Você está na página 1de 719

Aula 01 –

Scanning and
Skimming
False Cognates

Escola Naval 2021


Teacher Andrea Belo
Teacher Andrea Belo
Aula 01: Escola Naval 2021

Sumário
Apresentação .................................................................................................................... 3

Scanning: investigar e responder ....................................................................................... 4

Skimming: compreender e solucionar ................................................................................ 6

Scanning & Skimming: ler e achar as respostas: “Just do it” ............................................... 9

Curiosidades Scanning/Skimming .................................................................................... 13

Scanning e Skimming em pequenos textos ...................................................................... 16

Scanning e Skimming em textos longos............................................................................ 17

Cognates and False Cognates ........................................................................................... 18

6.1 Questões inéditas ...................................................................................................................... 23

Exercícios na área da Carreira Militar ............................................................................... 27

Gabarito de outras questões............................................................................................ 35

Exercícios respondidos com comentários ......................................................................... 36

Considerações finais ........................................................................................................ 51

Referências bibliográficas ................................................................................................ 52

Traduções ........................................................................................................................ 54

Aula 01 – Scanning, Skimming and False Cognates 2


www.estrategiamilitar.com.br 61
Teacher Andrea Belo
Aula 01: Escola Naval 2021

Apresentação
Foi apresentada, junto ao cronograma de estudos, de forma sintetizada, uma introdução às
técnicas Skimming e Scanning. Estudamos e praticamos exercícios fazendo leituras dinâmicas e
rápidas, com o objetivo de se concentrar em palavras chaves, em ideias.
Recordando, Scanning, é a “varredura” do texto. É ler com atenção a primeira página de um
jornal, procurar uma palavra no dicionário, pesquisar na internet um artigo sobre determinado
assunto em busca de informações específicas, necessárias naquele momento, entre outros.
Durante as leituras dos textos presentes na prova do Escola Naval, você fará um
rastreamento, procurará algo que realmente interessa e seguir selecionando partes essenciais dos
textos até encontrar a informação desejada. Você pode voltar no texto time after time com o foco
no que precisa para responder aos exercícios. Concentre-se que vai dar certo! Let’s go!
Em complemento ao Scanning, vimos que a técnica Skimming é uma estratégia que ajuda você
a ler o texto mais rápido também. Dessa vez, prestando atenção ao layout do texto, título, subtítulo,
cognatos, falsos cognatos, primeiras e/ou últimas linhas de cada parágrafo, informação não verbal
(figuras, tirinhas, anúncios, gráficos, tabelas etc). Por exemplo, quando você espera para ser
atendidos em um consultório médico e fica folheando aleatoriamente uma revista, você está usando
a técnica do Skimming.
Em ambas técnicas que usaremos bastante agora, você vai, naturalmente e, na sequência das
nossas aulas, obter a ideia geral de cada texto, identificar o assunto e prever informações essenciais
ao texto.
No momento da prova, você vai ler cuidadosamente todos os parágrafos, percebendo a
função e razão de haver certas palavras destacadas, ou em caixa alta, em negrito ou em itálico etc.
Você deve estar atento aos detalhes dos textos, mas não se esquecer, é claro, que são
colaboradores diretos da sua aprovação os seus conhecimentos gerais, curiosidade ao que está
acontecendo atualmente em nosso país e no mundo e também seu estudo constante de todas as
disciplinas que abrangem os conteúdos da prova de melhores instituições do Brasil.
Vamos dar continuidade ao uso das técnicas, que certamente, serão valiosas e você também
deve estudar vários cognatos e falsos cognatos, que podem aparecer em sua prova.
Come on! Vamos resolver mais exercícios com diferentes tipos de questões e aprimorar seu
vocabulário com cognatos e falsos cognatos e, é claro, com as traduções no fim do material e assim,
day after day, você sentir-se mais preparado e confiante – o melhor candidato!

Aula 01 – Scanning, Skimming and False Cognates 3


www.estrategiamilitar.com.br 61
Teacher Andrea Belo
Aula 01: Escola Naval 2021

Scanning: investigar e responder


Ler textos presentes na prova exige concentração, capacidade interpretativa e muita, muita
atenção. E, você já sabe que, usando Scanning facilita sua compreensão, encontrando as palavras-
chave, que serão “guia” para encontrar a resposta da questão.
Alguns autores, além de escrever sobre Scanning, chamam essa técnica de Selectivity, por ser
uma forma de leitura seletiva, já que selecionamos os trechos onde se deseja encontrar aquela
determinada informação.
O que realmente importa não é o nome da técnica, mas, se você está analisando, em suas
leituras, as palavras cognatas, o vocabulário essencial e o que mais for questionado nos enunciados
das questões, levando à compreensão mais eficiente.
Na hora de sua prova, a escolha da estratégia de leitura deve estar de acordo com o objetivo
de cada questão e suas possíveis alternativas.
No decorrer das aulas, inúmeras questões serão solucionadas, todas elas detalhadamente,
com ênfase nas melhores formas de encontrar respostas, usando e chegar à sua aprovação.
Escolhendo o Scanning, você sabe exatamente o que está procurando: informações
específicas. Veja um esquema com as principais funções da técnica Scanning:

ESTRATÉGIA DE LEITURA
SELETIVA

FAZER A "VARREDURA" DO
SCANNING

TEXTO EM QUESTÃO

ATENTAR-SE EM PALAVRAS,
FRASES E TERMOS
IMPORTANTES

BUSCAR INFORMAÇÕES
OBJETIVAMENTE

NÃO LER TEXTO TODO E SIM,


ENCONTRAR RESPOSTAS
ESPECÍFICAS À PERGUNTA

CONECTAR PALAVRAS
PROCURADAS COM O
ENUNCIADO DA PERGUNTA

Aula 01 – Scanning, Skimming and False Cognates 4


www.estrategiamilitar.com.br 61
Teacher Andrea Belo
Aula 01: Escola Naval 2021

Scanning é, muitas vezes, chamado de “leitura de passar os olhos” porque você vai
justamente obter as informações fazendo uma rápida visualização. Você está em busca de algo
específico, que poderá facilitar o caminho até a resposta daquele exercício.
Vejamos agora, exemplo de questão que poderia ser da sua prova, explorando Scanning:

Vamos dar continuidade às análises e aprendizado, treinando a técnica Scanning.

Questão inédita – A opção correta sobre o texto é:


A) O texto trata de uma brincadeira na hora do corte de cabelo.
B) O barbeiro não quer cortar o cabelo de Calvin.
C) Adultos decidem como crianças devem cortar seus cabelos.
D) As crianças, como Calvin, não sabem qual é o corte ideal para a idade delas.
E) O texto trata do corte de cabelo de Calvin, decidido pela sua mãe.
Apenas com a técnica scanning, você “bate os olhos” e percebe que é um corte de cabelo,
Calvin sugere algo mas a decisão final é da sua mãe, certo? Então, você já acertaria a questão
só com a busca de palavras específicas, tais como “I want...”, sobre o que Calvin quer do corte
de cabelo e a mãe: “the usual “– sem traduzir mas, escaneando termos importantes.
Na letra A, fala de brincadeira, o que não é visto na examinação de palavras-chave. Falsa.
Quanto à letra B, temos a afirmação de o barbeiro não quer cortar o cabelo de Calvin mas não
vemos isso no texto, nem lendo e escaneando palavra por palavra. Falsa.
A letra C é falsa porque afirma algo geral e vimos, pela “varredura” do texto, que se trata de
Calvin e sua mãe e não de todos os adultos, como sugerido. Falsa.
A letra D está errada também. Podemos perceber que se trata de Calvin e não de todas as
crianças, como foi generalizado na letra D. Falsa.
A resposta correta é a letra E pois foi possível perceber com scanning, como analisamos no
início da resposta.
Sua meta é a aprovação e então, temos um caminho a percorrer juntos.
Vamos à técnica Skimming agora! Come on!

Aula 01 – Scanning, Skimming and False Cognates 5


www.estrategiamilitar.com.br 61
Teacher Andrea Belo
Aula 01: Escola Naval 2021

Skimming: compreender e solucionar

Agora é a vez de fazer uso do Skimming, estratégia tal que exige observação das informações
visuais que acompanham o texto (palavras destacadas, título, subtítulo, autor, fonte, data, layout do
texto, tabelas, fotos, referências, enfim, aquela "leitura por cima", para explorar ao máximo as
informações importantes presentes e perceber a ideia geral do texto.
Deve-se, porém, lembrar-se de que essa estratégia, assim como qualquer outra, só funciona
se você estiver concentrado durante a leitura.
Isso porque, além das informações ali contidas, resumirem o texto, cada palavra em destaque
é uma dica do assunto e, muitas vezes, é a própria resposta solicitada.
Skimming, ao começar a ler seu texto, vai proporcionar a você, a compreensão geral - General
comprehension - sobre qual assunto o texto discorre com dicas visuais de fácil percepção a um futuro
aluno de uma excelente universidade, não é verdade?
Veja um esquema com as principais funções da técnica Skimming:

ESTRATÉGIA DE LEITURA
DE EXAMINAÇÃO

CAPTAR IDEIAS PRINCIPAIS


SKIMMING

OBSERVAÇÃO GERAL SEM


DETER-SE EM UM PONTO
ESPECÍFICO

RECONHECER DICAS SOBRE


O ASSUNTO

NÃO LER TEXTO TODO E


SIM, EXPLORAR A LEITURA
EM GERAL

INVESTIGAÇÃO DA IDEIA
CENTRAL DO TEXTO

Aula 01 – Scanning, Skimming and False Cognates 6


www.estrategiamilitar.com.br 61
Teacher Andrea Belo
Aula 01: Escola Naval 2021

Para praticar a técnica de Skimming em um texto, que poderia ser, por exemplo, o artigo
abaixo, retirado do jornal The Guardian, observe a questão e os comentários, para que você saiba
como encontrar a resposta:

Aula 01 – Scanning, Skimming and False Cognates 7


www.estrategiamilitar.com.br 61
Teacher Andrea Belo
Aula 01: Escola Naval 2021

Questão – De acordo com o início do texto:

Bom, no início do texto, praticando Scanning e Skimming, já eliminamos as alternativas A e B,


que afirmam calcular o tempo na internet e tecnologia (e o texto é especificamente sobre uso de
telefone celular e não sobre internet ou tecnologia de forma geral).
A alternativa C, não tem relação com o que é perguntado: “Do you know how much time you
spend on your phone?” (Você sabe quanto tempo passa em seu telefone?), já que é uma pergunta
direcionada ao leitor e não sobre a escritora mencionada.
E a alternativa E, afirma que se pergunta número exato de horas mas, não há nada de exato
na pergunta e sim uma reflexão de quanto tempo se passa no celular, direcionada, como eu disse,
ao leitor.
A alternativa D é a que contempla, de forma correta essa reflexão: tempo que você, leitor,
passa em seu telefone. Alternativa correta, por análise geral – skimming de informações.

Em cada exercício do nosso material, estamos fazendo uma análise cuidadosa e completa.
Isso vai te oferecer condições seguras para o dia da sua prova.
Independente da universidade escolhida, você precisará encontrar uma forma de ler e
responder as questões para ingressar nessas universidades. Todo exercício é preparo!
Fique atento aos “vestígios” que aparecem, são colaboradores direto do seu acerto nas
questões, junto às técnicas, que vamos treinar em toda essa aula. É importante que você saiba como
selecionar o que lê, onde focar sua atenção no momento de procurar uma informação específica e
essencial à questão.
Podemos excluir algumas opções de resposta a partir de análises. E podemos, também,
excluir alternativas com informações “inventadas”, ou seja, inexistentes no texto, que são colocadas
dentre as alternativas de resposta para gerar dúvida.
Eu costumo nomear essas armadilhas de “tricky” words, ou seja, pegadinhas. Vamos analisar
muitas no decorrer do material, com a prática de vários exercícios na teoria e na lista de questões
comentadas.
Fique atento! Very important!

Aula 01 – Scanning, Skimming and False Cognates 8


www.estrategiamilitar.com.br 61
Teacher Andrea Belo
Aula 01: Escola Naval 2021

Scanning & Skimming: ler e achar as respostas: “Just do it”


Já comprovamos que existem várias formas de facilitar a leitura em diferentes situações da
sua prova.
Identificar palavras ou ideias, procurar um verbo, um adjetivo, uma afirmação, uma negação,
entre outras informações essenciais na hora de resolver a prova.
O melhor candidato – você – deve aprender a usar técnicas e, junto aos seus conhecimentos,
chegar ao seu propósito = a sua aprovação.

Skimming e scanning são expressões do Inglês equivalentes a: ler superficialmente e ler


rapidamente, respectivamente.
Essas técnicas, como vimos anteriormente, ajudam você a obter mais rapidamente a
informação dos textos, como eu disse em outros capítulos, não sendo necessário ler cada palavra
contida em seu contexto.
Você vai fazer perguntas ao texto: o que se espera desse texto? Quais são as partes
importantes? Que locais posso reconhecer (se há imagem vinculada), Qual é o assunto central? Que
palavras ajudam a reconhecer do que se trata sua leitura?
E assim, percorrer o caminho ideal para chegar à conclusões, que certamente, levarão você à
aprovação.
O objetivo de compreender os textos vai depender diretamente da sua capacidade em
relacionar ideias, estabelecer referências e fazer deduções lógicas para buscar respostas às
questões.
Com a prática que estamos desenvolvendo nos exercícios, você vai, naturalmente, saber
identificar palavras que sinalizam o que se pede na questão, percebendo os elementos que
colaboram com a resposta correta.
Utilizando a técnica Scanning e também Skimming nas leituras, agora juntas na mesma
questão, será como o título desse capítulo – ler e achar as respostas: Just do it!
Para isso, você deve deixar de lado aquele hábito de ler palavra por palavra, lembrar tudo o
que sabe sobre o assunto e prestar atenção ao contexto em que as questões estão inseridas.
Usando o mesmo texto, que respondemos um a pergunta usando a técnica skimming, vamos
agora usar scanning e, escaneando informações, resolver outra questão, similar às da sua prova:

Aula 01 – Scanning, Skimming and False Cognates 9


www.estrategiamilitar.com.br 61
Teacher Andrea Belo
Aula 01: Escola Naval 2021

Na frase do texto “... until you realize it”, o termo sublinhado se refer a:

Aula 01 – Scanning, Skimming and False Cognates 10


www.estrategiamilitar.com.br 61
Teacher Andrea Belo
Aula 01: Escola Naval 2021

Bom, escaneando a frase em que o pronome “it” aparece, percebemos, de acordo com
regaras gramaticais existentes, que é, de fato, um pronome que se refere a palavras no singular.
“IT” é o pronome específico de substituição de um objeto ou um animal, por exemplo. Assim,
já eliminamos as alternativas C e E, em que se afirma que “it” se refere a algo no plural: minutos e
dias.
A alternativa A, é, até agora, a melhor opção, pois “ until you realize it”, é até você perceber
isso, provavelmente, isso seja o tempo gasto com o celular mas, continuemos a analisar as
alternativas.
E a alternativa B, afirma que “it” se refere a um verbo mas, como vimos quando falei a
tradução da frase “até você perceber isso”, é notável que it não está conectado com o verbo e sim
com o tempo gasto com o celular.
A alternativa D diz que se refere ao celular de Adrienne mas, a pergunta é para o leitor, você,
e não a escritora.
Assim, a melhor opção realmente é a letra A, em que se afirma que “it” se refere ao tempo:
exatamente o que o pronome quer dizer.
Viu como scanning funciona? Você vai direto ao ponto, no local em que a palavra está e o que
deve substitui-la ou encaixar ou seja qual for a pergunta da prova.
Muitas pessoas consideram scanning e skimming como técnicas preciosas e verdadeiras
estratégias de leitura, já que você consegue ler um grande volume de informação com prática.
Podemos afirmar que o skimming consiste em observar o texto rapidamente, para detectar o
assunto, sem preocupar com os detalhes, permitindo ao leitor identificar rapidamente o sentido
geral do texto.
Geralmente a leitura no skimming é realizada com a velocidade maior que a leitura normal e
assim, é mais abrangente.
E o scanning é uma técnica que consiste em localizar a informação específica desejada com
os olhos. É uma rápida visualização do texto como o nome diz, como um scanner, trabalhando
rapidamente, lendo a informação contida naquele espaço.
Agora, vejamos um esquema em que há informações sobre as técnicas que vimos e aplicamos
nos exercícios. Vamos lá.

Aula 01 – Scanning, Skimming and False Cognates 11


www.estrategiamilitar.com.br 61
Teacher Andrea Belo
Aula 01: Escola Naval 2021

Esquema aprovado? Vamos lá? Rumo à aprovação!

Aula 01 – Scanning, Skimming and False Cognates 12


www.estrategiamilitar.com.br 61
Teacher Andrea Belo
Aula 01: Escola Naval 2021

Curiosidades Scanning/Skimming
As estratégias de leitura que estamos usando nessa aula, têm a finalidade de viabilizar a sua
leitura sem que, necessariamente, você aprenda todas as palavras que existem em Inglês.
Seu vocabulário vai se estender, day after day, se você estiver lendo da forma que estou
explicando e mostrando a você através dos exercícios da de provas anteriores, aqui resolvidos e
comentados em detalhes.
O caminho do sucesso é o estudo contínuo e persistência em aprender.
Além disso, as traduções oferecidas no fim do material, como já adiantei, são preciosas
ferramentas de estudo com intuito de enriquecer vocabulário, tanto para ensinar você a interpretar
textos quanto escrever parágrafos inteiros sabendo muitas palavras.
Há curiosidades sobre as técnicas que estamos utilizando? Sim!
Quando pesquisamos sobre preparar-se para provas de mestrado e doutorado, bem como
especializações que exigem proficiência em Inglês, além das provas de várias instituições, a sugestão
e verdadeira instrução é sempre ler textos utilizando das técnicas Scanning e Skimming. Por quê?
A resposta só pode ser uma: com essas técnicas, conseguimos ler textos com agilidade e
qualidade, já que a assimilação é rápida.
A principal vantagem dessas técnicas é que são um tipo de leitura dinâmica e ensinam você a
reconhecer vocábulos inseridos dentro da frase.
E, reconhecer “dados” incorporados ao texto, é o grande segredo.
Assim, você enxerga e compreende o assunto através dos blocos de palavras juntas. Seus
olhos fazem “paradas” tão rápidas que não se percebe.
Uma fonte muito usada nas provas são as tirinhas do “Hagar, The Terrible” – Hagar, o horrível.
São curtas, críticas e cheias de assuntos interessantes para as questões.
Escolhi essa tirinha de “Hagar, o Horrível”, por ser um exemplo de fonte explorada com
assuntos explorados em muitas disciplinas. Você vai ver.
Vamos olhar para a história, fazer uma leitura com os olhos rapidamente, tentar entender a
essência e extrair o que se compreende com as palavras que chamam a atenção.

Questão Inédita Teacher Andrea Belo – identificação do assunto/interpretação.


Veja a sugestão de perguntas da forma como você deve iniciar a resolução.
Qual é o assunto que você consegue perceber através da leitura da tirinha do Hagar?

Aula 01 – Scanning, Skimming and False Cognates 13


www.estrategiamilitar.com.br 61
Teacher Andrea Belo
Aula 01: Escola Naval 2021

Quem são os personagens? Sobre o que conversam? Como são as reações deles?
Tente responder comprovando com Scanning e Skimming e veja como fica fácil.

https://nebusresearch.files.wordpress.com/2014/06/chris-browne_hagar-the-horrible_19-june-2014.gif

Você deve “ler” os quadrinhos. As expressões faciais. As reações. Os movimentos. Tudo


são indicadores do assunto. É primordial que você leia a figura ou um texto como se estivesse
desvendando um mistério, decifrando um código.
No quadrinho em questão, quando você vê a palavra “beer” no primeiro quadrinho,
pronunciada pela esposa, percebe que ela não está muito satisfeita provavelmente porque
Hagar está bêbado (percebemos pelas estrelinhas ao redor da cabeça dele).
Como resposta, Hagar pronuncia números indicando que não sabe quantas cervejas
bebeu. A expressão que contém palavras cognatas, “serious problem”, que a mulher dele diz,
mostra que ela está irritada, pois aponta o dedo, demonstração de nervosismo, raiva, fúria.
Ele, “sem noção” do que diz e, perceptivelmente alcoolizado (inclusive com as estrelinhas
ao redor da cabeça), responde com as palavras “math tutor”, que podemos deduzir uma
manifestação de que ele revela não saber matemática e, assim, precisa de um tutor, um
professor de matemática.
Conseguiu identificar o assunto? Encontrou palavras que facilitaram?
As “chaves” da resolução são as técnicas e atenção!

Bom, essa “leitura rápida” foi feita em segundos.


Isso que você está aprendendo e deve fazer, em todas as questões da sua prova. Vamos
para o próximo capítulo. Mas antes, deixo aqui um esquema representativo a seguir.

Aula 01 – Scanning, Skimming and False Cognates 14


www.estrategiamilitar.com.br 61
Teacher Andrea Belo
Aula 01: Escola Naval 2021

#FICAADICA

Vejamos outros textos com o uso do Scanning e Skimming.

Aula 01 – Scanning, Skimming and False Cognates 15


www.estrategiamilitar.com.br 61
Teacher Andrea Belo
Aula 01: Escola Naval 2021

Scanning e Skimming em pequenos textos


Após utilizarmos as técnicas necessárias com análise de detalhes fundamentais, o caminho
da aprovação está fácil, percebeu? Como você está se saindo?
Você já deve ter notado que usar Skimming e Scanning é basicamente fazer uma leitura
rápida, mas, não estou falando de velocidade, e sim de atingir os objetivos esperados.
Até porque, posteriormente, você pode voltar ao texto – quantas vezes quiser – e retomar
informações sobre o que você procura.
Estamos, em cada exercício, identificando o que é essencial para ter uma noção geral do que
se trata o texto. É uma construção de conhecimento, com foco ao que realmente interessa,
economizando tempo e resolvendo as questões da prova facilmente.
Nesse capítulo, vou exemplificar o uso dessas técnicas juntas em textos pequenos e
posteriormente, em textos mais longos. Veja essas frases:

Após exames, pessoas com níveis alterados de glicemia, a comprovação de


ringope é imediata. Além de necessitar o uso de insulina, é obrigatório evitar o
consumo de lagofe, para não agravar mais ainda as condições do paciente, que
pode fatalmente sabafar, se não tomar os devidos cuidados com a saúde.

Sendo uma questão, você conseguiria ler as palavras que inventei - ringope,
lagofe e sabafar - substuindo-as, de forma automática, pelos termos corretos à
compreensão do parágrafo acima e responder qual é o assunto? Sim, diabetes. E
ringope é diabetes, lagofe - açúcar, sabafar - falecer, não é?

Você percebeu que não demorou tanto tempo para identificar as palavras inventadas e, o
melhor, compreendeu o parágrafo inteiro por dois motivos: você aprendeu a ler de forma dinâmica,
dando importância às palavras peculiares, aquelas que indicam o assunto e/ou o que precisa saber
sobre tal assunto.
E porque o uso das técnicas estudadas em conexão ao vocabulário aprendido a cada exercício
resolvido, proporciona autoconfiança, garantia da sua aprovação.
A falta de vocabulário é considerada uma das causas para o atraso na leitura. Porém, durante
seus estudos, você percebe que as palavras desconhecidas, não devem ser encaradas como
empecilho, mas desafios: você vai procurar o sentido entre as palavras.

Aula 01 – Scanning, Skimming and False Cognates 16


www.estrategiamilitar.com.br 61
Teacher Andrea Belo
Aula 01: Escola Naval 2021

Scanning e Skimming em textos longos


A partir de diferentes formas que facilitam suas leituras, você já precisa ter em mente que vai
conseguir ler, utilizando as técnicas estudadas para compreender o texto ou parte dele – parte
essencial para encontrar o que se pede.
E, acelerando sua leitura pouco a pouco, tanto em textos curtos ou em textos maiores, o tipo
de varredura feita pelos seus olhos fará com que seu cérebro responda aos estímulos que você
enviou.
E, dessa forma, dando-lhes significados, que ajudam a encontrar as respostas certas.
Você estará, no decorrer das aulas, ressaltando o que é mais ou o que é menos interessante
em cada texto. O que é mais ou menos relevante. E assim, caminhar para as soluções das questões.
A leitura dos textos, apesar de muitas vezes apresentarem um conteúdo longo e repleto de
palavras desconhecidas, ficará viável.
Isso porque, com o uso de técnicas, vinculadas às análises cautelosas e com muita atenção
que se deve ter antes de responder uma questão, mesmo se for simples.
Pode ser, inclusive, que você considere simples demais e, no teor de cada pergunta, a
interpretação exija de você um cuidado ainda maior. É necessário muita atenção e o bom uso das
técnicas ensinadas. Don’t forget it!
Vejamos questões inéditas em continuação à resolução de possíveis questões.
Agora, com as questões inéditas e também com as questões de anos anteriores, que vamos
solucionar e comentar, uma a uma.
Aqui, teremos exemplos de textos curtos, longos, imagens, tirinhas, entre outros, analisando
cada possibilidade de exercício que já estiveram ou podem estar presentes em sua prova.
Vamos explorar, nessa aula, bastante exercícios de escrita, como é cobrado na segunda fase
da Fuvest, para aproveitar as técnicas de scanning e skimming aqui aprendidas.
Mas, como eu disse, as técnicas são excelentes para a prática da leitura rápida, dinâmica, com
agilidade e sabedoria para, no dia da sua prova, você gabaritar a prova de Inglês.
Come on!

Aula 01 – Scanning, Skimming and False Cognates 17


www.estrategiamilitar.com.br 61
Teacher Andrea Belo
Aula 01: Escola Naval 2021

Cognates and False Cognates


Eu falei para você, na aula de introdução, a importância de conhecer os falsos cognatos já que,
cognatos, pela semelhança com nossa língua, ajudam na leitura, enquanto os falsos cognatos,
podem atrapalhar quando não se sabe o que realmente significa.

O termo cognato se refere a palavras que têm a mesma origem. Por exemplo, Novembro
(Português) e November (em Inglês) são cognatos.

É normal que as pessoas usem o termo “falsos cognatos” quando se referem a palavras que
são escritas ou faladas de forma parecida, e portanto nos fazem pensar que as duas palavras têm
o mesmo significado, mas não têm. E aparecem muito nas provas.

Um dos exemplos mais comuns de falsos cognatos, que confundem a mente das pessoas, é o
caso das placas escritas “push” e “pull”. Isso porque, “push”, em Inglês, se assemelha ao verbo
puxar como “puxe”, em Português, mas o significado de “push” é empurrar.

Podemos, inclusive, ver pessoas puxando a porta ao invés de empurrar, não é mesmo?

Apesar de bastante utilizado, este termo não é o mais adequado de todos. Cognatos se
referem, de fato, à origem das palavras, e ao pensar em falsos cognatos não estamos pensando na
origem, e sim nos seus significados em duas línguas diferentes.

E, claro que duas palavras podem ter a mesma origem mas, por diversos acontecimentos ao
longo do tempo estas palavras acabaram adquirindo significados diferentes.

Um termo também comum e melhor para expressar os falsos cognatos é “false friends”, ou
falsos amigos. Veja um exemplo:

Nessa imagem, por exemplo, se for para ler ou escrever


que a mulher está com uma roupa elegante, diríamos:
“The woman is elegant because of her
apparel”
Eu sei que o falso cognato apparel lembra a palavra
aparelho e não parece nada com vestimenta, mas
vestimenta é a tradução de apparel.

Aula 01 – Scanning, Skimming and False Cognates 18


www.estrategiamilitar.com.br 61
Teacher Andrea Belo
Aula 01: Escola Naval 2021

Os falsos cognatos, também chamados de falsos amigos são, muitas vezes, considerados
inimigos na hora dos estudos.
Mas você não deve se apegar aos detalhes, não vai parar sua leitura a cada vírgula e sim, usará
skimming e scanning - ler com os olhos para compreender o assunto e também “escanear” as
informações necessárias e solicitadas nas questões, como eu já te disse antes.
Você vai precisar de uma lista com os falsos cognatos mais comuns em provas. Vou mostrar,
então, uma lista com diversos outros casos que enganam pessoas, para você não se confundir e sim,
aprimorar seus conhecimentos.
Vamos nos preparar bem. Agora, vejamos alguns exemplos de falsos cognatos na lista que
preparei para você, com muitos presentes em provas anteriores diversas.

LISTA DE FALSOS COGNATOS MAIS COMUNS E FREQUENTES NAS PROVAS

FALSOS COGNATOS E SEUS SIGNIFICADOS WATCH OUT! - CUIDADO!

Abstract – resumo Abstrato – conceptual


Actually - na verdade, de fato Atualmente – These days, today, nowadays
Accent - sotaque Assento – seat
Adept- especialista, bom conhecedor de... Adepto - supporter
Agenda - pauta diária, pauta para reuniões Agenda - appointment book
Alias – pseudônimo Aliás – By the way
Alms – esmola Almas – souls
Animus – hostilidade, inimizade Animado – Excited
Annotate - observar Anotar – to take note, to write down
Application– registro, inscrição Aplicação - investment
Appointment - compromisso profissional Apontamento – note
Appreciation – gratidão, reconhecimento Apreciação – judgement
Argument (n) – discussão Argumento - reasoning, point
Arm - braço Arma - gun
Army – exercito Arma – gun
Assist – ajudar, dar assistência Assistir - Watch
Attend - assistir, participar de Atender - to help; to answer; to see, to
Audience – plateia, público Audiencia – court appearance
Balcony - sacada Balcão - counter
Baton – cassetete Batom – lipstick
Barracks – quartel Barraca – tent
Bond – vínculo, elo Bonde – trolley car

Aula 01 – Scanning, Skimming and False Cognates 19


www.estrategiamilitar.com.br 61
Teacher Andrea Belo
Aula 01: Escola Naval 2021

Bonnet – touca, capô de carro Boné – cap


Braces – aparelho dental Braços – arms
Candid – sincero Cândido – innocent, naive
Carton - caixa de papelão Cartão – card
Cartoon – desenho animado Cartão – card
Chef – cozinheiro, mestre cuca Chefe – boss
Cigar - charuto Cigarro - cigarette
College - faculdade Colégio - school
Commodity - artigo, mercadoria Comodidade - comfort
Compromise - entrar em acordo Compromisso - appointment; date
Content - conteúdo Contente – glad, happy
Convict - réu Convicto - Sure
Costume - fantasia (roupa) Costume - custom, habit
Cup - xícara Copo - glass
Curse – maldição, xingamento. Curso – course
Dessert - sobremesa Deserto - desert
Data – dados Data – date
Dependable – confiável Dependente – Dependant
Devolve – transferir Devolver – return, give back, refund
Discussion – debate, opiniões, considerações Discussão – argument
Diversion – desvio, trajeto Diversão – fun
Educated - com bom nível de escolaridade Educado - well-mannered, polite
Enroll - registrar-se inscrever-se, alistar-se Enrolar - to roll
Expiation – penitência, castigo Espiar – To spy
Exquisite - belo, refinado Esquisito - strange, odd
Expert – especialista Esperto – clever, smart
Fabric – tecido Fábrica – factory
Fate – destino Fato – fact
File – arquivo Fila – line
Gracious – benéfico Gracioso – graceful
Gratuity – gorjeta Gratuito – For free, gratuit
Gravy – molho, caldo Grave – Serious
Grip - agarrar firme Gripe - cold, flu
Heydey – apogeu Ei, dia – Hey day
Azar - bad luck
Hazard - risco, arriscar

Aula 01 – Scanning, Skimming and False Cognates 20


www.estrategiamilitar.com.br 61
Teacher Andrea Belo
Aula 01: Escola Naval 2021

Hostage – refém Hóspede – guest


Idiom - expressão idiomática Idioma – language
Injury – ferida, ferimento Injuria – insult, offence
Intend - pretender, ter intenção Entender – understand
Intoxication – embriaguez, efeito de drogas Intoxicação – poisoning
Jest – zombo, brincadeira Gesto – gesture
Lecture - palestra, aula Leitura – reading
Legend – lenda Legenda - subtitle
Livraria - book shop
Library - biblioteca
Lanche – snack
Lunch – almoço
Luxúria – lust
Luxury - luxo
Magazine - department store
Magazine - revista
Maior – bigger
Mayor – prefeito
Matar – to kill
Mate – colega, companheiro
Mistura - mix, mixture
Moisture - umidade
Notícia - news
Notice - notar, perceber-se
Novela - soap opera
Novel – romance
Official – officer
Office – escritório
Oração – prayer
Oration – discurso (formal)
Orquídea - Orchid
Orchard – pomar
Parentes - relatives
Parents - pais
Pasta – briefcase
Pasta – massa, macarrão
Físico – physicist
Physician – médico
Telephone – Phone, telephone
Phony – impostor
Polícia – Police
Policy – Apólice, política (ideais políticos)
Porco – pig
Pork – carne de porco
Porta – door
Port – porto
Prato – plate, dish
Prate – tagarelar, falar muito
Preservative – condom
Preservative – conservante
Pretender - to intend, to plan
Pretend - fingir
Procurar - to look for
Procure - conseguir, adquirir
Pular - to jump
Pull - puxar
Puxar - to pull
Push - empurrar
Realizar - make come true, to carry out
Realize - perceber, notar
Recipiente – container
Recipient – recebedor
Reclamar – complain
Reclaim – recuperar
Recordar, lembrar – remember, remind
Record – gravar
Refrigerante - soft drink, soda
Refrigerant - substância usada em aparelhos
Resumir – sumarize
Resume - retomar, reiniciar

Aula 01 – Scanning, Skimming and False Cognates 21


www.estrategiamilitar.com.br 61
Teacher Andrea Belo
Aula 01: Escola Naval 2021

Retired – aposentado Retirado – removed


Senior – idoso Senhor - gentleman, sir
Sensible – sensato Sensível – sensitive
Service – atendimento Serviço – job
Sort – tipo, espécie Sorte – luck, fate
Stranger - desconhecido Estrangeiro - foreigner
Stupid – burro Estúpido - impolite, rude
Supper – jantar, ceia Super – super
Support - apoiar Suportar (tolerar) - can stand
Sympathetic – compreensivo Simpático - nice
Tax - imposto Taxa - rate; fee
Tent – barraca Tentar – to try
Thicket – moita, mato fechado Ticket, bilhete – ticket
Toss – arremessar Tosse – cough
Turn - vez, volta, virar, girar Turno - shift; round
Ultimately – em última análise Ultimamente – Lately
Valorous – corajoso, destemido Valoroso, de valor, valuable
Vicious – defeituoso, impuro Viciado – addicted
Vine – videira Vinho – wine

Meu conselho para você é o seguinte: leia a lista, estude bastante, sempre. Quanto mais você
ler artigos jornalísticos, reportagens, textos em geral usados como fonte para elaboração das provas,
junto ao material aqui desenvolvido, mais preparado você vai estar.

Vejamos questões inéditas em continuação à resolução de possíveis questões.

Aula 01 – Scanning, Skimming and False Cognates 22


www.estrategiamilitar.com.br 61
Teacher Andrea Belo
Aula 01: Escola Naval 2021

6.1 Questões inéditas


O jornal britânico The Guardian, é publicado diariamente em Londres e tem grande
circulação, considerado um dos jornais mais lidos do mundo.
Talvez por ser um veículo de notícias conhecido internacionalmente e ter boa fama, é usado
na hora da elaboração das provas.
Por esse motivo, elaborei questões inéditas com um artigo do jornal The Guardian, publicado
em Abril/2019, uma notícia polêmica e um tema pertinente para que possamos analisar e dar
continuidade aos estudos de sucesso. Come on!

Questão inédita – Fonte: The Guardian

https://www.theguardian.com/lifeandstyle/2019/apr/01/five-ways-to-get-more-fibre-in-your-diet acesso 5/4/2019

Aula 01 – Scanning, Skimming and False Cognates 23


www.estrategiamilitar.com.br 61
Teacher Andrea Belo
Aula 01: Escola Naval 2021

Questão inédita– Teacher Andrea Belo/2019


Questão 1 – A opção correta presente no texto é:
A) O texto trata de fibras solúveis e insolúveis.
B) As fibras se referem a alimentos de digestão complexa.
C)Adultos devem ingerir mais de 30g de carboidratos por dia.
D) As maçãs são alimentos oferecidos pela BDA como auxílio na dieta.
E) As palavras “fibre” e “roughage” são sinônimos no texto.
Comentários:
Em continuação ao uso das técnicas aprendidas, vamos começar a ler nosso texto, sem traduzi-
lo por inteiro, mas verificando o que indica o assunto. Palavras tais como “fibre, diet, risk,
câncer, carbohydrates, apple”, entre outras, deixam clara a ideia de que a preocupação com a
saúde é abordada assim como doenças e importância de dietas e esse tema.
Vejamos, atenciosamente, qual opção se encaixa melhor no texto.
Ao verificar a alternativa “A”, afirmando que o texto trata de fibras solúveis e insolúveis, até
encontramos a palavra “fibre”, mas, no início do texto, fala-se apenas de fibras insolúveis – até
compara-se a palavra “fibre” e “roughage”, ambas dizem respeito às fibras insolúveis. Aqui,
vale a pena lembrar-se como é importante estar em dia com as notícias sobre qualquer
assunto, inclusive saúde. Há vertentes que defendem fibras solúveis como melhores à saúde,
pois fibras solúveis, como aveia e cascas de frutas, misturam-se no estômago e inibem a
absorção de glicose, controlando o colesterol e o diabetes.
As fibras insolúveis de que trata o texto, por sua vez, se você fizer uma pesquisa para entender
melhor sobre isso ou, se já leu e tem conhecimentos sobre o assunto, saberia que frutas como
a maçã e algumas verduras de cor escura, passam pelo estômago sem sofrer alterações,
regulam o movimento peristáltico e ainda são responsáveis pela limpeza de toxinas e bactérias
nocivas. Até a foto mostra maçãs, exemplo de fibra insolúvel. A alternativa “A” está incorreta.
Já comentei com você que, informações “inventadas”, ou seja, não encontradas no texto
geralmente estão presentes em opções falsas na questão.
Na letra “B”, afirma-se que as fibras se referem a alimentos de digestão complexa. Além de
seus conhecimentos prévios auxiliarem muito (já que fibras são consideradas auxiliares na
digestão e absorção do alimento e não o contrário) seria fácil saber que a palavra “complexa”
não está presente no texto e que fibras ajudam na digestão, que se torna mais fácil e não
complexa. A alternativa “B” está errada.
Na letra “C”, afirma-se que adultos devem ingerir mais de 30g de carboidratos por dia. A
palavra “mais” em inglês, geralmente, se não for uma comparação ou adequação ao que se
refere, aparece, na maior parte das vezes como “more”. No texto, a afirmação é de que se deve
consumir 30g e não mais de 30g – “adults should get 30g a day”. Alternativa “C” está incorreta.

Aula 01 – Scanning, Skimming and False Cognates 24


www.estrategiamilitar.com.br 61
Teacher Andrea Belo
Aula 01: Escola Naval 2021

Na letra “D”, afirma-se que maçãs são alimentos oferecidos pela BDA como auxílio na dieta. A
“BDA”, que no texto informa que é a Associação Britânica de Dieta, é mencionada, mas em
momento nenhum essa Associação oferece maçã ou qualquer outro alimento para dieta das
pessoas. A alternativa não está de acordo com o texto e por isso, pode ser descartada.
Lembra que já comentei com você que, muitas vezes, palavras são “inventadas” nas
alternativas de escolha.
Quando isso acontece, fica simples, pois você vai excluir e continuar sua análise.
Na letra “E”, afirma-se que as palavras “fibre” e “roughage” são sinônimos no texto. Sim,
quando se usa o termo “or” – ou em inglês, é para dar opções de um ou outro, ambos termos
dizem respeito às fibras insolúveis – “Fiber, or roughage, refers to indigestible carbohydrates...”
logo no início do texto. É a alternativa correta.
Após análise, temos a alternativa que melhor se encaixa no texto, a letra “E”, que é, na verdade,
a única que representa o assunto com informação verdadeira, de acordo com a fonte de leitura
e os passos seguidos para melhor resolver a questão.
Você teria acertado? Está ficando simples, não é?
Vejamos outra questão para treinar seus conhecimentos e técnicas e ficar ainda mais claro
tudo que você está aprendendo e usando nas questões.

Questão inédita - Teacher Andrea Belo/2019


Questão 2 - Os termos abaixo sublinhados podem ser substituídos, respectivamente, sem que
haja prejuízo do sentido, por:
I. “A fiber-rich diet...” (linha 1) → fibre
II. “... including a reduced risk of heart disease…” (linha 2) → illness
III. “… may be a reason why” (linha 4) → because
Estão corretas:
apenas I e III.
apenas III.
apenas II.
apenas II e III.
apenas I e II.

Aula 01 – Scanning, Skimming and False Cognates 25


www.estrategiamilitar.com.br 61
Teacher Andrea Belo
Aula 01: Escola Naval 2021

Comentários:
Já sabemos que o texto trata de alimentação saudável, com ênfase no consumo de fibras.
Vamos analisar os números I, II e III, para que você possa compreender o vocabulário envolvido
e depois, encontre a resposta correta.
No número I, “A fiber-rich diet...”, a troca seria feita de “fiber” para “fibre”, o que é possível
pois ambas significam a mesma coisa. Isso acontece porque há diferenças em relação aos
termos usados em diferentes países ou regiões
Nos EUA, é comum o isso de “fiber” enquanto os países europeus falantes da língua inglesa,
usam “fibre”, assim como usam “centre” ao invés de “center” (EUA).
No número II, “... including a reduced risk of heart disease…”, a troca seria feita de “disease”
para “illness”, o que é possível pois são sinônimos.
O que você precisa saber é que illness se refere às doenças físicas e também mentais, mas, não
se usa illness para se falar de algo simples como um resfriado, por exemplo. Por sua vez, disease
geralmente se refere a qualquer doença e, na maior parte das vezes, doenças que são
transmissíveis de uma pessoa para outra, como uma infecção. E não se usa disease para doença
mental e sim "mental illness".
No número III, “… may be a reason why”, a troca seria de “why” para “because”, mas, apesar
de ambas significarem porque, why é usado em perguntas enquanto because é usado em
respostas. E, quando se fala de “o porquê”, o motivo ou razão, daí usamos “why”, como na
frase que estamos analisando – “a reason why” - a razão por que algo acontece.
Bom, pela análise feita, agora já sabemos que só são possíveis de substituição a I e a II.
Pelo que vimos acima, fica fácil marcar a alternativa letra “E”, já que todas as outras apontam
números que não estão corretos. E veja como fica a análise:
Na letra A, apenas I e III, a III não é possível de substituição sem prejuízo;
Na letra B, apenas III, está incorreta porque a II é a única que em não pode haver substituição.
Na letra C, apenas II, está incorreta porque não é apenas o número II mas também o I.
Na letra D, apenas II e III, está incorreta porque a III não é possível de substituição sem prejuízo.
Na letra E, apenas I e II, é nossa resposta.
E agora, vamos aos exercícios de provas de anos anteriores, para praticar questões já
cobradas e aumentar seus conhecimentos sobre como essas questões são elaboradas.

Aula 01 – Scanning, Skimming and False Cognates 26


www.estrategiamilitar.com.br 61
Teacher Andrea Belo
Aula 01: Escola Naval 2021

Exercícios na área da Carreira Militar

Vamos fazer diferentes questões de inúmeras bancas para praticar. Let’s go!

Questão 01 (EFOMM/2018)

Questão 02 (EAM/2018)

Aula 01 – Scanning, Skimming and False Cognates 27


www.estrategiamilitar.com.br 61
Teacher Andrea Belo
Aula 01: Escola Naval 2021

Questão 03 (EEAR/2017)

The Bottom Line on Facebook Depression 1 Facebook, the most popular social media
platform, does not make people more depressed on its own. Instead, what the research
shows is that Facebook – when used as a surveillance device – leads to a greater risk of
feelings of 5 envy. And the more those feelings of envy increase, the more likely it is for
a person to start feeling depressed. The key to stopping these feelings is to not use
Facebook primarily as a surveillance method to spy on your family and friends’ lives.
Instead, use it as a social network where you 10 share your own information, photos and
updates, as well as consume other’s updates and shares. Healthy use of Facebook will
protect you against the possibility of feeling more depressed after using it. It’s a simple
thing you can try for yourself – especially if you feel more envious after checking
Facebook.

(Fonte: Psych Central – World of Psychology)

Questão 03 (EEAR/2017)
Reading the text leads to the conclusion that Facebook
a) may lessen depressive feelings.
b) alone doesn’t cause people to be more depressed.
c) is a social tool that can help loneliness if it is used in a healthy way.
d) can lead to unconscious envy that will result in depression feelings.

Aula 01 – Scanning, Skimming and False Cognates 28


www.estrategiamilitar.com.br 61
Teacher Andrea Belo
Aula 01: Escola Naval 2021

Questão 04 (Colégio Naval/2018)

Questão 05 e 06 (ESPCEX/2017)

Aula 01 – Scanning, Skimming and False Cognates 29


www.estrategiamilitar.com.br 61
Teacher Andrea Belo
Aula 01: Escola Naval 2021

Questão 07 (EPCAR/2018)

Questão 08 (EPCAR/2018)

Aula 01 – Scanning, Skimming and False Cognates 30


www.estrategiamilitar.com.br 61
Teacher Andrea Belo
Aula 01: Escola Naval 2021

Questão 09 (ITA/2016)

Adaptado de: www.fundersandfounders.com/9.types-of-intelligence. (acesso:13/8/2015)

Questão 09 - Os tipos de inteligência que se associam às definições I , II , III e IV da figura


são, respectivamente:
a) inter-personal, existential, intra-personal, linguistic.
b) inter-personal, linguistic, intra-personal, existential.
c) intra-personal, linguistic, inter-personal existential.
d) intra-personal, inter-personal, existential, linguistic.
e) existential, linguistic, inter-personal, intra-personal.

Aula 01 – Scanning, Skimming and False Cognates 31


www.estrategiamilitar.com.br 61
Teacher Andrea Belo
Aula 01: Escola Naval 2021

Questão 10 (IME/2018)

Choose the correct option.


a) Dutch researchers produced penicillin for it was cheaper than before. Concerning the supply
of it, the increase was automatic.
b) The first clinical use of penicillin was not immediate owing to the impact of its discovery.
This delay changed the History of Medicine.
c) The efforts by the Netherlands to produce the new drug weren't regarded as valuable until
around a decade ago.
d) It is impossible that penicillin’s first clinical use dates back more than 75 years.

Aula 01 – Scanning, Skimming and False Cognates 32


www.estrategiamilitar.com.br 61
Teacher Andrea Belo
Aula 01: Escola Naval 2021

e) The US and Great Britain succeeded in producing penicillin right after 1943.

Questão 11 (Colégio Naval/2019) Texto para perguntas 11 e 12

Questão 11 – Read the extract from the text


“Ten minutes after you get to work and stop reading the morning paper”
Mark the option that can replace the expression get to
(A) drive to
(B) go to
(C) find
(D) arrive at
(E) come from

Aula 01 – Scanning, Skimming and False Cognates 33


www.estrategiamilitar.com.br 61
Teacher Andrea Belo
Aula 01: Escola Naval 2021

Questão 12 – Read the extract from the text

“The year is 2020, and it’s 7:45 on a rainy Monday morning (...)”

What’s the correct question referring to the underlined information?


(A) What’s the weather?
(B) What’s the weather like?
(C) How’s the weather?
(D) How’s the climate like?
(E) What’s the climate?

Aula 01 – Scanning, Skimming and False Cognates 34


www.estrategiamilitar.com.br 61
Teacher Andrea Belo
Aula 01: Escola Naval 2021

Gabarito de outras questões

1. A 2. D 3. B

4. E 5. B 6. E

7. D 8. A 9. C

10. C 11. D 12. B

Aula 01 – Scanning, Skimming and False Cognates 35


www.estrategiamilitar.com.br 61
Teacher Andrea Belo
Aula 01: Escola Naval 2021

Exercícios respondidos com comentários

Questão 01 (EFOMM/2018)

Comentários:
“Ruth queria ser transferida para outro departamento, mas a sua inscrição foi [...] porque seu
próprio departamento está com falta de pessoal”. Teremos a aula destinada aos verbos mas
aqui, o importante é escanear os termos importantes à resposta.
Na letra A, “recusada” está correto. O verbo to turn down indica “recusar” ou “rejeitar”, usado
no contexto em que é apresentado no trecho, por exemplo, em que uma inscrição para uma
vaga foi indeferida. Alternativa correta.
Na letra B, “bem vestido” é incorreto. O verbo não se encaixa de maneira alguma ao contexto.
Alternativa incorreta.
Na letra C, “aparecida” é incorreto. O verbo to turn up indica algo que aparece em determinado
lugar. Portanto, não se encaixa neste contexto. Alternativa incorreta.
Na letra D, “virada” é incorreto. O verbo to turn over indica algo que foi virado, como uma
página, por exemplo. Portanto, não se encaixa neste contexto. Alternativa incorreta.
Na letra E, “desviado” é incorreto. Apesar de to turn away também poder ser interpretado
como “rejeitar” ou “recusar”, é mais comumente usado para se falar de pessoas. Mas há ainda
o sentido de “desviar”, como desviar algo da luz, por exemplo. Portanto, não se encaixa neste
contexto. Alternativa incorreta.

Aula 01 – Scanning, Skimming and False Cognates 36


www.estrategiamilitar.com.br 61
Teacher Andrea Belo
Aula 01: Escola Naval 2021

Questão 02 (EAM/2018)

Comentários:
No cartum, o future tense é usado para expressar
Na letra A, “um plano” é incorreto. Apesar de o uso do future tense (will) ser comumente usado
para indicar um plano de se fazer algo, no cartum a frase “This will never end” (Isso nunca irá
acabar) não soa como um plano, mas uma previsão. Alternativa incorreta.
Na letra B, “uma ordem” é incorreta. No cartum, a frase “This will never end” (Isso nunca irá
acabar) não indica uma ordem, mas uma previsão. Alternativa incorreta.
Na letra C, “uma possibilidade” é incorreta. O future tense não é usado para indicar
possiblidade; isso acontece com o auxiliar “can” (pode). Além disso, no cartum, a frase “This
will never end” (Isso nunca irá acabar) não soa como uma possibilidade, mas uma certeza.
Alternativa incorreta.
Na letra D, “uma previsão” está correta. No cartum, This will never end” (Isso nunca irá acabar)
indica uma certeza, uma previsão de que não haverá um fim.
Na letra E, “uma oferta” é incorreto. O future tense não é usado para se indicar uma oferta,
mas afirmar uma certeza. Alternativa incorreta.

Aula 01 – Scanning, Skimming and False Cognates 37


www.estrategiamilitar.com.br 61
Teacher Andrea Belo
Aula 01: Escola Naval 2021

Questão 03 (EEAR/2017)
The Bottom Line on Facebook Depression 1 Facebook, the most popular social media platform, does
not make people more depressed on its own. Instead, what the research shows is that Facebook – when
used as a surveillance device – leads to a greater risk of feelings of 5 envy. And the more those feelings
of envy increase, the more likely it is for a person to start feeling depressed. The key to stopping these
feelings is to not use Facebook primarily as a surveillance method to spy on your family and friends’
lives. Instead, use it as a social network where you 10 share your own information, photos and updates,
as well as consume other’s updates and shares. Healthy use of Facebook will protect you against the
possibility of feeling more depressed after using it. It’s a simple thing you can try for yourself – especially
if you feel more envious after checking Facebook.
(Fonte: Psych Central – World of Psychology)
Questão 03 (EEAR/2017)
Reading the text leads to the conclusion that Facebook
a) may lessen depressive feelings.
b) alone doesn’t cause people to be more depressed.
c) is a social tool that can help loneliness if it is used in a healthy way.
d) can lead to unconscious envy that will result in depression feelings.

Comentários:
No enunciado diz: Ler o texto, leva à conclusão que o Facebook:
Na letra A, “may lessen depressive feelings” - pode diminuir os sentimentos depressivos. Ele
pode causar sentimentos de inveja, que por sua vez pode causar depressão. Alternativa
incorreta.
Na letra B, “alone doesn’t cause people to be more depressed” - em si mesmo não faz com que
as pessoas estejam mais deprimidas. Facebook, the most popular social media platform, does
not make people more depressed on its own. O Facebook, a plataforma de mídia social mais
popular, em si mesmo não torna as pessoas mais deprimidas. Alternativa correta.
Na letra C, “is a social tool that can help loneliness if it is used in a healthy way” - é uma
ferramenta social que pode ajudar na solidão se for usada de forma saudável. O texto diz que
o uso saudável do Facebook irá protegê-lo contra a possibilidade de se sentir mais deprimido,
não solitário. Alternativa errada.
Na letra D, “can lead to unconscious envy that will result in depression feelings” - pode levar a
uma inveja inconsciente que irá resultar em sentimentos de depressão. O texto diz que a
pessoa pode se sentir mais invejosa depois de verificar o Facebook. Portanto, não é uma inveja
inconsciente. Errada também.

Aula 01 – Scanning, Skimming and False Cognates 38


www.estrategiamilitar.com.br 61
Teacher Andrea Belo
Aula 01: Escola Naval 2021

Questão 04 (Colégio Naval/2018)

Comentários:
“Reassistir não só te lembra dos pontos identificáveis para se atentar durante a sua viagem,
mas também adiciona à empolgação da sua próxima exploração”.

Na letra A, Trip não está funcionando como sujeito, enquanto o pronome it está. Portanto, ele
não se refere ao vocábulo. Alternativa incorreta.
Na letra B, Spots funciona como objeto, e podemos notar que o pronome it está funcionando
como sujeito. Portanto, não há como ele se referir ao vocábulo. Alternativa incorreta.
Na letra C, Excitement está no final da frase, depois do pronome it – logo, não há como o
pronome se referir a essa palavra. Alternativa incorreta.
Na letra D, Exploration está no final da frase, depois do pronome it – logo, não há como o
pronome se referir a essa palavra. Alternativa incorreta.
Na letra E, Rewatch está correto, pois ele é o sujeito principal do trecho – “não só te lembra
dos pontos identificáveis para se atentar durante a sua viagem, mas também adiciona”. O
pronome it é utilizado para que não haja a repetição do termo, mas podemos notar que há um
paralelismo quanto às ações do sujeito na frase. Alternativa correta.

Aula 01 – Scanning, Skimming and False Cognates 39


www.estrategiamilitar.com.br 61
Teacher Andrea Belo
Aula 01: Escola Naval 2021

Questão 05 e 06 (ESPCEX/2017)

Questão 05 – Comentários:
No primeiro parágrafo, o termo lies aparece em “The challenge of conducting future military
operations within megacitites [...] lies in understanding the dynamic” (O desafio de conduzir
operações militares futuras em megalópoles está em entender a dinâmica).
Na letra A, “Ele fica deitado ao sol por muito tempo” é incorreto. Aqui, a expressão “to lie”
indica “deitar-se”, portanto, não corresponde ao mesmo sentido do trecho no parágrafo 1.
Alternativa incorreta.
Na letra B, “A competência dele está na sua capacidade de se comunicar” está correto. Assim
como no primeiro parágrafo, “lies” indica que algo se pauta ou se baseia em outra coisa.
Alternativa correta.

Aula 01 – Scanning, Skimming and False Cognates 40


www.estrategiamilitar.com.br 61
Teacher Andrea Belo
Aula 01: Escola Naval 2021

Na letra C, “Eu posso ver na cara dela que ela está mentindo” é incorreto. Aqui, “to lie” indica
“mentir”, portanto, não corresponde ao mesmo sentido do trecho no parágrafo 1. Alternativa
incorreta.
Na letra D, “Um crocodilo gigante fica à espera da sua presa” é incorreto. A expressão “to lie in
wait” indica “ficar à espera”, portanto, não corresponde ao mesmo sentido do trecho no
parágrafo 1. Alternativa incorreta.
Na letra E, “Este item sempre fica aí para a próxima reunião” é incorreto. A expressão “to lie
over” indica que algo paira sobre algum lugar, em um sentido de simplesmente ser deixado em
algum lugar. Portanto, não corresponde ao mesmo sentido do trecho no parágrafo 1.
Alternativa incorreta.

Questão 06
Comentários:
Na letra A, já podemos identificar o erro no primeiro termo, “is”. Em “How [...] language
communities interact in megacities?” temos a presença de um verbo (interatc – interagir), que,
em uma pergunta, precisa do auxiliar “do” ou “does” – neste caso, “do”, já que o sujeito está
no plural. Alternativa incorreta.
Na letra B, os dois primeiros termos estão corretos, mas o último não. Em “What role [...]
language play...”, temos uma pergunta e a presença de um verbo (play – assumir), portanto,
precisa-se do auxiliar “do” ou “does” – neste caso, “does”, já que o sujeito está no singular.
Alternativa incorreta.
Na letra C, podemos identificar o erro no primero termo, “are”. Em “How [...] language
communities interact in megacities?” temos a presença de um verbo (interatc – interagir), que,
em uma pergunta, precisa do auxiliar “do” ou “does” – neste caso, “do”, já que o sujeito está
no plural. O “are” seria possível se o verbo estivesse no gerúndio. Alternativa incorreta.
Na letra D, podemos identificar o erro no primero termo, “are”. Em “How [...] language
communities interact in megacities?” temos a presença de um verbo (interatc – interagir), que,
em uma pergunta, precisa do auxiliar “do” ou “does” – neste caso, “do”, já que o sujeito está
no plural. O “are” seria possível se o verbo estivesse no gerúndio. Alternativa incorreta.
Na letra E, todos os termos estão corretos. Nos primeiro e terceiro casos, as perguntas são
feitas com a presença do verbo principal que, portanto, precisa de um auxiliar, seja “do” ou
“does” (o que depende do sujeito). Já no segundo caso, temos uma voz passiva, que exige,
portanto, o verbo “to be” – como “is” ou “are” (dependendo do sujeito). Portanto, temos “How
do language communities interact in megacities? What tensions are caused by multiple
language communities in urban space? What role does language play in the power structures
(government or otherwise) of megacities?”. Alternativa correta.

Aula 01 – Scanning, Skimming and False Cognates 41


www.estrategiamilitar.com.br 61
Teacher Andrea Belo
Aula 01: Escola Naval 2021

Questão 07 (EPCAR/2018)

Comentários:
Na letra A, “Sobrinha” é incorreto, pois não corresponde ao masculino de daughter (filha).
Alternativa incorreta.
Na letra B, “Irmã” é incorreto, pois não corresponde ao masculino de daughter (filha).
Alternativa incorreta.
Na letra C, “Irmão” é incorreto, pois não corresponde ao masculino de daughter (filha).
Alternativa incorreta.
Na letra D, “Filho” está correto, pois corresponde ao ao masculino de daughter (filha).
Alternativa correta.

Questão 08 (EPCAR/2018)

Comentários:
A frase “Rumores de que Hitler ainda está vivo circulam desde os anos de 1970” significa que:
Na letra A, “algumas pessoas ainda consideram essa hipótese” está correto. Se ainda há
rumores sobre Hitler estar vivo desde 1970, é porque algumas pessoas ainda acreditam nisso.
Alternativa correta.
Na letra B, “é uma crença antiga que não é mais aceita” é incorreto. Os rumores ocorrem desde
os anos 70, portanto, muitos ainda acreditam nele. Alternativa incorreta.
Na letra C, “foi um rumor ocorrido em 1970” é incorreto. Os rumores vêm circulando desde
1970, não apenas em 1970. Alternativa incorreta.

Aula 01 – Scanning, Skimming and False Cognates 42


www.estrategiamilitar.com.br 61
Teacher Andrea Belo
Aula 01: Escola Naval 2021

Na letra D, “Hitler está vivo desde 1970” é incorreto. Segundo o trecho, são rumores que
circulam desde 1970 que afirmam que Hitler ainda vive. Alternativa incorreta.

Questão 09 (ITA/2016)
A questão a seguir ITA/2016 refere-se à figura a seguir:

Adaptado de: www.fundersandfounders.com/9.types-of-intelligence. (acesso:13/8/2015)

Questão 09 - Os tipos de inteligência que se associam às definições I , II , III e IV da figura


são, respectivamente:
f) inter-personal, existential, intra-personal, linguistic.
g) inter-personal, linguistic, intra-personal, existential.
h) intra-personal, linguistic, inter-personal existential.
i) intra-personal, inter-personal, existential, linguistic.
j) existential, linguistic, inter-personal, intra-personal.

Aula 01 – Scanning, Skimming and False Cognates 43


www.estrategiamilitar.com.br 61
Teacher Andrea Belo
Aula 01: Escola Naval 2021

Comentários:
Ao olhar a imagem, rapidamente, para escaneá-la, você percebe que há tipos de inteligência
ao redor de uma cabeça.
Sem ler, nesse momento, o que há ao redor da imagem, vamos focar naquilo que é solicitado
na questão. Exatamente – pede-se os tipos de inteligência dos números I, II, III e IV,
respectivamente.
Como aplicar a técnica Scanning agora e daqui para frente? Procurando palavras-chave. No
número I, “escaneamos” as palavras “understanding, yourself, feel”.
E, assim, forma-se a ideia de “entender você mesmo”, sentir, que é algo interpessoal, uma
relação com o “lado de dentro”.
O número II, com as palavras “words e express”, define algo linguístico: expressar palavras.
Aqui já saberíamos a resposta, só fazendo Scanning de informações, certo?
Mas, vamos continuar escaneando a questão para ter certeza. O número III, “people’s feelings”
e a figura de duas pessoas se comunicando, sugere algo intrapessoal, relação pessoal “de fora”
e na imagem, há duas pessoas de mãos dadas.
No número IV, “questions” já demonstra ligação com algo existencial e o desenho é de alguém
meditando, relação existencialista. Bingo: a alternativa “C” é a que segue a sequência certa e
é a alternativa que se deve marcar.
As alternativas “A”, “B”, “D” e “E”, trazem sequências de tipos de inteligência que não fazem
sentido em relação à imagem que analisamos.
Seriam descartadas logo que você começasse a conectar as palavras escaneadas com as
possíveis respostas.

Questão 10 (IME/2018)

Aula 01 – Scanning, Skimming and False Cognates 44


www.estrategiamilitar.com.br 61
Teacher Andrea Belo
Aula 01: Escola Naval 2021

Choose the correct option.


a) Dutch researchers produced penicillin for it was cheaper than before. Concerning the supply
of it, the increase was automatic.
b) The first clinical use of penicillin was not immediate owing to the impact of its discovery.
This delay changed the History of Medicine.
c) The efforts by the Netherlands to produce the new drug weren't regarded as valuable until
around a decade ago.
d) It is impossible that penicillin’s first clinical use dates back more than 75 years.
e) The US and Great Britain succeeded in producing penicillin right after 1943.

Aula 01 – Scanning, Skimming and False Cognates 45


www.estrategiamilitar.com.br 61
Teacher Andrea Belo
Aula 01: Escola Naval 2021

Comentários:
Desde o início do texto, pelo título, já podemos rastrear informações importantes à resposta a
partir da técnica Scanning aprendida. Encontramos palavras que facilitam a leitura, tais como
discovery, penicillin, 75 years e clinical use.
Essas palavras “escaneadas”, definem o assunto tratado e ajuda na compreensão do que é
explorado no texto – a descoberta da penicilina e algo relacionado ao tempo por causa da
referência de 75 anos – e assim já sabemos que é um artigo científico (também observado pela
fonte revista Science, no link abaixo do texto, que você também deve estar atento ao ler cada
questão da prova). Vamos lá. Pede-se a opção correta.

Na letra “A”, afirma-se “Dutch researchers produced penicillin for it was cheaper than before.
Concerning the supply of it, the increase was automatic”, que agora, não traduziremos para
não influenciar na maneira que vamos usar a técnica Scanning. Até porque, você já sabe das
traduções no fim do nosso material.

Vamos continuar a análise da alternativa “A”. As palavras “Dutch, produced, penicilina,


increase, automatic” – nos dá a ideia, na sequência que as lemos, escaneando-as, seguinte: os
holandeses produziram a penicilina e o aumento foi automático.

Bom, primeiramente, não foram apenas os holandeses. Fala-se no texto, de países baixos
(Netherlands) mas também há outros países envolvidos (EUA e Grã-Bretanha). E, não há, no
texto, nenhuma pista ou comprovação de que o aumento foi imediato. Não há a palavra
“imediato” nem sinônimo dela. Já poderíamos descartar essa alternativa. Mas, caso não
encontrássemos outra melhor, a analisaríamos novamente.

Na letra “B”, “The first clinical use of penicillin was not immediate owing to the impact of its
discovery. This delay changed the History of Medicine”, as palavras que ajudam a entender a
afirmação aqui são “clinical use of penicilin, not immediate, impact, Discovery e History of
Medicine”. Ligando esses termos, teríamos o resultado: “O uso clínico da penicilina... algo não
foi imediato... teve impacto... algo envolvendo descoberta e a história da medicina.” Ao
procurar a palavra ‘impacto’, percebemos que ela aparece no início do texto, junto à palavra
imediato – “...immediate and profound.” Se na letra “B”, afirma que não foi imediato, de
qualquer forma, está incorreta pois a palavra ‘imediato’ aparece para definir um fato e não
uma negação. Da mesma forma que fizemos na letra “A”, podemos descartar essa alternativa,
porém, analisa-la novamente se não encontrarmos outra melhor.

Aula 01 – Scanning, Skimming and False Cognates 46


www.estrategiamilitar.com.br 61
Teacher Andrea Belo
Aula 01: Escola Naval 2021

Na letra “C”, “The efforts by the Netherlands to produce the new drug weren't regarded as
valuable until around a decade ago”, fazendo uma leitura rápida para escanear as palavras
facilitadoras, podemos apontar: “Netherlands to produce, drug, weren’t, valuable e decade”,
que resultariam na ideia: Holandeses produzem, nova droga (no caso, a penicilina), negação de
algo – por causa do apóstrofo “n’t”, exemplos: don’t, didn’t, weren’t –, valorizada e década. No
texto, quando se fala de sucesso (success) há uma ligação particularmente com os holandeses
(This success... particularly in the Netherlands).

É um sinal de que essa pode ser a melhor opção. A pista da negação (weren’t) junto à palavra
década (decade), indicam que não houve valor (valuable) em uma década. No texto, é
exatamente isso – há informações desse ‘valor’ apenas nos últimos 10-15 anos (Information
about... in the last 10-15 years...) em relação aos holandeses. Isso nos certifica de que a
alternativa correta é a letra “C”.

Na letra “D”, “It is impossible that penicillin’s first clinical use dates back more than 75 years”,
o termo impossible – também “escaneada” com a técnica que estudamos – é uma palavra
perigosa. A menos que essa palavra esteja no texto, é difícil afirmar que algo é impossível se
isso não foi dito com clareza. Impossível é um termo forte, “pesado” para se usar e, geralmente,
é utilizado em alternativas falsas. Aqui, já podemos descartar essa alternativa porque sabemos
a correta, mas, poderíamos analisá-la novamente se não soubéssemos ainda.

Na letra “E”, “The US and Great Britain succeeded in producing penicillin right after 1943”, além
dessa afirmação não mencionar os holandeses, que aparecem no texto, a alternativa afirma,
escaneando palavras, que EUA e Grã-Bretanha obtiveram sucesso após 1943 (The US and Great
Britain succeeded right after 1943) e, o texto informa que não foi após mas no ano de 1943
(...incredibly successful by 1943.) E assim, sabemos que essa opção não está correta de acordo
com o texto e de acordo com as técnicas que usamos para ler as alternativas, uma por uma.

Conclusão fácil: Mesmo que você não soubesse as palavras do texto inteiro, seria ainda assim
possível marcar a alternativa “C” como correta se você aplicar as técnicas que expliquei
anteriormente. Logo, acertaria a questão.
Daí, a “não-necessidade” da tradução nesse momento e sim como complemento de seus
estudos ao final da aula.
Será válido para construção de estruturas, aprendizado de vocábulos dos mais variados tipos e
que, depois, ajudará em qualquer prova de qualquer área da Carreira Militar.

Questão 11 (Colégio Naval/2019) Texto para perguntas 11 e 12

Aula 01 – Scanning, Skimming and False Cognates 47


www.estrategiamilitar.com.br 61
Teacher Andrea Belo
Aula 01: Escola Naval 2021

Questão 11 (Colégio Naval/2019) – Read the extract from the text


“Ten minutes after you get to work and stop reading the morning paper”
Mark the option that can replace the expression get to
(A) drive to
(B) go to
(C) find
(D) arrive at
(E) come from

Aula 01 – Scanning, Skimming and False Cognates 48


www.estrategiamilitar.com.br 61
Teacher Andrea Belo
Aula 01: Escola Naval 2021

Comentários:
A frase diz que “10 minutos depois você chega no trabalho e para de ler o jornal” e pede para
marcar a opção que melhor substitui a expressão get to que nesta frase possui o sentido de
chegar. Analisemos alternativa por alternativa.
Na letra A, temos “drive to” que significa “dirige para”, dando um sentido de que ainda não
chegou no trabalho 10 minutos depois e que vai começar a dirigir para chegar. Mas a frase
original mostra que depois de 10 minutos você chega no trabalho. A frase fica “10 minutos
depois você dirige para o trabalho e para de ler o jornal”. Alternativa errada.
Na letra B, temos “go to” que significa que a pessoa ainda não chegou no trabalho também e
10 minutos depois e que vai começar a ir. A frase fica “10 minutos depois você vai para o
trabalho e para de ler o jornal”. Veja que você vai, não quer dizer que chega no trabalho.
Alternativa errada.
Na letra C, temos “find” que significa “encontra” e esta opção além de não ter um conectivo
não encaixa na frase. Veja como a frase ficaria “10 minutos depois você encontra trabalho e
para de ler jornal”. O sentido mudou totalmente e, portanto, alternativa errada.
Na letra D, temos “arrive at” que significa “chega no” e este sentido é exatamente o que
encaixa na frase sem que perca seu significado inicial. O verbo to arrive significa chegar,
exatamente o sentido mostrado na frase. Logo, alternativa correta é a letra D.
Na letra E, temos “come from” que significa “vem de” e esta opção não encaixa na frase, pois
ela fica “10 minutos depois você vem do trabalho e para de ler o jornal”. Perceba que o sentido
ficou inverso, como se o personagem saísse do trabalho. Portanto, alternativa errada.

Questão 12 (Colégio Naval/2019) – Read the extract from the text

“The year is 2020, and it’s 7:45 on a rainy Monday morning (...)”

What’s the correct question referring to the underlined information?


(A) What’s the weather?
(B) What’s the weather like?
(C) How’s the weather?
(D) How’s the climate like?
(E) What’s the climate?

Aula 01 – Scanning, Skimming and False Cognates 49


www.estrategiamilitar.com.br 61
Teacher Andrea Belo
Aula 01: Escola Naval 2021

Comentários:
A palavra “rainy” significa “chuvoso” e está caracterizando a manhã da segunda feira do texto.
Veja que esta palavra está descrevendo algo. Veja que a palavra “rainy” se refere a um
determinado período, portanto ela descreve o “tempo” (weather).

A palavra “climate” significa clima e se refere a uma condição do estado da atmosfera em um


longo período. Assim a melhor opção é a letra B que faz a pergunta “Como está o tempo? ” e
a resposta seria “chuvoso” (rainy). Vejamos as outras alternativas.

Na letra A, temos “What’s the weather? ” apesar de usar a palavra weather, a pergunta está
incompleta, pois quando queremos saber como está alguma coisa é necessário o uso da palavra
“like” no final da pergunta. Portanto, alternativa incorreta.

Na letra B, como já mostrado anteriormente alternativa correta é a letra B.

Na letra C, temos “How is the weather? ” apesar de usar a palavra weather, a pergunta está
incompleta novamente, pois quando queremos saber como está alguma coisa é necessário o
uso da palavra “like” no final da pergunta. Ficaria certo se fosse “Ho is the weather like? ”.
Portanto, alternativa incorreta novamente.

Na letra D, temos “How is the climate like? ” está errado ao usar a palavra climate que como
mostrado anteriormente se refere a períodos de longo prazo. Portanto alternativa incorreta.

Na letra E, como já mostrado anteriormente, a palavra climate se refere a períodos de longo


prazo. Além disso falta também o uso da palavra like para se saber como algo está. Portanto
alternativa incorreta.

Aula 01 – Scanning, Skimming and False Cognates 50


www.estrategiamilitar.com.br 61
Teacher Andrea Belo
Aula 01: Escola Naval 2021

Considerações finais
Outra aula concluída, ufa!!! Mais um passo até a sua aprovação! As técnicas Scanning e
Skimming ficaram mais claras em relação ao seu melhor uso. E os falsos cognatos, analisados com
maior cuidado, não é mesmo?
Continuaremos a estudar os conteúdos de forma minuciosa e prática, com sucesso!
É importante lembrar de fazer listas de vocabulário das palavras que você achou difíceis a
cada aula, em cada exercício ou lista, a fim de reescrevê-las e então, recordá-las nos momentos de
pausa entre as aulas.
Minha sugestão é que você faça a leitura dessas palavras consideradas “novas” para vê-las
novamente. Isso te ajudará nas questões em que esses vocábulos reaparecem. Acontece muito com
a classe dos verbos, por exemplo.
A cada lista de exercício resolvida ou mesmo a cada exercício que você faça, perceberá como
fica mais fácil identificar um verbo já visto no tempo passado ou particípio.
É sua conquista de etapas e que tornará você, um candidato mais bem preparado e confiante
para realizar uma excelente prova.
É importante lembrar também do nosso Fórum de dúvidas e outras redes sociais
complementares para que seus estudos avancem cada vez mais.

Aula 01 – Scanning, Skimming and False Cognates 51


www.estrategiamilitar.com.br 61
Teacher Andrea Belo
Aula 01: Escola Naval 2021

Referências bibliográficas

BARRETO, Tania Pedroza; GARRIDO, Maria Line; SILVA, João Antenor de C., Inglês Instrumental.
Leitura e compreensão de textos. Salvador, Ba UFBA, 1995, p. 64.
BROWN. H. Douglas. Principles of Language Learning and Teaching. Prentice Hall International, 1988.
COMPEDELLI, Samira Yousseff. Português, Literatura, Produção de texto & Gramática – São Paulo: Ed.
Saraiva, 2002.
CORREIA, Clese Mary P. Reading Specific Purposes. Salvador/ Ba: UFBA, 1999.
COSTEIRA, Adriana Araújo de M. Reading Comprehension Skills. João Pessoa/PB: ETFP, 1998.
CRYSTAL David. Cambridge University Press 1997. The Cambridge Encyclopedia of Language.
Cambridge University Press 1997
FREEMAN. Diane Larsen. MURCIA. Marianne Celce. The Grammar Book, 1999.
DYE, Joan., FRANFORT, Nancy. Spectrum II, III A Communicative Course in English. USA, Prentice Hall,
1994.
FAVERO, Maria de Lourdes Albuquerque (org.). Dicionário de educadores no Brasil: da colônia aos
dias atuais. Rio de Janeiro : UFRJ, MEC, INEP, 1999.
FRANKPORT, Nancy & Dye Hoab. Spectrum II, III Prentice Hall Regents Englewood Cliffs, New Jersy,
1994.
GADELHA, Isabel Maria B. Inglês Instrumental: Leitura, Conscientização e Prática. Teresina: EDUFFI,
2000.
GUANDALINI, Eiter Otávio. Técnicas de Leitura em Inglês: ESP – English For Specific Purposes: estagio
1. São Paulo: Texto novo, 2002.
GRELLET, Françoise. Developing Reading Skills. Cambridge University Press, 1995
HOLAENDER, Arnon & Sanders Sidney. A complete English Course. São Paulo. Ed. Moderna, 1995.
HUTCHINSON, Tom & WATERS, Alan. English for Specific Purposes. Cambridge: Cambridge University
Press, 1996
KRASHEN. Stephen D. Second Language Acquisition and Second Language Learning, Prentice-Hall
International, 1988.
LAENG, Mauro. Dicionário de pedagogia. Lisboa : Dom Quixote, 1973.
LEFFA, Vilson J. Metodologia do ensino de línguas. In: BOHN, H.; VANDRESEN, P. (org.). Tópicos de
linguística aplicada: o ensino de línguas estrangeiras. Florianópolis: Editora da UFSC, 1988. p. 211-
231.

Aula 01 – Scanning, Skimming and False Cognates 52


www.estrategiamilitar.com.br 61
Teacher Andrea Belo
Aula 01: Escola Naval 2021

LIBERATO, Wilson. Compact English Book Inglês Ensino Médio. São Paulo: FTD, Vol. Único, 1998
Mc ARTHUR. The Oxford Companion to the English Language. Oxford University Press 1992
Fromkin. Victoria. An Introduction to Language
MARQUES, Amadeu. Inglês Série Brasil. ed. Atica. São Paulo: 2004. Vol. Único.
MURPHY, Raymond: Essencial Grammar in Use Oxford. New York Ed. Oxford University, 1997.
OLIVEIRA, Luciano Amaral. English For Tourism Students. Inglês para Estudantes de Turismo: São
Paulo, Rocca, 2001.
OLIVEIRA, Sara Rejane de F. Estratégias de leitura para Inglês Instrumental. Brasília: UNB, 1994.
QUINTANA, et alli. First Certificate. Master Class Oxford. New York, 2004: Ed. Oxford University.
PAULINO, Berenice F. et all. Leitura em textos em Inglês – Uma Abordagem Instrumental. Belo
Horizonte: Ed. Dos Autores, 1992.
PEREIRA, Edilberto Coelho. Inglês Instrumental. Teresina: ETFPI, 1998.
RODGES, Theodore. Jack C. Richards. Approaches and Methods in Language Teaching. Cambridge
University Press, 2001.
RODMAN Robert. Harcourt Brace 1993. English as a Global Language
STEWART, B., HAINES S. First Certificate, MasterClass. UK – Oxford 2004.
SILVA, João Antenor de C., GARRIDO, Maria Lina, BARRETO, Tânia Pedrosa. Inglês Instrumental:
Leitura e Compreensão de Textos. Salvador: Centro Editorial e Didático, UFBA. 1994
SOARES, Moacir Bretãs. Dicionário de legislação do ensino. 19.ed. Rio de Janeiro : FGV, 1981.
SOUZA, Adriana Srade F. Leitura em Língua Inglesa: Uma abordagem Instrumental. São Paulo: Disal,
2005.
TUCK, Michael. Oxford Dictionary of Computing for Learners of English. Oxford: Oxford University
Press, 1996.
TOTIS, Verônica Pakrauskas. Língua Inglesa: leitura. São Paulo: Cortez, 1991.
Livros eletrônicos:
Dicionário Houaiss da Língua Portuguesa, Editora Objetiva, 2001.
MOURãO, Janaína Pereira. "Skimming x Scanning"; Brasil Escola. Disponível em
<https://brasilescola.uol.com.br/ingles/skimming-x-scanning.htm>. Acesso em 20 de março de 2019.
www.newsweek.com - Acesso em 18 de março de 2019.
http://www.galaor.com.br/tecnicas-de-leitura/ - Acesso em 19 de março de 2019.
Expressões Idiomáticas (continuação)" em Só Língua Inglesa. Virtuous Tecnologia da
Informação,2008-2019. Consultado em 03/04/2019 às 22:09. Disponível na Internet em
http://www.solinguainglesa.com.br/conteudo/Expressoes5.php

Aula 01 – Scanning, Skimming and False Cognates 53


www.estrategiamilitar.com.br 61
Teacher Andrea Belo
Aula 01: Escola Naval 2021

Traduções

O Ponto Principal da Depressão no Facebook


O Facebook, a plataforma de mídia social mais popular, em si mesmo não torna as
pessoas mais deprimidas. Em vez disso, o que a pesquisa mostra é que o Facebook -
quando usado como um dispositivo de monitoramento - leva a um maior risco de
sentimentos de inveja.
E quanto mais esses sentimentos de inveja aumentam, mais provável é que uma
pessoa comece a sentir-se deprimida. A chave para parar esses sentimentos é não
usar o Facebook principalmente como um método de vigilância para espionar a vida
de sua família e amigos.
Em vez disso, use-o como uma rede social onde você compartilha suas próprias
informações, fotos e atualizações, além de fazer uso de outras atualizações e
compartilhamentos. O uso saudável do Facebook irá protegê-lo contra a
possibilidade de se sentir mais deprimido depois de usá-lo. É uma coisa simples que
você pode tentar por si mesmo - especialmente se você se sentir mais invejoso
depois de verificar o Facebook.

Aula 01 – Scanning, Skimming and False Cognates 54


www.estrategiamilitar.com.br 61
Teacher Andrea Belo
Aula 01: Escola Naval 2021

O desafio de realizar futuras operações militares dentro das megacidades (cidades com mais de dez
milhões de habitantes) consiste em compreender as complexidades dinâmicas e multidimensionais
dessas áreas urbanas. As operações militares em megacidades, seja de combate ou de outra forma,
serão semelhantes às de outros ambientes urbanos, mas serão complicadas por fatores únicos ao
meio ambiente. Em primeiro lugar, as megacidades são em grande parte multilíngues. Embora isso
possa ser dito das grandes cidades em geral, a escala do multilinguismo nas megacidades amplifica
seus efeitos. Por exemplo, na cidade de Nova York (NYC) - uma megacidade metropolitana de mais
de dezoito milhões de pessoas - nove línguas estrangeiras são faladas por comunidades de cem mil
ou mais. A língua também desempenha um papel na determinação da identidade e da comunidade
linguística em que se decide viver.
Por exemplo, a maioria dos russos em Nova York tende a viver no sul de Brooklyn e em Staten Island,
enquanto os que falam chinês tendem a se agrupar em Manhattan e em Sunset Park. Nas
megacidades, a língua, a cultura e o contexto regional andam de mãos dadas e muitas vezes
ultrapassam as identidades étnicas. Para entender completamente o contexto de uma megacidade,
devemos entender o papel das línguas usadas em suas comunidades. Como as comunidades
linguísticas interagem em megacidades? Que tensões são causadas por múltiplas comunidades
linguísticas no espaço urbano? Qual o papel que a língua desempenha nas estruturas de poder
(governo ou não) das megacidades?

Aula 01 – Scanning, Skimming and False Cognates 55


www.estrategiamilitar.com.br 61
Teacher Andrea Belo
Aula 01: Escola Naval 2021

O ano é 2020 e são 7:45 em uma manhã chuvosa


de segunda-feira e você está no seu carro e a
caminho do trabalho. Você vira à esquerda e
depois, vira à direita. Alguns minutos depois, você
para em um semáforo. Quando a luz fica verde e
não há outros carros no cruzamento, você
continua no seu caminho. Dez minutos depois,
você chega ao trabalho e para de ler o jornal da
manhã. Então, você sai do seu carro e diz:
"Obrigado!" Seu carro responde: "De nada!" Esse
possível futuro pode parecer irreal, mas na
verdade muitas empresas de automóveis já estão
testando carros robóticos sem motorista carros ',
nas estradas hoje (embora os carros ainda não
falem muito).
Nos anos 80, a Alemanha e os Estados Unidos
testaram os primeiros carros sem motorista e, por
empresas de 2020, como Volvo, GM, Nissan e
BMW, planejam vender carros sem motorista.
Carros sem motorista não são realmente "sem
motorista" - os "motoristas" são computadores
que usam radar, mapas de computador e outras
tecnologias modernas. Eles oferecem muitas
vantagens.
Talvez o mais importante deles seja menos mortes
causadas por acidentes de viação. Para Por
exemplo, em 1968, mais de 53.000 pessoas
perderam a vida em acidentes de carro no país.
Nos EUA Este número caiu para menos de 33.000,
mas ainda é um número alto. E, além disso, as
pessoas gastam menos tempo presas em
engarrafamentos e não haverá necessidade de
pessoas a ter uma carta de condução. Uma das
principais desvantagens da nova tecnologia, no
entanto, é o custo. Não é grátis US $ 5.000 a US $
10.000 são adicionados a o preço de um carro
novo. No entanto, em algum momento de sua vida,
você provavelmente estará sentado em um carro
robótico e sem motorista, a caminho do trabalho
ou da escola. O futuro está quase aqui. você está
pronto para isto?

Aula 01 – Scanning, Skimming and False Cognates 56


www.estrategiamilitar.com.br 61
Teacher Andrea Belo
Aula 01: Escola Naval 2021

I – Entender você mesmo, o que Spacial – Visualizar o Naturalista: Entender, conviver


você sente e o que você quer. mundo em 3D. com as coisas e ler a natureza.

W
Os tipos de Musical: Discernir sons, batidas,
tons, ritmo e timbre.
II – Encontrar as palavras
certas para expressar o que inteligência
When
você quer dizer.
Lógico-matemático: Quantificar
– por Mark Vital coisas, fazer hipóteses e prová-las.

hen
I – Corporal Cinestésica:
Coordenar sua mente e
corpo.
III – Sensibilizando-se com os
sentimentos e razões das pessoas.
en
IV – Enfrentar questões sobre o
porquê de nossas vidas e morte.

hen hen hen


Aula 01 – Scanning, Skimming and False Cognates 57
www.estrategiamilitar.com.br 61
Teacher Andrea Belo
Aula 01: Escola Naval 2021

Aula 01 – Scanning, Skimming and False Cognates 58


www.estrategiamilitar.com.br 61
Teacher Andrea Belo
Aula 01: Escola Naval 2021

Após pouco mais de 75 anos de uso clínico da penicilina, o mundo pode ver que seu
impacto foi imediato e profundo. Em 1928, um evento casual no laboratório de
Alexander Fleming em Londres mudou o curso da medicina. No entanto, a
purificação e o primeiro uso clínico da penicilina levariam mais de uma década. A
cooperação sem precedentes dos Estados Unidos / Grã-Bretanha para produzir
penicilina foi incrivelmente bem-sucedida em 1943. Esse sucesso ofuscou os esforços
para produzir penicilina durante a Segunda Guerra Mundial na Europa,
principalmente na Holanda. As informações sobre esses esforços, disponíveis apenas
nos últimos 10 a 15 anos, fornecem novas ideias sobre a história do primeiro
antibiótico. Pesquisadores na Holanda produziram penicilina usando seus próprios
métodos de produção e comercializaram em 1946, o que acabou aumentando a
oferta de penicilina e diminuindo o preço. A incomum coincidência envolvida na
descoberta da penicilina demonstra as dificuldades em encontrar novos antibióticos
e deve lembrar aos profissionais de saúde o gerenciamento hábil desses
medicamentos extraordinários.

Aula 01 – Scanning, Skimming and False Cognates 59


www.estrategiamilitar.com.br 61
Teacher Andrea Belo
Aula 01: Escola Naval 2021

- Quantas cervejas
você bebeu? - Você está certa! Eu vou
- Hagar, você tem um contratar um professor
problema sério! de matemática!
- Humm, três? Cinco?
Oito? Seis? Quatro?

Aula 01 – Scanning, Skimming and False Cognates 60


www.estrategiamilitar.com.br 61
Teacher Andrea Belo
Aula 01: Escola Naval 2021

5 formas de obter mais fibras em sua dieta:


As fibras ajudam a reduzir o risco de doenças cardíacas e de cancro das entranhas, contudo poucas
pessoas ingerem-nas corretamente. Aqui mostramos como fica sua ingestão.
FOTO MAÇÃS
A fibra, substância fibra, refere-se aos carboidratos indigestos. Uma dieta rica em fibras está ligada a
benefícios à saúde, incluindo a redução dos riscos de doenças cardíacas e câncer de intestino.
Enquanto as diretrizes do Reino Unido dizem que adultos devem consumir 30g por dia, menos de 1
pessoa em 10 atingem esse objetivo. Entenda o que, em sua comida, pode te ajudar: uma maçã típica
tem de 2 a 3 gramas de fibra e um pãozinho sesame bagel tem 4 gramas. Jo Greening, quem
representa a BDA, Associação Britânica da Dieta, diz que vale a pena checar os rótulos, o quanto
diferentes marcas tem diferentes níveis de fibra.

Aula 01 – Scanning, Skimming and False Cognates 61


www.estrategiamilitar.com.br 61
,

Aula 02
Verb Tenses

Escola Naval 2021

Teacher Andrea Belo


Teacher Andrea Belo
Aula 02: EN - Verb Tenses

Sumário

Apresentação ..............................................................................................................................................................................3
2.Verb to be .................................................................................................................................................................................4
1. Simple Present ...................................................................................................................................................................7
2. Simple Past .........................................................................................................................................................................9
3. Future: will x going to .......................................................................................................................................................12
4. Gerund .............................................................................................................................................................................15
5. Present Continuous ou Present Progressive .....................................................................................................................18
6. Past Continuous ou Past Progressive ................................................................................................................................19
7. Present Perfect .................................................................................................................................................................20
8. Past Perfect ......................................................................................................................................................................22
9. Future Perfect...................................................................................................................................................................24
10. Present Perfect Continuous ...........................................................................................................................................26
11. Past Perfect Continuous ................................................................................................................................................28
12. Future Perfect Continuous ............................................................................................................................................30
13. Modal verbs ..................................................................................................................................................................32
Verbo Modal CAN ..........................................................................................................................................................................32
Verbo Modal Could ........................................................................................................................................................................33
Verbo Modal May ..........................................................................................................................................................................33
Verbo Modal Might .......................................................................................................................................................................34
Verbo Modal Must .........................................................................................................................................................................34
Verbos Modais Should/Ought to ...................................................................................................................................................35
Verbo Modal Shall .........................................................................................................................................................................36
Verbos Modais Will e Would .........................................................................................................................................................36
14. Imperative tense ...........................................................................................................................................................37
15. Phrasal Verbs – introdução ...........................................................................................................................................39
16. Questão inédita ............................................................................................................................................................40
17. Questões de anos anteriores .........................................................................................................................................42
18. Gabarito ...............................................................................................................................................................................48
19. Exercícios respondidos e comentados ..................................................................................................................................49
18. Considerações finais ......................................................................................................................................................60
Referências bibliográficas ..........................................................................................................................................................61
Traduções ..................................................................................................................................................................................63

Aula 02: Exploring Verb Tenses in texts 2


www.estrategiamilitares.com.br
67
Teacher Andrea Belo
Aula 02: EN - Verb Tenses

Apresentação
Chegou a vez da nossa aula de verbos, uma das mais importantes de todo o material. Se você
identifica o verbo e consegue entendê-lo no contexto em que ele aparece, os resultados são
garantidos. E isso é muito importante. As interpretações são essenciais, é claro.
O vocabulário também. Mas os verbos são a “alma” da frase, eles apresentam as ideias do
texto e nos levam ao assunto, ao tema, ao que de fato se tratam os textos. Vamos estudá-los!

Verbo é a classe de palavras que exprime ação, que indicam acontecimentos representados
em um determinado tempo. Originada do latim, “verbum” significa, de fato, “palavra”.
Muitas pessoas acreditam que aprender as conjugações dos verbos seja complicado. Mas não
é. Em primeiro lugar, os verbos são essenciais para ajudar na interpretação.
Quanto mais enraizadas são as regras verbais, mais naturalmente você as usará no dia da
prova, extraindo os verbos dos textos e demarcando-os, para realizar uma leitura global do texto em
questão, encontrando as respostas procuradas.

E, saber os verbos, ajudará você a extrapolar o uso da língua inglesa na hora da prova, além
de aumentar a possibilidade de aplicá-los nos contextos exigidos nos exercícios. Isso porque, ao
testar o seu raciocínio e a sua capacidade de compreender textos em Inglês, os verbos e suas devidas
conjugações, em cada tempo verbal, economizam seu tempo e direcionam a sua atenção ao que
deve ser respondido.
Uma dica interessante é reconhecer o verbo assim que você ler cada frase do texto, seja qual
for a forma que a leitura for apresentada.
O verbo vem logo após o sujeito, que executa a ação. Por exemplo, se a frase é “The doctors
work at the hospital”, quem realiza a ação são os médicos (doctors) e a ação realizada é trabalhar
(work), que é o nosso verbo. Certo?
Eis que estamos diante de uma classe de palavras que favorece você a construir seus
pensamentos: os verbos, por excelência! Ao analisá-los, devido à importância que eles têm,
estudaremos exemplos juntos, com suas peculiaridades e diferentes flexões, ampliando sua
competência linguística e fazer bom uso do aprendizado em sua prova.
Vamos, então, passar por todos os tempos verbais, esclarecendo dúvidas e lembrando que,
alguns tempos verbais são poucos explorados nas provas mas, vamos “passar por eles” rapidamente
para que o conteúdo fique completo. E, podem “cair” em alguma oportunidade. Let’s go!

Aula 02: Exploring Verb Tenses in texts 3


www.estrategiamilitares.com.br
67
Teacher Andrea Belo
Aula 02: EN - Verb Tenses

2.Verb to be

O verbo to be é aquele assunto que as pessoas definem como algo que se estuda a vida inteira
e ainda assim não sabe ao certo como se usa.
É um verbo ensinado todas as vezes que se inicia um curso de Inglês e, por esse motivo, muita
gente considera “chato” estudar Inglês para iniciantes.
O verbo to be tem sua importância e vou deixar claro como se usa e o porquê dessa
importância. To be significa ser ou estar. Não existe uma regra para saber se, na frase, é ser ou estar,
depende do contexto e a ideia sobre o que se refere.
Gosto de dizer que o to be não é um verbo difícil e sim exclusivo, já que pode ser usado em
diferentes frases, tanto como verbo principal quanto como verbo auxiliar.

O que diferencia o to be dos demais verbos da língua inglesa, é que em todos os outros,
utilizamos a raiz para fazer frases, o to be muda por inteiro.
Veja – verbo jogar (to play) I play, You play (Eu jogo, você joga) ou o verbo dançar (to dance)
I dance, you dance, they dance (Eu danço, você dança, eles dançam) enquanto o verbo ser – Eu sou,
ele é, eles são fica: I am, he is, they are, sem ao menos usar as letras “be” para iniciar as conjugações.
Você não vai dizer “I be, you be, we be”, como na maioria nos outros verbos, 99% deles são
conjugados através da raiz, do radical.
Usado como verbo ser, as frases geralmente usam adjetivos (Eu sou alto/baixo – I am
tal/short) ou para dizer a profissão (Ele é engenheiro – He is an engineer) entre outros exemplos.
O verbo to be como “estar” expressará sentidos de ação ou de se estar em algum lugar – I am
happy. She is in the supermarket – Eu estou feliz. Ela está no supermercado.
Em todos os tempos verbais que você vai estudar aqui, terá a explicação, seguida das formas
afirmativa, negativa e interrogativa, para que você compreenda melhor o uso dos verbos.
Nas frases afirmativas, o verbo to be é simples, conforme estudamos e, muitas vezes, somos
obrigados a decorá-los (I am, you are, he is, she is, it is, you are, we are, they are).
O motivo pelo qual you are se repete é que as palavras “você” e “vocês”, em Inglês, são iguais:
you. Assim, na conjugação, you are significa você é/você está e também vocês são/vocês estão
dependendo do contexto.
Na forma interrogativa em Inglês, o verbo to be se posiciona no início da frase, antes do
sujeito. A conjugação fica: Am I? Are you?, Is he?, Is she?, Is it?, Are you?, Are we?, Are they?

Aula 02: Exploring Verb Tenses in texts 4


www.estrategiamilitares.com.br
67
Teacher Andrea Belo
Aula 02: EN - Verb Tenses

Na forma negativa, com a adição da partícula de negação “not” nos verbos, a conjugação fica:
I am not, you are not, he is not, she is not, it is not, you are not, we are not, they are not. Se esses
verbos aparecerem na forma abreviada, encontramos I’m not, you aren’t, he isn’t, she isn’t, it isn’t,
you aren’t, we aren’t, they aren’t. E também há as formas no tempo passado do verbo to be.
Preparei um esquema para resumir as três formas do TO BE no presente e no passado –
afirmativa, interrogativa e negativa – e, além do verbo to be, teremos os esquemas de todos os
tempos verbais que vamos estudar em cada capítulo de sua aula.
Ficará melhor para você visualizar e saber ler e encontrar os verbos em suas leituras. Vejamos
o to be:

Agora, o esquema no passado (Verb to be in the Past), assim como no presente, vejamos:

Aula 02: Exploring Verb Tenses in texts 5


www.estrategiamilitares.com.br
67
Teacher Andrea Belo
Aula 02: EN - Verb Tenses

PAST

Para ler, interpretar e encontrar as respostas corretas, é necessário que você saiba, além do
verbo to be, todo o conteúdo que vamos explorar no cronograma de estudos em nosso material.
Cada aula será um complemento para a próxima.
E você também precisa estar atento às notícias do Brasil e do mundo, ler jornais, revistas,
estar com seus estudos em dia e de forma constante.
Sempre digo que, ler textos das fontes usadas pela banca na hora de preparar as provas é um
dos exercícios importantes a se fazer.
Uma vez preparado para interpretar a questão completa, você pode realizar as provas de
qualquer instituição e se sair bem. Vamos à prática.
Vamos observar um texto que fez parte de uma questão das provas de Carreiras Militares,
com a presença do verbo to be várias vezes no texto e, o verbo to be indicaria a resposta,
dependendo da pergunta que aparecesse. Vejamos.
Agora, vamos estudar o tempo Present Simple e todos os outros tempos verbais necessários
para resolver sua prova com mérito, contextualizando gramática e vocabulário. Come on!

Aula 02: Exploring Verb Tenses in texts 6


www.estrategiamilitares.com.br
67
Teacher Andrea Belo
Aula 02: EN - Verb Tenses

1. Simple Present
Em Inglês, o Simple Present, tempo verbal Presente Simple, pode ser usado para expressar
uma ação habitual, aquilo que fazemos com frequência, por exemplo: I study every day (Eu estudo
todos os dias), I sometimes watch TV (Eu assisto TV às vezes), I often use the computer. (Eu uso o
computador com frequência) etc.
Usamos o Present Simple também para exprimir verdades, fatos imutáveis: Birds sing.
(Pássaros cantam), Babies need their moms. (Bebês precisam de suas mães) etc. Usamos esse tempo
também para informar situações, opiniões, fatos em geral: Technology grows day by day. (A
tecnologia cresce dia após dia), I love music. (Eu amo música) etc.
Temos que fazer um esclarecimento para facilitar o estudo de todos os tempos verbais. Você
sabe por que, ao se falar do verbo que será usado em uma frase, tem a preposição “to” antes dele?
Por exemplo, o sujeito I, o verbo to study e o complemento very much, formam a frase I study very
much – Eu estudo muito, mas o “to” não aparece na frase.
Isso porque, o verbo em sua forma original, no infinitivo, ou seja, sem conjugação, está
acompanhado da preposição “to” enquanto em uma frase, o verbo é conjugado e não usamos mais
o “to” antes dele. Se você procurar no dicionário os verbos ler, escrever e trabalhar – to read, to
write e to work. Mas, ao escrever as frases “Eu leio, Eu escrevo e Eu trabalho” fica: I read, I write e I
work.
Temos três formas em todos os tempos verbais: afirmativa, negativa e interrogativa. Há dois
auxiliares que acompanham as frases interrogativas no presente: Do e Does. Por exemplo, ao dizer
“Você trabalha? ou “Ela trabalha?” em Português, apenas colocamos o ponto de interrogação no fim
da frase. Na forma interrogativa em Inglês, precisamos adicionar “Do” no início da pergunta – Do
you work? (Você trabalha?) e, para sujeitos no singular, classificados como terceira pessoa do
singular (he/she/it), usamos “Does” – Does she work? (Ela trabalha?), demonstrando que as orações
estão no tempo presente.
Na forma negativa, com a adição da partícula de negação “not” nos auxiliares, eles se tornam
do not/don’t e does not/doesn’t, formas abreviadas ou não – I don’t work (Eu não trabalho), She does
not work ou She doesn’t work (Ela não trabalha), He does not work/He doesn’t work (Ele não
trabalha).
Dificilmente você encontra a explicação da existência desses auxiliares. Vou esclarecer e
justificar para você. É simples. Primeiro, os verbos em Inglês não tem terminações como em
Português – Eu estudo, tu estudas, ele estuda, nós estudamos, vós estudais, eles estudam – sendo
apenas “study” para todos os sujeitos e acréscimo de -s, -es ou -ies para terceiras pessoas – I study,
you study, she/he/it studies, we study, they study.
Segundo, se em Português dizemos “Ela estuda” e “Ela estuda?” igual, mudando apenas a
entonação, como saberíamos o tempo da frase se não fosse demarcada pelos auxiliares Do e Does –
Do you study? e Does she study? Faz sentido, não é mesmo?
E, nas negativas, enquanto em Português temos a presença do “não” em todas as frases – Eu
não trabalho, você não trabalha, ela não trabalha etc tanto para presente quanto no passado ou
futuro, veja:

Aula 02: Exploring Verb Tenses in texts 7


www.estrategiamilitares.com.br
67
Teacher Andrea Belo
Aula 02: EN - Verb Tenses

Exemplos: Eu não trabalhei, você não trabalhou, ele não trabalhará etc – como saberíamos o
tempo se não houvesse os auxiliares don’t e doesn’t demonstrando presente? – Do you work? e Does
she work? Entendeu? Got it?
As frases afirmativas são formadas por um sujeito, um verbo principal e o complemento, que
pode ser onde, quando aconteceu, com quem, porque ou qualquer outra informação que alguém
executou. Lembrando que, ao ser conjugado nas terceiras pessoas do singular (e/she/it), precisamos
acrescentar “s”, “ies” ou “es”, exemplos: I run. She runs. (Eu corro. Ela corre.)
As frases interrogativas são formadas por um auxiliar (Do para sujeito no plural ou Does para
sujeito no singular) no início da frase, um sujeito, um verbo principal e o complemento (onde,
quando aconteceu, com quem, porque ou qualquer outra informação), exemplos: Do you run? Does
he run? (Você corre? Ele corre?)
As frases negativas são formadas por um sujeito, auxiliar don’t ou doesn’t, um verbo principal
e o complemento (onde, quando aconteceu, com quem, porque ou qualquer outra informação),
exemplos: I don’t run. She doesn’t run. (Eu não corro. Ela não corre.)
Vejamos, como exemplo, o verbo estudar – TO STUDY, conjugado em todas as pessoas do
singular e plural nas três formas – afirmativa, negativa e interrogativa no Present Simple:

Agora, vamos aos estudos do tempo Past Simple e suas particularidades.

Aula 02: Exploring Verb Tenses in texts 8


www.estrategiamilitares.com.br
67
Teacher Andrea Belo
Aula 02: EN - Verb Tenses

2. Simple Past
Em Inglês, o Simple Past, tempo verbal Passado Simples, é usado para demonstrar uma ação
que já aconteceu e ficou no passado, tal como um jogo que acabou, um evento que passou ou
alguém que chegou, por exemplo.
Por isso, as frases no passado simples são geralmente acompanhadas de uma expressão de
tempo definida como yesterday, que significa ontem – I worked yesterday (Eu trabalhei ontem).
Para narrar ações que já ocorreram, além de yesterday, outras expressões mais comuns que
indicam o passado são: “last” - last night, last Sunday, last week (noite passada, domingo passado,
semana passada.
Outro termo é o “ago” – two years ago, ten minutes ago (dois anos atrás, dez minutos atrás)
etc.
É importante salientar que, a palavra “atrás” é usada no tempo passado demonstrando
justamente o tempo. Se fosse a preposição “atrás”, apontando o lugar seria “behind” (He is behind
me – Ele está atrás de mim), ok?
Estudaremos sobre isso na aula de preposições.
Temos também as três formas, como nos demais tempos verbais: afirmativa, negativa e
interrogativa.
Há apenas um auxiliar que acompanha as frases interrogativas e negativas: did/didn’t.
Por exemplo, ao dizer “Você trabalhou?”, adicionarmos “Did” no início da pergunta – Did you
work? para qualquer sujeito.
E usamos, em frases negativas: did not/didn’t, forma abreviada ou não – I didn’t work (Eu não
trabalhei), She did not work ou She didn’t work (Ela não trabalhou) etc.
Perceba que o verbo volta à sua forma original “work” tanto na forma interrogativa quanto
negativa e, só apresenta terminações ou diferenças em sua escrita na forma afirmativa.
A explicação da existência do auxiliar did também não é justificada e sim vista como
obrigatória no tempo passado.
Mas, assim como no presente, os verbos em Inglês não tem terminações como em Português
– Eu trabalhei, tu trabalhou, ele trabalhou, nós trabalhamos, vós trabalhastes, eles trabalharam –
sendo apenas “worked” para todos os sujeitos e acréscimo de -ed, para qualquer sujeito quando o
verbo for regular – I worked, you worked, she/he/it worked, we worked, they worked.
Explicarei, em seguida, o que acontece quando os verbos são irregulares. Felizmente, são
minoria e isso colabora com seus estudos.

Aula 02: Exploring Verb Tenses in texts 9


www.estrategiamilitares.com.br
67
Teacher Andrea Belo
Aula 02: EN - Verb Tenses

Verbos regulares são aqueles em que cujas terminações no tempo Past Simple apenas sofrem
o acréscimo das partículas “d” e “ed” na maioria dos verbos – She danced rock (Ela dançou rock –
verbo to dance – jogar).
Se o verbo terminar em vogal + a letra “y”, recebem “ed” – He played baseball (Ele jogou
beisebol). Mas, se terminar em consoante + a letra “y”, troca-se o “y”por “ied” – She cried yesterday.
(Ela chorou ontem – verbo to cry – chorar).
Caso o verbo termine com a sequência consoante/vogal/consoante, se dobra a última
consoante e também acrescenta “ed” - I preferred the blue pen. (Eu preferi a caneta azul – verbo to
prefer – preferir).
Se o verbo terminar com a vogal “e”, simplesmente recebe “d” – He arrived yesterday. (Ele chegou
ontem – verbo to arrive – chegar).
Verbos irregulares são aqueles em que as conjugações no tempo Past Simple sofrem diversas
alterações, como mudança das letras (por exemplo o verbo to write – escrever – se torna wrote),
acréscimo de letras (por exemplo o verbo to hear – ouvir – se transforma em heard), entre outras
modificações que acontecem.
Alguns verbos mudam completamente, como é o caso do verbo to buy – comprar – se
transforma em bought e o verbo to be – ser/estar – que se transforma em was/were para singular e
plural.
Por causa dessas transformações nos verbos irregulares, muitas pessoas acreditam que eles
sejam difíceis ou complicados.
E, na verdade, não há nada de complicado nisso.
Com o uso dos verbos irregulares nos exercícios diversos, eles vão se tornando familiares para
você.
E não podemos esquecer que a quantidade de verbos irregulares é bem menor que os
regulares.
Como eu já havia dito antes, 90% são os mais fáceis, com -ed acrescido no final deles.
Os verbos diferentes que passam por diversificações, além de ser apenas 10% da língua
inglesa, se repetem nos exercícios de provas de anos anteriores.
E, quanto mais questões você resolver e se dedicar aos estudos com leituras complementares
e muito esforço, passará a conhecê-los e se sentirá confiante com o passar do tempo.
Vejamos exemplos do verbo TO STUDY, conjugado nas três formas – afirmativa, negativa e
interrogativa no tempo Past Simple:

Aula 02: Exploring Verb Tenses in texts 10


www.estrategiamilitares.com.br
67
Teacher Andrea Belo
Aula 02: EN - Verb Tenses

O esquema ajuda a compreender melhor e memorizar cada estrutura, os auxiliares e,


consequentemente, lembrar das formas afirmativas, negativas e interrogativas quando aparecer nas
frases dos textos no dia da prova.
Vamos testar seus conhecimentos de tempo futuro agora. Let’s go!

Aula 02: Exploring Verb Tenses in texts 11


www.estrategiamilitares.com.br
67
Teacher Andrea Belo
Aula 02: EN - Verb Tenses

3. Future: will x going to


Quando se fala no tempo futuro, em Inglês, muitas pessoas dizem: futuro é “will” ou então
“going to”. E as perguntas sobre a diferença entre will e going to são frequentes.
As gramáticas, de uma forma geral, conseguem, incrivelmente, complicar estruturas tão
simples! Vamos simplificar e entender como pode ser simples, de fato.
Portanto, você vai aprender o uso de will e going to de um modo prático para acertar as
questões que envolvem o tempo futuro conectado com um vasto vocabulário e, com certeza, outros
tempos verbais, que estão sendo aprendidos aos poucos.
As frases com o auxiliar “will” são imediatas pois, colocando-se will antes do verbo, pronto –
transformou a ideia de presente em futuro, veja: – I will work (Eu trabalharei), She will dance (Ela
dançará). They will study (Eles estudarão).
Mas, se comparado ao “going to”, a dúvida é como usar um outro no tempo futuro em inglês.
Vamos esclarecer isso.
Usamos “will” quando vamos expressar algo no futuro indicando
uma certa incerteza e, muitas vezes, as frases em que o “will” é
bem aplicado, encontramos algumas expressões comuns no
tempo futuro, tais como I think (eu acho que…), probably
(provavelmente), I guess (eu acho) maybe (talvez).
Há outras, mas com essas são mais comuns em frases indicativas de futuro nas provas.
Vejamos exemplos com “will” e as expressões expostas acima.
Para ficar claro, vamos citar exemplos tais como “Eu provavelmente viajarei em dezembro” –
I will probably travel in December.
Outro exemplo: “Ela talvez viajará nas férias” – She will maybe travel on vacation. Se você
tem quase certeza do que vai fazer, se já planejou algo – não é uma regra seguida 100% das vezes –
mas é melhor usar o “going to”, como na frase “Planejei a viagem, vou no próximo sábado.”– I
planned the trip. I am going next Saturday.
As frases afirmativas são formadas por um sujeito, o auxiliar will, um verbo principal e o
complemento - “Ela vai dançar amanhã cedo” – She will dance tomorrow in the morning.
As frases interrogativas são formadas pelo auxiliar will no início da frase, um sujeito, um verbo
principal e o complemento (onde, quando aconteceu, com quem, porque ou qualquer outra
informação), exemplos: Will you run? Will he run? (Você vai correr? Ele vai correr?)
As frases negativas são formadas por um sujeito, auxiliar will not ou abreviado won’t, um
verbo principal e o complemento (onde, quando aconteceu, com quem, porque ou qualquer outra
informação), exemplos: I won’t run. She won’t run. (Eu não correrei. Ela não correrá.)

Aula 02: Exploring Verb Tenses in texts 12


www.estrategiamilitares.com.br
67
Teacher Andrea Belo
Aula 02: EN - Verb Tenses

Vejamos exemplos do verbo TO STUDY, conjugado no futuro simples com WILL no “esquema”
em todas as formas – afirmativa, negativa e interrogativa para fixar melhor.

Por sua vez, o going to é usado para expressar algo no futuro indicando uma certeza, planos
fixos, já definidos, por exemplo: I’m going to marry in 2021 – Eu vou me casar em 2021.
Nessa frase, a pessoa afirmou que vai se casar no ano de 2021 porque certamente já planejou
o casamento, marcou a data no cartório, preparou-se financeiramente para a festa e outros
elementos necessários para esse evento.
Quando fazemos uma previsão como um palpite, também é aconselhado o uso do going to,
por exemplo: It’s going to be an excellent year for me – Vai ser um excelente ano para mim.
As frases afirmativas são formadas por um sujeito, o verbo to be como auxiliar de cada sujeito
(I am, you are, he is, she is, it is, we are, they are), o verbo principal e o complemento – “She is going
to dance tomorrow. It is the Christmas school presentation. Ela vai dançar amanhã. É a apresentação
natalina na escola.”

Aula 02: Exploring Verb Tenses in texts 13


www.estrategiamilitares.com.br
67
Teacher Andrea Belo
Aula 02: EN - Verb Tenses

Percebemos que a pessoa que vai se apresentar foi ou está sendo preparada para o evento
que vai acontecer, houve um planejamento e, apresentações natalinas, acontecem todos os anos
nas escolas, algo que se realiza nas escolas no mês de dezembro.
Nas frases interrogativas, o verbo to be vai para a começo da pergunta. Em seguida, temos
um sujeito, o verbo principal e o complemento – Is she going to dance at Christmas school
presentation tomorrow? Ela vai dançar na apresentação natalina da escola amanhã? sendo que a
pergunta provavelmente foi feita com a certeza da resposta que sim, apenas para confirmar por
causa do uso do going to.
As frases negativas são formadas por um sujeito, o verbo to be seguido do not, o verbo
principal e o complemento – “She is not going to dance tomorrow/ She isn’t going to dance
tomorrow. They are not going to travel – Eles não vão viajar. Assim, as formas abreviadas são: is not
= isn’t (para terceiras pessoas do singular) e are not = aren’t (para plural em geral).
Vejamos exemplos do verbo TO STUDY, conjugado no futuro com GOING TO, em forma de
“esquema” em todas as formas – afirmativa, negativa e interrogativa para você fixar melhor.

Agora, estudaremos o gerúndio em nossa aula. Esse tempo verbal é muito importante na
construção de outras estruturas em que são necessários, tais como o gerúndio nos tempos da
vertente Continuous (Present e Past Continuous). Let’s go!

Aula 02: Exploring Verb Tenses in texts 14


www.estrategiamilitares.com.br
67
Teacher Andrea Belo
Aula 02: EN - Verb Tenses

4. Gerund
Em Inglês, o gerúndio é um pouco diferente do que conhecemos em Português. Faz parte de
estruturas em que agregamos a partícula -ing no fim dos verbos.
Porém, com algumas exceções que trataremos aqui.
Gerúndio, de um modo geral, pode ser definido com algo que transmite a ideia de ações
prolongadas ou ações ainda em desenvolvimento.
O gerúndio pode, por exemplo, transformar o verbo em substantivo, pode atuar como sujeito,
como predicado, e, na maioria das vezes, é usado para complementar verbos.
O gerúndio foi inserido nesse capítulo antes dos demais tempos verbais justamente porque
agora estudaremos o Present e o Past Continuous, cujas estruturas utilizam o gerúndio, além do
verbo to be, já estudado em nosso material.
Vejamos algumas regras ortográficas que precisam ser observadas ao
acrescentar -ing aos verbos. O gerúndio será sempre utilizado após
preposições, por exemplo:
“I have chances of being promoted in this company” (Eu tenho chances de
ser promovido nessa empresa).
Outro exemplo: Margareth has her reasons for behaving different - Margareth tem seus
motivos para comportar-se diferente.
Também usamos o gerúndio os verbos to go – ir e to come – vir, quando fizerem referência à
atividades físicas, tais como:
- go fishing, go bowling, go swimming, go skiing, go riding, go jogging, go shopping, go hiking,
go boating
Veja perguntas:
“ – I go swimming every Saturday “(Eu nado todos os sábados), “Will you come fishing with
me?” (Você virá pescar comigo?) e I don’t want to go bowling tonight (Eu não quero ir ao boliche
essa noite, por exemplo.
A palavra swimming, no exemplo acima, se refere ao verbo nadar e significa, de fato, nadar.
Mas, há também os casos em que os verbos com –ing no final, serão substantivos.
Por exemplo: Swimming helps me to relax (Nadar me ajuda a relaxar, como se fosse a natação,
o ato de nadar) e Reading is very important to the students (Ler é muito importante para os alunos,
como se fossem as leituras, o ato de ler importante).
As palavras nos exemplos com -ing, na verdade, tornaram-se sujeitos e não verbos.

Aula 02: Exploring Verb Tenses in texts 15


www.estrategiamilitares.com.br
67
Teacher Andrea Belo
Aula 02: EN - Verb Tenses

Há casos em que os verbos, necessitam -ing quando há duas ações, ou seja, dois verbos em
uma só frase.
São esses os exemplos:
“to admit, to avoid, to appreciate, to consider, to continue, to delay, to detest, to deny, to
dislike, to enjoy, to escape, to finish, to forgive, to imagine, to include, to keep, to mention, to miss,
to practice, to recommend, to resist, to risk, to suggest, to try, to understand e to quit.”
Esses verbos, quando inseridos em frases, necessitam que o segundo verbo, logo após deles,
tenham o acréscimo de -ing.
Com exercícios e muita prática em seus estudos, isso ficará fácil e natural para você.
Veja alguns exemplos: I admit getting angry sometimes (Eu admito que fico nervoso às vezes),
I enjoy studying English (Eu gosto de estudar Inglês) e They deny doing that (Eles negam que fizeram
aquilo).
Existe uma regra em que os verbos terminados pela letra “e”, perdem o “e”, ao usar -ing.
São exemplos os verbos to drive (dirigir) e to save (economizar), She is driving now (Ela está
dirigindo agora) e He is saving money for his future (Ele está economizando dinheiro para o seu
futuro).
Após algumas expressões em Inglês, precisamos usar o -ing como regra também.
São elas:
“ can’t stand, it’s worth, be used to, can’t help, feel like, it’s no good, look forward to, what
about, how about, it’s no use, in spite of.”
Vejamos exemplos:
“I can’t help laughing now” (Não consigo não rir agora), “I can’t stand explaining you
something thousands of times” (Não aguento explicar a você a mesma coisa mil vezes) e “It’s worth
visiting that museum” (Vale a pena visitar aquele museu).
Vejamos exemplos de alguns verbos, em forma de “esquema” no Gerúndio, conforme
as regras e exemplos de como melhor usar, de acordo com a teoria e explicações estudadas.

Aula 02: Exploring Verb Tenses in texts 16


www.estrategiamilitares.com.br
67
Teacher Andrea Belo
Aula 02: EN - Verb Tenses

Agora, estudaremos os tempos verbais da vertente Continuous (Present e também past


Continuous), que usam o gerúndio em suas estruturas para a elaboração de frases nas formas
afirmativa, negativa e interrogativa.
Assim, já ficará mais simples para compreender tais tempos verbais. Vamos lá!

Aula 02: Exploring Verb Tenses in texts 17


www.estrategiamilitares.com.br
67
Teacher Andrea Belo
Aula 02: EN - Verb Tenses

5. Present Continuous ou Present Progressive


Em continuação ao assunto gerúndio e o uso do -ing nos verbos, vamos falar do Present
Continuous ou Present Progressive, pois esse tempo verbal é conhecido nessas duas denominações.
O que você precisa saber, essencialmente, é que este tempo verbal é formado pelo verbo to
be e outro verbo, no caso, o verbo principal da frase. Isto significa que se você souber conjugar o
verbo to be e também souber o gerúndio dos verbos, a estrutura do Present Continuous está
formada.
As frases afirmativas são formadas por um sujeito, o verbo to be na afirmativa, o verbo
principal e o complemento - “Ela está estudando agora.” – She is studying now. Outros exemplos:
He is working at this moment. (Ele está trabalhando nesse momento), They are reading a magazine.
(Eles estão lendo uma revista).
As frases interrogativas são formadas pelo o verbo to be na forma afirmativa no início da
frase, o verbo principal e o complemento – “Ela está estudando agora?” – Is he working at this
moment? (Ele está trabalhando nesse momento?) e as frases negativas são formadas por um sujeito,
o verbo to be na forma negativa, o verbo principal e o complemento - “Ela não está estudando
agora.” – She is not/isn’t studying now.
Vejamos exemplos do verbo TO STUDY, no Present Continuous, em nosso “esquema”:

Vamos agora ao estudo do Past Continuous.

Aula 02: Exploring Verb Tenses in texts 18


www.estrategiamilitares.com.br
67
Teacher Andrea Belo
Aula 02: EN - Verb Tenses

6. Past Continuous ou Past Progressive


Falar do Past Continuous, também é o mesmo que falar de Past Progressive, pois esse tempo
verbal é conhecido nas duas formas.
O que você precisa saber, essencialmente, é que este tempo verbal é formado pelo verbo to
be, desta vez conjugado no passado e outro verbo, a ação principal da frase.
Mais uma vez, se você souber conjugar o verbo to be no passado e também souber o gerúndio
dos verbos, a estrutura do Past Continuous está formada.
As frases afirmativas são formadas por um sujeito, o verbo to be no passado, na forma
afirmativa, o verbo principal e o complemento - “Ela estava estudando.” – She was studying.
As frases interrogativas são formadas pelo o verbo to be no passado e na forma afirmativa no
início da frase, o verbo principal e o complemento – “Ela estava estudando?” – Was she working? e
Were they reading a magazine? (Eles estavam lendo uma revista?).
As frases negativas são formadas por um sujeito, o verbo to be no passado e na forma
negativa, o verbo principal e o complemento - “Ela não estava estudando.” – She was not/ wasn’t
studying. Vejamos nosso “esquema”, com o verbo TO STUDY, conjugado no Past Continuous.

Agora, estudaremos um tempo verbal muito importante,: o Present Perfect.

Aula 02: Exploring Verb Tenses in texts 19


www.estrategiamilitares.com.br
67
Teacher Andrea Belo
Aula 02: EN - Verb Tenses

7. Present Perfect
O Present Perfect é considerado difícil, é visto como algo complexo mas, basta entender a
maneira certa de usá-lo e encontrá-lo nas frases, que ele se torna mais simples do que parece.
Vou mostrar, na minha forma de ensinar, como pode ser descomplicado, ok?
Present Perfect é um tempo verbal que descreve uma ação em que estão conectados o passado
e o presente. Ou seja, o Present Perfect conta fatos que ocorreram em um tempo indefinido do
passado e ainda não foram concluídos.
Em Português, não temos um tempo que corresponda a esse. E, por isso, ao invés de dizer
“Tenho estudado para essa prova desde 2017”, as pessoas dizem “Eu estudo para essa prova desde
2017”, usando o presente para contar algo que já começou e ainda acontece, diferente em Inglês,
que o tempo verbal desse capítulo faz esse papel.
Por esse motivo, o Present Perfect é, muitas vezes, julgado e considerado um tempo verbal
complicado, difícil de aprender. Mas, como eu disse, ao compreender o uso certo, ficará simples.
Vejamos algumas regras de uso correto do Present Perfect. Em primeiro lugar, ações que “vêm
acontecendo recentemente”, por exemplo: I have been sad recently (Ando triste recentemente/
Tenho estado triste recentemente) ou They have run every day at the park (Eles correm todos os
dias no parque/Eles têm corrido todos os dias no parque).
Outro uso do Present Perfect: ações que acabaram de acontecer - We have just finished our
work. (Nós acabamos de terminar nosso trabalho) e She has just looked that magazine. (Ela acabou
de olhar aquela revista).
E, uma das formas mais comuns de se encontrar o Present Perfect é quando algo aconteceu
em um momento indefinido, como: You have played video game for a long time. (Você joga/tem
jogado vídeo game por muito tempo) e I have helped you a lot. (Eu ajudo/tenho ajudado você
bastante).
As frases afirmativas têm a seguinte estrutura: um sujeito, um verbo auxiliar have/has
(terceiras pessoas do singular) e o verbo principal no particípio passado - She has studied “Ela tem
estudado”, They have worked very much. (Eles têm trabalhado muito).
Para verbos regulares, o particípio passado apenas acrescenta –ed, assim como no Past
Simple de verbos regulares. É igual. E isso é ótimo porque facilita bastante na hora da prova.
Se, por sua vez, o verbo for irregular, o particípio não segue nenhuma regra, sendo
geralmente feita troca de letras ou acréscimo de -en no final deles, como por exemplo to write
(escrever), que no passado é wrote, no particípio fica written para qualquer sujeito.
Pelo menos isso, não é? Veja: He has eaten a lot these days – Ele tem comido muito esses
dias. O verbo to eat (comer), que no passado fica ate, por ser irregular, no particípio, muda para
eaten mas o usamos para todos os sujeitos, assim como os regulares.

Aula 02: Exploring Verb Tenses in texts 20


www.estrategiamilitares.com.br
67
Teacher Andrea Belo
Aula 02: EN - Verb Tenses

As frases interrogativas são formadas pelo verbo auxiliar have/has no início das perguntas,
um sujeito e o verbo principal no particípio - Has she studied “Ela tem estudado?”, Have they worked
very much? (Eles têm trabalhado muito?). Aqui também, usa-se o mesmo verbo para todos os
sujeitos.
As frases negativas são formadas por um sujeito, um verbo auxiliar have/has na forma
negativa has not/hasn’t e have not/haven’t e o verbo principal no particípio - She hasn’t studied “Ela
não tem estudado”, They haven’t worked very much. (Eles não têm trabalhado muito).

Vejamos nosso “esquema”, com o verbo TO STUDY, conjugado no Present Perfect:

Agora, estudaremos o Past Perfect em nossa aula. Esse tempo verbal também é geralmente
dito como sendo complicado por fazer arte dos tempos de denominação “Perfect”.
E você verá que é simples e depois vamos exercitar com muitas questões com esses tempos
verbais dentro dos textos. Basta compreender as regras e como usá-los.

Aula 02: Exploring Verb Tenses in texts 21


www.estrategiamilitares.com.br
67
Teacher Andrea Belo
Aula 02: EN - Verb Tenses

8. Past Perfect
O Past Perfect é um tempo verbal que descreve uma ação no passado, que ocorreu antes de
outra. Como assim? Bom, o Past Perfect conta fatos correlacionados com outros que aconteceram.
É simplesmente dizer o que houve em decorrência de outro fator, como por exemplo: “Fui
promovido porque vendi/tive vendido muito esse mês – I got the promotion because I had sold a lot
this month. Ser promovido só aconteceu porque a pessoa vendeu muito, antes de receber a
promoção.
Então, “vender muito” foi expresso, no Past Perfect, como a ação anterior ao ganho da
promoção (had sold = got the promotion), certo?
Assim como o Present Perfect, o Past Perfect tem suas regras de uso e facilitará para você
encontrá-lo nos textos e entender por que foi usado naquele momento.
Em primeiro lugar, ações que aconteceram por causa de outras, como vimos no parágrafo
anterior. Então, para ficar claro, veja: O criminoso fugiu. Então, a polícia chegou. A polícia só chegou
depois que o criminoso fugiu. Logo, o fato de a polícia chegar será expresso no Past Simple enquanto,
o criminoso fugir, que aconteceu antes, estará no Past Perfect: The criminal had run away when the
police arrived. (to run away = fugir/to arrive = chegar).
Usamos Past Perfect para fatos que “tinham/haviam acabado” de acontecer, com o uso do
advérbio just, como também vimos no Present Perfect - We had just left when you called. (Nós
tínhamos acabado de partir quando você ligou).
E, outros advérbios que encontramos no Past Perfect são: already, when, by the time, never,
ever, before, after, para enfatizar a ideia de que a ação estava totalmente acabada antes da que será
descrita: She had already decided not to go. (Ela já tinha decidido-+-- não ir.) e I asked my friend if
he had ever gone to London. (Perguntei ao meu amigo se ele já tinha ido em Londres).
As frases afirmativas têm a seguinte estrutura: um sujeito, o verbo auxiliar had (para todos
os sujeitos) e o verbo principal no particípio passado - She had studied “Ela tinha estudado”, They
had worked very much. (Eles tinham trabalhado muito).
Aqui também, mesma observação: para verbos regulares, o particípio passado apenas
acrescenta –ed e, quando verbo é irregular, o particípio não segue nenhuma regra, como vimos os
verbos to write (escrever), que no passado é wrote, no particípio fica written.
As frases interrogativas são formadas pelo verbo auxiliar had no início das perguntas, um
sujeito e o verbo principal no particípio - Had she studied “Ela tinha estudado?”, Had they worked
very much? (Eles tinham trabalhado muito?).
As frases negativas são formadas por um sujeito, um verbo auxiliar had na forma negativa
had not/hadn’t e o verbo principal no particípio - She hadn’t studied “Ela não tinha estudado”, They
hadn’t worked very much. (Eles não tinham trabalhado muito).

Aula 02: Exploring Verb Tenses in texts 22


www.estrategiamilitares.com.br
67
Teacher Andrea Belo
Aula 02: EN - Verb Tenses

Vejamos nosso “esquema”, com o verbo TO STUDY, conjugado no Past Perfect:

Agora, estudaremos o último tempo verbal Perfect, o Future Perfect.


E você verá que esse tempo verbal também é simples. Basta saber como usá-lo.
Como eu disse antes, algumas estruturas e formas verbais são dificilmente encontradas nas
provas e outras são mais comuns, como o Present Perfect, que acabamos de ver.
Veremos todas para não haver dúvidas.

Aula 02: Exploring Verb Tenses in texts 23


www.estrategiamilitares.com.br
67
Teacher Andrea Belo
Aula 02: EN - Verb Tenses

9. Future Perfect
O Future Perfect é usado para expressar ações que vão terminar, em um certo tempo no
futuro, como se você já soubesse ou como se estivesse prevendo o que vai acontecer. Vamos
aprender esse tempo verbal através de exemplos.
Em uma frase, se queremos dizer que o vôo de uma pessoa será às 20h, por exemplo, não há
tempo de chegar no aeroporto e embarcar às 20h se a pessoa sair de casa às 19:30h. Então, diremos:
Quando ela chegar no aeroporto, o avião já terá partido – When she gets there, the plane will have
left.
Outro exemplo para ficar mais claro: By next month, I will have finished my book. (No próximo
mês, eu terei terminado meu livro). Percebemos que, o sujeito, que provavelmente seja o escritor
do livro, fez uma previsão para o término de sua ação, dizendo que o livro estará terminado no
próximo mês.
Por isso, no Future Perfect, é comum encontrarmos expressões temporais que acompanhem
as frases, tais como: before (antes), by (em, no, na), by the time (quando) etc. Como eu já disse, o
Future Perfect se refere a eventos que serão terminadas em determinado ponto do futuro, ou seja,
em um tempo posterior ao do momento da ação principal da frase.
As frases afirmativas têm a seguinte estrutura: um sujeito, o verbo auxiliar que representa o
futuro: will, o verbo auxiliar to have (para todas as pessoas como sujeito) e o verbo principal no
particípio - She will have arrived there before you notice it (Ela terá chegado lá antes que você
perceba), demonstrando que ela saiu e vai chegar em um determinado lugar antes que a outra
pessoa perceba.
As frases interrogativas são formadas pelo auxiliar will, dessa vez antes dos sujeitos, logo no
início das frases, o sujeito, o verbo to have e o verbo principal no particípio – Will she have arrived
at the airport by 8pm? (Ela terá chegado no aeroporto às 8pm?).
E as frases negativas são formadas pelo sujeito, auxiliar will, dessa vez na forma negativa, will
not/won’t, o verbo to have e o verbo principal no particípio – She will not have arrived at the airport
by 8pm (Ela não terá chegado no aeroporto às 8pm).
Vale ressaltar que, para construir frases no Future Perfect, não importa se as ações serão
realmente concluídas no futuro: o que importa é a projeção que o sujeito vai fazer para usar tal
tempo verbal, veja: The boy will have paid all his debts in December – significando O garoto terá pago
todas as suas dívidas até Dezembro. Mesmo que ele não pague seus débitos até o fim do ano, é o
que se espera que aconteça. E, por isso, a previsão de que ele, provavelmente terá pago, foi
expressada no Future Perfect, ok?

Aula 02: Exploring Verb Tenses in texts 24


www.estrategiamilitares.com.br
67
Teacher Andrea Belo
Aula 02: EN - Verb Tenses

Vejamos o Future Perfect em nosso “esquema”.

Agora, estudaremos os tempos da vertente “Perfect Continuous”, em que há auxiliar to have,


verbos no particípio e também no gerúndio, vamos lá?
E, daqui por diante, vamos estudar e praticar com exercícios variados.

Aula 02: Exploring Verb Tenses in texts 25


www.estrategiamilitares.com.br
67
Teacher Andrea Belo
Aula 02: EN - Verb Tenses

10. Present Perfect Continuous


O Present Perfect Continuous é um tempo verbal usado para enfatizar a continuidade de uma
ação que se iniciou no passado e se prolonga até hoje. Pouco usado em provas mas, como os textos
são de diferentes fontes, pode aparecer ou ajudar na compreensão de alguma alternativa na hora
de sua prova.
A definição se parece com a do Present Perfect, que também é usado para indicar algo que
começou e ainda não terminou, certo? Então, temos que analisar as diferenças para não haver
confusão.
Vou definir as diferenças através de exemplos para que você possa visualizar bem e
encontrar, nos textos, cada tempo verbal no dia da sua prova.
Na seguinte frase: I have studied English for 6 years. (Eu estudo/tenho estudado Inglês há 6
anos), a ação teve início 6 anos atrás e ainda continua até o presente momento – a pessoa ainda
está estudando Inglês.
O Present Perfect Continuous é mais simples do que isso. É quando algo está acontecendo no
mesmo momento em que o sujeito está falando, veja: I have been studying English for 6 years, seria
também “Eu tenho estudado Inglês a 6 anos” mas o falante, o sujeito da frase, está estudando Inglês
agora, nesse momento, lembrando-se que já estuda esse idioma a 6 anos, ou seja, estudando e
enfatizando a ação de que começou a estudar no passado.
O Present Perfect, por sua vez, revela algo que começou e ainda acontece, mas, não
necessariamente que a pessoa esteja fazendo o que diz.
Outro exemplo para ficar mais clara a diferença para você: Eu estou preparando uma sopa,
seria I am preparing a soup, usando Present Continuous, certo? Mas, Eu estou preparando uma sopa
a 15 minutos, seria I have been preparing a soup for 15 minutes, no Present Perfect Continuous, ou
seja, comecei a sopa e ainda estou preparando-a, a ação não terminou e ainda está sendo feita, ok?
Outro exemplo: alguém está limpando a casa o chão ainda está molhado, usamos o Present
Perfect Continuous – She has been cleaning the house and the floor is still wet – pois ela tem limpado
a casa e não terminou, já que o chão ainda está molhado (wet).
As frases afirmativas têm a seguinte estrutura: um sujeito, o verbo auxiliar to have (ou has
nas terceiras pessoas do singular) o verbo to be no particípio (been) para todos os sujeitos e o verbo
principal no gerúndio (com -ing) - She has been singing (Ela tem cantado), demonstrando que ela
começou a cantar, ainda está cantando. A pessoa que falou isso, o sujeito da frase, provavelmente,
está vendo ou ouvindo quem está cantando, ou com admiração ou fazendo algum tipo de
observação sobre o que vê/ouve.
As frases interrogativas são formadas pelo to have (ou has nas terceiras pessoas do singular)
antes do sujeito, o verbo to be no particípio (been) para todos os sujeitos e o verbo principal no
gerúndio (com -ing) - Has she been singing? (Ela tem cantado?).
E as frases negativas são formadas pelo sujeito, o verbo to have not (ou has not nas terceiras
pessoas do singular) o verbo to be no particípio (been) para todos os sujeitos e o verbo principal no
gerúndio (com -ing) - She hasn’t been singing (Ela não tem cantado).

Aula 02: Exploring Verb Tenses in texts 26


www.estrategiamilitares.com.br
67
Teacher Andrea Belo
Aula 02: EN - Verb Tenses

Vejamos uma observação que preparei para facilitar a sua compreensão sobre esses tempos
verbais. Em seguida, o esquema do verbo to study, conjugado no Present Perfect Continuous.

Não confunda:
• Present Continuous
• Present Perfect Continuous
• Present Perfect
O Present Continuous expressa uma ação que está ocorrendo no momento, agora:
She is dancing now. (Ela está dançando agora.)
O Present Perfect Continuous expressa algo que começou no passado e continua até o presente:
He has been dancing for one hour. (Ele está dançando há uma hora.)
O Present Perfect expressa ações que acabaram em um tempo não definido do passado:
She has danced. (Ela dançou. – pode dançar novamente, todos os dias já que não definiu quando).

Vamos ao Past Perfect Continuous agora. Preparado?

Aula 02: Exploring Verb Tenses in texts 27


www.estrategiamilitares.com.br
67
Teacher Andrea Belo
Aula 02: EN - Verb Tenses

11. Past Perfect Continuous


O Past Perfect Continuous, que também pode ser encontrado com o nome Past Perfect
Progressive, como os outros tempos verbais Continuous em geral, é usado para enfatizar ações
anteriores à outras, ambas no passado.
Por exemplo, um aluno estudou por 8 horas seguidas e ficou cansado. A ação de ficar cansado
veio como consequência dos estudos. E, aconteceu depois que ele estudou por longas horas.
Porém, ambas ações já aconteceram – estudar e ficar cansado – o aluno provavelmente já
descansou e o que aconteceu já passou. Para descrever esses acontecimentos do passado, usando
dois verbos na frase, um deles será expresso no passado (ficar cansado) e o verbo que descreve o
fato anterior ao cansaço, no Past Perfect Continuous, tempo justamente exclusivo para
acontecimentos anteriores a outros.
Vejamos o exemplo acima explicado, agora em Inglês, com o uso do tempo verbal Present
Perfect Continuous: He got tired because he had been studying for 8 hours – to get tired = ficar
cansado, no passado = got tired) e had been studying = tinha estudado ou tinha ficado estudando 8
horas, se for traduzir literalmente.
Parece que não é simples, mas é.
Na seguinte frase: I had been written e-mails all night
long before I went to bed (Eu fiquei escrevendo e-mails
a noite inteira antes de ir para cama), ambas ações já
aconteceram: escrever emails a noite inteira e depois ir
para a cama dormir.

A última coisa feita foi ir para cama dormir e, por isso, o verbo went to bed. Já a ação escrever
e-mails, o que o sujeito fez antes de dormir, está no Past Perfect Continuous – had been writing.
É muito comum, no Past Perfect Continuous, o uso de advérbios diversos na elaboração das
sentenças, como when (quando), since (desde), before (antes), after (depois) etc. porque esses
advérbios proporcionam a ligação entre os eventos que aconteceram no passado.
As frases afirmativas do Past Perfect Continuous têm a seguinte estrutura: um sujeito, o verbo
auxiliar to have no passado: had, o verbo to be no particípio (been) para todos os sujeitos e o verbo
principal no gerúndio (com -ing) - She had been singing long hours and she won the music festival.
(Ela tinha ficado cantando ou simplesmente ela tinha cantado antes de ganhar o festival de música.
E ganhou: won), demonstrando que praticou e ganhou, duas ações que aconteceram.
As frases interrogativas são formadas pelo had antes do sujeito, no início das frases, o verbo
to be no particípio (been) para todos os sujeitos e o verbo principal no gerúndio (com -ing) - Had she
been singing before she won the festival? (Ela tinha cantado? Ou Ela tinha ficado cantando antes que
ganhasse o festival?).

Aula 02: Exploring Verb Tenses in texts 28


www.estrategiamilitares.com.br
67
Teacher Andrea Belo
Aula 02: EN - Verb Tenses

E as frases negativas, por sua vez, são formadas pelo sujeito, pelo had na negativa: had
not/hadn’t, o verbo to be no particípio (been) para todos os sujeitos e o verbo principal no gerúndio
(com -ing) - She hadn’t been singing before... (Ela não tinha cantado ou não tinha ficado cantando
antes de...)
Vejamos o esquema do verbo to study, conjugado no Past Perfect Continuous.

Agora, em complemento aos tempos “Perfect Continuous”, vejamos o Future Perfect


Continuous no próximo capítulo e outros tempos verbais e curiosidades adiante. Vamos lá!

Aula 02: Exploring Verb Tenses in texts 29


www.estrategiamilitares.com.br
67
Teacher Andrea Belo
Aula 02: EN - Verb Tenses

12. Future Perfect Continuous


O Future Perfect Continuous, que também pode ser encontrado com o nome Future Perfect
Progressive indica uma ação que será completada em algum momento no futuro. É um tempo verbal
pouco usado em Inglês, o menos usado de todos os tempos aqui estudados.
Porém, você precisa entender bem o uso e regras dele para que, caso apareça em sua prova,
seja simples encontrá-lo e saber como responder as perguntas sobre isso.
Uma das características marcantes desse tempo verbal é que ele expressa algo especial, pois,
quando utilizado, é expressa uma intenção real do que se quer dizer. Vejamos exemplos: By October
of this year, I will have been searching for a job for three months. (Em Outubro deste ano, fará três
meses que estarei procurando um emprego).
O sujeito está dizendo algo que vai acontecer daqui a 3 meses, considerando que ele estamos
em Julho, já que 3 meses contados a partir de Julho, é Outubro. E, se o sujeito não encontrar o
emprego que procura, completará 3 meses a procura de algo que ainda não encontrou.
Vejamos outro exemplo, com o uso do tempo verbal Future Perfect Continuous: At ten o’clock,
I will have been waiting for you for two hours (Às 22h, terei ficado esperando você por duas horas).
Então, agora são exatamente 20h e daqui 2 horas (22h), a pessoa já terá esperado outra por 2 horas,
ou seja, ficará 2 horas esperando alguém.
Vejamos um exemplo:
Na seguinte frase: Next year, the teacher will have been
working at the school for more than 5 years – quer dizer
que no ano que vem, o professor terá trabalhado na escola
por mais de 5 anos. Então, o professor já trabalha a quase
4 anos no mesmo lugar e, no próximo ano, completará 5
ou mais anos trabalhando nessa escola.
As frases afirmativas do Future Perfect Continuous têm a seguinte estrutura: um sujeito, o
verbo auxiliar que representa o futuro: will, o verbo to have (para todos os sujeitos), o verbo to be
no particípio – been – e o verbo principal no gerúndio (com -ing) - She will have been singing for two
hours at 3pm. (Ela terá ficado cantando por 2 horas às 15h), demonstrando que ela está cantando,
são 13h e, daqui 2 horas, ela terá ficado fazendo a mesma coisa (cantando) por duas horas. Certo?
As frases interrogativas são formadas pelo will antes do sujeito, no início das frases, o sujeito,
o verbo auxiliar que representa o futuro: will, o verbo to have (para todos os sujeitos), o verbo to be
no particípio – been – e o verbo principal no gerúndio (com -ing) - Will she have been singing for two
hours at 3pm.
E as frases negativas, por sua vez, são formadas pelo sujeito, o auxiliar will na negativa – will
not/won’t, o verbo to have (para todos os sujeitos), o verbo to be no particípio – been – e o verbo
principal no gerúndio (com -ing) - She will not/won’t have been singing for two hours at 3pm.

Aula 02: Exploring Verb Tenses in texts 30


www.estrategiamilitares.com.br
67
Teacher Andrea Belo
Aula 02: EN - Verb Tenses

Vejamos o esquema do verbo to study, conjugado no Future Perfect Continuous.

Agora, estudaremos sobre os verbos modais e suas características, regras, como usar etc. E
vamos acrescentando conteúdo em seus estudos. Vamos lá!

Aula 02: Exploring Verb Tenses in texts 31


www.estrategiamilitares.com.br
67
Teacher Andrea Belo
Aula 02: EN - Verb Tenses

13. Modal verbs

Modal verbs são muito simples.


Verbo modal é o nome técnico classificado pela Gramática Normativa como um grupo de
palavras em Inglês que possuem suas características próprias. Como assim?
É porque esse grupo de verbos, considerados de certa forma auxiliares, não seguem as
mesmas regras que os outros verbos da língua inglesa.
O uso dos Modal verbs acontece para mudar ou, muitas vezes, complementar o sentido do
verbo principal, expressando ideias variadas, que podem ser: possibilidade, obrigação, dedução,
desejo, proibição, vontade, capacidade, entre outras.
Vamos estudar cada um deles separadamente e conhecer seus significados e maneiras de
usar. Nas frases afirmativas, eles vêm antes dos verbos principais.
Nas interrogativas, no início das frases e nas negativas, com o acréscimo de “not”, assim como
já vimos em outros auxiliares já estudados.
Agora, um por um para melhor compreensão.

Verbo Modal CAN


Can é usado para expressar, na maioria das vezes, capacidade ou habilidade. Mas pode
aparecer em frases também demonstrando possibilidade, alguma permissão informal ou fazendo
um pedido informal. Vejamos alguns exemplos:
You can park here. (Você pode estacionar aqui – Permissão)

It can happen to you one day. (Isto pode acontecer com você um dia – Possibilidade)

They can speak French. (Eles sabem/conseguem falar Francês – Habilidade)

I can't have done it! (Não posso ter feito isso! – Capacidade (como fui capaz de fazer isso!)

Can you help me? (Você pode me ajudar? – Pedido informal)

Can I use you phone? (Posso usar seu telefone? – Permissão informal)

Aula 02: Exploring Verb Tenses in texts 32


www.estrategiamilitares.com.br
67
Teacher Andrea Belo
Aula 02: EN - Verb Tenses

Verbo Modal Could


Could é usado quase que nas mesmas situações em que usamos can. Porém, com um “tom”
mais educado. Expressa expressar capacidade, habilidade, possibilidade, permissão formal e pedido
formal.
Geralmente está presente em perguntas com um pouco de formalidade, já que haverá outros
modais para casos de formalidade de fato.
Vejamos alguns exemplos:
Could you open the door, please? (Você poderia abrir a porta, por favor?)

I could see she was tired. (Eu podia ver que ela estava cansada.)

We could not smoke in that restaurant. (Nós não podíamos fumar naquele restaurante.)

He could swim when he was a kid. (Ele podia/conseguia nadar quando era criança)

If I win the lottery, I could buy a new house. (Se eu ganhasse na loteria, eu podia/poderia
comprar uma casa nova).

Verbo Modal May

May é usado para indicar permissão e possibilidade.


Pode também ser usado para expressar ações e acontecimentos que serão possíveis no futuro
e no presente.
Pode expressar deduções, fazer um pedido, pedir ou dar permissão. Pode oferecer ajuda.
Vejamos alguns exemplos:
May I help you? (Posso lhe ajudar? (*Aqui podemos usar can ou may na linguagem oral,
cotidiana, informal, mas, na escrita, o can seria inapropriado e o may, totalmente adequado)

I may call you later when I leave my job. (Eu posso ligar para você mais tarde quando eu sair
do meu trabalho.)

She may not know what happened. She looks like suspicious. (Ela não deve saber o que
aconteceu. Ela parece suspeita.)

Aula 02: Exploring Verb Tenses in texts 33


www.estrategiamilitares.com.br
67
Teacher Andrea Belo
Aula 02: EN - Verb Tenses

I may visit you next Sunday. It’s my day off. (Eu posso te visitar no próximo domingo. /É
provável que eu te visite no próximo domingo, é meu dia de folga.)

He may be sick, he didn’t come. (Ele deve estar doente, ele não veio. / É possível que esteja
doente)

Verbo Modal Might

Might é usado para indicar permissões mais formais, possibilidades remotas.


Também pode ser usado para descrever ações e acontecimentos possíveis em um momento
futuro ou presente, pode expressar deduções, às vezes fazer pedidos ou dar permissão.
É importante destacar que, might passa uma ideia de frases mais polida, ou seja, mais formal
e por isso é menos usado que os demais.
Vejamos exemplos.
Might I borrow you pen? Mine is not on my table. (Posso pegar sua caneta emprestada? A
minha não está e minha mesa.)
It might rain later because it is cloudy. (Pode ser que chova mais tarde/ Talvez chova mais
tarde porque está nublado.)
He might tell you why he got disappointed. (Pode ser que ele te conte por que ele ficou
decepcionado)
She might get home because of the traffic. (Ela deve chegar em casa tarde por causa do
trânsito.)
Why did he leave? I don’t know. Maybe might needed. (Por que ele foi embora? Não sei. Talvez
ele precisasse ir.)

Verbo Modal Must


Must é usado para exprimir obrigações e deduções (se for na afirmativa) e expressar proibição
(na negativa) – must not/mustn’t.
Vejamos exemplos.

Aula 02: Exploring Verb Tenses in texts 34


www.estrategiamilitares.com.br
67
Teacher Andrea Belo
Aula 02: EN - Verb Tenses

You must help her, it’s your mom! (Você deve ajudá-la, é sua mãe!)

The doctor said you must stop smoking, or you’ll die. (O médico falou que você deve parar de
fumar ou você vai morrer)

Children must not watch this video. (As crianças não devem assistir esse vídeo.)

Bom, como must não tem forma específica para o tempo passado, usamos had to para
expressar uma obrigação no passado:

Yesterday I had to work up to 10pm. (Ontem tive que trabalhar até às 10 da noite.)

Em frases afirmativas, o must também pode ser substituído por have to para expressar
obrigação. Significarão dever, ter que fazer algo, must e have to com a mesma função e tradução.

Geralmente, na fala, linguagem informal, percebemos que o have to é usado com mais
frequência do que o must:

I must study for my test. (Eu tenho que/devo estudar para minha prova).

I have to study for my test. (Eu tenho que/devo estudar para minha prova).

Verbos Modais Should/Ought to

Should ou Ought to são modais usados na mesma função, com o mesmo significado.
São ambos usados para aconselhar, exprimir expectativas ou obrigações menos intensas.
Vejamos exemplos.
They should/ought to wear seat belt while driving. (Eles deveriam usar cinto de segurança no
carro enquanto está dirigindo)
You should not/ought not to walk alone after 9pm. (Você não deveria andar sozinho após 9h
da noite)
You should not/ought not to accept offers from strangers. (Você não deveria aceitar propostas
de estranhos.)
You shouldn't/ought not to say things like that to me. (Você não deveria dizer esse tipo de
coisa para mim.)
What should/ought to we do now? I have no idea. (O que devemos fazer agora? Eu não faço
ideia).

Aula 02: Exploring Verb Tenses in texts 35


www.estrategiamilitares.com.br
67
Teacher Andrea Belo
Aula 02: EN - Verb Tenses

Verbo Modal Shall

Shall é usado para formar orações que remetem a ações futuras, que ainda vão acontecer.
Shall só é usado na primeira pessoa do singular (I) e do plural (We).
Este modal é visto com mais frequência em perguntas ou quando se oferece algo, sugerindo
alguma coisa ou fazendo algum convite É considerado bem formal, expressa polidez.
Vejamos exemplos.
You can count on me. I shall arrive tomorrow. (Você pode contar comigo. Eu chegarei
amanhã.)
We shall arrive tomorrow. (Nós chegaremos amanhã.)
Shall I call her? (Ligo para ela? – como se fosse uma dúvida se ligo ou não para a pessoa, que
pode estar ocupada ou não quer falar.)
Shall I open the window? (Abro a janela? – como se fosse uma dúvida se o clima está quente
ou frio e se a pessoa realmente pode abrir a janela.)
We shall need the money in December. (Vamos precisar do dinheiro em Dezembro.)
Shall I carry your luggage? (Quer que eu carregue sua bagagem?)

Verbos Modais Will e Would


Will e Would são modais, porém são muito mais usados na função de auxiliar do futuro e
frases condicionais, consecutivamente.
Will, acompanhando verbos principais, coloca-os no tempo futuro, como já vimos no capítulo
específico do tempo futuro – I will travel tomorrow – Eu viajarei amanhã.
Would é basicamente a característica da polidez, da delicadeza.
E acompanha verbos principais, colocando-os na função de condicional – I would travel if I
had money. (Eu viajaria se tivesse dinheiro. O would também é usado como o passado de will.
Vejamos exemplos.
I will study very much on the weekend. (Vou estudar muito no fim de semana).
Will you cook lunch? (Você vai fazer almoço?)
Would you walk on the cemetery at night? (Você andaria no cemitério a noite?)

Agora, estudaremos o tempo verbal Imperative, que geralmente usamos para dar ordens.
Mas, veremos que há outros usos do imperativo.
Vamos lá! Temos muitos exercícios ainda para praticar! Come on!

Aula 02: Exploring Verb Tenses in texts 36


www.estrategiamilitares.com.br
67
Teacher Andrea Belo
Aula 02: EN - Verb Tenses

14. Imperative tense


Imperative Tense é o tempo verbal em inglês usado para expressar ordens, pedidos, oferecer
instruções e também para aconselhar alguém.
Para elaborar frases no Imperativo, basta usar os verbos em Inglês no infinitivo (sua forma
original sem conjugação) sem a preposição “to”. O “to” é usado para mostrar os verbos separados,
ou seja, fora das frases, não contextualizados.
Lembre-se de que o Imperativo é usado apenas com o verbo em sua forma infinitiva, sem a
preposição “to” e, quase 100% das vezes, no início de frases.
Mas o modo Imperativo não é, por sua vez, conjugado no passado ou na forma contínua. Não
há essas variações no Imperativo.
E, para expressar negação, usa-se o Don’t no início das frases, como por exemplo Don’t repeat
that. (Não repita isso).
Quando um verbo é mencionado com “to”, geralmente é antes da frase ser elaborada,
apontando a ação a ser usada em determinada oração: to go (verbo ir) Students go to the university
– Alunos vão à universidade.
O uso do tempo Imperativo costuma ser direto e, às vezes, passa a impressão de que a pessoa
foi rude ou um pouco sem educação. Isso porque é fácil perceber que, curiosamente, a palavra
please, inserida no contexto de um pedido ou ordem, é bem mais usada em países falantes da língua
inglesa do que por nós, brasileiros.
São diferenças culturais que não devem ser julgadas já que isso não torna ninguém melhor
ou pior do que o outro, mas sim, mostra-nos o quanto há variedades linguísticas e formas diversas
de se expressar como pessoa.
Então, é bom que você esteja atento que, em um pedido ou uma ordem, por exemplo, com
o intuito de amenizar e não parecer ser grosseiro, o sujeito certamente usará a palavra please nos
textos e nos exercícios que analisaremos e já estamos analisando nas aulas.
Vejamos exemplos de frases no modo Imperativo.
Turn on the TV, please. (Ligue a TV, por favor).
Look at the book now, please. (Olhe para o livro agora, por favor).
Hey, John, bring me a cup of water, please. (Ei, John, traga-me um copo de água, por favor).
Go fast! (Vá rápido!)
Come here, please. (Venha aqui, por favor).
Listen to your teacher. (Ouça sua professora).
Sit down/ Stand up. (Sente-se/Levante-se)
Close the door and the window. (Feche a porta e a janela).
Be careful. (Tome cuidado).

Aula 02: Exploring Verb Tenses in texts 37


www.estrategiamilitares.com.br
67
Teacher Andrea Belo
Aula 02: EN - Verb Tenses

Existem muitas situações em que encontramos frases no Imperativo. Por exemplo, nas placas
de sinalização em nossa cidade: stop (pare), Push (empurre), Insert the coin (insira a moeda).
Se você analisar, os manuais de instrução de qualquer assunto, tais como a montagem de um
produto novo, um eletrodoméstico que você tenha comprado ou até mesmo as receitas culinárias,
também são cheias deles.
Encontramos frases imperativas em ordem, sequência que devemos seguir para montar algo
ou preparar alguma coisa: “first, you...” (primeiro, você...), “then, you...” (então você...), “so, you...”
(daí, você...) “next,...” (em seguida,...), “after,...” (depois,...) e assim por diante com verbos no
Imperativo, para seguir os passos e cumprir uma meta. Exemplos:
First, break the egg. And then, join the flour. (Primeiro, quebre o ovo e então, junte à farinha)
First, connect the cables. Then, plug it. Finally, check your internet connection and...
(Primeiro,..)

Agora, estudaremos um pouco sobre os Phrasal verbs, já que depois teremos uma aula
exclusiva para explorar a fundo esse tema. Vamos lá!

Aula 02: Exploring Verb Tenses in texts 38


www.estrategiamilitares.com.br
67
Teacher Andrea Belo
Aula 02: EN - Verb Tenses

15. Phrasal Verbs – introdução


Teremos uma aula inteira, em nosso material, exclusiva para estudar e compreender melhor
os phrasal verbs, já que são bastante usados na elaboração das provas de Inglês.
É um assunto que necessita de atenção e, aqui, farei uma introdução, para que esse tópico
não falte nessa aula inteira, que é destinada aos verbos e suas conjugações.
Phrasal Verbs, definidos de uma maneira mais simples, são verbos que vem acompanhados
por preposições ou advérbios.
Ou seja, é uma combinação de palavras formada por um verbo e uma preposição ou advérbio.
São também conhecidos como verbos preposicionados ou, em alguns livros e gramáticas,
classificados como expressões verbais, porque esses verbos especiais, quando combinados com
partículas adverbiais ou com preposições, mudam completamente o significado do verbo usado em
sua composição.
Quando você tenta traduzir essas combinações, esses phrasal verbs, palavra por palavra,
elas poderão ficar totalmente sem sentido, já que são verbos interpretados sempre em conjunto.
Para exemplificar, vamos pensar no verbo to call, que, em Português, significa chamar ou
ligar, telefonar (I called you last night = Eu liguei para você ontem a noite).
Esse verbo, quando usado junto às preposições in e off, por exemplo, tornam-se outros
verbos com outros significados, veja:
To call in: convidar – I will probably call my neighbor in to the party – Eu provavelmente vou
convidar meu vizinho para a festa.
To call off: cancelar – I have to call off the meeting with you, I’m sorry – Eu tenho que cancelar
a reunião com você, desculpe-me.
Como afirmei que os phrasal verbs não podem ser traduzidos literalmente, a melhor forma
de aprendê-los é praticando: respondendo exercícios e lendo textos, fontes da sua prova.
Quanto mais intensificado for seu estudo, mais vocabulário, incluindo phrasal verbs, você
aprenderá.
Esse assunto é tão importante em Inglês, que existem vários dicionários de phrasal verbs. Na
aula em que trataremos de regras, explicações variadas, maneiras de usá-los, preposições mais
utilizadas na construção dos phrasal verbs, entre outros detalhes essenciais, haverá uma lista com a
sugestão de alguns deles que aparecem com frequência nas provas.
Há inúmeros trechos de questões com os phrasal verbs e vou mostrar um deles aqui, além de
vários outros no decorrer das aulas, principalmente, na aula específica aos phrasal verbs, para que
você tenha uma percepção de como eles são cobrados.

Aula 02: Exploring Verb Tenses in texts 39


www.estrategiamilitares.com.br
67
Teacher Andrea Belo
Aula 02: EN - Verb Tenses

16. Questão inédita

Questão inédita Teacher Andrea Belo


Read the text and answer the question below.

Aula 02: Exploring Verb Tenses in texts 40


www.estrategiamilitares.com.br
67
Teacher Andrea Belo
Aula 02: EN - Verb Tenses

Questão 01 (Inédita – Teacher Andrea Belo)


De acordo com o texto, a pergunta “Do you and your partner have a healthy financial
relationship?” indica:
a) The question involves the relationship of marriage and divorce because of money.
b) The question involves a business deal represented by the word partner.
c) The question is a curiosity about how a wedding life is in many aspects.
d) The question shows that the author talks about two tasks: health and relationship.
e) The question makes couples think about their relationship involving money tasks.
Comentários:
THE TELEGRAPH é uma fonte usada na elaboração de algumas provas e é uma ótima opção de
aprimoramento de vocabulário. Muitas questões que estamos analisando e ainda vamos
analisar em nossas aulas, mostram textos retirados de The Telegraph, assim como nessa
questão inédita elaborada para você.
Você pode perceber que o assunto é família, pois há a palavra family no lugar em que o jornal
mostra o assunto, abaixo do nome The Telegraph. Há um casal vestido de noivos e moedas ao
redor deles, indicando a relação do dinheiro e do casamento, como já se pergunta no início do
artigo: Do you and your partner have a healthy financial relationship? – Você e seu parceiro
têm uma relação financeira saudável? E durante o texto, palavras cognatas indicam que se fala
de finanças no casamento: husband, finance, money, married 11 years etc. Agora vamos às
alternativas.
Na letra “A”, diz que a pergunta envolve a relação de casamento e divórcio (marriage/ divorce)
mas o texto é sobre de casamento x dinheiro e não se fala em divórcio por causa de dinheiro.
Mesmo que isso possa acontecer, no texto não há essa informação. Alternativa errada.
A alternativa “B”, diz que a pergunta envolve negócios por causa da palavra partner, que pode
significar sócio. Mas o artigo cita partner como parceiro(a) de casamento (marido e esposa) e
não em uma empresa. Alternativa errada.
Na letra “C”, diz que a pergunta é uma curiosidade sobre muitos aspectos (many aspects). Essa
palavra many é perigosa. O texto fala da questão financeira e não de muitos aspectos do
casamento. Atenção à essas palavras que generalizam. Alternativa errada.
Na letra “D”, diz que o autor do texto fala de saúde e relacionamento (healthy and relationship)
mas não é separadamente como a alternativa aponta e sim, sobre um relacionamento saudável
em relação à vida financeira (healthy financial relationship) Alternativa errada.
Na letra “E”, diz que a pergunta leva o casal a pensar sobre seu relacionamento envolvendo o
assunto dinheiro (relationship involving money tasks), que é justamente o que propõe o texto.
Alternativa correta.
Agora vamos à nossa lista de questões de exercícios de provas anteriores para praticar ainda
mais.

Aula 02: Exploring Verb Tenses in texts 41


www.estrategiamilitares.com.br
67
Teacher Andrea Belo
Aula 02: EN - Verb Tenses

17. Questões de anos anteriores

Você vai, agora, responder questões selecionadas de provas já realizadas em anos anteriores.
Depois, como em todas as nossas aulas, haverá o gabarito e as questões comentadas.
Vamos treinar várias Carreiras Militares, entre elas, questões EM, para adquirir experiência e
treinar vocabulário.

AFA/2017
TEXT: Howard Gardner: ‘Multiple intelligences’ are not ‘learning styles’ by Valerie Strauss

The fields of psychology and education were revolutionized 30 years ago when we now
worldrenowned psychologist Howard Gardner published his 1983 book Frames of Mind: The
Theory of Multiple Intelligences, which detailed a new model of human intelligence that went
beyond the traditional view that there was a single kind that could be measured by standardized
tests.
Gardner’s theory initially listed seven intelligences which work together: linguistic, logical-
mathematical, musical, bodily-kinesthetic, interpersonal and intrapersonal; he later added an
eighth, naturalist intelligence and says there may be a few more. The theory became highly popular
with K-12¹ educators around the world seeking ways to reach students who did not respond to
traditional approaches, but over time, ‘multiple intelligences’ somehow became synonymous with
the concept of ‘learning styles’. In this important post, Gardner explains why the former is not the
latter.
It’s been 30 years since I developed the notion of ‘multiple intelligences’. I have been gratified by
the interest shown in this idea and the ways it’s been used in schools, museums, and business
around the world. But one unanticipated consequence has driven me to distraction and that’s the
tendency of many people, including persons whom I cherish, to credit me with the notion of
‘learning styles’ or to collapse ‘multiple intelligences’ with ‘learning styles’. It’s high time to relieve
my pain and to set the record straight.
First a word about ‘MI theory’. On the basis of research in several disciplines, including the study
of how human capacities are represented in the brain, I developed the idea that each of us has a
number of relatively independent mental faculties, which can be termed our ‘multiple
intelligences’. The basic idea is simplicity itself. A belief in a single intelligence assumes that we
have one central, all-purpose computer, and it determines how well we perform in every sector of
life. In contrast, a belief in multiple intelligences assumes that human beings have 7 to 10 distinct
intelligences.
Even before I spoke and wrote about ‘MI’, the term ‘learning styles’ was being bandied about in
educational circles. The idea, reasonable enough on the surface, is that all children (indeed all of
us) have distinctive minds and personalities. Accordingly, it makes sense to find out about learners
and to teach and nurture them in ways that are appropriate, that they value, and above all, are
effective.

Aula 02: Exploring Verb Tenses in texts 42


www.estrategiamilitares.com.br
67
Teacher Andrea Belo
Aula 02: EN - Verb Tenses

Two problems: first, the notion of ‘learning styles’ is itself not coherent. Those who use this term
do not define the criteria for a style, nor where styles come from, how they are recognized/
assessed/ exploited. Say that Johnny is said to have a learning style that is ‘impulsive’. Does that
mean that Johnny is ‘impulsive’ about everything? How do we know this? What does this imply
about teaching? Should we teach ‘impulsively’, or should we compensate by ‘teaching reflectively’?
What of learning style is ‘right-brained’ or visual or tactile? Same issues apply.
Problem #2: when researchers have tried to identify learning styles, teach consistently with those
styles, and examine outcomes, there is not persuasive evidence that the learning style analysis
produces more effective outcomes than a ‘one size fits all approach’. Of course, the learning style
analysis might have been inadequate. Or even if it is on the mark, the fact that one intervention
did not work does not mean that the concept of learning styles is fatally imperfect; another
intervention might have proved effective. Absence of evidence does not prove non-existence of a
phenomenon; it signals to educational researchers: ‘back to the drawing boards’.
Here’s my considered judgment about the best way to analyze this lexical terrain: Intelligence: We
all have the multiple intelligences. But we signed out, as a strong intelligence, an area where the
person has considerable computational power. Style or learning style: A hypothesis of how an
individual approach the range of materials. If an individual has a ‘reflective style’, he/she is
hypothesized to be reflective about the full range of materials. We cannot assume that
reflectiveness in writing necessarily signals reflectiveness in one’s interaction with the others.
Senses: Sometimes people speak about a ‘visual’ learner or an ‘auditory’ learner. The implication
is that some people learn through their eyes, others through their ears. This notion is incoherent.
Both spatial information and reading occur with the eyes, but they make use of entirely different
cognitive faculties. What matters is the power of the mental computer, the intelligence that acts
upon that sensory information once picked up.
These distinctions are consequential. If people want to talk about ‘an impulsive style’ or a ‘visual
learner’, that’s their prerogative. But they should recognize that these labels may be unhelpful, at
best, and ill-conceived at worst.
In contrast, there is strong evidence that human beings have a range of intelligences and that
strength (or weakness) in one intelligence does not predict strength (or weakness) in any other
intelligences. All of us exhibit jagged profiles of intelligences. There are common sense ways of
assessing our own intelligences, and even if it seems appropriate, we can take a more formal test
battery. And then, as teachers, parents, or self assessors, we can decide how best to make use of
this information. (Adapted from https://www.washingtonpost.com/news/answer-sheet)

Questão 01 (AFA/2017) - The text


a) aims at highlighting distinctive mind barriers related to learning.
b) provides the reader with a bird’s-eye-view of Gardner’s landmark publication.
c) develops a considerable set of psychological and mental implications.
d) concerns about spending 30 years to measure people’s intelligence.

Aula 02: Exploring Verb Tenses in texts 43


www.estrategiamilitares.com.br
67
Teacher Andrea Belo
Aula 02: EN - Verb Tenses

Questão 02 (AFA/2017) - In the sentence “there was a single kind that could be measured by
standardized tests” (lines 07 and 08), it is possible to find an option to substitute the pronoun
accordingly in
a) when.
b) which
c) how.
d) whom.

Questão 03 (AFA/2017) - In the

Questão 03 (EPCAR/2018)

Questão 04 (Escola Naval/2018) -

Aula 02: Exploring Verb Tenses in texts 44


www.estrategiamilitares.com.br
67
Teacher Andrea Belo
Aula 02: EN - Verb Tenses

Questão 05 (Escola Naval/2018)

TEXT (EAM/2019) para questões 06 e 07


There's nowhere like Scotland. Scotland is a country in a country. It is part of Great Britain (England,
Scotland and Wales), and of the United Kingdom (England, Scotland, Wales and Northern Ireland).
Scotland is in the far northwest of Europe, between the Atlantic Ocean and the North Sea. It is
often cold and grey, and it often rains a lot. But the people of Scotland love their country, and many
visitors to Scotland love it too. They love the beautiful hills and mountains of the north, the sea and
the eight hundred islands, and the six cities: Edinburgh, Glasgow, Aberdeen, Dundee, Inverness and
Stirling. The country is special, and Scottish people are special too: often warm and friendly. There
are about five million people in Scotland. Most Scots live in the south, in or near the big cities of
Edinburgh and Glasgow. Most of the north of the country is very empty; not many people live there.
A Scottish person is also called a Scot, but you cannot talk about a Scotch person: Scotch means
whisky, a drink made in Scotland. Scottish people are British, because Scotland is part of Great
Britain, but you must not call Scottish people English! The Scots and the English are different. These
days everyone in Scotland speaks English. But, at one time, people in the north and west of Scotland
did not speak English. They had a different language, a beautiful language called Gaelic. About
60,000 people, 1% of the people in Scotland, speak Gaelic now. But many more want Gaelic in their
lives because it is part of the story of Scotland. Adapted from: FLINDERS, S. Factfiles Seotland. OUP, 2010.

Questão 06 - Say if the following statements are T (TRUE) or F (FALSE) about Scotland. Then, mark
the correct option, from top to bottom.
( ) It is part of Great Britain but not of the United Kingdom.
( ) It is located between the Atlantic Ocean and the North Sea.
( ) The weather there is usually cold, grey, and rainy.
( ) There are 80 islands in the country.
( ) Tourists can see hills, mountains and the sea there.

Aula 02: Exploring Verb Tenses in texts 45


www.estrategiamilitares.com.br
67
Teacher Andrea Belo
Aula 02: EN - Verb Tenses

(A) (F) (T) (F) (T) (T)


(B) (T) (F) (T) (F) (F)
(C) (F) (T) (T) (T) (T)
(D) (F) (T) (T) (F) (T)
(E) (T) (F) (F) (T) (F)

Questão 07 (EAM/2019) - About Scotland’s people/population, it is correct to say that


(A) they are often friendly and kind.
(B) most of them live in the north.
(C) they are also called Scotch.
(D) they do not speak English.
(E) nobody speaks Gaelic anymore.

Questão 08 (EFOMM/201) -

Aula 02: Exploring Verb Tenses in texts 46


www.estrategiamilitares.com.br
67
Teacher Andrea Belo
Aula 02: EN - Verb Tenses

Questão 09 (EAM/2018) Para questões 09 e 10

Questão 10 (EAM/2018 adaptada) About the picture, it is correct to say that


(A) they are practicing a sport that everybody can play if they want.
(B) basketball is played by only two people at the same time.
(C) they should play in a different place to not disturb anybody.
(D) they do play all weekends.
(E) they play and talk at the same time.

Aula 02: Exploring Verb Tenses in texts 47


www.estrategiamilitares.com.br
67
Teacher Andrea Belo
Aula 02: EN - Verb Tenses

18. Gabarito

1–B 08 – E
2–B 09 – B
3–A 10 – A
4–C
5–D
6–D
7–A

Aula 02: Exploring Verb Tenses in texts 48


www.estrategiamilitares.com.br
67
Teacher Andrea Belo
Aula 02: EN 2021

19. Exercícios respondidos e comentados

AFA/2017

TEXT: Howard Gardner: ‘Multiple intelligences’ are not ‘learning styles’ by Valerie Strauss

The fields of psychology and education were revolutionized 30 years ago when we now
worldrenowned psychologist Howard Gardner published his 1983 book Frames of Mind: The
Theory of Multiple Intelligences, which detailed a new model of human intelligence that went
beyond the traditional view that there was a single kind that could be measured by standardized
tests.
Gardner’s theory initially listed seven intelligences which work together: linguistic, logical-
mathematical, musical, bodily-kinesthetic, interpersonal and intrapersonal; he later added an
eighth, naturalist intelligence and says there may be a few more. The theory became highly popular
with K-12¹ educators around the world seeking ways to reach students who did not respond to
traditional approaches, but over time, ‘multiple intelligences’ somehow became synonymous with
the concept of ‘learning styles’. In this important post, Gardner explains why the former is not the
latter.
It’s been 30 years since I developed the notion of ‘multiple intelligences’. I have been gratified by
the interest shown in this idea and the ways it’s been used in schools, museums, and business
around the world. But one unanticipated consequence has driven me to distraction and that’s the
tendency of many people, including persons whom I cherish, to credit me with the notion of
‘learning styles’ or to collapse ‘multiple intelligences’ with ‘learning styles’. It’s high time to relieve
my pain and to set the record straight.
First a word about ‘MI theory’. On the basis of research in several disciplines, including the study
of how human capacities are represented in the brain, I developed the idea that each of us has a
number of relatively independent mental faculties, which can be termed our ‘multiple
intelligences’. The basic idea is simplicity itself. A belief in a single intelligence assumes that we
have one central, all-purpose computer, and it determines how well we perform in every sector of
life. In contrast, a belief in multiple intelligences assumes that human beings have 7 to 10 distinct
intelligences.
Even before I spoke and wrote about ‘MI’, the term ‘learning styles’ was being bandied about in
educational circles. The idea, reasonable enough on the surface, is that all children (indeed all of
us) have distinctive minds and personalities. Accordingly, it makes sense to find out about learners
and to teach and nurture them in ways that are appropriate, that they value, and above all, are
effective.

Aula 02 – Verb Tenses 49


www.estrategiamilitar.com.br 67
Teacher Andrea Belo
Aula 02: EN 2021

Two problems: first, the notion of ‘learning styles’ is itself not coherent. Those who use this term
do not define the criteria for a style, nor where styles come from, how they are recognized/
assessed/ exploited. Say that Johnny is said to have a learning style that is ‘impulsive’. Does that
mean that Johnny is ‘impulsive’ about everything? How do we know this? What does this imply
about teaching? Should we teach ‘impulsively’, or should we compensate by ‘teaching reflectively’?
What of learning style is ‘right-brained’ or visual or tactile? Same issues apply.
Problem #2: when researchers have tried to identify learning styles, teach consistently with those
styles, and examine outcomes, there is not persuasive evidence that the learning style analysis
produces more effective outcomes than a ‘one size fits all approach’. Of course, the learning style
analysis might have been inadequate. Or even if it is on the mark, the fact that one intervention
did not work does not mean that the concept of learning styles is fatally imperfect; another
intervention might have proved effective. Absence of evidence does not prove non-existence of a
phenomenon; it signals to educational researchers: ‘back to the drawing boards’.
Here’s my considered judgment about the best way to analyze this lexical terrain: Intelligence: We
all have the multiple intelligences. But we signed out, as a strong intelligence, an area where the
person has considerable computational power. Style or learning style: A hypothesis of how an
individual approach the range of materials. If an individual has a ‘reflective style’, he/she is
hypothesized to be reflective about the full range of materials. We cannot assume that
reflectiveness in writing necessarily signals reflectiveness in one’s interaction with the others.
Senses: Sometimes people speak about a ‘visual’ learner or an ‘auditory’ learner. The implication
is that some people learn through their eyes, others through their ears. This notion is incoherent.
Both spatial information and reading occur with the eyes, but they make use of entirely different
cognitive faculties. What matters is the power of the mental computer, the intelligence that acts
upon that sensory information once picked up.
These distinctions are consequential. If people want to talk about ‘an impulsive style’ or a ‘visual
learner’, that’s their prerogative. But they should recognize that these labels may be unhelpful, at
best, and ill-conceived at worst.
In contrast, there is strong evidence that human beings have a range of intelligences and that
strength (or weakness) in one intelligence does not predict strength (or weakness) in any other
intelligences. All of us exhibit jagged profiles of intelligences. There are common sense ways of
assessing our own intelligences, and even if it seems appropriate, we can take a more formal test
battery. And then, as teachers, parents, or self assessors, we can decide how best to make use of
this information.
(Adapted from https://www.washingtonpost.com/news/answer-sheet)

Aula 02 – Verb Tenses 50


www.estrategiamilitar.com.br 67
Teacher Andrea Belo
Aula 02: EN 2021

Questão 01 (AFA/2017) - The text


a) aims at highlighting distinctive mind barriers related to learning.
b) provides the reader with a bird’s-eye-view of Gardner’s landmark publication.
c) develops a considerable set of psychological and mental implications.
d) concerns about spending 30 years to measure people’s intelligence.
Comentários:
Na letra A, há um erro ao afirmar que o texto tem como objetivo (o verbo aims indica objetivo)
destacar (highlight) diferentes barreiras mentais relacionadas ao aprendizado (aims at highlighting
distinctive mind barriers related to learning.) Na realidade, o texto foca no trabalho de Howard
Gardner.
A letra B representa exatamente a ideia do texto: provides the reader with a bird’s-eye-view of
Gardner’s landmark publication. Ele fornece (provides) ao leitor uma visão geral da publicação de
Gardner. Alternativa correta.
A alternativa C erra ao dizer que o texto desenvolve implicações mentais e psicológicas: develops a
considerable set of psychological and mental implications. Na verdade, o texto trata da diferença
entre múltiplas inteligências (multiple intelligences) e estilos de aprendizado (learning styles).
A alternativa D está incorreta porque diz que há preocupações (concerns) pelo fato de serem
necessários 30 anos para medir a inteligência de alguém: concerns about spending 30 years to
measure people’s intelligence. Na realidade, o trabalho de Gardner foi publicado aproximadamente
30 anos atrás. Portanto, o texto relaciona “30 anos” com a publicação do trabalho de Gardner, e não
com o tempo de mensuração da inteligência.

Questão 02 (AFA/2017) - In the sentence “there was a single kind that could be measured by
standardized tests” (lines 07 and 08), it is possible to find an option to substitute the pronoun
accordingly in
a) when.
b) which
c) how.
d) whom.

Aula 02 – Verb Tenses 51


www.estrategiamilitar.com.br 67
Teacher Andrea Belo
Aula 02: EN 2021

Comentários:
O pronome em questão é that.
A alternativa A está incorreta pois when é usado para designar lugar.
A alternativa B está correta porque a palavra which é um pronome que substitui perfeitamente o
pronome that nessa situação. Pode ser traduzido como “o/a qual”.
A alternativa C está incorreta pois how é usado para designar “como” ou “de que forma”.
A alternativa D está incorreta pois whom é usado como objeto na frase, e não como pronome
relativo.
OBS.: No trecho, encontramos os verbos no tempo passado simples (simple past). There was, could
e measured. É interessante perceber que there was é o passado de there is, could é passado de can
e measured é passado de measure. Dessa forma, vamos trabalhando o assunto da aula e nos
familiarizando com os diferentes tempos verbais.

Questão 03 (EPCAR/2018)

Comentários:
A frase “Rumores de que Hitler ainda está vivo circulam desde os anos de 1970” significa que:
Na letra A, “algumas pessoas ainda consideram essa hipótese” está correto. Se ainda há rumores
sobre Hitler estar vivo desde 1970, é porque algumas pessoas ainda acreditam nisso. Alternativa
correta.
Na letra B, “é uma crença antiga que não é mais aceita” é incorreto. Os rumores ocorrem desde os
anos 70, portanto, muitos ainda acreditam nele. Alternativa incorreta.
Na letra C, “foi um rumor ocorrido em 1970” é incorreto. Os rumores vêm circulando desde 1970,
não apenas em 1970. Alternativa incorreta.
Na letra D, “Hitler está vivo desde 1970” é incorreto. Segundo o trecho, são rumores que circulam
desde 1970 que afirmam que Hitler ainda vive. Alternativa incorreta.

Aula 02 – Verb Tenses 52


www.estrategiamilitar.com.br 67
Teacher Andrea Belo
Aula 02: EN 2021

Questão 04 (Escola Naval/2018) -

Comentários:
“Greg [...] e [...] a sua perna enquanto [...] em Bariloche”.
Na letra A, felt (sentiu) já causa uma certa estranheza ao contexto. “Greg sentiu e quebrou a sua
perna enquanto estava esquiando em Bariloche” – além disso, has broken não se encaixa neste
contexto, pois se trata de uma ação que não tem impacto no presente, ela aconteceu apenas em um
determinado momento. Alternativa incorreta.
Na letra B, felt (sentiu) já causa uma certa estranheza ao contexto. “Greg sentiu e tinha quebrado a
sua perna enquanto tem esquiado em Bariloche” – as ações não condizem com uma única linha
temporal, então nenhum sentido é estabelecido. Alternativa incorreta.
Na letra C, “Greg caiu e quebrou a sua perna enquanto estava esquiando em Bariloche” estabelece
uma única linha temporal nos acontecimentos, por isso, apenas o simple past é usado. Alternativa
correta.
Na letra D, fell (caiu) está correto, mas has broken e had been skiing, como visto nas alternativas A
e B, não se encaixam neste contexto. Alternativa incorreta.
Na letra E, felt (sentiu) já causa uma certa estranheza ao contexto. “Greg sentiu e quebrado a sua
perna enquanto estava esquiando em Bariloche” – além disso, broken (quebrado) não se encaixa
neste contexto. Alternativa incorreta.

Questão 05 (Escola Naval/2018) -

Aula 02 – Verb Tenses 53


www.estrategiamilitar.com.br 67
Teacher Andrea Belo
Aula 02: EN 2021

Comentários:
Peter: [...] a Mary não estava no trabalho ontem?
Jane: Ela estava doente.
Na letra A, “Quando” é incorreto. “Quando a Mary não estava no trabalho ontem?” não estabelece
um sentido na própria frase ou no diálogo. Alternativa incorreta.
Na letra B, “Onde” é incorreto. “Onde a Mary não estava no trabalho ontem?” não estabelece um
sentido na própria frase ou no diálogo. Alternativa incorreta.
Na letra C, “Quem” é incorreto. “Quem a Mary não estava no trabalho ontem?” não estabelece um
sentido na própria frase ou no diálogo. Alternativa incorreta.
Na letra D, “Por que” está correto. “Por que a Mary não estava no trabalho ontem?”, “Ela estava
doente” – por mais que a resposta não comece com “Because” (Porque), a justificativa corresponde
à pergunta de Peter. Alternativa correta.
Na letra E, “Qual” é incorreto. “Qual a Mary não estava no trabalho ontem?” não estabelece um
sentido na própria frase ou no diálogo. Alternativa incorreta.

TEXT (EAM/2019) para questões 06 e 07


There's nowhere like Scotland. Scotland is a country in a country. It is part of Great Britain (England,
Scotland and Wales), and of the United Kingdom (England, Scotland, Wales and Northern Ireland).
Scotland is in the far northwest of Europe, between the Atlantic Ocean and the North Sea. It is
often cold and grey, and it often rains a lot. But the people of Scotland love their country, and many
visitors to Scotland love it too. They love the beautiful hills and mountains of the north, the sea and
the eight hundred islands, and the six cities: Edinburgh, Glasgow, Aberdeen, Dundee, Inverness and
Stirling. The country is special, and Scottish people are special too: often warm and friendly. There
are about five million people in Scotland. Most Scots live in the south, in or near the big cities of
Edinburgh and Glasgow. Most of the north of the country is very empty; not many people live there.
A Scottish person is also called a Scot, but you cannot talk about a Scotch person: Scotch means
whisky, a drink made in Scotland. Scottish people are British, because Scotland is part of Great
Britain, but you must not call Scottish people English! The Scots and the English are different. These
days everyone in Scotland speaks English. But, at one time, people in the north and west of Scotland
did not speak English. They had a different language, a beautiful language called Gaelic. About
60,000 people, 1% of the people in Scotland, speak Gaelic now. But many more want Gaelic in their
lives because it is part of the story of Scotland.
Adapted from: FLINDERS, S. Factfiles Seotland. OUP, 2010.

Aula 02 – Verb Tenses 54


www.estrategiamilitar.com.br 67
Teacher Andrea Belo
Aula 02: EN 2021

Questão 06 - Say if the following statements are T (TRUE) or F (FALSE) about Scotland. Then, mark
the correct option, from top to bottom.
( ) It is part of Great Britain but not of the United Kingdom.
( ) It is located between the Atlantic Ocean and the North Sea.
( ) The weather there is usually cold, grey, and rainy.
( ) There are 80 islands in the country.
( ) Tourists can see hills, mountains and the sea there.

(A) (F) (T) (F) (T) (T)


(B) (T) (F) (T) (F) (F)
(C) (F) (T) (T) (T) (T)
(D) (F) (T) (T) (F) (T)
(E) (T) (F) (F) (T) (F)

Comentários:
A primeira afirmativa é falsa. A Escócia é parte da Grã-Bretanha e também é parte do Reino Unido.
“It is part of Great Britain (England, Scotland and Wales), and of the United Kingdom (England,
Scotland, Wales and Northern Ireland).”
A segunda afirmativa é verdadeira. O texto diz que o país fica na região noroeste da Europa e que
fica entre o oceano atlântico e o mar do norte. “Scotland is in the far northwest of Europe, between
the Atlantic Ocean and the North Sea.”.
A Terceira afirmativa é verdadeira. O texto confirma a informação de que a Escócia é normalmente
fria, cinzenta e chuvosa. “It is often cold and grey, and it often rains a lot”. A palavra “often”
expressa a ideia de que algo acontece com frequência.
A quarta afirmativa é falsa. O texto diz que há 800 ilhas na Escócia, e não 80. “the eight hundred
islands”. Este trecho evidencia isso.
A quinta afirmativa é verdadeira. “They love the beautiful hills and mountains of the north, the
sea…”. Este trecho diz que turistas e habitantes adoram as colinas e montanhas do norte, o mar…
Temos, então, a sequência: F / T / T / F / T
A alternativa correta é a letra D.

Aula 02 – Verb Tenses 55


www.estrategiamilitar.com.br 67
Teacher Andrea Belo
Aula 02: EN 2021

Questão 07 (EAM/2019) - About Scotland’s people/population, it is correct to say that


(A) they are often friendly and kind.
(B) most of them live in the north.
(C) they are also called Scotch.
(D) they do not speak English.
(E) nobody speaks Gaelic anymore.
Comentários:
A alternativa A está correta. “Scottish people are special too: often warm and friendly”. Este trecho
confirma que as pessoas escocesas são amigáveis e gentis.
A alternativa B está incorreta. O texto diz que a maioria das pessoas vivem no sul do país, sendo o
norte do país bem vazio. “. Most Scots live in the south, in or near the big cities of Edinburgh and
Glasgow. Most of the north of the country is very empty”.
A alternativa C está incorreta. Não se deve chamar um escocês de scotch, pois essa palavra significa
whisky, que é uma bebida típica da Escócia. “you cannot talk about a Scotch person: Scotch means
whisky”.
A alternativa D está incorreta. O texto diz que todos falam inglês na Escócia, hoje em dia. “These
days everyone in Scotland speaks English”.
A alternativa E está incorreta. Aproximadamente 60.000 pessoas, ou 1% da população, ainda fala
Gaélico. “About 60,000 people, 1% of the people in Scotland, speak Gaelic now”.

Questão 08 (EFOMM/201) -

Aula 02 – Verb Tenses 56


www.estrategiamilitar.com.br 67
Teacher Andrea Belo
Aula 02: EN 2021

Comentários:
“Locais em Raja Ampat dizem que além do dano ao recife, o acidente também exerceu uma grande
sobrecarga na economia local”. A palavra em negrito tem o sentido similar a
Na letra A, “logo que” é incorreto. Essa expressão não corresponde a um sinônimo de besides (além
de), pois não indica adição. Alternativa incorreta.
Na letra B, “ao invés de” é incorreto. Essa expressão não corresponde a um sinônimo de besides
(além de), pois não indica adição. Alternativa incorreta.
Na letra C, “apesar de” é incorreto. Essa expressão não corresponde a um sinônimo de besides (além
de), pois não indica adição, mas concessão. Alternativa incorreta.
Na letra D, “desde que” é incorreto. Essa expressão não corresponde a um sinônimo de besides
(além de), pois não indica adição, mas condição. Alternativa incorreta.
Na letra E, “assim como” está correto. A expressão corresponde ao mesmo significado de besides
(além de), pois ambos indicam adição. “Locais em Raja Ampat dizem que assim como o dano ao
recife, o acidente também exerceu uma grande sobrecarga na economia local”. Alternativa correta.

Questão 09 (EAM/2018) Para questões 09 e 10

Aula 02 – Verb Tenses 57


www.estrategiamilitar.com.br 67
Teacher Andrea Belo
Aula 02: EN 2021

Comentários:
O que eles estão fazendo na figura?
Na letra A, “As crianças jogam basquete todo dia” é incorreto. O enunciado faz a pergunta no
gerúndio utilizando -ing (What are they doing), portanto, a resposta deve ser equivalente, e indicar
uma ação que está sendo realizada. Não se trata do que as crianças fazem todos os dias, mas do que
elas estão fazendo naquele momento. Alternativa incorreta.
Na letra B, “Eles estão jogando basquete agora” está correto. O enunciado faz a pergunta no
gerúndio utilizando -ing (What are they doing), portanto, a resposta deve ser equivalente, e indicar
uma ação que está sendo realizada. “They are playing basketball now” (Eles estão jogando basquete
agora) indica não só uma ação constante no presente (present continuous), como também atende
ao contexto da imagem. Alternativa correta.
Na letra C, “Os garotos não jogaram basquete ontem” é incorreto. O enunciado faz a pergunta no
gerúndio utilizando -ing (What are they doing), portanto, a resposta deve ser equivalente, e indicar
uma ação que está sendo realizada. A frase no passado não corresponde à pergunta feita no
enunciado, e em relação à figura, não há apenas garotos. Alternativa incorreta.
Na letra D, “Eles nunca jogam basquete juntos” é incorreto. O enunciado faz a pergunta no gerúndio
utilizando -ing (What are they doing), portanto, a resposta deve ser equivalente, e indicar uma ação
que está sendo realizada. A resposta no presente não corresponde à pergunta feita no enunciado,
nem condiz com a figura. Alternativa incorreta.
Na letra E, “O garoto e a garota não jogarão basquete” é incorreto. O enunciado faz a pergunta no
gerúndio utilizando -ing (What are they doing), portanto, a resposta deve ser equivalente, e indicar
uma ação que está sendo realizada. Uma resposta no futuro (future tense) não corresponde à
pergunta feita no enunciado, nem condiz com a figura. Alternativa incorreta.

Questão 10 (EAM/2018) About the picture, it is correct to say that


(A) they are practicing a sport that everybody can play if they want.
(B) basketball is played by only two people at the same time.
(C) they should play in a different place to not disturb anybody.
(D) they do play all weekends.
(E) they play and talk at the same time.

Aula 02 – Verb Tenses 58


www.estrategiamilitar.com.br 67
Teacher Andrea Belo
Aula 02: EN 2021

Comentários:
Na letra A, há a afirmação de que eles estão praticando um esporte que todos podem jogar se quiser.
Pode ser a alternativa correta, já que é, de fato, um esporte que qualquer pessoa pode jogar se
quiser mas, vamos analisar as outras alternativas para verificar se há alguma delas melhor do que a
letra A.
Na letra B, a palavra “only” faz com que a alternativa esteja errada pois, basquete não é jogado
apenas por duas pessoas (only two people) e sim quantas quiserem.
Na letra C, diz que eles deveriam jogar em outro lugar para não incomodar as pessoas mas, não há
comprovação de que estão incomodando, então, não faz sentido afirmar isso.
Na letra D, diz que eles jogam todos os fins de semana e, não há como comprovar isso sem um texto
com essa afirmação.
Na letra E, diz que eles jogam e conversam ao mesmo tempo. Pode até ser verdade mas, na figura
não tem algum “balão” ou característica que determine que há, de fato, uma conversa entre eles. E
assim, não podemos fazer tal afirmação.

Aula 02 – Verb Tenses 59


www.estrategiamilitar.com.br 67
Teacher Andrea Belo
Aula 02: EN 2021

18. Considerações finais

Concluímos mais uma aula, outro passo até a sua aprovação! Nota-se o progresso nos
estudos, o ganho de vocabulário e uma maior tranquilidade para enfrentar os exercícios que surgem.
E você vai se acostumando a equilibrar seus estudos de forma sistematizada, estudando cada vez
mais e com mais dedicação.

Outro detalhe importante para seu sucesso nos estudos, é continuar fazendo listas de
vocabulário das palavras e verbos, principalmente os irregulares, que aparecem em forma de lista
em inúmeras fontes de pesquisa.
Isso te ajudará nas questões futuras e torna você, como eu disse antes, um candidato mais
bem preparado e confiante para realizar uma excelente prova.
É importante lembrar também do nosso Fórum de dúvidas, exclusivo do Estratégia
Concursos. Será minha forma de responder o que mais você precise saber para que os conteúdos
fiquem ainda mais claros.

E, caso queira, acesse minhas redes sociais para aprender mais palavras e contar com dicas
importantes, que colaboram diretamente com seus estudos dia após dia.

Aula 02 – Verb Tenses 60


www.estrategiamilitar.com.br 67
Teacher Andrea Belo
Aula 02: EN 2021

Referências bibliográficas

BARRETO, Tania Pedroza; GARRIDO, Maria Line; SILVA, João Antenor de C., Inglês Instrumental.
Leitura e compreensão de textos. Salvador, Ba UFBA, 1995, p. 64.
BROWN. H. Douglas. Principles of Language Learning and Teaching. Prentice Hall International, 1988.
COMPEDELLI, Samira Yousseff. Português, Literatura, Produção de texto & Gramática – São Paulo: Ed.
Saraiva, 2002.
CORREIA, Clese Mary P. Reading Specific Purposes. Salvador/ Ba: UFBA, 1999.
COSTEIRA, Adriana Araújo de M. Reading Comprehension Skills. João Pessoa/PB: ETFP, 1998.
CRYSTAL David. Cambridge University Press 1997. The Cambridge Encyclopedia of Language.
Cambridge University Press 1997
FREEMAN. Diane Larsen. MURCIA. Marianne Celce. The Grammar Book, 1999.
DYE, Joan., FRANFORT, Nancy. Spectrum II, III A Communicative Course in English. USA, Prentice Hall,
1994.
FAVERO, Maria de Lourdes Albuquerque (org.). Dicionário de educadores no Brasil: da colônia aos
dias atuais. Rio de Janeiro : UFRJ, MEC, INEP, 1999.
FRANKPORT, Nancy & Dye Hoab. Spectrum II, III Prentice Hall Regents Englewood Cliffs, New Jersy,
1994.
GADELHA, Isabel Maria B. Inglês Instrumental: Leitura, Conscientização e Prática. Teresina: EDUFFI,
2000.
GUANDALINI, Eiter Otávio. Técnicas de Leitura em Inglês: ESP – English For Specific Purposes: estagio
1. São Paulo: Texto novo, 2002.
GRELLET, Françoise. Developing Reading Skills. Cambridge University Press, 1995
HOLAENDER, Arnon & Sanders Sidney. A complete English Course. São Paulo. Ed. Moderna, 1995.
HUTCHINSON, Tom & WATERS, Alan. English for Specific Purposes. Cambridge: Cambridge University
Press, 1996
KRASHEN. Stephen D. Second Language Acquisition and Second Language Learning, Prentice-Hall
International, 1988.
LAENG, Mauro. Dicionário de pedagogia. Lisboa : Dom Quixote, 1973.
LEFFA, Vilson J. Metodologia do ensino de línguas. In: BOHN, H.; VANDRESEN, P. (org.). Tópicos de
linguística aplicada: o ensino de línguas estrangeiras. Florianópolis: Editora da UFSC, 1988. p. 211-
231.

Aula 02 – Verb Tenses 61


www.estrategiamilitar.com.br 67
Teacher Andrea Belo
Aula 02: EN 2021

LIBERATO, Wilson. Compact English Book Inglês Ensino Médio. São Paulo: FTD, Vol. Único, 1998
Mc ARTHUR. The Oxford Companion to the English Language. Oxford University Press 1992
Fromkin. Victoria. An Introduction to Language
MARQUES, Amadeu. Inglês Série Brasil. ed. Atica. São Paulo: 2004. Vol. Único.
MURPHY, Raymond: Essencial Grammar in Use Oxford. New York Ed. Oxford University, 1997.
OLIVEIRA, Luciano Amaral. English For Tourism Students. Inglês para Estudantes de Turismo: São
Paulo, Rocca, 2001.
OLIVEIRA, Sara Rejane de F. Estratégias de leitura para Inglês Instrumental. Brasília: UNB, 1994.
QUINTANA, et alli. First Certificate. Master Class Oxfor. New York, 2004: Ed. Oxford University.
PAULINO, Berenice F. et all. Leitura em textos em Inglês – Uma Abordagem Instrumental. Belo
Horizonte: Ed. Dos Autores, 1992.
PEREIRA, Edilberto Coelho. Inglês Instrumental. Teresina: ETFPI, 1998.
RODGES, Theodore. Jack C. Richards. Approaches and Methods in Language Teaching. Cambridge
University Press, 2001.
RODMAN Robert. Harcourt Brace 1993. English as a Global Language
STEWART, B., HAINES S. First Certificate, MasterClass. UK – Oxford 2004.
SILVA, João Antenor de C., GARRIDO, Maria Lina, BARRETO, Tânia Pedrosa. Inglês Instrumental:
Leitura e Compreensão de Textos. Salvador: Centro Editorial e Didático, UFBA. 1994
SOARES, Moacir Bretãs. Dicionário de legislação do ensino. 19.ed. Rio de Janeiro : FGV, 1981.
SOUZA, Adriana Srade F. Leitura em Língua Inglesa: Uma abordagem Instrumental. São Paulo: Disal,
2005.
TUCK, Michael. Oxford Dictionary of Computing for Learners of English. Oxford: Oxford University
Press, 1996.
TOTIS, Verônica Pakrauskas. Língua Inglesa: leitura. São Paulo: Cortez, 1991.
Livros eletrônicos:
Dicionário Houaiss da Língua Portuguesa, Editora Objetiva, 2001.
MOURãO, Janaína Pereira. "Skimming x Scanning"; Brasil Escola. Disponível em
<https://brasilescola.uol.com.br/ingles/skimming-x-scanning.htm>. Acesso em 20 de março de 2019.
www.newsweek.com - Acesso em 18 de março de 2019.
http://www.galaor.com.br/tecnicas-de-leitura/ - Acesso em 19 de março de 2019.
Expressões Idiomáticas (continuação)" em Só Língua Inglesa. Virtuous Tecnologia da
Informação,2008-2019. Consultado em 03/04/2019 às 22:09. Disponível na Internet em
http://www.solinguainglesa.com.br/conteudo/Expressoes5.php

Aula 02 – Verb Tenses 62


www.estrategiamilitar.com.br 67
Teacher Andrea Belo
Aula 02: EN 2021

Traduções

TEXT: Howard Gardner: ‘Multiple intelligences’ are not ‘learning styles’ by Valerie Strauss

The fields of psychology and education were revolutionized 30 years ago when we now
worldrenowned psychologist Howard Gardner published his 1983 book Frames of Mind: The
Theory of Multiple Intelligences, which detailed a new model of human intelligence that went
beyond the traditional view that there was a single kind that could be measured by standardized
tests.
Gardner’s theory initially listed seven intelligences which work together: linguistic, logical-
mathematical, musical, bodily-kinesthetic, interpersonal and intrapersonal; he later added an
eighth, naturalist intelligence and says there may be a few more. The theory became highly popular
with K-12¹ educators around the world seeking ways to reach students who did not respond to
traditional approaches, but over time, ‘multiple intelligences’ somehow became synonymous with
the concept of ‘learning styles’. In this important post, Gardner explains why the former is not the
latter.
It’s been 30 years since I developed the notion of ‘multiple intelligences’. I have been gratified by
the interest shown in this idea and the ways it’s been used in schools, museums, and business
around the world. But one unanticipated consequence has driven me to distraction and that’s the
tendency of many people, including persons whom I cherish, to credit me with the notion of
‘learning styles’ or to collapse ‘multiple intelligences’ with ‘learning styles’. It’s high time to relieve
my pain and to set the record straight.
First a word about ‘MI theory’. On the basis of research in several disciplines, including the study
of how human capacities are represented in the brain, I developed the idea that each of us has a
number of relatively independent mental faculties, which can be termed our ‘multiple
intelligences’. The basic idea is simplicity itself. A belief in a single intelligence assumes that we
have one central, all-purpose computer, and it determines how well we perform in every sector of
life. In contrast, a belief in multiple intelligences assumes that human beings have 7 to 10 distinct
intelligences.
Even before I spoke and wrote about ‘MI’, the term ‘learning styles’ was being bandied about in
educational circles. The idea, reasonable enough on the surface, is that all children (indeed all of
us) have distinctive minds and personalities. Accordingly, it makes sense to find out about learners
and to teach and nurture them in ways that are appropriate, that they value, and above all, are
effective.

Aula 02 – Verb Tenses 63


www.estrategiamilitar.com.br 67
Teacher Andrea Belo
Aula 02: EN 2021

Two problems: first, the notion of ‘learning styles’ is itself not coherent. Those who use this term
do not define the criteria for a style, nor where styles come from, how they are recognized/
assessed/ exploited. Say that Johnny is said to have a learning style that is ‘impulsive’. Does that
mean that Johnny is ‘impulsive’ about everything? How do we know this? What does this imply
about teaching? Should we teach ‘impulsively’, or should we compensate by ‘teaching reflectively’?
What of learning style is ‘right-brained’ or visual or tactile? Same issues apply.
Problem #2: when researchers have tried to identify learning styles, teach consistently with those
styles, and examine outcomes, there is not persuasive evidence that the learning style analysis
produces more effective outcomes than a ‘one size fits all approach’. Of course, the learning style
analysis might have been inadequate. Or even if it is on the mark, the fact that one intervention
did not work does not mean that the concept of learning styles is fatally imperfect; another
intervention might have proved effective. Absence of evidence does not prove non-existence of a
phenomenon; it signals to educational researchers: ‘back to the drawing boards’.
Here’s my considered judgment about the best way to analyze this lexical terrain: Intelligence: We
all have the multiple intelligences. But we signed out, as a strong intelligence, an area where the
person has considerable computational power. Style or learning style: A hypothesis of how an
individual approach the range of materials. If an individual has a ‘reflective style’, he/she is
hypothesized to be reflective about the full range of materials. We cannot assume that
reflectiveness in writing necessarily signals reflectiveness in one’s interaction with the others.
Senses: Sometimes people speak about a ‘visual’ learner or an ‘auditory’ learner. The implication
is that some people learn through their eyes, others through their ears. This notion is incoherent.
Both spatial information and reading occur with the eyes, but they make use of entirely different
cognitive faculties. What matters is the power of the mental computer, the intelligence that acts
upon that sensory information once picked up.
These distinctions are consequential. If people want to talk about ‘an impulsive style’ or a ‘visual
learner’, that’s their prerogative. But they should recognize that these labels may be unhelpful, at
best, and ill-conceived at worst.
In contrast, there is strong evidence that human beings have a range of intelligences and that
strength (or weakness) in one intelligence does not predict strength (or weakness) in any other
intelligences. All of us exhibit jagged profiles of intelligences. There are common sense ways of
assessing our own intelligences, and even if it seems appropriate, we can take a more formal test
battery. And then, as teachers, parents, or self assessors, we can decide how best to make use of
this information. (Adapted from https://www.washingtonpost.com/news/answer-sheet)

Aula 02 – Verb Tenses 64


www.estrategiamilitar.com.br 67
Teacher Andrea Belo
Aula 02: EN 2021

TEXTO: Howard Gardner:


'Múltiplas inteligências' não são 'estilos de aprendizagem' de Valerie Strauss

Os campos da psicologia e da educação foram revolucionados há 30 anos, quando agora o psicólogo


de renome mundial Howard Gardner publicou seu livro Frames of Mind: The Theory of Multiple
Intelligences, de 1983, que detalhava um novo modelo de inteligência humana que ia além da visão
tradicional de que havia um tipo único que pode ser medido por testes padronizados.
A teoria de Gardner listou inicialmente sete inteligências que trabalham juntas: linguística, lógico-
matemática, musical, cinestésica, interpessoal e intrapessoal; mais tarde, ele acrescentou uma
oitava inteligência naturalista e diz que pode haver mais algumas. A teoria se tornou muito popular
entre os educadores de ensino fundamental e médio do mundo todo, buscando maneiras de
alcançar os alunos que não responderam às abordagens tradicionais, mas com o tempo,
'inteligências múltiplas' de alguma forma tornaram-se sinônimos do conceito de 'estilos de
aprendizagem'. Neste importante post, Gardner explica por que o primeiro não é o último.
Faz 30 anos que desenvolvi a noção de 'inteligências múltiplas'. Fiquei satisfeito com o interesse
demonstrado nessa idéia e com a maneira como ela foi usada em escolas, museus e negócios em
todo o mundo. Mas uma consequência imprevista me levou à distração e essa é a tendência de
muitas pessoas, incluindo pessoas que eu aprecio, de me creditar com a noção de 'estilos de
aprendizagem' ou de colapsar 'inteligências múltiplas' com 'estilos de aprendizagem'. É hora de
aliviar minha dor e esclarecer as coisas.
Primeiro uma palavra sobre a "teoria do MI". Com base em pesquisas em várias disciplinas, incluindo
o estudo de como as capacidades humanas são representadas no cérebro, desenvolvi a ideia de que
cada um de nós tem várias faculdades mentais relativamente independentes, que podem ser
denominadas nossas 'inteligências múltiplas'. A idéia básica é a própria simplicidade. A crença em
uma única inteligência pressupõe que temos um computador central para todos os fins e determina
o desempenho de todos os setores da vida. Em contraste, uma crença em múltiplas inteligências
pressupõe que os seres humanos tenham 7 a 10 inteligências distintas.
Mesmo antes de falar e escrever sobre "MI", o termo "estilos de aprendizagem" estava sendo usado
nos círculos educacionais. A idéia, suficientemente razoável na superfície, é que todas as crianças
(de fato todos nós) tenham mentes e personalidades distintas. Consequentemente, faz sentido
descobrir sobre os alunos e ensiná-los e alimentá-los de maneiras apropriadas, que valorizem e,
acima de tudo, sejam eficazes.

Aula 02 – Verb Tenses 65


www.estrategiamilitar.com.br 67
Teacher Andrea Belo
Aula 02: EN 2021

Dois problemas: primeiro, a noção de "estilos de aprendizagem" não é coerente. Aqueles que usam
esse termo não definem os critérios para um estilo, nem de onde vêm os estilos, como são
reconhecidos / avaliados / explorados. Diga que se diz que Johnny tem um estilo de aprendizado
"impulsivo". Isso significa que Johnny é "impulsivo" em relação a tudo? Como nós sabemos disso? O
que isso implica no ensino? Devemos ensinar 'impulsivamente' ou devemos compensar 'ensinando
de forma reflexiva'? Qual o estilo de aprendizagem é "cérebro direito" ou visual ou tátil? Os mesmos
problemas se aplicam.
Problema # 2: quando os pesquisadores tentam identificar estilos de aprendizagem, ensinar
consistentemente com esses estilos e examinar os resultados, não há evidências convincentes de
que a análise do estilo de aprendizagem produza resultados mais efetivos do que uma abordagem
de "tamanho único". Obviamente, a análise do estilo de aprendizagem pode ter sido inadequada.
Ou mesmo se estiver certo, o fato de uma intervenção não funcionar não significa que o conceito de
estilos de aprendizagem seja fatalmente imperfeito; outra intervenção pode ter se mostrado eficaz.
Ausência de evidência não prova a inexistência de um fenômeno; sinaliza para os pesquisadores
educacionais: "de volta às pranchetas".
Aqui está o meu julgamento considerado sobre a melhor maneira de analisar esse terreno lexical:
Inteligência: todos nós temos múltiplas inteligências. Mas assinamos, como uma inteligência forte,
uma área em que a pessoa tem um poder computacional considerável. Estilo ou estilo de
aprendizagem: Uma hipótese de como um indivíduo aborda a variedade de materiais. Se um
indivíduo tem um "estilo reflexivo", é provável que ele reflita sobre toda a gama de materiais. Não
podemos assumir que a refletividade na escrita necessariamente sinaliza reflexividade na interação
de uma pessoa com as outras.
Sentidos: às vezes as pessoas falam sobre um aluno 'visual' ou 'auditivo'. A implicação é que algumas
pessoas aprendem com seus olhos, outras através de seus ouvidos. Essa noção é incoerente. Tanto
a informação espacial quanto a leitura ocorrem com os olhos, mas fazem uso de faculdades
cognitivas inteiramente diferentes. O que importa é o poder do computador mental, a inteligência
que age sobre essas informações sensoriais, uma vez captadas.
Essas distinções são consequenciais. Se as pessoas querem falar sobre 'um estilo impulsivo' ou um
'aprendiz visual', essa é uma prerrogativa deles. Mas eles devem reconhecer que esses rótulos
podem ser inúteis, na melhor das hipóteses, e mal concebidos na pior.
Por outro lado, há fortes evidências de que os seres humanos têm uma gama de inteligências e que
a força (ou fraqueza) em uma inteligência não prediz força (ou fraqueza) em outras inteligências.
Todos nós exibimos perfis irregulares de inteligências. Existem maneiras de senso comum de avaliar
nossas próprias inteligências e, mesmo que pareça apropriado, podemos fazer uma bateria de teste
mais formal. E então, como professores, pais ou auto avaliadores, podemos decidir a melhor forma
de usar essas informações.

Aula 02 – Verb Tenses 66


www.estrategiamilitar.com.br 67
Teacher Andrea Belo
Aula 02: EN 2021

There's nowhere like Scotland. Scotland is a country in a country. It is part of Great Britain (England,
Scotland and Wales), and of the United Kingdom (England, Scotland, Wales and Northern Ireland).
Scotland is in the far northwest of Europe, between the Atlantic Ocean and the North Sea. It is
often cold and grey, and it often rains a lot. But the people of Scotland love their country, and many
visitors to Scotland love it too. They love the beautiful hills and mountains of the north, the sea and
the eight hundred islands, and the six cities: Edinburgh, Glasgow, Aberdeen, Dundee, Inverness and
Stirling. The country is special, and Scottish people are special too: often warm and friendly. There
are about five million people in Scotland. Most Scots live in the south, in or near the big cities of
Edinburgh and Glasgow. Most of the north of the country is very empty; not many people live there.
A Scottish person is also called a Scot, but you cannot talk about a Scotch person: Scotch means
whisky, a drink made in Scotland. Scottish people are British, because Scotland is part of Great
Britain, but you must not call Scottish people English! The Scots and the English are different. These
days everyone in Scotland speaks English. But, at one time, people in the north and west of Scotland
did not speak English. They had a different language, a beautiful language called Gaelic. About
60,000 people, 1% of the people in Scotland, speak Gaelic now. But many more want Gaelic in their
lives because it is part of the story of Scotland.

Não há lugar como a Escócia. A Escócia é um país em um país. Faz parte da Grã-Bretanha
(Inglaterra, Escócia e País de Gales) e do Reino Unido (Inglaterra, Escócia, País de Gales e
Irlanda do Norte). A Escócia fica no extremo noroeste da Europa, entre o Oceano Atlântico e
o Mar do Norte. Geralmente é frio e cinza, e chove muito. Mas o povo da Escócia ama seu
país, e muitos visitantes da Escócia também o amam. Eles adoram as belas colinas e
montanhas do norte, o mar e as oitocentas ilhas e as seis cidades: Edimburgo, Glasgow,
Aberdeen, Dundee, Inverness e Stirling. O país é especial, e o povo escocês também é especial:
geralmente caloroso e amigável. Existem cerca de cinco milhões de pessoas na Escócia. A
maioria dos escoceses vive no sul ou nas grandes cidades de Edimburgo e Glasgow. A maior
parte do norte do país está muito vazia; muitas pessoas não moram lá. Uma pessoa escocesa
também é chamada de escocês, mas não se pode falar de um escocês: escocês significa
uísque, uma bebida feita na Escócia. O povo escocês é britânico, porque a Escócia faz parte
da Grã-Bretanha, mas você não deve chamar o povo escocês de inglês! Os escoceses e os
ingleses são diferentes. Hoje em dia, todos na Escócia falam inglês. Mas, ao mesmo tempo,
as pessoas no norte e oeste da Escócia não falavam inglês. Eles tinham um idioma diferente,
um idioma bonito chamado gaélico. Cerca de 60.000 pessoas, 1% das pessoas na Escócia,
falam gaélico agora. Mas muitos mais querem gaélico em suas vidas porque faz parte da
história da Escócia.

Aula 02 – Verb Tenses 67


www.estrategiamilitar.com.br 67
Aula 03 –
Articles and
Nouns

Escola Naval 2021

Teacher Andrea Belo


Teacher Andrea Belo
Aula 03: Escola Naval 2021

Sumário

Introdução ......................................................................................................................... 3
Artigo definido the ............................................................................................................ 4
Artigos indefinidos A e AN ................................................................................................. 6
Substantivos ...................................................................................................................... 8
Substantivo comum ........................................................................................................................... 8
Substantivos próprios ........................................................................................................................ 8
Substantivo composto ..................................................................................................................... 10
Substantivos abstrato e concretos .................................................................................................. 12
Substantivo coletivo ........................................................................................................................ 13
Substantivos contáveis .................................................................................................................... 15
Substantivos incontáveis ................................................................................................................. 16
Curiosidades sobre artigos ............................................................................................... 23
Curiosidades sobre substantivos ...................................................................................... 25
Questões inéditas ............................................................................................................ 27
Questões de anos anteriores ........................................................................................... 31
Gabarito .......................................................................................................................... 38
Respostas comentadas .................................................................................................... 39
Considerações finais ........................................................................................................ 51
Referências bibliográficas ................................................................................................ 52
Traduções ........................................................................................................................ 54

Aula 03 – Articles and Nouns 2


www.estrategiamilitar.com.br 61
Teacher Andrea Belo
Aula 03: Escola Naval 2021

Introdução
Chegou a vez da nossa aula sobre artigos e também sobre substantivos.
Mesmo que haja um vasto conhecimento sobre as regras de gramática em Inglês, sem os
artigos e os substantivos, a comunicação seria impossível. Isso porque, etimologicamente,
substantivo significa, literalmente, aquilo que está debaixo, a base.
E, se os artigos vêm antes dos substantivos, determinando-os, ambos são fundamentais aos
textos.
Vamos falar um pouco de artigos. Quando eu explico o uso dos artigos em Inglês, sempre
gosto de lembrar que, o artigo definido THE, por exemplo, está na lista das palavras mais usadas na
língua inglesa. Então, aprender a usar o the e os outros artigos, de forma correta, é de grande
importância para melhorar seu vocabulário em Inglês.
Os artigos estão em todo lugar, em todos as frases dos textos e inseridos nos contextos. Por
isso, é essencial que você tenha conhecimento de como usar os artigos de forma correta, mesmo
sabendo que há regras, muitas regras.
Mas é assim também em Português e em todas as línguas e, saber mais sobre cada item, que
estará presente em sua prova, deixará você devidamente preparado.
Os artigos em Inglês, são classificados como definidos e indefinidos e colaboram
imensamente na interpretação dos textos, pois são os artigos que revelam, que mostram para você
se a palavra que foi mencionada no texto aparece no contexto pela primeira vez ou essa palavra está
retomando alguma ideia já expressada anteriormente, ajudando em sua compreensão do texto
como um todo. Em nossa aula, ficará fácil e claro para você.
O substantivo, por sua vez, é a classe de palavras que nomeia seres, objetos, lugares,
sentimentos, ou seja, denomina e classifica a maioria das palavras constituintes das frases que
usamos para nos comunicar. A palavra substantivo é derivada da família de substância e substancial,
que significam, fundamental.
Nessa aula, estudaremos todos os artigos e todos os substantivos em Inglês, com exemplos e
explicações completas, já que esses termos estão entre as palavras mais importantes de um idioma,
pois estão presentes em nossos discursos, nos textos que escrevemos e lemos e em nossos
pensamentos quando queremos expressar opiniões.
Vamos estudá-los!

Aula 03 – Articles and Nouns 3


www.estrategiamilitar.com.br 61
Teacher Andrea Belo
Aula 03: Escola Naval 2021

Artigo definido THE


Os artigos definidos que temos na língua portuguesa são: o, a, os e as. Em Inglês, eles são
representados por apenas uma palavra: o artigo definido the.
Definite Article, o artigo definido, é o termo usado para se referir a algo específico ou algo
que já foi mencionado anteriormente.
Como já afirmei, existe apenas um artigo definido em Inglês, the, e ele é utilizado diante de
substantivos tanto no singular quanto no plural, pois a única palavra the significa o, a, os e as.
Exemplos simples: The car is blue – O carro é azul. The cars are blue – Os carros são azuis. The
table is big – A mesa é grande. The tables are big – As mesas são grandes.
Observe também outro exemplo = The backpack is red – A mochila é vermelha – A cor
vermelha especifica o substantivo mochila (backpack). Assim, não se trata de uma mochila qualquer,
e sim da mochila vermelha, determinada pelo artigo definido the. Na maioria das vezes, esse artigo
é usado antes dos substantivos, para defini-los.
Vejamos outros casos do correto uso do artigo definido the.
Usamos o artigo definido antes de substantivos já mencionados, como por exemplo: Tom
wrote a book. The book is about art – Tom escreveu um livro. O livro é sobre artes. O artigo the foi
usado para determinar o livro que já havia sido mencionado: o livro que Tom escreveu.
Usamos o artigo definido também para substantivos considerados únicos em sua espécie
como a lua, o sol etc: the moon (a lua), the sun (o sol), the sky (o céu), the planet Earth (o planeta
Terra), the universe (o universo): The sun is shining – O sol está brilhando.
Outro caso em que usamos o artigo definido é quando se fala de algo que se remete a nomes
geográficos de um rio, um mar, um oceano, canais, polos, desertos, golfos, ilhas e montanhas, veja:
the Mississipi River, the Pacif ocean, the English Channel (O Canal da Mancha), the South Pole (o polo
Sul) , the Sahara desert, the Gulf of Mexico, the Bahamas, the Alps etc. Exemplo: The Pacific Ocean
is very big. O oceano pacífico é muito grande.
Um caso interessante do uso do artigo definido é quando se fala de adjetivos que estão sendo
usados como substantivos no plural, como: the poor (os pobres), the powerful (os poderosos), the
strong (os fortes), the good (os bons), the bad (os maus) etc. Você deve estar perguntando: e os
substantivos no singular, como por exemplo, o poderoso? Como fica a frase? Respondo a você que,
precisa usar o nome, o substantivo, veja: The powerful man helped the boy – O poderoso homem
ajudou o garoto.
Usamos o artigo definido também para nomes compostos de países, como os Estados Unidos,
o Reino Unido, os Emirados Árabes e a República Dominicana: The United States, The United
Kingdom, The United Arab Emirates, The Dominican Republic. He lives in the USA – Ele mora nos EUA,
sendo in the = nos = em + os, o e artigo definido os = the.
Quando os nomes próprios indicam um grupo familiar, ou seja, o nome de uma família inteira,
usamos também o artigo definido: The Smiths went to the restaurant yesterday – Os Smith (os
membros da família Smith) foram ao restaurante ontem. É muito importante ressaltar que, para
nomes próprios em geral e nomes de pessoas, não se usa artigo definido, nunca.

Aula 03 – Articles and Nouns 4


www.estrategiamilitar.com.br 61
Teacher Andrea Belo
Aula 03: Escola Naval 2021

Apenas quando estiver se referindo a um nome próprio que já possuam the em sua estrutura,
como The Economist, The New York Times, The Beatles, The Washington Post, The Empire States,
The Louvre, The Kremlin, The Taj Mahal, The Vatican etc.
Então, ao dizer Susan is my friend – Susan é minha amiga, não podemos jamais escrever ou
falar "The Susan is my friend" – nunca, certo?
Usamos o artigo definido também quando se trata do grau superlativo em Inglês (teremos
uma aula destinada aos adjetivos, em que explicarei, com detalhes, os graus comparativo e
superlativo), como por exemplo: Bob is the tallest guy in our classroom – Bob é o cara mais alto da
nossa sala. Kathy is the shortest girl in our classroom – Kathy é a garota mais baixa da nossa sala.
Agora vejamos os casos em que não devemos usar o artigo definido, como já citei acima, por
exemplo, em nomes próprios.
Portanto, não se deve colocar the antes de nomes de cidades, estados, países e continentes.
Para dizer que a Europa é um grande continente, ficaria: Europe is a big continent, sem o artigo no
início da frase.
E também não usamos o artigo definido the antes de nomes de disciplinas acadêmicas
esportes, ciências, cores, estações do ano, meses e dias da semana. Vejamos exemplos.
Tennis is a great sport. O tênis é um ótimo esporte.
Biology is an important science. A Biologia é uma ciência importante.
Yellow is Paul's favorite color. O amarelo é a cor favorita de Paulo.
Também não devemos usar o artigo definido para substantivos que estiverem no plural,
porém utilizados em sentido genérico, como por exemplo as pessoas (referindo-se a muitas pessoas,
a uma maioria de pessoas): People watch too much TV – As pessoas assistem muita televisão, sem
the para “as” pessoas.
Outro caso em que também não usamos o artigo definido é quando há substantivos abstratos
ou nomes que indicam algum tipo de material: Hope is stronger than fear – A esperança é mais forte
do que o medo. E Silk produces soft clothes – A seda produz roupas macias.
Devemos ficar atentos quando houver referência a cargos e títulos, pois, quando usados sós
na oração, usa-se o artigo definido.
Mas, omite-se o artigo definido quando o nome próprio estiver junto, veja: The president is
on TV now – O presidente está na TV agora. Porém, veja essa outra frase: President Kennedy was
murdered – O presidente Kennedy foi assassinado.
Quando vamos construir frases no passado e futuro, usando last e next, não usamos o artigo
the antes deles, pois são expressões temporais: I will travel next month.
Eu vou viajar no próximo mês e não “in the next month” ou em referência ao passado last
month e nunca “in the last month”, isso não existe.
Agora, em complemento ao capítulo que você estudou sobre o artigo definido, vamos estudar
os artigos indefinidos a e an. Come on!

Aula 03 – Articles and Nouns 5


www.estrategiamilitar.com.br 61
Teacher Andrea Belo
Aula 03: Escola Naval 2021

Artigos indefinidos A e AN
Os Indefinite articles – artigos indefinidos que temos na língua portuguesa: um, uma, uns,
umas, em Inglês, são representados por apenas duas palavras: a e an e são usados quando nos
referimos a algo em geral, algo não especificado. Eles apenas podem ser usados antes de
substantivos que estão no singular.
Muitas pessoas aprendem erroneamente que a se usa antes de palavras que se inicia com
consoante enquanto an se usa para palavras que se iniciam com vogal. Na verdade, essa regra
funciona quase sempre, mas os artigos indefinidos também dependem do som das letras iniciais das
palavras em que são empregados.
Por exemplo, dizer “um homem velho’ em Inglês é an old man, por causa da letra “o” no início
da palavra old. Mas, dizer “um homem honesto” é an honest man, por causa do som de “o” da
palavra honest.
Apesar de começar com a letra “h”, essa letra não é pronunciada na palavra honest, como na
maioria em que o “h” possui o som de “r”, por exemplo, na palavra house. Se o som for da consoante
“r”, aplicamos o artigo indefinido a: He wants to buy a house (Ele quer comprar uma casa).
Outros casos em que devemos analisar a fonética para usar o artigo correto, é em palavras
cuja vogal inicial tem o som de consoante, que geralmente começam com “u” e “eu”, mas tem o
som de “y”, que é consoante em Inglês: This is a university. (Essa é uma universidade), I am reading
a European Journal. (Estou lendo um jornal europeu)
Apesar de iniciar com a vogal “o”, a palavra one tem o som da consoante “w” e, por isso,
também pede o uso do artigo indefinido a e não an: She lives in a one-story small house (Ela mora
em uma pequena casa de um andar).
Em regra geral, na maior parte das palavras que começam com vogais, usa-se o artigo
indefinido an e, por sua vez, o artigo indefinido a, é usado para palavras que iniciam com consoante:
She left an hour ago (Ela saiu há uma hora), He is a jornalist (Ele é um jornalista), I will buy a new
skirt (Eu comprarei uma saia nova), He will see an elephant at the zoo (Ele verá um elefante no
zoológico) etc.
O artigo a deve ser usado antes das palavras few e little quando têm sentido positivo, como:
She can see a few buildings far away (Ela consegue avistar alguns prédios de longe), como se fosse
“um pouco de prédios” e por isso a few, mas a melhor tradução é alguns prédios. Outro exemplo:
She wants just a few milk with coffee (Ela quer apenas um pouco de leite com café).
Agora vejamos os casos em que não devemos usar os artigos indefinidos. Como eu havia dito
antes, os artigos indefinidos a e an não devem ser usados quando a ideia principal da frase se refere
a substantivos no plural: a e an não significam uns/umas nunca e só podem ser usados
exclusivamente no singular. Explicarei o que fazer com palavras no plural. Vamos lá.
Não devemos usar artigos indefinidos antes de substantivos incontáveis (embora isso
aconteça em Português).
Então, usamos some, como por exemplo: She gave me some advice (Ela me deu um conselho)
e como conselho em Inglês é incontável – advice – usamos some e não o artigo an.

Aula 03 – Articles and Nouns 6


www.estrategiamilitar.com.br 61
Teacher Andrea Belo
Aula 03: Escola Naval 2021

Em um esquema simples usando os artigos definidos e indefinidos de forma resumida para


ajudar você a se lembrar das regras, poderíamos dizer que:

Usamos o artigo indefinido "A" quando as palavras


possuem uma consoante como letra inicial e, além da grafia,
sua pronúncia também deve ser de consoante.

Usamos o artigo indefinido "AN" quando as palavras


possuem uma vogal como letra inicial e, além da grafia,
sua pronúncia deve ser de uma das vogais também.

Usamos o artigo definido "THE" para palavras contáveis e


incontáveis, femininas e masculinas e, tanto para o singular
como para o plural, já que apenas THE significa "o,a, os, as".

Agora, vamos aos substantivos. Let’s go!

Aula 03 – Articles and Nouns 7


www.estrategiamilitar.com.br 61
Teacher Andrea Belo
Aula 03: Escola Naval 2021

Substantivos
Os Nouns, substantivos em Inglês, existem para nomear os seres em geral, representados por
pessoas, lugares, instituições, grupos, elementos da natureza etc.
São aquelas palavras presentes em nossos pensamentos, em nossos discursos, na fala e na
escrita. Os substantivos integram as classes gramaticais e estão relacionados à formação e ao
significado de palavras.
De acordo com uma pesquisa da palavra substantivo, no Dicionário Etimológico, percebemos
que substantivo se origina se substância, parte essencial de alguma coisa. E, conforme a Nova
Gramática do Português Brasileiro, o termo substantivo significa literalmente o que está debaixo, na
base.
A definição de substantivo em Inglês – NOUN – em variados dicionários é praticamente a
mesma: “a noun is a kind of word that is usually the name of something such as a person, place,
thing, quality, or idea.”- um substantivo é um tipo de palavra que geralmente nomeia coisas, tais
como uma pessoa, lugar, qualidade ou ideia. Em Inglês, há mais substantivos do que qualquer outra
classe de palavras, sabia?
Vamos estudar, a parte fundamental dos textos que você vai ler e interpretar em sua prova,
que são, de fato, os substantivos.
Os substantivos podem ser: common noun (substantivo comum), proper noun (substantivo
próprio), compound noun (substantivo composto), countable/uncountable nouns – substantivos
contáveis e incontáveis, com as variações de singular/plural (singular e plural) nos substantivos e
certas regras que estudaremos.
Veremos cada um deles com exemplos, já que há peculiaridades e regras que envolvem todos
os tipos de substantivos. Vamos começar pelo substantivo comum.

Substantivo comum

Os common nouns (substantivos comuns) são aqueles que nomeiam seres da mesma espécie,
fazem referência a uma pessoa, lugar ou coisa de forma geral. São os mais presentes nas provas,
pois estão nos textos que você vai precisar ler no dia da prova.
São escritos com a letra inicial minúscula, exceto se iniciarem a frase. Exemplos: book (livro),
table (mesa), city (cidade) etc.

Substantivos próprios

Por sua vez, os proper nouns (substantivos próprios) são aqueles que representam o nome de
uma determinada pessoa, de uma entidade, de lugares em geral. Também aparecem bastante nas
provas para nomear pessoas e lugares de referência nos textos variados.

Aula 03 – Articles and Nouns 8


www.estrategiamilitar.com.br 61
Teacher Andrea Belo
Aula 03: Escola Naval 2021

Sempre devem ser escritos com letra maiúscula.


Além de pessoas e lugares, são substantivos próprios, em Inglês, os dias da semana, dos
meses, de documentos históricos e de instituições.
Exemplos: Robert, Mississipi River, Cambridge University, Monday, January – Roberto, Rio
Mississippi, Universidade Cambridge, segunda-feira, janeiro.
Vamos fazer uma pequena comparação com exemplos para ficar clara a diferença entre
substantivos comuns e próprios, sendo que, as palavras da primeira coluna se relacionam com as da
segunda, assim: para cada exemplo de substantivo comum, há um próprio que o exemplifique, como
school (escola) em relação à Oxford University, que é um nome, um substantivo próprio nomeando
uma universidade, exemplo de escola. E assim por diante. Veja:

Substantivos Substantivos

Proper Nouns
Common Nouns

comuns: próprios:
SCHOOL OXFORD / CAMBRIDGE
CAR PORSHE / FERRARI
TEA EARL GREY
MAN PAUL SMITH
WEBSITE ESTRATEGIA.COM
MOVIE AVENGERS
MOUNTAIN EVEREST
CITY / STATE SÃO PAULO
ANIMAL MICKEY MOUSE /
GARFIELD
OCEAN
ATLANTIC OCEAN
HOUSE
WHITE HOUSE
TOWER
EIFFEL TOWER
STATUE
STATUE OF LIBERTY
RIVER
MISSISSIPI RIVER

Aula 03 – Articles and Nouns 9


www.estrategiamilitar.com.br 61
Teacher Andrea Belo
Aula 03: Escola Naval 2021

Substantivo composto

Quando temos a combinação de dois ou mais substantivos comuns ou próprios formando


um só, são compound nouns (substantivos compostos), tais como driving license (carteira de
habilitação) e washing machine (máquina de lavar roupas).
Às vezes não estão separados por itens, mas são compostos por duas palavras: bedroom
(quarto), motorcycle (motocicleta), haircut (corte de cabelo), policeman (policial ou agente de
polícia) etc.
Em substantivos compostos com preposição ou advérbio, pluralizamos o seu componente
principal (geralmente é a primeira palavra): sister-in-law – sisters-in-law (cunhadas), godmother –
godmothers (madrinhas), maidservant – maidservants (criadas), runner-up – runners-up (vice-
campeões).
Vejamos uma lista com alguns substantivos compostos que já apareceram e podem aparecer
novamente nas provas:
ARMCHAIR

AIRLINE

AIRPORT

BACKGROUND
COMPOUND NOUNS

BATHROOM

BUS STOP

CLASSMATE

CUPBOARD

DAYDREAM

DOWNSTAIRS

EGGPLANT

EVERYBODY

FRIENDSHIP

FOOTPRINT

GENTLEMAN

GOLDFISH

HEADACHE

HOMEWORK

HIGHWAY

HAIRCUT

INSIDE

INTAKE

Aula 03 – Articles and Nouns 10


www.estrategiamilitar.com.br 61
Teacher Andrea Belo
Aula 03: Escola Naval 2021

LADYBUG

LIGHTHOUSE

MOONLIGHT

MOTHER-IN-LAW

NOTEBOOK
COMPOUND NOUNS
NEWSPAPER

OUTDOOR

OUTSIDE

PARTNERSHIP

PHOTOCOPY

POPCORN

PETSHOP

OUTDOOR

ONLINE

POLICEMAN

POPCORN

RAINBOW

SKATEBOARD

TIMETABLE

WEEKEND

YOURSELF

Aula 03 – Articles and Nouns 11


www.estrategiamilitar.com.br 61
Teacher Andrea Belo
Aula 03: Escola Naval 2021

Substantivos abstrato e concretos

Assim como em Português, também temos também os substantivos abstratos – abstract


nouns – e os substantivos concretos – concrete nouns.
Os abstratos representam algo imaginado, pois designam coisas intangíveis tais como ações,
sentimentos, ideias, conceitos e qualidades.
Exemplos de substantivos abstratos: peace, honesty, education, chaos, stress etc – paz,
honestidade, educação, caos, estresse.
Os concretos se referem às coisas que podemos ver e perceber ao nosso redor, já que
designam seres e coisas da realidade física, coisas que podemos ter contato através dos nossos cinco
sentidos: visão, audição, tato, paladar e olfato.
Exemplos de substantivos concretos: roses, sunset, sandwich, bus, voice, school etc. – rosas,
pôr do sol, sanduíche, ônibus, voz, escola etc.

Aula 03 – Articles and Nouns 12


www.estrategiamilitar.com.br 61
Teacher Andrea Belo
Aula 03: Escola Naval 2021

Substantivo coletivo

Há também os collective nouns (substantivos coletivos) que denotam um conjunto de número


indeterminado de seres ou coisas, ou seja, designam um grupo de seres da mesma espécie.
Perceba que, o substantivo coletivo é uma única palavra que, mesmo usada no singular, indica
o agrupamento.
Entender e saber aplicar os conceitos de gramática e ortografia da nossa própria língua é
essencial no nosso dia a dia.
A língua portuguesa, quando comparada à língua inglesa, possui mais coletivos inseridos nos
discursos, nos textos em geral.
Mas, podemos notar que, em qualquer língua, a riqueza de vocabulário permite que a maioria
das palavras sejam substituídas por outras, mais simples e poucas pessoas usam os substantivos
coletivos, de fato.
Em textos científicos, acadêmicos, artigos, reportagens, revistas e jornais que você deve ler,
esses substantivos coletivos podem aparecer, como em Português, usa-se enxame, coletivo de
abelhas, para se referir a esses insetos.
Em Português, temos também a palavra arquipélago, coletivo de ilhas, que é mais comum
aparecer em leituras mais técnicas, assim como as suas fontes de estudo. Outros exemplos são
cardume - coletivo de peixes, banca - para examinadores e elenco - de atores.
Há coletivos bastante incomuns, em todas as línguas. São palavras quase não usadas ou que
não se escuta, como por exemplo em Português, girândola, coletivo de foguetes e também atilho,
coletivo de espigas de milho.
Em Inglês, do mesmo modo, há coletivos básicos, usados no cotidiano, como a palavra forest
(para um grupo de árvores), que muitas pessoas nem sabem que é classificada como um substantivo
coletivo.
Como exemplo de coletivos mais acessíveis, que podem aparecer em provas são considerados
simples, temos family (família) para classificar parentes de uma mesma linhagem, temos o coletivo
audience (público) para pessoas, entre outros.
Alguns, como em Português, são atípicos, como o coletivo flock (rebanho ou bando de
pássaros) e swarm (enxame de abelhas).
Vejamos, a seguir, uma lista de coletivos como curiosidade, já que raramente são
disponibilizados em materiais que tratam desse assunto, vistos geralmente de forma superficial. E
você estará preparado, caso apareça alguma dessas palavras em sua prova.

Aula 03 – Articles and Nouns 13


www.estrategiamilitar.com.br 61
Teacher Andrea Belo
Aula 03: Escola Naval 2021

Substantivos Coletivos em Inglês: Collective Nouns

Agenda = tasks (tarefas agendadas) Elephant = herd (manada)


Ambush = people (legião) Faculty = academics (acadêmicos)
Ants = colony (colônia de formigas) Fleet = aircrafts (esquadrilha de aeronaves)
Anthology = poems (antologia) Fleet = ships (esquadra de navios)
Apes = troop (macacos) Flock = tourists (turistas)
Army = soldiers (soldados) Forest = trees (Floresta)
Audience= spectators (espectadores) Galaxy = stars (estrelas)
Battery = tests (bateria de exames) Grove = trees (bosque de árvores)
Bats = colony (morcegos) Handful = children (crianças)
Beavers = colony (castores) Huddle = lawyers group (advogados)
Bench = magistrates (magistrados) Indians = tribe (tribo)
Bevy = beauties (beldades) Jury = judges (juízes)
Board = directors (diretores) Neverthriving = jugglers (malabaristas,
Brood = researchers (pesquisadores) mágicos, ilusicionistas)
Cast = actors/actresses (elenco) Mob = criminals (criminosos)
Camels = caravan (caravana de camelos) Orchestra = musicians (músicos)
Chain = islands (arquipélago) Tigers = Ambush (alcateia de tigres)
Chorus = angels (coro de anjos) Thieves = gang (ladrões)
Clowns = troupe (palhaços) Panel = experts (especialistas)
Clump = trees (arvoredo) Peloton = cyclists (ciclistas)
Collection = objects (coleção) People = crowd (multidão)
Congregation = workshippers (adoradores) Shush = librarians (bibliotecários)
Crew = sailors (tripulação) Squad = soldiers (esquadrão de soldados)
Convoy = lorries/truck(frota de caminhão) Staff = employees (funcionários)
Deck = cards (cartas) Sheep = A flock of sheep (rebanho)

Dogs = pack (matilha de cães) Soldiers = army (exército)


Drift = lecturesrs (conferencistas) Team = athletes (atletas)

Aula 03 – Articles and Nouns 14


www.estrategiamilitar.com.br 61
Teacher Andrea Belo
Aula 03: Escola Naval 2021

Substantivos contáveis

Temos também substantivos contáveis – countable nouns – são aqueles que podem ser
contados, enumerados.

Os substantivos contáveis possuem forma no plural e no singular, como podemos ver em one
book (um livro), two houses (duas casas) e one hundred years (cem anos).

Por serem classificados como contáveis, às vezes aparecerão precedidos de números ou


precedidos dos artigos definidos a/an, que vimos anteriormente.

Veja alguns exemplos: a pear, an apple, a pair of shoes (uma pera, uma maçã, um par de
sapatos).

Aula 03 – Articles and Nouns 15


www.estrategiamilitar.com.br 61
Teacher Andrea Belo
Aula 03: Escola Naval 2021

Substantivos incontáveis

Os substantivos incontáveis – uncountable nouns – são aqueles que não podem ser contados
ou enumerados. Assim, são usados somente no singular e por isso, não são antecedidos pelos artigos
a/an, não tem plural e não são usados depois de números.

Veja: I have good news for you. (Eu tenho boas notícias para você), She needs some help (Ela precisa
de ajuda).

Alguns exemplos comuns, sempre presentes nos textos das provas são: money (dinheiro),
music (música – estilo musical), bread (pão), entre outros que veremos as regras adiante.
Devemos nos lembrar que, ao contrário do Inglês, usamos, em Português, alguns destes
substantivos como contáveis.
Temos o costume de dizer, por exemplo, Por favor, me dê uma água, que não pode ser
traduzido Please, give me a water, está errada essa construção. O certo é: Please, give me a glass of
water – Por favor, me dê um copo de água ou Please, give me a bottle of water – Por favor, me dê
uma garrafa de água, já que copo e garrafa são contáveis.
Vejamos alguns exemplos de substantivos incontáveis que, você precisa saber o que usar
junto a eles para que seja possível usá-los em frases com sentido de contagem, já que não se pode
usar a ou an.
Um deles é o substantivo salt (sal) – para ser contável, dizemos uma colher de sal, por
exemplo – a teaspoon of salt.
A palavra money (dinheiro) – não existe “um dinheiro” e, para ser contável, usamos o nome
da moeda a que se refere, como por exemplos ten dollars – 10 dólares, five 5 pounds – 5 libras, one
real – 1 real.
Outro substantivo incontável é music. Primeiro, porque a palavra music não se refere apenas
a uma música específica (que é song) como em Português pois music se refere a um estilo musical –
He loves rock music (Ele ama rock, o estilo rock, certo?).
Temos a palavra bread (pão), que, em Português, é usada como contável, mas em Inglês não
se conta o pão e sim as fatias – I want a loaf of bread (Quero uma fatia de pão) e coffee (café), em
que não se conta um café e sim a xícara – I’d like a cup of coffee (Eu gostaria de uma xícara de café).
Para a palavra queijo – cheese, também não se fala: Eu quero um queijo e sim um pedaço,
uma fatia de queijo – I’d like a piece of cheese, a slice of cheese.
E daqui por diante, vamos ver outros possíveis substantivos que podem estar em sua prova.
Arroz, em Inglês – rice – para demonstrar quantidade, é correto usar um saco de arroz, um
grão de arroz: a bag of rice, a grain of rice.
E também papel – paper – não falamos um papel, como é usado em Português. Temos que
dizer uma resma de papel ou uma folha de papel: a ream of paper, a sheet of paper.

Aula 03 – Articles and Nouns 16


www.estrategiamilitar.com.br 61
Teacher Andrea Belo
Aula 03: Escola Naval 2021

Para o substantivo conselho – advice – que, para se falar que vai dar um conselho é I will give
you some advice. O mesmo acontece com informação – uma informação seria some information pois
“one” information ficaria errado e não se usa nem sequer informalmente.
Vamos ao uso da palavra notícia ou notícias em Inglês – news – que para dizer uma notícia ou
alguma notícia é a piece of news, some news. Esta palavra é sempre usada com verbos no singular.
Por exemplo: The news is fantastic. (As notícias são fantásticas). Mesmo que traduzida por
“notícias”, o verbo to be usado deve ser no singular: is.
Um substantivo bastante interessante é cabelo um Inglês – hair – porque, apesar de ser incontável:
pode ser usado como substantivo contável, significando de fio(s) de cabelo. Veja dois diferentes
exemplos do uso de hair:
My hair is black – Meus cabelos são pretos.
Her hair is brown. When she finds a gray hair, she immediately pulls it out - Os cabelos dela
são marrom. Quando ela acha um fio grisalho, ela o arranca imediatamente.
Vejamos outros substantivos incontáveis, para ficar ainda mais claro, separados por
categoria, facilitando a visualização.
Preparei uma lista, para você, com uncountable nouns divididos em food (comida), feelings
(sentimentos) e everyday stuff (coisas do dia-a-dia):

Aula 03 – Articles and Nouns 17


www.estrategiamilitar.com.br 61
Teacher Andrea Belo
Aula 03: Escola Naval 2021

Podemos dizer, por exemplo, um suco, enquanto em Inglês, não existe “one juice” e sim some
juice, a cup of juice, a glass of juice ou a liter of juice que seriam a representação de possíveis
traduções: um pouco de suco, um copo de suco ou um litro de suco, por serem substantivos
incontáveis.
Eis, agora, alguns exemplos ilustrativos com countable and uncountable food and drink (alimentos e
bebidas contáveis e incontáveis) para você comparar e perceber como é diferente em Inglês
comparados ao Português.
Perceba, por exemplo, que a palavra cereal, em Inglês, é incontável enquanto em Português,
dizemos cereais. Veja outros exemplos:

Outros exemplos bastante interessantes são os numbers (números em Inglês), que, no plural,
não sofrem mudança alguma de flexão, em sua maioria.

Em casos onde há substantivos com algum número e hífen, forma-se uma expressão e o plural
fica sem o -s, já que no inglês os adjetivos são invariáveis, como podemos ver em:

I have an 18-hour day (Tenho uma jornada de 18 horas)


He was in a five-star hotel (Ele estava um hotel cinco estrelas)

She works an 8-hour period on Mondays. (Ela trabalha um período de 8 horas na segunda-feira)
E os numerais seguintes: dozen (dúzia), hundred (cem/centena), thousand (mil) e million
(milhão) e billion (bilhão), permanecem sem o -s: 8 million people - E não 8 millions para 8 milhões.

Aula 03 – Articles and Nouns 18


www.estrategiamilitar.com.br 61
Teacher Andrea Belo
Aula 03: Escola Naval 2021

Veja: 200,000 - two hundred thousand – Three pens (três canetas) Three million dollars (três
milhões de dólares), two billion (dois bilhões).
Se, por acaso, for acrescentado –s ao final dos números, estamos nos referindo a números
indefinidos:
Dozens of clients bought the new product (Dezenas de clientes compraram o novo produto).
E, por falar em palavras no singular e no plural, vejamos o que fazer para que os substantivos
sejam flexionados de forma correta. Em Português, a maioria das palavras apenas precisam do
acréscimo da letra -s no final delas.
Em Inglês, há outras possibilidades e regras a seguir, de fato.
Plural de palavras terminadas em -ch, -sh, -s, -x, -z e -o, usa-se -es no final delas, veja: church
= churches (igrejas), dish = dishes (pratos), kiss = kisses (beijos), box = boxes (caixas) e tomato =
tomatoes (tomates).
E, para substantivos terminados em -f ou -fe, essas terminações são substituídas por -ves:
shelf = shelves (prateleiras), leaf = leaves (folhas), thief = thieves (ladrões) e wolf = wolves, entre
outras.
Substantivos terminados em -y, colocamos -ies no final das palavras e retiramos o -y que
estava lá como por exemplo family = families e baby = babies.
Em relação a plural irregular, devido a serem palavras que sofrem grandes alterações quando
flexionadas, o melhor jeito de aprender é fazendo exercícios, leituras de variados textos em Inglês.
É a prática que ensina verdadeiramente. E, quando estudados com atenção, fica fácil
memorizar.
O substantivo pé, que é foot em Inglês, muda para feet, com letras e no lugar de o.
Dizer pessoa, em Inglês, é person, mas pessoas, é people. Até aceita-se dizer persons para
duas pessoas, mas quando se fala de muitas pessoas, mais de três, um grupo de pessoas, é people.
Os substantivos person/people merecem um pouco a mais de atenção pois people, pode
significar também povo, no sentido demográfico e pode significar gente, como eu disse
anteriormente, pessoas.
Assim, o plural de people no sentido de povo tem acréscimo de -s:
The South people enjoys barbecue. (O povo do Sul curte gosta de churrasco).
Como a palavra people está no singular, o verbo to enjoy, correspondente no Present Simple,
vai fazer a concordância, sendo acrescentado de -s para a terceira pessoa – o povo (ele) gosta.
Agora, nessa frase:
Some Indian peoples have different cultural habits – Alguns povos indianos têm costumes
culturais estranhos, o verbo está fazendo a concordância com o plural peoples.
E ainda existem algumas palavras que tem letra -s no final, mas estão no singular, sendo iguais
quando usadas no plural, como por exemplo headphones (fones de ouvido) e pants (calças).

Aula 03 – Articles and Nouns 19


www.estrategiamilitar.com.br 61
Teacher Andrea Belo
Aula 03: Escola Naval 2021

Há alguns substantivos que têm origem grega ou latina e aparecem com frequência em textos
científicos em Inglês e são diferentes, portanto, merecem atenção.
Veja alguns deles a seguir, que podem estar presentes em uma das questões de sua prova.
São eles:
✓ alumnus - alumni (ex-alunos, alunos graduados),
✓ axis - axes (eixos),
✓ bacillus - bacilli (bacilos),
✓ bacterium - bacteria (bactérias),
✓ cactus - cacti (cactos),
✓ corpus - corpora (corpora),
✓ criterion - criteria (critérios),
✓ datum - data (dados),
✓ encyclopedia - encyclopedae (enciclopédias),
✓ erratum - errata (erratas),
✓ formula - formulae ou formulas (fórmulas),
✓ fungus - fungi ou funguses (fungos),
✓ medium - media (meios de comunicação),
✓ nucleus - nuclei (núcleos),
✓ phenomenon - phenomena (fenômenos),
✓ stimulus - stimuli (estímulos),
✓ stratum - strata (estratos),
✓ vertebra - vertebrae (vértebras)

Algumas gramáticas ainda apresentam uma categoria especial para classificar o plural com o
nome de parelhas. As parelhas são a demonstração do plural de alguns substantivos que têm duas
partes iguais e uma possível tradução para eles seria “um par de”, como óculos (glasses).
Mas podemos simplesmente chamar de plural irregular, como veremos uma lista adiante,
que preparei como complemento dos seus estudos em relação a esse assunto.
Essas palavras só existem no plural, não possuem singular, apesar de que, em Português,
podemos usá-las no singular, como:
Onde estão os meus óculos de sol? – There are my sunglasses? - E nunca “Where is my
sunglasses?”. Haverá outros exemplos adiante para melhor compreensão.
Não podemos nos esquecer também do que chamamos de falsos plurais. Alguns livros e
estudiosos linguistas afirmam que existem plurais nomeados falsos porque são idênticos na escrita
tanto quando são usados no singular quanto no plural.
Os exemplos de plurais falsos são os nomes de disciplinas, de ciências e outros substantivos
terminados em -ics, que são singulares:
Politics is a complicated thing to talk about – Política é uma coisa complicada de se falar sobre.

Aula 03 – Articles and Nouns 20


www.estrategiamilitar.com.br 61
Teacher Andrea Belo
Aula 03: Escola Naval 2021

Temos como outros exemplos, além de politics, acoustics (acústica), athletics (atletismo),
electronics (eletrônica), genetics (genética), linguistics (linguística), mathematics (matemática),
physics (física) e statistics (estatística):
General Statistics can show you the real results of our project. – A estatística geral pode
mostrar a você os resultados reais do nosso projeto.
Mas, há uma importante observação a ser feita: se, por acaso, essas palavras forem usadas
com outro sentido, o verbo que as acompanha poderá ir para o plural, como: “What are your
politics?” - Quais são suas políticas? E “Your family genetics prove the similarities among your
relatives.” (As genéticas de sua família provam as semelhanças entre os seus parentes.
Vou apresentar uma lista, de possíveis palavras que possuem plurais irregulares, das variadas
exceções acima e que podem aparecer nos textos das provas.

IRREGULAR PLURAL NOUNS LIST


(LISTA DE SUBSTANTIVOS COM PLURAL IRREGULAR)

alga = algae (algas marinhas) life = lives (vidas)


analysis = analyses (análises) man = men (homens)
appendix = appendices (indices) oasis = oases (oásis)
basis = bases (base) oats = oats (aveia)
binoculars = binoculars (binóculos) ox = oxen (bois)
bus = buses (ônibus) parenthesis = parentheses (parênteses)
child = children (crianças) salmon = salmon (salmões)
clothes = clothes (roupas) scales = scales (balanças)
crisis = crises (crises) scarf = scarves (lenços, cachecóis)
diagnosis = diagnoses (diagnósticos) scissors = scissors (tesouras)
dwarf = dwarves (anões) series = séries (series de TV)
elk = elk (alces) sheep = sheep (ovelhas)
fish = fish (peixes) synthesis = syntheses (sínteses)
foot = feet (pés) synopsis = synopses (sinopses)
gentleman = gentleman (cavalheiros) surroundings = surroundings (arredores)
glasses = glasses (óculos) species = species (espécie)
half = halves (metades) stairs = stairs (escadas)
hypothesis = hypotheses (hipóteses) thanks = thanks (obrigado(a)
jeans = jeans (calças jeans) thesis = theses (teses)
knife = knives (facas) tooth = teeth (dentes)
matrix = matrices/matrixes (matrizes) trout = trout (trutas)
mouse = mice (ratos) woman = women (mulheres)

Aula 03 – Articles and Nouns 21


www.estrategiamilitar.com.br 61
Teacher Andrea Belo
Aula 03: Escola Naval 2021

Nouns examples
Exemplos de substantivos

Agora, para complementar as explicações e exemplos, vamos falar um pouco das inúmeras
curiosidades que envolvem os artigos e os substantivos.

Aula 03 – Articles and Nouns 22


www.estrategiamilitar.com.br 61
Teacher Andrea Belo
Aula 03: Escola Naval 2021

Curiosidades sobre artigos


Mostrarei a você algumas curiosidades divertidas em Inglês e ao mesmo tempo, pertinentes.
Além de serem fatos interessantes que podem fazer parte de alguma pergunta ou texto da sua
prova, envolvem o artigo definido the, estudado nessa aula.

Você por acaso sabe o que é pangrama? Tem alguma ideia? Bom, a palavra pangrama tem
origem grega, sendo que pan significa todos, grama significa letra e, um pangrama é uma frase em
que são usadas todas as letras do alfabeto de uma língua, mas essa frase precisa ter sentido
completo.
Os pangramas foram inventados na mesma época em que surgiu a tipografia, com o propósito
de avaliar os efeitos visuais de uma fonte, como uma forma de exercício. A criação de pangramas é
um passatempo divertido, que demanda criatividade e conhecimento da língua.

Se um pangrama deve usar todas as letras com o mínimo de palavras, obviamente, a


combinação de palavras precisa de artigos para conectá-las. E é isso mesmo.

O primeiro pangrama que existiu foi: The quick brown fox jumps over the lazy dog, traduzido:
A rápida raposa marrom pula por cima do cão preguiçoso. Perceba que, na sentença que gerou o
pangrama, a conecção de palavras foi feita com o uso do artigo definido the duas vezes, no início e
quase no final da frase: The quick brown fox jumps over the lazy dog.

Em um artigo chamado Interesting Notes no ano de 1885, essa frase foi uma sugestão da
companhia The Western Union, testando o sistema de telégrafo com transmissões da mensagem
The quick brown fox jumps over the lazy dog. Essa frase aparece, inclusive, como exemplo em alguns
programas da Microsoft.

Aula 03 – Articles and Nouns 23


www.estrategiamilitar.com.br 61
Teacher Andrea Belo
Aula 03: Escola Naval 2021

Em Português, temos pangramas também, veja: “Jane quer LP, fax, CD, giz, TV e bom whisky”,
em que estão presentes todas as 26 letras do nosso alfabeto em uma só frase e, com sentido. Legal,
não é? Você consegue elaborar algum pangrama?

Quer saber outra curiosidade sobre os artigos?


Vamos lá! Foi descoberto que, cerca de 11% de toda a língua inglesa é composta
exclusivamente pela letra “e”. Isso mesmo, a letra “e” está presente em milhares de palavras em
Inglês. Dizem que, uma, em cada oito letras que você vê em uma palavra escrita em Inglês, vai
possuir a letra “e”.

E eu te pergunto, há coincidência com alguma letra que possui no artigo definido que
estudamos?

Então, de todas as palavras vistas em frases onde encontramos a letra “e”, em Inglês, mais de 70%
possuem o artigo THE em sua construção, tanto para conectar substantivos e verbos como também
para dar sentido entre o sujeito, predicado e outros elementos da oração.

Veja, na imagem a seguir, exemplos de palavras usando todas as letras do alfabeto, de A a Z


em que TODAS usam as palavras realmente possuem letras E, conforme a curiosidade:

APPLE BELL CROCODILE DAUGHTER ELEPHANT

e e
e
FIREFIGHTER GIRAFFE HELICOPTER ICE-CREAM

JELLY LEMON

e
e ENGLISH

MONKEY ORANGE PEAR

QUEEN RED SEE


e
TABLE
e e
UMBRELLA VEGETABLES WAVE XYLOPHONE YELLOW ZEBRA

Aula 03 – Articles and Nouns 24


www.estrategiamilitar.com.br 61
Teacher Andrea Belo
Aula 03: Escola Naval 2021

Curiosidades sobre substantivos


Agora, vamos à algumas curiosidades sobre os substantivos, que também podem aparecer
nas provas, retiradas de fontes que publicam fatos curiosos e pesquisas, como as revistas Time,
Forbes e Newsweek e os jornais The Telegram e The Guardian.
Você sabe qual é a palavra mais antiga que existe em Inglês e que ainda é usada até hoje? É
o substantivo town, que significa cidade. Apesar de existir a palavra city, que também significa
cidade, town é menor do que city.
Ao se referir ao termo city, deve-se pensar em uma área mais populosa, já que antigamente,
só havia towns, menores, como se fossem municípios hoje.
Outra curiosidade: além da palavra mais antiga em Inglês ser um substantivo, a palavra maior
que existe a língua inglesa também é um substantivo. Sabe qual é? É o nome de uma doença
pulmonar, que se adquire com a inalação de cinza e pó de carvão. É difícil de escrever e de falar.
Considera-se que é um desafio conseguir pronunciá-la. Lá vai: a maior palavra da língua inglesa
é pneumonoultramicroscopicsilicovolcanoconiosis.

Mais uma curiosidade: você sabia que William Shakespeare, além de contribuir com a língua
inglesa em suas obras escritas, ele adorava inventar palavras?
Não existe uma lista para comprovar quantas palavras para saber o número exato de
vocábulos que ele inventou.
Mas, é comprovado que, quase todas, são substantivos comuns, próprios, compostos,
abstratos, entre outros. Por exemplo, birthplace (local de nascimento), torture (tortura) e bubble
(bolha) foram criadas por Shakespeare.

Aula 03 – Articles and Nouns 25


www.estrategiamilitar.com.br 61
Teacher Andrea Belo
Aula 03: Escola Naval 2021

No balão, outros exemplos de palavras inventadas por Shakespeare: elbow (cotovelo), gossip
(fofoca), bedroom (quarto), lonely (solitário), eyeball (globo ocular), entre muitas outras.

E a melhor curiosidade de todas: quantas palavras você acha que precisa saber para falar
Inglês fluentemente? Se você acha que precisa saber todas as palavras que existem no dicionário ou,
no mínimo, 90% delas para falar fluente esse idioma, você está enganado.

É comprovado o fato que, são necessárias apenas 2.000 palavras para se comunicar bem em
Inglês. Pense bem: o Dicionário Oxford, um dos maiores dicionários da língua inglesa, tem mais de
200 mil verbetes.

Isso mesmo, duzentas mil palavras das quais você precisa saber duas mil para adquirir
fluência. E, dessas 2 mil, mais da metade são substantivos.

= 200.000 words (200.000 palavras)


* You need 2.000 to communicate
(Você precisa de 2 mil para se comunicar)

Muitos estudiosos linguistas dizem que a produção oral não precisa de um vocabulário muito
extenso e isso significa que o seu vocabulário ativo (o que você usa para falar). Então, 80% das
palavras que você realmente vai precisar, se resumem em 2000 palavras em Inglês.

Claro que se você decorar 2000 palavras não vai “sair por aí” falando fluente. Será necessário
fazer combinações de substantivos, verbos e outros termos integrantes das orações. Mas a notícia
é boa, não é?

Gostou das curiosidades? Percebemos que, a língua inglesa, assim como qualquer outra
língua, sem os substantivos ou verbos, não é possível de ser falada ou escrita.

E você já aprendeu todos os tempos verbais e agora, os artigos e os substantivos, tão


importantes na hora da prova, na leitura dos textos das provas.

Aula 03 – Articles and Nouns 26


www.estrategiamilitar.com.br 61
Teacher Andrea Belo
Aula 03: Escola Naval 2021

Questões inéditas

Agora questões inéditas, similar às questões da prova, com um artigo da revista Time,
usada algumas vezes para elaborar questões da prova. O artigo completo foi publicado em
maio de 2019 mas vamos usar apenas um trecho para análise.
Aqui, exige-se que seja feita a interpretação, conectando substantivos, artigos, verbos etc,
com o domínio de muitos tópicos gramaticais, os quais estamos estudando no decorrer das
nossas aulas.

Questão Inédita 1 – Teacher Andrea Belo

http://time.com/5584275/oculus-quest-review

Aula 03 – Articles and Nouns 27


www.estrategiamilitar.com.br 61
Teacher Andrea Belo
Aula 03: Escola Naval 2021

Questão inédita 1 – (2019) – De acordo com o texto, sobre Oculus Quest, afirma-se que:
A ( ) As experiencias de realidade virtual com o Oculus Quest não dependem de nenhum
hardware adicional, sendo apenas necessário ligar o equipamento na TV.
B ( ) As experiencias de realidade virtual com o Oculus Quest fazem com que as pessoas
dependam desse hardware adicional em suas salas de TV.
C ( ) A realidade virtual é quase impossível com o Oculus Quest por causa do preço alto.
D ( ) As tentativas de realidade virtual com o Oculus Quest custaram muitos dólares.
E ( ) As experiencias de realidade virtual com o Oculus Quest estão na terceira tentativa, após
duas tentativas anteriores.
Comentários:
No enunciado, pede-se o que é correto afirmar sobre o Oculus Quest. É necessário que você
leia as fontes usadas nas variadas provas, como a revista Time, aqui usada. Assim, você já
saberia que, o Oculus Quest é o primeiro sistema de jogo integrado para realidade virtual (RV),
em que se pode jogar em qualquer lugar somente com controladores e um headset de RV.
Na letra A, é incorreto dizer que o Oculus Quest não depende do hardware porque, na linha 3,
o substantivo equipment (equipamento) mostra que o equipamento é dependente de um
hardware adicional.
Na letra B, afirma-se que experiências com o Oculus Quest fazem com que as pessoas
dependam do hardware. Mas, como vimos na alternativa “A”, o substantivo equipment
(equipamento) mostra que o equipamento é dependente de um hardware adicional e não que
as pessoas ficam dependentes.
A letra C é falsa pois, o preço alto é devido à alta tecnologia do equipamento e, onde tem a
palavra impossible, é sobre o hardware do qual dependem e fica impossível pegar só o fone e
começar a jogar como diz exatamente essa frase: ...makes it nearly impossible to just grab a
headset and start playing.
Na letra D, a afirmação de que as tentativas custaram caro é falsa pois, onde há o substantivo
numeral $399, é o preço do equipamento do qual se fala no artigo e não há os valores
anteriores.
A letra E é verdadeira, como podemos comprovar no início do segundo parágrafo, através do
substantivo third, que é a terceira versão: third major attempt e há uma primeira e segunda
versões citadas com outros substantivos, numerais primeiro e segundo em Inglês: Its first, the
polarizing Oculus Rift, brought... e Its second, the smaller, self-contained Oculus Go.
Vejamos outra questão inédita, com o mesmo texto da Time, porém, com alternativas
escritas em Inglês, como às vezes aparece nas provas, também exigindo a interpretação e
gramática.
Então, desde o enunciado, você já deve procurar as palavras cognatas e começar a conectar
ideias para entender o texto e as opções de escolha.

Aula 03 – Articles and Nouns 28


www.estrategiamilitar.com.br 61
Teacher Andrea Belo
Aula 03: Escola Naval 2021

Questão Inédita 2 – Teacher Andrea Belo

http://time.com/5584275/oculus-quest-review

Questão inédita 2 – (2019) – According to the text, it is INCORRECT about Oculus Quest:
A ( ) Oculus Quest virtual reality experiences are different from everything.
B ( ) To use Oculus Quest virtual reality, an additional hardware is necessary.
C ( ) The first version of this equipment was called Oculus Rift and limited degrees of freedom.
D ( ) The second version of this equipment, called Oculus Go, limited degrees of freedom.
E ( ) Oculus Quest costs $399 and for 128GB version, the price is $499.

Aula 03 – Articles and Nouns 29


www.estrategiamilitar.com.br 61
Teacher Andrea Belo
Aula 03: Escola Naval 2021

Comentários:
No enunciado, pede-se o que é incorreto sobre o Oculus Quest.
Na letra A, está certo dizer que as experiências com Oculus Quest são diferentes de tudo, como
propões a alternativa: Oculus Quest virtual reality experiences are different from everything, já
que a palavra unlike, é sinônimo de diferente, significa distinto/dessemelhante.
Na letra B, afirma-se que precisa do hardware adicional. Como há o substantivo experience e
o adjetivo dependant (você aprenderá sobre adjetivos em breve), que ambas são palavras
cognatas, experience e dependant – experiência e dependente, mostra que o equipamento é
dependente de um hardware adicional. Está certo também.
A letra C é falsa pois, quem teve graus limitados foi a segunda versão e não a primeira, como
vemos em: Its second, the smaller(...) but its single controller, limited degrees of freedom e é a
alternativa solicitada no enunciado, letra “C”, a incorreta.
Na letra D, a afirmação de que a segunda versão teve graus limitados está correta, como vimos
na alternativa anterior: Its second, the smaller(...) but its single controller, limited degrees of
freedom.
Na letra E, afirma-se os preços exatos mostrados no texto: Oculus Quest costs $399 and for
128GB version, the price is $499, é isso mesmo: $399 e $499 para versão 128GB. Está certo
também.
Algumas questões da prova são elaboradas em Inglês, desde o enunciado e terem suas
alternativas também em Inglês, exigem maior atenção ao vocabulário, aos substantivos que
você conhece, palavras que você está aprendendo dia após dia.
Agora você já estudou todos os tempos verbais, artigos, substantivos e, claro, métodos,
técnicas de leitura que ajudam muito a compreender os textos.
Você, daqui para frente, deve treinar, praticar exercícios e manter os estudos em dia.
Vamos resolver agora, questões de anos anteriores e depois que você treinar, verifique as
respostas comentadas para aprimorar seus conhecimentos.
Vamos praticar!

Aula 03 – Articles and Nouns 30


www.estrategiamilitar.com.br 61
Teacher Andrea Belo
Aula 03: Escola Naval 2021

Questões de anos anteriores


Você vai, agora, responder questões selecionadas de provas já realizadas em anos anteriores.
Depois, como em todas as nossas aulas, haverá o gabarito e as questões comentadas.
Vamos começar com questões da Escola Naval, de acordo coma sua instituição escolhida e
depois, vamos treinar de outras Carreiras Militares, para adquirir experiência e treinar vocabulário.

Questão 01 (Escola Naval/2018)


In the sentence "It helps lower stress, which weakens the immune system.” (2nd paragraph),
the underlined words mean, respectively,______and______.
a) raise / lessens
b) rise / strengthens
c) reduce / debilitates
d) eliminate / toughens
e) decrease / reproduces

Questão 02 (Colégio Naval/2018)

Aula 03 – Articles and Nouns 31


www.estrategiamilitar.com.br 61
Teacher Andrea Belo
Aula 03: Escola Naval 2021

Questão 02 (Colégio Naval/2018) All the underlined words in text I are adjectives, EXCEPT:
(A) social.
(B) global.
(C) long.
(D) executive.
(E) former.

Questão 03 – (EAM/2019)

Aula 03 – Articles and Nouns 32


www.estrategiamilitar.com.br 61
Teacher Andrea Belo
Aula 03: Escola Naval 2021

Questão 03 – (EAM/2019) It is FALSE to say that


(A) Nessie looked like a dragon, according to George Spicer.
(B) The BBC TV network did a search in Loch Ness.
(C) “Loch" means “lake", in Scottish Gaelic.
(D) Edward Mountain’s expedition found the Monster.
(E) Robert Williams’ photo was a fraud.

Questão 04– (EEAR/2018)


Homeless crack addict revitalizes small square in downtown São Paulo
A homeless man has chosen to occupy his free time revitalizing a small square on the corner of
avenues São João and Duque de Caxias, in downtown São Paulo. He planted pau-brasil, palm,
banana and avocado trees. He also planted boldo, sweet potatoes, beans, peppers and
ornamental plants, such as snake plants. Residents noticed the square’s gradual changes and
congratulated the author for the modifications.
Fonte: Folha de São Paulo Internacional – 21/03/2017
It is FALSE to say that
The words “small”, “sweet” e “ornamental”, underlined in the text, are __________.
a) nouns
b) adverbs
c) pronouns
d) adjectives

Questão 05 – (Questão inédita Militares)


Read the dialogue and mark the right option to fill in the gaps respectively.
A: Good afternoon, Jack!
B: Hey, Jen! How are you doing?
A: I’m alright. What about yourself?
B: I’m doing well.
A: _________ you at Johnnie’s house last weekend?
B: Yes, I ________.
A: Who ________ you go with?
B: I ________ with my wife. What about you? Were you there?
A: No, I ________. I _______ home.

Aula 03 – Articles and Nouns 33


www.estrategiamilitar.com.br 61
Teacher Andrea Belo
Aula 03: Escola Naval 2021

A) Did / sis / went / was / weren’t / was


B) Were / was / were / go / didn’t / went
C) Was / was / did / was / weren’t / was
D) Were / was / did / went / wasn’t / stayed
E) Did / did / did / went / didn’t / went

A: Good afternoon, Jack!


B: Hey, Jen! How are you doing?
A: I’m alright. What about yourself?
B: I’m doing well.

A: ___Were______ you at Johnnie’s house last weekend?


B: Yes, I ___was_____.
A: Who ___did_____ you go with?
B: I ___went_____ with my wife. What about you? Were you there?
A: No, I ___wasn’t_____. I ___stayed____ home.

Questão 06 – (Questão inédita Militares)


Read the dialogue and mark the right option
Read the sentences and mark the correct option to fill in the blanks respectively.
Elisa is ______ friend. ________ lives near my grandfather’s house. We love riding our bikes.
_______ bike is big. ________ is small. We love spending time together!
A) My / she / my / hers
B) Your / I / my / her
C) Her / he / her / mine
D) My / her / my / she
E) His / he / my / his
Elisa is __my____ friend. ___she_____ lives near my grandfather’s house. We love riding our
bikes. ___my____ bike is big. ___hers_____ is small. We love spending time together!

Aula 03 – Articles and Nouns 34


www.estrategiamilitar.com.br 61
Teacher Andrea Belo
Aula 03: Escola Naval 2021

Questão 07 – (Questão inédita Militares)


Read the dialogue and mark the right option
Which word is incorrectly used in: “You failed, my ability in this test is unpossible”
A) Failed
B) Unpossible
C) My
D) In
E) Test

Questão 08 – (Questão inédita Militares)


Read the dialogue and mark the right option
Use the verbs in the parentheses to complete the following statements.
I- I usually ______ (do) my homework after dinner, but now I ______ (do) a diferent task.
II- Eli _______ (eat) a lot. Sometimes, he _______ (want) to eat less
III- Alessandra _________(live) in Boston, United States. During the week, her daughters
_______ (work) as delivery girls for a big tech company.
Now mark the option which completes them respectively.

A) Am doing / am doing / is eating / wants / lives / working


B) Am doing / do / eats / is wanting / is living / works
C) Am doing / do / eat / is wanting / is living / works
D) Do / do / is eating / want / live / is working
E) Do / am doing / eats / wants / lives / work

I- I usually __do____ (do) my homework after dinner, but now I _am doing___ (do) a diferent
task.
II- Eli _eats___ (eat) a lot. Sometimes, he __wants___ (want) to eat less
III- Alessandra __lives____(live) in Boston, United States. During the week, her daughters
__work___ (work) as delivery girls for a big tech company.

Aula 03 – Articles and Nouns 35


www.estrategiamilitar.com.br 61
Teacher Andrea Belo
Aula 03: Escola Naval 2021

Texto para questão 09

AFA/2017
TEXT: Howard Gardner: ‘Multiple intelligences’ are not ‘learning styles’ by Valerie Strauss

The fields of psychology and education were revolutionized 30 years ago when we now
worldrenowned psychologist Howard Gardner published his 1983 book Frames of Mind: The
Theory of Multiple Intelligences, which detailed a new model of human intelligence that went
beyond the traditional view that there was a single kind that could be measured by standardized
tests.
Gardner’s theory initially listed seven intelligences which work together: linguistic, logical-
mathematical, musical, bodily-kinesthetic, interpersonal and intrapersonal; he later added an
eighth, naturalist intelligence and says there may be a few more. The theory became highly popular
with K-12¹ educators around the world seeking ways to reach students who did not respond to
traditional approaches, but over time, ‘multiple intelligences’ somehow became synonymous with
the concept of ‘learning styles’. In this important post, Gardner explains why the former is not the
latter.
It’s been 30 years since I developed the notion of ‘multiple intelligences’. I have been gratified by
the interest shown in this idea and the ways it’s been used in schools, museums, and business
around the world. But one unanticipated consequence has driven me to distraction and that’s the
tendency of many people, including persons whom I cherish, to credit me with the notion of
‘learning styles’ or to collapse ‘multiple intelligences’ with ‘learning styles’. It’s high time to relieve
my pain and to set the record straight.
First a word about ‘MI theory’. On the basis of research in several disciplines, including the study
of how human capacities are represented in the brain, I developed the idea that each of us has a
number of relatively independent mental faculties, which can be termed our ‘multiple
intelligences’. The basic idea is simplicity itself. A belief in a single intelligence assumes that we
have one central, all-purpose computer, and it determines how well we perform in every sector of
life. In contrast, a belief in multiple intelligences assumes that human beings have 7 to 10 distinct
intelligences.
Even before I spoke and wrote about ‘MI’, the term ‘learning styles’ was being bandied about in
educational circles. The idea, reasonable enough on the surface, is that all children (indeed all of
us) have distinctive minds and personalities. Accordingly, it makes sense to find out about learners
and to teach and nurture them in ways that are appropriate, that they value, and above all, are
effective.

Aula 03 – Articles and Nouns 36


www.estrategiamilitar.com.br 61
Teacher Andrea Belo
Aula 03: Escola Naval 2021

Two problems: first, the notion of ‘learning styles’ is itself not coherent. Those who use this term
do not define the criteria for a style, nor where styles come from, how they are recognized/
assessed/ exploited. Say that Johnny is said to have a learning style that is ‘impulsive’. Does that
mean that Johnny is ‘impulsive’ about everything? How do we know this? What does this imply
about teaching? Should we teach ‘impulsively’, or should we compensate by ‘teaching reflectively’?
What of learning style is ‘right-brained’ or visual or tactile? Same issues apply.
Problem #2: when researchers have tried to identify learning styles, teach consistently with those
styles, and examine outcomes, there is not persuasive evidence that the learning style analysis
produces more effective outcomes than a ‘one size fits all approach’. Of course, the learning style
analysis might have been inadequate. Or even if it is on the mark, the fact that one intervention
did not work does not mean that the concept of learning styles is fatally imperfect; another
intervention might have proved effective. Absence of evidence does not prove non-existence of a
phenomenon; it signals to educational researchers: ‘back to the drawing boards’.
Here’s my considered judgment about the best way to analyze this lexical terrain: Intelligence: We
all have the multiple intelligences. But we signed out, as a strong intelligence, an area where the
person has considerable computational power. Style or learning style: A hypothesis of how an
individual approach the range of materials. If an individual has a ‘reflective style’, he/she is
hypothesized to be reflective about the full range of materials. We cannot assume that
reflectiveness in writing necessarily signals reflectiveness in one’s interaction with the others.
Senses: Sometimes people speak about a ‘visual’ learner or an ‘auditory’ learner. The implication
is that some people learn through their eyes, others through their ears. This notion is incoherent.
Both spatial information and reading occur with the eyes, but they make use of entirely different
cognitive faculties. What matters is the power of the mental computer, the intelligence that acts
upon that sensory information once picked up.
These distinctions are consequential. If people want to talk about ‘an impulsive style’ or a ‘visual
learner’, that’s their prerogative. But they should recognize that these labels may be unhelpful, at
best, and ill-conceived at worst.
In contrast, there is strong evidence that human beings have a range of intelligences and that
strength (or weakness) in one intelligence does not predict strength (or weakness) in any other
intelligences. All of us exhibit jagged profiles of intelligences. There are common sense ways of
assessing our own intelligences, and even if it seems appropriate, we can take a more formal test
battery. And then, as teachers, parents, or self assessors, we can decide how best to make use of
this information. (Adapted from https://www.washingtonpost.com/news/answer-sheet)

Questão 09 (AFA/2017) - In the sentence “it’s been 30 years since I developed the notion of
‘multiple intelligences’” (lines 20 and 21), the contraction refers to
a) It has.
b) It been.
c) It is.
d) It was.

Aula 03 – Articles and Nouns 37


www.estrategiamilitar.com.br 61
Teacher Andrea Belo
Aula 03: Escola Naval 2021

Questão 10 (EPCAR/2019) - “Slavery continues today and harms people in every country in the
world” (lines 2 and 3). The highlighted words can be substituted for _____.

a) them
b) theirs
c) their
d) they

Gabarito
1. C 2. D
3. B 4. D
5. D 6. A
7. B 8. E
9. A 10. A

Aula 03 – Articles and Nouns 38


www.estrategiamilitar.com.br 61
Teacher Andrea Belo
Aula 03: Escola Naval 2021

Respostas comentadas

Questões Comentadas

Questão 01 (Escola Naval/2018)


In the sentence "It helps lower stress, which weakens the immune system.” (2nd paragraph),
the underlined words mean, respectively,______and______.
a) raise / lessens
b) rise / strengthens
c) reduce / debilitates
d) eliminate / toughens
e) decrease / reproduces

Comentários:
O contexto nos mostra claramente que “lower” dá a ideia de diminuição e “weakens” dá a ideia
de enfraquecimento.
A alternativa A está incorreta. “Raise” significa aumentar/crescer e “lessens” significa diminuir.
A alternativa B está incorreta. “Rise” significa levantar/aumentar e “strengthens” significa
reforçar/fortalecer.
A alternativa C está correta. “Reduce” significa reduzir/diminuir e “debilitates” significa
debilitar/enfraquecer.
A alternativa D está incorreta. “Eliminate” significa eliminar e “toughens” significa
endurecer/dificultar.
A alternativa E está incorreta. “Decrease” significa diminuir e “reproduces” significa reproduzir.

Aula 03 – Articles and Nouns 39


www.estrategiamilitar.com.br 61
Teacher Andrea Belo
Aula 03: Escola Naval 2021

Questão 02 (Colégio Naval/2018)

Questão 02 (Colégio Naval/2018) All the underlined words in text I are adjectives, EXCEPT:
(A) social.
(B) global.
(C) long.
(D) executive.
(E) former.

Comentários:
Teremos nossa aula de adjetivos, em que vamos trabalhar a função deles, assim como os
substantivos nessa aula de hoje. Vamos à análise das alternativas:
A alternativa A está errada. “Social” funciona como adjetivo caracterizando o termo “media”.

Aula 03 – Articles and Nouns 40


www.estrategiamilitar.com.br 61
Teacher Andrea Belo
Aula 03: Escola Naval 2021

A alternativa B está errada. “Global” funciona como adjetivo caracterizando o termo


“problem”.
A alternativa C está errada. “Long” funciona como adjetivo caracterizando o termo “break”.
A alternativa D está correta. “Executive” funciona como substantivo no trecho (o executivo).
A alternativa E está errada. “Former” funciona como adjetivo caracterizando o termo
“executive”.

Questão 03 – (EAM/2019)

Questão 03 – (EAM/2019) It is FALSE to say that


(A) Nessie looked like a dragon, according to George Spicer.
(B) The BBC TV network did a search in Loch Ness.
(C) “Loch" means “lake", in Scottish Gaelic.
(D) Edward Mountain’s expedition found the Monster.
(E) Robert Williams’ photo was a fraud.

Aula 03 – Articles and Nouns 41


www.estrategiamilitar.com.br 61
Teacher Andrea Belo
Aula 03: Escola Naval 2021

Comentários:
A alternativa A está correta. Há a presença do substantivo “dragon” (dragão). George Spicer
descreveu Nessie como um dragão. “George Spicer described that he saw Nessie and it was a
‘dragon’”.
A alternativa B está correta. A BBC TV fez uma busca detalhada no lago em 2003 e nada foi
encontrado. Eles concluíram que a história era um mito. “In 2003, the BBC TV network made a
show that did a detailed search of Loch Ness. They found nothing and concluded that the
Monster was a myth”.
A alternativa C está correta. “Loch Ness is a lake (or ‘loch’ in Scottish Gaelic)”. Este trecho diz
que ‘loch’ significa “lake” (lago) em gaélico escocês.
A alternativa D está incorreta. “In 1934, Edward Mountain sent an expedition to Loch Ness
from 9 am to 6 pm every day, for 5 weeks. They never found any evidence of the Monster”.
Este trecho diz que Edward Mountain enviou uma expedição de 5 semanas para encontrar o
monstro. O monstro, porém, nunca foi encontrado. A alternativa diz exatamente o contrário
disso.
A alternativa E está correta. O texto diz que a foto foi considerada fraude por volta de 1994.
“the photo was declared to be a hoax”. A palavra “hoax” é sinônima de fraude.

Questão 04– (EEAR/2018) It is FALSE to say that


Text
Homeless crack addict revitalizes small square in downtown São Paulo
A homeless man has chosen to occupy his free time revitalizing a small square on the corner of
avenues São João and Duque de Caxias, in downtown São Paulo. He planted pau-brasil, palm,
banana and avocado trees. He also planted boldo, sweet potatoes, beans, peppers and
ornamental plants, such as snake plants. Residents noticed the square’s gradual changes and
congratulated the author for the modifications.
Fonte: Folha de São Paulo Internacional – 21/03/2017

Questão 04 – (EEAR/2018) It is FALSE to say that


The words “small”, “sweet” e “ornamental”, underlined in the text, are __________.
a) nouns
b) adverbs
c) pronouns

Aula 03 – Articles and Nouns 42


www.estrategiamilitar.com.br 61
Teacher Andrea Belo
Aula 03: Escola Naval 2021

d) adjectives

Comentários:
A afirmativa A está incorreta. “Nouns” são os substantivos, e substantivos são palavras que
designam coisas, pessoas, grupos, lugares. As palavras “small”, “sweet” e “ornamental” estão
caracterizando as palavras “square”, “potatoes” e “plants” no texto, respectivamente.
A afirmativa B está incorreta. “Adverbs” são os advérbios, e advérbios são palavras que
designam ideias de modo, tempo, lugar, dúvida, negação, intensidade, afirmação. As palavras
“small”, “sweet” e “ornamental” estão caracterizando as palavras “square”, “potatoes” e
“plants” no texto, respectivamente.
A afirmativa C está incorreta. “Pronouns” são os pronomes, e pronomes são palavras que
substituem, referem-se ou acompanham um nome (substantivo). As palavras “small”, “sweet”
e “ornamental” estão caracterizando as palavras “square”, “potatoes” e “plants” no texto,
respectivamente. Os pronomes se relacionam com os substantivos. Portanto você poderia
pensar que as palavras sublinhadas se encaixam nessa descrição. Mas essas palavras estão
caracterizando os nomes, não substituindo, fazendo referência ou acompanhando-os.
A alternativa D está correta. As palavras sublinhadas são adjetivos porque acompanham o
substantivo, caracterizando-o. Dessa forma “small” (pequeno/a) caracteriza “square” (praça),
“sweet” (doce) caracteriza “potatoes” (batatas) e “ornamental” (ornamental/ornamentais)
caracteriza “plants” (plantas).

Questão 05 – (Questão inédita Militares)


Read the dialogue and mark the right option to fill in the gaps respectively.
A: Good afternoon, Jack!
B: Hey, Jen! How are you doing?
A: I’m alright. What about yourself?
B: I’m doing well.

A: _________ you at Johnnie’s house last weekend?


B: Yes, I ________.
A: Who ________ you go with?
B: I ________ with my wife. What about you? Were you there?
A: No, I ________. I _______ home.

Aula 03 – Articles and Nouns 43


www.estrategiamilitar.com.br 61
Teacher Andrea Belo
Aula 03: Escola Naval 2021

A) Did / sis / went / was / weren’t / was


B) Were / was / were / go / didn’t / went
C) Was / was / did / was / weren’t / was
D) Were / was / did / went / wasn’t / stayed
E) Did / did / did / went / didn’t / went

A: Good afternoon, Jack!


B: Hey, Jen! How are you doing?
A: I’m alright. What about yourself?
B: I’m doing well.
A: ___Were______ you at Johnnie’s house last weekend?
B: Yes, I ___was_____.
A: Who ___did_____ you go with?
B: I ___went_____ with my wife. What about you? Were you there?
A: No, I ___wasn’t_____. I ___stayed____ home.

Comentários:
Na primeira lacuna, precisamos usar uma palavra no passado. A expressão last weekend nos
dá essa certeza. A única palavra possível é “were”, pois a pergunta tem como objetivo saber se
Jack foi à casa de Johnnie no fim de semana anterior.
Na segunda lacuna, não poderíamos usar “were” por causa do sujeito “I”, que exige “was”.
Na terceira lacuna, como já sabemos que o diálogo está se desenrolando no passado e, além
disso, podemos ver o verbo “go” no infinitivo. Esses são os indicativos de que “did” é a palavra
que completa a lacuna.
Na quarta lacuna, já sabemos que precisamos nos manter no tempo passado. Por isso, “went”
é a palavra que completa essa lacuna, já que “went” é o passado do verbo “go”.
Na quinta lacuna, sabemos que precisamos manter o diálogo no passado e, que precisamos
completar a lacuna com uma “short answer” negativa porque a frase começa com “no”.
Portanto, “wasn’t” é a palavra a ser encaixada na lacuna, tendo em vista que a pergunta foi
feita com a palavra “were” e, a reposta é dada com “I”.
Na sexta lacuna, sabemos que precisamos de um verbo conjugado no passado, mas essa lacuna
nem seria necessária para resolver a questão, até porque outros verbos conjugados no “simple
past” poderiam se encaixar de forma adequada nessa lacuna.
Temos, portanto, a seguinte sequência: were / was / did / went / wasn’t / stayed.

Aula 03 – Articles and Nouns 44


www.estrategiamilitar.com.br 61
Teacher Andrea Belo
Aula 03: Escola Naval 2021

GABARITO: D

Questão 06 – (Questão inédita Militares)


Read the dialogue and mark the right option
Read the sentences and mark the correct option to fill in the blanks respectively.
Elisa is ______ friend. ________ lives near my grandfather’s house. We love riding our bikes.
_______ bike is big. ________ is small. We love spending time together!

A) My / she / my / hers
B) Your / I / my / her
C) Her / he / her / mine
D) My / her / my / she
E) His / he / my / his

Elisa is __my____ friend. ___she_____ lives near my grandfather’s house. We love riding our
bikes. ___my____ bike is big. ___hers_____ is small. We love spending time together!
Essa é uma questão interessante, pois não há uma única resposta para cada lacuna. Precisamos
resolver a questão pensando no contexto como um todo. Podemos depreender do trecho que
há duas amigas ou dois amigos que gostam de andar de bicicleta. A bicicleta de uma é grande
e a do(a) outro(a) é pequena. Elas adoram passar tempo juntas.
Comentários:
A primeira lacuna é preenchida adequadamente por “my”, pois percebe-se que a narrador(a)
fala de sua relação com Elisa.
A segunda lacuna é mais fácil, pois precisamos de um “subject pronoun” que se refira a Elisa
para preenchê-la. “She” é, portanto, a única opção aplicável aqui.
Na terceira lacuna, mais uma vez o contexto geral do trecho é o que nos indica o pronome
correto a ser utilizado.
Como é perceptível que o trecho fala da relação entre amigos(as), sendo um deles a Elisa e o(a)
narrador(a) o segundo elemento da relação, o melhor pronome para a lacuna é “my”.
Na quarta lacuna, após utilizar “my” na lacuna anterior, temos apenas “hers” como opção, já
que já falei da minha bicicleta, agora preciso falar da dela.
Ao analisar as possibilidades para as lacunas, percebemos que elas são várias. É importante,
porém, perceber que a segunda lacuna era a única que não dava possibilidade de nenhum outro
pronome além de “she”, porque tinha de funcionar como sujeito da frase e teria de referi-se a
Elisa.

Aula 03 – Articles and Nouns 45


www.estrategiamilitar.com.br 61
Teacher Andrea Belo
Aula 03: Escola Naval 2021

Ao analisarmos as alternativas, vemos que apenas a letra A nos dava o pronome “she” como
opção para a segunda lacuna. Dessa forma, poderíamos resolver a questão ao resolver a
segunda lacuna.
Fica o BIZU para você, meu querido aluno. Preste atenção nas alternativas, pois elas podem lhe
ajudar a resolver uma questão complexa, de maneira simples.
Temos a sequência “my / she / my / hers” – LETRA A

Questão 07 – (Questão inédita Militares)


Read the dialogue and mark the right option
Which word is incorrectly used in: “You failed, my ability in this test is unpossible”
A) Failed
B) Unpossible
C) My
D) In
E) Test

Comentários:
A alternativa A está correta. “Failed” é usado, nesse contexto, para dizer que falhou ou
fracassou no teste. A palavra está sendo empregada corretamente na frase.
A alternativa B está incorreta. O erro está em usar o prefixo “un” ao invés de “im” para dizer
impossível. A forma correta de escrever a palavra seria “impossible”.
A alternativa C está correta. “My” está sendo empregado de forma correta.
A alternativa D está correta. “In” está sendo usado de forma correta. “In” é a preposição
adequada.
A alternativa E está correta. “”Test” está sendo utilizado corretamente.
GABARITO: B

Aula 03 – Articles and Nouns 46


www.estrategiamilitar.com.br 61
Teacher Andrea Belo
Aula 03: Escola Naval 2021

Questão 08– (Questão inédita Militares)


Read the dialogue and mark the right option
Use the verbs in the parentheses to complete the following statements.
I- I usually ______ (do) my homework after dinner, but now I ______ (do) a diferent task.
II- Eli _______ (eat) a lot. Sometimes, he _______ (want) to eat less
III- Alessandra _________(live) in Boston, United States. During the week, her daughters
_______ (work) as delivery girls for a big tech company.
Now mark the option which completes them respectively.
A) Am doing / am doing / is eating / wants / lives / working
B) Am doing / do / eats / is wanting / is living / works
C) Am doing / do / eat / is wanting / is living / works
D) Do / do / is eating / want / live / is working
E) Do / am doing / eats / wants / lives / work

I- I usually __do____ (do) my homework after dinner, but now I _am doing___ (do) a diferent
task.
II- Eli _eats___ (eat) a lot. Sometimes, he __wants___ (want) to eat less
III- Alessandra __lives____(live) in Boston, United States. During the week, her daughters
__work___ (work) as delivery girls for a big tech company.
Comentários:
Na sentença I, a primeira lacuna pede que “do” seja utilizado no presente. Fato confirmado
pelo advérbio “usually” que indica frequência. Portanto, devemos preencher essa lacuna com
“do”. Já a palavra “now” pede que a segunda lacuna seja preenchida no gerúndio, já que dá a
ideia de que está acontecendo naquele momento. A segunda lacuna deve ser preenchida com
“am doing”.
Na sentença II, as duas lacunas possuem mais de uma opção que se encaixaria corretamente.
Mas é importante observar que, ao acertar as duas primeiras, a questão já estaria resolvida por
conta das opções apresentadas nas alternativas. Mas, de qualquer forma, a melhor opção para
preencher as lacunas era colocar ambos os verbos no presente simples, porque, nesse
contexto, dão a ideia de hábito, frequência. Portanto, “eats” e “wants” seriam o melhor encaixe
para esses verbos.
Na sentença III, seguindo o mesmo raciocínio da sentença anterior, a melhor opção para ambos
os verbos era colocá-los no presente simples. A melhor forma de encaixá-los seria “lives” e
“work”, respectivamente.
Temos a sequência “do / am doing / eats / wants / lives / work”

Aula 03 – Articles and Nouns 47


www.estrategiamilitar.com.br 61
Teacher Andrea Belo
Aula 03: Escola Naval 2021

GABARITO: E

Texto para questão 09

AFA/2017
TEXT: Howard Gardner: ‘Multiple intelligences’ are not ‘learning styles’ by Valerie Strauss

The fields of psychology and education were revolutionized 30 years ago when we now
worldrenowned psychologist Howard Gardner published his 1983 book Frames of Mind: The
Theory of Multiple Intelligences, which detailed a new model of human intelligence that went
beyond the traditional view that there was a single kind that could be measured by standardized
tests.
Gardner’s theory initially listed seven intelligences which work together: linguistic, logical-
mathematical, musical, bodily-kinesthetic, interpersonal and intrapersonal; he later added an
eighth, naturalist intelligence and says there may be a few more. The theory became highly popular
with K-12¹ educators around the world seeking ways to reach students who did not respond to
traditional approaches, but over time, ‘multiple intelligences’ somehow became synonymous with
the concept of ‘learning styles’. In this important post, Gardner explains why the former is not the
latter.
It’s been 30 years since I developed the notion of ‘multiple intelligences’. I have been gratified by
the interest shown in this idea and the ways it’s been used in schools, museums, and business
around the world. But one unanticipated consequence has driven me to distraction and that’s the
tendency of many people, including persons whom I cherish, to credit me with the notion of
‘learning styles’ or to collapse ‘multiple intelligences’ with ‘learning styles’. It’s high time to relieve
my pain and to set the record straight.
First a word about ‘MI theory’. On the basis of research in several disciplines, including the study
of how human capacities are represented in the brain, I developed the idea that each of us has a
number of relatively independent mental faculties, which can be termed our ‘multiple
intelligences’. The basic idea is simplicity itself. A belief in a single intelligence assumes that we
have one central, all-purpose computer, and it determines how well we perform in every sector of
life. In contrast, a belief in multiple intelligences assumes that human beings have 7 to 10 distinct
intelligences.
Even before I spoke and wrote about ‘MI’, the term ‘learning styles’ was being bandied about in
educational circles. The idea, reasonable enough on the surface, is that all children (indeed all of
us) have distinctive minds and personalities. Accordingly, it makes sense to find out about learners
and to teach and nurture them in ways that are appropriate, that they value, and above all, are
effective.

Aula 03 – Articles and Nouns 48


www.estrategiamilitar.com.br 61
Teacher Andrea Belo
Aula 03: Escola Naval 2021

Two problems: first, the notion of ‘learning styles’ is itself not coherent. Those who use this term
do not define the criteria for a style, nor where styles come from, how they are recognized/
assessed/ exploited. Say that Johnny is said to have a learning style that is ‘impulsive’. Does that
mean that Johnny is ‘impulsive’ about everything? How do we know this? What does this imply
about teaching? Should we teach ‘impulsively’, or should we compensate by ‘teaching reflectively’?
What of learning style is ‘right-brained’ or visual or tactile? Same issues apply.
Problem #2: when researchers have tried to identify learning styles, teach consistently with those
styles, and examine outcomes, there is not persuasive evidence that the learning style analysis
produces more effective outcomes than a ‘one size fits all approach’. Of course, the learning style
analysis might have been inadequate. Or even if it is on the mark, the fact that one intervention
did not work does not mean that the concept of learning styles is fatally imperfect; another
intervention might have proved effective. Absence of evidence does not prove non-existence of a
phenomenon; it signals to educational researchers: ‘back to the drawing boards’.
Here’s my considered judgment about the best way to analyze this lexical terrain: Intelligence: We
all have the multiple intelligences. But we signed out, as a strong intelligence, an area where the
person has considerable computational power. Style or learning style: A hypothesis of how an
individual approach the range of materials. If an individual has a ‘reflective style’, he/she is
hypothesized to be reflective about the full range of materials. We cannot assume that
reflectiveness in writing necessarily signals reflectiveness in one’s interaction with the others.
Senses: Sometimes people speak about a ‘visual’ learner or an ‘auditory’ learner. The implication
is that some people learn through their eyes, others through their ears. This notion is incoherent.
Both spatial information and reading occur with the eyes, but they make use of entirely different
cognitive faculties. What matters is the power of the mental computer, the intelligence that acts
upon that sensory information once picked up.
These distinctions are consequential. If people want to talk about ‘an impulsive style’ or a ‘visual
learner’, that’s their prerogative. But they should recognize that these labels may be unhelpful, at
best, and ill-conceived at worst.
In contrast, there is strong evidence that human beings have a range of intelligences and that
strength (or weakness) in one intelligence does not predict strength (or weakness) in any other
intelligences. All of us exhibit jagged profiles of intelligences. There are common sense ways of
assessing our own intelligences, and even if it seems appropriate, we can take a more formal test
battery. And then, as teachers, parents, or self assessors, we can decide how best to make use of
this information. (Adapted from https://www.washingtonpost.com/news/answer-sheet)

Questão 09 (AFA/2017) - In the sentence “it’s been 30 years since I developed the notion of
‘multiple intelligences’” (lines 20 and 21), the contraction refers to
a) It has.
b) It been.
c) It is.
d) It was.

Aula 03 – Articles and Nouns 49


www.estrategiamilitar.com.br 61
Teacher Andrea Belo
Aula 03: Escola Naval 2021

Comentários:
A alternativa A está correta. Trata-se de um verbo conjugado no tempo “present perfect”. Ele
deve, portanto, ser conjugado com o verbo auxiliar “have”, seguido do verbo principal
conjugado no “past participle”. Portanto, “it’s been” equivale a “it has been”.
A alternativa B está incorreta. Não se utiliza sujeito + verbo no “past participle”. Tem de haver
o verbo auxiliar “have” para que a estrutura esteja correta.
A alternativa C está incorreta. O uso do verbo “to be” no particípio “been” é o grande indicador
de que temos uma estrutura no “present perfect” e, portanto, faz-se necessário o uso do
auxiliar “have”.
A alternativa D está incorreta. “Was” é um verbo conjugado no “simple past” enquanto o trecho
está conjugado no “present perfect”. Não é possível usar “was” acompanhado de verbo no
particípio.

Questão 10 (EPCAR/2019) - “Slavery continues today and harms people in every country in the
world” (lines 2 and 3). The highlighted words can be substituted for _____.
a) them
b) theirs
c) their
d) they
Comentários:
Vamos lá, falar de pronomes e, é claro, muitos artigos e substantivos envolvidos.
“Slavery continues today and harms people in every country in the world”. A escravidão
continua hoje e afeta pessoas em todos os países do mundo.

Ao analisar a sentença traduzida, podemos observar que o trecho sublinhado atua como objeto
direto. A questão pede para substituirmos o trecho sublinhado por um pronome
correspondente.

A alternativa A está correta. “Them” é o pronome adequado para a substituição proposta pela
questão. “Them” é um pronome pessoal que é usado na função sintática de objeto da oração,
enquanto “They” é um pronome pessoal usado na função sintática de sujeito da oração.

A alternativa B está incorreta. “Theirs” é um pronome possessivo utilizado no final das frases.
A alternativa C está incorreta. “Their” é um pronome possessivo utilizado antes do substantivo
ao qual ele se relaciona.

Aula 03 – Articles and Nouns 50


www.estrategiamilitar.com.br 61
Teacher Andrea Belo
Aula 03: Escola Naval 2021

A afirmativa D está incorreta. “They” é um pronome pessoal usado na função sintática de


sujeito da oração. Nesse caso, o trecho sublinhado atua como objeto, e não sujeito. Por isso, o
pronome “them” é o mais adequado para substituir o trecho.

Considerações finais
Outra aula concluída, com artigos e substantivos. Você está chegando cada dia mais perto da
sua prova e da compreensão geral de tudo o que vai responder. O aprendizado é contínuo. Seus
estudos devem ser também.
É um passo a mais até a sua aprovação! As estruturas de frases com diferentes vocábulos e
interpretação de textos em Inglês dependem dos artigos, dos substantivos e de muito mais
conteúdos que vêm por aí, nas próximas aulas.
E, dia após dia, você se prepara, ficando confiante e seguro dos seus resultados. Vai dar certo
e você sabe disso!

Outro detalhe importante para seu sucesso nos estudos é a leitura de textos
complementares daquelas fontes da prova, tais como jornais e revistas internacionais, com
reportagens e artigos diversos para explorar seus conhecimentos. Aqui temos as traduções como
bônus e material complementar também.
Estude! Faça listas de exercícios, de palavras novas, de palavras que você acha difíceis. Até
mesmo os exercícios inéditos ou de anos anteriores aqui presentes, quando resolvidos mais de uma
vez, fica mais fácil identificar algo que antes parecia difícil.
É sua conquista de etapas e que tornará você, um candidato mais bem preparado e confiante
para realizar uma excelente prova.
É importante lembrar também do nosso Fórum de dúvidas, exclusivo do Estratégia
Concursos. Será minha forma de responder, no prazo máximo de 48 horas, o que mais você precise
saber para que os conteúdos fiquem ainda mais claros em seus estudos, certo?

Aula 03 – Articles and Nouns 51


www.estrategiamilitar.com.br 61
Teacher Andrea Belo
Aula 03: Escola Naval 2021

Referências bibliográficas
ACKLAM, Richard; CRACE, Araminta. Total English: Pre intermediate. 1 ed. Grã-Bretanha: Longman
do Brasil, 2005.
BAKER, M. In other words: a coursebook on translation. Routledge, 1992.
BLATT, Franz. Précis de Syntaxe Latine. Lyon, Paris: IAC, 1952.
BENTES, Anna Christina e Mussalim, Fernanda (org.). Introdução À Linguística, Domínios E Fronteiras.
6ª edição. Editora Cortez. São Paulo. 2006.
BOURGOGNE, Cleuza Vilas Boas & Silva Lilian Santos. Interação & Transformação. SP: Ed. Brasil,
1999.
BOWKER, L. & PEARSON, J. Working with Specialized Language. Routledge. Capítulos 1, 2, 8,10 e 11,
2002.
BUSSE, Winfried Busse & Mário Vilela. Gramática de Valências. Coimbra: Almedina,1986.
VENUTI, L. (ed.) 1992. Rethinking Translation: discourse, subjectivity, ideology. Routledge.
CARVALHO, José Herculano de. Estudos Lingüísticos. v. 2. Coimbra: Atlântida, 1969.
CHIMIM, Renata; Ilearn English student book, 4 / Renata Chimim, Viviane Kirmeliene; [obra coletiva
organizada e desenvolvida pela editora]. 1ª. ed. São Paulo: Pearson Education do Brasil, 2013.
CORBEIL, J.-Cl., ARCHAMBAULT, A. Michaelis Tech dicionário temático visual inglês-português-
francês-espanhol. Tradução: Marisa Soares de Andrade. São Paulo: Melhoramentos, 1997.
CUNHA, Celso. Nova Gramática do Português Contemporâneo. Rio de Janeiro: Nova fronteira,
terceira edição, 2001.
CUNNINGHAM, Gillie; REDSTON, Chris. Face2Face: Upper Intermediate. 1 ed. Brazil: Cambridge,
2001.

Aula 03 – Articles and Nouns 52


www.estrategiamilitar.com.br 61
Teacher Andrea Belo
Aula 03: Escola Naval 2021

DANIELS, H. Vygotsky and pedagogy. Educational Tasks Pedagogical Communication for Teachers.
Routledge, 3rd edition, 2001.
FAIRCLOUGH, N. Discourse and social change. Polity Press, 1992.
GENTZLER, E. Contemporary Translation Theory. Routledge, 1993.
HOUAISS, A., CARDIM, I. Dicionário universitário Webster inglês-português / português-inglês. São
Paulo: Record, 1998.
HYLAND, K. Genre and second language writing – For teachers and pedagogical professionals in
general, 2003.
HUTCHINSON, Tom & WATERS, Alan. English for Specific Purposes. Cambridge: Cambridge University
Press, 1996.
LAFACE, A. O dicionário e o contexto escolar. Revista Brasileira de Linguística, Unesp/Assis, v.9, 1982,
p. 165-179.
LOBATO, M.P. Lúcia. Teorias Linguísticas e ensino do português como língua materna. Brasília: UNB,
1999.
MICHAELIS Tech Dicionário Temático Visual: línguas estrangeiras – Pesquisa e tradução Marisa
Soares de Andrade. – São Paulo: Companhia Melhoramentos, 1997..
SILVA, João Antenor de C., GARRIDO, Maria Lina, BARRETO, Tânia Pedrosa. Inglês Instrumental:
Leitura e Compreensão de Textos. Salvador: Centro Editorial e Didático, UFBA. 1994.
SILVA, T.; MATSUDA, P. Second language writing research: perspectives on the process of knowledge
construction, 2001.
SILVEIRA BUENO, F. A formação histórica da língua portuguesa. 3. ed. São Paulo: Saraiva , 1967.
SIMPSON, J., WEINER, E. (eds.) Oxford English dictionary on CD-ROM. 2ed. Oxford : Oxford
University Press, 1999.
PASCHOALIN, Maria Aparecida; SPADOTO, Neuza Terezinha. Gramática, Teoria e Exercícios. Editora
FDT. São Paulo. 1996.
RIBEIRO, Manuel P. Nova gramática aplicada da língua portuguesa. Rio de Janeiro: Metáfora editora,
14ª edição, 2002.
TUCK, Michael. Oxford Dictionary of Computing for Learners of English. Oxford: Oxford University
Press, 1996.
CETEMFolha/NILC: Corpus de Extractos de Textos Electrónicos. Banco de dados. Disponível em:
http://acdc.linguateca.pt/cetenfolha>.Último acesso (vários acessos) em: 04.05.2019.
VENTURINI, Laercio. Origem e desenvolvimento da língua inglesa. Disponível em:
<http://www.startenglish.com.br/index.php?option=com_content&task=view&id=100&Itemid=97
>. Acesso em: 22 mai. 2012.
OXFORD photo dictionary. Oxford: Oxford University Press, 1992

Referências complementares (websites):

Aula 03 – Articles and Nouns 53


www.estrategiamilitar.com.br 61
Teacher Andrea Belo
Aula 03: Escola Naval 2021

www.richmond.com.br - Acesso em 18 de março de 2019.


http://www.sk.com.br/sk-perf.html - Acesso em 19 de março de 2019.
https://www.inglesnapontadalingua.com.br/2013/03/o-que-sao-falsos-cognatos.html - Acesso em
19 de março de 2019.
https://englishlive.ef.com/pt-br/blog/15-expressoes-idiomaticas-comuns-em-ingles/
https://www.infoescola.com/ingles/
https://www.solinguainglesa.com.br/conteudo/indice.php
https://www.inglesnapontadalingua.com.br
https://www.englishexperts.com.br/

Traduções

Experiências de realidade virtual


gratificantes são diferentes de qualquer
outra coisa que você vai experimentar em
sua TV na sua sala de estar. Infelizmente,
as experiências de RV satisfatórias são, em
sua maior parte, dependentes de um
hardware adicional. Isso torna quase
impossível apenas pegar um fone de
ouvido e começar a jogar, e eleva o preço
da admissão por algumas centenas de
dólares no mínimo.
O $399 Oculus Quest ($499 para a versão
de 128GB) é a terceira grande tentativa da
empresa de pregar a realidade virtual. Seu
primeiro, a polarização Oculus Rift, trouxe
VR para PCs altamente qualificados,
exigindo que os usuários configurassem
sensores estacionários para jogar. Seu
segundo, o menor, autocontido Oculus Go,
abandonou o PC por causa de alguma coisa
solta recém-encontrada, mas o seu único
controlador, com graus limitados de
liberdade, e a massacrante experiência
Aula 03 – Articles and Nouns 54
www.estrategiamilitar.com.br
eventualmente deixou muitas pessoas
61
entediadas.
Teacher Andrea Belo
Aula 03: Escola Naval 2021

TEXT: Howard Gardner: ‘Multiple intelligences’ are not ‘learning styles’ by Valerie Strauss

The fields of psychology and education were revolutionized 30 years ago when we now
worldrenowned psychologist Howard Gardner published his 1983 book Frames of Mind: The
Theory of Multiple Intelligences, which detailed a new model of human intelligence that went
beyond the traditional view that there was a single kind that could be measured by standardized
tests.
Gardner’s theory initially listed seven intelligences which work together: linguistic, logical-
mathematical, musical, bodily-kinesthetic, interpersonal and intrapersonal; he later added an
eighth, naturalist intelligence and says there may be a few more. The theory became highly popular
with K-12¹ educators around the world seeking ways to reach students who did not respond to
traditional approaches, but over time, ‘multiple intelligences’ somehow became synonymous with
the concept of ‘learning styles’. In this important post, Gardner explains why the former is not the
latter.
It’s been 30 years since I developed the notion of ‘multiple intelligences’. I have been gratified by
the interest shown in this idea and the ways it’s been used in schools, museums, and business
around the world. But one unanticipated consequence has driven me to distraction and that’s the
tendency of many people, including persons whom I cherish, to credit me with the notion of
‘learning styles’ or to collapse ‘multiple intelligences’ with ‘learning styles’. It’s high time to relieve
my pain and to set the record straight.
First a word about ‘MI theory’. On the basis of research in several disciplines, including the study
of how human capacities are represented in the brain, I developed the idea that each of us has a
number of relatively independent mental faculties, which can be termed our ‘multiple
intelligences’. The basic idea is simplicity itself. A belief in a single intelligence assumes that we
have one central, all-purpose computer, and it determines how well we perform in every sector of
life. In contrast, a belief in multiple intelligences assumes that human beings have 7 to 10 distinct
Aula 03 – Articles and Nouns
intelligences. 55
www.estrategiamilitar.com.br 61
Even before I spoke and wrote about ‘MI’, the term ‘learning styles’ was being bandied about in
educational circles. The idea, reasonable enough on the surface, is that all children (indeed all of
Teacher Andrea Belo
Aula 03: Escola Naval 2021

Two problems: first, the notion of ‘learning styles’ is itself not coherent. Those who use this term
do not define the criteria for a style, nor where styles come from, how they are recognized/
assessed/ exploited. Say that Johnny is said to have a learning style that is ‘impulsive’. Does that
mean that Johnny is ‘impulsive’ about everything? How do we know this? What does this imply
about teaching? Should we teach ‘impulsively’, or should we compensate by ‘teaching reflectively’?
What of learning style is ‘right-brained’ or visual or tactile? Same issues apply.
Problem #2: when researchers have tried to identify learning styles, teach consistently with those
styles, and examine outcomes, there is not persuasive evidence that the learning style analysis
produces more effective outcomes than a ‘one size fits all approach’. Of course, the learning style
analysis might have been inadequate. Or even if it is on the mark, the fact that one intervention
did not work does not mean that the concept of learning styles is fatally imperfect; another
intervention might have proved effective. Absence of evidence does not prove non-existence of a
phenomenon; it signals to educational researchers: ‘back to the drawing boards’.
Here’s my considered judgment about the best way to analyze this lexical terrain: Intelligence: We
all have the multiple intelligences. But we signed out, as a strong intelligence, an area where the
person has considerable computational power. Style or learning style: A hypothesis of how an
individual approach the range of materials. If an individual has a ‘reflective style’, he/she is
hypothesized to be reflective about the full range of materials. We cannot assume that
reflectiveness in writing necessarily signals reflectiveness in one’s interaction with the others.
Senses: Sometimes people speak about a ‘visual’ learner or an ‘auditory’ learner. The implication
is that some people learn through their eyes, others through their ears. This notion is incoherent.
Both spatial information and reading occur with the eyes, but they make use of entirely different
cognitive faculties. What matters is the power of the mental computer, the intelligence that acts
upon that sensory information once picked up.
These distinctions are consequential. If people want to talk about ‘an impulsive style’ or a ‘visual
learner’, that’s their prerogative. But they should recognize that these labels may be unhelpful, at
best, and ill-conceived at worst.
In contrast, there is strong evidence that human beings have a range of intelligences and that
strength (or weakness) in one intelligence does not predict strength (or weakness) in any other
Aula 03 – Articles and Nouns
intelligences. All of us exhibit jagged profiles of intelligences. There are common sense ways of 56
www.estrategiamilitar.com.br 61
assessing our own intelligences, and even if it seems appropriate, we can take a more formal test
battery. And then, as teachers, parents, or self assessors, we can decide how best to make use of
Teacher Andrea Belo
Aula 03: Escola Naval 2021

TEXTO: Howard Gardner:


'Múltiplas inteligências' não são 'estilos de aprendizagem' de Valerie Strauss

Os campos da psicologia e da educação foram revolucionados há 30 anos, quando agora o psicólogo


de renome mundial Howard Gardner publicou seu livro Frames of Mind: The Theory of Multiple
Intelligences, de 1983, que detalhava um novo modelo de inteligência humana que ia além da visão
tradicional de que havia um tipo único que pode ser medido por testes padronizados.
A teoria de Gardner listou inicialmente sete inteligências que trabalham juntas: linguística, lógico-
matemática, musical, cinestésica, interpessoal e intrapessoal; mais tarde, ele acrescentou uma
oitava inteligência naturalista e diz que pode haver mais algumas. A teoria se tornou muito popular
entre os educadores de ensino fundamental e médio do mundo todo, buscando maneiras de
alcançar os alunos que não responderam às abordagens tradicionais, mas com o tempo,
'inteligências múltiplas' de alguma forma tornaram-se sinônimos do conceito de 'estilos de
aprendizagem'. Neste importante post, Gardner explica por que o primeiro não é o último.
Faz 30 anos que desenvolvi a noção de 'inteligências múltiplas'. Fiquei satisfeito com o interesse
demonstrado nessa idéia e com a maneira como ela foi usada em escolas, museus e negócios em
todo o mundo. Mas uma consequência imprevista me levou à distração e essa é a tendência de
muitas pessoas, incluindo pessoas que eu aprecio, de me creditar com a noção de 'estilos de
aprendizagem' ou de colapsar 'inteligências múltiplas' com 'estilos de aprendizagem'. É hora de
aliviar minha dor e esclarecer as coisas.
Primeiro uma palavra sobre a "teoria do MI". Com base em pesquisas em várias disciplinas, incluindo
o estudo de como as capacidades humanas são representadas no cérebro, desenvolvi a ideia de que
cada um de nós tem várias faculdades mentais relativamente independentes, que podem ser
denominadas nossas 'inteligências múltiplas'. A idéia básica é a própria simplicidade. A crença em
uma única inteligência pressupõe que temos um computador central para todos os fins e determina
o desempenho de todos os setores da vida. Em contraste, uma crença em múltiplas inteligências
pressupõe que os seres humanos tenham 7 a 10 inteligências distintas.
Mesmo antes de falar e escrever sobre "MI", o termo "estilos de aprendizagem" estava sendo usado
nos círculos educacionais.
Aula 03 – Articles andANouns
idéia, suficientemente razoável na superfície, é que todas as crianças57
(de fato todos nós) tenham mentes e personalidades distintas. Consequentemente, faz sentido61
www.estrategiamilitar.com.br
descobrir sobre os alunos e ensiná-los e alimentá-los de maneiras apropriadas, que valorizem e,
Teacher Andrea Belo
Aula 03: Escola Naval 2021

Dois problemas: primeiro, a noção de "estilos de aprendizagem" não é coerente. Aqueles que usam
esse termo não definem os critérios para um estilo, nem de onde vêm os estilos, como são
reconhecidos / avaliados / explorados. Diga que se diz que Johnny tem um estilo de aprendizado
"impulsivo". Isso significa que Johnny é "impulsivo" em relação a tudo? Como nós sabemos disso? O
que isso implica no ensino? Devemos ensinar 'impulsivamente' ou devemos compensar 'ensinando
de forma reflexiva'? Qual o estilo de aprendizagem é "cérebro direito" ou visual ou tátil? Os mesmos
problemas se aplicam.
Problema # 2: quando os pesquisadores tentam identificar estilos de aprendizagem, ensinar
consistentemente com esses estilos e examinar os resultados, não há evidências convincentes de
que a análise do estilo de aprendizagem produza resultados mais efetivos do que uma abordagem
de "tamanho único". Obviamente, a análise do estilo de aprendizagem pode ter sido inadequada.
Ou mesmo se estiver certo, o fato de uma intervenção não funcionar não significa que o conceito de
estilos de aprendizagem seja fatalmente imperfeito; outra intervenção pode ter se mostrado eficaz.
Ausência de evidência não prova a inexistência de um fenômeno; sinaliza para os pesquisadores
educacionais: "de volta às pranchetas".
Aqui está o meu julgamento considerado sobre a melhor maneira de analisar esse terreno lexical:
Inteligência: todos nós temos múltiplas inteligências. Mas assinamos, como uma inteligência forte,
uma área em que a pessoa tem um poder computacional considerável. Estilo ou estilo de
aprendizagem: Uma hipótese de como um indivíduo aborda a variedade de materiais. Se um
indivíduo tem um "estilo reflexivo", é provável que ele reflita sobre toda a gama de materiais. Não
podemos assumir que a refletividade na escrita necessariamente sinaliza reflexividade na interação
de uma pessoa com as outras.
Sentidos: às vezes as pessoas falam sobre um aluno 'visual' ou 'auditivo'. A implicação é que algumas
pessoas aprendem com seus olhos, outras através de seus ouvidos. Essa noção é incoerente. Tanto
a informação espacial quanto a leitura ocorrem com os olhos, mas fazem uso de faculdades
cognitivas inteiramente diferentes. O que importa é o poder do computador mental, a inteligência
que age sobre essas informações sensoriais, uma vez captadas.
Aula 03 – Articles and Nouns 58
Essas distinções são consequenciais. Se as pessoas querem falar sobre 'um estilo impulsivo' ou um
www.estrategiamilitar.com.br 61
'aprendiz visual', essa é uma prerrogativa deles. Mas eles devem reconhecer que esses rótulos
Teacher Andrea Belo
Aula 03: Escola Naval 2021

Um ex-executivo do Facebook disse que a mídia social está causando grandes danos à
sociedade em todo o mundo. O executivo é um homem chamado Chamath Palihapitiya. Ele
entrou no Facebook em 2007 e tornou-se vice-presidente. Ele foi responsável por aumentar o
número de usuários que o Facebook tinha. Palihapitiya disse que se sente muito culpado por
conseguir que mais pessoas usem as redes sociais. Ele disse que as redes estão destruindo a
sociedade porque estão mudando o comportamento das pessoas. Vinte anos atrás, as pessoas
conversavam cara a cara. Hoje, as pessoas trocam mensagens e não falam. As pessoas também
se importam com o que as outras pessoas pensam delas. Eles postam fotos e esperam para ver
quantas
Aulapessoas gostam
03 – Articles and da foto. Eles ficam muito tristes se as pessoas não gostam da foto.
Nouns 59
www.estrategiamilitar.com.br
Palihapitiya disse que as pessoas devem fazer uma longa pausa nas mídias sociais para poder 61
experimentar a vida real. Ele quer que as pessoas se valorizem em vez de avaliar "corações,
Teacher Andrea Belo
Aula 03: Escola Naval 2021

There's nowhere like Scotland. Scotland is a country in a country. It is part of Great Britain (England,
Scotland and Wales), and of the United Kingdom (England, Scotland, Wales and Northern Ireland).
Scotland is in the far northwest of Europe, between the Atlantic Ocean and the North Sea. It is
often cold and grey, and it often rains a lot. But the people of Scotland love their country, and many
visitors to Scotland love it too. They love the beautiful hills and mountains of the north, the sea and
the eight hundred islands, and the six cities: Edinburgh, Glasgow, Aberdeen, Dundee, Inverness and
Stirling. The country is special, and Scottish people are special too: often warm and friendly. There
are about five million people in Scotland. Most Scots live in the south, in or near the big cities of
Edinburgh and Glasgow. Most of the north of the country is very empty; not many people live there.
A Scottish person is also called a Scot, but you cannot talk about a Scotch person: Scotch means
whisky, a drink made in Scotland. Scottish people are British, because Scotland is part of Great
Britain, but you must not call Scottish people English! The Scots and the English are different. These
days everyone in Scotland speaks English. But, at one time, people in the north and west of Scotland
did not speak English. They had a different language, a beautiful language called Gaelic. About
60,000 people, 1% of the people in Scotland, speak Gaelic now. But many more want Gaelic in their
lives because it is part of the story of Scotland.

Não há lugar como a Escócia. A Escócia é um país em um país. Faz parte da Grã-Bretanha
(Inglaterra, Escócia e País de Gales) e do Reino Unido (Inglaterra, Escócia, País de Gales e
Irlanda do Norte). A Escócia fica no extremo noroeste da Europa, entre o Oceano Atlântico e
o Mar do Norte. Geralmente é frio e cinza, e chove muito. Mas o povo da Escócia ama seu
país, e muitos visitantes da Escócia também o amam. Eles adoram as belas colinas e
montanhas do norte, o mar e as oitocentas ilhas e as seis cidades: Edimburgo, Glasgow,
Aberdeen, Dundee, Inverness e Stirling. O país é especial, e o povo escocês também é especial:
geralmente caloroso e amigável. Existem cerca de cinco milhões de pessoas na Escócia. A

Aula 03 – Articles and Nouns 60


www.estrategiamilitar.com.br 61
Teacher Andrea Belo
Aula 03: Escola Naval 2021

maioria dos escoceses vive no sul ou nas grandes cidades de Edimburgo e Glasgow. A maior
parte do norte do país está muito vazia; muitas pessoas não moram lá. Uma pessoa escocesa
também é chamada de escocês, mas não se pode falar de um escocês: escocês significa
uísque, uma bebida feita na Escócia. O povo escocês é britânico, porque a Escócia faz parte
da Grã-Bretanha, mas você não deve chamar o povo escocês de inglês! Os escoceses e os
ingleses são diferentes. Hoje em dia, todos na Escócia falam inglês. Mas, ao mesmo tempo,
as pessoas no norte e oeste da Escócia não falavam inglês. Eles tinham um idioma diferente,
um idioma bonito chamado gaélico. Cerca de 60.000 pessoas, 1% das pessoas na Escócia,
falam gaélico agora. Mas muitos mais querem gaélico em suas vidas porque faz parte da
história da Escócia.

Aula 03 – Articles and Nouns 61


www.estrategiamilitar.com.br 61
Aula 04 –
Adjectives and
Adverbs

ESCOLA NAVAL 2021

Teacher Andrea Belo


Teacher Andrea Belo
Aula 04: Escola Naval 2021

Sumário
Introdução ........................................................................................................................................................... 3
Adjetivos ............................................................................................................................................................. 4
Adjetivos – Grau comparativo .............................................................................................................................. 7
Comparativo de superioridade...................................................................................................................................... 7
Comparativo de igualdade .......................................................................................................................................... 10
Comparativo de inferioridade ..................................................................................................................................... 11
Adjetivos – Grau superlativo............................................................................................................................... 12
Superlativo de superioridade ...................................................................................................................................... 12
Superlativo de inferioridade ........................................................................................................................................ 13
Advérbios .......................................................................................................................................................... 14
Advérbios de modo ..................................................................................................................................................... 18
Advérbios de frequência ............................................................................................................................................. 19
Advérbios de tempo .................................................................................................................................................... 20
Advérbios de lugar ...................................................................................................................................................... 21
Advérbios de dúvida e/ou certeza............................................................................................................................... 23
Advérbios de intensidade ............................................................................................................................................ 26
Locuções adverbiais .................................................................................................................................................... 28
Advérbios: Grau comparativo ............................................................................................................................. 29
Comparativo de Igualdade.......................................................................................................................................... 29
Comparativo de superioridade.................................................................................................................................... 30
Comparativo de inferioridade ..................................................................................................................................... 31
Advérbios: Grau Superlativo ............................................................................................................................... 32
Superlativo de superioridade ...................................................................................................................................... 32
Superlativo de inferioridade ........................................................................................................................................ 33
Questão inédita ................................................................................................................................................. 34
Questões de anos anteriores para prática ........................................................................................................... 36
11. Gabarito ................................................................................................................................................. 45
12. Questões comentadas ............................................................................................................................. 46
13. Considerações finais ................................................................................................................................ 59
14. Referências bibliográficas ............................................................................................................................. 60
15. Traduções..................................................................................................................................................... 62

Aula 04 – English General Presentation 2


www.estrategiamilitar.com.br 70
Teacher Andrea Belo
Aula 04: Escola Naval 2021

Introdução
Identificar em uma frase, os advérbios e os adjetivos, é algo considerado tricky, ou seja, algo
melindroso, arriscado. Isso porque, em Inglês, como também em outras línguas, essas palavras nos
enganam bastante pela maneira como se apresentam nos textos.
Mas, você verá, nessa aula, como é simples a distinção desses dois termos.
Vamos, então, à nossa aula sobre adjetivos e advérbios, para que você possa aprender as
regras de uso de cada um deles para encontrá-los inseridos em sua prova, levando você a acertar o
que for perguntado acerca desse assunto.
Os adjectives – adjetivos, são palavras que têm o propósito de qualificar ou modificar os
substantivos. Às vezes, modificam pronomes também, como veremos nas explicações. Adjetivos têm
como meta proporcionar detalhes e descrever melhor pessoas, lugares, objetos etc.
E, com o acréscimo dos adjetivos nas frases, as ideias apresentadas se tornam mais
interessantes e os textos ficam mais interativos e atrativos na hora da leitura.
Os adverbs – advérbios, são palavras que também acrescentam informações importantes ao
texto, porém, geralmente, modificam os verbos ou os próprios adjetivos. O advérbio é um elemento
que indica a circunstância em que se encontra o verbo e assim, vai ressaltar as circunstâncias, ou
seja, o modo, a intensidade, o lugar, uma negação, uma afirmação, uma dúvida, entre outras
situações diversas nos textos.
Tanto adjetivos quanto advérbios são importantes elementos e, devemos sempre nos
lembrar de que, uma determinada palavra em sua prova, pode fazer com que o termo analisado
mude, de uma classe gramatical para outra, a depender do contexto em que está inserida.
Por isso, esse material está dividido em detalhadas etapas, com os tópicos explicitados, para
que você estude cada item, separadamente e com muitos exemplos.
Assim, no momento da prática de exercícios, você perceberá que estão envolvidos, em uma
só questão, diferentes assuntos, que abrangem todo o conteúdo a ser estudado por você – da
mesma forma que são elaboradas as provas das Carreiras Militares em geral.
Vou orientar você, mostrando as normas que precisam ser seguidas em relação aos adjetivos
e aos advérbios, para que você os encontre nos textos e saiba como reconhecê-los nas alternativas
de cada questão.
Vamos lá! Você consegue e será o melhor candidato!

Aula 04 – English General Presentation 3


www.estrategiamilitar.com.br 70
Teacher Andrea Belo
Aula 04: Escola Naval 2021

Adjetivos
Os adjectives (adjetivos) caracterizam os substantivos e expressam qualidades ou
características em relação a pessoas, objetos ou animais. Essas qualidades podem ser duradouras
ou permanentes, características que expressam estados passageiros e condições e, além de
qualidades, as características das ações de pessoas, objetos ou animais.
A classe de adjetivos está entre as maiores classes gramaticais de palavras, ou seja, possui um
número ilimitado de palavras, que aumenta gradativamente, de acordo com a evolução da língua.
Dessa forma, não conseguimos fazer uma lista com todos os adjetivos da língua inglesa,
principalmente porque, essa lista se tornaria desatualizada daqui em tempo.
A posição do adjetivo em uma frase pode variar, assim como sua terminação. E veremos cada
caso em particular adiante. Antes de relatar outros detalhes sobre os adjetivos, vou citar algumas
das terminações mais comuns para os adjetivos em geral, com 5 exemplos cada, certo?

TERMINAÇÕES COMUNS NOS ADJETIVOS:

-al = cultural, original, practical, sensational, logical.


-an = Brazilian, American, Canadian, Belgian, European.
-ant = brilliant, elegant, important, significant, relevant.
-ar = bipolar, familiar, nuclear, peculiar, popular.
-ble = acceptable, comfortable, flexible, horrible, possible.
-ed = advanced, haunted, liberated, refined, sophisticated.
-ent = different, efficient, excellent, fluent, violent.
-ful = careful, colorful, helpful, useful, wonderful.
-ic = artistic, dramatic, fantastic, scientific, tragic.
-ing = amazing, darling, interesting, good-looking,
outgoing.
-ive = creative, expensive, naive, receptive, talkative.
-less = effectless, faithless, hairless, limitless, useless.
-ous = conscious, dangerous, fabulous, jealous, marvelous.
-y = easy, fancy, friendly, messy, satisfactory.

Aula 04 – English General Presentation 4


www.estrategiamilitar.com.br 70
Teacher Andrea Belo
Aula 04: Escola Naval 2021

Em Inglês, especificamente, os adjetivos são invariáveis em relação ao gênero (masculino e


feminino) e número (singular e plural).
Isso significa que, um mesmo adjetivo é utilizado para caracterizar masculino, feminino,
singular e plural. Por exemplo, o adjetivo new, significa novo, nova, novos e novas, veja:
I have a new pen. (Tenho uma caneta nova.)
l have two new pens. (Tenho duas canetas novas.)
I have a new notebook. (Tenho um caderno novo.)
I have new notebooks. (Tenho cadernos novos.)
E, como são invariáveis no gênero, veja outros exemplos, com referência a pessoas:
The boy is blond. (O garoto é loiro.)
The boys are blond. (Os garotos são loiros.)
The girl is blond. (A garota é loira.)
The girls are blond. (As garotas são loiras.)
Comumente, os adjetivos são utilizados depois de verbos de ligação. Esses verbos,
denominados verbos de ligação, não indicam ação, nunca. Eles indicam estado, ligando uma
característica a um sujeito.
Alguns exemplos de verbos de ligação são os verbos ser, estar, permanecer, tornar-se, sendo
em Inglês – to be, to stay, to become etc), ou seja, o adjetivo é ligado ao substantivo por esse verbo
de ligação. Vejamos alguns exemplos:
Bob is a calm buy. (Bob é um garoto calmo). ou Bob is calm. (Bob é calmo).
That song is popular. (Aquela música é pop).
He gets talkative when talking about technology. (Ele fica conversador/tagarela/comunicativo
quando fala sobre tecnologia).
Uma mudança interessante, que inclusive, eu já havia comentado em nossa primeira aula, é
que, ao contrário do Português, que a ordem é o adjetivo após o substantivo, em Inglês, o adjetivo
vem sempre antes do substantivo. Exemplos:
Um garoto alto = A tall boy. A tall girl – Uma garota alta.
Um garoto baixo = A short boy. A short girl – Uma garota baixa.


Adjectives TALL (alto) Nouns • BOY (garoto)

(adjetivos) SHORT (baixo) •
(sibstantivos) GIRL (garota)

Aula 04 – English General Presentation 5


www.estrategiamilitar.com.br 70
Teacher Andrea Belo
Aula 04: Escola Naval 2021

Existem alguns adjetivos com terminações diferenciadas e incomuns, que merecem atenção.
É o caso daqueles terminados em –ing e –ed, que podem, a princípio, fazer com que você se lembre
dos tempos verbais Present Continuous e Past Simple.
Mas, se não há verbo to be que antecede a palavra com terminação -ing, não é Continuous.
E, se também não há alguma expressão que comprove o tempo passado, não se classifica
como Past Simple. Além disso, você sabe diferenciar um adjetivo de um verbo, quer ver?
Jannet’s job is boring. (O emprego de Janet é chato.)
Jannet is bored today. (Jannet está entediada hoje.)
The accident was shocking. (O acidente foi chocante.)
She is shocked. (Ela está chocada.)
É claro que você sabe, olhando os exemplos acima, que os adjetivos bored e boring não se
parecem, nem de longe, com verbos, ações.
E, como estão qualificando substantivos – Jannet = bored, job = boring, accident = shocking e
she = chocked, são, de fato, adjetivos dentro da frase.
Os adjetivos terminados em -ed indicam como alguém se sente, ao passo que os terminados
em -ing dizem algo sobre alguém.
Quando utilizamos mais de um adjetivo na mesma frase, devemos obedecer a uma ordem de
uso, de acordo com a ideia implícita em cada um:

opinion (opinião) + size/age (tamanho/idade) + shape (forma)


+ color (cor) + origin (origem) + material (material) + noun (substantivo)
Exemplo: The expensive big white Canadian wood house. (A grande casa canadense cara branca de
madeira).

Não é necessário utilizar todos os adjetivos na construção de frases e, geralmente, não


aparecem todos em uma só oração. O exemplo acima foi, intencionalmente, para mostrar a você
quais informações podem estar presentes e fazer parte de uma ideia qualificando alguma coisa
usando muitos adjetivos.
Está percebendo como você vai identificar adjetivos nas questões da prova?
Quando perguntar se o termo pode ser substituído por um adjetivo ou se o próprio adjetivo
pode substituir outro, você estará apto a responder com segurança.
Agora vamos às variações de grau dos adjetivos: Comparativo e Superlativo.

Aula 04 – English General Presentation 6


www.estrategiamilitar.com.br 70
Teacher Andrea Belo
Aula 04: Escola Naval 2021

Adjetivos – Grau comparativo


Os adjetivos podem possuir graus de comparação, divididos em comparativos e superlativos.
Comparative sentences, frases com uso de comparativos, como o próprio nome já indica, tem
como tarefa, comparar dois ou mais substantivos usando um adjetivo para realizar tal comparação.
Essa relação de comparação pode ser de superioridade (quando um substantivo possui mais
de uma qualidade que o outro), pode ser de igualdade (quando ambos os substantivos possuem a
mesma quantidade de certa qualidade) ou pode ser também de inferioridade (quando um possui
menos daquela qualidade que o outro).
Veremos cada um dos tipos de comparação de adjetivos com exemplos, da mesma maneira
que as provas das Carreiras Militares utilizam para elaborar as questões da prova.
Você precisa saber identificá-los para que possa substitui-los por outros quando solicitado.
Ou quando se pede para encontrar a alternativa em que não há comparação ou até mesmo
responder se uma frase específica possui elementos comparativos ou não e assim por diante.

Comparativo de superioridade

Estudar frases comparativas em Inglês, que são muitas, fica simples se você pensar qual é o
real objetivo de uma comparação: estabelecer um paralelo entre uma coisa e outra.
Exatamente isso: usamos o grau comparativo para equiparar uma pessoa ou uma coisa com
outra, para comparar as diferenças entre os dois elementos que se modificam.
A estrutura usada nas frases em que há o comparativo de superioridade é essa: substantivo
1 (sujeito) + verbo + adjetivo comparativo + than (do que) + substantivo 2 (objeto).
Mas, depende do número de letras que cada adjetivo tem. Isso mesmo, se o adjetivo é curto
ou longo, interfere na formação das frases.
E existem outras regras a serem seguidas. Vamos estudar cada uma delas.
Para adjetivos de uma sílaba, ou seja, até 5 letras, com o som de uma só sílaba, considerados
“curtos”, como tall – alto/a, short – baixo/a e outros, acrescentamos a terminação -er para fazer a
frase comparativa:

Bethy is taller than Jane – Bethy é mais alta do que Jane.


.
Se o adjetivo terminar em CVC: consoante + vogal + consoante, como hot, big, sad e wet,
entre outros, a consoante final deve ser duplicada antes da incluir -er + than no final do adjetivo:

Summer is hotter than Spring – O verão é mais quente do que a primavera.

Aula 04 – English General Presentation 7


www.estrategiamilitar.com.br 70
Teacher Andrea Belo
Aula 04: Escola Naval 2021

Os adjetivos que possuem cinco ou mais letras, geralmente com duas sílabas, tais como
simple, clever, common e quiet (simples, inteligente, comum e quieto), formam o comparativo com
a adição de -er ou pelo uso da palavra more antes do adjetivo + than. Vale lembrar que ambas as
formas são utilizadas e estão corretas:

Bethy is simpler than Jane. – Bethy é mais simples do que Jane.


Bethy is more simple than Jane. – Bethy é mais simples do que Jane.

No caso de outros adjetivos, com mais de 6 letras e considerados longos, tais como intelligent,
interesting, beautiful, comfortable etc (inteligente, interessante, bonito/a, confortável etc), apenas
se faz a comparação acrescentando more antes do adjetivo + than:

Bethy is more intelligent than Jane. – Bethy é mais inteligente do que Jane.

Por sua vez, adjetivos terminados em y, com até 5 letras, tais como pretty, busy, noisy, happy
etc – lindo/a, ocupado/a, barulhento/a, feliz, e etc, seguem a regra dos outros adjetivos
considerados “curtos”.
Porém, elimina-se a letra y, que é automaticamente substituída pela letra i e, assim, faz-se o
acréscimo de -ier + than:

Bethy is prettier than Jane. – Bethy é mais linda do que Jane.

Por último, os adjetivos que, quando usados em frases comparativas, mudam


completamente. São chamados de adjetivos irregulares, tais como good, bad e far – bom, mau e
longe, não seguem regras e cada um tem uma grafia diferente quando a comparação é feita, veja:

GOOD: Bethy is better than Jane. – Bethy é melhor do que Jane.

BAD: Bethy is worse than Jane. – Bethy é pior do que Jane.

FAR: Bethy’s house is further than Jane’s one. – A casa de Bethy é mais longe
do que a casa de Jane.

Agora, vamos visualizar as formas de se fazer frases comparativas em Inglês, usando qualquer
adjetivo que aparecer em sua prova.

Aula 04 – English General Presentation 8


www.estrategiamilitar.com.br 70
Teacher Andrea Belo
Aula 04: Escola Naval 2021

Assim, você poderá identificar o grau comparativo nas frases retiradas dos textos e saberá
responder qualquer questão sobre esse assunto com esse esquema:

Bethy is taller than Jane. – Bethy é mais alta do que Jane.

Agora, vejamos o comparativo de igualdade.

Aula 04 – English General Presentation 9


www.estrategiamilitar.com.br 70
Teacher Andrea Belo
Aula 04: Escola Naval 2021

Comparativo de igualdade

O grau comparativo de igualdade é usado para comparar dois elementos equivalentes entre
si. Geralmente, a comparação é feita com apenas dois elementos, dois sujeitos em uma frase.
A estrutura usada nas frases em que há o comparativo de igualdade é essa: substantivo 1
(sujeito) + verbo + as + adjetivo + as + substantivo 2 (objeto).
Essa estrutura é igual para todos os adjetivos da língua inglesa, não depende do número de
letras ou sílabas que cada adjetivo possui.
Vejamos exemplos:

Bethy is as intelligent as Jane – Bethy é tão inteligente quanto Jane.


Bethy is as pretty as Jane – Bethy é tão linda quanto Jane.
Bethy is as good as Jane – Bethy é tão bondosa quanto Jane.

Anne is as tall as John – Anne é tão alta quanto John.

E agora, vamos estudar o comparativo de inferioridade. Vamos lá!

Aula 04 – English General Presentation 10


www.estrategiamilitar.com.br 70
Teacher Andrea Belo
Aula 04: Escola Naval 2021

Comparativo de inferioridade

Por sua vez, o grau comparativo de inferioridade é usado para comparar dois elementos,
evidenciando que o primeiro é inferior ao segundo.
A estrutura usada nas frases em que há o comparativo de inferioridade é essa: substantivo 1
(sujeito) + verbo + less + adjetivo + substantivo 2 (objeto).
Essa estrutura também é igual para todos os adjetivos da língua inglesa, não depende do
número de letras ou sílabas que cada adjetivo possui.
Vejamos exemplos:

Bethy is less tall than Jane – Bethy é menos alta do que Jane.
Bethy is less intelligent than Jane – Bethy é menos inteligente do que Jane.
Bethy is less good than Jane – Bethy é menos bondosa do que Jane.

Algumas pessoas preferem usar a negação no comparativo de inferioridade (que é apenas


usar o verbo to be na forma negativa com o acréscimo de not) ao invés do comparativo de
inferioridade, para a frase “soar melhor” e não enfatizar algo inferior, veja exemplos:

Bethy is not as tall as Jane. – Bethy não é tão alta quanto Jane.
Beth and Jane are not as tall as Susan. - Bethy e Jane não são tão altas quanto Susan.

Elizabeth is less tall than John – Anne é menos alta do que John.

Aula 04 – English General Presentation 11


www.estrategiamilitar.com.br 70
Teacher Andrea Belo
Aula 04: Escola Naval 2021

Adjetivos – Grau superlativo


Os superlativos têm a função de intensificar, de forma única, a qualidade de um substantivo
em relação a todos os outros de um mesmo segmento.
Por exemplo, vimos, no capítulo anterior, que dizer que alguém é mais inteligente, tão
inteligente quanto ou tem menos inteligência, temos que falar de outra pessoa para fazer a
comparação.
No superlativo, se diz que alguém é o mais inteligente de todos. E pronto!
Essa relação de intensidade pode ser de superioridade ou de inferioridade que vamos ver agora com
exemplos para ficar claro.

Superlativo de superioridade

A estrutura usada nas frases em que há o superlativo de superioridade é essa: substantivo +


verbo + artigo THE + adjetivo no superlativo (veremos as regras).
Assim como no modo comparativo, analisando o número de letras/sílabas dos adjetivos, o
superlativo é mais simples, mas a formação de frases é diferente para o adjetivo curto ou longo.
Para os adjetivos curtos – de uma sílaba, até 5 letras – como tall – alto/a, short – baixo/a e
outros considerados “curtos”, acrescenta-se a terminação -est no final:

Bethy is the tallest girl in the class. – Bethy é a garota mais alta da sala.
.

Note que o artigo the é muito importante no superlativo, pois proporcionam ênfase de que
algo/alguém é o mais/a mais sobre o assunto tratado na frase.
Nos adjetivos longos, com mais de 5 letras e mais sílabas, tais como beautiful, interesting,
important etc, acrescentamos o termo the most antes do adjetivo.

Bethy is the most beautiful girl in the class. – O verão é a garota mais bonita da sala.

Por sua vez, adjetivos terminados em y, com até 5 letras, tais como pretty, busy, noisy, happy
etc – lindo/a, ocupado/a, barulhento/a, feliz, e etc, seguem a regra dos outros adjetivos
considerados “curtos” mas elimina-se a letra y, que é substituída pela letra i e, assim, faz-se o
acréscimo de -iest:

Bethy is the prettiest girl in the class. – Bethy é a garota mais linda da sala.

Aula 04 – English General Presentation 12


www.estrategiamilitar.com.br 70
Teacher Andrea Belo
Aula 04: Escola Naval 2021

Os adjetivos que terminam em CVC (consoante/vogal/consoante) apenas seguem regras de -


est nos curtos e the most nos longos, da mesma maneira, dobrando as letras:

Bethy is the thinnest girl in the class. – Bethy é a garota mais magra da sala.

Por último, os adjetivos irregulares que vimos no comparativo, tais como good, bad e far –
bom, mau e longe, também não seguem regras e cada um tem uma grafia diferente quando a
comparação é feita – the best, the worst e the furthest, veja:

GOOD: Bethy is the best student. – Bethy é a melhor aluna. (de todas)

BAD: Bethy is the worst student. – Bethy é a pior aluna. (de todas)

FAR: Bethy’s house is the furthest in this city. – A casa de Bethy é a mais longe nessa
cidade.

Vejamos agora como se usa o superlativo de inferioridade.

Superlativo de inferioridade

A estrutura usada nas frases em que há o superlativo de superioridade é essa: substantivo +


verbo + artigo THE LEAST + adjetivo – todos os adjetivos são usados com the least antes. A regra é
uma só para todos. Para adjetivos longos, curtos, com CVC ou com final y.

Bethy is the least thin student. – Bethy é a aluna menos magra. (de todas)

Bethy is the least intelligent student. – Bethy é a aluna menos inteligente. (de todas)

Bethy is the least funny student. – Bethy é a aluna menos engraçada. (de todas)

Agora, que falamos de forma abrangente sobre adjetivos, estudaremos a classe de advérbios
detalhadamente e com muitos exemplos de exercícios para fixar mais um conteúdo importante a
você para o dia da sua prova.
Vamos lá! Come on!

Aula 04 – English General Presentation 13


www.estrategiamilitar.com.br 70
Teacher Andrea Belo
Aula 04: Escola Naval 2021

Advérbios
Os advérbios têm quase a mesma função dos adjetivos, contudo, ao invés de agregar
características a substantivos, os advérbios viabilizam qualidades de modo, tempo e lugar aos verbos
e, algumas vezes, aos próprios adjetivos!
Portanto, os advérbios funcionam como modificador de verbos, de adjetivos e, muitas vezes,
de outros advérbios, usados para dizer quando, como ou onde alguma coisa aconteceu e,
geralmente, aparece depois de verbos principais, como veremos.
Vejamos como se dá a formação dos advérbios para ficar mais fácil falar das classificações
deles adiante.
Você deve compreender muito bem para não confundir, já que os advérbios são derivados de
adjetivos e, algumas vezes, possuem a mesma forma do adjetivo.
A maioria dos advérbios são formados pelo acréscimo da terminação -ly, que significa -mente.
Este acréscimo ocorre nos advérbios que indicam modo, que indicam frequência ou intensidade,
entre muitos outros.

Para advérbios de qualquer classificação, acrescenta-se -ly no final dos adjetivos. Para
adjetivos terminados pelo próprio y, trocamos o y por i e acrescentamos normalmente -ly.
Veja alguns exemplos:

crazy (louco) - crazily (loucamente)


easy (fácil) - easily (facilmente)
happy (feliz, alegre) - happily (felizmente, alegremente)
heavy (pesado) - heavily (pesadamente)
lucky (sortudo) - luckily (afortunadamente)

Aula 04 – English General Presentation 14


www.estrategiamilitar.com.br 70
Teacher Andrea Belo
Aula 04: Escola Naval 2021

Para os adjetivos terminados em -le, trocando-se o -le por -ly, forma-se o advérbio:

horrible (horrível) - horribly (horrivelmente)


incredible (inacreditável) – incredibly (inacreditavelmente)
probable (provável) - probably (provavelmente)
simple (simples) - simply (simplesmente)
subtle (sutil) - subtly (sutilmente)

Com alguns adjetivos terminados em -e (sem a letra L antes da letra E), mantemos o -e,
acrescentando, normalmente -ly; com a exceção de dois adjetivos especiais nesse caso: true e due:

brave (bravo) - bravely (bravamente)


immediate (imediato) - immediately (imediatamente)
Exceções: true (verdadeiro) - truly (verdadeiramente)
due (que se deve, adequado) - duly (pontualmente, a tempo)

Quando os adjetivos terminam em -ic acrescentam -ally e não somente -ly, veja:

automatic (automático) - automatically (automaticamente)


romantic (romântico) - romantically (romanticamente)
specific (específico) - specifically (especificamente)
tragic (trágico) - tragically (tragicamente)

De forma simples e direta, caso o adjetivo já termine em -ly, nada se acrescenta na formação
do advérbio:

Bethy is tired of her daily routine. (Justine está cansada da sua rotina diária.)
Bethy helps her sister friendly. (Bethy ajuda sua irmã amigavelmente.)

Aula 04 – English General Presentation 15


www.estrategiamilitar.com.br 70
Teacher Andrea Belo
Aula 04: Escola Naval 2021

Alguns advérbios são usados com a grafia igual aos adjetivos, ou seja, não acrescentamos
nenhuma terminação nem modificamos quaisquer letras. Mas lembre-se de que o sentido muda.
Enquanto o adjetivo bonito – beautiful qualifica o sujeito, o advérbio beautiful indica o modo que o
sujeito executou alguma ação, veja:

Bethy is beautiful. She sings beautiful. (Bethy é bonita. Ela canta bonito – de forma bonita)

Além de beautiful, outros exemplos em que adjetivos e advérbios ficam iguais nas frases em
ambas funções são fast, hard, right e late. Vejamos frases:

ADJETIVOS ADVÉRBIOS

Bethy is a fast swimmer. Bethy swims fast. Bethy ´s uma nadadora rápida.
Bethy nada rápido.
Bethy is a hard worker. Bethy works hard. Bethy é uma trabalhadora esforçada.
Bethy trabalha duro/com esforço.
Bethy is always right. Bethy thinks right. Bethy é/está sempre certa.
Bethy pensa certo.
Bethy is late. Bethy wakes up late sometimes. Bethy é/está atrasada.
Bethy acorda atrasada às vezes.

Alguns desses advérbios, que coincidem com os adjetivos, quando se coloca a terminação -ly,
muda-se o sentido da palavra e também o significado, como por exemplo hardly e lately:

Bethy works hard. Bethy trabalha duro/com esforço.


Bethy’s sick. She can hardly work. Bethy está doente. Ela mal consegue trabalhar.

Bethy is late. Bethy é/está atrasada.


Bethy’s sick. She didn’t work lately. Bethy está doente. Ela não trabalhou recentemente.

Aula 04 – English General Presentation 16


www.estrategiamilitar.com.br 70
Teacher Andrea Belo
Aula 04: Escola Naval 2021

Outra importante observação é que nem todas as palavras terminadas em -ly são advérbios.
Algumas são adjetivos, acredita? Apesar da terminação -ly ser comumente usada em advérbios e ser
uma característica particular deles, lonely, por exemplo, é o adjetivo solitário.
Vejamos outros:

Adjetivo lovely: adorável – Bethy is a lovely person. (Bethy é uma pessoa adorável).
Adjetivo friendly: amigável – Bethy is friendly with everybody. (Bethy é amigável com todos).
Adjetivo elderly: velho/idoso – Bethy’s mom is elderly. (A mãe de Bethy é uma idosa).

Um advérbio especial e diferente, que não usa -ly em sua grafia, é well, referente ao adjetivo
good. Uma pessoa pode ser boa/bondosa (como uma qualidade) ou pode ser boa em alguma coisa
que faz com dedicação, demonstrar uma habilidade.
Em Português, usamos apenas uma palavra para representar ambos, “bom de bondoso” e
“bom em algo que faça”. Mas em Inglês, veja:

Bethy is a good swimmer – Bethy é uma boa nadadora.


Bethy swims very well – Bethy nada muito bem. (é boa em natação).

Já vimos como formar os advérbios e, a maior parte deles sofre o acréscimo de -ly. Para
colocar os advérbios em uma frase, o que é necessário? Quando são usados? Vejamos.
Um advérbio pode ser usado para descrever um verbo, descrevendo quando, por que, onde
ou como uma ação ocorreu. Por exemplo, uma pessoa pode agir rapidamente, calmamente ou
silenciosamente. Na seguinte frase: Bethy falou ao telefone – Bethy talked on the phone, o verbo
falar pode ser modificado por um advérbio como lentamente. Ou, cuidadosamente:

Bethy talked on the phone slowly – Bethy falou ao telephone lentamente.


Bethy talked on the phone carefully – Bethy falou ao telephone cuidadosamente.

Os advérbios são inúmeros. Eles podem ser de modo, de lugar, de tempo, de frequência, de
intensidade, de certeza ou dúvida e, ainda existem as famosas locuções adverbiais. Eles aparecem
em diversas situações e enriquecem frases em geral.
Vamos, agora, analisar os tipos de advérbios que existem para que as explicações anteriores
sejam melhor compreendidas com exemplos de cada um com detalhes.

Aula 04 – English General Presentation 17


www.estrategiamilitar.com.br 70
Teacher Andrea Belo
Aula 04: Escola Naval 2021

Advérbios de modo

Como o próprio nome já nos diz, os adverbs of manner (advérbios de modo), indicam a
maneira em que uma ação ocorreu.
Os advérbios são formados a partir de adjetivos, em que se acrescenta o sufixo -ly e assim
deixam de ser adjetivos, passando a ser advérbios, como já vimos anteriormente.
Veja as comparações para que os advérbios de modo fiquem claros:

Bethy is bad (Bethy é má) – Bethy sings badly (Bethy canta mal)
Beth is slow (Bethy é lenta/devagar). - Bethy sings slowly. (Bethy canta vagarosamente).

Os advérbios de modo são bastante flexíveis e, normalmente, podem aparecer em três posições:
1 – Antes do sujeito:
Quickly Bethy organized her books – Rapidamente Bethy organizou seus livros.
2 – Entre o sujeito e o verbo:
Bethy quickly organized her books – Bethy rapidamente organizou seus livros.
3 – Após o verbo ou o objeto:
Bethy organized her books quickly – Bethy organizou seus livros rapidamente.

The girl organized the books quickly.

Aula 04 – English General Presentation 18


www.estrategiamilitar.com.br 70
Teacher Andrea Belo
Aula 04: Escola Naval 2021

Advérbios de frequência

Os adverbs of frequency (advérbios de frequência) são, de fato, os mais conhecidos do Inglês.


Eles são apenas palavras utilizadas para descrever com que frequência alguma atividade é
realizada, como sempre, às vezes, nuca, entre outros.
Essa frequência pode ser demonstrada pelos advérbios: daily (diariamente), weekly
(semanalmente), monthly (mensalmente), yearly (anualmente).

Bethy goes to the market weekly – Bethy vai ao mercado semanalmente)


Bethy reads her 3 favorite books yearly – Bethy lê seus 3 livros favoritos anualmente)

As formas mais comuns de se ver advérbios de frequência em frases é quando podemos


representar a frequência por porcentagem, como alguns livros fazem – 100% para always (sempre),
90% para usually e often (frequentemente), 50% para sometimes (às vezes), 30% para occasionally
(ocasionalmente/eventualmente), 10% ou menos para rarely e seldom (raramente) e 0% para never
(nunca). Vejamos em frases:

Bethy always talks on the phone – Bethy sempre fala ao telefone.


Bethy usually talks on the phone – Bethy frequentemente fala ao telefone.
Bethy sometimes talks on the phone – Bethy às vezes fala ao telefone.
Bethy ocasionally talks on the phone – Bethy eventualmente fala ao telefone.
Bethy rarely talks on the phone – Bethy raramente fala ao telefone.
Bethy never talks on the phone – Bethy nunca fala ao telefone.

Como você percebeu, os advérbios de frequência estão antes de um verbo principal. Mas,
eles podem variar a posição em que se encontram e não prejudicar o significado nem o sentido das
frases. Vejamos as frases acima, com troca de posições.

Bethy talks on the phone, always – Bethy fala ao telefone, sempre.


Usually, Bethy talks on the phone – Frequentemente, Bethy fala ao telefone.
Sometimes, Bethy talks on the phone – Às vezes, Bethy fala ao telefone.

Agora, vamos analisar os advérbios de tempo e como usá-los.

Aula 04 – English General Presentation 19


www.estrategiamilitar.com.br 70
Teacher Andrea Belo
Aula 04: Escola Naval 2021

Advérbios de tempo

Os adverbs of time (advérbios de tempo) indicam quando uma ação ocorreu e não precisam
da terminação -ly.
Geralmente, demonstram e caracterizam algo que aconteceu no tempo passado ou no tempo
futuro, enfatizando frases tanto no Past Simple como do tempo Future Simple.
Os advérbios de tempo, na maioria das vezes, aparecem no final das frases. Alguns deles são:

last week / last month / last year = semana passada / mês passado / ano passado.
next week / next month / next year = semana que vem / mês que vem / ano que vem.
OU
próxima semana, próximo mês, próximo ano.

She traveled last week. She will travel next week.


Ela viajou semana passada. Ela vai viajar na próxima semana.

Aula 04 – English General Presentation 20


www.estrategiamilitar.com.br 70
Teacher Andrea Belo
Aula 04: Escola Naval 2021

Advérbios de lugar

Os adverbs of place (advérbios de lugar) indicam, como o nome diz, a localização onde uma
ação aconteceu e eles não precisam da terminação -ly.
Podem aparecer após o verbo, na frente das orações, antes do sujeito, entre outras posições.
Vejamos exemplos pois há muitos casos e vou explicando a você, um a um.

Bethy walks everywhere – Bethy anda por todos os lugares/por toda parte.

Os advérbios here e there (aqui e lá) são muito comuns em frases para indicar lugar.

Come here! (Venha aqui!), para dizer “Venha até mim.”


The table is in here. (A mesa está aqui), para dizer “Quero mostrar a você que a mesa está aqui.”
Put it there. (Coloque isso lá!), para dizer “Leve isso para lá, para longe daqui.”
The table is in there. (A mesa está lá.) para dizer “Pode ir, a mesa já está lá.”

Here you are! I was looking for you!


(Aqui está você! Estava procurando você!)

I didn’t realize you were there. I was looking for you.


(Não percebi que você estava lá. Estava procurando você!)

Aula 04 – English General Presentation 21


www.estrategiamilitar.com.br 70
Teacher Andrea Belo
Aula 04: Escola Naval 2021

Alguns advérbios são, ao mesmo tempo, advérbios e preposições. Vejamos exemplos:

Advérbio usado como advérbio: Advérbio usado como preposição:

around The necklace rolled around in my fingers. I am wearing a necklace around my neck.
O colar se enrolou entre os meus dedos. Estou usando um colar em volta do pescoço.

behind Hurry! You are getting behind. Let's hide behind the door.
Depressa! Você está ficando para trás. Vamos nos esconder atrás da porta.

down Bethy fell down. Bethy made her way down the hill.
Bethy caiu. (para baixo) Bethy fez o caminho morro abaixo.

in We decided to drop in on Bethy. I dropped the letter in the box.


Decidimos visitar Bethy sem avisar. Eu coloquei a carta dentro da caixa.

off Let's get off at the next stop. She put the flowers off the vase.
Vamos descer na próxima parada. Ela colocou as flores fora do vaso.

on She rode on for two more hours. Put the books on the table.
Ela rodou por mais duas horas. Coloque os livros na mesa.

over She turned over and went back to sleep. I think I will hang the picture over my table.
Ela se virou e voltou a dormir. Acho que vou colocar o quadro acima da mesa.

Há advérbios de lugar que terminam em -where, para indicar um lugar específico, como:

I would like to go somewhere hot. (Eu gostaria de ir em algum lugar quente.)


Is there anywhere I can find a drugstore? (Há algum lugar em que eu encontre uma drogaria?
I have nowhere to go. (Eu não tenho lugar nenhum para ir.)
I keep looking for you everywhere! (Eu continuo procurando por você em todos os lugares.)

Aula 04 – English General Presentation 22


www.estrategiamilitar.com.br 70
Teacher Andrea Belo
Aula 04: Escola Naval 2021

Outros advérbios de lugar terminam em -wards, para expressar movimentos em um lugar em


particular, um lugar, uma direção específica:

Cats don't usually walk backwards. (Gatos geralmente não andam para trás.)

The ship sailed westwards. (O navio foi para o oeste/direção ocidental.)

The balloon drifted upwards. (O balão foi levado para cima.)

Let’s walk homewards until we arrive. (Vamos andar de volta para casa.)

The balloon drifted upwards.


O balão foi levado para cima.

Agora, vamos estudar alguns advérbios classificados como de dúvida ou de certeza.

Advérbios de dúvida e/ou certeza

Os nomes dos advérbios já indicam, em todos os subtítulos, sobre o que se tratam. Assim, os adverbs
of doubt or certainty (advérbios de dúvida ou certeza), vão justamente indicar o grau de dúvida ou
de certeza de algo que aconteceu, acontece ou acontecerá.
Esses advérbios aparecem com a terminação -ly. Exemplos:

Aula 04 – English General Presentation 23


www.estrategiamilitar.com.br 70
Teacher Andrea Belo
Aula 04: Escola Naval 2021

Bethy maybe dance tonight – Bethy talvez dance essa noite.


Bethy will clearly dance tonight – Bethy vai, sem dúvida, dançar essa noite.

Além de maybe, outros advérbios nessa categoria:

assuredly (indubitavelmente, sem dúvidas)

certainly (certamente, seguramente, evidentemente)

clearly (claramente, sem dúvidas, evidentemente)

definitely (definitivamente)

maybe (talvez)

perhaps (talvez - no início ou no final da frase)

possibly (possivelmente)

probably (provavelmente)

Entre os advérbios de dúvida e certeza, estão os que demonstram um ponto de vista. São os
advérbios que expressam a opinião do sujeito, o que ele pensa sobre a situação da frase. Esses
advérbios, por sua vez, também aparecem com a terminação -ly, comum na classe adverbial.

Bethy unluckily lost the game – Bethy por azar perdeu o jogo.

O sujeito da frase está expressando, através do advérbio unluckly, a ideia da falta de sorte de
Bethy em relação ao jogo. E, assim, a tradução do advérbio é “por azar”.

Aula 04 – English General Presentation 24


www.estrategiamilitar.com.br 70
Teacher Andrea Belo
Aula 04: Escola Naval 2021

Outros advérbios que evidenciam pontos de personally (pessoalmente)


vista: presumably (presumivelmente)
bravely (corajosamente)
rightly (com razão)
carelessly (de forma desleixada, negligente) seriously (seriamente)
cleverly (inteligentemente) stupidly (estupidamente, de modo imbecil)
confidentially (confidencialmente) theoretically (teoricamente)
disappointingly (de modo decepcionante, truthfully (na verdade)
desapontador)
unbelievably (inacreditavelmente)
foolishly (de forma tola, insensata)
unfortunately (infelizmente)
happily (por sorte, felizmente)
unluckily (por azar)
kindly (gentilmente, de bom grado)
wisely (sabiamente)
luckily (por sorte)
presumably (presumivelmente)
naturally (naturalmente)
technically (tecnicamente)
obviously (obviamente)

Já que esses advérbios exibem pontos de vista, a característica principal deles é que seja um
comentário com caráter opinativo.
Eles modificam a oração inteira, e não apenas palavras isoladamente:

Fortunately, Bethy decided to help us.


(Felizmente, Bethy decidiu nos ajudar), em que fortunately: ponto de vista.
Stupidly, I forgot my keys.
(Estupidamente/de forma estúpida, eu esqueci minhas chaves), em que stupidly: ponto de vista.

Fortunately, my friend decided to help me when I fell down in front of everybody.


(Felizmente, meu amigo decidiu me ajudar quando caí em frente todos)

Aula 04 – English General Presentation 25


www.estrategiamilitar.com.br 70
Teacher Andrea Belo
Aula 04: Escola Naval 2021

Advérbios de intensidade

De uma forma geral, os adjetivos e advérbios possuem funções parecidas. Mas, enquanto os
adjetivos tratam de substantivos (como pessoas e coisas são), os advérbios tratam, quase sempre,
dos verbos (como as ações são feitas).
No entanto, é possível utilizar os advérbios para modificar os adjetivos. São os chamados
modifiers (modificadores) e são utilizados antes dos adjetivos para enfatizar a intensidade em
relação àquela qualidade:

absolutely [absolutamente]
extremely [extremamente]
really [realmente]
so [tão]
such [tão]
too/very [muito, demais]

This cake was very delicious but extremely expensive.


Esse bolo estava muito delicioso, mas extremamente caro.

Os advérbios intensificadores, além de permitir que o sujeito se expresse com mais clareza,
também proporciona às frases uma sensação mais natural.
Esses advérbios são uma ferramenta importante, já que são uma forma essencial de mostrar
emoção ou alcance de uma emoção.
Esses advérbios modificadores de frases, fornecem contexto emocional a elas e fortalecem
os seus significados, já que diz que algo foi feito extremante, incrivelmente etc.
Alguns desses advérbios podem aumentam a intensidade negativamente também. As frases
soam negativas, mas elas existem:

She made the report awfully. Ela fez o relatório terrivelmente.

Aula 04 – English General Presentation 26


www.estrategiamilitar.com.br 70
Teacher Andrea Belo
Aula 04: Escola Naval 2021

Esses modificadores, por sua vez, dão força às palavras que elas modificam, só que com
conotação negativa, para intencionalmente, dar a ideia de gravidade em algo que talvez poderia ser
entendido como normal ou cotidiano.

Bethy is dreadfully sorry. (Bethy está terrivelmente arrependida)


Perceba que, nesse exemplo, o advérbio de intensidade reforça a ideia de que quer
fazer um pedido de desculpas sobre algo que aconteceu. Com o acréscimo do advérbio
de intensidade dreadfully, a frase ficou muito mais forte e impactante.

Voltando a falar de advérbios de intensidade em geral, veja outros que já apareceram e podem
aparecer novamente em textos da maioria das provas:

She is remarkable the champion of the competition.


(Ela é notavelmente a campeã da competição)

Outros intensificadores comuns:

amazingly (surpreendentemente)
at all (em absoluto)
especially (especialmente)
extraordinarily (extraordinariamente)
outrageously (escandalosamente)
phenomenally (fenomenalmente)
remarkably (notavelmente)
terribly (terrivelmente)
totally (totalmente)
unusually (incomumente)

Aula 04 – English General Presentation 27


www.estrategiamilitar.com.br 70
Teacher Andrea Belo
Aula 04: Escola Naval 2021

Locuções adverbiais

O que é uma locução adverbial? Locução adverbial é um conjunto de duas ou mais palavras que
desempenham a função de advérbio.
Essas locuções adverbiais se formam com uma preposição, às vezes uma preposição + substantivo
ou preposição + adjetivo ou ainda preposição + advérbio.
As locuções adverbiais são expressões em que, quando o conjunto de duas ou mais palavras são
agrupadas, desempenham a função de um advérbio e pode alterar o sentido desse verbo, ou de um
adjetivo ou até mesmo de outro advérbio.
Vejamos exemplos:

Algumas locuções adverbiais que aparecem again and again – repedidamente


nos textos das provas: arm in arm – de braços dados
a só/a sós - lonely at first – a principio
à toa - occasionally at most – no máximo
à vontade - at will, freely at once – imediatamente
absolutamente – not at all at random – ao acaso
ao acaso - without consideration by and large – em geral
ao contrário - in contrary day by day – dia após dia
às avessas - just the opposite fairly well – razoavelmente
às claras - openly, directly far and near/far and wide – em toda parte
às direitas - straightforward from now on – de hoje em diante
às pressas - fast hand in hand -de mãos dadas
de bom grado - of good will hardly ever – quase nunca
de cor - by heart head over hills – de cabeça para baixo
de fato, na verdade - in fact in full – por extenso
de má vontade - unwillingly likely – muito provável
é claro - of course little by little – pouco a pouco
em geral - generally once in a while – de vez em quando
em silêncio - silently sooner or late – mais cedo ou mais tarde
por via das dúvidas - just in case through and through- por complete
sem dúvida - no doubt to a certain extent – até certo ponto

Aula 04 – English General Presentation 28


www.estrategiamilitar.com.br 70
Teacher Andrea Belo
Aula 04: Escola Naval 2021

Advérbios: Grau comparativo


Assim como os adjetivos, os advérbios possuem grau comparativo e superlativo. Vejamos,
primeiramente, como se faz o grau comparativo, mas, desde já digo a você: é mais simples nos
advérbios do que nos adjetivos – e que bom, então, certo?
Comparativo de Igualdade

O comparative of equality – comparativo de igualdade nos advérbios tem a estrutura seguinte: as +


advérbio + as, significando: tanto/tão...quanto/como. Veja:

Bethy drives as carefully as Tom. – Bethy dirige tão cuidadosamente quanto Tom.
.
Does Bethy study as frequently as Tom? – A Bethy estuda tão frequentemente quanto Tom?

Na forma negativa, apenas acrescenta-se not, ficando: not as/not so + advérbio + as, significando –
não tão...como/quanto:

Bethy is not as carefully as Tom. – Bethy não é tão cuidadosamente quanto Tom.
.

E, para frases em outros tempos verbais, sem utilizar o verbo to be, conjuga-se a forma negativa
normalmente – como você aprendeu na aula de verbos – e a estrutura as/as:

Bethy doesn’t drive as carefully as Tom. – Bethy não dirige tão cuidadosamente quanto Tom.
.

Aula 04 – English General Presentation 29


www.estrategiamilitar.com.br 70
Teacher Andrea Belo
Aula 04: Escola Naval 2021

Comparativo de superioridade

O comparative of superiority – comparativo de superioridade tem diferentes estruturas quando


elaborado com advérbios considerados curtos ou longos de acordo com as letras/sílabas, lembra?
Eles têm esse formato: more + advérbio + than, significando: mais/do que. Veja:

Bethy travels more frequently than Tom. – Bethy viaja mais frequentemente do que Tom.
.

Does Bethy study more frequently than Tom? - Bethy estuda mais frequentemente do que Tom?

Na forma negativa, apenas acrescenta-se not no verbo to be ou conjuga-se a forma negativa do


Present Simple normalmente – como você aprendeu na aula de verbos:

Bethy doesn’t swim more easily than Tom. – Bethy não nada mais facilmente do que Tom.
.

Se o advérbio for um daqueles que fica igual, idêntico ao adjetivo em sua estrutura, como vimos o
adjetivo e o advérbio alto – high, por exemplo, segue a regra do adjetivo: -er no final do advérbio +
than:

Peter talks higher than Tom. – Peter fala mais alto do que Tom.
.

Aula 04 – English General Presentation 30


www.estrategiamilitar.com.br 70
Teacher Andrea Belo
Aula 04: Escola Naval 2021

Comparativo de inferioridade

O comparative of inferiority – também tem diferentes estruturas quando elaborado com


advérbios considerados curtos ou longos de acordo com as letras/sílabas, lembra?
Eles têm esse formato: less + advérbio + than, significando: mais/do que. Veja:

Bethy travels less frequently than Tom. – Bethy viaja menos frequentemente do que Tom.
.

Does Bethy study less frequently than Tom? - Bethy estuda menos frequentemente do que Tom?

Na forma negativa, apenas acrescenta-se not no verbo to be ou conjuga-se a forma negativa


do Present Simple normalmente – como você aprendeu na aula de verbos:

Bethy doesn’t swim less easily than Tom. – Bethy não nada menos facilmente do que Tom.
.
As traduções ficam “estranhas”, não ficam? Por esse motivo, prefere-se usar o comparativo
de igualdade na forma negativa para, ao invés de “desmerecer” algo ou alguém, se diz que
algo/alguém não pratica uma ação tão bem quanto outro.
Assim, fica mais “educado” e mais adequado. Veja a última comparação acima, usada com
comparativo de igualdade no lugar de inferioridade:

Bethy doesn’t swim as easily as Jane. – Bethy não nada tão facilmente quanto Jane.
.

Vejamos agora, o grau superlativo nos advérbios.

Aula 04 – English General Presentation 31


www.estrategiamilitar.com.br 70
Teacher Andrea Belo
Aula 04: Escola Naval 2021

Advérbios: Grau Superlativo


O grau superlativo é usado para demonstrar que algo dentro de um grupo se destaca, isto é, alcança
o grau máximo no aspecto em que é comparado.
No caso do grau superlativo para advérbios, temos as estruturas que usam expressões tais como
“o(a) mais” e também “o (a) menos”, para superioridade ou inferioridade.

Superlativo de superioridade

Para o superlativo de superioridade, usamos the most + advérbio ou -est para advérbios curtos, que
coincidem com os adjetivos, aqueles em que se usa a mesma palavra para ambas classes gramaticais
como vimos antes, lembra?
Vejamos exemplos de ambos casos:

Bethy does her homework the most quickly. – Bethy é a faz a tarefa de casa mais rapidamente.
(que todos).
.
Bethy talks the loudest in her classroom. – Bethy é a que fala mais alto em sua sala.
.
Em ambas frases, percebemos que poderíamos traduzir assim: “mais (...) de todos”, porque o
superlativo é justamente isso: mostrar que algo/alguém é mais, o máximo em alguma coisa,
incomparável a outros.

She talks the loudest in her classroom. – Ela é a que fala mais alto em sua sala.

Aula 04 – English General Presentation 32


www.estrategiamilitar.com.br 70
Teacher Andrea Belo
Aula 04: Escola Naval 2021

Superlativo de inferioridade

Para o superlativo de inferioridade dos advérbios, no lugar de the most, usa-se the least antes do
advérbio da frase.

Bethy drives the least carefully. – Bethy é a que dirige menos cuidadosamente.
.
No superlativo de advérbios, há algumas formas comparativas e superlativas irregulares,
cujos advérbios não seguem as regras apresentadas. Os exemplos que aparecem nas provas são the
best, the worst e, o que usamos, the furthest (o melhor, o pior e o mais longe) – sempre com o
sentido de ser “o mais alguma coisa de todos.”

Vimos no comparativo. Agora, vejamos no superlativo:

I walked seven blocks, my friends walked two, some of them four but I walked the furthest.
Eu caminhei sete quarteirões, meus amigos caminharam dois, alguns quatro
quarteirões, mas eu (fui o que) caminhei mais longe. (significando mais longe de todos).

She runs the furthest on the competition. – Ela é a que corre mais longe na competição.

Agora, teremos uma questão inédita que elaborei, similar a muitas instituições das Carreiras
Militares para você, abordando os assuntos que foram explorados nessa aula. Logo depois, os
exercícios de anos anteriores para você treinar. Vamos lá!

Aula 04 – English General Presentation 33


www.estrategiamilitar.com.br 70
Teacher Andrea Belo
Aula 04: Escola Naval 2021

Questão inédita

Questão inédita Teacher Andrea Belo

Questão 01 (Inédita – Teacher Andrea Belo)

De acordo com o texto, podemos afirmar que:

A ( ) Mais de 8,8% do candidatos políticos na Índia eram mulheres, de acordo com análises.

B ( ) De acordo com análises, bons candidatos na Índia oferecem festas antes das eleições.

C ( ) Na Índia, uma realidade perplexa é mostrada no texto e não aceita na política.

D ( ) Candidatos indianos passam por uma realidade perplexa, ambos sexos e qualquer idade.

E ( ) Apenas 8,8% de candidatos políticos na Índia eram mulheres, de acordo com análises.

Comentários:

O trecho do texto acima, retirado do jornal New York Times, uma fonte muito usada em provas
das Carreiras Militares, indica o assunto a partir das letras em negrito na lateral, em que há
palavras cognatas e mostra que se trata das eleições na Índia, falando especificamente das
candidatas mulheres: “In India’s election, women in...”
O trecho é curto e assim, fica possível ler e analisar cada alternativa para encontrar aquela que
melhor se encaixa com as informações oferecidas através da leitura.

Aula 04 – English General Presentation 34


www.estrategiamilitar.com.br 70
Teacher Andrea Belo
Aula 04: Escola Naval 2021

Na letra A, afirma-se que mais de 8,8% dos candidatos políticos eram mulheres de acordo com
a pesquisa que foi realizada. Porém, nas primeiras duas linhas do trecho apresentado, onde há
o número 8,8%, encontramos o advérbio only (apenas/somente), declarando que apenas 8,8%
são mulheres e não mais de 8,8%. Se fossem mais de 8, 8%, apareceria o termo de comparação
more than, que não está presente no texto. Alternativa errada.

Na letra B, afirma-se que bons candidatos oferecem festas antes das eleições. Além de não
aparecer o adjetivo good, que seria bom, para qualificar os candidatos, a palavra party, no
plural – parties – pode ser traduzida como festas ou partidos. Como o assunto é política, parties
quer dizer partidos políticos e não festas no texto. Alternativa errada.

Na letra C, afirma-se que uma realidade perplexa é mostrada e não aceita na política. O adjetivo
perplexa – perplexing – está presente no texto, porém, não há a informação de que essa
realidade não é aceita. Alternativa errada.

Na letra D, afirma-se que candidatos indianos passam por uma realidade perplexa, ambos
sexos e qualquer idade, mas, você já pode eliminar essa alternativa como resposta porque não
se fala de idade nem de sexo no texto. Alternativa descartada.

A letra E, que é a alternativa correta, afirmando que apenas 8,8% de candidatos políticos na
Índia eram mulheres, de acordo com análises. E vimos, na análise da letra A, que o advérbio
only significa apenas/somente. Alternativa correta.

Vamos continuar analisando outras questões, agora, questões diversas das Carreiras Militares
de anos anteriores para praticar. Let’s go! Let’s study!

Aula 04 – English General Presentation 35


www.estrategiamilitar.com.br 70
Teacher Andrea Belo
Aula 04: Escola Naval 2021

Questões de anos anteriores para prática

Vamos começar com uma questão ESCOLA NAVAL e, em seguida, outros de diferentes
instituições para treinar palavras em Inglês e praticar como eliminar alternativas. Let’s go!

Questão 01 – Escola Naval / 2018


Doctors Know Best
By Ted Spiker

Along with all the disease stomping, heart reviving, baby delivering, and overall people
healing they do, doctors have another full-time job: keeping themselves healthy. Scratch that
- keeping themselves healthiest. So instead of peeking into their medical practices, we looked
at what they actually practice - in their own lives. Use personal strategies and insider tips
from the best medical pros to supercharge your health this year.
( I)-______
"As soon as I feel an illness coming on, I go to sleep for at least nine hours," says Hilda
Hutcherson, MD, clinical professor of ob-gyn at Columbia University Medical Center. "I also lie
on the floor with my legs elevated and propped against the wall and breathe deeply for five
minutes." It helps lower stress, which weakens the immune system.
(II )-______
Instead of having a garden-variety green salad, Margaret McKenzie, MD, assistant professor
of surgery at the Cleveland Clinic, tosses napa cabbage, radicchio, edamame, and carrots with
ginger-soy dressing. "It gives me a lot of vitamins, antioxidants, and protein and makes me
feel full," she says.
(III)-______
[...] Gary Small, MD, professor of psychiatry and biobehavioral sciences at the University of
California, Los Angeles, and author of The Alzheimer"s Prevention Program, plays Scrabble
and Words With Friends on his smartphone most days. These word games are perfect brain
boosters, because they build not only verbal and math skills but also spatial abilities as you
position letters to create words. "Combining several mental tasks strengthens multiple neural
circuits," Dr. Small says. "It"s like cross-training for your brain."
(IV) - _____
Make your bedroom spalike: Dim the lights at least an hour before you go to bed; ban cell
phones, laptops, and the TV; ask your partner for a foot rub. "I do deep breathing exercises,"
Dr. Hutcherson says. "Sometimes I play relaxing music softly."
(V) - _____

Aula 04 – English General Presentation 36


www.estrategiamilitar.com.br 70
Teacher Andrea Belo
Aula 04: Escola Naval 2021

The most important meal is breakfast, says David Katz, MD, director and founder of Yale-
Griffin Prevention Research Center in Derby, Connecticut. He often has two breakfasts,
divvying up his morning meal so that he eats half before his workout and half after. "It helps
with portion control, and it establishes a daily eating pattern," Dr. Katz says. Plan your
breakfast at night to start the next day on a healthy note.
(Abridged from https ://www.fitnessmagazine.com/health/doctors-tips-tostay-healthy/)
QUESTÃO 01
The headlines below have been removed from the text and replaced by (I), (II), (III), (IV) and
(V). Number them to indicate the order they must appear to complete the text correctly.
Then mark the option that contains the right sequence.

( ) Fuel up for the day


( ) Take a time out
( ) Stay sharp
( ) Eat extra veggies
( ) Sleep easier

a) (II) (I) (V) (III) (IV)


b) (V) (I) (III) (II) (IV)
c) (I) (IV) (V) (III) (II)
d) (II) (IV) (III) (V) (I)
e) (V) (III) (IV) (II)(I)

Questão 02 – Texto para responder as questões 02 e 03 (Colégio Naval/2017)

Social media ’destroying how society works'

A former Facebook executive has said social media is doing great harm to society around the
world. The executive is a man called Chamath Palihapitiya. He ___________ Facebook in 2007
and ___________a vice president. He was responsible for increasing the number of users
Facebook had. Mr Palihapitiya said he feels very guilty about getting more people to use social
networks. He said the networks are destroying society because they are changing people's
behavior. Twenty years ago, people talked to each other face to face. Today, people message
each other and do not talk. People also really care about what other people think of them. They
post photos and wait to see how many people like the photo. They get very sad if people do not
like the photo.

Aula 04 – English General Presentation 37


www.estrategiamilitar.com.br 70
Teacher Andrea Belo
Aula 04: Escola Naval 2021

Mr. Palihapitiya said people should take a long break from social media so they can experience
real life. He wants people to value each other instead of valuing online "hearts, likes, and
thumbs-up". Palihapitiya also points out how fake news is affecting how we see the world; it is
becoming easier for large websites to spread lies. It is also becoming easier to hurt other people
online. Anyone can hide behind a fake user name and post lies about other people. Palihapitiya
said this was a global problem. He is worried about social media so much that he has banned
his children from using it. However, he did state that Facebook was a good company. He said:
"Of course, it's not all bad. Facebook overwhelmingly does good in the world."

Questão 02 – Read the statements to check if they are TRUE (T) or FALSE (F).
I- An ex-Facebook boss said social media is damaging society.
II- It is becoming more difficult for big websites to spread fake news.
III- People message each other today instead of talking face to face.
IV- Palihapitiya said social media does not change our behavior.
Choose the option that respectively represents the statements above.
(A) F/T/T/F
(B) F/F/T/T
(C) T/F/T/F
(D) T/T/F/T
(E) F/F/F/T

Questão 03 (Colégio Naval/2018)


All the underlined words in text I are adjectives, EXCEPT:
(A) social.
(B) global.
(C) long.
(D) executive.
(E) former.

Aula 04 – English General Presentation 38


www.estrategiamilitar.com.br 70
Teacher Andrea Belo
Aula 04: Escola Naval 2021

Questão 04 (EEAR/2018) - Read the text and answer questions 04 and 05


Economic crisis increases consumption of rice and beans in Brazil
The economic crisis is making the Brazilian consumer exchange meat for the traditional dish of
rice and beans. High unemployment and falling incomes, together with the low prices of these
products, caused by good harvest, are responsible for the increase in demand, __________ will
be 15% to 20% this month, compared to the prediction for the year. The average consumption
per capita is around 3, 5 kilos of rice and 1, 5 kilo of beans.
Fonte: Folha de São Paulo – Internacional -10/05/2017

04 – Choose the alternative that best completes the blank in the text:
a) who
b) which
c) whom
d) whose

Questão 05 – According to the text, the Brazilian consumer is _______.


a) changing rice and beans for meat
b) having more rice than beans on average
c) paying a very high price for rice and beans
d) reducing the demand for the traditional dish of rice and beans

Questão 06 – Text for questions 06 and 07 (EFOMM/2017)


Pidgins and creoles - Pidgin Languages
A pidgin is a system of communication which has grown up among people who do not
share a common language, but who want to talk to each other, for trading or other
reasons. Pidgins have been variously called ‘makeshift’, ‘marginal’, or ‘mixed’ languages.
They have a limited vocabulary, a reduced grammatical structure, and a much narrower
range of functions, compared to the languages which gave rise to them. They are the
native language of no one, but they are nonetheless a main means of communication for
millions of people, and a major focus of interest to those who study the way languages
change.
It is essential to avoid the stereotype of a pidgin language, as perpetrated over the years
in generations of children’s comics and films. The ‘Me Tarzan, you Jane’ image is far from
the reality.

Aula 04 – English General Presentation 39


www.estrategiamilitar.com.br 70
Teacher Andrea Belo
Aula 04: Escola Naval 2021

A pidgin is not a language which has broken down; nor is it the result of baby talk,
laziness, corruption, primitive thought processes, or mental deficiency. On the contrary:
pidgins are demonstrably creative adaptations of natural languages, with a structure
and rules of their own. Along with creoles, they are evidence of a fundamental process
of linguistic change, as languages come into contact with each other, producing new
varieties whose structures and uses contract and expand. They provide the clearest
evidence of language being created and shaped by society for its own ends, as people
adapt to new social circumstances. This emphasis on processes of change is reflected in
the terms pidginization and creolization.
Most pidgins are based on European languages – English, French, Spanish, Dutch, and
Portuguese – reflecting the history of colonialism. However, this observation may be the
result only of our ignorance of the languages used in parts of Africa, South America, or
South-east Asia, where situations of language contact are frequent. One of the best-
known non-European pidgins is Chinook Jargon, once used for trading by American
Indians in north-west USA. Another is Sango, a pidginized variety of Ngbandi, spoken
widely in west-central Africa.
Because of their limited function, pidgin languages usually do not last for very long –
sometimes for only a few years, and rarely for more than a century. They die when the
original reason for communication diminishes or disappears, as communities move
apart, or one community learns the language of the other. (Alternatively, the pidgin may
develop into a creole.) The pidgin French which was used in Vietnam all but disappeared
when the French left; similarly, the pidgin English which appeared during the American
Vietnam campaign virtually disappeared as soon as the war was over. But there are
exceptions. The pidgin known as Mediterranean Lingua Franca, or Sabir, began in the
Middle Ages and lasted until the 20th century.
Some pidgins have become so useful as a means of communication between languages
that they have developed a more formal role, as regular auxiliary languages. They may
even be given official status by a community, as lingua francas. These cases are known
as ‘expanded pidgins’, because of the way in which they have added extra forms to cope
with the needs of their users and have come to be used in a much wider range of
situations than previously. In time, these languages may come to be used on the radio,
in the press, and may even develop a literature of their own. Some of the most widely
used expanded pidgins are Krio (in Sierra Leone), Nigerian Pidgin English, and Bislama
(in Vanuatu). In Papua New Guinea, the local pidgin (Tok Pisin) is the most widely used
language in the country.
(CRYSTAL, David. The Cambridge Encyclopedia of Language, 3rd ed., 2010, p.344).

Aula 04 – English General Presentation 40


www.estrategiamilitar.com.br 70
Teacher Andrea Belo
Aula 04: Escola Naval 2021

Questão 06 - (EFOMM/2017) - In line 65, “(...) they have added extra forms to cope with the
needs of their users (...)”, the phrasal verb in bold is closest in meaning to:
( a ) foresee
( b ) support
( c ) respect
( d ) realize
( e ) handle

Questão 07 - (EFOMM/2017)
Which option can NOT be inferred from the text? A pidgin language is:
( a ) a simplified means of linguistic communication.
( b ) the native language of a speech community.
( c ) employed in situations such as commerce.
( d ) a contact language.
( e ) constructed impromptu, or by convention, between individuals or groups of people.

Questão 08 - TEXT for questions 08 and 09 (EPCAR/2019)

WHAT IS MODERN SLAVERY?


Slavery did not end with abolition in the 19th century. Slavery continues today and harms people
in every country in the world.
Women forced into prostitution. People forced to work in agriculture, domestic work and
factories. Children in sweatshops1 producing goods sold globally. Entire families forced to work
for nothing to pay off generational debts. Girls forced to marry older men.
There are estimated 40.3 million people in modern slavery around the world, including:
• 10 million children
• 24.9 million people in forced labour
• 15.4 million people in forced marriage
• 4.8 million people in forced sexual exploitation

Aula 04 – English General Presentation 41


www.estrategiamilitar.com.br 70
Teacher Andrea Belo
Aula 04: Escola Naval 2021

Someone is in slavery if they are:


• forced to work – through coercion, or mental or physical threat;
• owned or controlled by an ’employer’, through mental or physical abuse or the threat of
abuse;
• dehumanised, treated as a commodity or bought and sold as ‘property’;
• physically constrained or have restrictions placed on their freedom of movement.
Slavery has been a disgraceful aspect of human society for most of human history. However,
AntiSlavery International has refused to accept that this bloody status quo should be allowed
to persist (Aidan McQuade, former director).
Forms of modern slavery
Purposes of exploitation2 can range from forced prostitution and forced labour to forced
marriage and forced organ removal. Here are the most common forms of modern slavery.
• Forced labour – any work or services which people are forced to do against their will3 under
the threat of some form of punishment.
• Debt bondage or bonded labour – the world’s most widespread form of slavery, when people
borrow money they cannot repay and are required to work to pay off the debt, then losing
control over the conditions of both their employment and the debt.
• Human trafficking– involves transporting, recruiting or harbouring people for the purpose of
exploitation, using violence, threats or coercion.
• Descent-based slavery – where people are born into slavery because their ancestors were
captured and enslaved; they remain in slavery by descent.
• Child slavery – many people often confuse child slavery with child labour, but it is much worse.
Whilst4 child labour is harmful for children and hinders5 their education and development, child
slavery occurs when a child is exploited for someone else’s gain. It can include child trafficking,
child soldiers, child marriage and child domestic slavery.
• Forced and early marriage – when someone is married against their will and cannot leave the
marriage. Most child marriages can be considered slavery.
Many forms of slavery have more than one element listed above. For example, human
trafficking often involves advance payment for travel and a job abroad, using money often
borrowed from the traffickers. Then, the debt contributes to control of the victims. Once they
arrive, victims cannot leave until they pay off their debt.
Many people think that slavery happens only overseas, in developing countries. In fact, no
country is free from modern slavery, even Britain. The Government estimates that there are
tens of thousands people in modern slavery in the UK.
Modern slavery can affect people of any age, gender or race. However, contrary to a common
misconception6 that everyone can be a victim of slavery, some groups of people are much more
vulnerable to slavery than others.

Aula 04 – English General Presentation 42


www.estrategiamilitar.com.br 70
Teacher Andrea Belo
Aula 04: Escola Naval 2021

People who live in poverty7 and have limited opportunities for decent work are more vulnerable
to accepting deceptive job offers that can turn exploitative. People who are discriminated
against on the basis of race, caste, or gender are also more likely to be enslaved. Slavery is also
more likely to occur where the rule of law is weaker and corruption is rife.
Anti-Slavery International believes that we have to tackle8 the root causes of slavery in order
to end slavery for good. That’s why we published our AntiSlavery Charter, listing comprehensive
measures that need to be taken to end slavery across the world.
(Adapted from https://www.antislavery.org/slavery-today/modern-slavery/)
Glossary:
1. sweatshop – a factory where workers are paid very little and work many hours in very bad
conditions
2. exploitation – abuse, manipulation
3. will – wish, desire
4. whilst – while
5. to hinder – obstruct, stop
6. misconception – wrong idea/ impression
7. poverty – the condition of being extremely poor
8. to tackle – attack

Question 08 - The concept of slavery worked in the text is


a) a very hard work for which people are paid very little.
b) about slaves who hardly work.
c) about something that is legally owned by someone else.
d) the activity of having slaves.

Question 09 - Mark the alternative in which the verb “to continue” (line 2) is applied in the
sentence correctly.
a) Slavery continues to exist today, affecting continents and countries.
b) Today, new forms of slavery continues being tragic.
c) Poverty continue afflicting vast number of people.
d) New forms of slavery is continuing to come from poverty.

Aula 04 – English General Presentation 43


www.estrategiamilitar.com.br 70
Teacher Andrea Belo
Aula 04: Escola Naval 2021

Texto para responder a questão 10 - (AFA/2018)

TEXT Howard Gardner: ‘Multiple intelligences’ are not ‘learning styles’ by Valerie Strauss
The fields of psychology and education were revolutionized 30 years ago when we now worldrenowned
psychologist Howard Gardner published his 1983 book Frames of Mind: The Theory of Multiple
Intelligences, which detailed a new model of human intelligence that went beyond the traditional view
that there was a single kind that could be measured by standardized tests.
Gardner’s theory initially listed seven intelligences which work together: linguistic, logical-mathematical,
musical, bodily-kinesthetic, interpersonal and intrapersonal; he later added an eighth, naturalist
intelligence and says there may be a few more. The theory became highly popular with K-12¹ educators
around the world seeking ways to reach students who did not respond to traditional approaches, but
over time, ‘multiple intelligences’ somehow became synonymous with the concept of ‘learning styles’. In
this important post, Gardner explains why the former is not the latter.
It’s been 30 years since I developed the notion of ‘multiple intelligences’. (…)
First a word about ‘MI theory’. On the basis of research in several disciplines, including the study of how
human capacities are represented in the brain, I developed the idea that each of us has a number of
relatively independent mental faculties, which can be termed our ‘multiple intelligences’. The basic idea
is simplicity itself. A belief in a single intelligence assumes that we have one central, all-purpose
computer, and it determines how well we perform in every sector of life. In contrast, a belief in multiple
intelligences assumes that human beings have 7 to 10 distinct intelligences. (…)
Here’s my considered judgment about the best way to analyze this lexical terrain: Intelligence: We all
have the multiple intelligences. But we signed out, as a strong intelligence, an area where the person has
considerable computational power. Style or learning style: A hypothesis of how an individual approaches
the range of materials. If an individual has a ‘reflective style’, he/she is hypothesized to be reflective
about the full range of materials. We cannot assume that reflectiveness in writing necessarily signals
reflectiveness in one’s interaction with the others.
Senses: Sometimes people speak about a ‘visual’ learner or an ‘auditory’ learner. The implication is that
some people learn through their eyes, others through their ears. This notion is incoherent. Both spatial
information and reading occur with the eyes, but they make use of entirely different cognitive faculties.
What matters is the power of the mental computer, the intelligence that acts upon that sensory
information once picked up.
In contrast, there is strong evidence that human beings have a range of intelligences and that strength (or
weakness) in one intelligence does not predict strength (or weakness) in any other intelligences. All of us
exhibit jagged profiles of intelligences. There are common sense ways of assessing our own intelligences,
and even if it seems appropriate, we can take a more formal test battery. And then, as teachers, parents,
or selfassessors, we can decide how best to make use of this information.
(Adapted from https://www.washingtonpost.com/news/answer-sheet)

Questão 10 – In the fragment “why the former is not the latter” (line 10), the highlighted words
refer to
a) multiple intelligences / learning style.
b) over time / theory.
c) ways to reach students / traditional approaches.
d) traditional approaches / K-12 educators.

Aula 04 – English General Presentation 44


www.estrategiamilitar.com.br 70
Teacher Andrea Belo
Aula 04: Escola Naval 2021

11. Gabarito

Gabarito

1– B 2– C 3–D 4–B 5–B 6–E

7– A 8– D 9–A 10 – A

Aula 04 – English General Presentation 45


www.estrategiamilitar.com.br 70
Teacher Andrea Belo
Aula 04: Escola Naval 2021

12. Questões comentadas


Questão 01 – Escola Naval / 2018

Doctors Know Best


By Ted Spiker

Along with all the disease stomping, heart reviving, baby delivering, and overall people
healing they do, doctors have another full-time job: keeping themselves healthy. Scratch that
- keeping themselves healthiest. So instead of peeking into their medical practices, we looked
at what they actually practice - in their own lives. Use personal strategies and insider tips
from the best medical pros to supercharge your health this year.
( I)-______
"As soon as I feel an illness coming on, I go to sleep for at least nine hours," says Hilda
Hutcherson, MD, clinical professor of ob-gyn at Columbia University Medical Center. "I also lie
on the floor with my legs elevated and propped against the wall and breathe deeply for five
minutes." It helps lower stress, which weakens the immune system.
(II )-______
Instead of having a garden-variety green salad, Margaret McKenzie, MD, assistant professor
of surgery at the Cleveland Clinic, tosses napa cabbage, radicchio, edamame, and carrots with
ginger-soy dressing. "It gives me a lot of vitamins, antioxidants, and protein and makes me
feel full," she says.
(III)-______
[...] Gary Small, MD, professor of psychiatry and biobehavioral sciences at the University of
California, Los Angeles, and author of The Alzheimer"s Prevention Program, plays Scrabble
and Words With Friends on his smartphone most days. These word games are perfect brain
boosters, because they build not only verbal and math skills but also spatial abilities as you
position letters to create words. "Combining several mental tasks strengthens multiple neural
circuits," Dr. Small says. "It"s like cross-training for your brain."
(IV) - _____
Make your bedroom spalike: Dim the lights at least an hour before you go to bed; ban cell
phones, laptops, and the TV; ask your partner for a foot rub. "I do deep breathing exercises,"
Dr. Hutcherson says. "Sometimes I play relaxing music softly."
(V) - _____
The most important meal is breakfast, says David Katz, MD, director and founder of Yale-
Griffin Prevention Research Center in Derby, Connecticut. He often has two breakfasts,
divvying up his morning meal so that he eats half before his workout and half after. "It helps
with portion control, and it establishes a daily eating pattern," Dr. Katz says. Plan your
breakfast at night to start the next day on a healthy note.

(Abridged from https ://www.fitnessmagazine.com/health/doctors-tips-tostay-healthy/)

Aula 04 – English General Presentation 46


www.estrategiamilitar.com.br 70
Teacher Andrea Belo
Aula 04: Escola Naval 2021

QUESTÃO 01
The headlines below have been removed from the text and replaced by (I), (II), (III), (IV) and
(V). Number them to indicate the order they must appear to complete the text correctly.
Then mark the option that contains the right sequence.

( ) Fuel up for the day


( ) Take a time out
( ) Stay sharp
( ) Eat extra veggies
( ) Sleep easier

a) (II) (I) (V) (III) (IV)


b) (V) (I) (III) (II) (IV)
c) (I) (IV) (V) (III) (II)
d) (II) (IV) (III) (V) (I)
e) (V) (III) (IV) (II)(I)

Comentários:
Fuel up for the day – A afirmativa V corresponde à esta descrição. O trecho diz que o café da
manhã (breakfast) é a mais importante refeição do dia. Um bom café da manhã ajuda a
começar o dia de maneira saudável (healthy).
Take a time out – A afirmativa I corresponde à esta descrição. O trecho diz que deitar-se com
os pés para cima ajuda a diminuir (lower) o estresse, que enfraquece (weakens) o sistema
imunológico.
Stay sharp – A afirmativa III corresponde á esta descrição. O trecho diz que fazer exercícios
mentais ajudam a prevenir (prevention) a Doença de Alzheimer, e que combinar tarefas
mentais (mental tasks) fortalecem (strengthen) múltiplos circuitos neurais.

Eat extra veggies – A afirmativa II corresponde à esta descrição. O trecho diz que combinar
diversos vegetais ao invés de (instead of) comer apenas uma salada verde, é uma excelente
ideia para se abastecer de vitaminas, antioxidantes e proteínas que a fazem sentir-se cheia
(feel full).

Sleep easier – A afirmativa IV corresponde à esta descrição. O trecho diz que se evitarmos
luzes fortes (dim the lights = regular a intensidade das luzes), banirmos (ban) telas digitais e
procurarmos relaxar na última hora antes de nos deitarmos, o processo de adormecer torna-
se muito mais fácil.
Temos, então, a sequência: V / I / III / II / IV
A alternativa B está correta.

Aula 04 – English General Presentation 47


www.estrategiamilitar.com.br 70
Teacher Andrea Belo
Aula 04: Escola Naval 2021

Questão 02 – Texto para responder as questões 02 e 03

Social media ’destroying how society works'

A former Facebook executive has said social media is doing great harm to society around the
world. The executive is a man called Chamath Palihapitiya. He ___________ Facebook in 2007
and ___________a vice president. He was responsible for increasing the number of users
Facebook had. Mr Palihapitiya said he feels very guilty about getting more people to use social
networks. He said the networks are destroying society because they are changing people's
behavior. Twenty years ago, people talked to each other face to face. Today, people message
each other and do not talk. People also really care about what other people think of them. They
post photos and wait to see how many people like the photo. They get very sad if people do not
like the photo.

Mr. Palihapitiya said people should take a long break from social media so they can experience
real life. He wants people to value each other instead of valuing online "hearts, likes, and
thumbs-up". Palihapitiya also points out how fake news is affecting how we see the world; it is
becoming easier for large websites to spread lies. It is also becoming easier to hurt other people
online. Anyone can hide behind a fake user name and post lies about other people. Palihapitiya
said this was a global problem. He is worried about social media so much that he has banned
his children from using it. However, he did state that Facebook was a good company. He said:
"Of course, it's not all bad. Facebook overwhelmingly does good in the world."

Questão 02 – Read the statements to check if they are TRUE (T) or FALSE (F).
I- An ex-Facebook boss said social media is damaging society.
II- It is becoming more difficult for big websites to spread fake news.
III- People message each other today instead of talking face to face.
IV- Palihapitiya said social media does not change our behavior.
Choose the option that respectively represents the statements above.

Aula 04 – English General Presentation 48


www.estrategiamilitar.com.br 70
Teacher Andrea Belo
Aula 04: Escola Naval 2021

(A) F/T/T/F
(B) F/F/T/T
(C) T/F/T/F
(D) T/T/F/T
(E) F/F/F/T
Comentários:
A afirmativa I é verdadeira. O texto diz que ele chegou ao cargo de vice-presidente da empresa
e ele realmente diz que mídias sociais causam danos à sociedade. “A former Facebook
executive has said social media is doing great harm to society around the world”.
A afirmativa II é falsa. O texto diz que está ficando cada vez mais fácil, para os grandes sites,
difundir as fake news. “it is becoming easier for large websites to spread lies”.
A afirmativa III é verdadeira. O texto diz que 20 anos atrás, as pessoas conversavam
pessoalmente, mas que hoje as pessoas usam mensagens para se comunicar. “Twenty years
ago, people talked to each other face to face. Today, people message each other and do not
talk”.
A afirmativa IV é falsa. O texto diz que as redes estão destruindo a sociedade porque alteram
o comportamento das pessoas. “He said the networks are destroying society because they are
changing people's behavior”.

Questão 03 (Colégio Naval/2018)


All the underlined words in text I are adjectives, EXCEPT:
(A) social.
(B) global.
(C) long.
(D) executive.
(E) former.
Comentários:
A alternativa A está errada. “Social” funciona como adjetivo caracterizando o termo “media”.
A alternativa B está errada. “Global” funciona como adjetivo caracterizando o termo
“problem”.
A alternativa C está errada. “Long” funciona como adjetivo caracterizando o termo “break”.
A alternativa D está correta. “Executive” funciona como substantivo no trecho (o executivo).
A alternativa E está errada. “Former” funciona como adjetivo caracterizando o termo
“executive”.

Aula 04 – English General Presentation 49


www.estrategiamilitar.com.br 70
Teacher Andrea Belo
Aula 04: Escola Naval 2021

Questão 04 (EEAR/2018) - Read the text and answer questions 04 and 05


Economic crisis increases consumption of rice and beans in Brazil
The economic crisis is making the Brazilian consumer exchange meat for the traditional dish of
rice and beans. High unemployment and falling incomes, together with the low prices of these
products, caused by good harvest, are responsible for the increase in demand, __________ will
be 15% to 20% this month, compared to the prediction for the year. The average consumption
per capita is around 3, 5 kilos of rice and 1, 5 kilo of beans.
Fonte: Folha de São Paulo – Internacional -10/05/2017

04 – Choose the alternative that best completes the blank in the text:
a) who
b) which
c) whom
d) whose
Comentários:
Aprenderemos a usar os pronomes relativos adiante mas, já como ensinamento, preste
atenção nas primeiras explicações que seguem.
A alternativa A está incorreta. “Who” é utilizado para remeter a alguém, e não a alguma coisa.
A lacuna deve ser preenchida por uma palavra que remeta a “increase” para que o trecho tenha
sentido.
A alternativa B está correta. “Which” é a palavra mais adequada para retomar a palavra
“increase”.
A alternativa C está incorreta. “Whom” é utilizado para remeter a alguém (na função de objeto
na frase, e não na função de sujeito como apresentado no trecho), e não a alguma coisa. A
lacuna deve ser preenchida por uma palavra que remeta a “increase” para que o trecho tenha
sentido.
A alternativa D está incorreta. “Whose” significa “de quem”, e não “que”. A lacuna deve ser
preenchida por uma palavra que remeta a “increase” para que o trecho tenha sentido.

05 – According to the text, the Brazilian consumer is _______.


a) changing rice and beans for meat
b) having more rice than beans on average
c) paying a very high price for rice and beans
d) reducing the demand for the traditional dish of rice and beans

Aula 04 – English General Presentation 50


www.estrategiamilitar.com.br 70
Teacher Andrea Belo
Aula 04: Escola Naval 2021

Comentários:
A alternativa A está incorreta. O texto diz que o consumidor brasileiro está trocando a carne
pelo arroz com feijão, e não o contrário. “The economic crisis is making the Brazilian consumer
exchange meat for the traditional dish of rice and beans”.
A alternativa B está correta. O texto explicita isso no quando diz que as pessoas consomem
em média 3,5 kilos de arroz e 1,5 kilos de feijão. “The average consumption per capita is around
3, 5 kilos of rice and 1, 5 kilo of beans”.
A alternativa C está incorreta. O texto afirma que o consumo de arroz e feijão aumentou devido
ao baixo preço desses alimentos. “the low prices of these products, caused by good harvest,
are responsible for the increase in demand”.
A alternativa D está incorreta. O texto diz que o consumidor brasileiro está aumentando a
demanda pelo tradicional arroz com feijão, e não diminuindo. “High unemployment and falling
incomes, together with the low prices of these products, caused by good harvest, are
responsible for the increase in demand”

Questão 06 – Text for questions 06 and 07 (EFOMM/2017)


Pidgins and creoles - Pidgin Languages
A pidgin is a system of communication which has grown up among people who do not
share a common language, but who want to talk to each other, for trading or other
reasons. Pidgins have been variously called ‘makeshift’, ‘marginal’, or ‘mixed’ languages.
They have a limited vocabulary, a reduced grammatical structure, and a much narrower
range of functions, compared to the languages which gave rise to them. They are the
native language of no one, but they are nonetheless a main means of communication for
millions of people, and a major focus of interest to those who study the way languages
change.
It is essential to avoid the stereotype of a pidgin language, as perpetrated over the years
in generations of children’s comics and films. The ‘Me Tarzan, you Jane’ image is far from
the reality. A pidgin is not a language which has broken down; nor is it the result of baby
talk, laziness, corruption, primitive thought processes, or mental deficiency. On the
contrary: pidgins are demonstrably creative adaptations of natural languages, with a
structure and rules of their own. Along with creoles, they are evidence of a fundamental
process of linguistic change, as languages come into contact with each other, producing
new varieties whose structures and uses contract and expand. They provide the clearest
evidence of language being created and shaped by society for its own ends, as people
adapt to new social circumstances. This emphasis on processes of change is reflected in
the terms pidginization and creolization.

Aula 04 – English General Presentation 51


www.estrategiamilitar.com.br 70
Teacher Andrea Belo
Aula 04: Escola Naval 2021

Most pidgins are based on European languages – English, French, Spanish, Dutch, and
Portuguese – reflecting the history of colonialism. However, this observation may be the
result only of our ignorance of the languages used in parts of Africa, South America, or
South-east Asia, where situations of language contact are frequent. One of the best-
known non-European pidgins is Chinook Jargon, once used for trading by American
Indians in north-west USA. Another is Sango, a pidginized variety of Ngbandi, spoken
widely in west-central Africa.
Because of their limited function, pidgin languages usually do not last for very long –
sometimes for only a few years, and rarely for more than a century. They die when the
original reason for communication diminishes or disappears, as communities move
apart, or one community learns the language of the other. (Alternatively, the pidgin may
develop into a creole.) The pidgin French which was used in Vietnam all but disappeared
when the French left; similarly, the pidgin English which appeared during the American
Vietnam campaign virtually disappeared as soon as the war was over. But there are
exceptions. The pidgin known as Mediterranean Lingua Franca, or Sabir, began in the
Middle Ages and lasted until the 20th century.
Some pidgins have become so useful as a means of communication between languages
that they have developed a more formal role, as regular auxiliary languages. They may
even be given official status by a community, as lingua francas. These cases are known
as ‘expanded pidgins’, because of the way in which they have added extra forms to cope
with the needs of their users, and have come to be used in a much wider range of
situations than previously. In time, these languages may come to be used on the radio,
in the press, and may even develop a literature of their own. Some of the most widely
used expanded pidgins are Krio (in Sierra Leone), Nigerian Pidgin English, and Bislama
(in Vanuatu). In Papua New Guinea, the local pidgin (Tok Pisin) is the most widely used
language in the country.
(CRYSTAL, David. The Cambridge Encyclopedia of Language, 3rd ed., 2010, p.344).

Questão 06 - (EFOMM/2017) - In line 65, “(...) they have added extra forms to cope with the
needs of their users (...)”, the phrasal verb in bold is closest in meaning to:
( a ) foresee
( b ) support
( c ) respect
( d ) realize
( e ) handle

Aula 04 – English General Presentation 52


www.estrategiamilitar.com.br 70
Teacher Andrea Belo
Aula 04: Escola Naval 2021

Comentários:
A alternativa A está incorreta. “Foresee” é um verbo que significa: antecipar determinado
acontecimento ou informação. Este termo não substitui adequadamente o phrasal verb em
questão.
A alternativa B está incorreta. “Support” é um verbo que significa: dar apoio. Este termo não
substitui adequadamente o phrasal verb em questão.
A alternativa C está incorreta. “Respect” é um verbo que significa: respeitar. Este termo não
substitui adequadamente o phrasal verb em questão.
A alternativa D está incorreta. “realize” é um verbo que significa: perceber, dar-se conta de
algo. Este termo não substitui adequadamente o phrasal verb em questão.
A alternativa E está correta. “handle” é um verbo que significa: manejar, gerenciar, administrar
ou lidar com uma situação. “to cope with” significa: assimilar, lidar com uma situação. Portanto,
existe uma proximidade de significação entre eles.

Questão 07 - (EFOMM/2017)
Which option can NOT be inferred from the text? A pidgin language is:
( a ) a simplified means of linguistic communication.
( b ) the native language of a speech community.
( c ) employed in situations such as commerce.
( d ) a contact language.
( e ) constructed impromptu, or by convention, between individuals or groups of people.
Comentários:
A alternativa A está correta. O seguinte trecho corrobora esta afirmação: “They have a limited
vocabulary, a reduced grammatical structure, and a much narrower range of functions,
compared to the languages which gave rise to them”
A alternativa B está incorreta. O texto diz que “pidgin language” não é língua nativa de
ninguém. Isso se evidencia no trecho: “They are the native language of no one”.
A alternativa C está correta. O texto diz que o comércio é um dos motivos para o surgimento
de uma “pidgin language”. Isso é visto no trecho: “A pidgin is a system of communication which
has grown up among people who do not share a common language, but who want to talk to
each other, for trading or other reasons.
A alternativa D está correta. O texto afirma que a “pidgin language” é uma língua de contato
no trecho: “the languages used in parts of Africa, South America, or South-east Asia, where
situations of language contact are frequent”.

Aula 04 – English General Presentation 53


www.estrategiamilitar.com.br 70
Teacher Andrea Belo
Aula 04: Escola Naval 2021

A alternativa E está correta. O texto diz que essas línguas são desenvolvidas a partir de
convenções feitas entre pessoas que precisam se comunicar. “A pidgin is not a language which
has broken down; nor is it the result of baby talk, laziness, corruption, primitive thought
processes, or mental deficiency. On the contrary: pidgins are demonstrably creative
adaptations of natural languages, with a structure and rules of their own”.

Questão 08 - TEXT for questions 08 and 09 (EPCAR/2019)


WHAT IS MODERN SLAVERY?
Slavery did not end with abolition in the 19th century. Slavery continues today and harms people
in every country in the world.
Women forced into prostitution. People forced to work in agriculture, domestic work and
factories. Children in sweatshops1 producing goods sold globally. Entire families forced to work
for nothing to pay off generational debts. Girls forced to marry older men.
There are estimated 40.3 million people in modern slavery around the world, including:
• 10 million children
• 24.9 million people in forced labour
• 15.4 million people in forced marriage
• 4.8 million people in forced sexual exploitation
Someone is in slavery if they are:
• forced to work – through coercion, or mental or physical threat;
• owned or controlled by an ’employer’, through mental or physical abuse or the threat of
abuse;
• dehumanised, treated as a commodity or bought and sold as ‘property’;
• physically constrained or have restrictions placed on their freedom of movement.
Slavery has been a disgraceful aspect of human society for most of human history. However,
Anti Slavery International has refused to accept that this bloody status quo should be allowed
to persist (Aidan McQuade, former director).
Forms of modern slavery
Purposes of exploitation2 can range from forced prostitution and forced labour to forced
marriage and forced organ removal. Here are the most common forms of modern slavery.
• Forced labour – any work or services which people are forced to do against their will3 under
the threat of some form of punishment.
• Debt bondage or bonded labour – the world’s most widespread form of slavery, when people
borrow money they cannot repay and are required to work to pay off the debt, then losing
control over the conditions of both their employment and the debt.

Aula 04 – English General Presentation 54


www.estrategiamilitar.com.br 70
Teacher Andrea Belo
Aula 04: Escola Naval 2021

• Human trafficking– involves transporting, recruiting or harbouring people for the purpose of
exploitation, using violence, threats or coercion.
• Descent-based slavery – where people are born into slavery because their ancestors were
captured and enslaved; they remain in slavery by descent.
• Child slavery – many people often confuse child slavery with child labour, but it is much worse.
Whilst4 child labour is harmful for children and hinders5 their education and development, child
slavery occurs when a child is exploited for someone else’s gain. It can include child trafficking,
child soldiers, child marriage and child domestic slavery.
• Forced and early marriage – when someone is married against their will and cannot leave the
marriage. Most child marriages can be considered slavery.
Many forms of slavery have more than one element listed above. For example, human
trafficking often involves advance payment for travel and a job abroad, using money often
borrowed from the traffickers. Then, the debt contributes to control of the victims. Once they
arrive, victims cannot leave until they pay off their debt.
Many people think that slavery happens only overseas, in developing countries. In fact, no
country is free from modern slavery, even Britain. The Government estimates that there are
tens of thousands people in modern slavery in the UK.
Modern slavery can affect people of any age, gender or race. However, contrary to a common
misconception6 that everyone can be a victim of slavery, some groups of people are much more
vulnerable to slavery than others.
People who live in poverty7 and have limited opportunities for decent work are more vulnerable
to accepting deceptive job offers that can turn exploitative. People who are discriminated
against on the basis of race, caste, or gender are also more likely to be enslaved. Slavery is also
more likely to occur where the rule of law is weaker, and corruption is rife.
Anti-Slavery International believes that we have to tackle8 the root causes of slavery in order
to end slavery for good. That’s why we published our Anti Slavery Charter, listing comprehensive
measures that need to be taken to end slavery across the world.
(Adapted from https://www.antislavery.org/slavery-today/modern-slavery/)
Glossary:
1. sweatshop – a factory where workers are paid very little and work many hours in very bad
conditions
2. exploitation – abuse, manipulation
3. will – wish, desire
4. whilst – while
5. to hinder – obstruct, stop
6. misconception – wrong idea/ impression
7. poverty – the condition of being extremely poor
8. to tackle – attack

Aula 04 – English General Presentation 55


www.estrategiamilitar.com.br 70
Teacher Andrea Belo
Aula 04: Escola Naval 2021

Question 08 - The concept of slavery worked in the text is


a) a very hard work for which people are paid very little.
b) about slaves who hardly work.
c) about something that is legally owned by someone else.
d) the activity of having slaves.
Comentários:
A alternativa A está incorreta. No conceito de escravidão abordado pelo texto, as pessoas não
tem salário algum pelo trabalho realizado.
A alternativa B está incorreta. “Hardly” é um advérbio que significa “raramente”. Portanto, a
alternativa está incorreta pelo fato de que escravos, de forma geral, trabalham muito, e não
raramente.
A alternativa C está incorreta. “Something” significa alguma coisa. Isso já garante que a
afirmação está errada. Não se pode chamar uma pessoa de “coisa”. Como a escravidão
legalizada já acabou no mundo, o texto não trata de pessoas que “possuem” outras pessoas de
forma legal. Atualmente, toda e qualquer escravidão é ilegal.
A alternativa D está correta. O conceito de escravidão abordado no texto consiste em falar
sobre a atividade de possuir escravos.

Question 09 - Mark the alternative in which the verb “to continue” (line 2) is applied in the
sentence correctly.
a) Slavery continues to exist today, affecting continents and countries.
b) Today, new forms of slavery continues being tragic.
c) Poverty continue afflicting vast number of people.
d) New forms of slavery is continuing to come from poverty.
Comentários:
A alternativa A está correta. O verbo está corretamente conjugado com a letra ”S” acrescida
ao final da palavra, tendo em vista que “slavery” pode ser substituído pelo pronome de terceira
pessoa do singular “IT”.
A alternativa B está incorreta. “New forms of slavery” (novas formas de escravidão) está no
plural e, portanto, pode ser substituído pelo pronome de terceira pessoa do plural “THEY”.
Sendo assim, o verbo não deveria ter a letra “S” acrescida ao seu final e deveria ser escrita da
seguinte forma: “continue”.
A alternativa C está incorreta. “Poverty” (Pobreza) está no singular e, portanto, pode ser
substituído pelo pronome de terceira pessoa do singular “IT”. Sendo assim, o verbo deveria ter
a letra “S” acrescida ao seu final e deveria ser escrita da seguinte forma: “continues”.

Aula 04 – English General Presentation 56


www.estrategiamilitar.com.br 70
Teacher Andrea Belo
Aula 04: Escola Naval 2021

A alternativa D está incorreta. “New forms of slavery” (novas formas de escravidão) está no
plural e, portanto, pode ser substituído pelo pronome de terceira pessoa do plural “THEY”.
Sendo assim, o verbo conjugado no tempo verbal “Present Continuous” deveria apresentar o
pronome “ARE” ao invés de “IS”.

Texto para responder a questão 01

TEXT Howard Gardner: ‘Multiple intelligences’ are not ‘learning styles’ by Valerie Strauss
The fields of psychology and education were revolutionized 30 years ago when we now worldrenowned
psychologist Howard Gardner published his 1983 book Frames of Mind: The Theory of Multiple
Intelligences, which detailed a new model of human intelligence that went beyond the traditional view
that there was a single kind that could be measured by standardized tests.
Gardner’s theory initially listed seven intelligences which work together: linguistic, logical-mathematical,
musical, bodily-kinesthetic, interpersonal and intrapersonal; he later added an eighth, naturalist
intelligence and says there may be a few more. The theory became highly popular with K-12¹ educators
around the world seeking ways to reach students who did not respond to traditional approaches, but
over time, ‘multiple intelligences’ somehow became synonymous with the concept of ‘learning styles’. In
this important post, Gardner explains why the former is not the latter.
It’s been 30 years since I developed the notion of ‘multiple intelligences’. (…)
First a word about ‘MI theory’. On the basis of research in several disciplines, including the study of how
human capacities are represented in the brain, I developed the idea that each of us has a number of
relatively independent mental faculties, which can be termed our ‘multiple intelligences’. The basic idea
is simplicity itself. A belief in a single intelligence assumes that we have one central, all-purpose
computer, and it determines how well we perform in every sector of life. In contrast, a belief in multiple
intelligences assumes that human beings have 7 to 10 distinct intelligences. (…)
Here’s my considered judgment about the best way to analyze this lexical terrain: Intelligence: We all
have the multiple intelligences. But we signed out, as a strong intelligence, an area where the person has
considerable computational power. Style or learning style: A hypothesis of how an individual approaches
the range of materials. If an individual has a ‘reflective style’, he/she is hypothesized to be reflective
about the full range of materials. We cannot assume that reflectiveness in writing necessarily signals
reflectiveness in one’s interaction with the others.
Senses: Sometimes people speak about a ‘visual’ learner or an ‘auditory’ learner. The implication is that
some people learn through their eyes, others through their ears. This notion is incoherent. Both spatial
information and reading occur with the eyes, but they make use of entirely different cognitive faculties.
What matters is the power of the mental computer, the intelligence that acts upon that sensory
information once picked up.
In contrast, there is strong evidence that human beings have a range of intelligences and that strength (or
weakness) in one intelligence does not predict strength (or weakness) in any other intelligences. All of us
exhibit jagged profiles of intelligences. There are common sense ways of assessing our own intelligences,
and even if it seems appropriate, we can take a more formal test battery. And then, as teachers, parents,
or selfassessors, we can decide how best to make use of this information.
(Adapted from https://www.washingtonpost.com/news/answer-sheet)

Aula 04 – English General Presentation 57


www.estrategiamilitar.com.br 70
Teacher Andrea Belo
Aula 04: Escola Naval 2021

Questão 10 – In the fragment “why the former is not the latter” (line 10), the highlighted words
refer to
a) multiple intelligences / learning style.
b) over time / theory.
c) ways to reach students / traditional approaches.
d) traditional approaches / K-12 educators.

Comentários:
É importante entender que former e latter são palavras usadas para designar “anterior” e
“posterior”, respectivamente. Portanto, o trecho significa: o porquê de o anterior não ser o
posterior. Nesse caso, former e latter se referem a expressões anteriormente citadas no texto.
Former retoma “multiple intelligences” enquanto latter retoma “learning style”. “multiple
intelligences” é citado anteriormente no texto em relação a “learning style”. Isso permite
compreender de forma clara porque former se refere a “multiple intelligences” e latter se
refere a “learning styles”.

A alternativa A está correta. Ela está completamente de acordo com a explicação dada acima.

A alternativa B está incorreta. Former e latter são palavras utilizadas para retomar palavras
anteriormente citadas. Mas essas palavras são, normalmente, substantivos. Portanto, over
time não poderia ser retomado por former nem por latter.

As alternativas C e D são incorretas. Pode-se entender por que essas alternativas estão
incorretas analisando a explicação acima, na qual expliquei como os termos former e latter.

Aula 04 – English General Presentation 58


www.estrategiamilitar.com.br 70
Teacher Andrea Belo
Aula 04: Escola Naval 2021

13. Considerações finais


Concluímos mais uma aula, outro passo até a sua aprovação!
Desta vez, com adjetivos e advérbios. E estamos caminhando para maior vocabulário e mais
aprendizado de fato.
Nota-se o progresso em seus estudos e, provavelmente, uma maior tranquilidade para enfrentar os
exercícios que surgem. E você vai se acostumando a equilibrar seus estudos de forma sistematizada,
estudando cada vez mais e com mais dedicação.

Outro detalhe importante para seu sucesso nos estudos, é continuar fazendo listas de
vocabulário das palavras e verbos, principalmente os irregulares, que aparecem em forma de lista
em inúmeras fontes de pesquisa.
Isso te ajudará nas questões futuras e torna você, como eu disse antes, um candidato mais bem
preparado e confiante para realizar uma excelente prova.
É importante lembrar também do nosso Fórum de dúvidas, exclusivo do Estratégia
Concursos. Será minha forma de responder, no prazo máximo de 48 horas, o que mais você precise
saber para que os conteúdos fiquem ainda mais claros em seus estudos, certo?

E, caso queira, acesse minhas redes sociais para aprender mais palavras e contar com dicas
importantes, que colaboram diretamente com seus estudos dia após dia.

@teacherandreabelo

Teacher Andrea Belo

Aula 04 – English General Presentation 59


www.estrategiamilitar.com.br 70
Teacher Andrea Belo
Aula 04: Escola Naval 2021

14. Referências bibliográficas


ACKLAM, Richard; CRACE, Araminta. Total English: Pre intermediate. 1 ed. Grã-Bretanha: Longman
do Brasil, 2005.
BAKER, M. In other words: a coursebook on translation. Routledge, 1992.
BLATT, Franz. Précis de Syntaxe Latine. Lyon, Paris: IAC, 1952.
BENTES, Anna Christina e Mussalim, Fernanda (org.). Introdução À Linguística, Domínios E Fronteiras.
6ª edição. Editora Cortez. São Paulo. 2006.
BOURGOGNE, Cleuza Vilas Boas & Silva Lilian Santos. Interação & Transformação. SP: Ed. Brasil,
1999.
BOWKER, L. & PEARSON, J. Working with Specialized Language. Routledge. Capítulos 1, 2, 8,10 e 11,
2002.
BUSSE, Winfried Busse & Mário Vilela. Gramática de Valências. Coimbra: Almedina,1986.
CARVALHO, José Herculano de. Estudos Lingüísticos. v. 2. Coimbra: Atlântida, 1969.
CHIMIM, Renata; Ilearn English student book, 4 / Renata Chimim, Viviane Kirmeliene; [obra coletiva
organizada e desenvolvida pela editora]. 1ª. ed. São Paulo: Pearson Education do Brasil, 2013.
CORBEIL, J.-Cl., ARCHAMBAULT, A. Michaelis Tech dicionário temático visual inglês-português-
francês-espanhol. Tradução: Marisa Soares de Andrade. São Paulo: Melhoramentos, 1997.
CUNHA, Celso. Nova Gramática do Português Contemporâneo. Rio de Janeiro: Nova fronteira,
terceira edição, 2001.
CUNNINGHAM, Gillie; REDSTON, Chris. Face2Face: Upper Intermediate. 1 ed. Brazil: Cambridge,
2001.
DANIELS, H. Vygotsky and pedagogy. Educational Tasks Pedagogical Communication for Teachers.
Routledge, 3rd edition, 2001.
FAIRCLOUGH, N. Discourse and social change. Polity Press, 1992.
GENTZLER, E. Contemporary Translation Theory. Routledge, 1993.
HOUAISS, A., CARDIM, I. Dicionário universitário Webster inglês-português / português-inglês. São
Paulo: Record, 1998.
HYLAND, K. Genre and second language writing – For teachers and pedagogical professionals in
general, 2003.
HUTCHINSON, Tom & WATERS, Alan. English for Specific Purposes. Cambridge: Cambridge University
Press, 1996.
LAFACE, A. O dicionário e o contexto escolar. Revista Brasileira de Linguística, Unesp/Assis, v.9, 1982,
p. 165-179.
LOBATO, M.P. Lúcia. Teorias Linguísticas e ensino do português como língua materna. Brasília: UNB,
1999.
MICHAELIS Tech Dicionário Temático Visual: línguas estrangeiras – Pesquisa e tradução Marisa
Soares de Andrade. – São Paulo: Companhia Melhoramentos, 1997..
SILVA, João Antenor de C., GARRIDO, Maria Lina, BARRETO, Tânia Pedrosa. Inglês Instrumental:
Leitura e Compreensão de Textos. Salvador: Centro Editorial e Didático, UFBA. 1994.

Aula 04 – English General Presentation 60


www.estrategiamilitar.com.br 70
Teacher Andrea Belo
Aula 04: Escola Naval 2021

SILVA, T.; MATSUDA, P. Second language writing research: perspectives on the process of knowledge
construction, 2001.
SILVEIRA BUENO, F. A formação histórica da língua portuguesa. 3. ed. São Paulo: Saraiva , 1967.
SIMPSON, J., WEINER, E. (eds.) Oxford English dictionary on CD-ROM. 2ed. Oxford : Oxford
University Press, 1999.
PASCHOALIN, Maria Aparecida; SPADOTO, Neuza Terezinha. Gramática, Teoria e Exercícios. Editora
FDT. São Paulo. 1996.
RIBEIRO, Manuel P. Nova gramática aplicada da língua portuguesa. Rio de Janeiro: Metáfora editora,
14ª edição, 2002.
TUCK, Michael. Oxford Dictionary of Computing for Learners of English. Oxford: Oxford University
Press, 1996.
CETEMFolha/NILC: Corpus de Extractos de Textos Electrónicos. Banco de dados. Disponível em:
http://acdc.linguateca.pt/cetenfolha>.Último acesso (vários acessos) em: 04.05.2019.
COSTA, Daiane. As origens da língua inglesa. Disponível em:
http://englishmaze.wordpress.com/2011/01/25/as-origens-da-lingua-inglesa/Acesso em: 2/5/
2019.
VENTURINI, Laercio. Origem e desenvolvimento da língua inglesa. Disponível em:
<http://www.startenglish.com.br/index.php?option=com_content&task=view&id=100&Itemid=97
>. Acesso em: 22 mai. 2012.
OXFORD photo dictionary. Oxford: Oxford University Press, 1992

Referências complementares (websites):


www.richmond.com.br - Acesso em 18 de março de 2019.
http://www.sk.com.br/sk-perf.html - Acesso em 19 de março de 2019.
https://www.inglesnapontadalingua.com.br/2013/03/o-que-sao-falsos-cognatos.html - Acesso em
19 de março de 2019.
https://englishlive.ef.com/pt-br/blog/15-expressoes-idiomaticas-comuns-em-ingles/
https://www.infoescola.com/ingles/

https://www.solinguainglesa.com.br/conteudo/indice.php

https://www.inglesnapontadalingua.com.br

https://www.englishexperts.com.br/

Aula 04 – English General Presentation 61


www.estrategiamilitar.com.br 70
Teacher Andrea Belo
Aula 04: Escola Naval 2021

15. Traduções

TEXT Howard Gardner: ‘Multiple intelligences’ are not ‘learning styles’ by Valerie Strauss
The fields of psychology and education were revolutionized 30 years ago when we now worldrenowned
psychologist Howard Gardner published his 1983 book Frames of Mind: The Theory of Multiple
Intelligences, which detailed a new model of human intelligence that went beyond the traditional view that
there was a single kind that could be measured by standardized tests.
Gardner’s theory initially listed seven intelligences which work together: linguistic, logical-mathematical,
musical, bodily-kinesthetic, interpersonal and intrapersonal; he later added an eighth, naturalist
intelligence and says there may be a few more. The theory became highly popular with K-12¹ educators
around the world seeking ways to reach students who did not respond to traditional approaches, but over
time, ‘multiple intelligences’ somehow became synonymous with the concept of ‘learning styles’. In this
important post, Gardner explains why the former is not the latter.
It’s been 30 years since I developed the notion of ‘multiple intelligences’. (…)
First a word about ‘MI theory’. On the basis of research in several disciplines, including the study of how
human capacities are represented in the brain, I developed the idea that each of us has a number of
relatively independent mental faculties, which can be termed our ‘multiple intelligences’. The basic idea is
simplicity itself. A belief in a single intelligence assumes that we have one central, all-purpose computer,
and it determines how well we perform in every sector of life. In contrast, a belief in multiple intelligences
assumes that human beings have 7 to 10 distinct intelligences. (…)
Here’s my considered judgment about the best way to analyze this lexical terrain: Intelligence: We all have
the multiple intelligences. But we signed out, as a strong intelligence, an area where the person has
considerable computational power. Style or learning style: A hypothesis of how an individual approaches
the range of materials. If an individual has a ‘reflective style’, he/she is hypothesized to be reflective about
the full range of materials. We cannot assume that reflectiveness in writing necessarily signals
reflectiveness in one’s interaction with the others.
Senses: Sometimes people speak about a ‘visual’ learner or an ‘auditory’ learner. The implication is that
some people learn through their eyes, others through their ears. This notion is incoherent. Both spatial
information and reading occur with the eyes, but they make use of entirely different cognitive faculties.
What matters is the power of the mental computer, the intelligence that acts upon that sensory
information once picked up.
In contrast, there is strong evidence that human beings have a range of intelligences and that strength (or
weakness) in one intelligence does not predict strength (or weakness) in any other intelligences. All of us
exhibit jagged profiles of intelligences. There are common sense ways of assessing our own intelligences,
and even if it seems appropriate, we can take a more formal test battery. And then, as teachers, parents, or
selfassessors, we can decide how best to make use of this information.
(Adapted from https://www.washingtonpost.com/news/answer-sheet)

Aula 04 – English General Presentation 62


www.estrategiamilitar.com.br 70
Teacher Andrea Belo
Aula 04: Escola Naval 2021

TEXTO Howard Gardner: 'Múltiplas inteligências' não são 'estilos de aprendizagem' de Valerie Strauss

Os campos da psicologia e da educação foram revolucionados 30 anos atrás, quando agora o psicólogo de renome
mundial Howard Gardner publicou seu livro Frames of Mind: The Theory of Multiple Intelligences, de 1983, que
detalhava um novo modelo de inteligência humana que ia além da visão tradicional de que havia um tipo único que
pode ser medido por testes padronizados.
A teoria de Gardner listou inicialmente sete inteligências que trabalham juntas: linguística, lógico-matemática,
musical, cinestésica, interpessoal e intrapessoal; mais tarde, ele acrescentou uma oitava inteligência naturalista e diz
que pode haver mais algumas. A teoria se tornou muito popular entre os educadores de ensino fundamental e médio
do mundo todo, buscando maneiras de alcançar os alunos que não responderam às abordagens tradicionais, mas
com o tempo, 'inteligências múltiplas' de alguma forma tornaram-se sinônimos do conceito de 'estilos de
aprendizagem'. Neste importante post, Gardner explica por que o primeiro não é o último.
Faz 30 anos que desenvolvi a noção de 'inteligências múltiplas'. (…)
Primeiro, uma palavra sobre a "teoria do IM". Com base em pesquisas em várias disciplinas, incluindo o estudo de
como as capacidades humanas são representadas no cérebro, desenvolvi a ideia de que cada um de nós tem várias
faculdades mentais relativamente independentes, que podem ser denominadas nossas 'inteligências múltiplas'. A
idéia básica é a própria simplicidade. A crença em uma única inteligência pressupõe que temos um computador
central para todos os fins e determina o desempenho de todos os setores da vida. Em contraste, uma crença em
múltiplas inteligências pressupõe que os seres humanos tenham 7 a 10 inteligências distintas. (…)
Aqui está o meu julgamento considerado sobre a melhor maneira de analisar esse terreno lexical: Inteligência: todos
nós temos múltiplas inteligências. Mas assinamos, como uma inteligência forte, uma área em que a pessoa tem um
poder computacional considerável. Estilo ou estilo de aprendizagem: Uma hipótese de como um indivíduo aborda a
variedade de materiais. Se um indivíduo tem um "estilo reflexivo", é provável que ele reflita sobre toda a gama de
materiais. Não podemos assumir que a refletividade na escrita necessariamente sinaliza reflexividade na interação
de uma pessoa com as outras.
Sentidos: às vezes as pessoas falam sobre um aluno 'visual' ou 'auditivo'. A implicação é que algumas pessoas
aprendem com seus olhos, outras através de seus ouvidos. Essa noção é incoerente. Tanto a informação espacial
quanto a leitura ocorrem com os olhos, mas fazem uso de faculdades cognitivas inteiramente diferentes. O que
importa é o poder do computador mental, a inteligência que age sobre essas informações sensoriais, uma vez
captadas.
Por outro lado, há fortes evidências de que os seres humanos têm uma gama de inteligências e que a força (ou
fraqueza) em uma inteligência não prediz força (ou fraqueza) em outras inteligências. Todos nós exibimos perfis
irregulares de inteligências. Existem maneiras de senso comum de avaliar nossas próprias inteligências e, mesmo que
pareça apropriado, podemos fazer uma bateria de teste mais formal. E então, como professores, pais ou auto
avaliadores, podemos decidir qual a melhor forma de usar essas informações.

Aula 04 – English General Presentation 63


www.estrategiamilitar.com.br 70
Teacher Andrea Belo
Aula 04: Escola Naval 2021

Social media ’destroying how society works'

A former Facebook executive has said social media is doing great harm to society around the
world. The executive is a man called Chamath Palihapitiya. He ___________ Facebook in 2007
and ___________a vice president. He was responsible for increasing the number of users
Facebook had. Mr Palihapitiya said he feels very guilty about getting more people to use social
networks. He said the networks are destroying society because they are changing people's
behavior. Twenty years ago, people talked to each other face to face. Today, people message
each other and do not talk. People also really care about what other people think of them. They
post photos and wait to see how many people like the photo. They get very sad if people do not
like the photo.

Mr. Palihapitiya said people should take a long break from social media so they can experience
real life. He wants people to value each other instead of valuing online "hearts, likes, and
thumbs-up". Palihapitiya also points out how fake news is affecting how we see the world; it is
becoming easier for large websites to spread lies. It is also becoming easier to hurt other people
online. Anyone can hide behind a fake user name and post lies about other people. Palihapitiya
said this was a global problem. He is worried about social media so much that he has banned
his children from using it. However, he did state that Facebook was a good company. He said:
"Of course, it's not all bad. Facebook overwhelmingly does good in the world."

Aula 04 – English General Presentation 64


www.estrategiamilitar.com.br 70
Teacher Andrea Belo
Aula 04: Escola Naval 2021

Mídia social "destruindo o funcionamento da sociedade"

Um ex-executivo do Facebook disse que a mídia social está causando grandes danos à sociedade em
todo o mundo. O executivo é um homem chamado Chamath Palihapitiya. Ele ___________ Facebook
em 2007 e ___________ vice-presidente. Ele foi responsável por aumentar o número de usuários
que o Facebook tinha. Palihapitiya disse que se sente muito culpado por conseguir que mais pessoas
usem as redes sociais. Ele disse que as redes estão destruindo a sociedade porque estão mudando
o comportamento das pessoas. Vinte anos atrás, as pessoas conversavam cara a cara. Hoje, as
pessoas trocam mensagens e não falam. As pessoas também se importam com o que as outras
pessoas pensam delas. Eles postam fotos e esperam para ver quantas pessoas gostam da foto. Eles
ficam muito tristes se as pessoas não gostam da foto.

Palihapitiya disse que as pessoas devem fazer uma longa pausa nas mídias sociais para poder
experimentar a vida real. Ele quer que as pessoas se valorizem em vez de avaliar "corações, gostos
e polegares para cima" on-line. Palihapitiya também aponta como as notícias falsas estão afetando
a maneira como vemos o mundo; está se tornando mais fácil para sites grandes espalharem
mentiras. Também está se tornando mais fácil ferir outras pessoas online. Qualquer um pode se
esconder atrás de um nome de usuário falso e postar mentiras sobre outras pessoas. Palihapitiya
disse que este era um problema global. Ele está tão preocupado com as mídias sociais que proibiu
seus filhos de usá-las. No entanto, ele afirmou que o Facebook era uma boa empresa. Ele disse: "É
claro que nem tudo é ruim. O Facebook faz o maior número de coisas boas no mundo".

Aula 04 – English General Presentation 65


www.estrategiamilitar.com.br 70
Teacher Andrea Belo
Aula 04: Escola Naval 2021

Pidgins and creoles - Pidgin Languages


A pidgin is a system of communication which has grown up among people who do not share a common
language, but who want to talk to each other, for trading or other reasons. Pidgins have been variously
called ‘makeshift’, ‘marginal’, or ‘mixed’ languages. They have a limited vocabulary, a reduced
grammatical structure, and a much narrower range of functions, compared to the languages which
gave rise to them. They are the native language of no one, but they are nonetheless a main means of
communication for millions of people, and a major focus of interest to those who study the way
languages change.
It is essential to avoid the stereotype of a pidgin language, as perpetrated over the years in generations
of children’s comics and films. The ‘Me Tarzan, you Jane’ image is far from the reality. A pidgin is not a
language which has broken down; nor is it the result of baby talk, laziness, corruption, primitive thought
processes, or mental deficiency. On the contrary: pidgins are demonstrably creative adaptations of
natural languages, with a structure and rules of their own. Along with creoles, they are evidence of a
fundamental process of linguistic change, as languages come into contact with each other, producing
new varieties whose structures and uses contract and expand. They provide the clearest evidence of
language being created and shaped by society for its own ends, as people adapt to new social
circumstances. This emphasis on processes of change is reflected in the terms pidginization and
creolization.
Most pidgins are based on European languages – English, French, Spanish, Dutch, and Portuguese –
reflecting the history of colonialism. However, this observation may be the result only of our ignorance
of the languages used in parts of Africa, South America, or South-east Asia, where situations of
language contact are frequent. One of the best-known non-European pidgins is Chinook Jargon, once
used for trading by American Indians in north-west USA. Another is Sango, a pidginized variety of
Ngbandi, spoken widely in west-central Africa.
Because of their limited function, pidgin languages usually do not last for very long – sometimes for
only a few years, and rarely for more than a century. They die when the original reason for
communication diminishes or disappears, as communities move apart, or one community learns the
language of the other. (Alternatively, the pidgin may develop into a creole.) The pidgin French which
was used in Vietnam all but disappeared when the French left; similarly, the pidgin English which
appeared during the American Vietnam campaign virtually disappeared as soon as the war was over.
But there are exceptions. The pidgin known as Mediterranean Lingua Franca, or Sabir, began in the
Middle Ages and lasted until the 20th century.
Some pidgins have become so useful as a means of communication between languages that they have
developed a more formal role, as regular auxiliary languages. They may even be given official status by
a community, as lingua francas. These cases are known as ‘expanded pidgins’, because of the way in
which they have added extra forms to cope with the needs of their users, and have come to be used in
a much wider range of situations than previously. In time, these languages may come to be used on the
radio, in the press, and may even develop a literature of their own. Some of the most widely used
expanded pidgins are Krio (in Sierra Leone), Nigerian Pidgin English, and Bislama (in Vanuatu). In Papua
New Guinea, the local pidgin (Tok Pisin) is the most widely used language in the country.

Um pidgin é um sistema de comunicação que cresceu entre pessoas que não compartilham um
idioma comum, mas que desejam conversar entre si, por motivos comerciais ou outros. Os Pidgins

Aula 04 – English General Presentation 66


www.estrategiamilitar.com.br 70
Teacher Andrea Belo
Aula 04: Escola Naval 2021

foram chamados de idiomas 'improvisados', 'marginais' ou 'mistos'. Eles têm um vocabulário


limitado, uma estrutura gramatical reduzida e uma gama muito menor de funções, em comparação
com as línguas que as originaram. Eles não são a língua nativa de ninguém, mas são, no entanto, o
principal meio de comunicação para milhões de pessoas, e um foco importante de interesse para
quem estuda a maneira como os idiomas mudam.
É essencial evitar o estereótipo de uma linguagem pidgin, como perpetrada ao longo dos anos em
gerações de quadrinhos e filmes infantis. A imagem 'Me Tarzan, você Jane' está longe da realidade.
Um pidgin não é um idioma quebrado; nem é o resultado de conversa de bebê, preguiça, corrupção,
processos primitivos de pensamento ou deficiência mental. Pelo contrário: os pidgins são
comprovadamente adaptações criativas das linguagens naturais, com uma estrutura e regras
próprias. Juntamente com os crioulos, eles são evidências de um processo fundamental de mudança
lingüística, à medida que as línguas entram em contato uma com a outra, produzindo novas
variedades cujas estruturas e usos se contraem e se expandem. Eles fornecem a evidência mais clara
de que a linguagem está sendo criada e modelada pela sociedade para seus próprios fins, à medida
que as pessoas se adaptam às novas circunstâncias sociais. Essa ênfase nos processos de mudança
se reflete nos termos pidginização e creolização.
A maioria dos pidgins é baseada em idiomas europeus - inglês, francês, espanhol, holandês e
português - refletindo a história do colonialismo. No entanto, essa observação pode ser o resultado
apenas de nossa ignorância dos idiomas usados em partes da África, América do Sul ou sudeste da
Ásia, onde as situações de contato com o idioma são frequentes. Um dos pidgins não europeus mais
conhecidos é o Chinook Jargon, usado anteriormente para comércio por índios americanos no
noroeste dos EUA. Outro é o Sango, uma variedade pidginizada de Ngbandi, falada amplamente na
África centro-oeste.
Devido à sua função limitada, os idiomas pidgin geralmente não duram muito tempo - às vezes
apenas por alguns anos e raramente por mais de um século. Eles morrem quando o motivo original
da comunicação diminui ou desaparece, à medida que as comunidades se afastam ou uma
comunidade aprende o idioma da outra. (Como alternativa, o pidgin pode se transformar em
crioulo.) O francês pidgin usado no Vietnã quase desapareceu quando os franceses saíram; da
mesma forma, o pidgin inglês que apareceu durante a campanha americana do Vietnã praticamente
desapareceu assim que a guerra terminou. Mas há exceções. O pidgin conhecido como
Mediterrâneo Lingua Franca, ou Sabir, começou na Idade Média e durou até o século XX.
Alguns pidgins tornaram-se tão úteis como um meio de comunicação entre idiomas que eles
desenvolveram um papel mais formal, como idiomas auxiliares regulares. Eles podem até receber o
status oficial de uma comunidade, como lingua francas. Esses casos são conhecidos como "pidgins
expandidos", devido à maneira como eles adicionaram formulários extras para lidar com as
necessidades de seus usuários e passaram a ser usados em uma variedade muito maior de situações
do que anteriormente. Com o tempo, esses idiomas podem ser usados no rádio, na imprensa e
podem até desenvolver uma literatura própria. Alguns dos pidgins expandidos mais usados são Krio
(na Serra Leoa), nigeriano Pidgin English e Bislama (em Vanuatu). Na Papua Nova Guiné, o pidgin
local (Tok Pisin) é a língua mais usada no país.

WHAT IS MODERN SLAVERY?

Aula 04 – English General Presentation 67


www.estrategiamilitar.com.br 70
Teacher Andrea Belo
Aula 04: Escola Naval 2021

Slavery did not end with abolition in the 19th century. Slavery continues today and harms people
in every country in the world.
Women forced into prostitution. People forced to work in agriculture, domestic work and
factories. Children in sweatshops1 producing goods sold globally. Entire families forced to work
for nothing to pay off generational debts. Girls forced to marry older men.
There are estimated 40.3 million people in modern slavery around the world, including:
• 10 million children
• 24.9 million people in forced labour
• 15.4 million people in forced marriage
• 4.8 million people in forced sexual exploitation
Someone is in slavery if they are:
• forced to work – through coercion, or mental or physical threat;
• owned or controlled by an ’employer’, through mental or physical abuse,
• dehumanised, treated as a commodity or bought and sold as ‘property’;
• physically constrained or have restrictions placed on their freedom of movement.
Slavery has been a disgraceful aspect of human society for most of human history. However,
Anti Slavery International has refused to accept that this bloody status quo should be allowed
to persist (Aidan McQuade, former director).
Forms of modern slavery
Purposes of exploitation2 can range from forced prostitution and forced labour to forced
marriage and forced organ removal. Here are the most common forms of modern slavery.
• Forced labour – any work or services which people are forced to do against their will3 under
the threat of some form of punishment.
• Debt bondage or bonded labour – the world’s most widespread form of slavery, when people
borrow money they cannot repay and are required to work to pay off the debt, then losing
control over the conditions of both their employment and the debt.
• Human trafficking– involves transporting, recruiting or harbouring people for the purpose of
exploitation, using violence, threats or coercion.
• Descent-based slavery – where people are born into slavery because their ancestors were
captured and enslaved; they remain in slavery by descent.
• Child slavery – many people often confuse child slavery with child labour, but it is much worse.
Whilst4 child labour is harmful for children and hinders5 their education and development, child
slavery occurs when a child is exploited for someone else’s gain. It can include child trafficking,
child soldiers, child marriage and child domestic slavery.
• Forced and early marriage – when someone is married against their will and cannot leave the
marriage. Most child marriages can be considered slavery.

Aula 04 – English General Presentation 68


www.estrategiamilitar.com.br 70
Teacher Andrea Belo
Aula 04: Escola Naval 2021

Many forms of slavery have more than one element listed above. For example, human
trafficking often involves advance payment for travel and a job abroad, using money often
borrowed from the traffickers. Then, the debt contributes to control of the victims. Once they
arrive, victims cannot leave until they pay off their debt.
Many people think that slavery happens only overseas, in developing countries. In fact, no
country is free from modern slavery, even Britain. The Government estimates that there are
tens of thousands people in modern slavery in the UK.
Modern slavery can affect people of any age, gender or race. However, contrary to a common
misconception6 that everyone can be a victim of slavery, some groups of people are much more
vulnerable to slavery than others.
People who live in poverty7 and have limited opportunities for decent work are more vulnerable
to accepting deceptive job offers that can turn exploitative. People who are discriminated
against on the basis of race, caste, or gender are also more likely to be enslaved. Slavery is also
more likely to occur where the rule of law is weaker, and corruption is rife.
Anti-Slavery International believes that we have to tackle8 the root causes of slavery in order
to end slavery for good. That’s why we published our Anti Slavery Charter, listing comprehensive
measures that need to be taken to end slavery across the world.

O QUE É ESCRAVIDÃO MODERNA?


A escravidão não terminou com a abolição no século XIX. A escravidão continua hoje e prejudica as
pessoas em todos os países do mundo.
Mulheres forçadas à prostituição. Pessoas forçadas a trabalhar na agricultura, trabalho doméstico e
fábricas. Crianças em fábricas1 produzindo produtos vendidos globalmente. Famílias inteiras
forçadas a trabalhar por nada para pagar dívidas geracionais. Meninas forçadas a se casar com
homens mais velhos.
Estima-se que 40,3 milhões de pessoas na escravidão moderna em todo o mundo, incluindo:
10 milhões de crianças
• 24,9 milhões de pessoas em trabalho forçado
• 15,4 milhões de pessoas em casamento forçado
• 4,8 milhões de pessoas em exploração sexual forçada
Alguém está em escravidão se:
• forçado a trabalhar - por coerção ou ameaça mental ou física;
• de propriedade ou controlado por um "empregador", através de abuso físico ou mental ou ameaça
de abuso;
• desumanizado, tratado como uma mercadoria ou comprado e vendido como "propriedade";
• fisicamente constrangido ou com restrições impostas à sua liberdade de movimento.

Aula 04 – English General Presentation 69


www.estrategiamilitar.com.br 70
Teacher Andrea Belo
Aula 04: Escola Naval 2021

A escravidão tem sido um aspecto vergonhoso da sociedade humana durante a maior parte da
história humana. No entanto, a Anti Slavery International se recusou a aceitar que esse sangrento
status quo deva persistir (Aidan McQuade, ex-diretor).
Os objetivos da exploração2 podem variar de prostituição forçada e trabalho forçado a casamento
forçado e remoção forçada de órgãos. Aqui estão as formas mais comuns de escravidão moderna.
• Trabalho forçado - qualquer trabalho ou serviço que as pessoas sejam forçadas a fazer contra sua
vontade3 sob a ameaça de alguma forma de punição.
• Bondage da dívida ou trabalho escravo - a forma mais difundida de escravidão do mundo, quando
as pessoas tomam emprestado dinheiro que não podem pagar e são obrigadas a trabalhar para
quitar a dívida, perdendo o controle sobre as condições de seu emprego e da dívida.
• Tráfico de seres humanos - envolve o transporte, o recrutamento ou o acolhimento de pessoas
com a finalidade de exploração, usando violência, ameaças ou coerção.
• Escravidão baseada na descida - onde as pessoas nascem na escravidão porque seus ancestrais
foram capturados e escravizados; eles permanecem em escravidão por descendência.
• Escravidão infantil - muitas pessoas confundem escravidão infantil com trabalho infantil, mas é
muito pior. Enquanto o trabalho infantil4 é prejudicial para as crianças e dificulta sua educação e
desenvolvimento, a escravidão infantil ocorre quando uma criança é explorada em benefício de
outra pessoa. Pode incluir tráfico de crianças, crianças soldados, casamento infantil e escravidão
doméstica infantil.
• Casamento forçado e precoce - quando alguém é casado contra sua vontade e não pode deixar o
casamento. A maioria dos casamentos infantis pode ser considerada escravidão.
Muitas formas de escravidão têm mais de um elemento listado acima. Por exemplo, o tráfico de
pessoas geralmente envolve pagamento antecipado de viagens e emprego no exterior, usando
dinheiro frequentemente emprestado dos traficantes. Então, a dívida contribui para o controle das
vítimas. Quando chegam, as vítimas não podem sair até que paguem suas dívidas.
Muitas pessoas pensam que a escravidão acontece apenas no exterior, nos países em
desenvolvimento. De fato, nenhum país está livre da escravidão moderna, mesmo a Grã-Bretanha.
O governo estima que existem dezenas de milhares de pessoas em escravidão moderna no Reino
Unido.
A escravidão moderna pode afetar pessoas de qualquer idade, gênero ou raça. No entanto, ao
contrário de um equívoco comum6 de que todos podem ser vítimas de escravidão, alguns grupos de
pessoas são muito mais vulneráveis à escravidão do que outros.
As pessoas que vivem na pobreza7 e têm poucas oportunidades de trabalho decente são mais
vulneráveis a aceitar ofertas de emprego enganosas que podem se tornar exploradoras. As pessoas
que são discriminadas com base em raça, casta ou gênero também têm maior probabilidade de
serem escravizadas. A escravidão também é mais provável de ocorrer onde o estado de direito é
mais fraco e a corrupção é predominante.
A Anti-Slavery International acredita que devemos enfrentar8 as causas profundas da escravidão, a
fim de acabar com a escravidão para sempre. Foi por isso que publicamos nossa Carta Anti-
Escravidão, listando medidas abrangentes que precisam ser tomadas para acabar com a escravidão
em todo o mundo.

Aula 04 – English General Presentation 70


www.estrategiamilitar.com.br 70
Aula 05 –
Pronouns and
Prepositions

ESCOLA NAVAL 2021

Teacher Andrea Belo


Teacher Andrea Belo
Aula 05: EN 2021

Sumário

Introdução ........................................................................................................................ 3

Pronouns .......................................................................................................................... 4

Prepositions .................................................................................................................... 19

Questões inéditas ........................................................................................................... 25

Questões de anos anteriores ........................................................................................... 28

11. Gabarito................................................................................................................. 36

12. Questões comentadas ............................................................................................ 37

13. Considerações finais ............................................................................................... 50

14. Referências bibliográficas .......................................................................................... 51

5. Traduções ................................................................................................................... 53

Aula 05 – Pronouns and Prepositions 2


www.estrategiamilitar.com.br 62
Teacher Andrea Belo
Aula 05: EN 2021

Introdução
Vamos, então, à nossa aula sobre outros importantes tópicos gramaticais: os pronomes e suas
classes; os determinantes e suas propriedades; as preposições e suas variedades e, também, as
conjunções e suas famosas orações coordenadas e subordinadas, entre outras.
Na língua inglesa, existem muitas classes gramaticais, as quais estamos estudando no
decorrer das nossas aulas. Mas, a que substitui e determina os substantivos é, de fato, a classe dos
pronomes (pronouns). E, além de acompanhar os substantivos, define lhes os limites de significação.
Veremos muitos tipos de pronomes nessa aula.
Quantifiers – os determinantes, são expressões usadas para indicar informações a respeito da
quantidade de algo. Veremos muitos tipos deles.
Prepositions – as famosas preposições – as milhares de preposições que existem em Inglês, são
palavras que ligam termos e estabelece relação entre eles. Vamos estudar todas as possíveis
preposições que podem aparecer nas provas de Carreira Militar.
As Conjunctions – conjunções – são palavras que ligam duas palavras, ligando, assim, duas frases,
das quais veremos muitas e inúmeros exemplos para que fique claro o uso delas nas abundantes
orações em que aparecem.
As línguas, de forma geral, são sistemas que devemos internalizar naturalmente, mas, como
o conhecimento da língua consiste em uma fragmentação de conteúdos em regras (e essas cheias
de exceções), fica complicado aprender e usar bem todos esses tópicos gramaticais, todo esse
conteúdo proposto em nossas aulas, eu sei disso.
Até porque uma língua é parte integrante de uma cultura e de suas relações sociais e, como
brasileiro, você estuda a língua estrangeira como um elemento externo, para capacitar você a
conseguir ler e responder as questões no dia da sua prova.
Mas, se seu objetivo é ser aprovado, imagine que por trás de cada texto, há formas de
desvendar os elementos ali presentes através das análises linguísticas e das técnicas, que tanto
explico e ensino você, como ajuda para sua aprovação.
Entender todos os itens das aulas, dia após dia, é abrir uma porta, aliás, abrir portas para um
mundo novo: o conhecimento e propriedade para utilizar no dia da prova.
E você vai conseguir. Vou fornecer ferramentas específicas para simplificar o estudo de
pronomes, de determinantes, de preposições, de conjunções, assim como de todos os assuntos que
abrangem sua prova, e, uma vez preparado para uma dessas provas, da maneira como estou
explicando e contextualizando para você, o caminho para a aprovação ficará menor de ser
percorrido.
Vamos lá! Você consegue e será o melhor candidato!

Aula 05 – Pronouns and Prepositions 3


www.estrategiamilitar.com.br 62
Teacher Andrea Belo
Aula 05: EN 2021

Pronouns
Os prounouns – pronomes – são palavras que acompanham os substantivos e podem
substituí-los (direta ou indiretamente), retomá-los ou se referir a eles.
Estudaremos todos os tipos de pronomes, tais como: personal, possessive, subject, object,
adjective, reflexive, demonstrative, indefinite e interrogative pronouns . Vou explicar todos eles
usando a metodologia que eu desenvolvi, especialmente para o curso do Estratégia Carreiras
Militares, uma maneira facilitadora de compreender aquilo que a gramática complica.
Os pronomes servem para remeter, retomar ou qualificar palavras do texto. Sem os
pronomes, as orações ficariam confusas e longas demais, como por exemplo, ao citar os nomes de
3 pessoas em uma frase, podemos usar o pronome sujeito they (eles/elas), remetendo àquelas
pessoas, veja: Elizabeth, Lourdes and Alexander play golf together. They play golf together.
Os pronomes em Inglês, de acordo com a função que exercem, são classificados em:

❖ pronomes pessoais (personal pronouns: subject e object pronouns)


❖ pronomes possessivos (possessive pronouns: possessive adjectives and possessive prounouns)
❖ pronomes demonstrativos (demonstrative pronouns)
❖ pronomes reflexivos (reflexive pronouns)
❖ pronomes indefinidos (indefinite pronouns)
❖ pronomes relativos (relative pronouns)
❖ pronomes interrogativos (interrogative pronouns)

Personal pronouns

Os personal pronouns – pronomes pessoais, podem indicar pessoas, lugares e objetos. São
classificados em:
❖ Subject Pronouns: os pronomes que funcionam como sujeitos, como por exemplo:

Lavinia is beautiful – She is beautiful.

noun subject pronoun

❖ Object Pronouns: os pronomes que funcionam como objetos, por exemplo:

I want to go to the park with Lavinia – I want to go to the park with her.

noun object pronoun

Aula 05 – Pronouns and Prepositions 4


www.estrategiamilitar.com.br 62
Teacher Andrea Belo
Aula 05: EN 2021

Com meu método de explicação, o resumo é o seguinte:


Os pronomes sujeitos, subject prounouns – vêm no início das frases, em sua maioria.
Isso porque, o sujeito de uma frase é a pessoa que realiza a ação, certo?
Pode até ser que o subject pronoun apareça no meio de uma frase mas têm a função
de sujeito, de quem realiza a ação, como por exemplo:

He works at the office every day – Every day, he works at the office.

(Ele trabalha no escritório todos os dias.) (Todos os dias, ele trabalha no escritório.)

Já os object pronouns sempre aparecem no fim das frases, na função de objeto mesmo,
que seriam os objetos direto e indireto, mas isso não precisa ser lembrado para entender e
aprender os pronomes que estamos estudando.
O que você precisa saber (e que nenhum livro diz) é que os pronomes sujeitos e objetos
têm o mesmo significado, porém colocados em lugares diferentes da frase. Se eu falar “Ela
estuda” é “She studies” mas, “Eu estudo com ela”, com a palavra ela no final da frase, ela vira
um pronome objeto e fica: “I study with her”, entendeu o bizu?
Como esses pronomes estão correlacionados, fiz uma tabela para você. Veja qual pronome
sujeito se refere a cada pronome objeto abaixo, veja os dois tipos de personal pronouns:

EXAMPLES:

I am the person. It’s me. (Eu sou a pessoa. Sou eu).

You are the person. It’s you. (Você é a pessoa. É você).

He is the person. It’s him. (Ele é a pessoa. É ele).

She is the person. It’s her. (Ela é a pessoa. É ela).

It is the object. It’s it. (Ele é o objeto. É ele).

We are the people. It’s us. (Nós somos as pessoas. Somos nós).

You are the person. It’s you. (Vocês são as pessoas. São vocês).

They are the people. It’s them. (Eles são as pessoas. São eles).

Aula 05 – Pronouns and Prepositions 5


www.estrategiamilitar.com.br 62
Teacher Andrea Belo
Aula 05: EN 2021

Possessive Pronouns

Os pronomes a serem apresentados, em cada subcapítulo do nosso material, serão explicados


de forma prática e compreensível, com exemplos e quadros ilustrativos, propiciando uma melhor
compreensão. Vamos lá.
Sobre os possessive pronouns, são aqueles que indicam que algo pertence a alguém. São
classificados em possessive adjectives e possessive pronouns (pronomes adjetivos e pronomes
substantivos).
Veremos cada um deles separadamente, para que você possa diferenciá-los e encontrá-los
na prova, caso seja solicitado uma substituição ou análise de frases com eles.
Possessive Adjectives: Esses pronomes são sempre são acompanhados por substantivos,
modificando-os. E eles não têm singular e plural, por exemplo: my significa meu, minha, meus e
minhas, assim como your significa seu, sua, seus, suas e assim por diante, veja:

Before leaving, I got my wallet, my keys, my laptop and my books.

(Antes de sair, eu peguei minha carteira, minhas chaves, meu laptop e meus livros.)

EXAMPLES:

My car is black. (Meu carro é preto).

Where is your car? (Onde está o seu carro?)

His car is black. (O carro dele é preto).

Where is her car? (Onde está o carro dela?)

Where is its car? (Onde está o carro dela (loja/concessionária?)

Where is our car? (Onde está o nosso carro?)

Your car is black. (Seu carro é preto).

Their car is black. (O carro deles/delas é preto).

Aula 05 – Pronouns and Prepositions 6


www.estrategiamilitar.com.br 62
Teacher Andrea Belo
Aula 05: EN 2021

Possessive Pronouns: esses pronomes têm a função de substituir o substantivo,


diferentemente dos pronomes adjetivos, que sempre estão ligados a ele. Eles também não sofrem
flexão de grau - singular e plural.
O mais importante para você saber é que, esses pronomes sempre estarão no final das frases,
sempre! Assim fica mais fácil de encontrá-los nos textos das provas e também saber suas funções
corretas, já que são similares e não podem ser confundidos.
Na hora da sua prova, em sua análise das alternativas de escolha de cada pergunta, você
precisa usar as técnicas certas – inclusive aprendida em nossa primeira aula – para ler os textos com
maior agilidade e compreender termos que levarão à resolução com segurança.
Veja um exemplo e o quadro ilustrativo:

Whose umbrella is this? It’s mine. And the keys? They are mine.

(De quem é esse guarda-chuva? É meu. E as chaves? São minhas.)

EXAMPLES:

Whose books are these? De quem são esses livros? They are mine. (São meus).

Whose books are these? De quem são esses livros? They are yours. (São seus).

Whose books are these? De quem são esses livros? They are his. (São dele).

Whose books are these? De quem são esses livros? They are hers. (São dela).

Whose books are these? De quem...livros? They are its. ( São dela - loja/biblioteca).

Whose books are these? De quem são esses livros? They are ours. (São nossos).

Whose books are these? De quem são esses livros? They are yours. (São seus).

Whose books are these? De quem são esses livros? They are theirs. (São meus).

Aula 05 – Pronouns and Prepositions 7


www.estrategiamilitar.com.br 62
Teacher Andrea Belo
Aula 05: EN 2021

E agora, um quadro só, com os dois tipos de possessive pronouns, para ficar mais fácil ainda
de memorizar e comparar ambos pronomes. E, não se esqueça dos bizus de como memorizá-los com
mais facilidade. Vamos lá!

Aula 05 – Pronouns and Prepositions 8


www.estrategiamilitar.com.br 62
Teacher Andrea Belo
Aula 05: EN 2021

Reflexive Pronouns

Os reflexive pronouns – pronomes reflexivos, são aqueles que praticam a ação sobre si mesmo
(eu mesmo, você mesmo etc) e aparecem após os verbos. São usados de acordo com o sujeito da
oração, sempre. São palavras que têm os sufixos “-self” (singular) e “-selves” (plural).
Os reflexive pronouns possuem 3 funções:
❖ Função reflexiva: é quando há concordância com o sujeito e aparece depois dele:

Lisa was looking herself in the mirror. (Lisa estava se olhando no espelho).

❖ Função enfática: é quando o pronome concorda com o sujeito e se posiciona depois


desse sujeito ou então do objeto.

Lisa herself prepared the sandwich. (Lisa mesma preparou o sanduíche).

❖ Função idiomática: é quando o pronome vem precedido da preposição by, para indicar
que o sujeito praticou a ação sozinho.

Lisa likes studying by herself. (Lisa gosta de estudar sozinha. – só ela.)

Veja um exemplo e o quadro ilustrativo:

EXAMPLES:

I cut myself with a knife. Eu me cortei com a faca.

You cut yourself with a knife. Você se cortou com a faca.

He cut himself with a knife. Ele se cortou com a faca.

She cut herself with a knife. Ela se cortou com a faca.

It cut itself with a knife. Ele se cortou com a faca. (um animal, por exemplo)

We cut ourselves with a knife. Nós nos cortamos com a faca.

You cut yourselves with a knife. Vocês se cortaram com a faca.

They cut themselves with a knife. Eles se cortaram com a faca.

Aula 05 – Pronouns and Prepositions 9


www.estrategiamilitar.com.br 62
Teacher Andrea Belo
Aula 05: EN 2021

Vejamos uma tabela dos pronomes que estudamos até agora, com as devidas classificações.

Aconselho que você tente escrever exemplos para cada pronome acima com o intuito de
treinar. E, quando aparecerem pronomes nas questões , sublinhe, circule, encontre uma forma de
destacá-los e descobrir que pronome é para aprimorar os conhecimentos.

Já estudamos muitos pronomes: os pessoais, possessivos, e reflexivos. Ainda faltam os


pronomes demonstrativos, indefinidos, relativos e interrogativos.

Você já percebeu que os pronomes, em geral, são aquelas palavras que substituem ou
acompanham outras, principalmente os substantivos, certo?

Afinal, se analisarmos a própria palavra pronome, “pro” significa “em função de”. Logo, é
possível concluir que “pronome” é sinônimo de algo que está “em função do nome”.

Por isso, os pronomes existem para remeter, retomar ou qualificar outras palavras expressas
nos textos das provas de Carreira Militar.

E assim, se torna tão importante identificá-los, para ter a certeza de que cada alternativa faz
sentindo, analisando cada termo que faz parte dos textos.

Mas ainda há muito o que estudar. Preparado?

Então, vamos aos pronomes demonstrativos. Let’s go!

Aula 05 – Pronouns and Prepositions 10


www.estrategiamilitar.com.br 62
Teacher Andrea Belo
Aula 05: EN 2021

Demonstrative Pronouns

Os demonstrative pronouns – pronomes demonstrativos, são apenas 4 e são simples de


entender. São usados para situar a posição de algo no espaço em que se encontra.
São aqueles pronomes que mostram, que indicam algo que pode estar perto ou longe. Veja
uma tabela e uma ilustração com esses pronomes.

Pela simplicidade, os pronomes demonstrativos podem ser reconhecidos nos textos facilitar na
busca pelas respostas das questões que lá estiverem. Vamos estudar os pronomes indefinidos e dar
continuidade ao seu aprendizado.

Aula 05 – Pronouns and Prepositions 11


www.estrategiamilitar.com.br 62
Teacher Andrea Belo
Aula 05: EN 2021

Indefinite Pronouns

Os indefinite pronouns – pronomes indefinidos, recebem esse nome porque substituem ou


acompanham o substantivo, porém, de forma indeterminada, como?
Eles oferecem a ideia, por exemplo, de algum lugar, alguma coisa, alguém, qualquer lugar,
qualquer pessoa, entre outros nomes imprecisos dentro das orações.
Os pronomes indefinidos começam com as palavras some, any, no e every. E terminam com
os sufixos -body, -one, -thing, -where, entre outros, a depender do contexto.
Em geral, usa-se -some, -every e -no em frases afirmativas, -any em frases negativas e
interrogativas.
Aqui, vale lembrar algumas diferenças entre some e any, só para ilustrar: some para frases
afirmativas e any para negativas e interrogativas, ambas palavras significam algum, alguma, alguns,
algumas e, dependendo do contexto, nas negativas, significam nenhum/nenhuma, ok?
Vejamos alguns exemplos para ilustrar:

São muitos os pronomes indefinidos e, por isso, precisamos de analisar diferentes exemplos
que possam estar nos textos no dia da sua prova de Carreira Militar.
Vou mostrar variados exemplos abaixo, nos quadros que seguem:

I think somebody has arrived. Eu acho que alguém chegou.

I believe someone forgot the book. Eu acredito que alguém esqueceu o livro.

She said something is wrong. Ela disse que alguma coisa está errada.

He is somewhere in Europe. Ele está em algum lugar na Europa.

I have to get there someway. Eu tenho que chegar lá de alguma maneira.

Aula 05 – Pronouns and Prepositions 12


www.estrategiamilitar.com.br 62
Teacher Andrea Belo
Aula 05: EN 2021

I can’t see anybody in the room. Não consigo ver ninguém na sala.

He didn’t see anyone there. Ele não viu ninguém lá.

I won’t do anything at the moment. Não consigo fazer nada nesse momento.

I can’t go anywhere. Não posso ir em lugar nenhum/qualquer lugar.

She is going home anyway. Ela está indo para casa de qualquer forma/jeito.

She said nobody won the medal. Ela disse que ninguém ganhou a medalha.

No one came to college yesterday. Ninguém veio à faculdade ontem.

None of the exercises are right. Nenhum dos exercícios está certo.

There is nothing I can do. Não há nada que eu possa fazer.

She is going nowhere. Ela não está indo para lugar nenhum.

She is going home anyway. Ela está indo para de qualquer forma/jeito.

Everybody is going to the party. Todos vão para a festa.

I talked to everyone at the party. Eu falei com todos da festa.

She is doing everything she can. Ela está fazendo tudo o que pode.

I’ve been everywhere in this city. Já estive em todos os lugares dessa cidade.

He drove every way at the race. Ele dirigiu em todos os sentidos na corrida.

Aula 05 – Pronouns and Prepositions 13


www.estrategiamilitar.com.br 62
Teacher Andrea Belo
Aula 05: EN 2021

Os pronomes vistos acima, com os respectivos exemplos, são muito cobrados nas provas de
Carreira Militar, de forma geral.
E, podem aparecer outros, que estão no quadro aqui ao lado para você, já com os significados.
Assim, você pode visualizar de fato, em que situações eles aparecem e com dar sentido a eles
na hora em que você estiver lendo os textos da prova.

Uma vez dominado essas palavras, a compreensão e busca pelas respostas fica mais acessível,
não é mesmo?
Por isso, eu sempre gosto de enfatizar a importância de se conhecer os termos gramaticais,
sejam eles pronomes, verbos, preposições, artigos, ou melhor, como seria bom conhecer e dominar
todos eles para realizar uma excelente prova.
E, basta estudar! Conte comigo.
Em continuidade à aula, vamos estudar os pronomes relativos.

Aula 05 – Pronouns and Prepositions 14


www.estrategiamilitar.com.br 62
Teacher Andrea Belo
Aula 05: EN 2021

Relative Pronouns

Os relative pronouns – pronomes relativos, são palavras que exercem a função de sujeito ou
de objeto nas frases.
Quando for sujeito, haverá um substantivo antes desse pronome. Mas, quando aparecer após
um verbo, com ou sem preposição, então, estamos falando de um pronome relativo com função de
objeto.
Em ambas situações, o que importa saber é a estrutura e entender para que e onde são
usados tais pronomes nos textos da prova, independente da função que exercem.
Vejamos um quadro com os nomes dos pronomes e vamos falar dos detalhes de cada um
deles para compreender melhor.

Na função de sujeito, os pronomes relativos são: who (para pessoas), which (para
objetos/animais) e that (para pessoas e objetos: neutro).

The man who arrived is charming. (O homem que chegou é charmoso).

O pronome se refere ao homem que chegou: o sujeito

Na função de objeto, temos os pronomes relativos who/whom (para pessoas) e também


which (para objetos/animais) e that (para pessoas e objetos: neutro).

She didn’t say who arrived. (Ela não disse quem chegou).

O pronome se refere ao homem que chegou (objeto) e não a ela (sujeito)

Vejamos outros exemplos com o uso de diferentes pronomes relativos

The girl who disappeared studied with me. (A garota que desapareceu estudava comigo).
O pronome se refere à garota que desapareceu: o sujeito

They didn’t show who the magician was. (Eles não mostraram quem era o mágico).

O pronome se refere ao mágico que chegou: o sujeito

Aula 05 – Pronouns and Prepositions 15


www.estrategiamilitar.com.br 62
Teacher Andrea Belo
Aula 05: EN 2021

Veja o quadro ilustrativo:

Os pronomes vistos acima, com os respectivos exemplos, também são muito cobrados nas
provas.
E, independente da maneira como esses pronomes vierem nos textos, se você entender o
funcionamento de cada um deles dentro do contexto, saberá responder as questões da prova.
Assim, você pode visualizar de fato, em que situações eles aparecem e com dar sentido a eles
na hora em que você estiver lendo os textos da prova.

Aula 05 – Pronouns and Prepositions 16


www.estrategiamilitar.com.br 62
Teacher Andrea Belo
Aula 05: EN 2021

Interrogative Pronouns

Os interrogative pronouns – pronomes interrogativos, também chamados de Question


Words, são aquelas palavras utilizadas para fazer perguntas, termos que representam aquilo que
queremos saber: o lugar, o motivo, a hora, o dia, entre outros.
Vejamos alguns exemplos:

What is your favorite fruit? Grapes. (Qual é a sua fruta favorita?) Uvas.

Which fruit do you prefer, apple or pear? Pear. (Qual fruta você prefere, maçã ou pera?) Pera.

Where do you go on the weekends? To the club. (Onde você vai nos fins de semana?) Ao clube.

When is Christmas? It’s on December 24 th. (Quando é o Natal? É 24 de dezembro).

How do you go to work? By bus. (Como você vai para o trabalho?) De ônibus.

Who is that boy over there? It’s my brother. (Quem é aquele garoto lá?) É meu irmão.

To whom did he give the present? To me. (Para quem ele deu o presente?) Para mim.

Whose house is that ? It’s mine.(De quem é aquela casa?) Minha.

Why do you study English? Because I need. (Por que você estuda alemão?) Porque eu preciso.

Observação: Você percebeu que usamos dois pronomes interrogativos para fazer perguntas
com a palavra qual – WHAT/WHICH – sendo What para perguntas gerais “O quê/Qual” mas, se for
uma escolha – “Qual você prefere, esse ou aquele?”, usamos “Which”.
‘What’ e ‘Which’, apesar de terem a mesma tradução, o momento de uso é diferente para
cada um deles, já que ‘what’ não apresenta restrições como o pronome interrogativo ‘which’ é usado
somente quando há um número limitado de opções.

Aula 05 – Pronouns and Prepositions 17


www.estrategiamilitar.com.br 62
Teacher Andrea Belo
Aula 05: EN 2021

Vejamos o quadro ilustrativo:

Os pronomes interrogativos, nos textos da prova, são usados quando há perguntas dentro de
um contexto e, na maioria das vezes, a resposta logo em seguida.
Ou, às vezes, são leituras mais técnicas, reflexivas, artigos científicos, entre outros, em que
há perguntas sem resposta dentro dos textos. E precisam de respostas mais amplas do que um “sim”
ou um “não”. Por isso aparecem nas alternativas de resposta algumas opções como um lugar ou
horário e, se você souber qual é o pronome, acerta a questão.
Veremos questões com esses pronomes adiante para praticar.

Agora vamos estudar as preposições. Come on!

Aula 05 – Pronouns and Prepositions 18


www.estrategiamilitar.com.br 62
Teacher Andrea Belo
Aula 05: EN 2021

Prepositions
As prepositions (preposições) são palavras ou grupos de palavras que ligam e estabelecem relações
dentro das frases.
Nessas relações, um termo explica ou completa o sentido do outro. Vejamos as preposições mais
importantes e mais usadas nas provas: preposições de lugar e de tempo.
Vou mostrar uma imagem com as preposições principais e comentar sobre cada uma delas em
seguida. Isso porque, algumas preposições podem ser de lugar e de tempo, dependendo do
contexto. E vamos analisar para que você entenda bem. Vamos lá.

Prepositions of place

✓ In - dependendo do contexto pode significar: dentro de; em; de; no e na.


✓ On - dependendo do contexto pode significar: sobre a; em cima de; acima de; em; no; na.
✓ At - dependendo do contexto pode significar: à; em; na; no.
✓ To - dependendo do contexto pode significar: para; a.
✓ For - dependendo do contexto pode significar: para; durante; por.

Aula 05 – Pronouns and Prepositions 19


www.estrategiamilitar.com.br 62
Teacher Andrea Belo
Aula 05: EN 2021

In
A preposição in é utilizada nos seguintes casos:
1) Para indicar tempo, seja o ano, o mês, as estações do ano ou uma parte do dia.
Exemplos:
• I study in the evening. (Eu estudo a noite.)
• He plays volleyball in the afternoon. (Ele joga vôlei de tarde.)
• My birthday is in April. (Meu aniversário dela é em abril.)
• My brother was born in 2012. (Meu irmão nasceu em 2012.)
• We always visit New York in the summer. ( Nós sempre visitamos Nova Iorque no verão).
Nessa regra existe uma exceção em relação ao termo “night”. Nesse caso, a preposição utilizada é o
“at”, por exemplo: at night (à noite – madrugada, após dormir).

2) Para indicar lugar, seja uma cidade, um país ou qualquer local específico.
Exemplos:
• She lives in Italy. (Ela mora na Itália.)
• He works in São Paulo. (Ele trabalha em São Paulo.)
• Some people like to have a TV in the bedroom. (Algumas pessoas gostam de ter uma TV no
quarto.)
• They left the dog in the house. (Eles deixaram o cachorro na casa.)
• He is waiting for me in the living room. (Ele está me esperando na sala.)

On
A preposição on é utilizada nos seguintes casos:
1) Para indicar tempo. No entanto, diferentemente do “in” ela é usada para datas específicas.
Exemplos:
• I was born on March 10th. (Eu nasci no em 10 de março.)
• He studies English on Tuesdays and Thursdays .(Ele estuda Inglês às terças e quintas.)
• Joanne bought a new car on November 17th. (Joanne comprou um carro novo dia 17 de
novembro.)

Aula 05 – Pronouns and Prepositions 20


www.estrategiamilitar.com.br 62
Teacher Andrea Belo
Aula 05: EN 2021

• They always go out on Saturdays. (Eles sempre saem aos sábados.)


• I will travel on December 18th. ( Vou viajar dia 18 de dezembro.)
• She will go to the park on Sunday. (Ela vai ao parque no domingo.)

2) Para indicar lugares e objetos. Todavia, diferentemente do “in” ela é usada para locais e objetos
que possuem uma superfície. Nesse caso, ela significa em cima de.
Exemplos:
• The book is on the bed. (O livro está em cima da cama).
• The cushion is on the floor. (A almofada está no chão.)
• My dog sleeps on the couch. (Meu cachorro dorme em cima do/no sofá).
• I will put the paper on my desk. (Vou colocar o papel em cima da/na mesinha.)
• She left her blouse on that chair. (Ela deixou a blusa dela em cima daquela/naquela cadeira.)

3) Para referir meios eletrônicos onde informações são disponibilizadas.


Exemplos:
• I checked that information on the company’s website. (Eu chequei aquela informação no site
da empresa.)
• Don’t believe everything you see on the Internet. (Não acredite em tudo que você lê na
Internet.)
• The principal can’t talk to you now because he is on the phone. (O diretor não pode falar com
você agora pois está ao telefone.)
• They watched the concert live on TV. (Eles assistiram o show ao vivo na TV.)

4) Para indicar nomes de ruas ou avenidas.


Exemplos:
• I live on Alvaro Luiz Street. (Eu moro na rua Álvaro Luiz.)
• She wrote a book about a guy who lived on Seventh Street. (Ela escreveu um livro sobre um
cara que morava na Seventh Street.)
• There are many famous places on Oxford Street. (Há muitos lugares famosos na rua Oxford.)
• There is a bull statue on Wall Street, in New York. (Há uma estátua de um touro na Wall
Street.)
• They've got a company on Madison Avenue. (Eles têm uma empresa na Madison Avenue).
Entretanto, se ao endereço for acrescido o número, utiliza-se o "at":
Exemplos:

Aula 05 – Pronouns and Prepositions 21


www.estrategiamilitar.com.br 62
Teacher Andrea Belo
Aula 05: EN 2021

• I live at 300 Karl Street. (Eu moro na rua Karl, nº 300.)


• He wrote a book about a man who lived at 25 Baker Street. (Ele escreveu um livro sobre um
homem que morava na Baker Street, nº 25.)

At
A preposição at é utilizada nos seguintes casos:

1) Para indicar horários


Exemplo:
• I woke up at 6 am. (Acordei às 06:00h.)
• The concert is at 8 pm. (O show é às 20h.)
• Our flight will leave at 3 o'clock. (Nosso voo sairá às três horas.)
• My friends usually eat dinner at 10 pm. (Meus amigos geralmente jantam às 10h.)
• She finished the exam at 2 pm. (Ela terminou a prova às 14h.)

2) Para indicar locais específicos.


Exemplos:
• She has snacks at work every day. (Ela lancha no trabalho todos os dias.)
• I saw him reading the newspaper at the café. (Eu o vi lendo o jornal no café.)
• She is at the hospital to visit her mom. (Ela está no hospital para visitar sua mãe.)
• She is waiting for her dad at the airport. (Ela esta esperando o pai no aeroporto.)

To
A preposição to é utilizada nos seguintes casos:
1) Para indicar movimento, posição, destino ou direção.
We are going to Greece. (Nós vamos para a Grécia).

2) Para indicar duração de tempo (início e fim de um período).


I studied English from 2015 to 2018. (Eu estudei Inglês de 2015 a 2018).

Aula 05 – Pronouns and Prepositions 22


www.estrategiamilitar.com.br 62
Teacher Andrea Belo
Aula 05: EN 2021

3) Para indicar distância.


It’s about 2 blocks from the supermarket to my house. (São cerca de 2 quarteirões do supermercado
até minha casa).

4) Para indicar comparação entre coisas.


I prefer going to the movies to watch TV (Eu prefiro ir ao cinema a assistir TV).

5) Para indicar o motivo ou propósito. Nesse caso, a preposição é seguida de verbo.


We go out to relax and have fun. (Nós saímos para relaxar e nos divertir).

For
A preposição for é utilizada nos seguintes casos:
1) Para indicar duração de tempo.
I’ve worked at school for six years. (Trabalhei na escola por seis anos).

2) Para indicar finalidade. Nesse caso, é seguido de gerúndio.


A tape is used for fixing things. (Uma fita é usada para consertar as coisas).

3) Para indicar benefício ou favor.


Working out is very good for the health. (Fazer exercícios é muito bom para a saúde).

4) Para indicar motivo ou propósito. No entanto, diferentemente do “to” ele é seguido de


substantivo.
This space is for guests only. (Este espaço é só para convidados).

Vimos que as preposições são palavras curtas, que geralmente vem depois de um substantivo.
As preposições são muito comuns nas provas.
Pelo fato de haver muitas preposições com vários significados e, como não há uma regra
específica de como e quando devemos usá-las, o melhor jeito para aprender as preposições é lendo
muito, fazendo muitos exercícios de provas anteriores para aprender aos poucos.
Vou mostrar a você um quadro ilustrativo com as preposições que mais apareceram em
exercícios de variadas provas e que, são realmente mais comuns de forma geral.
E, como eu já disse, o ideal é o estudo constante. Conte comigo!

Aula 05 – Pronouns and Prepositions 23


www.estrategiamilitar.com.br 62
Teacher Andrea Belo
Aula 05: EN 2021

Agora, teremos questões inéditas que elaborei, aos moldes das grandes provas de Carreira
Militar do Brasil inteiro, para você treinar.

Aula 05 – Pronouns and Prepositions 24


www.estrategiamilitar.com.br 62
Teacher Andrea Belo
Aula 05: EN 2021

Questões inéditas

Questão inédita Teacher Andrea Belo


Read the text and answer the question below.

Questão 01 (Inédita – Teacher Andrea Belo)


De acordo com o texto, podemos afirmar que:
A ( ) Os gastos de hoje definem se você pode ou não gastar amanhã.
B ( ) Hoje é bem mais divertido para gastar do que esperar o dia de amanhã.
C ( ) Gastos, de forma geral, são algo divertido, hoje ou amanhã.
D ( ) Gastar hoje é mais divertido do que esperar para gastar amanhã.
E ( ) Os gastos em geral são contabilizados hoje e não esperam o amanhã.

Aula 05 – Pronouns and Prepositions 25


www.estrategiamilitar.com.br 62
Teacher Andrea Belo
Aula 05: EN 2021

Comentários:
O trecho do texto acima, publicado no dia 24 de julho de 2019, no jornal The Economist, uma
fonte muito usada em provas de Carreiras Militares, indicando o assunto a partir da imagem e
das letras em negrito, além das variadas palavras cognatas.
O trecho é curto, retirado de um grande artigo jornalístico mas, com apenas uma parte dele,
vai facilitar para que eu mostre a você o que é preciso fazer para encontrar a resposta.
Vamos ler e analisar cada alternativa para encontrar aquela que melhor se encaixa com as
informações oferecidas através da leitura.
Na letra A, afirma-se que “Os gastos de hoje definem se você pode ou não gastar amanhã”
mas, para ser verdade, provavelmente haveria uma compound sentence com a conjunção OR:
“Spending today define if you can or can’t spend tomorrow” (Gastar hoje define se você pode
ou não gastar amanhã) mas, no texto, onde se fala no verbo gastar (spend), a frase é “Spending
today is more fun than witing to spend tomorrow: Gastar hoje é mais divertido do que esperar
para gastar amanhã, com os verbos gastar (spend) e esperar (wait). Falsa.
Na letra B, afirma-se que hoje é bem mais divertido para gastar do que esperar o dia de
amanhã, mas, pela análise que fizemos sobre a letra A, vimos que, apesar de parecer
verdadeira, o “amanhã” que o texto se trata não é o dia de amanhã e sim o futuro, o amanhã,
os dias que virão. Cuidado com esse tipo de alternativa – O ITA coloca essas afirmações com
uma palavra que não é o que o texto quer dizer de fato para confundir você: que eu chamo de
“tricky” – pegadinhas.
Na letra C, afirma-se que gastos, de forma geral, são algo divertido, hoje ou amanhã. Falsa pela
análise que já fizemos anteriormente. E, não se usa, no texto, a conjunção OR, como na
alternativa apresentada. Falsa.
Na letra D, afirma-se que gastar hoje é mais divertido do que esperar para gastar amanhã.
Justamente o que fala no texto, não definindo o dia de amanhã, como na letra B mas, se
tratando do amanhã de forma geral, como a ideia representa. Resposta correta: D
A letra E, diz que os gastos em geral são contabilizados hoje e não esperam o amanhã. Além de
ser falsa pela descoberta do que foi exatamente afirmado no texto, não se fala em contabilizar
os gastos em nenhuma parte do texto.

Questão 02 (Inédita – Teacher Andrea Belo)


Choose the correct option:
A ( ) Retirement encourage people on spending money today.
B ( ) Spending tomorrow is better than spending today because of the retirement guarantees.
C ( ) Spending is good anyway: today or tomorrow or anytime you have money.
D ( ) Retirement is described as something full of possibilities but complex.
E ( ) When you accumulate money, you have to think about what to do with it.

Aula 05 – Pronouns and Prepositions 26


www.estrategiamilitar.com.br 62
Teacher Andrea Belo
Aula 05: EN 2021

Comentários:
Aqui, devemos analisar cada alternativa, para encontrar palavras que se assemelham às que
estão descritas em cada opção de resposta.

E também, achar, através do scanning, skimming e seus conhecimentos adquiridos, aula após
aula, o que há de absurdo, ou seja, algo que possamos identificar que não aparece no texto e
eliminar tal alternativa, para facilitar na busca pela melhor delas.

Na letra A, afirma-se que a aposentadoria (retirement) encoraja pessoas a gastar o dinheiro


mas, não há a comprovação disso no texto (e já falei que são alternativas a serem eliminadas).
Para ter a certeza, você continua em busca da alternativa que mais faça sentido ao buscar
aquelas ideias no texto. Vamos lá. É importante observar que o verbo encourage, pede a
preposição on como complemento, significando “em” – encorajar alguém a ou em fazer algo.
Na letra B, afirma-se que gastar hoje (today) é melhor do que amanhã (tomorrow) por causa
das garantias da aposentadoria (retirement guarantees) mas, novamente aqui, outra
informação que não está presente no texto. Falsa.

Na letra C, afirma-se que gastar hoje ou amanhã é bom de qualquer jeito: aqui atenção. São
afirmações que parecem ser verdade mas, de acordo com o texto, não são. Não se usa, no
texto, a conjunção OR, como na alternativa apresentada. Falsa.

Na letra D, afirma-se que a aposentadoria é cheia de possibilidades (full of possibilities) mas


complexa (but complex). Apesar de aparecer a conjunção de contraste BUT, importante para
revisar os conhecimentos dessa nossa aula, eis outra informação que parece ser verdade mas
não está presente no texto.

A letra E, além de ser a opção correta por eliminação do que há de errado nas outras, podemos
comprovar, através do falso cognato “amass”, que está correta. Atenção: amass não é amassar,
como parece, e sim, acumular, exatamente o que afirma nessa alternativa: quando você
acumula dinheiro (accumulate ou amass money), tem que pensar no que fazer (think about
what to do, no texto: decide what to do, sinônimo).
Alternativa correta: E

Aula 05 – Pronouns and Prepositions 27


www.estrategiamilitar.com.br 62
Teacher Andrea Belo
Aula 05: EN 2021

Questões de anos anteriores


Você agora vai resolver questões de anos anteriores de inúmeras e diferentes Carreiras Militares
para treinar vocabulário e exercícios variados. E, em seguida, as respostas comentadas. Vamos lá!

QUESTÃO 01 (EEAR/2017)

QUESTÃO 01 – Choose the best alternative to fill in the blanks subsequently.


a) a – some – the – some
b) the – the – the – the
c) the – some – some – a
d) some – a – some – the

QUESTÃO 02 – Choose the alternative in which all the words are uncountable.
a) salt – cake – egg
b) chocolate – butter – walnut
c) flour – baking powder – sugar
d) vanilla – cake tin – ice cream

Aula 05 – Pronouns and Prepositions 28


www.estrategiamilitar.com.br 62
Teacher Andrea Belo
Aula 05: EN 2021

2018/COLÉGIO NAVAL - Questão 03

QUESTÃO 03

Aula 05 – Pronouns and Prepositions 29


www.estrategiamilitar.com.br 62
Teacher Andrea Belo
Aula 05: EN 2021

2017/EPCAR
Questão 04

QUESTÃO 04 - The word


“workplace” (line 10)
Means:

a) an institution for people


who need professional care

b) a place where people work.

c) home for old people.

d) a school where people


live.

Aula 05 – Pronouns and Prepositions 30


www.estrategiamilitar.com.br 62
Teacher Andrea Belo
Aula 05: EN 2021

2018/ESCOLA NAVAL
Questão 05

Aula 05 – Pronouns and Prepositions 31


www.estrategiamilitar.com.br 62
Teacher Andrea Belo
Aula 05: EN 2021

QUESTÃO 05

Aula 05 – Pronouns and Prepositions 32


www.estrategiamilitar.com.br 62
Teacher Andrea Belo
Aula 05: EN 2021

2018/AFA - Questão 06

2018/AFA - Questão 06 - The text

a) points out how well Burkina Faso dealt with food shortage.

b) shows a remaining difficulty as well as its way out.

c) highlights the importance of urbanization for crop fields.

d) states that land degradation is a natural impact for today’s climatic

stability.

Aula 05 – Pronouns and Prepositions 33


www.estrategiamilitar.com.br 62
Teacher Andrea Belo
Aula 05: EN 2021

2017/EFOMM - Questão 07

Aula 05 – Pronouns and Prepositions 34


www.estrategiamilitar.com.br 62
Teacher Andrea Belo
Aula 05: EN 2021

2017/EFOMM - Questão 07 - Which option can NOT be inferred from the text? A pidgin
language is:
( a ) a simplified means of linguistic communication.
( b ) the native language of a speech community.
( c ) employed in situations such as commerce.
( d ) a contact language.
( e ) constructed impromptu, or by convention, between individuals or groups of people.

2017/EFOMM - Questão 08 - In line 57, “Lingua Franca” is:


( a ) a language that people use to communicate when they have different first languages.
( b ) a distinctive lect, or variety of English spoken by African Americans.
( c ) an informal language consisting of words and expressions that are not considered
appropriate for formal occasions.
( d ) a body of words and phrases that apply to a specific activity or profession, such as a
particular art form or a medical or scientific subject.
( e ) the language that a person has spoken from earliest childhood.

Aula 05 – Pronouns and Prepositions 35


www.estrategiamilitar.com.br 62
Teacher Andrea Belo
Aula 05: EN 2021

11. Gabarito

1–B 2– C 3– C 4– B

5– B 6–B 7–B 8– A

Aula 05 – Pronouns and Prepositions 36


www.estrategiamilitar.com.br 62
Teacher Andrea Belo
Aula 05: EN 2021

12. Questões comentadas

QUESTÃO 01 (EEAR/2017)

QUESTÃO 01 – Choose the best alternative to fill in the blanks subsequently.


a) a – some – the – some
b) the – the – the – the
c) the – some – some – a
d) some – a – some – the

QUESTÃO 02 – Choose the alternative in which all the words are uncountable.
a) salt – cake – egg
b) chocolate – butter – walnut
c) flour – baking powder – sugar
d) vanilla – cake tin – ice cream

Aula 05 – Pronouns and Prepositions 37


www.estrategiamilitar.com.br 62
Teacher Andrea Belo
Aula 05: EN 2021

Comentários QUESTÃO 01:

Essa questão discute a diferença de sentido entre definite e indefinite articles.


Primeiramente, a receita cita todos os ingredientes, assim:
Melt “a” chocolate seria incorrect, visto que já se foi especificado qual chocolate, nesse caso,
chocolate U. Além disso, como a quantidade de chocolate também já foi falada (50g),”some”
seria incorrect. Assim, o primeiro espaço deve ser preenchido com “the”.
Já na segunda frase, add “a” sugar está incorrect, pois sugar já foi mencionado antes. Da mesma
forma, sua quantidade foi determinada, por isso, “some” também seria incorrect. Novamente,
devemos usar “the”.
Na terceira frase, add “some” eggs fica incorrect, pois a quantidade de eggs foi determinada
anteriormente, portanto, devemos usar “the”.
Por fim, add “a” flour fica incorrect, pois esse noun já foi mencionado anteriomente. Além
disso, a quantidade foi determinada, logo não se pode usar “some”. Portanto, a opção correta
é “the”. Logo, correct alternative is B.

Comentários QUESTÃO 02:

Alternative A is incorrect, pois ”cake” é um noun contável, assim como “eggs”.

Alternative B is incorrect, pois o noun “chocolate” é contável.

Alternative C is correct, pois nenhum dos nouns apresentados é contável.

Aternative D is incorrect, pois “cake tin” (tabuleiro de bolo) é um noun contável. Essa é uma

expressão a se colocar no seu vocabulário.

Aula 05 – Pronouns and Prepositions 38


www.estrategiamilitar.com.br 62
Teacher Andrea Belo
Aula 05: EN 2021

2018/COLÉGIO NAVAL - Questão 03

QUESTÃO 03

Aula 05 – Pronouns and Prepositions 39


www.estrategiamilitar.com.br 62
Teacher Andrea Belo
Aula 05: EN 2021

COMENTÁRIOS:

Nessa questão precisamos ficar atentos, pois o enunciado pede a alternative que NÃO possui

um adjetivo comparativo.

Alternative A is incorrect, pois contém o comparativo “easier”, relativo ao adjective “easy”.

Alternative B is incorrect, pois possui o comparativo irregular “better”, referente ao adjective

“good”.

Alternative C is correct, pois não há nenhuma palavra comparativa.

Alternative D is incorrect, pois possui o comparativo regular “deeper”, relativo ao adjective

“deep”.

Alternative E is incorrect, pois possui o comparativo regular “affordable”, relativo ao

verb“afford”.

Aula 05 – Pronouns and Prepositions 40


www.estrategiamilitar.com.br 62
Teacher Andrea Belo
Aula 05: EN 2021

2017/EPCAR - Questão 04

Aula 05 – Pronouns and Prepositions 41


www.estrategiamilitar.com.br 62
Teacher Andrea Belo
Aula 05: EN 2021

QUESTÃO 04 - The word “workplace” (line 10) means

a) an institution for people who need professional care.

b) a place where people work.

c) home for old people.

d) a school where people live.

COMENTÁRIOS:

Alternative A is incorrect, pois “workplace” não é o nome próprio de uma instituição


(“institution”).

Alternative B is correct, pois “workplace” é formado pela junção de duas palavras : “work”
(verb “trabalhar”) e “place” (noun “lugar”).Juntas, essas palavras formam a ideia de “lugar de
trabalhar” (place were people work).

Alternative C is incorrect, pois “home” significa “lar”, não lugar de trabalho.

Alternative D is incorrect, pois “school”significa “escola”, não lugar de trabalho.

Aula 05 – Pronouns and Prepositions 42


www.estrategiamilitar.com.br 62
Teacher Andrea Belo
Aula 05: EN 2021

2018/ESCOLA NAVAL - Questão 05

Aula 05 – Pronouns and Prepositions 43


www.estrategiamilitar.com.br 62
Teacher Andrea Belo
Aula 05: EN 2021

2018/ESCOLA NAVAL - Questão 5

COMENTÁRIOS:

Number I se relaciona à expressão “take a time out” (“ tirando um tempo de descanso”), pois

aborda atitudes tomadas quando se percebe a doença vindo (“ the illness coming”).

Number II se relaciona à expressão “eat extra veggies”, pois aborda escolhas alimentares

para aumentar a quantidade de vitaminas, antioxidantes e proteínas.

Number III se relaciona à expressão “stay sharp” (“estar afiado”), pois aborda uma atividade

- jogar Scrabble - que deixa os circuitos neurais fortalecidos.

Number IV se relaciona à expressão “sleep easier”, pois discute o escurecimento do quarto,

o não uso do celular pelo menos 1 hora antes de dormir e uma massagem nos pés.

Number V se relaciona à expressão “fuel up for the day” (“abastecendo para o dia”), pois

aborda a importância do café da manhã (“breakfast”).

Assim, correct alternative is B.

Aula 05 – Pronouns and Prepositions 44


www.estrategiamilitar.com.br 62
Teacher Andrea Belo
Aula 05: EN 2021

2018/AFA - Questão 06

Aula 05 – Pronouns and Prepositions 45


www.estrategiamilitar.com.br 62
Teacher Andrea Belo
Aula 05: EN 2021

2018/AFA - Questão 06 - The text

a) points out how well Burkina Faso dealt with food shortage.

b) shows a remaining difficulty as well as its way out.

c) highlights the importance of urbanization for crop fields.

d) states that land degradation is a natural impact for today’s climatic stability.

COMENTÁRIOS:

Alternative A is incorrect, pois o texto afirma que Burkina Faso (line 82) é uma das

localidades em que o aumento dos preços dos alimentos culminou em instabilidade política

e social, sendo então, incorreto afirmar que ela lidou bem com a escassez de comidas (how

well Burkina dealt with food shortage).

Alternative B is correct, pois o texto mostra uma dificuldade que ainda existe (remaining): a

escassez alimentar. Porém, também oferece soluções no final do texto (its way out).

Alternative C is incorrect, pois o texto justamente mostra como a urbanização contribui para

reduzir a área agrícola disponível.

Alternative D is incorrect, de acordo com o trecho “Land degradation due to increased

human activities has impacted negatively on agricultural production” (lines 45 -47) mostra

que “land degradation” não é um “natural impact”.

Aula 05 – Pronouns and Prepositions 46


www.estrategiamilitar.com.br 62
Teacher Andrea Belo
Aula 05: EN 2021

2017/EFOMM - Questão 07

Aula 05 – Pronouns and Prepositions 47


www.estrategiamilitar.com.br 62
Teacher Andrea Belo
Aula 05: EN 2021

2017/EFOMM - Questão 07 - Which option can NOT be inferred from the text? A pidgin
language is:
( a ) a simplified means of linguistic communication.
( b ) the native language of a speech community.
( c ) employed in situations such as commerce.
( d ) a contact language.
( e ) constructed impromptu, or by convention, between individuals or groups of people.

Comentários:
A alternativa A está correta. O seguinte trecho corrobora esta afirmação: “They have a limited
vocabulary, a reduced grammatical structure, and a much narrower range of functions,
compared to the languages which gave rise to them”
A alternativa B está incorreta. O texto diz que “pidgin language” não é língua nativa de
ninguém. Isso se evidencia no trecho: “They are the native language of no one”.
A alternativa C está correta. O texto diz que o comércio é um dos motivos para o surgimento
de uma “pidgin language”. Isso é visto no trecho: “A pidgin is a system of communication which
has grown up among people who do not share a common language, but who want to talk to
each other, for trading or other reasons.
A alternativa D está correta. O texto afirma que a “pidgin language” é uma língua de contato
no trecho: “the languages used in parts of Africa, South America, or South-east Asia, where
situations of language contact are frequent”.
A alternativa E está correta. O texto diz que essas línguas são desenvolvidas a partir de
convenções feitas entre pessoas que precisam se comunicar. “A pidgin is not a language which
has broken down; nor is it the result of baby talk, laziness, corruption, primitive thought
processes, or mental deficiency. On the contrary: pidgins are demonstrably creative
adaptations of natural languages, with a structure and rules of their own”.

Aula 05 – Pronouns and Prepositions 48


www.estrategiamilitar.com.br 62
Teacher Andrea Belo
Aula 05: EN 2021

2017/EFOMM - Questão 08 - In line 57, “Lingua Franca” is:


( a ) a language that people use to communicate when they have different first languages.
( b ) a distinctive lect, or variety of English spoken by African Americans.
( c ) an informal language consisting of words and expressions that are not considered
appropriate for formal occasions.
( d ) a body of words and phrases that apply to a specific activity or profession, such as a
particular art form or a medical or scientific subject.
( e ) the language that a person has spoken from earliest childhood.

Comentários:
A alternativa A está correta. O texto diz que uma “língua franca” é uma versão aprimorada da
“pidgin”, que desenvolveu um papel mais formal. “Some pidgins have become so useful as a
means of communication between languages that they have developed a more formal role, as
regular auxiliary languages. They may even be given official status by a community, as lingua
francas”. Este trecho corrobora a afirmativa da alternativa.
A alternativa B está incorreta. “língua franca” não é um conceito atrelado à língua inglesa.
A alternativa C está incorreta. O texto diz que a “língua franca” é uma língua que desenvolveu
um papel mais formal. “developed a more formal role”. Exatamente o contrário do que diz a
alternativa.
A alternativa D está incorreta. A “língua franca” não é uma língua voltada para assuntos
científicos ou médicos, mas sim para assuntos que não exigem nível linguístico altamente
rebuscado.
A alternativa E está incorreta. A “língua franca” não é uma língua que se aprende na infância,
mas sim uma língua desenvolvida de acordo com a necessidade de determinado grupo. Não é
uma língua nativa de ninguém.

Aula 05 – Pronouns and Prepositions 49


www.estrategiamilitar.com.br 62
Teacher Andrea Belo
Aula 05: EN 2021

13. Considerações finais


Concluímos mais uma aula, outro passo até a sua aprovação!
E, como eu sempre digo, estamos caminhando para maior vocabulário e mais aprendizado de
fato, com importantes temas e tópicos, aula após aula.
Nota-se o progresso em seus estudos e, provavelmente, uma maior tranquilidade para
enfrentar os exercícios que surgem. E você vai se acostumando a equilibrar seus estudos de forma
sistematizada, estudando cada vez mais e com mais dedicação.

Outro detalhe importante para seu sucesso nos estudos, é continuar fazendo listas de
vocabulário das palavras e verbos, principalmente os irregulares, que aparecem em forma de lista
em inúmeras fontes de pesquisa.
Isso te ajudará nas questões futuras e torna você, como eu disse antes, um candidato mais
bem preparado e confiante para realizar uma excelente prova.
É importante lembrar também do nosso Fórum de dúvidas, exclusivo do Estratégia Militares.
Será minha forma de responder você, esclarecer o que mais você precise saber para que os
conteúdos fiquem ainda mais claros em seus estudos, certo?

E, caso queira, acesse minhas redes sociais para aprender mais palavras e contar com dicas
importantes, que colaboram diretamente com seus estudos dia após dia.

@teacherandreabelo

Teacher Andrea Belo

Teacher Andrea Belo

Aula 05 – Pronouns and Prepositions 50


www.estrategiamilitar.com.br 62
Teacher Andrea Belo
Aula 05: EN 2021

14. Referências bibliográficas


ACKLAM, Richard; CRACE, Araminta. Total English: Pre intermediate. 1 ed. Grã-Bretanha: Longman
do Brasil, 2005.
BAKER, M. In other words: a coursebook on translation. Routledge, 1992.
BLATT, Franz. Précis de Syntaxe Latine. Lyon, Paris: IAC, 1952.
BENTES, Anna Christina e Mussalim, Fernanda (org.). Introdução À Linguística, Domínios E Fronteiras.
6ª edição. Editora Cortez. São Paulo. 2006.
BOURGOGNE, Cleuza Vilas Boas & Silva Lilian Santos. Interação & Transformação. SP: Ed. Brasil,
1999.
BOWKER, L. & PEARSON, J. Working with Specialized Language. Routledge. Capítulos 1, 2, 8,10 e 11,
2002.
BUSSE, Winfried Busse & Mário Vilela. Gramática de Valências. Coimbra: Almedina,1986.
CARVALHO, José Herculano de. Estudos Lingüísticos. v. 2. Coimbra: Atlântida, 1969.
CHIMIM, Renata; Ilearn English student book, 4 / Renata Chimim, Viviane Kirmeliene; [obra coletiva
organizada e desenvolvida pela editora]. 1ª. ed. São Paulo: Pearson Education do Brasil, 2013.
CORBEIL, J.-Cl., ARCHAMBAULT, A. Michaelis Tech dicionário temático visual inglês -português-
francês-espanhol. Tradução: Marisa Soares de Andrade. São Paulo: Melhoramentos, 1997.
CUNHA, Celso. Nova Gramática do Português Contemporâneo. Rio de Janeiro: Nova fronteira,
terceira edição, 2001.
CUNNINGHAM, Gillie; REDSTON, Chris. Face2Face: Upper Intermediate. 1 ed. Brazil: Cambridge,
2001.
DANIELS, H. Vygotsky and pedagogy. Educational Tasks Pedagogical Communication for Teachers.
Routledge, 3 rd edition, 2001.
FAIRCLOUGH, N. Discourse and social change. Polity Press, 1992.
GENTZLER, E. Contemporary Translation Theory. Routledge, 1993.
HOUAISS, A., CARDIM, I. Dicionário universitário Webster inglês-português / português-inglês. São
Paulo: Record, 1998.
HYLAND, K. Genre and second language writing – For teachers and pedagogical professionals in
general, 2003.
HUTCHINSON, Tom & WATERS, Alan. English for Specific Purposes. Cambridge: Cambridge University
Press, 1996.
LAFACE, A. O dicionário e o contexto escolar. Revista Brasileira de Linguística, Unesp/Assis, v.9, 1982,
p. 165-179.
LOBATO, M.P. Lúcia. Teorias Linguísticas e ensino do português como língua materna. Brasília: UNB,
1999.
MICHAELIS Tech Dicionário Temático Visual: línguas estrangeiras – Pesquisa e tradução Marisa
Soares de Andrade. – São Paulo: Companhia Melhoramentos, 1997..
SILVA, João Antenor de C., GARRIDO, Maria Lina, BARRETO, Tânia Pedrosa. Inglês Instrumental:
Leitura e Compreensão de Textos. Salvador: Centro Editorial e Didático, UFBA. 1994.
SILVA, T.; MATSUDA, P. Second language writing research: perspectives on the process of knowledge
construction, 2001.

Aula 05 – Pronouns and Prepositions 51


www.estrategiamilitar.com.br 62
Teacher Andrea Belo
Aula 05: EN 2021

SILVEIRA BUENO, F. A formação histórica da língua portuguesa. 3. ed. São Paulo: Saraiva , 1967.
SIMPSON, J., WEINER, E. (eds.) Oxford English dictionary on CD-ROM. 2ed. Oxford : Oxford
University Press, 1999.
PASCHOALIN, Maria Aparecida; SPADOTO, Neuza Terezinha. Gramática, Teoria e Exercícios. Editora
FDT. São Paulo. 1996.
RIBEIRO, Manuel P. Nova gramática aplicada da língua portuguesa. Rio de Janeiro: Metáfora editora,
14ª edição, 2002.
TUCK, Michael. Oxford Dictionary of Computing for Learners of English. Oxford: Oxford University
Press, 1996.
CETEMFolha/NILC: Corpus de Extractos de Textos Electrónicos. Banco de dados. Disponível em:
http://acdc.linguateca.pt/cetenfolha>.Último acesso (vários acessos) em: 04.05.2019.
COSTA, Daiane. As origens da língua inglesa. Disponível em:
http://englishmaze.wordpress.com/2011/01/25/as-origens-da-lingua-inglesa/Acesso em: 2/5/
2019.
VENTURINI, Laercio. Origem e desenvolvimento da língua inglesa. Disponível em:
<http://www.startenglish.com.br/index.php?option=com_content&task=view&id=10 0&Itemid=97
>. Acesso em: 22 mai. 2012.
OXFORD photo dictionary. Oxford: Oxford University Press, 1992

Referências complementares (websites):

www.richmond.com.br - Acesso em 18 de março de 2019.


http://www.sk.com.br/sk-perf.html - Acesso em 19 de março de 2019.
https://www.inglesnapontadalingua.com.br/2013/03/o-que-sao-falsos-cognatos.html - Acesso em
19 de março de 2019.
https://englishlive.ef.com/pt-br/blog/15-expressoes-idiomaticas-comuns-em-ingles/
https://www.infoescola.com/ingles/

https://www.solinguainglesa.com.br/conteudo/indice.php

https://www.inglesnapontadalingua.com.br

https://www.englishexperts.com.br/

Aula 05 – Pronouns and Prepositions 52


www.estrategiamilitar.com.br 62
Teacher Andrea Belo
Aula 05: EN 2021

5. Traduções

Aqueles que economizam para a aposentadoria enfrentam muitos dilemas. Gastar hoje é mais
divertido do que esperar para gastar amanhã. Uma vez que as economias foram acumuladas,
você deve decidir o que fazer com elas. As possibilidades são muitas e complexas. E as pessoas
são propensas aos erros, compram quando os valores dos ativos estão altos e sofrem pânico
quando ativos entram em ação. A promessa dos conselheiros-robôs, que oferecem consultoria
financeira gerada por computador, é ajudar quem poupa com esses problemas de maneira
muito mais barata que os humanos.

Aula 05 – Pronouns and Prepositions 53


www.estrategiamilitar.com.br 62
Teacher Andrea Belo
Aula 05: EN 2021

DICAS DE VIAGEM - Como planejar férias com tema de filme

Aula 05 – Pronouns and Prepositions 54


www.estrategiamilitar.com.br 62
Teacher Andrea Belo
Aula 05: EN 2021

É mais fácil do que você imagina encontrar, visitar e curtir os lugares onde seus filmes favoritos
foram feitos.

Seja a trilogia "O Senhor dos Anéis" na Nova Zelândia ou o "Feriado romano" em Roma, muitos
filmes notáveis são filmados em locais atraentes em todo o mundo que os viajantes podem
querer visitar e desfrutar.

De acordo com Angela Tillson, gerente de locação de filmes em Kauai, que trabalhou no set de
filmes, incluindo "Jurassic Park: O Mundo Perdido" e "Os Descendentes", explorando um
destino de cenário de filme amado pelos olhos do filme, torna-o agradável período de férias.
"Ver um lugar com foco em um filme que você ama dará a você uma perspectiva que o turista
comum não costuma entender. Você certamente terá uma melhor impressão do local", disse
ela. Aqui estão suas dicas para começar .

Escolha o seu destino


Se você gosta de um filme, pode descobrir onde ele foi filmado olhando os créditos no final do
filme ou acessando o Internet Movie Database, também conhecido como IMDB, que
geralmente lista os locais de filmagem. Depois de conhecer o local, você pode começar a
planejar sua viagem. Ou então, considere fazer o que Tillson costuma fazer quando decide onde
passar férias: escolha um local que você está interessado em visitar e descubra quais filmes
foram filmados lá. "Às vezes, é divertido deixar um destino determinar o filme que você vai
viver, e não o contrário", disse Tillson.

Entre no clima
Antes de ir para o seu destino, não deixe de assistir ao filme novamente. Um replay não
apenas lembra os pontos identificáveis a serem observados durante a sua viagem, mas também
aumenta a emoção de sua próxima exploração.
Se o filme for baseado em um livro, considere a leitura do livro também. Pode haver detalhes
sobre o local em que o filme não toca. Além disso, os livros geralmente têm cenas que não
fazem parte das adaptações do filme, o que oferece uma visão mais profunda do destino.
Tillson também recomendou o download da trilha sonora ou da trilha sonora do filme e a
escutou durante toda a viagem.

Marque uma viagem temática

Aula 05 – Pronouns and Prepositions 55


www.estrategiamilitar.com.br 62
Teacher Andrea Belo
Aula 05: EN 2021

Algumas empresas de viagens vendem roteiros definidos focados em filmes populares. O


operador turístico de luxo Zicasso, por exemplo, faz uma viagem de oito dias, inclusive, à
Irlanda inspirada em "Guerra nas Estrelas: Os Últimos Jedi" e Wild Frontiers faz uma viagem de
onze dias à Índia inspirada no "The Best Exotic Marigold Hotel. "Tillson sugeriu fazer uma
pesquisa na web ou verificar com um agente de viagens para descobrir essas viagens. Além
disso, em alguns destinos, operadores turísticos e hotéis locais vendem passeios com temas de
filmes. Por exemplo, o St. Regis Priceville Resort oferece um tour que inclui um passeio de

helicóptero particular até Manawaiopuna Falls, famoso em "Jurassic Park", e um tour de


quadriciclo pelos locais de filmagem de filmes como "Os Caçadores da Arca Perdida" e "Piratas
do Caribe". O almoço está incluso e o custo é de US $ 5.674 para dois adultos.

Uma opção mais acessível, em Roma, é a excursão temática de “quatro dias romanos” da HR
Tours, onde os viajantes montam uma Vespa com um motorista e veem todos os locais do
filme; o custo é de 170 euros por pessoa. Pendure onde a equipe de filmagem estava Quando
não estão trabalhando, as equipes de filmagem gostam de ir a bares e restaurantes casuais que
servem saborosa culinária local, disse Tillson.

Descubra onde a equipe dos bastidores do seu filme passou o tempo perguntando ao conselho
de turismo do seu destino ou ao concierge do hotel e confira alguns dos pontos. "É outra
maneira de se envolver no filme e passar algum tempo em bares e restaurantes que você
normalmente não pensaria em conhecer", disse ela.

Aula 05 – Pronouns and Prepositions 56


www.estrategiamilitar.com.br 62
Teacher Andrea Belo
Aula 05: EN 2021

Preconceitos Mais Comuns

Aula 05 – Pronouns and Prepositions 57


www.estrategiamilitar.com.br 62
Teacher Andrea Belo
Aula 05: EN 2021

Quais são algumas das maneiras mais comuns pelas quais as pessoas discriminam umas às
outras? Algumas das áreas em que as pessoas demonstram intolerância são bem conhecidas,
como a raça. Mas outros são menos reconhecidos, ainda que mais comuns:
Idade: o envelhecimento é mais comum do que você pensa. As pessoas idosas são
consideradas inflexíveis e duras no passado, enquanto as pessoas mais jovens são vistas como
inexperientes e ingênuas. Um quinto dos adultos trabalhadores dizem que experimentam o
envelhecimento no local de trabalho.
Classe: O classismo geralmente assume a forma de discriminação por parte de pessoas mais
ricas contra pessoas menos favorecidas. No entanto, o classismo segue nos dois sentidos -
pessoas de status econômico mais baixo podem ver os ricos como esnobes da elite que,
embora seguros monetariamente, estão moralmente falidos.
Cor: diferente do racismo, o colorismo é discriminação baseada apenas na cor da pele de uma
pessoa; quão relativamente escuro ou claro eles são. Ocorre dentro e entre as raças. É comum
em sociedades multiétnicas e não brancas e sociedades com preconceito racial histórico.
Habilidade: 8 Geralmente chamada de habilismo, uma forma menos conhecida de preconceito
é a discriminação contra pessoas com deficiências visíveis, como aquelas em cadeiras de rodas
ou com dificuldades de aprendizagem. Os deficientes enfrentam discriminação não apenas de
seus dez colegas, mas também de instituições, escolas, empregadores e 11 proprietários de
terras que hesitam em acomodar os deficientes.
Sexo / Gênero: Possivelmente o preconceito mais universal e de longa duração é aquele
baseado no sexo ou no sexo de uma pessoa. Historicamente, o sexismo colocou os homens em
uma posição mais vantajosa que as mulheres.
Peso / tamanho: em resumo, o tamanho é uma discriminação com base no tamanho ou peso
corporal de uma pessoa. O dimensionismo trabalha com padrões sociais de beleza e
geralmente assume a forma de discriminação contra o excesso de peso - preconceito anti-
gordura.
Religião: A discriminação religiosa e a execução penal têm sido comuns ao longo da história.
Mas o preconceito baseado na afiliação religiosa não termina com a religião organizada; Os
ateus são propensos a discriminação e a serem discriminados.
Orientação Sexual: Geralmente, o preconceito baseado na orientação sexual inclui
discriminação contra aqueles de orientação não-heterossexual. A discriminação contra os não-
heterossexuais assume muitas formas, dependendo da sociedade. Em algumas sociedades, o
preconceito é aberto e tolerado, mas na maioria das sociedades ocidentais, o viés contra o não-
heterossexual é mais discreto.
País de origem: o nativismo é uma forma comum de discriminação contra imigrantes em um
país. Ao contrário de muitas outras formas de discriminação, o nativismo é muitas vezes
incentivado e imposto por algumas entidades públicas.
Que preconceito você tem? Qual preconceito você já experienciou?

Aula 05 – Pronouns and Prepositions 58


www.estrategiamilitar.com.br 62
Teacher Andrea Belo
Aula 05: EN 2021

Pidgins e crioulos

Aula 05 – Pronouns and Prepositions 59


www.estrategiamilitar.com.br 62
Teacher Andrea Belo
Aula 05: EN 2021

Idiomas Pidgin
Um pidgin é um sistema de comunicação que cresceu entre pessoas que não compartilham uma
linguagem comum, mas que querem conversar um com o outro, por negociação ou outros motivos.
Pidgins foram chamados de "improvisado", "marginal" ou "misto" línguas. Eles têm um vocabulário
limitado, uma reduzida estrutura gramatical e uma gama muito mais estreita de funções, em
comparação com as línguas que deram origem para eles. Eles são a língua nativa de ninguém, mas eles
são, no entanto, um meio principal de comunicação para milhões de pessoas, e um grande foco de
interesse para aqueles que estudam a maneira como as línguas mudam.
É essencial evitar o estereótipo de uma linguagem pidgin, como perpetrada ao longo dos anos em
gerações de quadrinhos e filmes infantis. The ‘Me Tarzan, sua imagem de Jane está longe da realidade.
Um pidgin não é um idioma que se decompõe; nem é o resultado de conversa de bebê, preguiça,
corrupção, processos de pensamento primitivo ou deficiência mental. Em pelo contrário: os pidgins
são comprovadamente criativos adaptações de linguagens naturais, com estrutura e regras própr ias.
Juntamente com os crioulos, eles são evidência de um processo fundamental de mudar, à medida que
os idiomas entram em contato com cada outro, produzindo novas variedades cujas estruturas e usa
contrato e expansão. Eles fornecem a mais clara evidência de linguagem sendo criada e modelada por
sociedade para seus próprios fins, à medida que as pessoas se adaptam a novas circunstâncias. Essa
ênfase nos processos de mudança se reflete nos termos pidginização e creolização.
A maioria dos pidgins são baseados em idiomas - inglês, francês, espanhol, holandês e Português -
refletindo a história do colonialismo. Contudo, esta observação pode ser o resultado apenas de nossa
ignorância das línguas usadas em partes do África, América do Sul ou Sudeste Asiático, onde situações
de contato com o idioma são frequentes. Um dos pidgins não europeus mais conhecidos são Chinook
Jargão, uma vez usado para negociação por índios americanos em noroeste dos EUA. Outro é o Sango,
um pidginizado variedade de Ngbandi, falada amplamente no centro-oeste África. Devido à sua função
limitada, o pidgin os idiomas geralmente não duram muito tempo - às vezes por apenas alguns anos e
raramente por mais de um século.
Eles morrem quando a razão original para comunicação diminui ou desaparece, como
comunidades se afastam ou uma comunidade aprende a linguagem do outro. (Como alternativa, o
pidgin pode se transformar em crioulo.) O francês pidgin que foi usado no Vietnã quase desapareceu
quando o Esquerda francesa; Da mesma forma, o inglês pidgin que apareceu durante a campanha
americana do Vietnã praticamente desapareceu assim que a guerra terminou. Mas há exceções. O
pidgin conhecido como A Lingua Franca do Mediterrâneo, ou Sabir, começou no Idade Média e durou
até o século XX.
Alguns pidgins tornaram-se tão úteis quanto meios de comunicação entre os idiomas que eles
desenvolveram um papel mais formal, já que idiomas auxiliares. Eles podem até receber oficiais
status por uma comunidade, como lingua francas. Estes casos são conhecidos como "pidgins
expandidos", devido à maneira como nos quais eles adicionaram formulários extras para lidar com o
necessidades de seus usuários e passaram a ser utilizados muito maior variedade de situações do que
anteriormente. Em tempo, esses idiomas podem ser usados no rádio, na imprensa, e pode até
desenvolver uma literatura por conta própria. Algumas das soluções expandidas mais usadas pidgins
são Krio (na Serra Leoa), Pidgin nigeriano Inglês e Bislama (em Vanuatu). Em Papua Nova Guiné, o
pidgin local (Tok Pisin) é o mais idioma amplamente utilizado no país.

Aula 05 – Pronouns and Prepositions 60


www.estrategiamilitar.com.br 62
Teacher Andrea Belo
Aula 05: EN 2021

Os médicos sabem melhor

Aula 05 – Pronouns and Prepositions 61


www.estrategiamilitar.com.br 62
Teacher Andrea Belo
Aula 05: EN 2021

Por Ted Spiker


Juntamente com toda a doença, o coração revivendo, entrega de bebês e pessoas em geral que
curam, os médicos têm outro emprego em período integral: mantendo-se saudável. Risque isso
- mantendo-se mais saudável. Então em vez de espiar suas práticas médicas, procuramos no
que eles realmente praticam - em suas próprias vidas. Usar estratégias pessoais e dicas dos
melhores médicos profissionais para sobrecarregar sua saúde este ano.
( I)-______
"Assim que sinto uma doença, vou dormir por pelo menos nove horas ", diz Hilda Hutcherson,
MD, clínica professor na Columbia University Medicai Center.
"Eu também deito no chão com as pernas elevadas e apoiadas contra a parede e respire
profundamente por cinco minutos. " ajuda a diminuir o estresse, o que enfraquece o sistema
imunológico.
(II) -______
Em vez de ter uma salada verde de variedade de jardim, Margaret McKenzie, MD, professora
assistente de cirurgia na a Cleveland Clinic, joga repolho napa, radicchio, edamame e cenoura
com molho de gengibre e soja. "Dá muitas vitaminas, antioxidantes e proteínas e faz me sinto
cheio ", diz ela.
III -______
[...] Gary Small, MD, professor de psiquiatria e ciências biocomportamentais da Universidade
da Califórnia, Los Angeles, e autor de The Alzheimer's Prevention Programa, joga Scrabble e
Words With Friends no seu smartphone na maioria dos dias. Estes jogos de palavras são
perfeito estimuladores cerebrais, porque constroem não apenas verbais e habilidades
matemáticas, mas também habilidades espaciais ao posicionar as letras para criar palavras.
"Combinando várias tarefas mentais fortalece vários circuitos neurais ", diz Dr. Small." como
treinamento cruzado para o seu cérebro ".
(IV) - _____
Torne o seu quarto espetacular: diminua as luzes pelo menos hora antes de ir para a cama;
proibir telefones celulares, laptops e a TV; peça ao seu parceiro uma massagem nos pés. "Eu
respiro fundo exercícios ", diz o Dr. Hutcherson." Às vezes eu toco música relaxante
suavemente. "
(V) - _____
A refeição mais importante é o café da manhã, diz David Katz, MD, diretor e fundador da Yale-
Griffin Prevention Centro de Pesquisa em Derby, Connecticut. Ele muitas vezes tem dois cafés
da manhã, dividindo sua refeição da manhã para que ele coma metade antes do treino e
metade depois. "Ajuda com a porção controle e estabelece um padrão alimentar diário ", Dr.
Katz diz. Planeje seu café da manhã à noite para começar o dia seguinte nota saudável.

Aula 05 – Pronouns and Prepositions 62


www.estrategiamilitar.com.br 62
Aula 06 –
Conjunctions
Direct Speech
Reported Speech

Escola Naval 2021 AFA 2021


Teacher Andrea Belo
Teacher Andrea Belo
Aula 06: Escola Naval 2021

Sumário
Introdução ........................................................................................................................ 3

Conjunctions ..................................................................................................................... 4

Compound sentences ...................................................................................................... 11

Direct Speech .................................................................................................................. 13

Indirect Speech/Reported speech ................................................................................... 14

Verb changes ..............................................................................................................................................15

Time expression changes ..........................................................................................................................17

Modal verbs changes.................................................................................................................................18

Questão inédita .............................................................................................................. 21

Questões de anos anteriores ........................................................................................... 23

11. Gabarito................................................................................................................. 33

12. Questões comentadas ............................................................................................ 34

13. Considerações finais ............................................................................................... 51

14. Referências bibliográficas .......................................................................................... 52

15. Traduções.................................................................................................................. 54

Aula 06 –Conjunctions/Clauses/Direct and Reported Speech 2


62
www.estrategiamilitar.com.br
Teacher Andrea Belo
Aula 06: Escola Naval 2021

Introdução

Vamos, então, à nossa aula sobre alguns tópicos considerados complexos: conjunctions and
clauses, direct/indirect ou reported speech (discursos direto e indireto).

As Conjunctions – conjunções – são palavras que ligam duas palavras, ligando, assim, duas
frases, das quais veremos muitas e inúmeros exemplos para que fique claro o uso delas nas
abundantes orações em que aparecem.

Estudaremos, então, as conjunções e suas famosas orações coordenadas e subordinadas, Os


discursos direto e indireto são usados quando queremos expressar as informações que alguém nos
relatou, nos contou. O discurso direto (direct speech) – ao relatar o que alguém disse, usando as
mesmas palavras que a pessoa utilizou, como mostrarei a você na teoria dessa aula. E, por sua vez,
o discurso indireto (indirect speech ou reported speech) – ao relatar o que foi dito, porém, usando
as nossas próprias palavras, que também explicarei com detalhes adiante.

Esses itens que selecionei para nossa aula 06 e que estudaremos agora, são de extrema
importância para solucionar questões em que as provas de Carreiras Militares exige mais de um
tempo verbal na mesma frase e assim, tenta confundir você ou apenas, descobrir se você aprendeu
bem os tempos verbais que já foram estudados para que você tenha domínio de identificá-los, como
você fará!

É claro que seu objetivo é ser aprovado. E, alcançar a aprovação depende de alguns passos,
tais como adotar uma postura positiva, estudar muito e dar o seu melhor. Assim, mais cedo ou mais
tarde vai alcançar sua vaga nas melhores universidades.

No caminho à aprovação, você vai resolver, durante a teoria e também no fim do material,
exercícios de provas anteriores bem como exercícios inéditos e, essas questões irão ajudar você a
colocar em prática o que aprende a cada dia.

Além disso, você estará avaliando seu conhecimento.

Vamos lá! Você consegue e estamos juntos!

Aula 06 –Conjunctions/Clauses/Direct and Reported Speech 3


62
www.estrategiamilitar.com.br
Teacher Andrea Belo
Aula 06: Escola Naval 2021

Conjunctions

As conjunctions – conjunções, são


As conjunções, chamadas também de linking words, connectors ou conectivos são palavras que
ligam duas partes de uma oração, para que a sentença possa fazer sentido. Observe um exemplo,
através dessas duas orações:
✓ I wanted to text you. (Eu queria te mandar uma mensagem.)
✓ I don’t have your number. (Eu não tenho o número do seu celular.)
Elas têm uma relação mas falta algo para unir essas frases e fazê-las ter um sentido maior:
I wanted to text you, but I don’t have your number.
(Eu queria te mandar uma mensagem, mas eu não tenho o número do seu celular.)
Essa palavra but exerceu o papel que precisávamos: uniu as duas sentenças e estabeleceu uma
lógica entre elas. Portanto, o but é uma conjunção.

Tipos de conjunções
Há três tipos de conjunções em inglês:
✓ Conjunções Coordenadas (Coordinating Conjunctions)

✓ Conjunções Subordinadas (Subordinating Conjunctions)

✓ Conjunções Correlativas (Correlative Conjunctions)

Conjunções Coordenadas

As conjunções coordenadas são as mais comuns e as que geralmente usamos ou identificamos


quando pensamos em linking words. Elas têm o papel de juntar orações.
Elas podem juntar orações independentes (ou seja, orações que possuem sentido completo por si
próprias, sem precisar de outra oração para fazer sentido), frases ou apenas palavras.
Na língua inglesa há sete conjunções coordenadas importantes:

For, And, Nor, But, Or, Yet, So

Vejamos as particularidades de cada uma delas.

Aula 06 –Conjunctions/Clauses/Direct and Reported Speech 4


62
www.estrategiamilitar.com.br
Teacher Andrea Belo
Aula 06: Escola Naval 2021

For [por]– Explica o motivo ou a proposta de algo (equivalente ao porquê).

I go to the park every week, for I love running.


[Eu vou ao parque toda semana, por amar correr.]
Peter though he had a great chance to be accepted at Oxford, for his grandfather was
the Dean of that university.
[Paul achava que tinha uma grande chance de ser aceito em Oxford, por seu avô ser o
reitor daquela Universidade.]

And [e] – Adiciona uma coisa à outra.

Daniel goes to the beach to surf and relax.


[O Daniel vai à praia para surfar e relaxar.]
I love red and white wine.
[Eu gosto de vinho branco e tinto.]

Nor [nem] – Utilizado para apresentar uma alternativa com ideia negativa à uma outra ideia
também negativa que já foi afirmada anteriormente.

The virus cannot live in immunized individuals, nor in the air.


[O vírus não pode viver em indivíduos imunizados, nem no ar.]
The guy didn’t have the chest of a body builder, nor did he have the six-pack abs.
[O cara não tinha o peitoral de um fisioculturista, nem tinha o abdômem tanquinho.]

But [mas] – Mostra contraste.

The game in the park is entertaining in the winter, but it’s better in the heat of summer.
[O jogo no parque é divertido no inverno, mas é melhor no calor do verão.]
She is very old but still sensual.
[Ela é muito velha, mas ainda é sensual.]

Aula 06 –Conjunctions/Clauses/Direct and Reported Speech 5


62
www.estrategiamilitar.com.br
Teacher Andrea Belo
Aula 06: Escola Naval 2021

Or [ou] – Apresenta uma alternativa ou uma escolha.

Those men play on teams: shirts or skins.


[Aqueles homens jogam em times: com camiseta ou sem camiseta.]
Do you want a boy or a girl, Mom?
[Você quer um menino ou uma menina, mamãe?]

Yet – Introduz uma ideia constratante que segue logicamente a ideia precedente, similar ao
“mas”.

I often take a book to read, yet I never seem to turn a single page.
[Eu frequentemente levo um livro para ler, mas parece que nunca viro uma só página.]
Dorian was the oldest of the girls, yet her accent was the most prominent.
[A Doriana era a mais velha das irmãs, mas seu sotaque era o mais proeminente.]

So [então, logo] – Indica efeito, resultado ou consequência.

I’ve started dating one of the soccer players, so now I have an excuse to often watch
the game.
[Eu comecei a namorar um dos jogadores de futebol, então agora eu tenho uma
desculpa para assistir aos jogos frequentemente.]
This is the easiest way to get there, so don’t say anything.
[Este é o caminho mais fácil para chegar lá, então não diga nada.]

Observe que as conjunções coordenadas geralmente ficam no meio de uma sentença e uma
vírgula é utilizada antes do linking word.
Com exceção se ambas as orações sejam muito curtas, neste caso a vírgula não será utilizada.
Quando uma conjunção coordenada conecta duas orações independentes (ou seja, que cada
uma possui sentido sozinha, sem precisar da outra oração para fazer sentido), ela é
acompanhada da vírgula.
A vírgula será utilizada quando but expressar contraste.

Aula 06 –Conjunctions/Clauses/Direct and Reported Speech 6


62
www.estrategiamilitar.com.br
Teacher Andrea Belo
Aula 06: Escola Naval 2021

Conjunções Subordinadas
Dos três tipos que há de conjunções, as subordinadas são as mais complexas de se reconhecer, mas
intuitivas de dominar..
As conjunções subordinadas introduzem as orações dependentes (ou seja, orações que não possuem
sentido completo por si próprias, elas precisam de outra oração para fazer sentido) prendendo-as a
uma oração independente (a que possui sentido completo por si só).
As conjunções subordinadas estabelecem uma relação de sentido entre a oração dependente com
o resto da frase. Há inúmeras conjunções subordinadas em inglês, as mais comuns são:

Aula 06 –Conjunctions/Clauses/Direct and Reported Speech 7


62
www.estrategiamilitar.com.br
Teacher Andrea Belo
Aula 06: Escola Naval 2021

Exemplos:

Although I’ve been here before, he’s just too hard to forget.
Embora eu já tenha vindo aqui antes, ele é muito difícil de esquecer.

I guess I’ll never be the same since I won this medal.


Eu acho que eu nunca mais serei o mesmo desde que eu ganhei essa medalha.

They are watching TV while my mom is making dinner.


Eles estão assistindo TV enquanto a minha mãe está fazendo o jantar.

If you leave her, she will die.


Se você deixá-la, ela morrerá.

I’m proud of you because you passed the exam.


Eu estou orgulhoso de você porque você passou no exame.

Talk to me before you leave.


Fale comigo antes de partir.

Once you go there, you never forget it.


Uma vez que você vai lá, você nunca esquece.

When I see you smile, I can do anything.


Quando eu vejo você sorrindo, eu consigo fazer qualquer coisa.

As orações podem ir em qualquer ordem, ou seja, tanto uma oração dependente como
uma independente pode começar a frase.
O que nunca muda é que a conjunção subordinada é a primeira palavra da oração
dependente.

Aula 06 –Conjunctions/Clauses/Direct and Reported Speech 8


62
www.estrategiamilitar.com.br
Teacher Andrea Belo
Aula 06: Escola Naval 2021

Conjunções Correlativas
As conjunções correlativas estão sempre em grupo. Elas vêm em pares e você precisa utilizar ambas
em lugares diferentes em uma oração para fazer sentido.
Por esse motivo, elas têm esse nome justamente pelo fato delas trabalharem juntas (co-) e por
relacionar um elemento de uma sentença com outro (relação).
Sua correlação sempre denota igualdade, e mostra a relação entre as ideias expressas em diferentes
partes da sentença:

as . . . as [como . . . como]
both . . . and [ambos . . . e]
either . . . or [ou . . . ou]
hardly . . . when [dificilmente. . . quando]
if . . . then [se . . . então]
just as . . . so [assim como . . . assim]
neither . . . nor [nem . . . nem]
no sooner . . . than [não antes . . . do que]
not . . . but [não . . . mas]
not only . . . but also [não somente . . . mas também]
rather . . . than [em vez . . . do que]
scarcely . . . when [mal. . . quando]
what with . . . and [o que com. . . e]
whether . . . or [se. . . ou]

Vejamos alguns exemplos:

I didn’t know whether you’d want the pizza or hamburger, so I got you both.
Eu não sabia se você iria querer pizza ou hamburger, então eu peguei os dois para você.

I want either the pizza or the hamburger.


Eu quero ou a pizza ou o hamburger.

Aula 06 –Conjunctions/Clauses/Direct and Reported Speech 9


62
www.estrategiamilitar.com.br
Teacher Andrea Belo
Aula 06: Escola Naval 2021

I’ll eat them both – not only the pizza but also the hamburger.
Eu comerei os dois – não só o pizza mas também o hamburger.

I’ll have both the pizza and the hamburger.


Eu vou querer ambos a pizza e o hamburger.

Há ainda uma outra conjunção, chamada de conjunção adverbial.


Ela estabelece uma ideia de conjunção que une duas orações, entretanto, por ter valor de advérbio,
não é comum que ela apareça junto às demais conjunções.
As conjunções adverbiais mais comuns (apesar de pouco usadas) e que podem aparecer em alguma
prova são:

Aula 06 –Conjunctions/Clauses/Direct and Reported Speech 10


62
www.estrategiamilitar.com.br
Teacher Andrea Belo
Aula 06: Escola Naval 2021

Compound sentences

Para entender o que são e porque existem as compound sentences – orações coordenadas e
subordinadas – vou explicar, antes, um detalhe da gramática para você.
Uma sentença simples (simple sentence), consiste em apenas uma estrutura, geralmente um só
verbo, explorando uma só ideia: I like salad (Eu gosto de salada), I am happy. (Eu estou feliz), entre
outras.
Uma compound sentence consiste em duas ou mais estruturas frasais independentes, assim sendo,
demonstra mais ideias, unidas por alguma conjunção, que muitas gramáticas chamam de palavras
de ligação, já que é o papel que elas exercem, veja exemplos:

Sentença 1: I like salad. (Eu gosto de salada).


Sentença 2: I like pasta. (Eu gosto de massas).
Compound sentence: I like salad and pasta. (Eu gosto de salada e de massas)

Sentença 1: She went to the mall yesterday. (Ela foi ao shopping ontem).
Sentença 2: She bought nothing at the mall. (Ela não comprou nada no shopping).
Compound sentence: She went to the mall yesterday, but she bought nothing.
(Ela foi ao shopping ontem mas ela não comprou nada).

Nota-se que as primeiras frases foram unidas pela conjunção and e a segunda frase, pela conjunção
but, além de outras, que já estudamos anteriormente e que são responsáveis em formar as
compound sentences.
Vamos recordá-las – as mais comuns são: FOR, AND, NOR, BUT, OR, YET, SO. Algumas gramáticas
dizem que há um esquema para se lembrar delas: Pensar na palavra FANBOYS, iniciais de cada
conjunção que se usa para elaborar compound sentences. Veja algumas:

Aula 06 –Conjunctions/Clauses/Direct and Reported Speech 11


62
www.estrategiamilitar.com.br
Teacher Andrea Belo
Aula 06: Escola Naval 2021

As frases dos exemplos anteriores com aproximadas traduções são:

He did not cheat on the game, for it was the wrong thing to do.
(Ele não trapaceou no jogo pois era a coisa errada de se fazer).

They got there early, and they got good seats.


(Eles chegaram lá cedo e conseguiram bons assentos).

They had no food at home, nor did they have money to go to the store.
(Eles não tinham comida em casa nem dinheiro para ir à loja).

I really need to go to there, but I am too tired to drive.


(Eu realmente preciso ir lá mas estou muito cansado para dirigir).

Should we start class now, or wait for the others to get here?
(Devemos começar a aula agora ou esperar pelos outros chegarem aqui?)

I am on a diet, yet I really want dessert.


(Eu estou de dieta, ainda que eu queira sobremesa).

Everyone was busy, so I went to the theater alone.


(Todos estão ocupados então eu fui ao teatro sozinho).

Quando essas frases aparecem na prova, estão dentro dos textos e, pode ser questionado se
as frases indicam contraste (but), adição (and) ou qualquer outra ideia de que vimos no início da
teoria de explicação das conjunções.

Conhecendo as conjunções e sabendo que estão inseridas da forma que estudamos, você vai
conseguir identificar o que for solicitado no dia da prova.

Agora, estudaremos Direct e Indirect Speech. Let’s go!

Aula 06 –Conjunctions/Clauses/Direct and Reported Speech 12


62
www.estrategiamilitar.com.br
Teacher Andrea Belo
Aula 06: Escola Naval 2021

Direct Speech

O discurso direto em Inglês – direct speech – é usado quando queremos reproduzir qualquer tipo de
informação que nos é relatada. Podemos dizer que, em suma, o discurso direto é utilizado para
repetir o que uma outra pessoa disse do jeito exatamente que foi dito, sem alteração. Veja:

She said, “I study every day because I want to be approved”.


Ela disse: “Eu estudo todos os dias porque quero ser aprovada”.

E sabe por que é importante estudar o discurso direto em relação à sua prova? Simplesmente pelo
fato que é possível encontrar (e aparece muitas vezes) o discurso direto em jornais e portais de
notícias, a fim de enfatizar e, consequentemente, deixar a notícia mais direta ou, às vezes, mais
dramática, vejamos:

“I won’t resign”, says the president during the meeting.


“Não irei renunciar”, diz o presidente durante a reunião.
“Our diplomatic relations are over”, affirm both candidates.
“Nossas relações diplomáticas acabaram”, afirmam ambos candidatos).

O discurso direto pode também ser encontrado em diálogos de narrativas ficcionais, pois permite
que traços da fala e de personalidade dos personagens envolvidos tenham destaque, atraindo a
atenção de seus leitores. Nos textos de provas podem aparecer por esse motivo: enfatizar alguma
parte do texto.

“I could tell you my adventures—beginning from this morning,” said Alice a


little timidly; “but it’s no use going back to yesterday, because I was a
different person then.” (Eu poderia lhes contar minhas aventuras –
começando por esta manhã", disse Alice um pouco tímida; mas não adianta
voltar a ontem, porque eu era uma pessoa diferente.)
Lewis Carrol, Alice no País das Maravilhas

Agora, vamos estudar o discurso indireto – indirect speech, também chamado de reported
speech, que, além de ter muitas possibilidades de uso, é mais recorrente ainda em muitas provas.

Aula 06 –Conjunctions/Clauses/Direct and Reported Speech 13


62
www.estrategiamilitar.com.br
Teacher Andrea Belo
Aula 06: Escola Naval 2021

Indirect Speech/Reported speech

O indirect speech/reported speech, ou discurso indireto, é, por sua vez, uma maneira de falar sobre
o que alguém disse, para repassar uma notícia, uma história.
A principal característica é que no discurso indireto, se fala na voz de quem está contando a ação e
não de quem a viveu.
Por esse motivo, existem algumas regras básicas para se usar bem o discurso indireto, como por
exemplo, a mudança dos tempos verbais.
Se você diz, por exemplo, que você quer um carro novo e alguém vai me contar, a s equência é a
seguinte:

- Eu quero um carro novo.


- Ele disse que queria um carro novo.

Em Inglês, é a mesma coisa. Observe que o verbo querer estava no presente quando você falou (eu
quero).
E foi automaticamente para o passado quando alguém contou o que você falou (ele/ela disse que
queria).
Veja em Inglês o exemplo do carro e outro, ilustrado:

(Você dizendo): – I want a new car .


(Alguém dizendo/contando o que ouviu): – He/She said that he/she wanted a new car.

(O pai dizendo): – I want to see your grades (Eu quero ver suas notas).
(A mãe dizendo à filha o que foi dito): – He said that he wanted to see your grades.
(Ele disse que ele queria ver as suas notas).

Aula 06 –Conjunctions/Clauses/Direct and Reported Speech 14


62
www.estrategiamilitar.com.br
Teacher Andrea Belo
Aula 06: Escola Naval 2021

Veja algumas mudanças que acontecem com os verbos quando o discurso direto é
transformado em discurso indireto, com exemplos abaixo de cada tempo verbal:

Verb changes

Aula 06 –Conjunctions/Clauses/Direct and Reported Speech 15


62
www.estrategiamilitar.com.br
Teacher Andrea Belo
Aula 06: Escola Naval 2021

Você percebeu que em todas as frases eu usei “He said”, que pode ser acompanhado ou não
de “that”. Mas, além do verbo to say (passado said), podemos também usar o verbo to tell (passado
told) em frases com indirect/reported speech.

Vejamos exemplos:

- I want a glass of water now. - Eu quero um copo de água agora.

He told me (that) he wanted this glass of water. Ele me disse que ele queria esse copo de água.

- I don’t want to work out today. - Eu não quero malhar.

He told me (that) he didn’t want to work out yesterday. Ele me disse que não queria malhar ontem.

A partir dos exemplos acima, podemos notar que algumas outras palavras, além dos tempos
verbais, se alteram com o discurso indireto. Se fosse, por exemplo, em Português:

- Eu comprei esse boné. (discurso direto)


Ele disse que comprou aquele boné. (discurso indireto)
Em Inglês, fica:
- I have bought this cap. He said he had bought that cap.
- Eu comprei esse boné. Ele disse que ele tinha comprado aquele boné.

Veja as possíveis modificações no discurso indireto, que podem perguntar na sua prova.

Aula 06 –Conjunctions/Clauses/Direct and Reported Speech 16


62
www.estrategiamilitar.com.br
Teacher Andrea Belo
Aula 06: Escola Naval 2021

Time expression changes

Pode ainda, haver outras alterações. Alguns lugares, além dos pronomes e das indicações de
tempo acima, também podem mudar com a passagem do discurso direto para o indireto.

They are meeting at my house. (Eles estão se encontrando na minha casa.)


He said that they were meeting at his house.
Ele disse que eles estavam se encontrando na casa dele.)

Aula 06 –Conjunctions/Clauses/Direct and Reported Speech 17


62
www.estrategiamilitar.com.br
Teacher Andrea Belo
Aula 06: Escola Naval 2021

I got here by train. (Eu cheguei aqui de trem.)

He said he had got there by train. (Ele disse que ele chegou lá de trem.)

E os verbos modais, já apresentados na aula 2, também mudam. Se você diz que pode fazer
algo, no momento que vou contar a alguém, digo: Ele(a) disse que podia fazer algo.
Assim como o posso se torna podia, em Inglês seria o can se tornar could, entendeu?
Vejamos a seguir a tabela e os exemplos para facilitar.

Modal verbs changes

I can dance rock. (Eu consigo dançar rock.)


He said he could dance rock. (Ele disse que ele conseguia dançar rock.)

Aula 06 –Conjunctions/Clauses/Direct and Reported Speech 18


62
www.estrategiamilitar.com.br
Teacher Andrea Belo
Aula 06: Escola Naval 2021

She must study a lot. (Ela deve estudar muito.)


She said she had to study a lot. (Ela disse que ela teve que estudar muito.)

Em um dos textos da prova do jornal The Economist que, inclusive, vamos explorar exercícios
em nossa aula adiante (às vezes aparecem nas provas), vou mostrar como uma das frases em
discurso direto poderia ter sido explorada para testar seus conhecimentos:

Na frase “Environmentalists may feel a twinge of fear…” (Ambientalistas podem sentir uma pontada
de medo...), no discurso direto, poderia ser questionado como ficaria no discurso indireto ou se o
verbo modal “may” mudaria na modificação de discurso direto para indireto, veja:

Questão: A frase “Environmentalists may feel a twinge of fear…” devidamente transformada em


discurso indireto, ficaria:

A ( ) He said that environmentalists may felt a twinge of fear.


B ( ) He said that environmentalists can felt a twinge of fear.
C ( ) He said that environmentalists may be felling a twinge of fear.
D ( ) He said that environmentalists may have felt a twinge of fear.
E ( ) He said that environmentalists might feel a twinge of fear.

Aula 06 –Conjunctions/Clauses/Direct and Reported Speech 19


62
www.estrategiamilitar.com.br
Teacher Andrea Belo
Aula 06: Escola Naval 2021

Comentários:

Aqui, devemos analisar cada alternativa, para encontrar a melhor alternativa para a sentença
transformada em discurso indireto, de acordo com as regras vistas acima, na explicação com
quadros/tabelas que preparei para você.

A primeira coisa a se observar é que o verbo modal “may”, ao direcionar-se à uma frase do discurso
indireto, passa de may para might, lembra?

Na letra A, afirma-se que o verbo feel vai para o passado – felt, mas, ao analisar a teoria acima,
podemos perceber que, como eu disse anteriormente, may se transforma em might. Falsa.

Na letra B, afirma-se que o verbo modal may muda para can e que o verbo feel vai para o passado –
felt, mas, ao analisar a teoria acima, podemos perceber que, como eu disse anteriormente, may se
transforma em might e jamais seria trocado um verbo modal por outro, pois, como vimos na aula de
verbos, o sentido também muda de acordo com a situação em que é encaixado. Falsa.

Na letra C, afirma-se que a estrutura inteira da frase muda de may feel para may be feeling, no
gerúndio , mas, ao analisar a teoria acima, podemos perceber que, como eu disse anteriormente,
may se transforma em might. Falsa.

Na letra D, afirma-se que estrutura inteira da frase muda de may feel para may have felt, no present
perfect, mas, ao analisar a teoria acima, podemos perceber que, como eu disse anteriormente, may
se transforma em might. Falsa.

A letra E, afirma-se que o verbo feel continua feel e o modal may muda para might, exatamente
como vimos na tabela ilustrativa. E podemos perceber que, como eu disse anteriormente, may
transformado em might está correto para discurso indireto com verbos modais.

Agora, teremos uma questão inédita que elaborei, aos moldes das melhores provas para você,
abordando os assuntos que foram explorados nessa aula.

Logo depois, os exercícios de diferentes bancas de anos anteriores para você treinar, como
sempre fazemos em nossas aulas. Vamos lá!

Aula 06 –Conjunctions/Clauses/Direct and Reported Speech 20


62
www.estrategiamilitar.com.br
Teacher Andrea Belo
Aula 06: Escola Naval 2021

Questão inédita

Questão inédita Teacher Andrea Belo

Question: Read the text and answer the question below.

Aula 06 –Conjunctions/Clauses/Direct and Reported Speech 21


62
www.estrategiamilitar.com.br
Teacher Andrea Belo
Aula 06: Escola Naval 2021

Questão 01 – De acordo com o primeiro parágrafo,


a) algumas pessoas se recolhem para comer o jantar do delivery.
b) muitas pessoas se recolhem para comer o jantar do delivery.
c) algumas caixas de delivery chegam nas casas das pessoas com atraso.
d) algumas pessoas se recusam a comer o jantar vindo de uma caixa encharcada.
e) as caixas encharcadas de delivery nunca chegam nas casas das pessoas.
Comentários:
Esse texto foi retirado de um trecho de um artigo do jornal The Economist, fonte comum no
preparo das provas. Vamos, é claro, usar as técnicas de Scanning e Skimming desde o início do
texto e analisando também a imagem, que há um entregador, já que desde o título, há a palavra
delivery no texto.
Você tem que encontrar a alternativa que apresenta uma ideia completa sobre algo que é
apresentado no primeiro parágrafo, solicitado no enunciado da questão. Vejamos cada
alternativa.
Ao verificar a alternativa “A”, afirmando “algumas pessoas se recolhem para...”, mas, sem
continuar lendo toda a frase, o verbo recoil, apesar de parecer recolher, é o verbo recusar e
por isso, essa alternativa está falsa.
N letra “B”, novamente, usa-se o verbo recolher para demonstrar algo e confundir você na hora
de escolher a melhor alternativa. Mas, vimos que recoil é recusar e não recolher.
Na letra “C”, afirma-se que caixas de delivery chegam nas casas das pessoas com atraso mas
não há essa informação em nenhum lugar do texto, sobre atraso de delivery. Falsa.
Na letra “D”, afirma-se: o que procuramos: o verbo recusar. E a informação é justamente o que
diz no texto, na primeira linha: “...those who recoil at eating supper out of a soggy box...”
(aqueles que se recusam a comer o jantar vindo de uma caixa encharcada). Alternativa correta.
Na letra “E”, afirma-se que as caixas nunca chegam nas casas das pessoas e, além dessa
alternativa estar incorreta porque não há essa informação no texto, a palavra nunca é perigosa
porque generaliza muito e não há o termo “never” no texto. Falsa.
Agora, vamos aos exercícios de anos anteriores, como sempre fazemos em nossas aulas,
para praticar ainda mais e, após resolver, verificar os comentários de cada alternativa e
aprender, cada vez mais, vocabulário e formas de resolver as questões no dia da sua prova.

Aula 06 –Conjunctions/Clauses/Direct and Reported Speech 22


62
www.estrategiamilitar.com.br
Teacher Andrea Belo
Aula 06: Escola Naval 2021

Questões de anos anteriores


Você agora vai resolver questões de provas de Carreiras Militares de anos anteriores,
incluindo a Escola Naval. E, em seguida, terá acesso às respostas comentadas. Vamos lá!

QUESTÃO 01 - EFOMM

Aula 06 –Conjunctions/Clauses/Direct and Reported Speech 23


62
www.estrategiamilitar.com.br
Teacher Andrea Belo
Aula 06: Escola Naval 2021

QUESTÃO 01 (EFOMM/2017) - In lines 2 – 3: “(…) and despite an unresolved lawsuit that has
delayed the project (...)”, the word in bold is formed by the prefix 'un'. In which option below
the word is not correctly formed by the same prefix?
( a ) unfinished
( b ) unfasten
( c ) unwrapped
( d ) undecided
( e ) unpolite

QUESTÃO 02 (EFOMM/2017). In lines 9 - 11: “The US authorities then in control began


excavation for larger locks in 1939”, the word in bold means:
( a ) An artificial waterway used for travel, shipping or irrigation.
( b ) A river or canal that vessels travel on.
( c ) A barrier that protects a harbor or shore from the full impact of waves.
( d ) A place on a river with a set of gates which open and close to allow boats in.
( e ) A barrier constructed across a waterway to control the flow or raise the level of water.

Texto para questão 03

Aula 06 –Conjunctions/Clauses/Direct and Reported Speech 24


62
www.estrategiamilitar.com.br
Teacher Andrea Belo
Aula 06: Escola Naval 2021

Questão 03 (EPCAR/2017) - The topic “ability” mentions people who


a) have poor sight.
b) have difficulty in learning.
c) get excited about helping the disabled.
d) face social and economical discrimination.

QUESTÃO 05 (Escola Naval/2017)

Aula 06 –Conjunctions/Clauses/Direct and Reported Speech 25


62
www.estrategiamilitar.com.br
Teacher Andrea Belo
Aula 06: Escola Naval 2021

04 (Escola Naval/2017)

05 (Escola Naval/2017)

(EsPCEx/2020)

Questão 06.

Aula 06 –Conjunctions/Clauses/Direct and Reported Speech 26


62
www.estrategiamilitar.com.br
Teacher Andrea Belo
Aula 06: Escola Naval 2021

Questão 07 (EsPCEx/2020 inédita) – “… yet few of us eat enough of it. Here’s how to up your
intake.” The underlined word can be substituted for
a) even
b) but
c) besides
d) otherwise
e) then

Questão 08. ITA/2018 - Texto para responder a questão 8

Aula 06 –Conjunctions/Clauses/Direct and Reported Speech 27


62
www.estrategiamilitar.com.br
Teacher Andrea Belo
Aula 06: Escola Naval 2021

Questão 08. Marque a opção correta de acordo com o sentido com que os verbos modais
sublinhados são empregados no texto.
I. I work as an editor at a publishing company. (line 1) → para indicar presente.
II. It’s given me a chance to think about what it really... (linha 11) → para indicar passado.
III. We never know what tomorrow might bring… (linha 13) → para indicar futuro.
IV. So it goes (linhas 16 e 17) → para indicar presente.
Estão corretas A ( ) I e II B ( ) I e IV. C. ( ) I, II e IV. D ( ) II, III e IV. E ( ) II e IV.

TEXTO PARA QUESTÕES 09 e 10 (AFA/2018)

Aula 06 –Conjunctions/Clauses/Direct and Reported Speech 28


62
www.estrategiamilitar.com.br
Teacher Andrea Belo
Aula 06: Escola Naval 2021

Questão 09 (AFA/2018) - Mark the INCORRECT option according to the text.

a) There are no clear explanations on reasons for food shortage.

b) If one applies the required solutions one solves food shortage problem.

c) Green house issues have contributed for experiencing food restriction.

d) Droughts and desertification also harm food production.

Questão 10 (AFA/2018) - The first paragraph states that crop failure, overpopulation and

poor government policies are the main cause of food scarcity in most countries Such
problems may represent respectively

a) urban, economic and social factors.

b) economic, social and environmental factors.

c) economic, environmental and political factors.

d) environmental, social and economic factors.

TEXTO PARA QUESTÕES 11 e 12 (EAM/2020 inéditas)

Aula 06 –Conjunctions/Clauses/Direct and Reported Speech 29


62
www.estrategiamilitar.com.br
Teacher Andrea Belo
Aula 06: Escola Naval 2021

Questão 11 – According to the text,

a) há pessoas que se recolhem para comer o jantar vindo de uma caixa encharcada.

b) há pessoas que se recolhem para comer o jantar vindo de um delivery com um ciclista.

c) algumas pessoas abominam a cultura do “food delivery system”.

d) algumas pessoas se recusam a comer o jantar vindo de uma caixa encharcada.

e) as caixas chegam encharcadas devido ao atraso do delivery.

Questão 12 – In the first paragraph, “by founding Takeaway.com in his university bedroom”,

the underlined word refers to

a) an old Dutchman.

b) tech billionaire.

c) cultural abomination.

d) online food delivery business.

e) online food.

Questão 13

Aula 06 –Conjunctions/Clauses/Direct and Reported Speech 30


62
www.estrategiamilitar.com.br
Teacher Andrea Belo
Aula 06: Escola Naval 2021

Questão 13 (CN/2018)

Aula 06 –Conjunctions/Clauses/Direct and Reported Speech 31


62
www.estrategiamilitar.com.br
Teacher Andrea Belo
Aula 06: Escola Naval 2021

Questão 14 (Escola Naval/2018)

QUESTÃO 14

Aula 06 –Conjunctions/Clauses/Direct and Reported Speech 32


62
www.estrategiamilitar.com.br
Teacher Andrea Belo
Aula 06: Escola Naval 2021

11. Gabarito

1–E 2– D 3– C 4–E 5– C
6–B 7 –B 8–C 9–B 10 – D
11 – C 12 – D 13 – E 14 – C

Aula 06 –Conjunctions/Clauses/Direct and Reported Speech 33


62
www.estrategiamilitar.com.br
Teacher Andrea Belo
Aula 06: Escola Naval 2021

12. Questões comentadas

QUESTÃO EFOMM/2016

Aula 06 –Conjunctions/Clauses/Direct and Reported Speech 34


62
www.estrategiamilitar.com.br
Teacher Andrea Belo
Aula 06: Escola Naval 2021

QUESTÃO 01 - In lines 2 – 3: “(…) and despite an unresolved lawsuit that has delayed the project
(...)”, the word in bold is formed by the prefix 'un'. In which option below the word is not
correctly formed by the same prefix?
( a ) unfinished
( b ) unfasten
( c ) unwrapped
( d ) undecided
( e ) unpolite
Comentários: O prefixo “un” indica negação.

Alternative A is correct, pois o adjective “finished” ganha ideia de negação pela adição de “un”
ao formar o adjective “unfinished”.

Alternative B is correct, pois a palavra “fasten” significa apertar, como por exemplo na fala da
aeromoça no avião, pedindo que apertemos os cintos de segurança: “lease, fasten your
seatbelts.”. Para dizer para soltarmos, a palavra seria “unfasten”, configurando uso de “un”
para negação.

Alternative C is correct, pois “wrapped” significa o adjective “embrulhado”, como embrulhado


para presente. A adição de “un” de fato traz a negativa “não embrulhado”.

Alternative D is correct, pois o adjective “decided” ganha ideia de negação com a adição do
prefixo “un”.

Alternative E is incorrect, pois o negative do adjective “polite” se dá pela adição do prefixo


“im”: “impolite”.

QUESTÃO 02. In lines 9 - 11: “The US authorities then in control began excavation for larger
locks in 1939”, the word in bold means:
( a ) An artificial waterway used for travel, shipping or irrigation.
( b ) A river or canal that vessels travel on.
( c ) A barrier that protects a harbor or shore from the full impact of waves.
( d ) A place on a river with a set of gates which open and close to allow boats in.
( e ) A barrier constructed across a waterway to control the flow or raise the level of water.

Aula 06 –Conjunctions/Clauses/Direct and Reported Speech 35


62
www.estrategiamilitar.com.br
Teacher Andrea Belo
Aula 06: Escola Naval 2021

Comentários:
Alternative A is incorrect, pois “lock” não é synonym de canal de água, “rio”, “waterway”,
mesmo que artificial.
Alternative B is incorrect, pois “locks” não é synonym de “river” (“rio”).
Alternative C is incorrect, pois “lock” não significa uma espécie de barreira (“barrier”) que
protege do impacto das ondas (“ondas”), o que implicaria que estaria localizada em uma região
litorânea.
Alternative D is correct, pois “locks” significa o conjunto de portões (“gates”) que abrem e
fecham em um canal, permitindo a passagem dos navios e barcos (“boats”).
Alternative E is incorrect, pois “lock” não é synonym de barreira (“barrier”). Essa alternative
descreve uma ideia de represa, que seria “dam”.

Texto para questão 03

Aula 06 –Conjunctions/Clauses/Direct and Reported Speech 36


62
www.estrategiamilitar.com.br
Teacher Andrea Belo
Aula 06: Escola Naval 2021

Questão 03 (EPCAR/2017) - In the sentence “Usually called ableism, a less well-known


form of prejudice” (lines 23 and 24), the underlined expression means
a) always.
b) rarely.
c) frequently.
d) seldom.
Comentários:
“Usually” é um advérbio de frequência que indica geralmente, quase sempre, situação usual.
Alternative A is incorrect, pois “always” é um advérbio que indica 100%, isto é, “sempre”.
Alternative B is incorrect, pois “rarely” indica frequência baixa, quase nula, “raramente”.
Alternative C is correct, pois “frequently” indica muitas vezes, frequentemente, usualmente.
Alternative D is incorrect, pois “seldom” indica atividade feita poucas vezes, como o “rarely”.

QUESTÃO 05 (Escola Naval/2017)

Aula 06 –Conjunctions/Clauses/Direct and Reported Speech 37


62
www.estrategiamilitar.com.br
Teacher Andrea Belo
Aula 06: Escola Naval 2021

04 (Escola Naval/2017)

05 (Escola Naval/2017)

Comentários questão 04: “Nearly” significa” quase” e “overseeing” significa supervisionar.

Alternative A is incorrect, pois “seldom” é um frequency adverb que indica baixa frequência e
“helping” é a forma no gerúndio de “help”, que significa “ajudar”.

Alternative B is incorrect, pois “nearby” traz a ideia de proximidade física, proximidade a um


lugar, não quantidade. E “joining” é a forma no gerúndio do verbo “join”, que pode ser
entendido como juntar-se.

Alternative C is incorrect, pois “certainly” indica certeza, ao invés de aproximação, de tentativa.


“Meeting” é a forma no gerúndio do verbo “meet”, que significa encontrar, conhecer.

Alternative D is incorrect, pois “quite” é um advérbio que indica intensidade. “Dis regarding” é
a forma no gerúndio do verbo “disregard”, que pode ser enendido como
“desprezar”,”negligenciar”.

Aula 06 –Conjunctions/Clauses/Direct and Reported Speech 38


62
www.estrategiamilitar.com.br
Teacher Andrea Belo
Aula 06: Escola Naval 2021

Alternative E is correct, pois “almost” significa “quase” e “supervising” é a forma no gerúndio


do verbo “supervise”, “supervisionar”.

Comentários questão 05:

Alternative A is incorrect, pois Cliff sofreu um acidente, não se machucou em uma competição
esportiva.

Alternative B is incorrect, pois Cliff classificou para as Olimpíadas, não para as Paraolimpíadas,
como afirma o final do sétimo parágrafo.

Alternative C is correct, Cliff carregou a bandeira durante a abertura dos jogos de 2000.

Alternative D is incorrect, pois Cliff não ganhou nenhuma medalha, como afirma-se no
parágrafo 12º.

Alternative E is incorrect, pois esse “book” foi escrito pelo narrador do texto, médico Bert R.
Mandelbaum, como se afirma no parágrafo 13º.

(EsPCEx/2020)

Questão 06.

Aula 06 –Conjunctions/Clauses/Direct and Reported Speech 39


62
www.estrategiamilitar.com.br
Teacher Andrea Belo
Aula 06: Escola Naval 2021

Comentários:
Essa questão analisa a frase “A fibre-rich diet is linked to health benefits including a reduced
risk of heart disease and bowel cancer.”
Vamos fazê-la por eleminação, começando pela alternative D. Ela está descartada, pois afirma
que “fiber diet is linked to ALL health benefits”, o que não é coerente com a informação do
texto. No texto se afirma que existem benefícios advindos da fibe-rich diet, inclusive citam-se
exemplos, mas não há informação que nos permita concluir que esses são TODOS os helth
benefits.
Agora vamos descartar a alternative E. Novamente, temos um problema de generalização. Na
frase, cita-se “bowel cancer” como exemplo, mas não há informação que nos permita concluir
que há redução de risco para todos os cânceres. Por isso, a alternative E está incorreta.
As alternatives A e C estão descartadas pelo mesmo motivo: introduzem quantificação ao noun
benefits, porém a frase original não nos permite concluir se são “lots of” ou “few” benefits, já
que não essa quantificação.
Assim, concluímos que a alternativa correta é B, que traz corretamente um dos exemplos da
frase, sobre o benefício da redução do risco de doença cardíaca.

Questão 07 (EsPCEx/2020 inédita) – “… yet few of us eat enough of it. Here’s how to up your
intake.” The underlined word can be substituted for
a) even
b) but
c) besides
d) otherwise
e) then
Comentários: É necessário voltar ao texto e analisar mais do que apenas o trecho mostrado
para compreendermos o significado de “yet” nesse caso. O trecho em questão é: “Roughage
helps reduce the risk of heart disease and bowel cancer, yet few of us eat enough of it .”
A parte anterior à vírgula traz uma afirmação de que fiber, ou roughage, faz bem à saúde.
Depois do “yet” há uma segunda afirmação de que poucos de nós comem a quantidade
suficiente de fiber. Essas duas afirmações estabelecem entre si uma relação de oposição, pois
deveríamos comer preferencialmente os alimentos que nos fazem bem. Assim sendo, essa
relação e oposição é justamente a função do “yet”. Para encontrarmos a alternativa correta,
precisamos de outra conjunção adversativa, que, no caso, é “but”, alternative B.
As alternatives A e C estão incorretas, pois “even” e “besides” trazem ideia de inclusão.
A alternative D está incorreta, pois corresponderia à expressão “caso contrário”, o que não se
encaixaria no sentido a frase.

Aula 06 –Conjunctions/Clauses/Direct and Reported Speech 40


62
www.estrategiamilitar.com.br
Teacher Andrea Belo
Aula 06: Escola Naval 2021

A alternative E está incorreta, pois “then” traz ideia de conclusão.

Questão 08. ITA/2018


Texto para responder as questões 08

Aula 06 –Conjunctions/Clauses/Direct and Reported Speech 41


62
www.estrategiamilitar.com.br
Teacher Andrea Belo
Aula 06: Escola Naval 2021

Questão 08. Marque a opção correta de acordo com o sentido com que os verbos modais
sublinhados são empregados no texto (adaptada)

I. I work as an editor at a publishing company. (line 1) → para indicar presente.

II. It’s given me a chance to think about what it really... (linha 11) → para indicar p assado.

III. We never know what tomorrow might bring… (linha 13) → para indicar futuro.

IV. Someone said to me:… (linha 35) → para indicar discurso direto.

Estão corretas
A ( ) I e II B ( ) I e IV. C. ( ) I, II e IV. D ( ) II, III e IV. E ( ) II e IV.

Comentários:
Vamos analisar os excertos.
I. Aprendemos, na aula sobre verbos, que o verbo em seu formato original, geralmente está no
tempo presente. Se o sujeito for singular, acrescenta-se o “s” mas, no caso do nº 1, o sujeito é
I (eu) e não se coloca “s” no verbo. Sendo assim, o item nº 1 está correto, indicando presente:
I work as na editor at a publishing company (Eu trabalho como editor em uma empresa de
publicações”. Afirmação correta.
II. Desde o início da frase, percebemos que é passado sim, já que há um verbo no particípio
(given) e, todos os tempos verbais que contém verbos no particípio indicam ações no passado
- ou já aconteceram ou iniciaram no passado e ainda acontecem mas tiveram início no passado.
Está correta também.
III. Nessa opção, há a palavra tomorrow, que leva você a talvez pensar que seja futuro, já que
tomorrow significa amanhã mas, o verbo principal (know), está no presente, conjugado sem
alteração e apenas para expressar “nunca sabemos o dia de amanhã/o que o amanhã pode
trazer a nós”: we never know what tomorrow might bring. Opção errada.
IV. “Someone said to me:...” tem o verbo say no passado (said) acompanhado da preposição
to, que é estrutura de discurso direto, conforme estudamos em nossa aula. Opção certa.
Portanto a resposta da questão é a letra “C”. Estão corretas os itens I, II e IV.

Aula 06 –Conjunctions/Clauses/Direct and Reported Speech 42


62
www.estrategiamilitar.com.br
Teacher Andrea Belo
Aula 06: Escola Naval 2021

QUESTÕES 09 e 10 (AFA/2018)

Aula 06 –Conjunctions/Clauses/Direct and Reported Speech 43


62
www.estrategiamilitar.com.br
Teacher Andrea Belo
Aula 06: Escola Naval 2021

Questão 09 (AFA/2018) - Mark the INCORRECT option according to the text.


a) There are no clear explanations on reasons for food shortage.
b) If one applies the required solutions one solves food shortage problem.
c) Green house issues have contributed for experiencing food restriction.
d) Droughts and desertification also harm food production.
Comentários:
Alternative A is incorrect, pois o texto afirma existiram diversas soluções práticas e aplicáveis
para o problema da escassez de comida nas lines 85-97.
Alternative B is correct, pois no parágrafo sobre “solutions” fica claro que o esforço coletivo e
individual é a chave para alcançar a resolução dessa problemática (“through concerted and
individual efforts”).
Alternative C is correct, pois (line 40) “mining of coal and oil emits green house gases” está
inserida como “environmental factor which have greatly contributed to food shortage”(lines
34-35, começo do parágrafo).
Alternative D is correct, pois as secas (“droughts”) e a desertificação dificultam as plantações,
principalmente nos países em desenvolvimento e, por isso, prejudicam (“harm”) a produção
de alimentos.

Questão 10 (AFA/2018) - The first paragraph states that crop failure, overpopulation and poor
government policies are the main cause of food scarcity in most countries Such problems may
represent respectively
a) urban, economic and social factors.
b) economic, social and environmental factors.
c) economic, environmental and political factors.
d) environmental, social and economic factors.
Comentários:
Alternative A is incorrect, pois “crop failure” (falha da plantação) não representa um problema
urbano, visto que se trata justamente de uma dificuldade ambiental.
Alternatives B and C are incorrect, pois como falado anteriormente, “crop failure” representa
um “environmental problem”.
Alternative D is correct, pois “crop failure” representa um “environmental problem”, enquanto
“overpopulation” (cidades superpopulosas) se encaixa na parte social e, por fim, “government
policies” (políticas governamentais) entram em “economic factor”.

Aula 06 –Conjunctions/Clauses/Direct and Reported Speech 44


62
www.estrategiamilitar.com.br
Teacher Andrea Belo
Aula 06: Escola Naval 2021

TEXTO PARA QUESTÕES 11 e 12 (EAM/2020 inéditas)

Questão 11 – According to the text,


a) há pessoas que se recolhem para comer o jantar vindo de uma caixa encharcada.
b) há pessoas que se recolhem para comer o jantar vindo de um delivery com um ciclista.
c) algumas pessoas abominam a cultura do “food delivery system”.
d) algumas pessoas se recusam a comer o jantar vindo de uma caixa encharcada.
e) as caixas chegam encharcadas devido ao atraso do delivery.

Comentários:
O início do texto aborda aqueles indivíduos que não aprovam a cultura do delivery de comida.
O autor faz uso de 3 possibilidades, talvez até um pouco cômicas ou caricatas, já que não se
trata de um texto puramente científico, de traços de perfis do indivíduo que não aprova essa
modernidade.
Trata-se de uma construção que se repete:
1º) Even those who recoil at eating supper out of a soggy box should admire Jitse Groen.
2º) Even those who fear being mowed down by curry-bearing cyclists should admire Jitse
Groen.

Aula 06 –Conjunctions/Clauses/Direct and Reported Speech 45


62
www.estrategiamilitar.com.br
Teacher Andrea Belo
Aula 06: Escola Naval 2021

3º) Even those who think the death of home cooking is a cultural abomination should admire
Jitse Groen.
Assim, compreende-se que se trata de 3 exemplos a fim de construir uma ideia mais global e
mais ampla do grupo de indivíduos que não lida bem, não aceita, o sistema de delivery de
comida. Por isso, a alternativa correta é C.
A alternativa A está incorreta, devido à interpretação do verbo “recoil”. Nesse caso, fica
estranho se o interpretarmos como “recolher”, isto é, interpretar que alguém se recolhe (talvez
em grupo, dentro de casa) frente à realidade de comer jantar vindo de caixa de entrega. Aqui,
a melhor interpretação seria pessoas que se “esquivam” frente à ideia de comer do delivery. A
alternativa D também trata do verbo “recoil”, por sua vez, usando a interpretação de
“recusar”. Essa não é uma interpretação possível para esse verbo.
Da mesma forma que a alternativa A, a alternativa B também interpreta equivocadamente
“recoil” como “recolher”. Além disso, também mistura informações do texto de forma errada,
pois diz que o jantar (informação constante na frase 1) está sendo trazido pelo ciclista
(informação constante na frase 2 e não necessariamente relacionada a jantar).
A alternativa E também traz uma informação que não aparece no texto. Vamos analisá-la com
calma. Parcialmente, os trechos dela aparecem no texto, como “soggy box” (caixa
“encharcada”) e “curry – bearing cyclists” (ciclistas carregadores de curry, isto é, os
entregadores de delivery). Contudo, foram unidos de outra forma diferente daquela presente
no texto, de modo a compor uma alternativa falsa, que cria uma relação de causa-
consequência entre a entrega do delivery e a condição da caixa de estar encharcada.

Questão 12 – In the first paragraph, “by founding Takeaway.com in his university bedroom”,
the underlined word refers to
a) an old Dutchman.
b) tech billionaire.
c) cultural abomination.
d) online food delivery business.
e) online food.
Comentários:
O termo sublinhado refere-se a um site, há “.com”. Então,
“The 41-year-old Dutchman, who cooked up the online food-delivery business by founding
Takeaway.com in his university bedroom”. Pela frase, podemos perceber que “Takeaway.com”
é o website referente ao negócio de entrega de comida fundado por um homem de atualmente
41 anos(pelo texto fica claro que seu nome é Jitse Groen).
Assim, a alternativa correta é D.
A alternativa A está incorreta, pois em “an old Dutchman” temos o indefinite article “na”, o
adjective “old” e o noun, núcleo da ideia da expressão, “Dutchman”, que representa aqueles
nascidos na Dutchland. Portanto, essa alternativa se refere a um homem.

Aula 06 –Conjunctions/Clauses/Direct and Reported Speech 46


62
www.estrategiamilitar.com.br
Teacher Andrea Belo
Aula 06: Escola Naval 2021

A alternativa B está incorreta, pois também se refere a um indivíduo, representado pelo noun
“billionaire”.

A alternativa C está incorreta, temos o adjective “cultural” e o noun “abomination”. Portanto,


a ideia principal dessa alternativa está em “abomination”, o que não é o caso.
A alternativa E está incorreta, pois o núcleo da ideia está no noun “food” (caracterizado pelo
adjective “online”). Portanto, essa alternativa se refere a “food”.

Questão 13

Aula 06 –Conjunctions/Clauses/Direct and Reported Speech 47


62
www.estrategiamilitar.com.br
Teacher Andrea Belo
Aula 06: Escola Naval 2021

Questão 13 (CN/2018)

Comentários:
A resposta para essa questão está no parágrafo que fala sobre o filme “Star Wars: the last Jedi”.
Esse parágrafo é encontrado sob o subtítulo “ Book a themed trip” e a frase específica de que
precisamos é: “ Luxury tour operator Zicasso, for example, has a eight-day trip, all inclusive, to
Ireland inspired by “Star Wars: The Last Jedi” and…”
Alternative A is incorrect, pois nada sobre “meals” (“refeições”) é mencionado,
Alternative B is incorrect, pois afirma sobre uma viagem de 11 dias, o que seria a locução
adjetiva “eleven-day trip” e vemos que no trecho temos a locução “eight-day trip”.
Alternative C is incorrect, pois não se afirmou nada sobre o preço dessa “trip” e o adjective
“cheap” refere-se justamente a um valor baixo, “barato”.
Alternative D is incorrect, pois o trecho afirma que o destino dessa viagem é a Irlanda
(“Ireland”).
Alternative E is correct, pois o trecho afirma que o “luxury tour operator Zicasso” fornece essa
viagem.

Aula 06 –Conjunctions/Clauses/Direct and Reported Speech 48


62
www.estrategiamilitar.com.br
Teacher Andrea Belo
Aula 06: Escola Naval 2021

Questão 14 (Escola Naval/2018)

Aula 06 –Conjunctions/Clauses/Direct and Reported Speech 49


62
www.estrategiamilitar.com.br
Teacher Andrea Belo
Aula 06: Escola Naval 2021

QUESTÃO 14 (EM/2018)

Comentários:

“Lower” é referente ao adjective “low”, que significa “pequeno”. Assim, “lower” significa

“diminuir”, nesse caso, “decrease” ou “reduce”. Já “weakens” tem a ver com o adjective

“weak”,que significa “fraco”.

Assim, “weakens” significa, nesse caso, “enfraquecer”, “debilitates”.

Portanto, correct alternative is C.

Aula 06 –Conjunctions/Clauses/Direct and Reported Speech 50


62
www.estrategiamilitar.com.br
Teacher Andrea Belo
Aula 06: Escola Naval 2021

13. Considerações finais


Outra aula alcançada com sucesso - outro passo até a sua aprovação!
E, dia após dia, os tópicos aprendidos aumentam, seu conhecimento fica mais amplo, o vocabulário
que você conhece se estende e a tendência é melhorar e ser capaz de alcançar a aprovação de fato.
Nota-se o avanço em seus estudos e, provavelmente, uma maior tranquilidade para enfrentar os
exercícios que surgem. E você vai se acostumando a equilibrar seus estudos de forma sistematizada,
estudando cada vez mais e com mais dedicação.

Outro detalhe importante para seu sucesso nos estudos, é continuar fazendo aquelas listas
de vocabulário que aconselhei você, com palavras, verbos variados e termos que você considere
importante de ser anotado, de ser revisto, estudado.
Isso te ajudará nas questões futuras e torna você, como eu disse antes, um candidato mais
bem preparado e confiante para realizar uma excelente prova.
É importante lembrar também do nosso Fórum de dúvidas, exclusivo do Estratégia Militares.
Será minha forma de responder você, esclarecer o que mais você precise saber para que os
conteúdos fiquem ainda mais claros em seus estudos, certo?

E, caso queira, acesse minhas redes sociais para aprender mais palavras e contar com dicas
importantes, que colaboram diretamente com seus estudos dia após dia.

@teacherandreabelo

Teacher Andrea Belo

Teacher Andrea Belo

Aula 06 –Conjunctions/Clauses/Direct and Reported Speech 51


62
www.estrategiamilitar.com.br
Teacher Andrea Belo
Aula 06: Escola Naval 2021

14. Referências bibliográficas


ACKLAM, Richard; CRACE, Araminta. Total English: Pre intermediate. 1 ed. Grã-Bretanha: Longman
do Brasil, 2005.
BAKER, M. In other words: a coursebook on translation. Routledge, 1992.
BLATT, Franz. Précis de Syntaxe Latine. Lyon, Paris: IAC, 1952.
BENTES, Anna Christina e Mussalim, Fernanda (org.). Introdução À Linguística, Domínios E Fronteiras.
6ª edição. Editora Cortez. São Paulo. 2006.
BOURGOGNE, Cleuza Vilas Boas & Silva Lilian Santos. Interação & Transformação. SP: Ed. Brasil,
1999.
BOWKER, L. & PEARSON, J. Working with Specialized Language. Routledge. Capítulos 1, 2, 8,10 e 11,
2002.
BUSSE, Winfried Busse & Mário Vilela. Gramática de Valências. Coimbra: Almedina,1986.
CARVALHO, José Herculano de. Estudos Lingüísticos. v. 2. Coimbra: Atlântida, 1969.
CHIMIM, Renata; Ilearn English student book, 4 / Renata Chimim, Viviane Kirmeliene; [obra coletiva
organizada e desenvolvida pela editora]. 1ª. ed. São Paulo: Pearson Education do Brasil, 2013.
CORBEIL, J.-Cl., ARCHAMBAULT, A. Michaelis Tech dicionário temático visual inglês-português-
francês-espanhol. Tradução: Marisa Soares de Andrade. São Paulo: Melhoramentos, 1997.
CUNHA, Celso. Nova Gramática do Português Contemporâneo. Rio de Janeiro: Nova fronteira,
terceira edição, 2001.
CUNNINGHAM, Gillie; REDSTON, Chris. Face2Face: Upper Intermediate. 1 ed. Brazil: Cambridge,
2001.
DANIELS, H. Vygotsky and pedagogy. Educational Tasks Pedagogical Communication for Teachers.
Routledge, 3 rd edition, 2001.
FAIRCLOUGH, N. Discourse and social change. Polity Press, 1992.
GENTZLER, E. Contemporary Translation Theory. Routledge, 1993.
HOUAISS, A., CARDIM, I. Dicionário universitário Webster inglês-português / português-inglês. São
Paulo: Record, 1998.
HYLAND, K. Genre and second language writing – For teachers and pedagogical professionals in
general, 2003.
HUTCHINSON, Tom & WATERS, Alan. English for Specific Purposes. Cambridge: Cambridge University
Press, 1996.
LAFACE, A. O dicionário e o contexto escolar. Revista Brasileira de Linguística, Unesp/Assis, v.9, 1982,
p. 165-179.
LOBATO, M.P. Lúcia. Teorias Linguísticas e ensino do português como língua materna. Brasília: UNB,
1999.
MICHAELIS Tech Dicionário Temático Visual: línguas estrangeiras – Pesquisa e tradução Marisa
Soares de Andrade. – São Paulo: Companhia Melhoramentos, 1997..
SILVA, João Antenor de C., GARRIDO, Maria Lina, BARRETO, Tânia Pedrosa. Inglês Instrumental:
Leitura e Compreensão de Textos. Salvador: Centro Editorial e Didático, UFBA. 1994.
SILVA, T.; MATSUDA, P. Second language writing research: perspectives on the process of knowledge
construction, 2001.

Aula 06 –Conjunctions/Clauses/Direct and Reported Speech 52


62
www.estrategiamilitar.com.br
Teacher Andrea Belo
Aula 06: Escola Naval 2021

SILVEIRA BUENO, F. A formação histórica da língua portuguesa. 3. ed. São Paulo: Saraiva , 1967.
SIMPSON, J., WEINER, E. (eds.) Oxford English dictionary on CD-ROM. 2ed. Oxford : Oxford
University Press, 1999.
PASCHOALIN, Maria Aparecida; SPADOTO, Neuza Terezinha. Gramática, Teoria e Exercícios. Editora
FDT. São Paulo. 1996.
RIBEIRO, Manuel P. Nova gramática aplicada da língua portuguesa. Rio de Janeiro: Metáfora editora,
14ª edição, 2002.
TUCK, Michael. Oxford Dictionary of Computing for Learners of English. Oxford: Oxford University
Press, 1996.
CETEMFolha/NILC: Corpus de Extractos de Textos Electrónicos. Banco de dados. Disponível em:
http://acdc.linguateca.pt/cetenfolha>.Último acesso (vários acessos) em: 04.05.2019.
COSTA, Daiane. As origens da língua inglesa. Disponível em:
http://englishmaze.wordpress.com/2011/01/25/as-origens-da-lingua-inglesa/Acesso em: 2/5/
2019.
VENTURINI, Laercio. Origem e desenvolvimento da língua inglesa. Disponível em:
<http://www.startenglish.com.br/index.php?option=com_content&task=view&id=100&Itemid=97
>. Acesso em: 22 mai. 2012.
OXFORD photo dictionary. Oxford: Oxford University Press, 1992

Referências complementares (websites):

www.richmond.com.br - Acesso em 18 de março de 2019.


http://www.sk.com.br/sk-perf.html - Acesso em 19 de março de 2019.
https://www.inglesnapontadalingua.com.br/2013/03/o-que-sao-falsos-cognatos.html - Acesso em
19 de março de 2019.
https://englishlive.ef.com/pt-br/blog/15-expressoes-idiomaticas-comuns-em-ingles/
https://www.infoescola.com/ingles/

https://www.solinguainglesa.com.br/conteudo/indice.php

https://www.inglesnapontadalingua.com.br

https://www.englishexperts.com.br/

Aula 06 –Conjunctions/Clauses/Direct and Reported Speech 53


62
www.estrategiamilitar.com.br
Teacher Andrea Belo
Aula 06: Escola Naval 2021

5.
15. Traduções

A China começou a entrar na era do consumo de carros em massa. É um grande


o
e histórico avanço. ”Então proclamou a agência de notícias estatal Xinhua no ano
passado. Os ambientalistas podem sentir uma pontada de medo nesse romance
crescente com o automobilismo. Mas uma rápida transformação social e
econômica está em andamento na China urbana, e o carro a está dirigindo.
Em 2002, a demanda por carros na China aumentou 56%, muito mais do que as
projeções mais otimistas. No ano seguinte, o crescimento acelerou para 75%,
antes de desacelerar em 2004 (quando o governo restringiu as regras de crédito
para compras de carros) para cerca de 15%. Mas em um mercado global lento, a
demanda da China continua hipnotizante. Poucos esperam que o crescimento
deste ano caia abaixo de 10%. Enquanto a economia continuar galopando em seu
atual clipe de alto índice, muitos esperam que as vendas de carros aumentem de
10 a 20% ao ano nos próximos anos.

Aula 06 –Conjunctions/Clauses/Direct and Reported Speech 54


62
www.estrategiamilitar.com.br
Teacher Andrea Belo
Aula 06: Escola Naval 2021

Os australianos não são conhecidos por seu amor pelas pessoas que viajam de
barco. Eles notoriamente afastaram um pequeno grupo de refugiados afegãos no
auge da guerra e, de maneira bastante divertida, realizaram uma campanha
assustadora com crocodilos e tubarões para impedir futuros imigrantes. Mas se
o aquecimento global continuar no ritmo atual, as ilhas vizinhas do Pacífico
poderão ser perdidas pelas inundações e a Austrália enfrentará um novo tipo de
intruso: os refugiados climáticos. Embora a Cruz Vermelha tenha produzido um
relatório há quatro anos estimando que 58% dos refugiados são causados por
fatores ambientais, ninguém fez nenhuma tentativa de resolver o problema.
Norman Myers, da Universidade de Oxford, alegou recentemente que poderia
haver um número estimado de 150 milhões de refugiados ambientais nos
próximos 50 anos, e metade deles poderia pousar na porta da Austrália. Mas a
ONU se recusa a conceder a eles o status de refugiado, e grupos de ajuda e
ambientalistas discutem sobre de quem são eles.

Aula 06 –Conjunctions/Clauses/Direct and Reported Speech 55


62
www.estrategiamilitar.com.br
Teacher Andrea Belo
Aula 06: Escola Naval 2021

As mulheres que trabalham no Japão são mais prováveis para se casar do que a
não se casar nos dias de hoje, o que é uma forte reversão do padrão tradicional.
Mas pela maioria delas, continuando a trabalhar após o casamento é uma escolha
mais fácil do que ter crianças. Apesar de algumas tentativas do governo e das
empresas para fazer o mundo do trabalho e paternidade compatíveis, mães
dizem que a cultura de negócios do Japão permanece hostil para eles.
Reuniões de negócios geralmente começam às 18h ou mais tarde. São
esperadas horas extras não-remuneradas e as empresas transferem
rotineiramente funcionários para cidades diferentes por longos anos.
Como resultado, muitas mulheres estão escolhendo trabalhar ao invés de ter
bebês, fazendo com que a taxa de natalidade japonesa caia para um nível recorde
em 1999 – um média de 1,34 bebês por mulher - um acréscimo de aflição desta
nação em envelhecimento.

Aula 06 –Conjunctions/Clauses/Direct and Reported Speech 56


62
www.estrategiamilitar.com.br
Teacher Andrea Belo
Aula 06: Escola Naval 2021

Aula 06 –Conjunctions/Clauses/Direct and Reported Speech 57


62
www.estrategiamilitar.com.br
Teacher Andrea Belo
Aula 06: Escola Naval 2021

ADEUS COISAS, OLÁ MINIMALISMO: PODE VIVER COM MENOS TORNÁ-LO MAIS FELIZ?
Fumio Sasaki possui um colchão dobrável, três camisas e quatro pares de meias. Depois de decidir a desprezo
posses, ele começou a se sentir mais feliz. Ele explica o porquê.
Deixe-me falar um pouco sobre mim. Tenho 35 anos, homem, solteiro, nunca fui casado. Eu trabalho como
editor de uma editora. Recentemente me mudei do bairro de Nakameguro, em Tóquio, onde vivi por uma
década, para um bairro chamado Fudomae em uma parte diferente da cidade. O aluguel é mais barato, mas
o movimento praticamente limpou minhas economias. Alguns de vocês podem pensar que eu sou um
perdedor: um adulto solteiro com pouco dinheiro. O velho eu teria sido muito envergonhado para admitir
tudo isso. Eu estava cheio de orgulho inútil. Mas eu honestamente não me importo com coisas assim mais.
A razão é muito simples: Estou perfeitamente feliz assim como eu sou. O motivo? Eu me livrei da maioria dos
meus bens materiais. Minimalismo é um estilo de vida em que você reduzir suas posses ao menos possível.
Viver com apenas os fundamentos nua não só forneceu benefícios superficiais, como o prazer de uma sala
arrumada ou a simples facilidade de limpeza, ele também levou a uma mudança mais fundamental. Me deu
uma chance de pensar no que realmente significa ser feliz. Pensamos que quanto mais tivermos, mais felizes
seremos. Nunca sabemos o que o amanhã pode trazer, por isso recolhemos e salvamos o máximo que
pudermos. Isso significa que precisamos de muito dinheiro, então gradualmente começamos a julgar as
pessoas por quanto dinheiro eles têm. Você se convence de que precisa ganhar muito dinheiro para não
perder o sucesso. E para você ganhar dinheiro, você precisa de todos os outros para gastar o seu dinheiro. E
assim vai. Então eu disse adeus a um monte de coisas, muitas das quais eu tinha por anos. E ainda agora eu
vivo cada dia com um espírito mais feliz. Eu sinto mais conteúdo agora do que eu já fiz no passado. Nem
sempre fui minimalista. Eu costumava comprar um monte de coisas, acreditando que todos esses bens
aumentariam a minha autoestima e levar a uma vida mais feliz. Eu adorava colecionar um monte de coisas
inúteis, e eu não podia jogar nada fora. Eu era um colecionador natural de quinquilharias que eu pensei que
me fez uma pessoa interessante. Ao mesmo tempo, porém, eu estava sempre comparando-me com outras
pessoas que tinham mais ou melhores coisas, o que muitas vezes me fez infeliz. Eu não conseguia me
concentrar em nada, e eu estava sempre perdendo tempo. O álcool era a minha fuga, e eu não tratava as
mulheres de forma justa. Eu não tentei mudar; Eu pensei que tudo isso era apenas parte de quem eu era, e
eu merecia ser infeliz. O meu apartamento não era horrivelmente confuso; Se minha namorada estava vindo
para o fim de semana, eu poderia fazer o suficiente arrumando para torná-lo parecer apresentável. Em um
dia usual, entretanto, havia uns livros empilhados em toda parte porque não havia bastante quarto em
minhas estantes. A maioria eu tinha manuseado através de uma ou duas vezes, pensando que eu iria lê-los
quando eu tinha tempo. O armário estava lotado com o que costumava ser a minha roupa favorita, a maioria
dos quais eu só usava algumas vezes. O quarto estava cheio de todas as coisas que eu tinha tomado como
passatempos e, em seguida, cansado de. Uma guitarra e um amplificador, cobertos com poeira. Pastas de
trabalho inglês conversacional eu planejava estudar uma vez que eu tinha mais tempo livre. Mesmo uma
câmera antiga fabulosa, que é claro que eu nunca tinha colocado um rolo de filme em. Pode soar como se
eu estou exagerando quando eu digo que eu comecei a se tornar uma nova pessoa. Alguém me disse: "tudo
que você fez é jogar fora as coisas", o que é verdade. Mas por ter menos coisas ao redor, eu comecei a me
sentir mais feliz a cada dia. Estou lentamente começando a entender o que é felicidade. Se você é qualquer
coisa como eu costumava ser-miserável, constantemente comparando-se com os outros, ou apenas
acreditando que sua vida é uma porcaria-eu acho que você deve tentar dizer adeus a algumas de suas coisas.
[...] Todos querem ser felizes. Mas tentar comprar a felicidade só nos faz feliz por um tempo.

Aula 06 –Conjunctions/Clauses/Direct and Reported Speech 58


62
www.estrategiamilitar.com.br
Teacher Andrea Belo
Aula 06: Escola Naval 2021

Aula 06 –Conjunctions/Clauses/Direct and Reported Speech 59


62
www.estrategiamilitar.com.br
Teacher Andrea Belo
Aula 06: Escola Naval 2021

NÓS GRAVAMOS CONVERSAÇÕES DE VC E ANALISAMOS COMO DIFERENTEMENTE ELES FALAM SOBRE


EMPREENDEDORES FEMININOS
1
Quando os capitalistas de risco (CRs) avaliam as propostas de investimento, a linguagem que eles usam para falar dos
2
empreendedores que os escrevem desempenha um papel importante, mas frequentemente oculto, na definição de
quem recebe financiamento 3 e por quê. Tivemos acesso a reuniões governamentais de tomada de decisões sobre
capital de risco na Suécia e pudemos observar os tipos de linguagem que os CRs usaram durante um período de dois
anos. Uma coisa importante se destacou: 5 A linguagem usada para descrever empreendedores do sexo masculino e
feminino era radicalmente diferente. E essas diferenças 6têm consequências muito reais para quem busca
financiamento - e para a sociedade em geral.
7
[…] Em todo o mundo, o capital de risco do governo é importante para colmatar lacunas financeiras
significativas e 8apoiar a inovação e o crescimento, uma vez que os CRs podem assumir riscos onde os bancos não
estão autorizados. Quando a incerteza é 9alta em relação à avaliação do potencial do produto e do mercado, por
exemplo, a avaliação do potencial do 10 empreendedor torna-se altamente central na tomada de decisões do governo
11
Na Suécia, cerca de um terço das empresas pertencem e são gerenciadas por mulheres, embora não lhes seja
concedida 12 uma proporção correspondente de financiamento governamental. Na verdade, as empresas pertencentes
a mulheres recebem muito menos - 13 apenas 13% enquanto 18%, o restante, vai para empresas que pertencem a
homens.
14
Isso nos traz de volta à nossa pesquisa. De 2009 a 2010, fomos convidados a observar silenciosamente 15
reuniões governamentais de tomada de decisões dos CRs e, mais importante, as conversas que tiveram sobre 16
empreendedores que solicitaram financiamento. […] Observamos discussões presenciais fechadas com 17 decisões
finais de financiamento para 125 aplicações de risco. Destes, 99 (79%) eram de empresários do sexo masculino e 26 18
(21%) eram de mulheres empresárias. O grupo de capitalistas de risco do governo observado incluiu 19 sete indivíduos:
duas mulheres e cinco homens. [… 20 Com algumas exceções, os financiadores produzem imagens estereotipadas de
mulheres como 21quem possui qualidades opostas àquelas consideradas importantes para serem empreende doras,
com CRs, questionando sua 22 credibilidade, confiabilidade, experiência e conhecimento.
23
Por outro lado, ao avaliar empreendedores do sexo masculino, os financiadores apoiaram -se em crenças
estereotipadas sobre homens 24que reforçaram seu potencial empreendedor. Os empreendedores do sexo masculino
eram comumente descritos como 25 assertivos, inovadores, competentes, experientes, conhecedores e com redes
estabelecidas. 26 Desenvolvemos pessoas empreendedoras masculinas e femininas com base em nossas descobertas
[…]. Essas personalidades 27 destacam algumas diferenças fundamentais sobre como os empreendedores foram
notados, dependendo de seu gênero. Homens 28 foram caracterizados, com potencial empreendedor, enquanto o
potencial empreendedor para mulheres foi 29 diminuído. Muitos rapazes e moças eram descritos como jovens, embora
jovens para homens fossem 30 vistos como promissores, enquanto mulheres jovens eram consideradas inexperientes.
Os homens eram elogiados por serem 31vistos como agressivos ou arrogantes, enquanto a experiência e a excitação
das mulheres eram temperadas por discussões 32 de suas deficiências emocionais. Da mesma forma, a cautela foi vista
de forma muito diferente, dependendo do sexo 33 do empreendedor.
34
Sem surpresa, esses estereótipos parecem ter desempenhado um papel em quem conseguiu financiamento
e quem não o fez. 35 Mulheres empreendedoras receberam apenas, em média, 25% do valor aplicado, enquanto os
homens 36 receberam, em média, 52% do que pediram. As mulheres também tiveram seu financiamento negado em
maior proporção 37do que os homens, sendo que quase 53% das mulheres tiveram suas solicitações recusadas, em
comparação com 38% dos homens. […] 38Tal estereotipagem inevitavelmente influenciará a distribuição do
financiamento, mas também poderá ter outras 39consequências importantes. Como o objetivo do capital de risco do
governo é usar o dinheiro dos impostos para estimular o crescimento e 40a criação de valor para a sociedade como um
todo, o viés de gênero apresenta o risco de que o dinheiro não esteja sendo investido em 41empresas com o maior
potencial. Isso não é apenas prejudicial para as mulheres empreendedoras; é potencialmente prejudicial para a
sociedade como um todo.
Fonte: Adaptado de Harvard Business Review <https://hbr.org/2017/05/we-recorded-vcs-conversa-tions-and-analyzedhow- differently-they-
talk-about-female-entrepreneurs>. Acesso em: 17 mai. 2017.

Aula 06 –Conjunctions/Clauses/Direct and Reported Speech 60


62
www.estrategiamilitar.com.br
Teacher Andrea Belo
Aula 06: Escola Naval 2021

Aula 06 –Conjunctions/Clauses/Direct and Reported Speech 61


62
www.estrategiamilitar.com.br
Teacher Andrea Belo
Aula 06: Escola Naval 2021

FRANK WHITTLE E A INVENÇÃO DO MOTOR DE JATO:


Seis lugares para rastrear seu gênio
Foi, de várias maneiras, um tipo de conquista muito britânica. Quando a turbina
começou a girar no "WU" - o protótipo de motor a jato desenvolvido pelo engenheiro
nascido em Coventry, Frank Whittle - foi um momento que mudou o mundo. Se você
estivesse passando pelos caminhos do Rugby, em Warwickshire, há mais de 80 anos,
você pode até ter ouvido. Um tamborilar da mecânica sincronização, construção e
construção, crescendo em intensidade para se tornar um rugido; um uivo vertiginoso
que alterar permanentemente a maneira como percorremos nosso planeta.
E, no entanto, poderia facilmente não ter acontecido. O triunfo de Whittle - em 12 de
abril de 1937 – foi conquistada em face da indiferença oficial e da dúvida científica, e só
foi retirada por um a menor amplitude financeira, com a Segunda Guerra Mundial se
aproximando de todos os lados. (...)
Aqui estava um visionário que começou a fomentar seu projeto para um motor a jato já
em 1927, e patenteou-o em 1930, mas teve que nadar contra a corrente depois de ver
sua ideia ridicularizada por o Ministério do Ar do Reino Unido - que, ao ver o projeto em
1929, o considerou "impraticável". Sem se deter, Whittle seguiu seu próprio caminho.
Em janeiro de 1936, ele fundou uma empresa privada, Power Jets Ltda, com o
engenheiro aeronáutico Rolf Dudley Williams e o oficial aposentado da RAF James
Collingwood Tinling. Com £ 2.000 de financiamento da O.T. Falk & Partners - um banco
de investimento que era conhecido por correr riscos - o trio começou a converter o que
havia sido criticado como fantasia em realidade.
O primeiro borrão de lâminas quando a WU (Whittle Unit) gritou em vida foi seguido
por um série de saltos para a frente. O Ministério do Ar fez seu primeiro pedido para as
ondas cerebrais de Whittle em janeiro de 1940. O primeiro avião britânico a jato decolou
de RAF Cranwell, Lincolnshire, em 15 de maio de 1941. O resto é muita história.
Nada disso ocorreu isoladamente. A história do motor a jato nunca pode ser contada
sem menciona Maxime Guillaume, que garantiu uma patente francesa para um motor a
jato com uma turbina a gás em 1921 (nenhum protótipo foi produzido, pois estava além
do escopo da tecnologia existente), e de Hans Von Ohain, que derrotou Whittle ao criar
o primeiro motor a jato totalmente operacional em 1939, enquanto a Alemanha
perseguia vantagens no conflito global.

Aula 06 –Conjunctions/Clauses/Direct and Reported Speech 62


62
www.estrategiamilitar.com.br
Aula 07 –
Linking words
Question words

Escola Naval 2021

Teacher Andrea Belo


Teacher Andrea Belo
Aula 07: Escola Naval 2021

Sumário

Introdução ........................................................................................................................ 3
Linking Words ................................................................................................................... 4
Exemplificação.............................................................................................................................................. 5
Contraste....................................................................................................................................................... 7
Resumo .......................................................................................................................................................... 9
Adição ............................................................................................................................................................ 9
Sequência ....................................................................................................................................................12
Question Words .............................................................................................................. 15
What ............................................................................................................................................................15
Which ...........................................................................................................................................................16
When ...........................................................................................................................................................17
Where ..........................................................................................................................................................17
Why ..............................................................................................................................................................18
Who/Whom ................................................................................................................................................19
Whose ..........................................................................................................................................................20
How ..............................................................................................................................................................21
Questão inédita .............................................................................................................. 23
Questões de anos anteriores ........................................................................................... 25
11. Gabarito................................................................................................................. 35
12. Questões comentadas ............................................................................................ 36
Considerações finais........................................................................................................ 54
14. Referências bibliográficas .......................................................................................... 55
15. Traduções.................................................................................................................. 58

Aula 07 - Linking words/Question words 2


www.estrategiamilitar.com.br 61
Teacher Andrea Belo
Aula 07: Escola Naval 2021

Introdução

Desta vez, vamos à nossa aula sobre os tópicos que proporcionam ligação de ideias: linking
words ou connectors (termos de ligação/conectivos) e também palavras especiais que levam à
possibilidade de fazer perguntas: questions words, que também são chamados de Wh questions.

Sobre linking words, é muito importante lembrar que, na aula 05, quando foram explicadas
as conjunções inseridas nas compound sentences, foram apresentadas a você inúmeras conjunções
com a função de linking words.

Em se tratando dos question words, muitos textos das provas trazem questionamentos sobre
temas polêmicos, descobertas, assuntos para reflexão, entre outros, utilizando esses termos nos
textos propriamente ditos e, são essenciais à compreensão do que se pergunta – se refere a um
lugar, ao tempo, a uma pessoa, ao motivo ou a qualquer tipo de informação que se pergunta.

Vamos estudar cada um deles e a situação em que são inseridos nas leituras que você vai
fazer e precisa conhecê-los.

Os tópicos que vamos estudar hoje são palavras necessárias para o vocabulário de quem
busca acertar o máximo possível de questões das Carreiras Militares.

Elas têm a função de unir, explicar, contrastar ou exemplificar frases e, são muito importantes
porque, com o uso de variados termos como os que estudaremos, as ideias são expressas com mais
exatidão nos textos, do jeito que a prova exige de você.

Como eu sempre digo, o seu objetivo é ser aprovado e para conseguir a sua aprovação, os
estudos e aprimoramento de conteúdo e assuntos se faz necessário e válido, sempre.

Vamos então estudar, praticar exercícios, tanto durante a teoria como também no fim do
material, treinando tudo o que aprendeu, em junção ao aprendizado de cada aula.

Vamos lá e conte comigo!

Aula 07 - Linking words/Question words 3


www.estrategiamilitar.com.br 61
Teacher Andrea Belo
Aula 07: Escola Naval 2021

Linking Words

Os conectivos – linking words – são aquelas palavras ou expressões que servem para estabelecer
uma conexão lógica entre frases e elementos de um texto.
Muitas delas são conjunções ou locuções conjuntivas, mas há também muitos advérbios,
preposições, entre outros, que estão presentes nos textos no dia da sua prova. Veja um exemplo
com “according to”:

“I am not sure but, according to Peter, everybody is going to the party”.


“Não tenho certeza mas, de acordo com Peter, todos vão para a festa”

Esses termos podem ser identificados por diversos nomes: linking words, words of transition,
connectives, words of connection, logical connectors, transition devices, cohesive devices, discourse
markers ou até connective adjuncts.
São também conhecidos como articuladores, já que ligam ideias dentro de textos e, na sua prova,
conectam aquilo que as questões exigem que você descubra, através das análises que sempre
fazemos nas aulas e o conhecimento necessário que você precisa e estou proporcionando mais para
o seu sucesso.
O papel dos famosos linking words é estabelecer relações entre contextos – uma ideia anterior e
uma ideia posterior. Essas relações podem ser de muitos tipos, tais como exclusão, concessão,
adição, condição etc. O uso delas confere ao texto coerência e coesão.
É um assunto muito importante pois, uma vez que linking words são fundamentais para a construção
de sentenças, é, consequentemente, essencial na produção dos textos de jornais e revistas, que são
usados nas proas de Carreiras Militares, já que são termos que colaboram diretamente com uma
boa comunicação.
Lembrando que as linking words não sofre flexão de grau (aumentativo ou diminutivo), número
(singular ou plural) ou gênero (feminino ou masculino). Ou seja, são palavras invariáveis.
Linking words são fundamentais na língua inglesa, porque são como peças-chave na hora da leitura
dos textos para colaborar com o raciocínio da mensagem que as frases querem proporcionar.
Para identificar os linking words nos textos que você vai ler, você precisa ter muito claro qual é a
ideia que o narrador quer expressar.
Como eu disse antes, a maioria desses termos ligam duas ideias numa sequência, como também
pode ligar duas ideias diferentes ou tentar dizer que uma coisa depende de outra.
Vejamos alguns exemplos com o que desejam expressar dentro do texto. Contudo, é preciso
entender que a lista de linking words é extensa e você deve, aula após aula, adicionar as que
aparecerem em uma lista de estudos e assim, aprender cada dia mais.

Aula 07 - Linking words/Question words 4


www.estrategiamilitar.com.br 61
Teacher Andrea Belo
Aula 07: Escola Naval 2021

Exemplificação
Para exemplificar, uma expressão muito comum é o tão usado “for example” (por exemplo), que
também pode ser substituído por “for instance” e aparece em muitos textos da sua prova.

There are many topics to study, for example, countable and uncountable nouns.
Há muitos tópicos para estudar, por exemplo, substantivos contáveis e incontáveis, que já estudamos .

Ou então, poderíamos dizer:

There are many topics to study, for instance, countable and uncountable nouns.
Há muitos tópicos para estudar, por exemplo, substantivos contáveis e incontáveis, que já estudamos .

Para exemplificar e dar ênfase em palavras, em ideias, também são usados outros linking words,
vistos no quadro abaixo e um exercício para treinar e visualizar melhor no contexto das provas típicas
de provas:

Aula 07 - Linking words/Question words 5


www.estrategiamilitar.com.br 61
Teacher Andrea Belo
Aula 07: Escola Naval 2021

Para ilustrar os linking words de exemplificação, vejamos um texto retirado do jornal The Economist,
fonte de elaboração de provas, em que aparece “for instance”.
Se a questão abordasse os exemplos decorrentes do parágrafo em que a expressão em questão
aparece, poderia ser assim:

Questão - According to the sentence “... India, for example, has only 11 judges for every 1
million people”, the underlined term refers to
(A) judges
(B) India
(C) 1 million people
(D) India population
(E) Every 1 million people

A resposta seria a letra A porque é a única que demonstra o exemplo exatos do que se
trata a referência de “for example” (por exemplo), logo após falar da India e, afirmar que há
apenas 11 juízes para cada 1 milhão de pessoas.

Agora vejamos linking words de contraste a seguir.

Aula 07 - Linking words/Question words 6


www.estrategiamilitar.com.br 61
Teacher Andrea Belo
Aula 07: Escola Naval 2021

Contraste
Para contrastar, o linking word comum é o “but” (mas), que também pode ser substituído por
“however” e aparece em muitos textos da sua prova, inclusive, perguntando se pode haver a devida
substituição.

She loves going to the beach but never on Saturdays, it’s crowded.
Ela ama ir à praia mas nunca aos sábados, é muito cheio.

Ou então, poderíamos dizer:

She loves going to the beach however she never goes on Saturdays, it’s crowded.
Ela ama ir à praia mas ela nunca vai aos sábados, é muito cheio.

Podemos usar, também para contrastar, “despite” e “in spite of”, que são seguidas por substantivos
ou gerúndios.

Despite losing the match, the players celebrated their efforts.


Apesar de perder o jogo, os jogadores comemoraram o esforço.

Ou então, poderíamos dizer:

In spite of the lost match, the players celebrated their efforts.


Apesar da perda do jogo, os jogadores comemoraram o esforço.

Para exemplificar, com ideia de contraste, também muito usadas, vejamos o quadro:

Aula 07 - Linking words/Question words 7


www.estrategiamilitar.com.br 61
Teacher Andrea Belo
Aula 07: Escola Naval 2021

Para construções seguidas por sujeito e um verbo, precisa-se adicionar “the fact that”:

Despite the fact that they lost the match, the players celebrated their efforts.
Apesar do fato de perder o jogo, os jogadores comemoraram o esforço.

Ou

In spite of the fact that they lost the match, the players celebrated their efforts.
Apesar do fato de perder o jogo, os jogadores comemoraram o esforço.

Agora vejamos linking words com a função de resumir a seguir.

Aula 07 - Linking words/Question words 8


www.estrategiamilitar.com.br 61
Teacher Andrea Belo
Aula 07: Escola Naval 2021

Resumo

Para resumir, há linking words comuns, tais como é o “in conclusion, in summary” (em síntese/ em
conclusão a, em suma), que são expressões geralmente usadas no começo das frases, indicando que
vamos resumir a ideia principal do que acabou de ser apresentada.

In conclusion, the meeting was very productive, and the ideas were well presented.
Em síntese, a reunião foi produtiva e as ideias foram bem apresentadas.

Ou então, poderíamos dizer:

In summary, the meeting was very productive, and the ideas were well presented.
Em síntese, a reunião foi produtiva e as ideias foram bem apresentadas.

Adição

Para adicionar, os linking words comuns são “and” e “also” e, outras expressões geralmente usadas
para adicionar ideias de maneira mais formal seriam “furthermore” e “moreover”, usadas bastante
em textos da sua prova.

The meeting was very productive. Moreover lots of ideas were presented.
A reunião foi muito produtiva. Além disso, muitas ideias foram apresentadas.

Ou então, poderíamos dizer:

The meeting was very productive and lots of ideas were presented.
A reunião foi muito produtiva e muitas ideias foram apresentadas.

Aula 07 - Linking words/Question words 9


www.estrategiamilitar.com.br 61
Teacher Andrea Belo
Aula 07: Escola Naval 2021

Para exemplificar, desta vez com ideia de adição, também muito usadas nas provas, vejamos o
quadro abaixo e um exercício para treinar e visualizar melhor como nas provas: :

Para ilustrar mais uma vez os linking words de adição, vejamos um texto retirado do jornal The
Guardian, que também aparece nas provas, em que aparece “and” muitas vezes no texto. E a
questão poderia ser assim:

Aula 07 - Linking words/Question words 10


www.estrategiamilitar.com.br 61
Teacher Andrea Belo
Aula 07: Escola Naval 2021

Questão – Percebe-se a ideia de adição no primeiro parágrafo, em

(A) book, literary criticism, philosophical reflection.

(B) literary criticism, philosophical reflection, imaginations.

(C) book, literary criticism, alternative versions.

(D) book, literary criticism, central idea.

(E) literary criticism, philosophical reflection.

Comentários:

A resposta seria a letra E porque a adição, representada desta vez pelo linking word and,

inúmeras vezes, mostra ideias de algo sendo adicionada a cada vez que aparece no texto,

conforme vimos exemplos acima.

Perceba que, a única opção em que a ordem das palavras está de acordo com o que é

adicionado, ou seja, logo após book e o linking word “and”, é a letra B de fato: “book... and ...

a characteristic blend of literary criticismand philosophical reflection...”

Agora vejamos linking words com a função de dar a ideia de sequência a seguir.

Aula 07 - Linking words/Question words 11


www.estrategiamilitar.com.br 61
Teacher Andrea Belo
Aula 07: Escola Naval 2021

Sequência
]

Para oferecer a ideia de sequência, há linking words fundamentais, tais como é o “first, second, after,
then, so”, entre outros, que são expressões geralmente usadas no começo das frases, indicando que
vamos resumir a ideia principal do que acabou de ser apresentada.

First, he decided to study. Then, he bought a good material and then dedicated a lot.
Primeiro, ele decidiu estudar. Daí, ele comprou um bom material e então dedicou-se muito.

Veja uma imagem com a sequência lógica muito usada nos textos, de uma forma geral e que, uma
vez conhecendo-os, você conseguirá identificar ideias questionadas no dia da sua prova.

Em narrativas, os linking words organizam a história numa sequência de eventos, facilitando para
você na compreensão do texto durante sua leitura e busca de respostas às perguntas apresentadas
na sua prova.

Aula 07 - Linking words/Question words 12


www.estrategiamilitar.com.br 61
Teacher Andrea Belo
Aula 07: Escola Naval 2021

Os linking words funcionam como conectivos em todas as frases. Assim, elas desempenham o papel
de conectar ideias unindo termos ou mesmo orações. Mas, quando oferecem a sequência, melhor
ainda para localizar você no assunto e as partes dele enquanto está sendo apresentado.
Por esse motivo, são elementos essenciais tanto na comunicação quanto na escrita, visto que eles
colaboram com a coesão e coerência textuais.
Há ainda, linking words que explicam a razão, a condição, a causa de algo, como podemos ver no
quadro abaixo e também outros exemplos de outros linking words adiante:

Due to the heavy rain the flight was cancelled.


Devido/em decorrência da chuva forte, o vôo foi cancelado.

Ou então, poderíamos dizer:

Because of the heavy rain the flight was cancelled.


Por causa/em decorrência da chuva forte, o vôo foi cancelado.

Aula 07 - Linking words/Question words 13


www.estrategiamilitar.com.br 61
Teacher Andrea Belo
Aula 07: Escola Naval 2021

E agora, vamos aos estudos dos Questions Words.


Are you ready?

Aula 07 - Linking words/Question words 14


www.estrategiamilitar.com.br 61
Teacher Andrea Belo
Aula 07: Escola Naval 2021

Question Words
Os Question Words são pronomes interrogativos utilizados para elaborar perguntas em Inglês. Eles
são empregados antes dos verbos auxiliares e modais para se questionar algo.
Vale lembrar que, em Português, podemos transformar qualquer afirmação em pergunta somente
mudando a entonação, o que é diferente em Inglês, como você tem visto em nossas aulas e
praticamos bem isso na aula de verbos, com os devidos auxiliares de cada tempo verbal.
Muitas vezes, os question words são chamados Wh Questions porque eles contêm as letras “W” e
“H” em sua estrutura, veja: What (O quê/qual), Which (O quê/qual), When (Quando), Who (Quem),
When (Quando), Why (Por quê), HoW (Como), entre outros que estudaremos.

What
What significa o quê/qual e é usado para perguntar sobre objetos, situações, assuntos diversos e
tudo aquilo que não sabemos. É o Wh question mais genérico de todos e, consequentemente, o mais
usado em textos dos mais variados tipos.
A pergunta feita com WHAT é geral, como abaixo, qual é o seu nome, endereço, o que você gosta,
que horas são e o que você faz, a resposta pode ser qualquer uma e não possui escolhas, como entre
duas ou mais coisas que você goste.
Se perguntar “o que você gosta?” – What do you like, a resposta pode ser o que vier em sua mente,
diferente de WHICH, que veremos a seguir.

WHAT'S
YOUR
NAME?

WHAT'S
WHAT DO
YOUR
YOU DO? ADDRESS?

WHAT

WHAT
WHAT DO
TIME IS
IT? YOU LIKE?

Aula 07 - Linking words/Question words 15


www.estrategiamilitar.com.br 61
Teacher Andrea Belo
Aula 07: Escola Naval 2021

Which
Which também significa o quê/qual porém, é usado quando temos opções limitadas, escolhas.
Enquanto what é usado para perguntas de um modo geral, o which é usado quando são oferecidas
opções de respostas.
Por exemplo, a pergunta sobre o que você gosta – “What do you like?”, usando o which, você
provavelmente precisaria escolher algo que gosta, como: “Which do you like, pizza or ice-cream?”,
em que sua resposta tem que ser uma das duas ou mais opções.
Veja outros exemplos:

WHAT DO YOU EAT IN


THE MORNING? WHICH DO YOU EAT IN THE
MORNING, BREAD OR TOAST?

WHICH
NAME DO
YOU PREFER,
TOM OR
JOHN?

WHICH JOB IS WHICH CITY


BETTER, WOULD YOU
BEING A LIVE, NEW
TEACHER OR YORK OR
A LAWYER? LONDON?
WHICH

WHICH KIND
OF MOVIE DO WHAT DO YOU
LIKE IN THE
YOU LIKE THE
MORNING,
MOST,
SUSPENSE OR MILK OR
JUICE?
DRAMA?

Aula 07 - Linking words/Question words 16


www.estrategiamilitar.com.br 61
Teacher Andrea Belo
Aula 07: Escola Naval 2021

When
When significa o quando e é usado para saber sobre tempo/período ou ocasião - quando algo
aconteceu, acontece ou irá acontecer.

WHEN IS
YOUR
BIRTHDAY?

WHEN ARE WHEN DID


YOU GOING YOU
TO DECIDE? GRADUATE?
WHEN?

WHEN WILL WHEN DO


YOU TRAVEL YOU GO TO
AGAIN? THE GYM?

Where
Where significa onde e é usado para saber sobre local, localização.

WHERE IS
YOUR
SCHOOL?

WHERE CAN WHERE DO


YOU LEAVE YOU WORK
YOUR CAR? OUT?
WHERE?

WHERE WILL WHERE DID


YOU YOU GO
GRADUATE? YESTERDAY?

Aula 07 - Linking words/Question words 17


www.estrategiamilitar.com.br 61
Teacher Andrea Belo
Aula 07: Escola Naval 2021

Why
Why significa porque e é usado para saber o motivo, a razão de algo acontecer, ter acontecido ou
porque vai acontecer.

A resposta é sempre because – why para perguntar e because para responder.

WHY ARE YOU


TIRED?
BECAUSE I
STUDIED A
LOT.
WHY ARE YOU
LEAVING? WHY DO YOU
BECAUSE I STUDY?
HAVE TO GO. BECAUSE I
WANT TO BE
WHY? APPROVED.

WHY WILL WHY DID YOU


YOU TRAVEL? GET LATE?
BECAUSE IT'S BECAUSE OF
MY VACATION. THE TRAFFIC.

Aula 07 - Linking words/Question words 18


www.estrategiamilitar.com.br 61
Teacher Andrea Belo
Aula 07: Escola Naval 2021

Who/Whom
Who e Whom significam quem, para saber sobre pessoas mas, são usados em diferentes situações
– enquanto who tem a função de sujeito, whom tem a função de objeto, como vou mostrar abaixo.
Se a pergunta for “Quem é o ator principal desse filme?”, é “Who is this movie main actor?” (a
resposta do who será o ator, que é o sujeito da pergunta).
Mas, se a pergunta for “Sobre quem é esse filme?”, é “Whom is this movie about?” (a resposta será
sobre quem é o filme, sendo o sujeito da frase o filme e não sobre quem é). Sobre quem é o objeto.
I o whom faz exatamente esse papel: de objeto direto ou indireto nas frases.
E ainda podem aparecer, nos textos da sua prova, perguntas com a preposição “to” no final, como
por exemplo: “To whom was she talking?” (Com quem ela estava falando?), também na função de
objeto. Vejamos exemplos.

WHO ARE YOU


TALKING TO?

WHO IS THE
AUTHOR? WHO IS YOUR
WHOM IS THIS TEACHER?
STORY ABOUT?
WHO?
WHOM?

TO WHOM WITH WHOM DID


SHOULD I TALK? YOU GO OUT?

Aula 07 - Linking words/Question words 19


www.estrategiamilitar.com.br 61
Teacher Andrea Belo
Aula 07: Escola Naval 2021

Whose
Whose significa de quem e é usado para saber quem é o dono de algo, para saber a quem pertence
alguma coisa. É sempre seguido de um nome e um verbo. Assim será na prova. Veja:

WHOSE PEN IS
THIS?

WHOSE BOOK IS WHOSE


THIS? TEACHER IS
THE BEST?

WHOSE?

WHOSE CARDS WHOSE ARE


ARE THOSE? THOSE BOOKS?

Aula 07 - Linking words/Question words 20


www.estrategiamilitar.com.br 61
Teacher Andrea Belo
Aula 07: Escola Naval 2021

How
Usamos a Question Word HOW (como) quando queremos descrever a forma como algo é feito e a
condição ou qualidade. Veja alguns exemplos abaixo e outros para melhor compreensão.

HOW ARE
HOW YOU
FAST IS HOW MANY
DOING? STUDENTS?
THAT
BUS? HOW MUCH
MONEY?

HOW HIGH IS
THE
MONUMENT? HOW OLD
HOW? ARE YOU?

HOW
OFTEN HOW FAR
DO YOU IS YOUR
STUDY? WORK?
HOW BIG HOW
IS YOUR TALL ARE
HOUSE? YOU?

Outros exemplos:
How was your class? - Como está sua aula?
How do you cook vegetables? - Como você cozinha verduras?
How do you know his name? - Como você sabe o nome dele?
How can I speak English fast? - Como eu posso falar Inglês rápido?

How old: usado para perguntar a idade de alguém ou algo.


How old is you mother? Quantos anos tem sua mãe?
How old is this building? Quantos anos tem esse edifício?

Aula 07 - Linking words/Question words 21


www.estrategiamilitar.com.br 61
Teacher Andrea Belo
Aula 07: Escola Naval 2021

How long: usando para perguntar há quanto tempo, quanto tempo.


How long have you been studying? Há quanto tempo você tem estudado?

How far: usando para perguntar a distância entre uma coisa e outra.
How far is the hotel from the school? Qual é a distância entre o hotel e a escola?

How many: quantos - usado para substantivos contáveis, quando o plural é possível.
How many students can you see? Quantos alunos você consegue ver?

How much: quanto - usado para substantivos incontáveis, quando o plural não é possível.
How much money do you need? Quanto de dinheiro você precisa?
How much sugar would you like? Quanto de açúcar você gostaria?

Preparei uma tabela para você:

Agora, teremos uma questão inédita que elaborei para você, abordando os assuntos que
foram explorados nessa aula. E depois, exercícios de anos anteriores para você treinar, como sempre
fazemos em nossas aulas. Vamos lá!

Aula 07 - Linking words/Question words 22


www.estrategiamilitar.com.br 61
Teacher Andrea Belo
Aula 07: Escola Naval 2021

Questão inédita

Questão inédita Teacher Andrea Belo


Read the text and answer the question below.

Questão inédita – According to the text:


The question “And what if you lost your hand?” is related to
a) what can happen if you don’t use one of your hands.
b) what senses you lose if you don’t have one of your hands.
c) what happens if you stop working because of your hand.
d) what happens if you lose hands control.

Aula 07 - Linking words/Question words 23


www.estrategiamilitar.com.br 61
Teacher Andrea Belo
Aula 07: Escola Naval 2021

e) what can you do if you lose one of your hands.

Comentários:

Esse texto foi retirado de um trecho de um artigo da revista Scientific American, fonte que às
vezes aparece nas provas das Carreiras Militares. Vamos, é claro, usar as técnicas de Scanning
e Skimming desde o início do texto.

Você tem que encontrar a alternativa que melhor representa a pergunta destacada em Inglês
“And what if you lost your hand?”, usando o Wh question what.

Ao verificar a letra “A”, usando o auxiliar da negativa “don’t use your hands”, percebemos que
está falsa porque na pergunta com what, há o verbo lost (to lose no passado), que é perder e
não fala sobre você não usar as mãos e sim não tê-las.

N letra “B”, usando a palavra “senses”, percebemos que está falsa porque na pergunta com
what, há a palavra lost (verbo lose no passado), que é perder e não fala sobre você não usar as
mãos e sim não tê-las.

Na letra “C”, afirmando “stop working”, percebemos que está falsa porque já analisamos que
a pergunta não se trata disso.

Na letra “D”, percebemos que está falsa porque também não se fala em perder o controle das
mãos e sim perder as mãos.

Na letra “E”, afirma-se o que procuramos: o que a pessoa faria se perdesse uma das suas mãos,
que é sobre o que se trata o texto.

Agora, vamos aos exercícios de anos anteriores, como sempre fazemos em nossas aulas,
para praticar ainda mais e, após resolver, verificar os comentários de cada alternativa e
aprender, cada vez mais, vocabulário e formas de resolver as questões no dia da sua prova.

Aula 07 - Linking words/Question words 24


www.estrategiamilitar.com.br 61
Teacher Andrea Belo
Aula 07: Escola Naval 2021

Questões de anos anteriores

Você agora vai resolver questões de provas das Carreiras Militares de anos anteriores,
incluindo a Escola Naval. E, em seguida, terá acesso às respostas comentadas. Vamos lá!

QUESTION 01 (AFA/2018)

Aula 07 - Linking words/Question words 25


www.estrategiamilitar.com.br 61
Teacher Andrea Belo
Aula 07: Escola Naval 2021

Questão 01 (AFA/2018) – Some environmental factors that have contributed to food shortage
are
a) land degradation, deforestation and fuel prices.
b) climatic change, combustion of fossil fuels and floods.
c) agriculture, fertilizers and cost of food production.
d) droughts, tropical storms and reduced investments in agriculture.

Questão 02 -

Aula 07 - Linking words/Question words 26


www.estrategiamilitar.com.br 61
Teacher Andrea Belo
Aula 07: Escola Naval 2021

Questão 03 (Escola Naval/2017)

Aula 07 - Linking words/Question words 27


www.estrategiamilitar.com.br 61
Teacher Andrea Belo
Aula 07: Escola Naval 2021

Questão 03 (Escola Naval/2017)

Questão 04 (EPCAR/2015)

Questão 04 (EPCAR/) – In the underlined sentences, there are verbs used in the Past Simple
a) six
b) five
c) four
d) three

Aula 07 - Linking words/Question words 28


www.estrategiamilitar.com.br 61
Teacher Andrea Belo
Aula 07: Escola Naval 2021

Questão 05 (EAM/inédita)

Questão 05 (EAM/inédita) - About the sentence “If you don’t, you probably know someone
who has one (or wants one)”, it is correct to say that the word which replaces “one” is
a) i-phone
b) computer
c) someone
d) you
e) company

Aula 07 - Linking words/Question words 29


www.estrategiamilitar.com.br 61
Teacher Andrea Belo
Aula 07: Escola Naval 2021

Questões 06, 07 e 08 (EEAR/ 2017)

TEXT: Bento boxes (TEXTO para QUESTÕES de 06 a 08)

People in offices in Japan often have a bento at lunchtime. They do not eat a meal in a
restaurant; they eat a bento in the office. They regularly have a menu for the bentos in the
office and they telephone a bento store with their orders. The people at bento store make the
bentos and take them to all the offices at about 12p.m.

Questão 06 – The underlined words in the text, often and regularly are
closest in meaning to
a) always.
b) seldom.
c) frequently.
d) sometimes.

Questão 07 – According to the text, Bento Boxes is a/ an.


a) office
b) kitchen
c) delivery
d) restaurant

Questão 08 – According to the text, the word “orders”, underlined, means


a) something organized in a particular way.
b) the situation that exists when everything is in the correct place.
c) A group of nuns who live according to a particular set of rules.
d) A request for something to be brought for someone in return for money.

Aula 07 - Linking words/Question words 30


www.estrategiamilitar.com.br 61
Teacher Andrea Belo
Aula 07: Escola Naval 2021

Questão 09 (ESA/inédita)

Questão 09 (ESA/inédita) – According to the text, it is correct to say that bowel is


a) gut
b) chin
c) elbow
d) bladder
e) thigh

Questão 10 (EsPCEx/2019)

Questão 10 (EsPCEx/2019) – According to the passage, Michael D. Coe’s book


[A] carbon dioxide separation technology for industrial and gas treating applications.
[B] estimation of soil carbon saturation that indicates its potential to store more carbon.
[C] the amount of carbon dioxide produced by the activities of a company.
[D] species that are particularly sensitive and disappear after a pollution event.
[E] long-term rise in the average temperature of the Earth’s climate system.

Aula 07 - Linking words/Question words 31


www.estrategiamilitar.com.br 61
Teacher Andrea Belo
Aula 07: Escola Naval 2021

11 (2016)

QUESTÃO 11 - In lines 2 – 3: “(…) and despite an unresolved lawsuit that has delayed the project
(...)”, the word in bold is formed by the prefix 'un'. In which option below the word is not
correctly formed by the same prefix?
( a ) unfinished
( b ) unfasten
( c ) unwrapped
( d ) undecided
( e ) unpolite

Aula 07 - Linking words/Question words 32


www.estrategiamilitar.com.br 61
Teacher Andrea Belo
Aula 07: Escola Naval 2021

QUESTÕES ITA/2018

Texto para responder as questões 12 a 15

Questão 12 - O texto é um(a):


A ( ) artigo de opinião.
B ( ) artigo científico.
C ( ) resumo de projeto de pesquisa.
D ( ) relato de experiência.
E ( ) chamada para publicação.

Questão 13 – Considere as seguintes afirmações:


I. A melhoria ou o aumento das funções cerebrais pode ser obtida via: estimulação
farmacológica, interface cérebro-máquina, implantes cerebrais ou mesmo remoção de
determinadas áreas do cérebro.
II. Atualmente, abordagens para melhoria das funções cerebrais envolvem exclusivamente
funções sensoriais, cognitivas e motoras.
III. O aumento das funções cerebrais pode vir a ser usado no tratamento de doenças
neurodegenerativas.

Aula 07 - Linking words/Question words 33


www.estrategiamilitar.com.br 61
Teacher Andrea Belo
Aula 07: Escola Naval 2021

Com base no texto, estão corretas


A ( ) apenas I e II.
B ( ) apenas I e III.
C ( ) apenas II e III.
D ( ) apenas III.
E ( ) todas.

Questão 14 – Com base no texto, é INCORRETO afirmar que estudos sobre a melhoria das
funções cerebrais
A ( ) devem-se aos avanços que foram conquistados no campo das Neurociências.
B ( ) são realizados tanto na esfera computacional quanto no domínio experimental.
C ( ) são realizados por cientistas de diferentes áreas.
D ( ) ainda estão apenas no campo da ficção científica.
E ( ) englobam questões de natureza ética e filosófica.

Questão 15 – Marque a opção que indica a que it se refere no seguinte excerto: "...it has
become a matter of reality…" (linha 2).
A ( ) advances in neural sciences
B ( ) treatment of a neurodegenerative disease
C ( ) some point in life
D ( ) science fiction
E ( ) augmentation of brain function

Aula 07 - Linking words/Question words 34


www.estrategiamilitar.com.br 61
Teacher Andrea Belo
Aula 07: Escola Naval 2021

11. Gabarito

1–B 2–D 3–B 4– C 5–A

6– C 7–C 8– C 9–A 10 – C

11 – E 12 – E 13 – B 14 – D 15 – E

Aula 07 - Linking words/Question words 35


www.estrategiamilitar.com.br 61
Teacher Andrea Belo
Aula 07: Escola Naval 2021

12. Questões comentadas

QUESTION 01 (AFA/2018)

Aula 07 - Linking words/Question words 36


www.estrategiamilitar.com.br 61
Teacher Andrea Belo
Aula 07: Escola Naval 2021

Questão 01 (AFA/2018) – Some environmental factors that have contributed to food shortage

are

a) land degradation, deforestation and fuel prices.

b) climatic change, combustion of fossil fuels and floods.

c) agriculture, fertilizers and cost of food production.

d) droughts, tropical storms and reduced investments in agriculture.

Comentários:

A informação acerca dos “environmental factor” está entre as lines 34 e 51.

Alternative A is incorrect, pois “fuel prices” não é mencionado como “environmental factor”

no texto.

Alternative B is correct, pois esses três fatores são discutidos no parágrafo que se relaciona ao

“environment”.

Alternative C is incorrect, pois “cost of food production” entra em “economic factors”, não

“environmental”.

Alternative D is incorrect, pois “reduced investments in agriculture” se relaciona às políticas

econômicas governamentais, não ao meio ambiente.

Aula 07 - Linking words/Question words 37


www.estrategiamilitar.com.br 61
Teacher Andrea Belo
Aula 07: Escola Naval 2021

Questão 02 -

Aula 07 - Linking words/Question words 38


www.estrategiamilitar.com.br 61
Teacher Andrea Belo
Aula 07: Escola Naval 2021

Questão 02 (Colégio Naval/2018)

Comentários:

Essa questão pede que encontremos a única alternativa que não traz uma palavra equivalente

(não um sinônimo) a“appealing” na frase “many noteworthy movies are filmed in appealing

locales”. Então, estamos procurando a alternative que não traz adjectives pos itivos, a fim de

tornarem o noun “locales” digno de ser visitado.

Alternative A is correct, pois traz uma ideia positiva para o noun “locales”, o que o torna um

destino atrativo aos viajantes.

Alternative B is correct, pois “pleasing” traz a ideia de “prazeroso”, positiva, portanto.

Alternative C is correct,pois “lovable” identifica um lugar bacana, já que remete ao adjective

“love”.

Alternative D is incorrect, pois se o noun “locales” recebe o adjective “repulsive”, isso o

caracteriza como um destino ruim para visitar, o que não faria sentido nessa frase.

Alternative E is correct, pois se um destino é descrito como “atractive”, isso traz uma ideia

positiva e, portanto, pode ser uma substituição viável.

Aula 07 - Linking words/Question words 39


www.estrategiamilitar.com.br 61
Teacher Andrea Belo
Aula 07: Escola Naval 2021

Questão 03 (Escola Naval/2017)

Aula 07 - Linking words/Question words 40


www.estrategiamilitar.com.br 61
Teacher Andrea Belo
Aula 07: Escola Naval 2021

Questão 03 (Escola Naval/2017)

Comentários:

Alternative A is incorrect, pois no 6º parágrafo se afirma que Gore gostaria de impedir que a

violência ocorresse(”Yet Gore wanted to prevent people from being violently injured in the first

place.”).

Alternative B is correct, como podemos perceber pelo conteúdo do 3º parágrafo do texto.

Alternative C is incorrect, pois, de acordo com o 5º parágrafo do texto, KAVI foi criado pelo Dr.

Rob Gore no hospital Kings County.

Alternative D is incorrect, pois, de acordo com a frase do 6º parágrafo: “Today, KAVI holds

weekly workshops for male and female students in three schools”.

Alternative E is incorrect, pois, de acordo com penúltimo parágrafo do texto: “While Gore still

regularly attends workshops, …”.

Aula 07 - Linking words/Question words 41


www.estrategiamilitar.com.br 61
Teacher Andrea Belo
Aula 07: Escola Naval 2021

Questão 04 (EPCAR/2015)

Questão 04 (EPCAR/2015) – In the underlined sentences, there are verbs used in the Past
Simple
a) six
b) five
c) four
d) three

Comentários:
Precisamos identificar os verbos nas sentenças sublinhadas. Uma estratégia para isso é
identificar os sujeitos (que são pessoas ou personal pronouns – I, you, he, she, etc...), pois o
verbos provavelmente estarão em seguida. Então temos:
I studied, I took, I was, the instructor used.
Perceba que precisamos apenas do verbs em past tense, porém, todos estão:
Regulares: study – studied and use – used.
Irregulares: take – took, be – was
Portanto, alternative C is correct.

Aula 07 - Linking words/Question words 42


www.estrategiamilitar.com.br 61
Teacher Andrea Belo
Aula 07: Escola Naval 2021

Questão 05 (EAM/inédita)

Questão 05 (EAM/inédita) - About the sentence “If you don’t, you probably know someone
who has one (or wants one)”, it is correct to say that the word which replaces “one” is
a) i-phone
b) computer
c) someone
d) you
e) company

Comentários:
Para responder a questão pedida precisamos olhar para mais que apenas o trecho mencionado.
“Do you have an iPad, iPod, iPhone, a smartwatch or a Mac computer? If you don’t, you
probably know someone who has one (or wants one)!”
A primeira parte desse trecho pergunta se o leitor possui alguns dos produtos famosos da
empresa Apple. Então, a parte inicial da segunda frase é uma condicional que verifica as
respostas negativas. Transformando-a em uma oração condicional completa teríamos “if you
don’t have an iPad, iPod, iPhone, a smartwatch or a Mac computer, ...”. Por fim, em seguida,
temos a oração consecutiva caso a condicional anterior seja atendida.
Nessa última frase, temos: “you probably know someone who has one (or wants one)”, isto é,
“you probably know someone who has an iPad, iPod, iPhone, a smartwatch or a Mac computer
(or wants an iPad, iPod, iPhone, a smartwatch or a Mac computer)”.

Aula 07 - Linking words/Question words 43


www.estrategiamilitar.com.br 61
Teacher Andrea Belo
Aula 07: Escola Naval 2021

Portanto, a resposta correta é alternative A.


A alternative B está incorreta, pois, especificamente no contexto, não se trata de qualquer
computer, mas de um Mac computer. Por isso, a alternative A está melhor que a B, já que
Iphones são produtos exclusivos da Apple.
A alternativa C está incorreta, pois não faria sentido a frase “you probably know someone who
has someone (or wnats someone)”. O mesmo para a alternative D: “you probably know
someone who has you (or wnats you)”.
A alternative E está incorreta, pois a company (no caso, Apple) ainda nem havia sido
mencionada, portanto, não poderia estar sendo retomada.

Questões 06, 07 e 08 (EEAR/ 2017)

TEXT: Bento boxes (TEXTO para QUESTÕES de 06 a 08)

People in offices in Japan often have a bento at lunchtime. They do not eat a meal in a
restaurant; they eat a bento in the office. They regularly have a menu for the bentos in the
office and they telephone a bento store with their orders. The people at bento store make the
bentos and take them to all the offices at about 12p.m.

Questão 06 – The underlined words in the text, often and regularly are
closest in meaning to
a) always.
b) seldom.
c) frequently.
d) sometimes.
Comentários:
As palavras sublinhadas são frequency adverbs que indicam uma frequência intermediária,
próxima da frequência elevada.
Alternative A is incorrect, pois “always” indica uma frequência total, “sempre”.
Alternative B is incorrect, pois “seldom” significa uma frequência intermediária próxima da
mais baixa.
Alternative C is correct, pois “frequently” indica uma frequência não de 100%, mas elevada.
Alternative D is incorrect, pois “sometimes” significa uma frequência mais baixa.

Aula 07 - Linking words/Question words 44


www.estrategiamilitar.com.br 61
Teacher Andrea Belo
Aula 07: Escola Naval 2021

Questão 07 – According to the text, Bento Boxes is a/ an.


a) office
b) kitchen
c) delivery
d) restaurant
Comentários:
Na letra C temos que "Bento Boxes" é um "delivery", o que é correto. De acordo com o texto
no trecho "They do not eat a meal in a restaurant; they eat a bento in the office", sabemos que
eles não comem uma refeição em um restaurante e sim comem um bento no escritório, ou
seja, uma ideia de um pedido de comida. Além disso temos ainda a frase "they telephone a
bento store with their orders", ou seja, eles telefonam para uma loja bento com seus pedidos,
novamente indicando delivery.

Portanto, não há como ser letras A, B ou D.

Questão 08 – According to the text, the word “orders”, underlined, means


a) something organized in a particular way.
b) the situation that exists when everything is in the correct place.
c) A group of nuns who live according to a particular set of rules.
d) A request for something to be brought for someone in return for money.
Comentários:
Alternative A is incorrect, pois o texto afirma que as pessoas vão ao bento boxes fazer “orders”
(pedidos) na hora do “lunch” (almoço), logo, não se trata de um escritório (“office”).
Alternative B is incorrect, pois o lugar trabalha com “pedidos”, então é um estabelecimento
comercial, não uma cozinha (“kitchen”).
Alternative C is correct, pois se as pessoas pedem (“orders”) no “lunch”, esse estabelecimento
é um comercial de delivery, isto é, que leva a comida até elas.
Alternative D is incorrect, pois o estabelecimento se diferencia do restaurante por entregar as
“orders”.

Aula 07 - Linking words/Question words 45


www.estrategiamilitar.com.br 61
Teacher Andrea Belo
Aula 07: Escola Naval 2021

Questão 09 (ESA/inédita)

Questão 09 (ESA/inédita) – According to the text, it is correct to say that bowel is

a) gut

b) chin

c) elbow

d) bladder

e) thigh

Comentários:

Essa questão trabalha principalmente conhecimento de vocabulário.

“Bowel” é sinônimo de “gut” ou “intestin”. Por isso, a alternativa correta é A.

Vou trazer o significado das outras alternativas para aumentarmos nosso vocabulário! Lembre-
se de anotar as novas palavras!

“Chin” é queixo, “Elbow” é cotovelo, “Bladder” é bexiga e “Thigh” é coxa.

Aula 07 - Linking words/Question words 46


www.estrategiamilitar.com.br 61
Teacher Andrea Belo
Aula 07: Escola Naval 2021

Questão 10 (EsPCEx/2019)

Questão 10 (EsPCEx/2019) – According to the passage, Michael D. Coe’s book


[A] carbon dioxide separation technology for industrial and gas treating applications.
[B] estimation of soil carbon saturation that indicates its potential to store more carbon.
[C] the amount of carbon dioxide produced by the activities of a company.
[D] species that are particularly sensitive and disappear after a pollution event.
[E] long-term rise in the average temperature of the Earth’s climate system.

Comentários:

Alternative A is incorrect, pois a expressão "carbon footprint" não se relaciona com nenhuma
tecnologia de "separation" para tratamento de gás.

Alternative B is incorrect, pois a "carbon footprint" não se relaciona com a quantidade de


"carbon saturation" em um determinado solo.

Alternative C is correct, pois a expressão “carbon footprint” se relaciona de fato às “pegadas


de carbono” de uma empresa, isto é, a quantidade de carbono que ela emite.

Alternative D is incorrect, pois a expressão não se refere a uma espécie de ser vivo.

Alternative E is incorrect, pois essa alternative descreve “aquecimento global” e não “emissões
de carbono”.

Aula 07 - Linking words/Question words 47


www.estrategiamilitar.com.br 61
Teacher Andrea Belo
Aula 07: Escola Naval 2021

11 (2016)

QUESTÃO 11 - In lines 2 – 3: “(…) and despite an unresolved lawsuit that has delayed the project
(...)”, the word in bold is formed by the prefix 'un'. In which option below the word is not
correctly formed by the same prefix?
( a ) unfinished
( b ) unfasten
( c ) unwrapped
( d ) undecided
( e ) unpolite

Aula 07 - Linking words/Question words 48


www.estrategiamilitar.com.br 61
Teacher Andrea Belo
Aula 07: Escola Naval 2021

Comentários:
O prefixo “un” indica negação.
Alternative A is correct, pois o adjective “finished” ganha ideia de negação pela adição de “un”
ao formar o adjective “unfinished”.
Alternative B is correct, pois a palavra “fasten” significa apertar, como por exemplo na fala da
aeromoça no avião, pedindo que apertemos os cintos de segurança: “lease, fasten your
seatbelts.”. Para dizer para soltarmos, a palavra seria “unfasten”, configurando uso de “un”
para negação.
Alternative C is correct, pois “wrapped” significa o adjective “embrulhado”, como embrulhado
para presente. A adição de “un” de fato traz a negativa “não embrulhado”.
Alternative D is correct, pois o adjective “decided” ganha ideia de negação com a adição do
prefixo “un”.
Alternative E is incorrect, pois o negative do adjective “polite” se dá pela adição do prefixo
“im”: “impolite”.
, pois “budgets overruns” ocorreram, ao invés de terem sido evitados (“prevented”).

QUESTÕES ITA/2018

Texto para responder as questões 12 a 15

Aula 07 - Linking words/Question words 49


www.estrategiamilitar.com.br 61
Teacher Andrea Belo
Aula 07: Escola Naval 2021

Questão 12 - O texto é um(a):


A ( ) artigo de opinião.
B ( ) artigo científico.
C ( ) resumo de projeto de pesquisa.
D ( ) relato de experiência.
E ( ) chamada para publicação.

Comentários:
O primeiro parágrafo do texto faz uma pequena introdução sobre o que será tratado no texto.
Afirma que o aumento da função cerebral já não é mais tema de ficção científica. Em seguida,
o segundo parágrafo inicia com “In this Research Topic, we welcome papers critically evaluating
the existing methods of brain augmentation...” (Neste tópico de pesquisa, são bem-vindos
artigos que avaliam criticamente os métodos existentes de aumento...) e segue citando outros
diversos tipos de trabalhos que o tópico de pesquisa aceita.
Assim é possível concluir que o texto faz uma chamada para publicação de trabalhos científicos
sobre o tema citado no primeiro parágrafo. Expressões como “we welcome...”, “... are also
welcome” enfatizam este tipo de publicação. Portanto a resposta da questão é a letra “E”.
Vamos analisar os outros itens.
Na letra A, não pode ser “artigo de opinião” pois o autor não dá sua opinião sobre o tema, além
disso não faz sentido o autor aceitar trabalhos e artigos científicos. Alternativa falsa.
Na letra B, também não pode ser “artigo científico” pois o autor não mostra um a pesquisa
científica apenas introduz um tema. Portanto, alternativa falsa.
Na letra C, não pode ser um “resumo de projetos de pesquisa” pois o autor não está resumindo
nenhum trabalho ou projeto ao dizer que aceita diversos tipos de trabalhos científicos, artigos,
comentários, entre outros projetos sobre o tema citado. Alternativa falsa.
Na letra D, não pode ser um “relato de experiência” pois o autor não está relatando nenhuma
experiência pessoal. Alternativa falsa novamente.

Questão 13 – Considere as seguintes afirmações:


I. A melhoria ou o aumento das funções cerebrais pode ser obtida via: estimulação
farmacológica, interface cérebro-máquina, implantes cerebrais ou mesmo remoção de
determinadas áreas do cérebro.
II. Atualmente, abordagens para melhoria das funções cerebrais envolvem exclusivamente
funções sensoriais, cognitivas e motoras.
III. O aumento das funções cerebrais pode vir a ser usado no tratamento de doenças
neurodegenerativas.

Aula 07 - Linking words/Question words 50


www.estrategiamilitar.com.br 61
Teacher Andrea Belo
Aula 07: Escola Naval 2021

Com base no texto, estão corretas

A ( ) apenas I e II.

B ( ) apenas I e III.

C ( ) apenas II e III.

D ( ) apenas III.

E ( ) todas.

Comentários:

Vamos analisar todas as afirmações.

I) De fato, esta afirmação é correta pois o texto diz que aprimoramentos (aumento ou
melhoria) das funções cerebrais podem ser obtidos através de implantes cerebrais,
estimulação de medicamentos (estimulação farmacológica), emprego de interfaces cérebro-
máquina, ablação de certas áreas do cérebro (implantes cerebrais ou mesmo remoção de
determinadas áreas do cérebro). Portanto afirmação verdadeira.

II) Afirmação falsa, pois não são somente as funções sensoriais, cognitivas e motoras que
melhoram as funções cerebrais. O texto em nenhum momento afirma que estas são somente
as únicas abordagens. O uso da palavra “exclusivamente” faz com que a afirmação seja falsa.

III) Afirmação verdadeira, pois, o texto, no início do primeiro parágrafo, afirma que “...it has
become a matter of reality that a person may consider at some point in life, for example as a
treatment of a neurodegenerative disease” ou seja, o aumento de funções cerebrais tornou-se
uma questão de realidade que uma pessoa pode considerar em algum momento de sua vida
por exemplo como tratamento de uma doença neurodegenerativa.

Portanto a resposta da questão é a letra “B”. As afirmações I e III são verdadeiras.

Aula 07 - Linking words/Question words 51


www.estrategiamilitar.com.br 61
Teacher Andrea Belo
Aula 07: Escola Naval 2021

Questão 14 – Com base no texto, é INCORRETO afirmar que estudos sobre a melhoria das
funções cerebrais
A ( ) devem-se aos avanços que foram conquistados no campo das Neurociências.
B ( ) são realizados tanto na esfera computacional quanto no domínio experimental.
C ( ) são realizados por cientistas de diferentes áreas.
D ( ) ainda estão apenas no campo da ficção científica.
E ( ) englobam questões de natureza ética e filosófica.

Comentários:

Vamos analisar item por item.

Na letra A, temos que “a melhoria das funções cerebrais se deve aos avanços que foram
conquistados no campo das Neurociências” é uma afirmação verdadeira. Logo no início do
texto temos o trecho “Due to advances in neural sciences, it has become...” ou seja, devido aos
avanços das ciências neurais o aumento das funções cerebrais não são mais tema de ficção
científica. Como a questão pede a afirmação incorreta, então alternativa errada.

Na letra B, “as melhorias das funções cerebrais são realizados tanto na esfera computacional
quanto no domínio experimental” é verdadeira pois no segundo parágrafo temos “The work
can be experimental or computational” mostrando que as pesquisas podem ser tanto
experimentais quanto computacionais. Afirmação correta e portanto alternativa falsa.

Na letra C, “as melhorias das funções cerebrais são realizados por cientistas de diferentes
áreas” é uma afirmação verdadeira pois o texto faz uma chamada de publicação e afirma que
aceita cientistas de diferentes áreas (“We welcome scientists from different fields” (linha 9)).
Portanto, alternativa falsa.

Na letra D, “as melhorias das funções cerebrais ainda estão apenas no campo da ficção
científica” é uma afirmação falsa pois logo na primeira frase o texto afirma que o aumento de
funções cerebrais não é mais tema de ficção científica – “Augmentation of brain function is no
longer just a theme of science fiction”. Afirmação falsa e, portanto, alternativa correta.
Portanto a resposta da questão é a letra “D”.

Aula 07 - Linking words/Question words 52


www.estrategiamilitar.com.br 61
Teacher Andrea Belo
Aula 07: Escola Naval 2021

Questão 15 – Marque a opção que indica a que it se refere no seguinte excerto: "...it has
become a matter of reality…" (linha 2).

A ( ) advances in neural sciences


B ( ) treatment of a neurodegenerative disease
C ( ) some point in life
D ( ) science fiction
E ( ) augmentation of brain function
(E) bens.
Comentários:

A palavra “it” se refere ao aumento de funções cerebrais citado na primeira frase. A frase
seguinte diz que devido aos avanços nas ciências neurais este aumento de funções cerebrais
poderá tornar-se uma questão de realidade que uma pessoa pode considerar em algum
momento de sua vida por exemplo como tratamento de uma doença neurodegenerativa.
Portanto a resposta da questão é a letra “E”.

Vamos analisar outras alternativas.

Na letra A, não pode ser “advances in neural sciences” pois é devido a estes avanços que algo
se tornou uma realidade para pessoas. Este algo não é o avanço na área de ciências neurais.
Alternativa falsa.

Na letra B, também não pode ser “treatment of a neurodegenerative disease” pois o autor cita
isto depois do uso de “it” e o “it” é usado para se referir a algo já citado anteriormente.
Portanto, alternativa falsa.

Na letra C, não pode ser um “some point in life” pois o autor cita isto depois do uso de “it” e o
“it” é usado para se referir a algo já citado anteriormente. Portanto, alternativa falsa.

Na letra D, não pode ser um “science fiction” pois não faz sentido dizer que a ficção científica
se tornou uma questão de realidade que uma pessoa pode considerar em algum momento de
sua vida por exemplo como tratamento de uma doença neurodegenerativa. Alternativa falsa
novamente.

Aula 07 - Linking words/Question words 53


www.estrategiamilitar.com.br 61
Teacher Andrea Belo
Aula 07: Escola Naval 2021

Considerações finais
Concluímos mais uma aula, outro passo até a sua aprovação!
E, como eu sempre digo, estamos caminhando para maior vocabulário e mais aprendizado de
fato, com importantes temas e tópicos, aula após aula.
Nota-se o progresso em seus estudos e, provavelmente, uma maior tranquilidade para
enfrentar os exercícios que surgem. E você vai se acostumando a equilibrar seus estudos de forma
sistematizada, estudando cada vez mais e com mais dedicação.

Outro detalhe importante para seu sucesso nos estudos, é continuar fazendo listas de
vocabulário das palavras e verbos, principalmente os irregulares, que aparecem em forma de lista
em inúmeras fontes de pesquisa.
Isso te ajudará nas questões futuras e torna você, como eu disse antes, um candidato mais
bem preparado e confiante para realizar uma excelente prova.
É importante lembrar também do nosso Fórum de dúvidas, exclusivo do Estratégia Militares.
Será minha forma de responder você, esclarecer o que mais você precise saber para que os
conteúdos fiquem ainda mais claros em seus estudos, certo?

E, caso queira, acesse minhas redes sociais para aprender mais palavras e contar com dicas
importantes, que colaboram diretamente com seus estudos dia após dia.

@teacherandreabelo

Teacher Andrea Belo

Teacher Andrea Belo

Aula 07 - Linking words/Question words 54


www.estrategiamilitar.com.br 61
Teacher Andrea Belo
Aula 07: Escola Naval 2021

14. Referências bibliográficas

ACKLAM, Richard; CRACE, Araminta. Total English: Pre intermediate. 1 ed. Grã-Bretanha: Longman
do Brasil, 2005.

BLATT, Franz. Précis de Syntaxe Latine. Lyon, Paris: IAC, 1952.

BENTES, Anna Christina e Mussalim, Fernanda (org.). Introdução À Linguística, Domínios E Fronteiras.
6ª edição. Editora Cortez. São Paulo. 2006.

BOURGOGNE, Cleuza Vilas Boas & Silva Lilian Santos. Interação & Transformação. SP: Ed. Brasil,
1999.

BOWKER, L. & PEARSON, J. Working with Specialized Language. Routledge. Capítulos 1, 2, 8,10 e 11,
2002.

BUSSE, Winfried Busse & Mário Vilela. Gramática de Valências. Coimbra: Almedina,1986.

CARVALHO, José Herculano de. Estudos Lingüísticos. v. 2. Coimbra: Atlântida, 1969.

CHIMIM, Renata; Ilearn English student book, 4 / Renata Chimim, Viviane Kirmeliene; [obra coletiva
organizada e desenvolvida pela editora]. 1ª. ed. São Paulo: Pearson Education do Brasil, 2013.

CORBEIL, J.-Cl., ARCHAMBAULT, A. Michaelis Tech dicionário temático visual inglês -português-
francês-espanhol. Tradução: Marisa Soares de Andrade. São Paulo: Melhoramentos, 1997.

CUNHA, Celso. Nova Gramática do Português Contemporâneo. Rio de Janeiro: Nova fronteira,
terceira edição, 2001.

CUNNINGHAM, Gillie; REDSTON, Chris. Face2Face: Upper Intermediate. 1 ed. Brazil: Cambridge,
2001.

DANIELS, H. Vygotsky and pedagogy. Educational Tasks Pedagogical Communication for Teachers.
Routledge, 3 rd edition, 2001.

Aula 07 - Linking words/Question words 55


www.estrategiamilitar.com.br 61
Teacher Andrea Belo
Aula 07: Escola Naval 2021

FAIRCLOUGH, N. Discourse and social change. Polity Press, 1992.

GENTZLER, E. Contemporary Translation Theory. Routledge, 1993.

HOUAISS, A., CARDIM, I. Dicionário universitário Webster inglês-português / português-inglês. São


Paulo: Record, 1998.

HYLAND, K. Genre and second language writing – For teachers and pedagogical professionals in
general, 2003.

HUTCHINSON, Tom & WATERS, Alan. English for Specific Purposes. Cambridge: Cambridge University
Press, 1996.

LAFACE, A. O dicionário e o contexto escolar. Revista Brasileira de Linguística, Unesp/Assis, v.9, 1982,
p. 165-179.

LOBATO, M.P. Lúcia. Teorias Linguísticas e ensino do português como língua materna. Brasília: UNB,
1999.

MICHAELIS Tech Dicionário Temático Visual: línguas estrangeiras – Pesquisa e tradução Marisa
Soares de Andrade. – São Paulo: Companhia Melhoramentos, 1997..

SILVA, João Antenor de C., GARRIDO, Maria Lina, BARRETO, Tânia Pedrosa. Inglês Instrumental:
Leitura e Compreensão de Textos. Salvador: Centro Editorial e Didático, UFBA. 1994.

SILVA, T.; MATSUDA, P. Second language writing research: perspectives on the process of knowledge
construction, 2001.

SILVEIRA BUENO, F. A formação histórica da língua portuguesa. 3. ed. São Paulo: Saraiva , 1967.

SIMPSON, J., WEINER, E. (eds.) Oxford English dictionary on CD-ROM. 2ed. Oxford : Oxford
University Press, 1999.

PASCHOALIN, Maria Aparecida; SPADOTO, Neuza Terezinha. Gramática, Teoria e Exercícios. Editora
FDT. São Paulo. 1996.

Aula 07 - Linking words/Question words 56


www.estrategiamilitar.com.br 61
Teacher Andrea Belo
Aula 07: Escola Naval 2021

RIBEIRO, Manuel P. Nova gramática aplicada da língua portuguesa. Rio de Janeiro: Metáfora editora,
14ª edição, 2002.

TUCK, Michael. Oxford Dictionary of Computing for Learners of English. Oxford: Oxford University
Press, 1996.

CETEMFolha/NILC: Corpus de Extractos de Textos Electrónicos. Banco de dados. Disponível em:


http://acdc.linguateca.pt/cetenfolha>.Último acesso (vários acessos) em: 04.05.2019.

COSTA, Daiane. As origens da língua inglesa. Disponível em:


http://englishmaze.wordpress.com/2011/01/25/as-origens-da-lingua-inglesa/Acesso em: 2/5/
2019.

VENTURINI, Laercio. Origem e desenvolvimento da língua inglesa. Disponível em:


<http://www.startenglish.com.br/index.php?option=com_content&task=view&id=100& Itemid=97
>. Acesso em: 22 mai. 2012.

OXFORD photo dictionary. Oxford: Oxford University Press, 1992

Referências complementares (websites):

http://www.sk.com.br/sk-perf.html - Acesso em 19 de março de 2019.

https://www.inglesnapontadalingua.com.br/2013/03/o-que-sao-falsos-cognatos.html - Acesso em
19 de março de 2019.

https://www.infoescola.com/ingles/

https://www.solinguainglesa.com.br/conteudo/indice.php

https://www.inglesnapontadalingua.com.br

https://www.englishexperts.com.br/

Aula 07 - Linking words/Question words 57


www.estrategiamilitar.com.br 61
Teacher Andrea Belo
Aula 07: Escola Naval 2021

15. Traduções

Aula 07 - Linking words/Question words 58


www.estrategiamilitar.com.br 61
Teacher Andrea Belo
Aula 07: Escola Naval 2021

Uma mão-robô ajuda amputados a “sentir” novamente

Seus olhos podem estar lendo isso, mas você provavelmente usou suas
mãos para chegar aqui. Todos os dias, seus dedos, pulsos e antebraços
conduzem a sinfonia dos movimentos motores, que permitem clicar no
mouse, digitar no teclado ou deslizar a tela. Mas e se você perdesse sua
mão? Que tipo de máquina poderia restaurar suas habilidades de
agarrar, clicar, tocar ou tocar?

Em um novo estudo, um grupo de pesquisadores da Universidade de


Utah reprojetou um sistema protético para permitir que um homem cuja
mão foi amputada sinta uma ampla gama de sensações, ajudando-o a
captar fisicamente uma variedade de objetos delicados, de um copo de
vinho para uma única uva. Embora outras próteses tenham permitido
que os usuários recebessem feedback sensorial, o sistema modificado
aumenta a detecção sutil de sensação na mão.

Muitas pessoas pensam no toque como um sentido, mas o toque é


realmente composto por muitos sentidos diferentes ”, diz Jacob George,
pesquisador de pós-doutorado que liderou o estudo. "Existem sensores
em sua mão para dor, temperatura, vibração, pressão - e, portanto, o
que estamos fazendo é identificar cada um desses pequenos sensores".

Aula 07 - Linking words/Question words 59


www.estrategiamilitar.com.br 61
Teacher Andrea Belo
Aula 07: Escola Naval 2021

O aumento da função cerebral não é mais apenas um tema de ficção científica. _____
(I) _____ avanços nas ciências neurais, tornou-se uma questão de realidade que uma
pessoa pode considerar em algum momento da vida, por exemplo, como tratamento de
uma doença neurodegenerativa. Atualmente, várias abordagens oferecem
aprimoramentos para as funções cerebrais sensoriais, motoras e cognitivas, _____ (II)
_____ para humor e emoções. Tais aprimoramentos podem ser alcançados
farmacologicamente, usando implantes cerebrais para gravações, estimulação e
administração de medicamentos, _____ (III) _____ empregando interfaces cérebro -
máquina, ou mesmo pela ablação de certas áreas do cérebro.

Neste tópico de pesquisa, congratulamo-nos com artigos que avaliam criticamente os


métodos existentes de aumento do cérebro, introduzindo novas abordagens e
sondando partes específicas dos circuitos cerebrais e mecanismos neuronais específicos
como candidatos a um aprimoramento. Congratulamo-nos com cientistas de diferentes
áreas: da neurociência de microcircuitos à neurociência de sistemas de redes de larga
escala e engenharia neural. O trabalho pode ser experimental ou computacional.
Revisões e trabalhos sobre questões filosóficas e éticas são _____ (IV) _____ bem -
vindos.

Embora o escopo de possíveis tópicos relevantes seja amplo, os autores são encorajados
a indicar claramente como seus estudos abordam o tema anunciado de aumento do
cérebro.

Aula 07 - Linking words/Question words 60


Notawww.estrategiamilitar.com.br
importante: Todas as contribuições para este Tópico de Pesquisa devem estar 61
dentro do escopo da seção e revista a que são enviadas, conforme definido em suas
Teacher Andrea Belo
Aula 07: Escola Naval 2021

Quando estudei Design de Interiores, participei de várias aulas de desenho. Fiquei


aterrorizada quando o instrutor usou meus desenhos como exemplo na frente da classe.
A maioria dos meus desenhos estava marcada com sujeira e oleosidade por causa das
minhas mãos suadas. Hoje em dia, qualquer trabalho de arte que faço é digital, e estou
feliz por não precisar mais me preocupar com as palmas das mãos suadas marcando o
papel. ” (Julia)

“No futuro, arte digital e arte real nunca serão diferentes. Eu uso os dois, e a arte digital
está completamente relacionada ao verdadeiro talento do desenho, porque eles estão
conectados e adotam o estilo artístico. Este método digital não tem limites. A pintura é
limitada, mas podemos corrigir nossos erros e leva muito mais tempo. Se você é bom
em criar arte digital, pode usar sua habilidade no photoshop, por exemplo, para fazer
com que sua arte pareça uma pintura real ou qualquer outro tipo de arte tradicional.
Além disso, na arte digital, você pode clicar facilmente em um botão para cancelá-lo.
Não é uma opção na arte tradicional, pois os erros existem para todos verem. ” (Yuuki)

Aula 07 - Linking words/Question words 61


www.estrategiamilitar.com.br 61
Aula 09 –
Relative Clauses
Quantifiers

Escola Naval 2021

Teacher Andrea Belo


Teacher Andrea Belo
Aula 09: Escola Naval 2021

Sumário

Introdução ........................................................................................................................ 3

Relative Clauses - Introduction .......................................................................................... 4

Relative pronoun THAT................................................................................................................................ 7

Relative pronoun WHICH ..........................................................................................................................10

Relative pronouns WHO and WHOM ......................................................................................................11

Relative pronoun WHOSE .........................................................................................................................15

Relative pronoun WHERE ..........................................................................................................................16

Relative pronoun WHEN ...........................................................................................................................17

Relative pronoun WHY ..............................................................................................................................19

Quantifiers ...................................................................................................................... 20

Diferentes exercícios na área da Carreira Militar ............................................................. 23

Gabarito.......................................................................................................................... 37

Exercícios respondidos com comentários ........................................................................ 38

Considerações finais........................................................................................................ 59

14. Referências bibliográficas .......................................................................................... 60

15. Traduções.................................................................................................................. 63

Aula 09 – Relative Clauses/Quantifiers 2


www.estrategiamilitar.com.br 68
Teacher Andrea Belo
Aula 09: Escola Naval 2021

Introdução

Desta vez, vamos à nossa aula sobre Relative Clauses (Orações Relativas), também presentes
nos textos das provas de Carreiras Militares e que, têm suas particularidades, as quais vamos estudar
a teoria e praticar com muitos exercícios no decorrer do material.
O que são Relatives Clauses? Relative Clauses são sentenças ou frases que oferecem uma
informação sobre algo ou alguém. A palavra “relative” deve ser entendida como “ligado a“ ou
“relacionado a” e, para facilitar, devemos lembrar dos relative Pronouns, que estudamos na aula
destinada aos pronomes e suas classificações.
As orações relativas são construídas similares às adverbiais, que estudamos anteriormente.
Então, possuem uma oração independente e outra dependente.
Assim, nos textos da maioria das provas, não seria diferente – aparecem, nos textos, as
orações dos mais variados tipos, com a exploração do vocabulário, dos termos de diferentes
classificações gramaticais, do sentido que cada frase tem, inseridas nos vários assuntos trabalhados.
Diante das provas, faz-se necessário conhecer muitos tipos de frases para sentir-se mais
seguro na hora de analisar cada texto e responder às questões.
O estudo das orações, como fazemos dia a dia em nosso material, abrange tópicos gramaticais
presentes nas entrelinhas do que se pergunta na sua prova.
E isso vai ampliar os conhecimentos que você já possui, pois a língua escrita possui prestígio
maior do que a língua falada, visto que é considerada um referencial de uso correto porque segue
regras e vai aparecer na sua prova para interpretação e busca de respostas do que é questionado.
Vamos estudar também os Quantifiers – os determinantes, são expressões usadas para
indicar informações a respeito da quantidade de algo. Veremos muitos tipos deles.
Assim, meu objetivo, como sua professora e orientadora, é dar a você condições para que
você aprenda os tópicos da sua prova, assim com as orações relativas e quantifiers de hoje e que
você reconheça as variedades da língua em suas diversas situações de uso.
Vamos então estudar as particularidades de todas as orações relativas, praticar exercícios,
tanto durante a teoria como também na lista de questões, treinando tudo o que aprendeu, em
junção ao aprendizado de cada aula.
Vamos lá, busque ser o melhor candidato e conte comigo!

Aula 09 – Relative Clauses/Quantifiers 3


www.estrategiamilitar.com.br 68
Teacher Andrea Belo
Aula 09: Escola Naval 2021

Relative Clauses - Introduction

As Relative Clauses são orações que exercem a função de adjetivos. Por esse motivo, em
algumas gramáticas, também são chamadas de Adjectives Clauses. E são, em Português, orações
subordinadas.
Elas são empregadas com o intuito de informar sobre uma pessoa ou determinada coisa e
assim, funcionam como complementos de um substantivo ou um pronome.
Essas orações - relative clauses - que vamos estudar cada particularidade, para compreender
bem, são construídas com a adição de partículas que têm o objetivo de adicionar informações à
oração principal.
Essas partículas são os pronomes relativos.
Em Inglês, os pronomes que já estudamos em aulas anteriores e que são utilizados nas
orações relativas são: who, whom, which, whose e that. Outros pronomes relativos, menos usados
mas que estudaremos também são when, where e why.
Eles são responsáveis por unir uma frase à outra bem como fornecer informações adicionais
à frase anterior.
Cada pronome relativo é utilizado para complementar uma oração de forma diferente.
Vejamos, em suma, como é a classificação dessas orações, para depois estudar cada uma delas
separadamente com exemplos.
Os pronomes relativos podem exercer diferentes funções na frase (sujeito, objeto ou posse)
e, determinam qual será a relative clause.

Defining relative clauses

Non-Defining relative clauses

Aula 09 – Relative Clauses/Quantifiers 4


www.estrategiamilitar.com.br 68
Teacher Andrea Belo
Aula 09: Escola Naval 2021

As Relative Clauses podem ser Defining Relative Clauses ou Non-defining Relative Clauses,
veja:

Defining relative clauses (orações restritivas): Essas orações são usadas para definir sobre

quem ou sobre o que estamos falando. Não exigem o uso de vírgulas e os pronomes relativos

utilizados nelas são: who, whom, which, whose, where, when, why e that, como estudaremos

adiante, um a um.

Non-defining relative clauses (orações explicativas): Um pouco diferente das defining relative

clauses, por sua vez, as non-defining relative clauses não fornecem informações essenciais

sobre o que antecede a frase. Aqui, as informações adicionadas geralmente aparecem entre

vírgulas. E os pronomes mais utilizados são: who, whom, whose e which.

As defining clauses, independente do pronome relativo a ser usado, aparecem de forma mais
direta nas frases, ou seja, como eu afirmei anteriormente, sem estar entre parênteses ou entre
vírgulas. Elas destacam pessoas ou coisas à qual se referem.

As non-defining clauses são orações que trazem informações extras sobre o sujeito ou objeto
das frases. Geralmente, são colocadas entre vírgulas ou parênteses.

Se tentarmos retirá-las da oração principal, ela dá a impressão de que está incompleta,


apesar de manter o sentido.

Aula 09 – Relative Clauses/Quantifiers 5


www.estrategiamilitar.com.br 68
Teacher Andrea Belo
Aula 09: Escola Naval 2021

Resumidamente, eis as características principais das Defining e Non-defining clauses:

Veremos cada pronome para saber qual dos dois tipos podem ser usadas com cada um deles.

Aula 09 – Relative Clauses/Quantifiers 6


www.estrategiamilitar.com.br 68
Teacher Andrea Belo
Aula 09: Escola Naval 2021

Relative pronoun THAT

O pronome relativo that é o que mais parece em frases de forma geral, em diálogos informais,
em filmes, em séries, em textos variados, entre outros, já que pode ser usado quando se trata de
pessoas ou de objetos. Atenção: é usado apenas em Defining Relative Clauses.
Vou introduzir oferecendo dois exemplos para você compreender melhor, desde o início da
explicação. É como se fôssemos dizer: “Eu tenho um amigo que eu admiro muito.” ou “Eu tenho um
cachorro que sabe nadar”, em que se fala de coisas/animais referentes à pessoa ou se fala da própria
pessoa, usando o mesmo pronome: that.
As frases acima seriam: “I have a friend that I admire a lot.” E “I have a dog that knows how to swim”.
No caso de THAT, foi escolhido para ser o primeiro da lista de todos que explicarei nessa aula,
justamente porque serve para todas as orações relativas.
Eu costumo chamar esse pronome de pronome carta coringa, que pode ser utilizado em qualquer
oração, quando se há alguma dúvida em relação a qual usar para desenvolver as frases desejadas.
É importante observar que, se vier um sujeito após o pronome that, ele pode ser omitido sem
prejudicar o sentido da frase. E às vezes pergunta-se isso em questões da sua prova.
Vejamos outros exemplos:

These are the flowers that I bought for you.


(Esses são as flores que eu comprei para você.)

Aqui, por causa do sujeito “I”, após o pronome that, ele poderia ser omitido:
These are the flowers I bought for you.

These are the boys that I was talking about.


(Esses são os garotos que eu estava falando.)

Aqui também, por causa do sujeito “I”, após o pronome that, ele poderia ser omitido:
These are the boys I was talking about

Aula 09 – Relative Clauses/Quantifiers 7


www.estrategiamilitar.com.br 68
Teacher Andrea Belo
Aula 09: Escola Naval 2021

He spoke so well that everybody was surprised.


(Ele falou tão bem que todos ficaram surpresos.)

Aqui também, por causa do sujeito “everybody”, após that, ele poderia ser omitido:
(He spoke so well everybody was surprised.)

Agora vejamos exemplos em que o pronome that não pode ser omitido, já que, não há sujeitos após
os mesmos e sim verbos, que “pedem” que o pronome esteja presente:

The cat that was hidden is mine.


(O gato que estava escondido é meu).

Aqui, por causa do verbo “was hidden” e não um sujeito, após that, ele não poderia ser
omitido: The cat that was hidden is mine.
Veja outro exemplo:

The girl that won the competition is the best.


(A garota
Aula 09 – Relative Clauses/Quantifiers que ganhou a competição é a melhor.) 8
www.estrategiamilitar.com.br 68
Teacher Andrea Belo
Aula 09: Escola Naval 2021

Aqui, também por causa do verbo “won” (Past Simple) e não um sujeito, após that, ele não poderia
ser omitido: The girl that won the game is the best.
Vejamos o pronome that em um dos textos da Newsweek, fonte usada em algumas provas das
Carreiras Militares, para exemplificar:

O texto está falando de Christoph Oswald e suas aventuras com as mulheres e, no segundo
parágrafo, foi usado o pronome that, se referindo a um programa de celular que promete algo: “...
Symbian Dater, a program that promises to turn the cell phone into …”.
Assim como estudamos, o that pode se referir a pessoas ou coisas e oferecer informações sobre
elas. No caso, foi informado que é um programa que promete algo.
Vamos ao estudo do pronome which.

Aula 09 – Relative Clauses/Quantifiers 9


www.estrategiamilitar.com.br 68
Teacher Andrea Belo
Aula 09: Escola Naval 2021

Relative pronoun WHICH

O pronome relativo which é usado quando se trata de coisas/objetos e animais e nunca usado

para pessoas. Pode ser também traduzido como que.

Diferente de that, é usado em defining ou non-defining relative clauses.

No caso de WHICH, é válido observar que, muitas vezes, pergunta-se em questões da sua prova se

which pode ser substituído por that ou vice-versa e, devemos analisar cuidadosamente se tais

pronomes, nessas frases, referem-se ao sujeito ou ao objeto.

Como uma observação, lembre-se de que, se vier um sujeito após o pronome which, assim como

acontece com o pronome that, ele também pode ser omitido sem prejudicar o sentido da frase, que

pode ser uma provável pergunta da sua prova.

Vejamos exemplos:

Orca whale, which is big, is very dangerous.


(A baleia orca, que é grande, é muito perigosa.)

Aqui, por causa do verbo “is”, após o pronome which, ele não pode ser omitido.

Aula 09 – Relative Clauses/Quantifiers 10


www.estrategiamilitar.com.br 68
Teacher Andrea Belo
Aula 09: Escola Naval 2021

Outro exemplo:

This is the document which you need to sign.


(Este é o documento que você precisa assinar.)

Aqui também, por causa do sujeito “you”, após o pronome which, ele pode ser omitido:
This is the document you need to sign.

Vamos aos pronomes who e whom.

Relative pronouns WHO and WHOM


Os pronomes relativos who e whom se referem apenas à pessoas e nunca objetos ou animais.

A diferença entre who e whom é bastante sutil e deve-se prestar atenção à explicação e aos
exemplos para não gerar confusão na hora de sua prova.

Observe: enquanto who se refere à uma pessoa que executa a ação da oração, whom é, por sua vez,
utilizado quando se trata de alguém que recebe uma ação. Exemplos:

Aula 09 – Relative Clauses/Quantifiers 11


www.estrategiamilitar.com.br 68
Teacher Andrea Belo
Aula 09: Escola Naval 2021

The charming guy who works with me has asked me out.


(O homem charmoso que trabalha comigo me chamou para sair.)

Aqui, você pode perceber que o pronome who faz referência ao homem charmoso, ou seja,
the charming guy, que foi quem chamou a pessoa para sair, executou a ação. Outro exemplo:

George Lee, who was the school’s president, discoursed yesterday.


(George Lee, que era o presidente da escola, discursou ontem.)

No exemplo anterior, você pode perceber mais uma vez que o pronome who faz
referência a George Lee, ou seja, o presidente da escola, que foi quem discursou, executou também
a ação.

Você percebeu que, usei exemplos com frases entre vírgulas ou não. Ambos casos se usa o
pronome relativo who.

Agora, vejamos exemplos com o pronome relativo whom, para que a diferença entre o uso de who
e whom fique clara a você.

Aula 09 – Relative Clauses/Quantifiers 12


www.estrategiamilitar.com.br 68
Teacher Andrea Belo
Aula 09: Escola Naval 2021

I don’t know the name of the boy whom she is going out with.
(Eu não sei o nome do garoto quem/ com quem ela está saindo).

Aqui, você pode perceber que o pronome whom faz referência a quem recebeu a ação: ela.
O sujeito da oração é “I” e o garoto de que trata a frase é aquele com quem ela está saindo, como
um adjetivo para a pessoa com quem alguém está saindo. Veja outro exemplo:

The boy to whom you gave the present is my brother.


(O garoto para quem você deu o presente é meu irmão).

Aqui também, você pode perceber que o pronome whom faz referência a quem recebeu a
ação: meu irmão.

Vejamos o pronome who em um dos textos do New York Times, fonte também usada para preparar
provas, para exemplificar:

Aula 09 – Relative Clauses/Quantifiers 13


www.estrategiamilitar.com.br 68
Teacher Andrea Belo
Aula 09: Escola Naval 2021

O texto está falando da Warner Bros Pictures, de cinema, de filmes e, no segundo parágrafo, foi
usado o pronome who, duas vezes.
Conforme estudamos, o pronome who, sempre se refere a pessoas, certo?
Na primeira vez que who aparece, ele está se referindo a Bryan Singer:
“... The last director – Bryan Singer, who directed X-Men and its sequel, was named…”
corretamente como estudamos.
Na segunda vez que who está na frase, está também se referindo a pessoas – desta vez, a Joseph
McGinty Nichol:
“... replace Joseph McGinty Nichol, known as McG, who left the project after refusing to board…”
explicando algo sobre Joseph, como estudamos.
Agora, vamos ao pronome whose. Come on!

Aula 09 – Relative Clauses/Quantifiers 14


www.estrategiamilitar.com.br 68
Teacher Andrea Belo
Aula 09: Escola Naval 2021

Relative pronoun WHOSE

O pronome relativo whose é usado para indicar posse, para indicar que algo pertence a alguém.
A tradução que melhor representa o whose seria o cujo/cuja em Português.
Vejamos exemplos:

That bag, whose owner in unknown, is in the lost and found department.
(Aquela mala, cujo dono é desconhecido, está no departamento de achados e perdidos.)

That teacher, whose class is attractive, is preparing a speech for tomorrow.


(Aquela professora, cuja aula é atrativa, está preparando um discurso para amanhã.)

Vamos ao pronome relativo where.

Aula 09 – Relative Clauses/Quantifiers 15


www.estrategiamilitar.com.br 68
Teacher Andrea Belo
Aula 09: Escola Naval 2021

Relative pronoun WHERE


O pronome relativo where é usado para indicar o lugar que a frase está fazendo referência, o local
do que se trata a informação principal da oração com esse pronome relativo.
É um dos pronomes relativos mais simples, pois tem o mesmo significado de quando é pronome
interrogativo: onde e se encaixa de forma prática nas frases.
Vejamos exemplos:

The house where I live is big, beautiful and comfortable.


(A casa onde/em que eu moro é grande, bonita e confortável.)

The college where she studies is modern.


(A faculdade onde/em que ela estuda é moderna.)

Vamos ao pronome relativo when.

Aula 09 – Relative Clauses/Quantifiers 16


www.estrategiamilitar.com.br 68
Teacher Andrea Belo
Aula 09: Escola Naval 2021

Relative pronoun WHEN


O pronome relativo when é usado para indicar tempo: o dia, a semana, o mês, enfim, o período do
que se trata a informação principal da oração com esse pronome relativo.
É um pronome relativo simples também, pois, assim como o where, tem o mesmo significado como
pronome interrogativo: quando.
Vejamos exemplos:

That was the day when I started working in this company.


(Aquele foi o dia quando/em que eu comecei a trabalhar nessa empresa.)

January is the month when I graduate.


(Janeiro é o mês quando/em que eu me formo.)

Aula 09 – Relative Clauses/Quantifiers 17


www.estrategiamilitar.com.br 68
Teacher Andrea Belo
Aula 09: Escola Naval 2021

Vejamos um texto, que usaremos em nossa lista de exercícios adiante, do jornal Thee
Economist, explorado em muitas provas, com o pronome when para ilustrar e exemplificar:

Percebemos que, nas três vezes em que o pronome when aparece no texto, refere-se a um
período de tempo, contextualiza o moment, veja: “... when Spain was a republic” (quando a
Espanha era república) , “... when Spain opened up to tourists” (quando a Espanha abriu-se
aos turistas) e “... when families of eight or tem were common” (quando famílias com oito ou
dez filhos era comum). Todos períodos do que se trata o assunto do texto.
Vamos ao último pronome relativo: why.

Aula 09 – Relative Clauses/Quantifiers 18


www.estrategiamilitar.com.br 68
Teacher Andrea Belo
Aula 09: Escola Naval 2021

Relative pronoun WHY


O pronome relativo why é usado para indicar a razão, o motivo do que se trata a informação principal
da oração.
É um pronome relativo que também significa porque, como na hora de fazer perguntas, em que está
na função de pronome interrogativo. Podemos também traduzir como “por causa de”.
Vejamos exemplos:

There are three different ways to go. That’s the reason why I got confused.
(Há três diferentes caminhos para ir lá. Essa é a razão por quê/pela qual eu fiquei confuso.)

I don’t know the reason why she is angry.


(Eu não sei a razão por quê/pela qual ela está nervosa.)

Vamos ao estudo dos quantifiers. Let’s go!

Aula 09 – Relative Clauses/Quantifiers 19


www.estrategiamilitar.com.br 68
Teacher Andrea Belo
Aula 09: Escola Naval 2021

Quantifiers
Os determinantes – quantifiers, são expressões usadas para indicar informações a respeito da

quantidade de alguma coisa. Para estudar e entender bem os tipos de quantifiers, é importante

lembrar de quando estudamos os substantivos.

Você se lembra que havia substantivos classificados com contáveis – countable – aqueles que

podemos contar, não necessitando de nenhuma unidade de medida, permitindo a forma singular e

plural. E os substantivos incontáveis – uncountable – que não podemos contar, tornando-se

necessária uma unidade de medida. Esses substantivos só apresentam a forma singular.

Agora vejamos os quantifiers que mais aparecem na sua prova:

“Much”, “Many” e “A Lot of”

As três palavras indicam a mesma coisa: uma grande quantidade. No entanto, o uso de cada um

depende exatamente do tipo de substantivo que há na frase:

“Much” é usado nos casos de substantivos não-contáveis.


Example: How much sugar do you need? A few. Quanto de açúcar você precisa? Um pouco.

“Many” é usado nos casos de substantives contáveis.


Example: How many students are there? Six. Quantos estudantes estão lá? Seis.

“A lot of” é usado em ambos os casos.


Example: I have a lot of things to do. Eu tenho muitas coisas pra fazer.

Aula 09 – Relative Clauses/Quantifiers 20


www.estrategiamilitar.com.br 68
Teacher Andrea Belo
Aula 09: Escola Naval 2021

“Too” e “So”

Esses dois pronomes são usados para intensificar algo. A diferença é que “too” dá a impressão de

exagero, às vezes com uma ideia negativa, enquanto “so” indica uma grande quantidade, sempre

com ideia positiva.

Examples: There is too much traffic jam in SP. = Há congestionamento demais em SP.
I love you so much. = Eu te amo muito.

“Little”, “Few” e “Enough”

Os quantifiers “little” e “few” significam ambos uma pequena quantidade de alguma coisa. A

diferença entre os dois é que enquanto “little” se refere aos substantivos não-contáveis, “few” se

refere aos contáveis.

Examples: There are a few people in the classroom. = Há poucas pessoas na sala de aula.
Add a little sugar, please. = Adicione um pouco de açúcar, por favor.

Por sua vez, “enough” se refere a algo suficiente, podendo ser usado tanto para substantivos

contáveis como incontáveis.

Examples: There isn’t enough food. = Essa comida não é suficiente/ não é o bastante.
I drove fast enough to get there in time.
Eu dirigi rápido o suficiente/ o bastante para chegar lá na hora.

Aula 09 – Relative Clauses/Quantifiers 21


www.estrategiamilitar.com.br 68
Teacher Andrea Belo
Aula 09: Escola Naval 2021

Então, “enough” significa suficiente ou bastante, certo? Mas não basta saber a tradução.

preciso saber como usar enough em Inglês de forma correta. Afinal, enough pode ser usado em

várias classes gramaticais em inglês.

Mas, o que importa mesmo é aprender como identificar enough nos textos da sua prova. Em

geral, podemos dizer que o uso mais frequente de enough é como vimos acima, antes ou depois de

substantivos.

Ainda podemos usar enough em algumas expressões fixas, veja:

✓ Enough is enough! I don’t want to argue anymore. (Já chega! Não quero mais discutir.)
✓ I’m just annoyed with her because she’s behaved so badly. Fair enough! ( Eu só estou
chateada com ela porque ela se comportou muito mal. Nada mais justo!)
✓ I can’t eat anymore. I’ve had enough. (Não aguento mais comer. Estou satisfeito/cheio.)
✓ I’ve had enough of your excuses (Já estou cheio de suas desculpas/cansado de suas
desculpas.)
✓ The doctors tried everything they could, but it was not enough to save him. (Os médicos
tentaram tudo que podiam, mas não foi suficiente para salvá-lo.)
✓ You’ve been practicing the flute all afternoon. Enough! (Você tocou flauta a tarde inteira.
Basta! Chega!)

Vamos aos exercícios para praticar os pronomes relativos inseridos nos textos da sua prova
de anos anteriores.
Let’s go!

Aula 09 – Relative Clauses/Quantifiers 22


www.estrategiamilitar.com.br 68
Teacher Andrea Belo
Aula 09: Escola Naval 2021

Diferentes exercícios na área da Carreira Militar


Vamos resolver exercícios de diferentes Carreiras Militares e, entre eles, claro, questões da
Escola Naval.
QUESTÃO AFA/2014 - Texto para responder a questão 01

QUESTÃO 01 (AFA/2014)

Aula 09 – Relative Clauses/Quantifiers 23


www.estrategiamilitar.com.br 68
Teacher Andrea Belo
Aula 09: Escola Naval 2021

QUESTÃO 02 (Colégio Naval/2018) – Texto para responder a questão 02

Aula 09 – Relative Clauses/Quantifiers 24


www.estrategiamilitar.com.br 68
Teacher Andrea Belo
Aula 09: Escola Naval 2021

QUESTÃO 02 (CN/2018) -

QUESTÃO 03
(Escola Naval/2018)

Aula 09 – Relative Clauses/Quantifiers 25


www.estrategiamilitar.com.br 68
Teacher Andrea Belo
Aula 09: Escola Naval 2021

QUESTÃO 04 (EPCAR/2015)

QUESTÃO 04 (EPCAR/2015)

Aula 09 – Relative Clauses/Quantifiers 26


www.estrategiamilitar.com.br 68
Teacher Andrea Belo
Aula 09: Escola Naval 2021

QUESTÃO 05 (EAM/2020 inédita)

A solicitor engaged by former members of the Supreme Horse Racing Club said
on Thursday evening he was “very optimistic” the 29 horses it has hitherto
owned, including the top-class chaser Kemboy, will be able to race this season.
The Thurles-based Patrick Kennedy was speaking after the ruling body of Irish
horse racing in effect kicked SHRC out of the sport by declaring it was no longer
“permitted to be owners or part-owners of racehorses”.
The announcement from Horse Racing Ireland represents the moment when
its patience expired over the club’s repeated failure to provide answers to its
questions.
Following complaints from club members about alleged irregularities in the
way SHRC was run, HRI got involved this summer and its concerns have not
been allayed. (The Guardian/November 2019)

Questão 05 – In the third paragraph “… Following complaints from club members about
alleged irregularities in the way SHRC was run, …” The verb “run” refers to:
a) in a hurry irregularity
b) in a hurry action
c) something urgent
d) a kind of function
e) the behave of something

Aula 09 – Relative Clauses/Quantifiers 27


www.estrategiamilitar.com.br 68
Teacher Andrea Belo
Aula 09: Escola Naval 2021

QUESTÃO 06 (EEAR/2017) - Texto para questões 06 e 07

April

There was a nice little girl which name was April. One day, she asked her parents why she was
called April. They answered that it was because she was born in April. The little girl liked her
name and the month April too. Her parents made her a party, all her friends celebrated with
her, and she received a lot of presents. One day her mother became pregnant and April had a
little brother. Her brother was born in February and everybody came and suggested names for
the new baby. April didn’t understand what the problem was. If the baby was born in February,
the correct name should be February.

Fonte: adapted from Thoughts and Notions.

QUESTÃO 06 – According to the text all the alternatives are correct, except:
a) April liked the fourth month of the year.
b) Her father and mother made her a party.
c) April was her mother’s favorite month.
d) April received a lot of gifts.

QUESTÃO 07 – The underlined word in the text is


a) an adjective
b) a pronoun
c) a noun
d) a verb

Aula 09 – Relative Clauses/Quantifiers 28


www.estrategiamilitar.com.br 68
Teacher Andrea Belo
Aula 09: Escola Naval 2021

QUESTÕES ITA/2014

Texto para responder as questões 08 a 10

Questão 08 – A Grant Study, pesquisa realizada pela Universidade de Harvard,

A ( ) teve por objetivo investigar o comportamento de pessoas idosas e felizes.

B ( ) possibilitou o levantamento gigantesco de dados sobre pesquisadores de Harvard.

C ( ) comprovou que John F. Kennedy foi um homem extremamente feliz.

D ( ) chama-se the 75 year old Grant Study por ser homenagem à faixa etária analisada.

E ( ) comprovou que felicidade na vida adulta está atrelada às relações afetivas ao longo da
vida.

Aula 09 – Relative Clauses/Quantifiers 29


www.estrategiamilitar.com.br 68
Teacher Andrea Belo
Aula 09: Escola Naval 2021

Questão 09 – Assinale a opção cuja reescrita não altera o sentido de: “Though all identities are
confidential, it was recently discovered that John F. Kennedy was a sample participant ”

A ( ) John F. Kennedy was a sample participant, although nobody knew that.

B ( ) In spite of being a sample participant, John F. Kennedy’s identity was never discovered.

C ( ) The study was confidential; thus the participation of John F. Kennedy was never
discovered.

D ( ) Besides being a confidential study, John F. Kennedy said he used t o be a participant.

E ( ) In spite of the fact that all identities are kept confidential, it was recently found out that
John F. Kennedy was a sample participant.

Questão 10 – Substituindo os adjetivos long e comprehensive, respectivamente, por easy e rich


na oração “Harvard conducted one of the longest and most comprehensive studies of human
development” (linha 1), teremos:

A ( ) the most easy - the richest

B ( ) the easiest - the most rich

C ( ) the more easy - the richer

D ( ) the easiest - the richest

E ( ) the most easy - the most rich

QUESTÕES IME/2018
Texto para responder as questões 11 a 13

Aula 09 – Relative Clauses/Quantifiers 30


www.estrategiamilitar.com.br 68
Teacher Andrea Belo
Aula 09: Escola Naval 2021

IME/2018

Aula 09 – Relative Clauses/Quantifiers 31


www.estrategiamilitar.com.br 68
Teacher Andrea Belo
Aula 09: Escola Naval 2021

Questão 11 – Choose the correct option.


(A) Hypatia did not marry so she could study; consequently, she followed the role model of the
time.
(B) Although Hypatia remained celibate, her father had been doubtful she would ever do so
because of the activities society expected her to perform.
(C) Professions were chosen according to the social class a woman occupied, centered around
family tradition and faith.
(D) Not many gaps have to be filled about Hypatia's academic life now; we know almost
everything because she continued her father's work.
(E) Some people didn't seem to understand science regardless of religion, and this might
explain why scientific concepts were seen as an obstacle.

Questão 12 – Choose a synonym for the expression to the extent that in the sentence “ (…)
Hypatia has come to symbolize this tragedy to the extent that her death has been cited as the
end of the classical world.” in lines 24 and 25.

(A) as well as
(B) in contempt of
(C) by easy stages
(D) for the record that
(E) to the degree that

Questão 13 – Choose the appropriate continuation for the sentence “By all accounts, Hypatia
was an extraordinary woman ___________” in line 26.
(A) since she was assigned a profession different from her father's.
(B) not only for her time, but for any time.
(C) as she lived for the Gods.
(D) devoted to teaching, which was a common activity for women at that time.
(E) of slight intellectual power.

Aula 09 – Relative Clauses/Quantifiers 32


www.estrategiamilitar.com.br 68
Teacher Andrea Belo
Aula 09: Escola Naval 2021

QUESTÃO ESA/2020 inédita

Texto para responder a questão 14

QUESTÃO 14 (ESA/2020 inédita) – Read the extract from the text

“He liked to take apart televisions and put them back together again”

The pronoun “them” refers to:

a) equipment in general

b) Apple equipment

c) Steve Jobs’ hobbies

d) televisions

e) things to take apart

Aula 09 – Relative Clauses/Quantifiers 33


www.estrategiamilitar.com.br 68
Teacher Andrea Belo
Aula 09: Escola Naval 2021

QUESTÃO 15 (EsPCEx/2019) – Texto para questão 15

QUESTÃO 15 (EsPCEx/2019) – Texto para questão 15

According to the text, choose the correct statement.

[A] A new sustainable material has already been chosen.

[B] Lego has already reduced the size of their packaging.

[C] Lego is planning to reduce the size of their products.

[D] Lego’s raw material will continue to be the same.

[E] They are going to hire 100 specialists in 2030.

Aula 09 – Relative Clauses/Quantifiers 34


www.estrategiamilitar.com.br 68
Teacher Andrea Belo
Aula 09: Escola Naval 2021

QUESTÃO 16
(EFOMM/2016)
Texto para
questão 16

QUESTÃO 16 (EFOMM/2016)

Aula 09 – Relative Clauses/Quantifiers 35


www.estrategiamilitar.com.br 68
Teacher Andrea Belo
Aula 09: Escola Naval 2021

QUESTÃO 16 (EFOMM/2016) - Complete the sentences with the correct verb tenses.

1. Little ____________ how inconvenient he can be. 2. Not until she received the call

_______________ relieved. 3. Not for one moment _____________ your honesty. 4. Under no

circumstances _______________ class.

( a ) does he realize/ she had breathed/would I doubt/ should you miss.

( b ) does he realize/ did she breathe/would I doubt/ should you miss

( c ) he realizes/ did she breath/ I would doubt/ you should miss

( d ) he will realize/ she breathed/ I would doubt/ should you miss

( e ) he realized/ she would breathe/would I doubt/ you should miss.

Aula 09 – Relative Clauses/Quantifiers 36


www.estrategiamilitar.com.br 68
Teacher Andrea Belo
Aula 09: Escola Naval 2021

Gabarito

1–A 2– D 3– A 4– B 5–E

6–C 7–A 8– E 9– D 10 – D

11 – E 12 – E 13 – B 14 – D 15 –B 16 – B

Aula 09 – Relative Clauses/Quantifiers 37


www.estrategiamilitar.com.br 68
Teacher Andrea Belo
Aula 09: Escola Naval 2021

Exercícios respondidos com comentários

QUESTÃO AFA/2014 - Texto para responder a questão 01

Aula 09 – Relative Clauses/Quantifiers 38


www.estrategiamilitar.com.br 68
Teacher Andrea Belo
Aula 09: Escola Naval 2021

QUESTÃO 01 (AFA/2014)

Comentários:

Alternative A is correct, pois a expressão diz que “climate changes” são causadas por “human

activities” e a alternative diz que “human activities” são responsáveis por “climate changes”, o

que é equivalente.

Alternative B is incorrect, pois a informação ficou trocada. Mudança climáticas são causadas

por atividades humanas, porém a alternative afirma que as mudanças climáticas são

responsáveis por várias atividades humanas.

Alternative C is incorrect, pois o noun “changes” recebeu o adjective “human”, o que está

trocado, pois o noun “changes” está acompanhado da ideia do adjective “climate”.

Alternative D is incorrect, pois afirma que “climate changes” são as responsáveis pelo atual

comportamento (“behavior”) humano.

Aula 09 – Relative Clauses/Quantifiers 39


www.estrategiamilitar.com.br 68
Teacher Andrea Belo
Aula 09: Escola Naval 2021

QUESTÃO 02 (Colégio Naval/2018) – Texto para responder a questão 02

Aula 09 – Relative Clauses/Quantifiers 40


www.estrategiamilitar.com.br 68
Teacher Andrea Belo
Aula 09: Escola Naval 2021

QUESTÃO 02 (CN/2018) -

Comentários:

Essa questão pergunta qual o objetivo central do texto II.

Alternative A is incorrect, pois o texto não se destina especificamente a “students”.

Alternative B is incorrect, pois o texto não foca principalmente no filme “Lord of rings”e na

visita ao seu cenário na Nova Zelândia.

Alternative C is incorrect, pois o texto foca que as pessoas viagem (“travel”) para cenários

usados em filmagens de filmes famosos, não que esses filmes sejam assistidos (“watch”).

Alternative D is correct, pois o texto foca em discutir diversos aspectos das viagens com

temáticas em filmes.

Alternative E is incorrect, pois o texto não foca nas agências de viagens (“travel companies”).

QUESTÃO 03
(Escola Naval/2018)

Aula 09 – Relative Clauses/Quantifiers 41


www.estrategiamilitar.com.br 68
Teacher Andrea Belo
Aula 09: Escola Naval 2021

QUESTÃO 03
(Escola Naval/2018)

Comentários:

Alternatives B and E are incorrect, pois usam

o verb to be no plural “are” quando o sujeito

é singular “skin” e pede, portanto, a forma “is”.

Alternatives C and D are incorrect, pois a leitura

do texto nos permite perceber que a ideia central é a de que não devemos tratar a dehydration
como um problema de curto prazo e rapidamente resolvido quando tomamos um copo de
água, visto que as consequências para a pele, a longo prazo, se ela não receber a quantidade
de água de que necessita podem ser sérias. Portanto, correct alternative is A.

Aula 09 – Relative Clauses/Quantifiers 42


www.estrategiamilitar.com.br 68
Teacher Andrea Belo
Aula 09: Escola Naval 2021

QUESTÃO 04 (EPCAR/2015)

QUESTÃO 04 (EPCAR/2015)

Comentários:
A frase analisada é “Also, in digital art you can easily click a button to cancel it”.

Percebemos que “it” se refere a “digital art”, como “digital” é adjective e “art” é noun, o núcleo
do sintagma é “art” e alternative B is correct.

Aula 09 – Relative Clauses/Quantifiers 43


www.estrategiamilitar.com.br 68
Teacher Andrea Belo
Aula 09: Escola Naval 2021

QUESTÃO 05 (EAM/2020 inédita)

A solicitor engaged by former members of the Supreme Horse Racing Club said
on Thursday evening he was “very optimistic” the 29 horses it has hitherto
owned, including the top-class chaser Kemboy, will be able to race this season.
The Thurles-based Patrick Kennedy was speaking after the ruling body of Irish
horse racing in effect kicked SHRC out of the sport by declaring it was no longer
“permitted to be owners or part-owners of racehorses”.
The announcement from Horse Racing Ireland represents the moment when
its patience expired over the club’s repeated failure to provide answers to its
questions.
Following complaints from club members about alleged irregularities in the
way SHRC was run, HRI got involved this summer and its concerns have not
been allayed. (The Guardian/November 2019)

Questão 05 – In the third paragraph “… Following complaints from club members about alleged
irregularities in the way SHRC was run, …” The verb “run” refers to:
a) in a hurry irregularity
b) in a hurry action
c) something urgent
d) a kind of function
e) the behave of something

Comentários:
Aqui temos uma questão que busca interpretar a aplicação de um vocábulo em um contexto.
Geralmente, “run” significa “correr” , porém, aqui, temos o sentido de “gerenciar”.
“... in the way SHRC was run...” = a forma que SHRC era gerenciado.
Assim, a alternativa correta é E.
As alternativas A e B estão incorretas, pois “run” não se refere aos nouns “irregularities” e
“action”.
A alternativa C está incorreta, pois, nesse caso, o sentido de “run” não tem a ver com urgência.
A alternativa D está incorreta, pois o noun function remete à ideia do “gerente”, mas não do
verb “run”, nesse caso. Importante não cair nesse detalhe!

Aula 09 – Relative Clauses/Quantifiers 44


www.estrategiamilitar.com.br 68
Teacher Andrea Belo
Aula 09: Escola Naval 2021

QUESTÃO 06 (EEAR/2017) - Texto para questões 06 e 07

April

There was a nice little girl which name was April. One day, she asked her parents why she was
called April. They answered that it was because she was born in April. The little girl liked her
name and the month April too. Her parents made her a party, all her friends celebrated with
her, and she received a lot of presents. One day her mother became pregnant and April had a
little brother. Her brother was born in February and everybody came and suggested names for
the new baby. April didn’t understand what the problem was. If the baby was born in February,
the correct name should be February. Fonte: adapted from Thoughts and Notions.

QUESTÃO 06 – According to the text all the alternatives are correct, except:

a) April liked the fourth month of the year.

b) Her father and mother made her a party.

c) April was her mother’s favorite month.

d) April received a lot of gifts.

Comentários:

Alternative A is correct, de acordo com o trecho “The little girl liked her name and the month
April too”. O mês April é o quarto mês (“fourth month”).

Alternative B is correct, pois o trecho “her parents made her a party” é equivalente a dizer “her
father and her mother made her a party”.

Alternative C is incorrect, pois o texto não fala qual o mês favorito da mãe de April.

Alternative D is correct, pois “gifts” e “presents” são sinônimos e o text afirma que “she
received a lot of presents”.

Aula 09 – Relative Clauses/Quantifiers 45


www.estrategiamilitar.com.br 68
Teacher Andrea Belo
Aula 09: Escola Naval 2021

QUESTÃO 07 – The underlined word in the text is

a) an adjective

b) a pronoun

c) a noun

d) a verb

Comentários:

Alternative A is correct, pois “little” acompanha o noun “girl” dando a ele uma característica,
logo, trata-se de um adjective.

Alternative B is incorrect, pois “pronouns” substituem nouns e, nesse caso, “little” acompanha
o noun “girl”.

Alternative C is incorrect, pois “little” acompanha um noun, não é um noun.

Alternative D is incorrect, pois verbs indicam ações e podem ser conjugados em diferentes
tempos verbais, o que não é o caso de “little”.

QUESTÕES ITA/2014
Texto para responder as questões 08 a 10

Aula 09 – Relative Clauses/Quantifiers 46


www.estrategiamilitar.com.br 68
Teacher Andrea Belo
Aula 09: Escola Naval 2021

Questão 08 – A Grant Study, pesquisa realizada pela Universidade de Harvard,

A ( ) teve por objetivo investigar o comportamento de pessoas idosas e felizes.

B ( ) possibilitou o levantamento gigantesco de dados sobre pesquisadores de Harvard.

C ( ) comprovou que John F. Kennedy foi um homem extremamente feliz.

D ( ) chama-se the 75 year old Grant Study por ser homenagem à faixa etária analisada.

E ( ) comprovou que felicidade na vida adulta está atrelada às relações afetivas ao longo da
vida.

Comentários:

Na letra A, segundo o texto (l. 3), a pesquisa investigou “healthy male Harvard college students
who… of their lifetime...” (estudantes masculinos saudáveis de Harvard que, ao longo de suas
vidas...). Portanto, não se referiu a pessoas idosas e felizes, o que torna a letra A incorreta.

Sobre a letra B, pesquisa possibilitou o levantamento de “an exorbitant amount of data that
deeply reflects the human condition” (l. 7), ou seja, “uma quantidade exorbitante de dados que
relfetem profundamente a condição humana” – portanto, não sobre pesquisadores de
Harvard. Isso torna a letra B incorreta.

O texto aponta que John F. Kennedy, na verdade, foi um dos participantes da pesquisa, mas
isso só foi descoberto recentemente (l. 5). No entanto, não é apontado que ele foi um homem
extremamente feliz, o que torna a letra C incorreta.

A pesquisa não é uma homenagem a essa faixa etária, mas um estudo extenso sobre o
desenvolvimento humano, que levou, de fato, cerca de 75 anos. Portanto, a letra D é incorreta.

Segundo o texto (l. 8-9), “warm relationships between parents, spouses, children and friends
have the greatest impact on your health and happiness in old age”, ou seja, “relacionamentos
afetuosos entre pais, esposas, filhos e amigos têm o maior impacto na sua saúde e felicidade
na idade avançada”. Portanto, a pesquisa comprovou que a felicidade na vida adulta está
atrelada a relações afetivas ao longo da vida, o que torna a alternativa E correta.

Aula 09 – Relative Clauses/Quantifiers 47


www.estrategiamilitar.com.br 68
Teacher Andrea Belo
Aula 09: Escola Naval 2021

Questão 09 – Assinale a opção cuja reescrita não altera o sentido de: “Though all identities are
confidential, it was recently discovered that John F. Kennedy was a sample participant ”

A ( ) John F. Kennedy was a sample participant, although nobody knew that.

B ( ) In spite of being a sample participant, John F. Kennedy’s identity was never discovered.

C ( ) The study was confidential; thus the participation of John F. Kennedy was never
discovered.

D ( ) Besides being a confidential study, John F. Kennedy said he used to be a participant.

E ( ) In spite of the fact that all identities are kept confidential, it was recently found out that
John F. Kennedy was a sample participant.

Comentários:
“Apesar de todas as identidades serem confidenciais, foi descoberto recentemente que John
F. Kennedy foi um dos participantes”.
Na letra A, a informação de “it was recently discovered” (foi descoberto recentemente) foi
descartada, o que altera o sentido da frase do trecho original. Tem-se apenas “John F. Kennedy
foi um participante, apesar de ninguém saber disso”. Portanto, a alternativa é incorreta.
Na letra B, “John F. Kennedy’s identity was never discovered” (a identidade de John F. Kennedy
nunca foi descoberta) contradiz a informação original de que “foi descoberto recentemente
que John F. Kennedy foi um dos participantes”. Portanto, a alternativa é incorreta.
Na letra C, a informação de que a identidade de John F. Kennedy nunca foi descoberta contradiz
o trecho original de que foi descoberto recentemente que John F. Kennedy foi um dos
participantes”. Portanto, a alternativa é incorreta.
Na letra D, “Além de ser um estudo confidencial, John F. Kennedy disse que ele costumava ser
um participante” adiciona e contradiz informações que não existem no trecho original,
alterando, assim, completamente o seu sentido. Portanto, a alternativa D é incorreta.
Na letra E, “Apesar do fato de que todas as identidades são mantidas confidenciais, foi
descoberto recentemente que John F. Kennedy foi um participante”. Aqui, “found out”
corresponde ao mesmo sentido de “discovered”; além disso, podemos ver que o sentido
permaneceu o mesmo em relação ao trecho original. Portanto, a alternativa está correta

Aula 09 – Relative Clauses/Quantifiers 48


www.estrategiamilitar.com.br 68
Teacher Andrea Belo
Aula 09: Escola Naval 2021

Questão 10 – Substituindo os adjetivos long e comprehensive, respectivamente, por easy e rich


na oração “Harvard conducted one of the longest and most comprehensive studies of human
development” (linha 1), teremos:

A ( ) the most easy - the richest

B ( ) the easiest - the most rich

C ( ) the more easy - the richer

D ( ) the easiest - the richest

E ( ) the most easy - the most rich

Comentários:

No trecho em questão, “long” e “comprehensive” estão na forma de superlativo, portanto,


“the longest” (maior/mais extenso) e “the most comprehensive” (mais abrangente).

No caso de “easy” e “rich”, as formas de superlativo corretas são “the easiest” e “the richest”.
É importante lembrar que, no inglês, adiciona-se “-est” para adjetivos de até duas sílabas, além
de acrescentar o artigo “the” – assim, tem-se “o mais fácil” e “o mais rico”.

O uso de “-er” serve apenas para o comparativo (“easier than”, por exemplo). Além disso,
“most” (ou “more”, no comparativo), como em “the most comprehensive” só é utilizado se o
adjetivo tiver três ou mais sílabas.

Portanto, as letras A, B, C e E são incorretas. Boa questão para relembrar o conteúdo de


comparativos e superlativos já aprendidos. A alternativa correta, então, é a letra D.

QUESTÕES IME/2018
Texto para responder as questões 11 a 13

Aula 09 – Relative Clauses/Quantifiers 49


www.estrategiamilitar.com.br 68
Teacher Andrea Belo
Aula 09: Escola Naval 2021

IME/2018

Aula 09 – Relative Clauses/Quantifiers 50


www.estrategiamilitar.com.br 68
Teacher Andrea Belo
Aula 09: Escola Naval 2021

Questão 11 – Choose the correct option.


(A) Hypatia did not marry so she could study; consequently, she followed the role model of the
time.
(B) Although Hypatia remained celibate, her father had been doubtful she would ever do so
because of the activities society expected her to perform.
(C) Professions were chosen according to the social class a woman occupied, centered around
family tradition and faith.
(D) Not many gaps have to be filled about Hypatia's academic life now; we know almost
everything because she continued her father's work.
(E) Some people didn't seem to understand science regardless of religion, and this might
explain why scientific concepts were seen as an obstacle.
Comentários:
No terceiro parágrafo (linhas 31-34), “Hypatia, por outro lado, levou a vida de um respeitado
acadêmico na universidade de Alexandria. Ela nunca se casou e permaneceu celibatária
durante a sua vida, devotando-se ao saber e ao ensino”. Portanto, o texto não afirma que ela
deixou de se casar para poder estudar – não se casar acabou sendo uma consequência natural
da vida que ela escolheu levar. Além disso, ela não seguiu o modelo da época, já que foi contra
o papel da mulher da sociedade daquele tempo. Isso torna a letra A incorreta.
Segundo o texto (linhas 26-28), “Téon se recusou a impor a sua filha o papel tradicional
designado às mulheres, e a criou como alguém criaria um filho segundo a tradição grega:
ensinando-a seu próprio negócio”. Então, o pai de Hypatia não ficou em dúvida sobre a filha
permanecer celibatária já que a sociedade esperava que ela exercesse, já que ele mesmo
procurou criá-la fora dos padrões daquela civilização. Isso torna a letra B incorreta.
No segundo parágrafo (29-30), “As mulheres gregas de todas as classes se ocupavam com o
mesmo tipo de trabalho, a maioria centrado nas necessidades domésticas da família”.
Portanto, as profissões não eram escolhidas de acordo com a classe social que a mulher
ocupava, nem tinham o foco da tradição ou fé da família. Isso torna a letra C incorreta.
Ela estava prosseguindo com um programa iniciado pelo seu pai, mas isso não significa que se
sabe quase tudo sobre sua vida acadêmica; afinal, o início do texto afirma que mais se sabe
sobre sua morte do que sobre a sua vida. Portanto, a letra D é incorreta.
No segundo parágrafo (linha 17), o autor afirma que Hypatia era vista como um “obstáculo”
(stumbling block) para aqueles que haviam aceitado a “verdade” do Cristianismo, não fosse
pelo seu carisma, charme e excelência em tornar conceitos filosóficos e matemáticos
complexos compreensível para os seus alunos; conceitos esses que iam contra os
ensinamentos da nova igreja. Portanto, conceitos científicos eram vistos como obstáculos
porque muitas pessoas não aceitavam que a ciência independia da religião; isso torna a letra
E correta.

Aula 09 – Relative Clauses/Quantifiers 51


www.estrategiamilitar.com.br 68
Teacher Andrea Belo
Aula 09: Escola Naval 2021

Questão 12 – Choose a synonym for the expression to the extent that in the sentence “ (…)
Hypatia has come to symbolize this tragedy to the extent that her death has been cited as the
end of the classical world.” in lines 24 and 25.

(A) as well as

(B) in contempt of

(C) by easy stages

(D) for the record that

(E) to the degree that

Comentários:

A expressão “to the extent that” indica “ao ponto que”. Portanto, “Hypatia simbolizou essa
tragédia ao ponto que sua morte foi citada como o fim do mundo clássico”. Portanto, carrega
um sentido de causa.

Na letra A, “as well as” expressa “assim como”, portanto, marca uma comparação, o que não
cabe como sinônimo para a expressão em questão. Isso torna a alternativa incorreta.

Na letra B, “in contempt of” expressa “em desprezo a”, o que contradiz o contexto do trecho,
já que a morte de Hypatia foi o símbolo da tragédia. Isso torna a alternativa incorreta.

Na letra C, “by easy stages” expressa “aos poucos”. “Ao ponto que” não tem o mesmo sentido
de “aos poucos”, então, não podem ser considerados sinônimos. Isso torna a alternativa
incorreta.

Na letra D, “for the record that” expressa “para que se conste que”. “Ao ponto que” não impõe
uma constatação como a primeira expressão, logo, não podem ser considerados sinônimos.
Isso torna a alternativa incorreta.

Na letra E, “to the degree that” expressa “ao grau que”. Se compararmos “ao ponto que” e “ao
grau que” temos, praticamente, o mesmo significado. Portanto, a alternativa E está correta.

Aula 09 – Relative Clauses/Quantifiers 52


www.estrategiamilitar.com.br 68
Teacher Andrea Belo
Aula 09: Escola Naval 2021

Questão 13 – Choose the appropriate continuation for the sentence “By all accounts, Hypatia
was an extraordinary woman ___________” in line 26.
(A) since she was assigned a profession different from her father's.
(B) not only for her time, but for any time.
(C) as she lived for the Gods.
(D) devoted to teaching, which was a common activity for women at that time.
(E) of slight intellectual power.

Comentários:
Depois de citar a sua vida de ensinamentos e a sua trágica morte como o fim do mundo clássico,
o autor afirma que “Em todo caso, Hypatia foi uma mulher extraordinária ...”.
Na letra A, “já que ela foi designada para uma profissão diferente da do seu pai” não se encaixa
no contexto, pois sabemos que Hypatia seguiu os ensinamentos do pai: “Téon se recusou a
impor a sua filha o papel tradicional designado às mulheres, e a criou como alguém criaria um
filho segundo a tradição grega: ensinando-a seu próprio negócio” (linhas 26-28). Isso torna a
alternativa incorreta.
Na letra B, “não apenas para a sua época, mas para qualquer uma” corresponde
adequadamente ao contexto, já que o autor descreve a grande influência de H ypatia para o
saber daquela época, o que permanece até os dias de hoje. Portanto, a alternativa B está
correta.
Na letra C, “já que ela vivia pelos deuses” não se adequa ao contexto. No texto, nada é citado
acerca das suas crenças; além disso, ela era considerada uma bruxa por supostamente impedir
seu grande amigo Orestes – que era pagão – a aceitar a “verdadeira fé”. Portanto, a alternativa
é incorreta.
Na letra D, “devota ao ensinamento, que era uma atividade comum para as mulheres naquela
época” não se adequa ao contexto, já que “As mulheres gregas de todas as classes se ocupavam
com o mesmo tipo de trabalho, a maioria centrado nas necessidades domésticas da família” (l.
29-30). Portanto, a alternativa é incorreta.
Na letra E, “de pouco poder intelectual” contradiz o contexto, uma vez que o texto afirma
adiante que “Os antigos escritores concordam que ela foi uma mulher de enorme poder
intelectual” (l. 34-35). Portanto, a alternativa é incorreta.

Aula 09 – Relative Clauses/Quantifiers 53


www.estrategiamilitar.com.br 68
Teacher Andrea Belo
Aula 09: Escola Naval 2021

QUESTÕES ESA/2020 inédita

Texto para responder a questão 14

QUESTÃO 14 (ESA/2020 inédita) – Read the extract from the text

“He liked to take apart televisions and put them back together again”

The pronoun “them” refers to:

a) equipment in general

b) Apple equipment

c) Steve Jobs’ hobbies

d) televisions

e) things to take apart

Aula 09 – Relative Clauses/Quantifiers 54


www.estrategiamilitar.com.br 68
Teacher Andrea Belo
Aula 09: Escola Naval 2021

Comentários:
Aqui temos uma questão que requer o entendimento da função dos pronouns. Pronouns nos
ajudam a evitar a repetição de palavras e à construção de frases curtas e desconfortáveis de
ler.
Para encontrarmos a quem o pronome se refere, vamos fazer o processo reverso e desacoplar
as duas frases. Assim, teremos:
“he liked to take apart televisions. And he liked to put televisions back together again”.
Assim, percebemos que “them” substitui “televisions” e que a alternativa correta é D.
Outra alternativa rapidamente é descartada ao entendermos os sentidos dos verbos das frases.
“take apart” significa desmontar e “put together” significa remontar. Portanto, a alternativa
correta precisaria trazer um objeto que pudesse ser desmontado e remontado, o que não é o
caso da alternativa C.
As alternativas A, B e E são descartadas ao percebermos que nenhum dos nouns mencionados
nelas já apareceram no texto anteriormente e, por isso, não poderiam estar sendo retomados
por um pronoun com o objetivo de substituir e evitar a repetição desse noun.

QUESTÃO 15 (EsPCEx/2019) – Texto para questão 15

Aula 09 – Relative Clauses/Quantifiers 55


www.estrategiamilitar.com.br 68
Teacher Andrea Belo
Aula 09: Escola Naval 2021

QUESTÃO 15 (EsPCEx/2019) – Texto para questão 16

According to the text, choose the correct statement.

[A] A new sustainable material has already been chosen.

[B] Lego has already reduced the size of their packaging.

[C] Lego is planning to reduce the size of their products.

[D] Lego’s raw material will continue to be the same.

[E] They are going to hire 100 specialists in 2030.

Comentários:

Alternative A is incorrect, de acordo com o trecho "there is no official definition of a sustainable


material". Ou seja, the new material ainda não foi escolhido ("hasn't already been chosen).

Alternative B is correct, pois o texto afirma no último parágrafo que uma das medidas já
tomadas para reduzir a "carbon footprint" foi justamente a redução "of packaging size".

Alternative C is incorrect, pois Lego já fez essa redução.

Alternative D is incorrect, visto que a ideia central do texto é justamente o objetivo da Lego de
substituir o plástico usado por um "sustainable material".

Alternative E is incorrect, pois 2030 é o prazo final para a substituição proposta. No segundo
parágrafo se afirma que 100 especialistas serão chamados para realizar esses estudos e
escolher um novo material, a fim de que em 2030 esse objetivo tenha sido atingido. Assim, eles
não podem ser contratados em 2030.

Aula 09 – Relative Clauses/Quantifiers 56


www.estrategiamilitar.com.br 68
Teacher Andrea Belo
Aula 09: Escola Naval 2021

QUESTÃO 16 (EFOMM/2016) - Texto para questão 16

Aula 09 – Relative Clauses/Quantifiers 57


www.estrategiamilitar.com.br 68
Teacher Andrea Belo
Aula 09: Escola Naval 2021

QUESTÃO 16 (EFOMM/2016) - Complete the sentences with the correct verb tenses.

1. Little ____________ how inconvenient he can be. 2. Not until she received the call
_______________ relieved. 3. Not for one moment _____________ your honesty. 4. Under no
circumstances _______________ class.

( a ) does he realize/ she had breathed/would I doubt/ should you miss.

( b ) does he realize/ did she breathe/would I doubt/ should you miss

( c ) he realizes/ did she breath/ I would doubt/ you should miss

( d ) he will realize/ she breathed/ I would doubt/ should you miss

( e ) he realized/ she would breathe/would I doubt/ you should miss.

Comentários:

Item 1 deve ser preenchido com “does he realize”, pois a construção correta é “little do/does
+ personal pronoun + verb”. Alternative C, D e E are incorrect por isso.

Item 2 deve ser preenchido com “did she breathe”, pois a construção é “did + persoal pronoun
+ verb”. Alternative A, D e E are incorrect por não usarem essa construção. Perceba que
alternativa C usa “did she breath” ao invés de “breathe”, o que faz dela incorrect, pois “breath”
(“respiração”) é noun e “breathe” (“respirar”) é verb.

Item 3 deve ser preenchido com “would I doubt”, pois a construção é “would + personal
pronoun + verb”. Alternatives C e D are incorrect por não usarem essa construção.

Item 4 deve ser preenchido com “should you miss”, pois a construção é “should + personal
pronoun + verb”. Alternatives C e E are incorrect por não usarem essa construção.

Logo, correct alternative is B.

Aula 09 – Relative Clauses/Quantifiers 58


www.estrategiamilitar.com.br 68
Teacher Andrea Belo
Aula 09: Escola Naval 2021

Considerações finais
Outra etapa vencida em sua caminhada das aulas de Inglês rumo à sua aprovação.

Em virtude da apresentação de cada tema de nossas aulas, seus conhecimentos se tornam mais
aguçados e seu conhecimento geral fica abrangente – e você – mais bem preparado.

Por mais que que você estude muito, sempre há detalhes complexos e assim, o estudo das
regras gramaticais é importante – que eu explico de forma dinâmica para facilitar. Afinal, você terá
que saber interpretar textos – e isso exige vocabulário e palavras sem fim.

Organize seus estudos. Acompanhe as aulas na sequência que as preparei para você. Crie o
hábito de ler. Ler é fundamental para ampliar o vocabulário.

Faça os exercícios que aparecem durante a teoria, os exercícios da lista de questões para fixar
os conteúdos estudados em sua mente. Leia as traduções que ofereço no fim do material. Fique
atento às palavras novas. Estude sempre, cada dia mais. Estudar e dedicar-se é o segredo da
aprovação, acredite! Além da prática de exercícios , comece a usar seus conhecimentos para falar
de forma correta, assim você fixará mais a maneira adequada do uso da gramática.

É importante lembrar também do nosso Fórum de dúvidas do Estratégia Militares e outras


redes sociais complementares para que seus estudos avancem cada vez mais.

Aula 09 – Relative Clauses/Quantifiers 59


www.estrategiamilitar.com.br 68
Teacher Andrea Belo
Aula 09: Escola Naval 2021

14. Referências bibliográficas

ACKLAM, Richard; CRACE, Araminta. Total English: Pre intermediate. 1 ed. Grã-Bretanha: Longman
do Brasil, 2005.

BLATT, Franz. Précis de Syntaxe Latine. Lyon, Paris: IAC, 1952.

BENTES, Anna Christina e Mussalim, Fernanda (org.). Introdução À Linguística, Domínios E Fronteiras.
6ª edição. Editora Cortez. São Paulo. 2006.

BOURGOGNE, Cleuza Vilas Boas & Silva Lilian Santos. Interação & Transformação. SP: Ed. Brasil,
1999.

BOWKER, L. & PEARSON, J. Working with Specialized Language. Routledge. Capítulos 1, 2, 8,10 e 11,
2002.

BUSSE, Winfried Busse & Mário Vilela. Gramática de Valências. Coimbra: Almedina,1986.

CARVALHO, José Herculano de. Estudos Lingüísticos. v. 2. Coimbra: Atlântida, 1969.

CHIMIM, Renata; Ilearn English student book, 4 / Renata Chimim, Viviane Kirmeliene; [obra coletiva
organizada e desenvolvida pela editora]. 1ª. ed. São Paulo: Pearson Education do Brasil, 20 13.

CORBEIL, J.-Cl., ARCHAMBAULT, A. Michaelis Tech dicionário temático visual inglês -português-
francês-espanhol. Tradução: Marisa Soares de Andrade. São Paulo: Melhoramentos, 1997.

CUNHA, Celso. Nova Gramática do Português Contemporâneo. Rio de Janeiro: Nova fronteira,
terceira edição, 2001.

CUNNINGHAM, Gillie; REDSTON, Chris. Face2Face: Upper Intermediate. 1 ed. Brazil: Cambridge,
2001.

Aula 09 – Relative Clauses/Quantifiers 60


www.estrategiamilitar.com.br 68
Teacher Andrea Belo
Aula 09: Escola Naval 2021

DANIELS, H. Vygotsky and pedagogy. Educational Tasks Pedagogical Communication for Teachers.
Routledge, 3 rd edition, 2001.

FAIRCLOUGH, N. Discourse and social change. Polity Press, 1992.

GENTZLER, E. Contemporary Translation Theory. Routledge, 1993.

HOUAISS, A., CARDIM, I. Dicionário universitário Webster inglês-português / português-inglês. São


Paulo: Record, 1998.

HYLAND, K. Genre and second language writing – For teachers and pedagogical professionals in
general, 2003.

HUTCHINSON, Tom & WATERS, Alan. English for Specific Purposes. Cambridge: Cambridge University
Press, 1996.

LAFACE, A. O dicionário e o contexto escolar. Revista Brasileira de Linguística, Unesp/Assis, v.9, 1982,
p. 165-179.

LOBATO, M.P. Lúcia. Teorias Linguísticas e ensino do português como língua materna. Brasília: UNB,
1999.

MICHAELIS Tech Dicionário Temático Visual: línguas estrangeiras – Pesquisa e tradução Marisa
Soares de Andrade. – São Paulo: Companhia Melhoramentos, 1997..

SILVA, João Antenor de C., GARRIDO, Maria Lina, BARRETO, Tânia Pedrosa. Inglês Instrumental:
Leitura e Compreensão de Textos. Salvador: Centro Editorial e Didático, UFBA. 1994.

SILVA, T.; MATSUDA, P. Second language writing research: perspectives on the process of knowledge
construction, 2001.

SILVEIRA BUENO, F. A formação histórica da língua portuguesa. 3. ed. São Paulo: Saraiva , 1967.

Aula 09 – Relative Clauses/Quantifiers 61


www.estrategiamilitar.com.br 68
Teacher Andrea Belo
Aula 09: Escola Naval 2021

SIMPSON, J., WEINER, E. (eds.) Oxford English dictionary on CD-ROM. 2ed. Oxford : Oxford
University Press, 1999.

PASCHOALIN, Maria Aparecida; SPADOTO, Neuza Terezinha. Gramática, Teoria e Exercícios. Editora
FDT. São Paulo. 1996.

RIBEIRO, Manuel P. Nova gramática aplicada da língua portuguesa. Rio de Janeiro: Metáfora editora,
14ª edição, 2002.

TUCK, Michael. Oxford Dictionary of Computing for Learners of English. Oxford: Oxford University
Press, 1996.

CETEMFolha/NILC: Corpus de Extractos de Textos Electrónicos. Banco de dados. Disponível em:


http://acdc.linguateca.pt/cetenfolha>.Último acesso (vários acessos) em: 04.05.2019.

COSTA, Daiane. As origens da língua inglesa. Disponível em:


http://englishmaze.wordpress.com/2011/01/25/as-origens-da-lingua-inglesa/Acesso em: 2/5/
2019.

VENTURINI, Laercio. Origem e desenvolvimento da língua inglesa. Disponível em:


<http://www.startenglish.com.br/index.php?option=com_content&task=view&id=100&Itemid=97
>. Acesso em: 22 mai. 2012.

OXFORD photo dictionary. Oxford: Oxford University Press, 1992

Referências complementares (websites):

http://www.sk.com.br/sk-perf.html - Acesso em 19 de março de 2019.

https://www.infoescola.com/ingles/

https://www.solinguainglesa.com.br/conteudo/indice.php

https://www.inglesnapontadalingua.com.br

https://www.englishexperts.com.br/

Aula 09 – Relative Clauses/Quantifiers 62


www.estrategiamilitar.com.br 68
Teacher Andrea Belo
Aula 09: Escola Naval 2021

15. Traduções

Aula 09 – Relative Clauses/Quantifiers 63


www.estrategiamilitar.com.br 68
Teacher Andrea Belo
Aula 09: Escola Naval 2021

O papel das mulheres na sociedade espanhola tem mudado rapidamente


desde que o país se tornou uma democracia após a morte do general Franco
em 1975. Ele havia eliminado reformas liberais introduzidas na década de
1930, quando a Espanha era uma república. Para as mulheres especificamente,
esses incluíram uma lei de divórcio benevolente e direitos de propriedade. Na
década de 1930, muitas mulheres desempenharam um papel importante.
Grande parte à esquerda, muitas vezes lutando lado a lado com homens nas
milícias pró-República durante o período de 1936-39 na guerra civil. Mas
depois disso, o novo regime, para a maior parte aplaudida pela igreja, era de
recolocá-las em casa como esposas e mães, com divórcio proibido e trabalhar
fora era algo de se franzir a testa.

A mudança começou na década de 1960, quando a Espanha abriu-se para os


turistas. Diante da concorrência de norte europeus com ideias “sexualmente
liberadas”, as mulheres espanholas “declararam guerra a eles, aos homens e
aos mais velhos”, nas palavras de Lucia Graves, autora de“ A Mulher
desconhecida ”, que narra sua vida como uma Inglesa casada com um espanhol
na época. Essa autoafirmação agressiva continua. Não totalmente com
sucesso. Nas universidades, as alunas agora superam os homens. Um típico
casal tem um ou dois filhos nos dias de hoje, muito longe desde os dias em que
famílias de oito ou dez eram comum. Mas as mulheres espanholas ainda
enfrentam os problemas de suas irmãs no norte da Europa. O progresso delas
no trabalho muitas vezes é bloqueado, seu salário geralmente é menor do que
o dos homens.

Aula 09 – Relative Clauses/Quantifiers 64


www.estrategiamilitar.com.br 68
Teacher Andrea Belo
Aula 09: Escola Naval 2021

Aula 09 – Relative Clauses/Quantifiers 65


www.estrategiamilitar.com.br 68
Teacher Andrea Belo
Aula 09: Escola Naval 2021

A escassez de alimentos é um problema sério para o mundo e é predominante na África


Subsaariana. A escassez de alimentos é causada por fatores econômicos, ambientais e
fatores sociais, como falha na colheita, superpopulação e políticas governamentais
pobres são a principal causa de alimentos escassez na maioria dos países. Fatores
Ambientais determinar o tipo de culturas a serem produzidas em um determinado lugar,
fatores econômicos determinam a compra e capacidade de produção e fatores sócio-
políticos determinam distribuição de alimentos para as massas. A escassez de alimentos
atingindo impactos negativos de longo e curto prazo que incluem fome, desnutrição,
aumento da mortalidade e agitação política
É necessário abordar coletivamente os questão da insegurança alimentar, tanto de
emergência quanto medidas de prazo.
Causas da escassez de alimentos
Existem vários fatores sociais que causam escassez de alimentos. A taxa de aumento
populacional é maior do que o aumento na produção de alimentos. O mundo está
consumindo mais do que produz, levando ao declínio no estoque de alimentos
armazenamento e aumento dos preços dos alimentos.
O aumento da população levou ao desmatamento de terras agrícolas para
assentamentos humanos redução da produção agrícola (Kamdor, 2007).
A superlotação da população em um determinado local resulta em urbanização de
campos agrícolas anteriormente ricos.
Destruição de florestas para assentamentos humanos, particularmente
floresta tropical tropical levou a mudanças climáticas, como secas prolongadas e
desertificação. População aumento significa mais poluição, pois as pessoas consomem
mais combustível em carros, indústria, cozinha doméstica. O efeito resultante
aumenta a poluição do ar e da água, o que afeta a clima e produção de alimentos.
Fatores ambientais contribuíram muito à escassez de alimentos. A mudança climática
reduziu a produção agrícola. A mudança no clima é majoritariamente causados por
atividades humanas e, em certa medida, atividades naturais. Maior combustão de
combustíveis fósseis devido ao aumento da população através de usinas, motores
transporte e mineração de carvão e petróleo emite estufa gases que continuaram
afetando o clima mundial.
Desmatamento de florestas tropicais devido à pressão humana mudou os padrões
climáticos e as estações das chuvas, e levou à desertificação que não pode suportar uma
colheita

Aula 09 – Relative Clauses/Quantifiers 66


www.estrategiamilitar.com.br 68
Teacher Andrea Belo
Aula 09: Escola Naval 2021

Produção. Degradação da terra devido ao aumento da população humana atividades


impactou negativamente a agricultura produção (Kamdor, 2007). Desastres naturais
como inundações, tempestades tropicais e secas prolongadas estão aumentar e ter
impactos devastadores na segurança alimentar particularmente nos países em
desenvolvimento. Existem vários fatores econômicos que contribuem para a escassez
de alimentos.
Fatores econômicos afetam a capacidade dos agricultores de se envolverem na
produção agrícola. Há também efeitos a longo prazo dos alimentos escassez. Isso inclui
aumento no preço dos alimentos
Como resultado, procura e forças de fornecimento. Aumento do custo de produção de
alimentos devido ao aumento dos preços dos combustíveis com seca persistente nas
regiões produtoras de grãos contribuiu para o aumento do preço dos alimentos no
mundo. O aumento do preço do petróleo levou ao aumento do preço de
fertilizantes, transporte de alimentos e também industrial agricultura. O aumento dos
preços dos alimentos culminou em instabilidade e agitação social em várias nações do
mundo em 2007, nos países do México, Camarões, Brasil, Burkina Faso, Paquistão, Egito
e Bangladesh entre outras nações (Kamdor, 2007).
Solução para problema de escassez de alimentos
Existem algumas soluções para o problema da alimentação escassez. É necessário
reduzir a produção de carbono emissões e poluição para reduzir as conseqüências
climáticas mudar através de esforços concertados e individuais. Lá é necessário investir
em energia limpa, como energia solar, nuclear, energia geotérmica em residências e
indústrias, porque eles não têm efeitos adversos no meio ambiente (Kamdor, 2007).
As nações ricas devem ajudar as nações pobres a desenvolver e usar energia limpa e
renovável, a fim de estabilizar as emissões de efeito estufa na atmosfera (Watson, sd).
O governo precisa trabalhar em consulta com órgãos climáticos, o Banco Mundial e as
Nações Unidas em projetos que visam promover o meio ambiente verde.
Conclusão
As causas da escassez de alimentos são bem conhecidas e pode ser resolvido se medidas
apropriadas para resolver o problema é levado e efetivamente implementado.
As causas ambientais da escassez de alimentos são mudanças na clima e poluição devido
a atividades humanas como excesso de pasto e desmatamento que pode ser controlado
através da legislação.

Aula 09 – Relative Clauses/Quantifiers 67


www.estrategiamilitar.com.br 68
Teacher Andrea Belo
Aula 09: Escola Naval 2021

Aula 09 – Relative Clauses/Quantifiers 68


www.estrategiamilitar.com.br 68
TO BE, THERE TO BE,
PRESENT CONTINUOUS

VERB TO BE Forma afirmativa: You are a student.


O verbo to be equivale aos verbos ser e/ou estar em português. Verbo depois do sujeito.
A conjugação do verbo to be no presente simples (simple present) é:

AFFIRMATIVE-AFIRMATIVA Forma negativa: You are not (aren’t) a student.


I am I’m Eu sou/estou Verbo depois do sujeito com “not”.
You are You’re Você é/está
He is He’s Ele é/está Forma interrogativa: Are you a student?
She is She’s Ela é/está Verbo antes do sujeito.
It is It’s Ele/Ela é/está
We are We’re Nós somos/estamos Exemplos:
You are You’re Vocês são/estão I’m Brazilian.
They are They’re Eles são/estão She’s a very intelligent girl.
They are engineers at Petrobras.
NEGATIVE-NEGATIVA You are on the 10th floor.
I’m not in São Paulo now. I’m in Rio.
Eu não sou /
I am not I’m not ---x--- He isn’t a good soccer player.
estou
We are not doctors, we are engineers.
You are not You’re not You aren’t Você não é / está
Are you a scientist at Instituto Vital Brasil?
He is not He’s not He isn’t Ele não é / está
Are they near the beach?
She is not She’s not She isn’t Ela não é / está
Is it a good Hospital?
Ele/Ela não é /
It is not It’s not It isn’t
está
VERBOS THERE IS, THERE ARE
Nós não somos / Os verbos  There is, There are estão no tempo presente e
We are not We’re not We aren’t
estamos equivalem ao verbo haver no sentido de existir em português. There
Vocês não são / is é usado antes de coisas ou pessoas no singular, e there are é usado
You are not You’re not You aren’t antes de coisas ou pessoas no plural.
estão
Eles não são / As formas dos verbos there is e there are são:
They are not They’re not They aren’t
estão
AFFIRMATIVE-AFIRMATIVA

INTERROGATIVE-INTERROGATIVA There is There’s há/existe- Usado no singular

am I? Eu sou / estou? There are --x-- há/existem- Usado no plural

are you? Você é / está?


NEGATIVE-NEGATIVA
is he? Ele é / está?
There is not There isn’t
is she? Ela é / está?
There are not There aren’t
is it? Ele/Ela é / está?

are we? Nós somos / estamos? INTERROGATIVE-INTERROGATIVA


are you? Vocês são/ estão? Is there?
are they? Eles são / estão? Are there?

PROMILITARES.COM.BR 191
TO BE, THERE TO BE, PRESENT CONTINUOUS

Exemplos: Ação temporária que ocorre num período de tempo no


There is much information on TV. presente, mas não necessariamente no momento em que se fala.
There are many students in that school. Exemplos:
There isn’t any car in the garage. Paul is reading a book of Jorge Amado.
There aren’t many people in the stadium. The U-2 is performing at Maracanã.
Is there much sugar in the pot? People are eating less meat nowadays.
Are there three rooms in that house?
Indicar uma ação futura.
VERBOS NO PRESENT CONTINUOUS/ Exemplo:
PROGRESSIVE Hurry up! The plane is leaving soon.
O Present Continuous ou Progressive indica uma ação que está She is visiting her mother in Niterói tomorrow.
em progresso no momento da fala. Também pode indicar ações I’m on vacation at the moment, but I’m starting a new course
temporárias no presente, ações futuras e acontecimentos que parecem next week.
infindáveis no presente (hábitos irritantes).
A conjugação do “present continuous” é:
Descreve e enfatiza uma série de acontecimentos infindáveis.
Nesse tipo de ação geralmente usamos advérbios como “always,
AFIRMATIVA NEGATIVA INTERROGATIVA forever or constantly” e outros advérbios de frequência.
I am studying (Eu Exemplo:
I’m not studying Am I studying?
estou estudando) She’s always trying to show that she knows more than everyone
else.
You are drinking You’re not
(Você está drinking (you Are you drinking? I’m constantly having to remind you to tidy up your things.
bebendo) aren’t)

He is singing (Ele He’s not singing Formação do sufixo “ing” com alguns verbos:
Is he singing?
está cantando) (He isn’t) • Verbos de uma só sílaba, terminados por consoante precedida
de uma única vogal, dobram essa consoante para o acréscimo
She is working de –ing:
She’s not working
(Ela está Is she working?
(She isn’t) beg begging
trabalhando)
hit hitting
It is raining (está It’s not raining (It stop stopping
Is it raining?
chovendo) isn’t)

We are painting We’re not • Verbos teminados em –e perdem o –e para o acréscimo de –ing:
(Nós estamos painting (We Are we painting? bake baking
pintando) aren’t)
love loving
You are reading You’re not rise rising
(Vocês estão reading (You Are you reading?
lendo) aren’t)
• Verbos terminados em –ee conservam essa terminação para
They are walking They’re not o acréscimo de –ing:
(Eles estão walking (They Are they walking? agree agreeing
andando) aren’t)
flee fleeing
see seeing

FORMAÇÃO DO PRESENT • Verbos terminados em –ie perdem essa terminação e recebem


CONTINUOUS OU PROGRESSIVE –y antes da colocação de –ing:
die dying
We are studying English lie lying
tie tying
Verbo Objeto
Verbo Prncipal
Auxiliar +
ING • Qualquer verbo terminado em –y é simplesmente acrescido
Sujeito de –ing:
obey obeying
Ação ocorre no momento em que se fala.
study studying
Exemplo:
play playing
What are you doing now? I’m fixing my father’s car.
cry crying
Listen! Somebody is knocking at the door.
What is that music you’re playing?

192 PROMILITARES.COM.BR
TO BE, THERE TO BE, PRESENT CONTINUOUS

• Verbos terminados em –l precedido de uma única vogal Skype is now used by 170 million people around the world (each
dobram essa consoante para o acréscimo de –ing (regra month), not just on their computers, but on the move – on their
válida para o inglês britânico); quando o –l for precedido mobile phones and even on their tablet devices.
de duas vogais, a duplicação não é efetuada: Microsoft wants to tap in to this connected community, but it’s
label labelling paying a huge price for a business that isn’t even profitable.
steal stealing (Rory Cellan-Jones, BBC News. http://www.bbc.co.uk/worldservice/learningenglish/
language/wordsinthenews/2011/05/110511witn_skype_page.shtml)
feel feeling
Ao se ler “Microsoft is buying Skype”, é correto afirmar que
• Verbos com mais de uma sílaba, terminados por consoante a) a Microsoft está vendendo o Skype.
precedida de apenas uma vogal, dobram essa consoante, b) o Skype está sendo vendido pela Microsoft.
se o acento tônico recair na segunda sílaba; se não, apenas
c) a Microsoft está comprando o Skype.
recebem –ing:
d) o Skype está se despedindo da Microsoft.
begin beginning
e) a Microsoft está perdendo o Skype.
permit permitting
master mastering 07. (MACKENZIE 2000) Em inglês, “Você está esperando alguma
open opening carta?” seria:
a) “Have you been waiting for a chart?”.
EXERCÍCIOS DE b) “Are you expecting a letter?”.

FIXAÇÃO
c) “Are you attending any lecture?”.
d) “Are you staying for the lecture?”.
e) “Have you been hoping for a lecture?”.

01. (UFRRJ) Complete the sentence with the correct verb. 08. (FAAP 1997) Assinale a alternativa que completa corretamente a
“_________biological agents in that tube”. frase: “The whole world __________ against drugs now”.
a) There is c) Is e) Is there a) is fighting d) has fought
b) There are d) There is not b) fought e) fight
c) had been fighting
02. (EEAR) Choose the best alternative to answer the question below:
“Are you a pilot?”. 09. (G1 1996) Check the correct answer.
a) No, we are. c) No, I am not. – What ___________ you ___________ now?
b) Yes, you are. d) Yes, I weren’t. – I ________________ my car.
a) do/do - am wash c) are/doing - am washing
03. (EEAR) “An umbrella __________ a very ordinary object.”
b) are/do - washing d) do/doing - washing
Choose the best alternative to complete the blank in the Paragraph:
a) is c) weren’t 10. (UNESP 1988) Assinale a alternativa correta.
b) were d) are “Many countries ______ with nuclear reactors.”
a) is experimenting d) would experiment
04. “Don’t worry, you’ll get there in time. _____ another train at 11
b) experiments e) are experimenting
a.m.”
c) experimenting
a) There c) There are
b) It’s d) There is
EXERCÍCIOS DE
05. (FAAP) Complete: “_________ many people living in China”.
a) Is there c) There is e) Is TREINAMENTO
b) Are d) There are

06. (UNIOESTE 2012) Leia. 01. (FAAP-1997) Assinale a alternativa que preenche corretamente
a lacuna da frase a seguir: “In my school ________________ many
MICROSOFT IS BUYING SKYPE
foreign students”.
One is the giant business, whose software powers more than
a) has d) have
90% of the world’s computers. The other is the firm, which has
revolutionised the way many communicate. Now Skype is being b) there are e) there have
swallowed up by Microsoft. c) there is
It’s just eight years since Skype started helping people to make
calls over the internet for nothing, and this is the third time it’s been 02. (ESFCEX) “We ______ for our English test at the moment”.
bought and sold. a) are studying
Microsoft has been struggling to prove it can compete with the b) are study
likes of Google and Apple. Now as it tries to make an impact on the
c) study
mobile-phone world, it wants Skype to help it become a bigger force.
d) are studyng

PROMILITARES.COM.BR 193
TO BE, THERE TO BE, PRESENT CONTINUOUS

03. (UEL) “The point __________ exactly where you ________ now”. number fall even further. The idea of a totally efficient market 15should
As lacunas da frase são corretamente preenchidas pela alternativa: dismay Panini, which will sell fewer packs as a result. But as in all
markets, behaviour is not strictly rational. 16Despite entreaties, your
a) are - is d) are – are correspondent’s son is prepared to tear out most of his stickers to get
b) is - is e) there is – there are hold of Lionel Messi.
c) is – are (Fonte: http://www.economist.com/news/finance-and-economics/2160
3019-got-got-got-got-got-need-stickernomics Acesso: 13/ago/2014)

04. (FGV) Assinale a alternativa correta:


Marque a opção em que o uso do ing denota ação contínua.
a) “There are a soldier in the hospital.”
a) […] disturbing number of adults […] (ref. 6)
b) “There are some soldiers in the hospital.”
b) Collecting them is no idle pursuit […] (ref. 7)
c) “There is a lot of soldiers in the hospital.”
c) […] your first sticker […] has a 640/640 probability of being
d) “There is little soldiers at the hospital.” needed. (ref. 8)
e) “There are much soldiers in the hospital.” d) According to Sylvain Sardy and Yvan Velenik […] (ref. 9)
e) […] the market is not being rigged. (ref. 10)
05. (FUVEST) Assinale a alternativa que preenche corretamente a
lacuna:
07. (UFV 2004) Leia o texto a seguir.
“__________ little sugar in my juice. I would like more”.
HAS TECHNOLOGY RUINED CHILDHOOD?
a) Are there d) There are 1
Today, parents are increasingly worried about the safety of their
b) Is there e) There aren’t children, and because of this, they are not letting their children out to
c) There is play. As a result, children are no longer playing outside but shutting
themselves away in their rooms and losing themselves in individualistic
06. (ITA 2015) Leia. activities such as television viewing and computer games.
STICKERNOMICS
2
Yet, if they had the chance, they would rather get out of the
house and go to the cinema, see friends or play sport. In fact, when
Football albums asked what their idea of a good day was, only 1 in 7 said that they
Got, got, got, got, got, need would turn on the television.
THE World Cup is still two weeks away, but for children worldwide 3
British teenagers have always retreated to their bedrooms,
(plus 6disturbing numbers of adults) the race to complete the Brazil leaving the younger children to play in communal spaces such as the
2014 sticker book started long ago. 1Panini, an Italian firm, has sitting room, garden or kitchen. However, children from the age of 9
produced sticker albums for World Cups since Mexico 1970; this year’s are now turning to their bedrooms as a place to socialise.
version has 640 stickers to collect. 7Collecting them is no idle pursuit, 4
Bedroom culture is a phenomenon of the past 20 years with
however. Getting every slot filled delivers an early lesson in probability, families getting smaller and homes getting more spacious. Increasing
the value of statistical tests and the importance of liquidity. prosperity has also contributed to the rise of the bedroom culture.
When you start an album, 8your first sticker (in Britain, they come Of British children aged 6 to 17, 72% have a room they do not
5
in packs of five) has a 640/640 probability of being needed. 2As the have to share with a sibling, 68% have their own music installation,
spaces get filled, the odds of opening a pack and finding a sticker 34% have an electronic games controller hooked up to the television,
you want fall. 9According to Sylvain Sardy and Yvan Velenik, two 21% have a PC. Only 1%, on the other hand, have an Internet
mathematicians at the University of Geneva, the number of sticker connection in their bedroom.
packs that you would have to buy on average to fill the album by
mechanically buying pack after pack would be 899. 11That assumes
6
On average children devote 5 hours a day to screen media. Even
there is no supply shock to the market (the theft of hundreds of so, only 1 child in 100 can be classed as a real screen addict, a child
thousands of stickers in Brazil in April 12left many fearful that Panini who spends a worrying 7 hours or more watching TV or playing
would run short of cards). computer games.
It also assumes that 10the market is not being rigged. Panini Although children generally have a few favourite programmes,
7

says that 3each sticker is printed in the same volumes and randomly they mostly use television to kill time when they are bored and have
distributed. In a 2010 paper Messrs Sardy and Velenik gamely played nothing special to do. Moreover, the distinction between individualistic
the role of “regulator” by checking the distribution of stickers for a media use and social activities such as chatting with friends is less
660-sticker album 13sold in Switzerland for that year’s World Cup. Out extreme than is commonly assumed. Children gossip about television
of their sample of 6,000 stickers, they expected to see each sticker soap characters, make contact with other children on the Internet,
9.09 times on average (6,000/660), 4which was broadly borne out in and visit friends to admire 6their new computer games.
practice. 8
As the use of PCs proliferates, reading skills are expected to
Even in a fair market, it is inefficient to buy endless packs as an suffer. Nevertheless, 57% of children say they still enjoy reading, and
individual (not to mention bloody expensive for the parents). The 1 in 5 teenagers can be classed as a book-lover.
answer is to create a market for collectors to swap their unwanted 9
As a result of the bedroom culture, it is becoming rarer for
stickers. The playground is 14one version of this market, 5where a child children over the age of 10 to watch television with their parents.
who has a card prized by many suddenly understands the power of Once in their rooms, children tend to stay up watching television for
limited supply. Sticker fairs are another. As with any market, liquidity as long as they wish. Consequently it is getting 8harder to control
counts. The more people who can be attracted into the market with children’s viewing.
their duplicate cards, the better the chances of finding the sticker you 10
One father told researchers that 9he drew the line at 9 pm. His
want. son, on the other hand, said: “They tell us to go up at about 9.30 or
Messrs Sardy and Velenik reckon that a group of ten astute 10 or something, and then we just watch until they come up and tell
sticker-swappers would need a mere 1,435 packs between them to us to switch it off at 11 or 11.30.”
complete all ten albums, if they take advantage of Panini’s practice of
Choose the alternative in which the capital word -ING form is an
selling the final 50 missing stickers to order. Internet forums, where
example of the present continuous:
potentially unlimited numbers of people can swap stickers, make this

194 PROMILITARES.COM.BR
TO BE, THERE TO BE, PRESENT CONTINUOUS

a) “a child who spends a WORRYING 7 hours or more” (paragraph 10. (EEAR 2011)
6) DOING THINGS
b) “INCREASING prosperity has also contributed to the rise of” Evan: Hey, Chloe. What are you doing?
(paragraph 4)
Chloe: I’m drawing a picture for the story I wrote in school today.
c) “children from the age of 9 are now TURNING to their bedrooms”
(paragraph 3) Evan: Oh, yeah? Terrific. Can I see it? It’s very pretty. I like the colors.

d) “children say they still enjoy READING” (paragraph 8) Chloe: Thanks, Dad.

e) “harder to control children’s VIEWING” (paragraph 9) Evan: Chloe, go look in the kitchen. Is Mom _____ coffee?
Chloe: Uh, no, Daddy, she’s still _______ the dishes.
08. (FATEC 2004) Leia. Evan: Oh. I guess I can wait a few minutes. I have to make a call.
BUNKER DOWN Chloe: Are you calling Grandma?
FORGET HIDING IN THE basement. Brits worried about their safety Evan: No. It’s a business call.
can now purchase a completely bombproof house, made by the steel (Taken from American Streamline)
manufacturer Corus. The Surefast shelter, launched earlier this month, is
GLOSSARY:
constructed out of steel panels that are slotted together and filled with to guess – achar, imaginar
concrete. But don’t expect to just throw it together at the last minute:
it takes several people 10 hours - and the help of a heavy crane - to Fill in the blanks with the correct verbs, respectively
assemble the two-story, 50,000 pounds structure. In tests the shelter
a) making - making
has successfully withstood everything from car bombs to blowtorches.
Still, it offers no protection from biological or chemical weapons. For b) making - doing
clean air, inhabitants had best outfit their bombproof homes with the c) doing - making
Dominick Hunter Group’s regenerative NBC filtration system. (The d) doing - doing
British Army is now installing it in its tanks.) Breathable air doesn’t come
cheap, either: a filter to support 10 people starts at 50,000 pounds.
(Newsweek, April 14, 2003.)
EXERCÍCIOS DE
Assinale a alternativa que apresenta o uso correto do presente
contínuo como em “The British Army is now installing it in its tanks.”.
a) The British Army is liking the new program.
COMBATE
b) The British Army is understanding the needs of the population.
c) The British Army is listening to the population. 01. (EFOMM) “The companies are expanding their business. They
__________ big now and __________ a lot of employees working
d) The British Army is preferring the new general.
there”.
e) The British Army is possessing many tanks.
a) is - there is d) are - are there
b) are - there is e) is - is there
09. (CN) Read the text below.
c) are - there are
HELPING AT A HOSPITAL
Every year many young people finish school and then take a year
02. (EFOMM) “Paul comes here every afternoon, but he ______________
off before they start work or go to college. Some of them go to other
tomorrow. His mother is very ill”.
countries and work as volunteers. Volunteers give their time to help
people. For example, they work in schools or hospitals, other help with a) are coming d) is coming
conservation. b) coming e) aren’t coming
Mike Coleman is 19 and ______________in Omaha, Nebraska, c) isn’t coming
in the United States. He wants to become a teacher but now he
______________ in Namibia. He’s working in a hospital near Katima 03. (UNESP) Assinale a alternativa correta:
Mulilo. He says, “I’m working with the doctors and nurses here to
“__________ a river near my house and It __________ a stream of
help sick people. I’m not a doctor but I can do a lot of things to help.
Amazon river”.
For example, I help carry people who can’t walk. Sometimes I go to
villages in the mobile hospital, too. There aren’t many doctors here so a) Is there – is d) There are – are
they need help from people like me. I don’t get any money, but that’s b) There are – is e) There is – is
OK, l’m not here for the money.” c) There is – are
“I’m staying here for two months, and I’m living in a small house
with five other volunteers. The work is hard and the days are long, 04. (EFOMM) “The companies are expanding their business and they
but I’m enjoying my life here. I’m learning a lot about life in Southern need all the help they can get. So they __________ several people”.
África and about myself! When I finish the two months’ work, I want
to travel in and around Namibia for three weeks. For example, I want a) employ d) employing
to see the animals in the Okavango Delta in Botswana.” b) is employing e) are employing
(http://vyre-legacy-access.cambridge.org) c) isn’t employing

Which verb forms respectively complete the gaps in text I? 05. (UFRS) “Today _____ more than 103 million dogs and cats
a) is living - leaves living in American homes, a total greater than the combined human
b) lives - is living populations of Spain and France.
c) is living - lives a) has d) have
d) leaves - is living b) there are e) is
e) leaves - is leaving c) there is

PROMILITARES.COM.BR 195
TO BE, THERE TO BE, PRESENT CONTINUOUS

06. (OSEC-SP) “Jane is an excellent student. At this moment she


_______ her lessons”. ANOTAÇÕES
a) studing
b) studied
c) is studied
d) has studying
e) is studying

07. (IME) “The 9:15 bus to town never stops here but it ____ at this
moment because an elderly woman ____ the street”.
a) is stopping - crosses
b) is stopping - is crossing
c) stops - is crossing
d) are stopping - are crossing
e) stopped - crossed

08. (FMU-SP) “I __________ there and I ______ a bird now”.


a) am looking – am seeing
b) am looking – seeing
c) look – am seeing
d) am looking – see
e) looking – seeing

09. (UF-BA) Escolha a pergunta cuja resposta é: “No, he isn’t”.


a) Are the students cheating?
b) Is Mary worrying about her future?
c) Is Tom working in the lab?
d) Is the lesson interesting?
e) Does he know how to enjoy his life?

10. (ITA) “The United States ____________ a big country”.


a) is
b) are
c) have
d) a e b são corretas.
e) todas são corretas.

GABARITO
EXERCÍCIOS DE FIXAÇÃO
01. B 04. D 07. B 10. E
02. C 05. D 08. A
03. A 06. C 09. C
EXERCÍCIOS DE TREINAMENTO
01. B 04. B 07. C 10. B
02. A 05. C 08. C
03. C 06. E 09. B
EXERCÍCIOS DE COMBATE
01. C 04. E 07. B 10. A
02. C 05. B 08. D
03. E 06. E 09. C

196 PROMILITARES.COM.BR
SIMPLE PRESENT

SIMPLE PRESENT Quando estabelece um fato, uma verdade universal, uma


informação científica.
O “Simple Present” é usado para ações do dia a dia, habituais.
Nesse caso, a frase vem geralmente acompanhada por um advérbio Exemplos:
de frequência. The earth goes round the sun.
Usado também para indicar fatos reais, uma verdade universal, Brazilian people drink more beer than Americans.
ações no futuro, horário oficial, opinião e narrativas. Nas terceiras She loves David.
pessoas do singular, na forma afirmativa, acrescentamos –S, –ES ou
–IES, sufixos que variam de acordo com a terminação do verbo. Nas
formas interrogativas e negativas usamos os auxiliares “Does” para as Numa narrativa quando descreve características regulares
terceiras pessoas e “Do” para as demais, e os verbos permanecem na de uma pessoa ou coisa.
forma base (infinitivo sem o “to”). Exemplo:
A conjugação do “Simple Present” é: I live in a town on the south part of the country. I work at
home, but I often visit other countries abroad on business. I
AFIRMATIVA NEGATIVA INTERROGATIVA have three children, one is still at school and the other goes to
University of Michigan. I speak French and German, and I know
I don’t (do not)
I like ( Eu gosto ) Do I like? a bit of Chinese. I don’t like sports but I enjoy soccer. I don’t
like
smoke, but I drink a lot of beer.
You like You don’t (do
Do you like?
( Você gosta ) not) like
Quando se estabelece uma opinião.
He likes He doesn’t (does Exemplos:
Does He like?
( Ele gosta ) not) like She thinks Paris is better than New York to live.
She likes She doesn’t They believe they will succeed at that company.
Does she like?
( Ela gosta ) (does not) like

It likes It doesn’t (does Descrevendo um evento que acontecerá no futuro,


Does it like? geralmente com hora e/ou data oficial.
( Ele/Ela gosta ) not) like
Verbos que são sempre usados no “Simple Present”
We like We don’t (do
Does we like? • Opinions: assume, believe, consider, feel, think, find,
( Nós gostamos ) not) like
suggest, suppose.
You like You don’t (do • Mental states: expect, forget, imagine, know, mean, notice,
Do you like?
( Vocês gostam ) not) like remember, understand.
• Emotions / Feeling: care, envy, fear, hate, like, love, mind,
They like They don’t (do
Do they like? prefer, regret, want, wish, detest.
( Eles gostam ) not) like
• Possession: belong to, have, hold, own, possess.
• Measure / Relation: contain, cost, hold, measure, weigh,
Formação do “Simple Present”: fit, carry.
The students always play soccer. • Perception: touch, see, hear, smell.
• Others: apply for, depend on, deserve, differ, exist, interest,
Sujeito Adv. de Verbo Complemento look (resemble).
Frequência Principal
O “Simple Present” é usado para ações rotineiras, habituais Nota:
ou indicando costume. Alguns desses verbos podem ser usados no “Present Continuous”
Nesse caso, a frase vem geralmente acompanhada por um quando estiverem exercendo a função de ação ( action ) no contexto.
advérbio de frequência. Veja a diferença:
Exemplos: Exemplos:
Mary always cooks for her husband. She considers her father a loveable person. (opinião dela em
relação ao seu pai, usa o “Simple Present”).
The engineer never works overtime.
She is considering her father’s proposal in her room. (neste
Do you always read the newspaper at night?
caso, ela está pensando sobre o problema é uma ação, usa o “Present
Continuous”).

PROMILITARES.COM.BR 197
SIMPLE PRESENT

Todos os verbos no “Simple Present” são acrescidos de um –S na 03.


3ª pessoa do singular (he, she, it).
Exemplo:
love loves
take takes
think thinks

Verbos terminados em –CH, -O, -SH, -SS, -X e –Z recebem a


According to the text, Garfield:
terminação –ES.
a) Thinks all days are beautiful.
Exemplo:
b) Prefers the kitchen only when it’s raining.
catch catches
c) Prefers staying outside if it’s a beautiful day.
go goes
d) Thinks days are beautiful in the kitchen even when it’s raining.
slash slashes
miss misses 04. (UEMG) A alternativa que completa corretamente a frase: “I am
mix mixes taking my umbrella just in case it____________” é a:
buzz buzzes a) rains d) is rain
b) rain e) “a” and “c” are correct.
Verbos terminados em –Y precedidos de consoante substituem o c) is raining
–Y por –IES.
Exemplo: 05. (UFPB) International advertising can be a risky business. When
carry carries McDonald's launched Le Big Mac in Paris, it discovered that in local
slang this meant "the big pimp". It is not just a question of language
copy copies either; national advertising styles also vary considerably. The British
study studies like humour and irony in their ads, whereas the Germans regard this
worry worries approach as frivolous. The French are more sexist than the British
and will use seminaked women in almost any context. The Italians
generally like to see beautiful people wearing beautiful clothes driving
Verbos terminados em –Y precedidos de vogal recebem a beautiful cars. These are not just national stereotypes, but based on
terminação –S, normalmente. hard experience. Different countries also prefer different products.
Exemplos: The text is PREDOMINANTLY in the:
enjoy enjoys a) present tense. d) present perfect tense.
stay stays b) past tense. e) present progressive tense.
play plays c) future tense.
buy buys
06. (UNESP 2013) Observe a tirinha.
Nota: Os auxiliares “do” e “does” são usados em frases
afirmativas para indicar ênfases, sempre concordando com o
sujeito da oração.
Exemplos:
I do love chocolate.
She does think Miami is the best place to live in The USA.

EXERCÍCIOS DE No trecho do primeiro quadrinho – she’s sick and tired of smelling beer

FIXAÇÃO –, ’s pode ser reescrito como


a) “is”. c) “goes”. e) “has”.
b) “was”. d) “does”.
01. (EPCAR) “John is a good student, so he ____ to school every day”.
07. (UFRRJ 2004) Leia.
a) go
TECHNOLOGICAL HURDLES FOR TERRORISTS
b) to go
To be successful, a terrorist or terrorist organization has to
c) come overcome formidable technical challenges. First, the terrorist has to
d) goes obtain a sufficiently lethal strain of a disease pathogen. Second, he
must know how to handle and store the pathogen correctly and
02. (UNITAU) “Mom often ____ when we ____ a joke”. safely. Third, he must know how to produce it in bulk. Tiny amounts
a) laugh - tell of a microorganism are lethal enough to ravage a field of crops, a herd
of animals, or a city of people, assuming the pathogen is delivered
b) laughs - tell
precisely to the target. However, 2biological agents do not survive well
c) laughs - tells outside the laboratory. In reality only a fraction of the biological agent
d) laughes - tell would reach the target population, so vastly larger amounts would be
e) laugh - tells needed to launch a catastrophic attack.

198 PROMILITARES.COM.BR
SIMPLE PRESENT

Considering the array of technological hurdles involved, it is 10. (UFSC) “He ____ The Times now, but on Sundays he ____ The
surprising that few terrorist attacks with biological weapons have been Observer”.
attempted. What is more, those attempts produced few casualties. a) is reading - reads d) to read - is reading
Recently, anthrax-laced letters killed five people in the United States.
That is tragic enough, but the 1casualties were fewer than might b) read - reads e) are reading - reads
have occurred from a small explosive or even a pistol. Researchers c) reads - read
calculate that since 1975, in 96 percent of the attacks worldwide in
which chemical agents were used no more than three people were
killed or injured. EXERCÍCIOS DE
(Awake! September 22, 2002.)

In the sentence, "[...] biological agents do not survive well [...]"(ref. 2),
TREINAMENTO
the use of the present tense implies
a) doubt. d) objectivity. 01. (ESPCEX/AMAN 2018)
b) condition. e) certainty. MILITARY OPERATIONS IN MEGACITIES
c) probability. A linguistic perspective
The challenge of conducting future military operations within
08. (PUC-RS 2000) Leia o texto. megacities (cities with populations over ten million) lies in understanding
COUPLE SELLS ADS TO PAY FOR WEDDING the dynamic and multidimensional complexities of these urban
areas. Military operations in megacities, whether combat-oriented or
Talk about a marriage of love and money. otherwise, will be similar to those in other urban environments, but
Tom Anderson and Sabrina Root paid for their $14,000 wedding will be complicated by factors unique to the megacity environment.
this weekend by selling advertising space at the ceremony and First and foremost, megacities are largely multilingual. While this
reception. can be said of large cities in general, the scale of multilingualism in
Everything from the wedding rings to a week at a penthouse in megacities magnifies its effects. For instance, in New York City (NYC) –
Cancun, Mexico, were donated after Anderson got 24 companies to a metropolitan megacity of over eighteen million people - nine foreign
sponsor the nuptials in exchange for having their names appear six languages are spoken by communities of one hundred thousand or
times from the invitations to the thank-you cards. larger. Language also plays a role in determining one's identity and the
language community in which one decides to live. For example, the
Anderson, 24, a bartender, spent his own money for his wife's
majority of Russian speakers in NYC tend to live in south Brooklyn and
$1,400 engagement ring while Root, 33, a hair stylist, paid $1,600
Staten Island, while Chinese speakers tend to cluster in Manhattan and
for the dress.
Sunset Park. In megacities, language, culture, and regional context go
The 1groom got the idea of corporate sponsorships 3while hand in hand and often reach beyond ethnic identities.
working in a small struggling animation studio that often had to
In order to fully understand the context of a megacity, we must
barter for services.
understand the role of the languages used in its communities. How
"So I was in a sales mode, and I got to thinking", he told the _____(1)_____ language communities interact in megacities? What
Philadelphia Inquirer, 2which ran a photo of the couple sitting among tensions _____(2) _____ caused by multiple language communities in
their corporate-sponsored wedding "gifts" in its Sunday edition. urban space? What role______(3) _____ language play in the power
The bride's perfume came from a local distributor, and coffee was structures (government or otherwise) of megacities?
provided gratis from a neighborhood supplier. (Adapted from Military Review - Jan/Feb 2016)
Advertisers had their names appear on the invitations and thank-
you cards, on cards at the buffet, on scrolls at the dinner table, in an Choose the alternative containing the correct words to respectively
ad placed in a local independent newspaper and in a verbal "thank complete gaps (1), (2) and (3) in paragraph 3.
you" that followed the first toast. a) is - does - do d) are - do - does
The Inquirer said the groom had bought two addresses on the b) do - are - do e) do - are - does
Internet's World Wide Web, namely: sponsoredwedding.com and c) are - is - do
weddingsponsors.com.
(Monday, August 16, 1999 - Reuters) 02. (UFSM 2001) Imagine it: Your plane touches down at Charles de
Gaulle and you take out your 1portable voice recognition-translation
Choose the alternative that best completes the sentence: "Today's device. You set the dial to "Français." Et voilà! You are free 3to roam
issue of The Inquirer ______ that another couple wants to have their Paris without anyone sneering at your high school French. Sound like
wedding sponsored. It also _________ a photo of them". science fiction? 2Machines that recognize your voice and translate your
a) says - runs language have already converged. Prototypes of real-time devices are in
b) say - run use, and they will probably be on the market in a decade or two. But
before we shell out $299.99 for this shiny new gadget, let us pause to
c) said - running
bid farewell to the dream of an idiomatic common ground - to the hope
d) saying - ran for mutual intelligibility and a linguistic brotherhood of man.
e) says - running (Lingua Franca, New York, May/June 2000.)

Assinale a alternativa que melhor expressa a ideia da pergunta:"Sound


09. (EFOMM) “The children usually ____ in the afternoon, but today
like science fiction?".
they ____ in the garden”.
a) Is the sound like in science fiction?
a) studies - are playing
b) Do you sound like science fiction?
b) are studying - play
c) Does it sound like science fiction?
c) study - plays
d) Do you like the sound of science fiction?
d) study - are playing
e) Does science fiction have sound?
e) studies - is playing

PROMILITARES.COM.BR 199
SIMPLE PRESENT

03. (UFRGS 1998) “Tess of the D'Urbervilles is the story of the seduction, Quais os verbos que devem preencher as lacunas II a III respectivamente?
betrayal, and destruction of an innocent girl, Tess Durbeyfield, who is a) are rising - raise
led by her foolish parents into thinking she comes from an ancient
noble family, the D'Urbervilles. Encouraged to claim Kingship with the b) are raising - rise
family, Tess is seduced by the suave, plausible Alec D'Urberville, who c) are rising - rise
abandons her when she bears his baby. The child dies, and Tess finds a d) is raising - raise
new love with the egotistic, self-righteous Angel Clare. When he hears
e) is rising - rise
her story on their wedding night, he too abandons her. In despair, Tess
murders Alec. She 1finds a few fleeting days of happiness with Clare,
who returns to her before she is captured and hanged. In the famous 08. (MACKENZIE) Indicate the alternative that best completes the
last lines of the novel, which could fit any other of Hardy's works following sentence: “She __________________ his proposal, but she
almost as well, ‘Justice’ was done, and the President of the Immortals _______________ a decision for a while”.
... had ended his sport with views through compassionate eyes the a) considers – doesn't need to make
difference between the fate human beings deserve and the one that b) is considering – doesn't want to make
they suffer."
c) has considered – had to take
The correct verbal forms of the nouns "seduction", "betrayal", and
d) has been considering – is taking
"destruction" are:
e) considered – needs to take
a) seduce - betray - destroy. d) seduct - betrayal - destruct.
b) seduct - betray - destroy. e) seduce - betray - destruct.
09.
c) seduce - betrayal - destruct. SHALL WE DANCE?
planets SPIN.
04. Complete with Simple Present or Present Continuous (Progressive):
lightning leaps.
“Today he _______________ jeans and T-shirts, but he usually
_______________ a suit at work”. atoms dance.
a) is wearing - wears d) wear - are wearing and so do we.
b) wears - is wearing e) has wearing - wearing Skirts bloom at a square dance in Albany, Oregon.
c) wearing - wear "It's friendship set to music," says Marilyn Schmit, who met her
husband on a square dance date 16 years ago.
05. (G1 1996) Assinale a alternativa que preenche corretamente as (Cathy Newman NATIONAL GEOGRAPHIC SENIOR
WRITER NATIONAL GEOGRAPHIC - JULY 2006)
lacunas: “He ____________ pudding. Why __________ him something
else?”.
From the first kick of a baby's foot to the last "Anniversary
a) don't like - do you offer Waltz," we dance - to internal rhythms and external sounds. Before
b) doesn't likes - don't you offer the written word, humans spoke the language of dance. It's as ancient
as the 3,400-year-old image of a man with a lute, dancing on a clay
c) doesn't like - don't you offers
plaque discovered in the Middle East.
d) doesn't like - don't you offer
We dance, not just with our bodies, but from the heart. "Dance
e) aren't like - do not you offer is bodies sounding off," says Judith Lynne Hanna, an anthropologist
at the University of Maryland. We pour out love and hate, joy and
06. (EFEI) Assinale a alternativa correta: sorrow; appeal to the spirits, gods, and nature; flirt, seduce, court;
a) Isadora eats never breakfast quickly. celebrate birth, death, and everything in between. We even presume
to reorder the world, as if, in the Shaker song, by "turning, turning
b) Isadora never eats her breakfast quickly.
we come round right." Dance is so profane, some religions ban it; so
c) Isadora never quickly eat her breakfast. sacred, others claim it.
d) Isadora never eats her quickly breakfast. Dance in America can hardly contain itself. We dance - from Florida
e) Isadora never eat her breakfast quickly. to Alaska, from horizon to horizon and sea to sea, in the ballrooms
of big cities and whistle-stop bars, in Great Plains Grange halls,
07. (ITA 1997) Leia. church basements, barrio nightclubs, and high school auditoriums.
3We do the polka, waltz, fox-trot, tarantella, jitterbug, samba, salsa,
BRAZIL'S NETWORK BOOM
rumba, mambo, tango, bomba, cha-cha, merengue, mazurka, conga,
BRAZIL IS ON THE VERGE OF A NETWORK SURGE. BUT EXACTLY Charleston, two-step, jerk, swim, Watusi, twist, monkey, electric slide,
HOW THEY'LL ALL IS STILL UP IN THE AIR. Harlem shake, shim sham shimmy, fandango, garba, gourd dance,
Probably the only thing that Brazil's two pay TV heavyweights, corn dance, hora, hopak - as if our lives depended on it. Some believed
Globo and TVA, agree ___(I)___ is that the country's multichannel just that: A medieval superstition averred that dancing in front of Saint
business is on the verge of a boom. Vitus's statue ensured a year of good health.
The two companies, which have fought one of the most We dance out of anguish, to attain solace, and, sometimes, in an
IMPASSIONED battles for dominance to be found anywhere in the attempt to heal. "I remember a couple," says Lester Hillier, owner of
pay TV world, ___(II)___ the intensity of their cable and wireless a dance studio in Davenport, Iowa. The husband was a retired farmer.
competition and extending it to direct-to-home television this year. His wife, a housewife, wore flat shoes and a floral housedress. "One
And with the number of Brazilian pay TV subscribers expected to ___ of their sons had been killed. The devastated parents had a dance
(III)___ fivefold to 5 million by the end of the decade, both sides are lesson booked the day after it happened. They insisted on coming
FEVERISHLY putting together new programming services to make their anyway," Hillier recalls.
packages as ALLURING as possible. (...) As the lesson hour drifted to a close, the couple asked for one last
(Ian Katz. Multichannel News. International, April, 1996) dance. They wanted a waltz. And when it ended, she rested her head
on his chest; he wrapped his arms around her shoulders. Then they
stood still, clinging to one another.

200 PROMILITARES.COM.BR
SIMPLE PRESENT

Dance, like the rhythm of a beating heart, is life. It is, also, the a) have c) do e) mustn’t
space between heartbeats. It is, said choreographer Alwin Nikolais, b) don’t d) must
what happens between here and there, between the time you start
and the time you stop. "It is," says Judith Jamison, artistic director of
05. (EFOMM) “John normally _____ a drink before meals, but now he
the Alvin Ailey American Dance Theater, 4"as close to God as you are
____ a tomato juice”.
going to get without words."
a) has, is having d) is having, have
To dance is human. To dance is divine.
Glossary:
b) is having, has e) had, have
anniversary - celebração de bodas c) have, is having
lute - alaúde (instrumento musical de cordas)
clay - argila
averred - afirmava 06. (EFOMM) Mark the correct option to complete the sentence below.
solace - consolo
“I’ve just finished reading a short story called “Dangerous”. It’s about
The present tense of the verbs in the text subtitle ("Planets spin [...] a woman who _____ her husband because she doesn’t want to lose
and so do we") is used to express him”.
a) future events. d) unexpected actions. a) was killing c) kill e) kills
b) non-repeated actions. e) permanent truths b) killed d) is killing
c) temporary agenda.
07. (EN) Which of the alternatives below completes the sentence correctly?
10. (UFAL 1999) Leia. “After the referee __________ the whistle, the match will start”.
CADBURY WORLD a) is blowing d) blowing
"Cadbury world is a chocolate theme park inaugurated in 1991. b) blows e) does blows
IT ____(I)____ half a million visitors every year and combines an
exhibition with a brief factory tour. The exhibition opens with Mayan c) blew
and Aztec use of cocoa and the "discovery" by the Europeans of the
"New World" together with the spicy drink known as chocolate. The 08. (EFOMM) “The initials V.I.P. ________Very Important Person”.
exhibition tells the arrival of cocoa in England as a luxury drink, the a) means d) accounts for
establishment of John Cadbury's shop in Bull Street, Birmingham,
b) is about e) stand for
in the 1830s, selling tea, coffee and chocolate, the discovery by his
sons of a new process (II) cocoa and the establishment of a flourishing c) describes
cocoa and chocolate business in the 1870s. Cocoa became a popular
family drink, marketed by Cadbury as "absolutely pure, therefore 09. (EN) Which sequence best completes the sentence below?
best." “When you stay ______ after midnight, your attention surely _______”.
Preencha corretamente a lacuna (I) do texto: a) woken – dropped d) awake – drops
a) is going to attract d) has attracted b) awake – has dropped e) woken – drops
b) attracted e) was attracted c) awoken – has dropped
c) attracts
10. (AFA) Marque a alternativa correta.
I. From 8 to 10 p.m. frequently Robert studies.
EXERCÍCIOS DE II. Robert frequently studies from 8 to 10 p.m.

COMBATE III. Robert frequently from 8 to 10 p.m. studies.


IV. Robert studies frequently from 8 to 10 p.m.

a) Only sentence I is correct.


01. (EN) “Water ____ at 100 degrees Centigrade”.
b) Only sentence III is wrong.
a) boil c) to boil e) are boiling
c) Sentences III and IV are correct.
b) boils d) boiling
d) Only sentence II is correct.

02. (EPCAR) Indicate the alternative that best completes the following
sentence: “She __________ German very well.” GABARITO
a) speaks and writes c) speak and writes EXERCÍCIOS DE FIXAÇÃO
b) speaks and write d) speak and write 01. D 04. E 07. E 10. A
02. B 05. A 08. A
03. (EFOMM) “Every time she ____ to school she says hello to her
03. D 06. A 09. A
friends and ____ to her classroom”.
EXERCÍCIOS DE TREINAMENTO
a) get, go d) has got, have gone
01. E 04. A 07. B 10. C
b) got, goes e) will get, had gone
02. C 05. D 08. B
c) gets, goes
03. A 06. B 09. E
04. (EN) Which of the sentences below completes the dialogue EXERCÍCIOS DE COMBATE
correctly? 01. B 04. B 07. B 10. D
Paul: Do you have to take him home tonight? 02. A 05. A 08. E
James: In fact, I _____ . He is taking a taxi. 03. C 06. E 09. D

PROMILITARES.COM.BR 201
SIMPLE PRESENT

ANOTAÇÕES

202 PROMILITARES.COM.BR
NOUNS, PLURAL AND GENDER

FORMAÇÃO DOS SUBSTANTIVOS Alguns sufixos formam substantivos abstratos:


Os substantivos são usados para identificar ou dar nome às • TION
pessoas, coisas e qualidades que nos circundam. Exemplos:
Exemplo: information, situation, solution, definition, promotion
student, man, house, sky, Monday, France, hope, information.
• SION
Substantivos com sufixos que formam algumas profissões: Exemplos:
• R explosion, persuasion, invasion, conclusion, decision,
Exemplos: conversion, recession
baker, builder, designer , manager, speaker, teacher
• MENT
• ER Exemplos:
Exemplos: amusement, judgment, excitement, argument, statement,
arrangement
engineer, footballer, gardener, lawyer, photographer,
trumpeter, butcher, carpenter, grocer, plumber, usher
• NESS
Exemplos:
• OR
sadness, readiness, usefulness, redness, business
Exemplos:
actor, author, director, doctor, editor, professor, solicitor,
surveyor, tailor • ANCE - ENCE - ANCY - ENCY
Exemplos:
independence, attendance, accountancy, efficiency,
• IST nuisance, emergency, conscience
Exemplos:
cellist, pianist, violinist, geologist, physicist, economist, • ABILITY - IBILITY
scientist, chemist, artist
Exemplos:
probability, respectability, possibility, responsibility
• IAN
Exemplos: • ISM
mathematician, musician, politician, statistician, historian, Exemplos:
librarian, technician
classicism, communism, realism, liberalism, socialism,
romanticism
• ANT
Exemplos: Outros sufixos que também formam substantivos.
accountant, assistant, attendant, consultant, informant, • AL
inhabitant
Exemplos:
arrival, committal, denial, dismissal, proposal, refusal,
• ENT withdrawal
Exemplos:
resident, superintendent • DOM
Exemplos:
kingdom, wisdom, random, condom

PROMILITARES.COM.BR 203
NOUNS, PLURAL AND GENDER

• HOOD
king queen rei / rainha
Exemplos:
likelihood, neighborhood, childhood, fairhood nephew niece sobrinho(a)

son daughter filho(a)


• TH
uncle aunt tio(a)
Exemplos:
breadth, depth, length, width widower widow viúvo / viúva

wizard witch feiticeiro(a), bruxo(a)


GENDER OF NOUNS -
ram/sheep ewe carneiro macho/fêmea
GÊNERO DOS SUBSTANTIVOS
Em alguns casos, o feminino em inglês é formado a partir da husband wife esposo / esposa
adição do sufixo -ESS ao substantivo masculino.

MASCULINO FEMININO TRADUÇÃO Alguns substantivos apresentam apenas uma forma, tanto
para o masculino quanto para o feminino.
author authoress autor(a)
MASCULINO/FEMININO TRADUÇÃO
baron baroness barão/baronesa
artist artista
count countess conde/condessa
child criança/filho(a)
heir heiress herdeiro(a)
cook cozinheiro(a)

Em alguns casos, há mudanças na ortografia. cousin primo(a)

dancer dançarino(a)
MASCULINO FEMININO TRADUÇÃO
driver motorista
actor actress Ator/atriz
journalist jornalista
Embaixador/
ambassador ambassadress
embaixatriz neighbor vizinho(a)
duke duches Duque/duquesa painter pintor(a)
emperor empress Imperador/imperatriz baby bebê
prince princess Príncipe/princesa parent pai/mãe
waiter waitress Garçom/garçonete person pessoa

prisoner prisioneiro(a)
Alguns substantivos masculinos apresentam formas irregulares
de feminino. reporter repórter

singer cantor(a)
MASCULINO FEMININO TRADUÇÃO
teacher professor(a)
bachelor spinster solteiro / solteira

bull cow touro / vaca


Para distinguir o gênero dos substantivos que designam
cock/rooster hen galo / galinha a maioria das espécies de animais, utilizam-se as formas male
(macho) e female (fêmea)
dog bitch cão / cadela
MASCULINO FEMININO TRADUÇÃO
drake duck pato(a)
male elephant female elephant elefante/elefanta
drone bee zangão / abelha
male bear female bear urso(a)
fox vixen raposa macho/fêmea
zebra macho/
friar/monk nun frade, monge / freira male zebra female zebra
fêmea
horse/stallion mare cavalo / égua

204 PROMILITARES.COM.BR
NOUNS, PLURAL AND GENDER

THE PLURAL OF NOUNS - Substantivos terminados em –Y, precedidos de consoante,


perdem o -Y e acrescenta-se -IES para formar o plural.
PLURAL DOS SUBSTANTIVOS
Como regra geral, o plural em inglês é formado a partir da adição de -S. SINGULAR PLURAL TRADUÇÃO

SINGULAR PLURAL TRADUÇÃO baby babies bebê(s)

author authors autor(es) country countries país(es)

book books livro(s) fly flies mosca(s)

computer computers computador(es) lady ladies senhora(s)

girl girls menina(s), garota(s) spy spies espião(ões)

student students aluno(s)


Substantivos terminados em –Y, precedidos de vogal, acres-
window windows janela(s) centa-se -S para formar o plural.

SINGULAR PLURAL TRADUÇÃO


Substantivos terminados em -CH, -SH, -S, -SS, -X e –Z
acrescentamos –ES para formar o plural.
day days dia(s)
SINGULAR PLURAL TRADUÇÃO
key keys chave(s)
match matches fósforo(s)

bus buses ônibus boy boys garoto(s)

kiss kisses beijo(s)


guy guys rapaz(es)
fox foxes raposa(s)

topaz topazes topázio(s) Observação

bush bushes arbusto(s) Os nomes de família terminados em -Y formam seu plural


acrescentando apenas -S, independentemente de o -Y aparecer
precedido de consoante ou vogal.
Substantivos terminados em –O com plural –ES Exemplo: The Kennedys, The Caseys etc.

SINGULAR PLURAL TRADUÇÃO


Alguns substantivos terminados em -F ou -FE perdem o -F ou o -FE
cargo cargoes cargo(s) e acrescenta-se -VES para formar o plural.

domino dominoes dominó(s) SINGULAR PLURAL TRADUÇÃO


echo echoes Eco(s)
calf calves bezerro(s)
hero heroes herói(s)
half halves metade(s)
potato potatoes batata(s)
knife knives faca(s)
tomato tomatoes tomate(s)
leaf leaves folha(s)

Terminados em –O com plural –S life lives vida(s)


SINGULAR PLURAL TRADUÇÃO
loaf loaves pão(pães)
memo memos memorando(s)
photo photos foto(s) self selves ego(s)
video videos vídeo(s)
shelf shelves prateleira(s)
kilo kilos quilo(s)
piano pianos piano(s) thief thieves ladrão(ões)

dynamo dynamos dinamo(s) wife wives esposa(s)


solo solos solo(s)
studio studios estúdio(s) wolf wolves lobo(s)

PROMILITARES.COM.BR 205
NOUNS, PLURAL AND GENDER

Nota: Substantivos de origem latina ou grega formam o plural


Os substantivos dwarf (anão), wharf (cais), hoof (casco), das seguintes formas:
handkerchief (lenço) e scarf (cachecol) podem formar o plural tanto
pelo acréscimo de -VES, como acrescentando apenas -S (regra
• Terminados em -US
geral).
Exemplo: SINGULAR PLURAL LATINO
dwarf- dwarves/dwarfs
wharf- wharves/wharfs cactus cacti

hoof- hooves/hoofs fungus fungi


handkerchief- handkerchieves/handkerchiefs
radius radii
scarf- scarves/scarfs
stimulus stimuli
Os demais substantivos terminados em -F, -FE ou -FF seguem
a regra geral para formar plural, acrescentando apenas -S.
• Terminados em -A
SINGULAR PLURAL TRADUÇÃO
SINGULAR PLURAL LATINO
roof roofs telhado(s)
alga algae
chief chiefs chefe(s)
formula formulae
gulf gulfs golfo(s)
larva larvae
cliff cliffs penhasco(s)
vertebra vertebrae
safe safes cofre(s)

belief beliefs crença(s) • Terminados em -UM, -ON

SINGULAR PLURAL LATINO


Alguns substantivos têm formas irregulares de plural.

SINGULAR PLURAL TRADUÇÃO addendum addenda

child children criança(s) bacterium bacteria


die dice dado(s) (jogo)
curriculum curricula
foot feet pé(s)

goose geese ganso(s) datum data

louse lice piolho(s) erratum errata


man men homem(ns)
medium media
mouse mice camundongo(s)

tooth teeth dente(s) memorandum memoranda

um centavo criterion criteria


penny pence/pennies
centavos

woman women mulher(s) phenomenon phenomena

ox oxen boi(s)
polyhedron polyhedra

Nos compostos separados por hífen, o componente


substantivo é que recebe o sufixo de plural. • Terminados em -EX, -IX

SINGULAR PLURAL TRADUÇÃO SINGULAR PLURAL LATINO

brother-in-law brothers-in-law cunhado(s) index indices

looker-on lookers-on espectador(es) matrix matrices

appendix appendices

206 PROMILITARES.COM.BR
NOUNS, PLURAL AND GENDER

• Terminados em -IS
phonetics Fonética

SINGULAR PLURAL LATINO physics Física

analysis analyses politics Política

axis axes
Exemplo:
basis bases The ethics is important in the world business.
crisis crises
Alguns substantivos são tipicamente utilizados apenas no
diagnosis diagnoses plural, concordando com o verbo no plural.
hypothesis hypotheses
PLURAL TRADUÇÃO
oasis oases
clothes roupa(s)
parenthesis parentheses
goods mercadoria(s)
synopsis synopses
jeans jeans
thesis theses
people pessoas

Substantivos incontáveis apresentam apenas forma de police polícia


singular, logo concorda com o verbo no singular.
blues tristeza(s)
SINGULAR TRADUÇÃO fish peixe(s)
coffee café(s) sheep carneiro(s)
joy alegria(s) contents conteúdo
pride orgulho(s) pants calças
news notícia(s) riches riqueza(s)
information informação(ões) scissors tesoura

Exemplo: trousers calças

The news is spread all over the world. cattle gado

clergy clero
Substantivos que terminam em -S, mas estão no singular,
concordam com o verbo no singular. dice dados

SINGULAR TRADUÇÃO binoculars binóculo

news Notícia glasses óculos

measles Sarampo pliers alicate

mumps Caxumba scales balança

billiards Bilhar spectacles óculos


bowls Boliche suspensórios/aparelho
braces
ortodôntico
dominoes Dominó

draughts/checkers Damas pyjamas pijama

athletics Atleta shorts calção

ceramics Cerâmica arms arma

classics Clássico customs alfândega

economics Economia earnings salário


ethics Ética stairs escada
gymnastics Ginástica lodgings quarto para aluguel
mathematics Matemática

PROMILITARES.COM.BR 207
NOUNS, PLURAL AND GENDER

Exemplo: • ED
The pants are in the laundry. Exemplos:
Nota: disappointed, embarrassed, worried, satisfied, fascinated,
reserved
A palavra “people” só vai para o plural com –S quando se referir
a povo(s).
Exemplo: Prefixos mais comuns:
The native peoples of Philippines and their problems.
PREFIXO SIGNIFICADO EXEMPLOS
ADJECTIVES - ADJETIVOS
Os adjetivos em inglês são usados geralmente antes dos anti-government,
anti- contra
anti-racist, anti-war
substantivos.
Exemplo:
wrong place, big house, small room, nice girl, tall man, thin autobiography,
auto- próprio
boys, black cars automobile

declassify,
Alguns dos principais sufixos que formam adjetivos: de- reverso, contrário decontaminate,
• Y demotivate
Exemplos:
healthy, cloudy, hungry, dirty, easy, funny, noisy, lucky disagree, displeasure,
dis- reverso, contrário
disqualify
• LY
Exemplos: downgrade,
down- reduzir, diminuir
brotherly, cowardly, fatherly, leisurely, friendly, monthly, downhearted
lively
extraordinary,
• LIKE extra- além, a mais
extraterrestrial
Exemplos:
businesslike, childlike, godlike, lifelike, workmanlike hyperactive,
hyper- extremo
hypertension
• ISH
Exemplos: illegal, impossible,
il-, im-, in-, ir- negação
childish, Spanish, foolish, reddish, Danish, snobbish insecure, irregular

• FUL
Exemplos: inter- entre interactive, international
beautiful, shameful, careful, skillful, doubtful, useful,
wonderful, harmful, hopeful
muito grande, megabyte, mega-deal,
mega-
importante megaton
• LESS
Exemplos:
aimless, meaningless, careless, pointless, useless, hopeless, midday, midnight,
mid- no meio, meio
lifeless mid-October

• ABLE incorreto, misaligned, mislead,


Exemplos: mis-
contrário misspelt
acceptable, considerable, eatable, readable, recognizable,
respectable, believable
non-payment,
non- negação
non-smoking
• IVE
Exemplos:
administrative, explosive, intensive, possessive, destructive, overcook, overcharge,
over- muito, demais
progressive overrate

• ING outdo, out-perform,


Exemplos: out- ir além, demais
outrun
daring, shocking, exciting, charming, boring, understanding

post- após post-election, post-war

208 PROMILITARES.COM.BR
NOUNS, PLURAL AND GENDER

05. (FATEC) Assinale a alternativa que só apresenta formas de


PREFIXO SIGNIFICADO EXEMPLOS feminino:
a) actress - baroness - wife d) father - actor - brother
pre- antes prehistoric, pre-war b) sister - granddaughter - son e) maid - wife - bull
c) baby - artist - cousin
pro-communist,
pro- a favor
pro-democracy 06. (UEL) Uma das alternativas abaixo apresenta somente formas de
masculino. Qual?
re- novamente reconsider, redo, rewrite
a) brother - son - actress d) queen - mare - vixen
b) niece - uncle - boy e) bachelor - fox - stewardess
semi- metade semicircle, semi-retired
c) gentleman - husband - king

sub- sob, abaixo submarine, sub-agent 07. (CESGRANRIO 1995) Most names of jobs are used for both men
and women. Mark the one that refers only to males:
super- acima, além de superhero, supermodel a) psychiatrist d) doctor
b) social scientist e) researcher
tele- a distância television, telepathic c) waiter

trans- através transatlantic, transfer 08. (UEPB 2014)


HOW MONEY WORKS: WILL CHINA ON US ALL?
ultra-compact, It’s no secret China has been booming while the West declines.
ultra- extremamente
ultrasound In fact, it’s been growing so fast it’s expanding overseas, too: buying
up businesses in the UK, U.S. and elsewhere. So, how worried should
remover, reverso, undo, unpack,
un- we be?
não unhappy, unhealthy
Napoleon once said, apparently. “Let China sleep because when
undercook, she wakes she’ll shake the world”.
under- menos que, abaixo
underestimate Indeed, for much of the industrial revolution, China was taking
a nap — so to speak. But in 1978 things began to change. The
Communist country encouraged private enterprise and unleashed its
biggest asset: 975 million citizens.
EXERCÍCIOS DE

FIXAÇÃO
Where then ensued mass migrations to urban areas where people
took up jobs in factories to manufacture goods for export. Since then
the economy dubbed ‘the dragon’ has doubled its slice of the global
economy and it’s predicted that by 2016 China will be the world’s
biggest economy.
01. (EEAR) – Mother to small son: “Be sure to wash your arms before
you put on your new shirt.” Can anything stand in the way of the Asian powerhouse?
– Small son: “Should I wash for long or short sleeves?” (From Yahoo Finance UK Friday Mar 8, 2013.)
All words below describe male family members, except:
Which group of words from text consists ONLY of nominal groups?
a) son c) mother
a) the industrial revolution - the Communist country - the global
b) uncle d) grandfather economy - the Asian powerhouse
b) buying up - the Communist country - economy dubbed - the
02. (EEAR) “Good day! My name is Sheila. I’m from Melbourne, global economy
Australia. My _______ is from Montreal, Canada. We live in Sydney. A
lot of _______ living in Australia come from other _______”. c) the industrial revolution - to manufacture goods - buying up - the
Asian powerhouse
Choose the best alternative to complete the blanks in the text:
d) the Asian powerhouse - the global economy - buying up - look up
a) husband – peoples – country
e) the Asian powerhouse - the Communist country - the industrial
b) husband – people – countries revolution - took up
c) husbands – persons – country
d) husbands – person – countries 09. (UNIOESTE 2012) Mark the noun that does NOT have the same
plural form as in residents:
03. (MIN.AER.) “Jennifer, _____, didn’t do a good job, but the ____ a) criminals c) women e) dealers
was very well in that movie”. b) forces d) slums
a) the actriss - artist c) the actress - actor
b) the actrix - author d) the actoress - actor 10. (EEAR) Choose the best alternative that shows the irregular plural
form:
04. (EPCAR) Mark the option that presents a mistake of gender. a) Mice – children – goose – woman
a) My niece is called Mary. b) Mice – children – geese – women
b) Prince Diana died in car accident. c) Mouses – kids – goose – women
c) My uncle is a good host. d) Mice – child – geese – woman
d) My father is a very wealthy man.

PROMILITARES.COM.BR 209
NOUNS, PLURAL AND GENDER

EXERCÍCIOS DE 15
Then, of course, there is the coconut 16which, when cut in two

TREINAMENTO and dried 18in the sun, produces oil. 5Plait three 6blades of grass and
13

20
dip 19into this oil, 14light... And you have a lamp. A lamp which not
so very long ago was still used 17throughout the islands”.
Nouns in English can be divided into "countable" or "uncountable"
01. (MACKENZIE) Assinale a alternativa em que aparecem somente (e.g.: apple X water). In order to indicate some kind of "measurement"
formas de feminino: in the case of uncountable nouns, another noun is required (e.g.:
"glasses" or "liters" of water).
a) mother - sister - wife - bachelor
Accordingly, the expression below that is equivalent to the structure
b) girl - aunt - Miss - spinster "blades of grass" (ref. 6) is
c) widow - brother-in-law - son-in-law - mother-in-law a) structures of steel. d) floors of wood.
d) father - son - uncle - doctor b) classes of Chinese. e) letters of complaint.
e) baroness - countess - heir - actress c) cups of coffee.

02. (UFRGS 2015) 04. (UERJ 2004)


The study of game 13aesthetics is a very recent practice, spanning BODY DECORATION
less than two 7decades. Unlike game studies in 4mathematics or
the social sciences, wich are much older, games became subject To look fiercer, brave, attractive, powerful and wealthy, people of
to humanistic study only after computer and video games became different cultures all over the world decorate their bodies and faces.
popular. This lack of persistent interest might seem 15odd, but only if Tattooing is one of the oldest known forms of decorating the
we see traditional games and computer games as intrinsically similar, human body. Egyptians tattooed the dead to protect them against evil
wich 19they are not. We might try to explain this lack by noting that spirits. Mummies from 2000 B.C. have been found with tattoos, and
games are usually seen as trivial and 16lowbrow by the aesthetic and anthropologists have even found prehistoric tattooed men.
theoretical elites who cultivate the 5analysis of artistic 14media objects: Some of the native Polynesian people of New Zealand, known
literature, the visual arts, theatre, music, etc. But this does not explain as the Maori, wore very intricate full facial tattoos. Each one was so
the fact that aesthetic studies of games are now not only possible, unique it was like a thumbprint. Many years ago, when Maori chiefs
but even encouraged and supported with funding. What happened signed documents and deeds with Europeans, they drew pictures of
to cause this change? their face patterns as a signature.
A possible explanation could be that digital games, unlike The Japanese also use tattooing. They consider it an art form, and
traditional games or sports, consist of non-ephemeral content (stored preserve the works of great tattoo masters in museums. Sometimes
words, sounds and images), which 8places them much closer to the people are like peacocks, dressing up to attract that special someone's
ideal object of the 9Humanities, the work of art. Thus, they become attention.
visible and textualizable for 29the aesthetic observer, in a way the
Among the Fulani nomads in Nigeria, the men compete to lure
6
previous 10phenomena were not.
girls by getting all done up in jewelry, painting wild patterns on their
However, 21this sudden visibility, probably also caused by the faces and wearing colorful beaded costumes. They also exaggerate
tremendous economic and cultural 12success of computer games, their eyes and make wild facial expressions.
produces 26certain blind spots in the aesthetic observer, especially
In our contemporary society, some people pierce their ears, have
if 18he/she is trained in textual/visual analysis, as is usually the case.
their noses made smaller, have their teeth straightened or capped or
Instead of treating 22the new phenomena carefully, and as 24objects of
made whiter, get hair implants and extensions, have fat sucked off
30
a study for which no methodology yet exists, they are analyzed with
their hips or have parts of the body plumped up. All in the pursuit of
tools that happen to be at hand, such as 27film or narrative theories.
the ultimate personal statement. I guess beauty truly is in the eye of
Therefore 28we need to outline and promote 23a methodology for
the beholder.
25
the aesthetic study of games, 20which, 31given the current 17nascent
(http://www.zuzu.org)
state of the field, 32will doubtlessly give way to more sophisticated
11
approaches in the years to come.
The title "Body Decoration" combines two nouns.
(Adapted from: Aarseth, Espen. Playing Research: Methodological approaches to
game analysis. Available at: <http://hypertext.rmit.edu.au/dac/ papers/Aarseth.pdf>. The same pattern is also found in the following pair:
Accessed on July 26th, 2014.) a) tattooed men and art form.
Select the alternative which presents only nouns in their plural forms. b) tattoo masters and hair implants.
a) mathematics (ref. 4) – analysis (ref. 5) – previous (ref. 6) c) hair extensions and facial tattoos.
b) decades (ref. 7) – analysis (ref. 5) – places (ref. 8) d) face patterns and different culture.
c) humanities (ref. 9) – phenomena (ref. 10) – approaches (ref. 11)
05. (FATEC 1998) Considere o texto.
d) places (ref. 8) – success (ref. 12) – approaches (ref. 11)
JAPAN
e) aesthetics (ref.13) – mathematics (ref.4) – media (ref. 14)
Sick TV
03. (PUC-RS 2007) DID HUNDREDS OF JAPANESE KIDS suffer a TV-induced epileptic
attack week - or were many simply petrified by a really scary mouse?
COCONUT TREE, COCO PALM
That's the question doctors and psychologists were trying to answer
“A thousand 7years ago, the coconut 8tree did not even exist after more than 700 Japanese, most of them children, were rushed to
in Tahiti. 1It was the pioneering Polynesians who 9first brought this hospitals after viewing an episode of the hugely popular cartoon show
21
plant with them in their migrations. A 2tree of 10life in every sense of "Pokemon", or "Pocket Monster". Some suffered convulsions, others
the 11phrase, its nut 12supplies water, milk and 3edible pulp; its "heart" arrived dizzy and nauseated. The apparent trigger was an onscreen
is eaten in salads; its 22trunk serves as 23framework for Tahitian 24huts, "explosion", followed by several seconds of flashing red lights
and its palms are 4woven as 25roofing. from the face of ratline Pikachu, the show's star character. Doctors

210 PROMILITARES.COM.BR
NOUNS, PLURAL AND GENDER

struggled to pinpoint the cause of the seizures, citing everything from 10. (JFS) Complete as sentenças a seguir utilizando as formas corretas
"optically induced epilepsy" to group hysterics. Japanese television de cada vocábulo apresentado:
executives were equally confused. "Those types of scenes are seen – He bought a _____________car.
on many animation programs", said one of the show's producers. TV
– She is a _____________ rock star.
Tokyo, which broadcast the cartoon, has pulled "Pocket Monsters".
(NEWSWEEK, JANUARY 5, 1998.) – I have good _______________ to give you.
– The police _______________ in front of the building.
Na 1ª linha do texto lê-se: "Did hundreds of Japanese KIDS suffer a
a) sport – famous – informations – is
TV-induced epileptic attack last week- [...]".
b) sport – famous – informations – are
Indique a alternativa em que todas as palavras têm a mesma forma de c) sports – famous – informations – are
plural de "KIDS".
d) sport – famouses – information – is
a) attack - mouse d) show - child
e) sports – famous – information – are
b) episode - Japanese e) explosion - Japanese
c) cartoon - trigger
EXERCÍCIOS DE

COMBATE
06. (MACK) Which alternative shows the correct plural form of the
words given?
a) mouse/mice - goose/geese - phenomenon/phenomena - deer/deer
b) mouse/mices - chick/chicken - person/persons - child/children
c) mouse/mouses - goose/geeses - deer/deers - news/news 01. (AFA) “The _______ was very elegant last night. She is the
d) mouse/mouses - new/newses - bus/buses - person/people director’s ______”.
e) mouse/mises - child/children - police/polices - news/news a) wife - spinster c) steward - host
b) husband - wife d) lady - wife
07. (PUC-RIO) Check the only word that could be used in the plural
form: 02. (AFA) In the following alternatives, all the nouns are feminine,
a) information d) energy EXCEPT:
b) land e) employment a) heiress - jewess - empress - mare
c) research b) spinster - niece - vixen - hen
c) daughter - cock - witch - aunt
08. (PUC-PR) Match the columns below so that the words in the d) lioness - cow - ewe - bee
second column fit the sentences provided in the first one:
1. Tom and Mary love their ___________. 03. (EFOMM) Choose the option that contains only nouns.
2. Put the oranges inside those __________. a) absentee - craftsmanship - golden - confusion
3. I can't walk. My __________ are aching a lot. b) assistant - friendliness - kindness - frosty
4. The __________ are flying south. c) adhesive - mindful - backwards - slowly
5. The cat is hunting the __________. d) wisdom - amazement - ammunition - cruelty
6. When Jane fell over, she broke two of her __________. e) heroic - wooden - poetic - clockwise
( ) feet ( ) geese
( ) teeth ( ) mice 04. (PUC-CAMP) Quais os substantivos que correspondem aos verbos
“breathe”, “prescribe”, “disappear”?
( ) children ( ) boxes
a) breath – prescription - disappearance
Choose the correct alternative: b) breth – prescribing – disappearance
a) 6 – 3 – 1 – 4 – 5 – 2 c) breeth – prescription – disappearment
b) 3 – 6 – 1 – 4 – 5 – 2 d) braeth – prescrivition – dispparition
c) 3–6–2–4–5–1 e) breathingly – prescition – disparition
d) 5 – 3 – 4 – 2 – 6 – 1
e) 3 – 6 – 4 – 1 – 2 – 5 05. (EFOMM) In “[…] and despite an unresolved lawsuit that has
delayed the project (...)”, the word in bold is formed by the prefix un.
09. (JFS) Give the correct plural form of the words: “buffalo – eskimo In which option below the word is not correctly formed by the same
– concerto – person – manservant – thesis – sister-In-Law – Die”. prefix?
a) buffalos - eskimos - concertos - personas - menservants - theses - a) unfinished d) undecided
sisters-in-law - dice b) unfasten e) unpolite
b) buffaloes - eskimos - concertos - persons - menservants - theses - c) unwrapped
sisters-in-law - dice
c) buffaloes - eskimos - concertoes - persons - manservants - theses 06. (EFOMM) The prefix un as in unclear is used correctly in all
- sisters-in-law - dice alternatives EXCEPT:
d) buffaloes - eskimos - concertos - persons - menservants - theses - a) unconscious. d) undetermined.
sister-in-laws - deaths b) uninteresting. e) unexpensive.
e) buffalos - eskimoes - concertoes - personas - manservants - thesis c) unfold.
- sister-in-laws - deaths

PROMILITARES.COM.BR 211
NOUNS, PLURAL AND GENDER

07. (AFA) Mark the option which shows the appropriate plural form
for the word “phenomenon”. ANOTAÇÕES
a) phenomenae
b) phenomena
c) phenomenons
d) phenomenos

08. (AFA) “I’m still doubtful about the characters of the new novel.
You probably agree that the suspects reactions were totally unexpected
as their explanations didn’t make their intentions clear enough in the
testimony. We could first deduce that the fake nun was the prime
suspect for having poisoned the stallion and the sow. On the other
hand, the cunning steward and the wily widower had extravagant
posture after the judgment”.
Considering the underlined words in the text, you could say that:
a) two words are female.
b) three words are female.
c) all of words are male.
d) just one word is female.

09. (ITA) Dadas as afirmações de que o plural de:


1. BASIS é BASIS
2. DATUM é DATAS
3. BUSINESS é BUSINESSES

Constatamos que está(estão) correta(s):


a) apenas a afirmação nº 1.
b) apenas a afirmação nº 2.
c) apenas a afirmação nº 3.
d) apenas as afirmações nº 1 e nº 3.
e) todas as afirmações.

10. (ITA) Dadas as afirmações de que o plural de:


1. CHIEF é CHIEVES
2. RADIUS é RADII
3. LEAF é LEAVES

Constatamos que está (estão) correta(s):


a) apenas a afirmação nº 1.
b) apenas a afirmação nº 2.
c) apenas a afirmação nº 3.
d) as afirmações nº 2 e nº 3.
e) todas as afirmações.

GABARITO
EXERCÍCIOS DE FIXAÇÃO
01. C 04. B 07. C 10. B
02. B 05. A 08. A
03. C 06. C 09. C
EXERCÍCIOS DE TREINAMENTO
01. B 04. B 07. B 10. E
02. C 05. C 08. B
03. C 06. A 09. B
EXERCÍCIOS DE COMBATE
01. D 04. A 07. B 10. D
02. C 05. E 08. A
03. D 06. E 09. C

212 PROMILITARES.COM.BR
PERSONAL PRONOUNS

PRONOMES SUJEITOS PRONOMES OBJETOS


(SUBJECT PRONOUNS) (OBJECT PRONOUNS)
Os Pronomes Sujeitos (Subject Pronouns) são usados como sujeito Os Pronomes Objetos (Object Pronouns) substituem os objetos
da oração ou substituem pessoas ou coisas que também estão na diretos ou indiretos, e sempre devem vir após verbos ou, em alguns
função de sujeito. casos, preposições.
I – eu ME – me, mim; -migo
YOU – te; ti; -tigo; você
YOU – tu;você
SINGULAR HIM – o; lhe; ele
SINGULAR HE – ele
HER – a; lhe; ela
SHE – ela
IT – o; a; lhe; ele; ela
IT – ele/ela (animais ou objetos)

US – nos; -nosco; nós


WE – nós PLURAL YOU – vos; -vosco; vocês
PLURAL YOU – vós; vocês THEM – os; as; lhes; eles
THEY – eles
• Usados como objeto do verbo.
Exemplos:
• Pronomes pessoais usados como sujeito da oração: He still loves her.
Exemplos: I’m going to see him tomorrow.
I am very poor. Take me to your office.
Won’t you give me something else?
She sat down on the river bank. • Usados como objeto, porém depois de preposição.
They went around the world together. Exemplos:
Look at them.
• Usados para substituir termos da oração: Bernado wants to go with you.
Exemplos: Between you and me
Women never go to the movies alone. They prefer to go
with their boyfriends.
• Substituindo nomes.
The executive will feel more energetic after gym. He will
feel healthy. Exemplos:
Pass Victória the pen = Pass her the pen.
• Usados em respostas simplificadas: I’ll visit Paulo next month = I’ll visit him next month.
Exemplos: Talk to Carla and Pedro = Talk to them.
Who wants a drink? I do.
Does he live in Tijuca? Yes, he does. PRONOMES ADJETIVOS POSSESSIVOS
Is Bia an architect? No, she isn’t. (POSSESSIVE ADJECTIVES)
Os Adjetivos Possessivos (Possessive Adjectives) são usados
antes do substantivo para indicar que aquele substantivo pertence a
• O pronome “IT” é usado em várias expressões em que o alguém, e são relacionados ao sujeito da oração.
sujeito não é determinado.
Exemplos:
It is raining/snowing/freezing/windy/cloudy/cold/hot etc.
It is half past seven
It is me
It is time to go

PROMILITARES.COM.BR 213
PERSONAL PRONOUNS

MY – meu(s); minha(s) PRONOMES POSSESSIVOS


YOUR – teu(s); tua(s); seu(s); sua(s); de você
(POSSESSIVE PRONOUNS)
SINGULAR HIS – dele(s); seu(s); sua(s)
Os Pronomes Possessivos (Possessive Pronouns) são usados com o
HER – dela(s); seu(s); sua(s) intuito de substituir o substantivo, ou seja, o objeto possuído na frase.
ITS – dele(s); dela(s); seu(s); sua(s)

Estão sempre relacionados ao sujeito.


MINE – meu(s); minha(s)
OUR – nosso(s); nossa(s) YOURS – teu(s); tua(s); seu(s); sua(s); de você
PLURAL YOUR – vosso(s); vossa(s); seu(s); sua(s); de vocês SINGULAR HIS – dele(s); seu(s); sua(s)
THEIR – deles; delas; seu(s); seu(s) HERS – dela(s); seu(s); sua(s)
ITS – dele(s); dela(s); seu(s); sua(s)
• Pronomes adjetivos possessivos usados antes do
substantivo que é a coisa possuída por alguém.
Exemplos: OURS – nosso(s); nossa(s)
My car; Her room; Our bank account PLURAL YOURS – vosso(s); vossa(s); seu(s); sua(s); de vocês

   THEIRS – deles; delas; seu(s); seu(s)


Meu carro Sua sala Nossa conta bancária

Sala dela Os pronomes possessivos são usados depois do substantivo e
quando o substantivo estiver subentendido.
Their children Exemplos:
This car is yours.

Seus filhos seu / seus



Deles / Delas Paula is in her car. I am in mine.
• Os adjetivos possessivos referem-se ao sujeito da oração.
my car


Have you hurt your hand?
  This is your bike. That is hers.

her bike


She is going to clean her house.
 
Os pronomes possessivos também concordam com o sujeito.
Exemplos:
They decided to park their car here. Whose pen is this? It’s Paulo’s. It’s his.
   

Where is Carla’s book? Hers is on the floor.


 

PRONOMES REFLEXIVOS (REFLEXIVE PRONOUNS)


Os Pronomes Reflexivos (Reflexive Pronouns) são usados para expressar ação reflexiva, ou seja, indica que o sujeito da oração executou e sofreu
a ação do verbo. Pode também ser utilizado quando queremos dizer que o sujeito realizou a ação sem a ajuda de ninguém; neste caso, o pronome
reflexivo ser precedido da preposição “by”. Usa-se também o pronome reflexivo para expressar ênfase; neste caso, o pronome deve vir logo após
o sujeito, e deve combinar com a coisa ou pessoa que está enfatizando.
MYSELF – me; mesmo(a); próprio(a); mim mesmo(a)
YOURSELF – te; ti; mesmo(a); próprio(a); ti mesmo(a); você mesmo(a);
SINGULAR HIMSELF – se; si; mesmo; próprio; a ele mesmo.
HERSELF – se; si; mesma; própria; a ela mesma.
ITSELF – se; si; mesmo(a); próprio(a); a ele / ela mesmo(a).

OURSELVES – nos; mesmos(as); próprios(as); a nós mesmos(as);


PLURAL YOURSELVES – vos; mesmos(as); próprios(as); a vós mesmos(as);
THEMSELVES – se; si; mesmos(as); próprios(as); a eles / elas mesmos(as);

214 PROMILITARES.COM.BR
PERSONAL PRONOUNS

• Usados com verbos reflexivos, em que o sujeito e o REFERÊNCIA PRONOMINAL -


objeto da ação são os mesmos.
Exemplos:
PRONOUN REFERENCE
O pronome geralmente substitui outro termo.
Look at yourself in the mirror!
Nas questões de prova, a banca pergunta a que termo do texto o
We enjoyed ourselves in Maceió.
pronome se refere.
Have you hurt yourself with that knife?
Muitas vezes esse termo que o pronome se refere está anterior a
• O pronome reflexivo é usado em sentenças em que o ele, mas em alguns casos aparece depois. Os pronomes referenciais
sujeito pratica a ação por ele mesmo. podem ser todos os tipos de pronomes, como os: pronomes pessoais,
objetos, possessivos, reflexivos, demonstrativos e relativos.
Exemplos:
I’ll just get myself the ball before the game starts.
 
Uso dos pronomes referenciais.
He made himself a drink. Exemplos:
After buying the new car, João sold it.
 

I’ve just bought myself a new car.


 

She lives by herself. Victor, take the seats out of the car and fix them.
 

• Usado para enfatizar ou realçar o sujeito ou o objeto


da oração.
Exemplos:
The principal (sujeito) himself called the student.
Thales loved the book (objeto) itself, but he didn’t like the The manager told the employees that they would receive a bonus.
film.
Why don’t you like him, the student next to the window?
• Há casos em que os pronomes reflexivos são usados
com preposição e também quando as preposições são
parte do “phrasal verb”.
Exemplos:
After he had received his orders, the soldier left the barracks.
Look after yourself. / Take care of yourself.
Don’t do it for him, let him do it for himself.
She lives by herself. / He’s old enough to go to Manaus by
himself.

SUBJECT OBJECT POSSESSIVE POSSESSIVE REFLEXIVE


PRONOUNS PRONOUNS ADJECTIVES PRONOUNS PRONOUNS

I ME MY MINE MYSELF

YOU YOU YOUR YOURS YOURSELF

HE HIM HIS HIS HIMSELF

SHE HER HER HERS HERSELF

IT IT ITS ITS ITSELF

WE US OUR OURS OURSELVES

YOU YOU YOUR YOURS YOURSELVES

THEY THEM THEIR THEIRS THEMSELVES

PROMILITARES.COM.BR 215
PERSONAL PRONOUNS

EXERCÍCIOS DE O pronome “it” em “It will be dismantled” refere-se a

FIXAÇÃO a) “embaixadas”.
b) “livros”.
c) “doações”.
d) “biblioteca”.
e) “torre de livros”.

07. (FCC) “I’ve seen ______ books on the table. Have you seen
01. (AFA) “The film BACK TO THE FUTURE, Michael J. Fox plays a ______?”.
teenager who accidentally travels back 30 years in time. There he
a) your - my c) your - mine e) yours - my
meets his parents, who are still teenagers and haven’t fallen in love
yet. He has to help them get together, because if they don’t get b) yours - mine d) your - me
married, he won’t exist!”
In the text above, the pronoun they refers to TEXTO PARA AS QUESTÕES 08 E 09:
a) H.G.Wells and Michael J. Fox. FOOD SHORTAGE CAUSES, EFFECTS AND SOLUTIONS
b) the accidentally travels. Food shortage is a serious problem facing the world and is
prevalent in sub-Saharan Africa. The scarcity of food is caused by
c) 30 years in time.
economic, environmental and social factors such as crop failure,
d) M.J.Fox’s parents in the film. overpopulation and poor government policies are the main cause
of food scarcity in most countries. Environmental factors determine
02. (EEAR) “Daisy Ridley, who plays a mysterious scavenger named the kind of crops to be produced in a given place, economic factors
Rey, and John Boyega, who plays a disaffected Stormtrooper named determine the buying and production capacity and socio-political
Finn, are among the new heroes of “The Force Awakens” and are factors determine distribution of food to the masses. Food shortage
bracing themselves for the biggest roles of their careers.” has far reaching long and short term negative impacts which include
The word “themselves” refers to: starvation, malnutrition, increased mortality and political 1unrest.
There is need to collectively address the issue of food insecurity using
a) Daisy Ridley and John Boyega. both emergency and long term measures.
b) Princess Leia and Han Solo. Causes of food shortages?
c) The Force Awakens. [...] There are several economic factors that contribute to food
d) Roles and careers. shortage. Economic factors affect the ability of farmers to engage in
agricultural production. 7Poverty situation in developing nations have
03. (EFOMM) “Typhoon Ida left a trail of destruction in ___________ reduced their capacity to produce food, as most farmers cannot afford
wake. It swept the country from coast to coast”. seed and fertilizers. They use poor farming methods that cannot 8yield
a) mine c) his e) its enough, even substantial use. Investments in agricultural research and
developing are very low in developing nations. 9Recent global financial
b) ours d) hers crisis have led to increase in food prices and reduced investments in
agriculture by individuals and governments in developed nations
04. (EFOMM) “She didn’t buy the furniture because _______ was too resulting in reduced food production.
expensive”.
[...]
a) it c) they
Solution to problem of food shortage:
b) its d) them
There are some solutions to the problem of food shortage. There
is need to reduce production of carbon emissions and pollution to
05. (UNESP 1988) Assinale a alternativa que completa corretamente a reduce the resultant climatic change through concerted and individual
frase “People should know about ______”. efforts. There is need to invest in clean energy such as solar, nuclear,
a) yourself c) himself e) yourselves and geothermal power in homes and industries, because 10they don’t
b) herself d) themselves have adverse effects on the environment (Kamdor, 2007). Rich nations
should help poor nations to develop and use clean and renewable
energy in order to stabilize green house emissions into the atmosphere
06. (UNIOESTE 2012) Leia.
(Watson, nd). Government need to work in consultation with climatic
ARGENTINA BUILDS A TOWER OF BOOKS bodies, World Bank and the UN to engage in projects aimed at
This is simply an audio and visual celebration of the book – any promoting green environment.
books, all books, in whichever language you like. Works by Jane [...]
Austen, Dickens, Henry Blake, Ernest Hemingway, Cervantes, Vargas Adapted from http://www.paypervids.com/food-shortage-
Llosa, Tolstoy and Argentina's own favourites, Borges and Sabato, line causeseffects-solutions/. Acesso em: 14 fev 2017.
the walls of this tower, each wrapped in plastic for its own protection.
The United Nations has designated the city as the 2011 World 08. (EPCAR/AFA 2018) The personal pronoun “they” (ref. 10) refers to
Book Capital. a) adverse effects.
This book tower is 25 metres high and lined with 30,000 donations b) homes and industries.
from more than 50 embassies. It'll be dismantled at the end of the c) geothermal power.
month and the books will form the beginning of a multi-lingual library.
d) some kinds of clean energy.
The Buenos Aires Book Fair, one of the biggest in the world,
has just ended, recording more visitors than ever before. The city
09. (EPCAR/AFA 2018) In “Poverty situation in developing nations
boasts hundreds of bookshops and some cafes even supply works by
have reduced their capacity to produce food, as most farmers cannot
Argentina's most renowned literary icon, Jorge Luis Borges, to read
afford seed and fertilizers” (ref. 7), the underlined word means
over coffee. Buenos Aires is a city that loves its books and now it has
a tower to prove it. a) poverty situation.
(Daniel Schweimler, BBC News, Buenos Aires. Fonte: http://www.bbc.co.uk/ b) developing nations.
worldservice/learningenglish/language/wordsinthenew s/2011/05/110516_witn
_buenosaires_books_page1.shtml)
c) seed and fertilizers.
d) most farmers.

216 PROMILITARES.COM.BR
PERSONAL PRONOUNS

10. At the Fourth World Conference on Women, Hillary Clinton 19


This is our hope. This is the faith that I go back to the South with.
stated, “It’s time for to say here in Beijing, and for the world When we allow 10freedom to 12ring, when we let 7it ring from every
to hear, that it is no longer acceptable to discuss’ rights women’s as state and every city, we will speed up that 14day when all of God’s
separate from human rights. children, black men and white men, Jews and Gentiles, Protestants
(Uel 1996) Assinale a letra correspondente à alternativa que preenche and Catholics, will join hands and sing the old Negro spiritual, “Free at
corretamente a lacuna da frase apresentada. last! free at last! thank God Almighty, we are free at last!”
(Adaptado de: LUTHER KING JR., Martin. I have a dream. Disponível em: <http://
a) we c) our e) ourselves www.archives.gov/press/exhibits/dream-speech.pdf>. Acesso em: 06 set. 2013.)
b) us d) ours
Assinale a alternativa que apresenta os referentes de their (ref. 5), its
(ref. 6) e it (ref. 7), respectivamente.
EXERCÍCIOS DE
a) all white people (ref. 8) – dream (ref. 9) – freedom (ref. 10)

TREINAMENTO b) all white people (ref. 8) – this nation (ref. 11) – ring (ref. 12)
c) many of our white brothers (ref. 13) – this nation (ref. 11) – freedom
(ref. 10)
01. (UPF 2015) Answer question(s) according to text. d) many of our white brothers (ref. 13) – this nation (ref. 11) – day
KING COUNTY KICKS OFF BAG RECYCLING (ref. 14)
CAMPAIGN WITH BUS FULL OF BAGS e) Negro community (ref. 15) – meaning (ref. 16) – freedom (ref.10)

03. (UFSM 2014)


NATURE & HEALTH - A DIRECT CONNECTION
Green spaces promote physical, psychological weII-being
By Julia Collins

The link between nature and health has finally been solidified.
2

British scientists Richard Mitchell, of Glasgow University, and Frank


Popham, of the University of St. Andrews, distinguished a direct
connection between contact with natural environments and improved
health.
4
The scientists studied a total of 941 million people between 2001
and 2005 to determine the links between fatal illnesses and access to
green spaces.
3
The evidence of their study shows 7green spaces have the most
positive effects on circulatory health, though 6they have little effect
on lung cancer.
1
“Evidence suggests that contact with such environments
In the sentence “The best way to recycle plastic bags is to stuff them has independent salutogenic effects, for example, green spaces
all in a single bag, tie it up and bring them back to this store”, the independently promote physical activity,” according to their findings
expressions “them – it – them” refer, respectively, to: published in the British medical journal The Lancet.
a) plastic bags - a single bag - plastic bags. However, 8they also say 5improved health is not only positively
b) the best way - a single bag - plastic bags. affected by physical activity in natural environments. “Several studies
have shown that contact (either by presence or visual) with green
c) a single bag - plastic bags - a single bag.
spaces can be psychologically and physiologically restorative, reducing
d) this store - a single bag - plastic bags. blood pressure and stress levels and possibly promoting faster healing
e) plastic bags - this store - plastic bags. in patients after surgical intervention,” their study says.
Young again
02. (UFRGS 2014) Leia. Nature photography isn't the only outdoor activity that can aid
I am happy to join 1__________ you today in what will go down health. Golf, gardening, and simply sitting outside in the sunshine
in history as the greatest demonstration for freedom in the history of have similar effects on mental and physical wellbeing.
our nation. According to Richard Louv, author of 13critically acclaimed Last
In the process 2__________ gaining our rightful place we must not Child in the Woods: Saving Our Children From Nature-Deficit Disorder,
be guilty of wrongful deeds. Let us not seek to satisfy our thirst for “the 10same benefits that come to children from nature also come to
freedom 3__________ drinking from the cup of bitterness and hatred. adults almost immediately: 11reduced stress, accentuated senses, and
We must forever conduct our struggle on the high plane of dignity increased attention span.”
and discipline. We must not allow our creative protest to degenerate “The bottom line is 12it's not a bitter pill. [Getting outside] is
into physical violence. 4The marvelous new militancy which has something that's fun that makes us all healthier and stimulates our
engulfed the 15Negro community must not lead us to distrust of 8all bodies and our minds and our souls. Nobody loses; everybody gains.”
white people, for 13many of our white brothers, as evidenced by 5their
(<http://ericksontribune.com/2010/09/nature-health-a-direct- connection/>.
presence here today, have come to realize that their destiny is tied Acesso em: 20 jul. 2013 - adaptado)
up with our destiny and their freedom is 17inextricably bound to our
freedom. We cannot walk alone. I have a 9dream that one day 11this Assinale verdadeira (V) ou falsa (F) em cada afirmativa a seguir.
nation will rise up and 18live out the true 16meaning of 6its creed: “We
( ) “they” (ref. 6) refere-se a “green spaces” (ref. 7).
hold these truths to be self-evident: that all men are created equal.”
I have a dream that my four little children will one day live in a nation ( ) “they” (ref. 8) refere-se a “41 million people” (ref. 9).
where they will not be judged by the color of their skin but by the ( ) “same benefits” (ref. 10) refere-se a “reduced stress, accentuated
content of their character. senses, and increased attention span” (ref. 11).

PROMILITARES.COM.BR 217
PERSONAL PRONOUNS

( ) “it” (ref. 12) refere-se a “critically acclaimed Last Child in the 06. (UEMG 2017)
Woods: Saving our Children from Nature- Deficit Disorder” (ref. 13). HOW A YOUNG STUDENT’S INNOVATIVE IDEA HOPES TO BOOST
A sequência correta é RESPONSE TIMES FOR EMTS
a) F – V – V – V. c) F – F – V – F. e) V – F – V – F. By Woody Brown on June 1, 2015

b) V – V – F – F. d) V – F – F – V. Drones have been at the forefront of the national conversation for


years now. As the components needed to create them grow smaller
04. (UFTM 2012) and more affordable, many companies and organizations have started
PSYCHOLOGY OF MONEY exploring the potential that drones could have to improve our daily
STUDY: THE RICH REALLY ARE MORE SELFISH lives. Whether by delivering a product with unprecedented speed or
By Brad Tuttle August 12, 2011 taking photographs and video from new heights, drones have many
capabilities, most of which we have yet to discover. One young man,
“Lower-class” individuals – i.e., folks without much money or however, has envisioned a new way to use drones that could save
education – demonstrate more compassion and empathy than their thousands of lives.
wealthy counterparts, according to a series of psychological studies. One of the greatest obstacles facing first responders and
In social scientist speech, “self-oriented behavior” is more likely to emergency medical technicians [EMTs] when it comes to the difficult
be exhibited by people with good education, prestigious jobs, high business of saving lives is time. Think of your daily commute: people
income, and overall higher-ranking social status. in the United States spend an average of 25.5 minutes traveling one-
[...] way to work every day. In bumperto-bumper traffic, blaring sirens
3
How the heck can researchers measure something like empathy? and flashing lights are often not enough to clear a fast path for an
One study, for instance, asked participants to identify the emotions ambulance to reach someone in need. During cardiac arrest, there are,
on display in photos of people with different facial expressions. at most, a few minutes to save a person’s life. After that, the mortality
Those with high-school-only educations showed “greater empathetic rate rises steeply. With stakes this high, every second counts.
accuracy” than participants with college educations. Alec Momont, a graduate student in engineering at Delft University
[...] of Technology in the Netherlands, recognized this problem and saw a
way to significantly reduce deaths that result from delayed emergency
Another study cited in the paper involved giving participants 10 care. What if ambulances could fly? Or rather, what if we could make
points, which would later be traded in for money. The individuals a drone that functioned like a stripped-down, lightweight automatic
given the points were to divide them up between themselves and an external defibrillator [AED]? AEDs, which can be found in schools,
anonymous partner. 4Guess who shared more of their points? sports arenas and many government buildings, are significantly more
We found that individuals reporting lower subjective socioeconomic effective than cardiopulmonary resuscitation [CPR] at preventing
status gave more to their partner than did upper-socioeconomic-status fatalities resulting from cardiac arrest. CPR can be helpful, but an AED
participants. In this context, the next time you’re called “low-class,” is better, and very few people have AEDs in their cars or homes.
5
consider it a compliment. As his master’s degree project, Momont built a prototype of this
(http://moneyland.time.com. Adaptado.) lifesaving drone. It contained an AED, a microphone and speakers.
The average travel time, according to him, could be cut by 90 percent.
In the excerpt of the sixth paragraph – Guess who shared more of their Here’s how it works: In the event of cardiac arrest, a paramedic would
points? (ref. 4) – the word their refers to respond to a call by flying the drone at a speed of 60 mph to the
a) anonymous partners. d) upper-class participants. scene of the emergency. The paramedic would then give instructions
b) participants. e) researchers. to someone near the victim, who would position the AED. Once in
place, the AED would operate automatically. The paramedic would be
c) lower-class individuals.
able to see through the camera whether or not the pads on the AED
have been correctly positioned, and how the victim responds.
05. (PUC-RS 2008)
A dramatized video released by Momont’s university demonstrates
It is an old saying that "Order is Heaven's First Law", and like
all of this functionality. In it, a young woman calls emergency services
many other old sayings, it contains a much deeper philosophy
in a panic because her father has had a heart attack. A calm-voiced
than appears immediately on the surface. Getting things into
EMT answers and guides her through the surprisingly simple process
a better order is the great secret of progress, and 1we are now
of finding and using the drone. Fewer than two minutes after she
able to fly through the air, not because the laws of Nature have
makes the call, her father sits up and hugs her.
altered, but because we have learnt to arrange things in the right
order to produce this result - the things 2themselves had existed [...]
from the beginning of the world, but what was wanting was the We live in a world where drones have, so far, been used mostly in
introduction of a Personal Factor which, by an intelligent perception armed conflict. Momont, however, has a different vision. In the near
of the possibilities contained in the laws of Nature, should be able future that he describes, tens of thousands of needless deaths will
to bring into working reality ideas which previous generations would be prevented with his ingenious invention. That is certainly welcome
have laughed at as the absurd fancies of an unbalanced mind. (...) news, especially in the United States, which deals with skyrocketing
Now the first thing in any investigation is to have some idea of what numbers of heart-related ailments and disabilities. “Let’s use drones
you are looking for, just as you would not go up a tree to find fish, for a good purpose,” Momont says. “Let us use drones to save lives.”
though you would for birds' eggs. (Adapted from: <http://www.verizonwireless.com/news/article/2015/05/ambu
(TROWARD, T. (1915), The creative process in the individual. lance- dronescould- save-thousands-of-lives.html>. Access on: 03 Oct. 2016.)
Dodd, Mead & Co., New York. pp 1-2.)
Mark T (true) and F (false) for the correlations between pronouns and
The pronoun "themselves" (ref. 2) is used their referents in the text.
a) as the complement to the verb "had existed". ( ) it (paragraph 5) –> video.
b) to emphasize the subject of the verb "had existed". ( ) who (paragraph 4) –> victim.
c) in relation to people taken in general. ( ) them (paragraph 1) –> drones.
d) to specify which things are arranged. ( ) which (paragraph 1) –> capabilities.
e) as a personal pronoun.

218 PROMILITARES.COM.BR
PERSONAL PRONOUNS

The correct sequence is 19


Initially SpaceX plans to reduce the cost of a Falcon 9 rocket with
a) F, T, F, T. b) T, T, F, T. c) T, F, T, T. d) T, F, F, F. a reused booster to $43 million per flight, a savings of 30 percent.
But this is only the beginning. Musk wants to make nearly all of the
Falcon 9 launch system reusable, and he wants to make launches and
07. (PUC-RJ 2017)
landings routine. “Rapid and complete reusability is really important
FALCON SOARS INTO SPACE AND LANDS IN THE OCEAN to make a rocket cost effective, like an airplane,” he said. “We’ve got
Musk’s Falcon 9 rocket had just shot 200 km up into space and to ultimately get rockets to that point.” “A Falcon 9 might fly as many
flown almost horizontal to the planet at six times the speed of sound, as 100 times before retirement”, he added.
before falling back to Earth. Then, somehow, it landed like a feather (Eric Berger Retrieved and adapted from http://arstechnica.com/science/2016/04/ like-
on a robotic barge in the ocean. The Falcon even found time to put an a-boss-falcon-soars-into-space-and-lands-in-the-ocean/ Access on July 2nd, 2016.)
inflatable space habitat into orbit, too.
In terms of reference, it is correct to affirm that
Engines and boosters have been dropping into the big drink
from the moment NASA began launching Mercury astronauts into a) “its” (ref. 3) refers to “five months ago” (ref. 2).
space. Most of those rockets sunk to the bottom of the ocean. Some b) “it” (ref. 5) refers to “New Shepard rocket” (ref. 4).
components of the space shuttle were recovered, of course, and the c) “which” (ref. 8) refers to “space” (ref. 7).
orbiter itself landed on a runway. But never before Friday has a rocket
blasted into space and then returned to make a vertical landing at sea. d) “They” (ref. 11) refers to “Masten Space Systems” (ref. 10).
These are 1heady times for a new generation of private spaceflight e) “which” (ref. 17) refers to “vehicle” (ref. 16).
companies. It was only 2five months ago that Blue Origin launched
3
its 4New Shepard rocket to space before landing 5it vertically in West 08. (EN 2017) Based on the text below, answer the question.
Texas. SpaceX followed suit in December when its larger and more TOO MANY THIRD GRADERS CAN’T READ THIS SENTENCE
powerful Falcon 9 not only flew to space, but delivered a satellite into
9 Feb. 2017 – Editor’s Picks
orbit and touched down at a landing site along the Florida coast near
its launch pad. Now SpaceX has taken the significant step of landing Two-thirds of U.S. third graders face challenges that will impact
at sea. their future, including academic struggles that could lead to dimmer
6
That’s critical, because it’s much more fuel efficient for a rocket academic and career prospects. Sadly, only one in three U.S. students
to touchdown on a ship below the point where it releases its payload demonstrates reading proficiency at the end of third grade. This has
into orbit, rather than go all the way back to a landing site near the alarming consequences for these children, and for our country.
Florida coast. SpaceX estimates that only one-half of its launches will A report released today from the Business Roundtable (BRT)
have enough fuel to fly back to the coast after fulfilling their primary shed light on this troubling trend in American education, and advises
missions. business leaders on how they can help put more children on a path
After trying reusability with the 7space shuttle, 8which proved to success. (…)
far more expensive to refurbish for subsequent flights, NASA has I’ve heard it said that before third grade, students are learning
largely given up on reusable spaceflight. Russia, Europe, and other to read, while after third grade, they’re reading to learn. Grade three
government agencies have too. 9So it fell to the new space companies, is a crossroads in a life’s journey. If you’ve read this far, then you
with their ethos of low-cost, to make frequent launches as a means of understand why is so important. Not enough of our young learners
opening up access to space, in order to push the technology forward. can say the same.
SpaceX and Blue Origin have been the most visible proponents of I encourage you to read the BRT report. As you read, please
reusable launch vehicles. But other firms, including XCOR, 10Masten consider ways to help our schools and our teachers keep students on
Space Systems, and Virgin Galactic, are interested too. 11They do not paths to bright futures.
seek so much to win lucrative government contracts, but to get lots
of people and stuff into space, to create a space economy, and to [...]
set about the business of colonizing the solar system. “This was a (Adapted from http://www.linkedin.com.)
really good milestone for the future of spaceflight,” Musk said. “This
is another step to the stars”, he added. By reading the text, we CANNOT state that the pronoun
12
Despite SpaceX’s arresting success on Friday, the job is not yet a) “their” in “Two-thirds of U.S. third graders face challenges that
done. 13The venerable space shuttle offers a 14sobering lesson for will impact their future […]” refers to “Two- thirds of U.S. third
these new space companies. 15Whereas NASA said in the 1970s the graders”.
shuttle would slash the cost of delivering payloads into space to $25 a b) “these” in “This has alarming consequences for these children
pound, it ended up costing closer to $25,000 a pound. It’s one thing […]” refers to “only one in three U.S. students”.
to land a rocket, and it’s another thing to fly it again without spending c) “they” in “[…] how they can help put more children on a path to
a lot of time and money. success.” refers to “business leaders”.
Musk must prove that his Falcon 9 rocket can be re-flown with d) “they” in “[…] while after third grade, they’re reading to learn.”
modest modifications. After the first SpaceX landing in December, the refers to “students”.
company performed a static firing test of the 16vehicle, 17which went
e) “you” in “I encourage you to read the BRT report.” refers to the
well until one of the nine engines showed thrust fluctuations. That
reader.
rocket will stand as a monument outside the company’s headquarters
in Hawthorne, California. Perhaps the Falcon 9 that landed at sea will
be flown again. 09. (CESESP) Peter and Anne are friends of ______ . I like ______ very
much. I suppose ________ also like me.
18
“It should fly again,” Musk said Friday. SpaceX will try to return
the booster back to Cape Canaveral, in Florida, by Sunday. After a) me – they- she
running a series of tests on the Falcon, the company plans to fire its b) mine – them – they
engines 10 times in a row on the ground. “If things look good, it will c) yours – her – she
be qualified for reuse,” Musk said. “We’re hoping to relaunch it on an
orbital mission, let’s say by June”, he added. d) them – them – they
e) him – him – he

PROMILITARES.COM.BR 219
PERSONAL PRONOUNS

10. (EN) Which is the correct way to complete the excerpt below? 06. (ITA) Assinale a alternativa correta.
THE LEGACY OF HARTLEPOOL a) Both he and I teach English.
He thought that if he had the courage to visit the Long Gallery, the b) Both him and I teach English.
portraits of ________ ancestors would come to life in ______ frames. c) Both he and me teach English.
_______ would point ______ fingers and say: “We did ______ duty.
We spent the money as __________ . was meant to be spent [...]". d) Both him and me teach English.

a) him – its – we – ours – ours – he e) n.d.a.

b) him – theirs – they – their – ours – it


07. (STA CASA) “Why don’t they enjoy __________ instead of watching
c) her – their – we – ours – our – he that dull TV programme”?
d) his – their – they - their – our – it a) yourself
e) his – its – they – theirs – our – it b) ourselves
c) yourselves
d) themselves
EXERCÍCIOS DE

COMBATE
e) himself

08. (UNESP) Assinale a alternativa que preenche corretamente a


lacuna da frase apresentada:
“Catherine is making __________ a dress”.
01. (AFA) “The Kremlin hoping a young strongman can preserve its
brutal victory in Chechnya.” a) to him d) herself
(Newsweek, September, 2006) b) to her e) they
c) himself
Mark the alternative that completes the blanks of the statement
below. 09. (CESGRANRIO) Mark the option which completes the following
“The word its is a (an) __________ pronoun and refers to__________”. sentences with the adequate pronouns:
a) personal - young strong man I. Businessmen have __________ own priorities.
b) reflexive - Chechnya II. Everyone must feel happy with __________ working habits.
c) possessive adjective - The Kremlin hoping III. Working from home allows a mother to spend more time with
d) objective - victory __________ children.
IV. If you have never tried to work at home, you cannot discuss
02. (EFOMM) “As a tool, the computer assists __________ to perform __________ disadvantages.
a lot of activities”. a) I. his - II. their - III. her - IV. their
a) we d) they b) I. their - II. its - III. their - IV. its
b) us e) to mine c) I. their - II. his - III. her - IV. its
c) ourselves d) I. its - II. your - III. its - IV. their
e) I. his - II. his - III. their - IV. your
03. (AFA) The pronoun the best substitutes the underlined words in
the sentence: “The buyers found it easy.” is
10. (EFOMM) In which sentence “it” is not correctly used?
a) you. c) they.
a) I can not bear it to see women crying.
b) he. d) we.
b) She found it strange being in that flat.

04. (EN) In the excerpt: “But if libraries are going to survive the digital c) We owe it to society to help those who need help.
age, they need to be more about helping patrons filter vast quantities d) I take it that you won't be working tomorrow.
of digital information rather than access to analog materials.” the e) I'll leave it to John to let them know.
pronoun “they” refers to:
a) libraries.
b) the digital age.
GABARITO
c) patrons. EXERCÍCIOS DE FIXAÇÃO
d) quantities of digital information. 01. D 04. A 07. C 10. B
e) analog materials. 02. A 05. D 08. D
03. E 06. E 09. B
05. (EN) “Path, the mobile phone-based social network founded by EXERCÍCIOS DE TREINAMENTO
former Facebook employee Dave Morin, which only allows its users to
01. A 04. B 07. B 10. D
have 150 friends, is gaining users in a week.”
02. C 05. B 08. B
The pronoun its refers to:
03. E 06. C 09. B
a) employee.
EXERCÍCIOS DE COMBATE
b) Path.
01. C 04. A 07. D 10. A
c) Dave Morin.
02. B 05. B 08. D
d) Facebook.
03. C 06. A 09. C
e) mobile.

220 PROMILITARES.COM.BR
INTERROGATIVE AND
DEMONSTRATIVE PRONOUNS

PRONOMES INTERROGATIVOS Exemplo:


How long has Ana studied English? Há quanto tempo Ana
(INTERROGATIVE PRONOUNS) estuda inglês?
Usamos os pronomes interrogativos para fazermos perguntas.
How much is this car? Quanto custa este carro?
Eles também são conhecidos como WH- Questions.
How many English dictionaries do you have? Quantos dicionários
PRONOMES TRADUÇÃO de inglês você tem?
How often do you go to the movies ? Com que frequência você
Who? Quem? vai ao cinema?

Whom? Quem? How far did you run? Qual a distância que você correu?
How old are you? Quantos anos você tem?
What? Que? O que?

Which? Qual? Pronomes interrogativos usados com substantivos contáveis e


incontáveis.
Whose? De quem?
How many Quantos , quantas Substantivos contáveis
When? Quando?
How much Quanto Substantivos incontáveis
Where? Onde? Aonde?

Why? Por que?

How? Como? PRONOMES DEMONSTRATIVOS


(DEMONSTRATIVE PRONOUNS)
Exemplo: Os pronomes demonstrativos fazem referência a objeto(s) ou
Who is that girl? Quem é aquela garota? pessoa(s) dentro do contexto em que se encontram.
Whom do you see at the door? Quem você vê à porta?
SINGULAR PLURAL
What is that on the floor? Que é aquilo no chão?
Which of the two is going to São Paulo? Qual dos dois vai para This These
São Paulo? isto, este, esta estes, estas
Whose pencil is this? De quem é este lápis?
That Those
When do you go to Salvador? Quando você vai a Salvador?
isso, esse, essa esses, essas
Where does Liliam live? Onde Liliam mora?
aquilo, aquele, aquela aqueles, aquelas
Why are you nervous? Por que você está nervoso?
How do I begin the Math exercise? Como começo o exercício de
matemática? Exemplo:
This building is very old.
Há algumas expressões interrogativas formadas a partir do That car is from Italy.
pronome HOW These students are prepared for the test.
Those soldiers are going to Iraq next week.
How long? Quanto tempo?,

How much? Quanto? (antes de substantivos incontáveis) Usamos os pronomes demonstrativos para fazer referência de
alguém ou algo dentro de um contexto.
How many? Quanto? (antes de substantivos contáveis)
Exemplo:
How often? Com que frequência?, Near São Paulo’s Luz station there is a “cracolândia”. The police
doesn’t have control over this. (O pronome “this” faz referência à
How far? Qual a distância? Quão longe?,
cracolândia.)
How old? Quantos anos? I don’t like these apples. I prefer those ones. (“those” refere-se
a “apples”.)

PROMILITARES.COM.BR 221
INTERROGATIVE AND DEMONSTRATIVE PRONOUNS

EXERCÍCIOS DE A lacuna no texto (ref. 2) pode ser corretamente preenchida por:

FIXAÇÃO a) “When do”.


b) “How is”.
c) “When is”.
d) “Who was”.
e) “What are”.

07. (UPE 2013)


01. (UNESP) Assinale a alternativa que completa corretamente a frase:
“__________ is your hat?”
a) When c) Whose e) How many
b) Who d) Where

02. (FUVEST) Choose the question for the statement: “Plague cropped
up in 1994, in India”.
a) How long did plague crop up in India? No 2º quadrinho, qual expressão interrogativa completa CORRETAMENTE
a pergunta?
b) How did plague crop up in 1994?
a) How far c) How much e) How often
c) When did plague crop up in India?
b) How about d) How long
d) What did plague crop up in India?
e) Why did plague crop up in India? 08. (UFSJ 2013)
LGBT RIGHTS IN BRAZIL
03. (UEL) Assinale a letra correspondente à alternativa que preenche Lesbian, gay, bisexual, and transgender (LGBT) people in Brazil
corretamente a lacuna da frase: “_________ video do you want to take?” enjoy most of the same legal protections available to non-LGBT people.
a) When c) What e) Where On May 5, 2011, the Supreme Federal Court voted in favor of
b) Why d) How allowing same-sex couples the same 112 legal rights as married couples.
2The decision was approved by 10 – 0 with one abstention, and it will
04. (EEAR) Read the dialogue and mark the right expression to give same-sex couples in stable partnerships the same financial and
complete it correctly. social rights enjoyed by those in opposite-sex relationships.
A: _____________ have you been working as a sergeant?
1
The list of various LGBT rights in Brazil has expanded since the end of
the military dictatorship in 1985, and the creation of the new constitution
B: For about 2 years.
of Brazil of 1988. In 2009, a survey conducted in 10 Brazilian cities found
a) How c) How much that 7.8% of men identified as gay, with bisexual males accounting for
b) How long d) How many another 2.6% of the total population (for a total of 10.4%). The Brazilian
lesbian population was 4.9% of females, with bisexual women reaching
05. (EN 2016) Which question word best completes the paragraph 1.4% (for a total of 6.3%). There are no nation-wide statistics.
below? According to the Guinness World Records, the São Paulo Gay
So if smoking is so bad for you, __________ is it so hard to quit? Pride Parade is the world's largest LGBT Pride celebration, with 4
Stopping smoking is difficult for several reasons: nicotine is highly million people in 2009. Brazil had 60.002 same-sex couples in the
addictive, rewarding psychological aspects of smoking and genetic same home, according to the Brazilian Census of 2010 (IBGE). The
predisposition. South American country has 300 active LGBT organizations.
(Adapted from http://www.spine-health.com/wellness/stop-smoking) (Adapted from http://en.wikipedia.org/wiki/LGBT_rights_in_Brazil.
Access on August 22nd, 2012.)
a) what c) who e) why
In the sentence “The decision was approved by 10 – 0 with one
b) when d) whose abstention, and it will give same-sex couples in stable partnerships
the same financial and social rights enjoyed by those in opposite-sex
06. (UEA 2014) Considere o texto. relationships” (ref.2), the word those could be substituted by
AMAZON TRIBES a) social rights. c) people.
The Amazon is the world’s largest rainforest. It is also the ancestral b) financial rights. d) rights.
home of one million Indians. They are divided into about 400 tribes,
each with its own language, culture and territory. 1Many have had 09. (CN 2017) Match the questions to the answers.
contact with outsiders for almost 500 years. Others – ‘uncontacted’ 1. Are you on vacation? ( ) Yes, we are.
tribes – have had no contact at all. 2. Where’s she from? ( ) She’s from Italy.
How do they live? 3. Are you and Ana married? ( ) No, she isn’t.
Most Indians live in settled villages by the rivers, and grow 4. Is Julian Italian? ( ) No, I’m not.
vegetables and fruits like manioc, corn, beans and bananas. They also Choose the option with the correct order.
hunt and fish, using plant-based poisons to stun the fish. Some tribes
use shotguns for hunting, others use bows and arrows, spears, or a) 3 – 2 – 4 – 1 c) 3–4–2–1 e) 1 – 4 – 3 – 2
blowguns with darts tipped with curare. Only a few Amazonian tribes b) 1 – 2 – 4 – 3 d) 1 – 4 – 2 – 3
are nomadic; they tend to live deep in the forest away from the rivers.
They grow some crops but rely more on hunting and gathering. 10. (UNESP) Assinale a pergunta correta para a resposta: “For two
2
__________ their problems? weeks”.
Almost all the Indians’ problems revolve around land: 3outsiders a) How long have you had it?
either want their land, or something on or underneath it. The key b) How many times do you have it?
threats are a massive boom in oil and gas exploration, rampant illegal c) How far did you have it?
logging and the rapid spread of ranching and farming.
d) How much money you have got it?
(www.survivalinternational.org. Adaptado.)
e) There is how long you’ve got it?

222 PROMILITARES.COM.BR
INTERROGATIVE AND DEMONSTRATIVE PRONOUNS

EXERCÍCIOS DE the pages, but a statement released by the American Academy of

TREINAMENTO Pediatrics (AAP) this week says reading in the early years is essential.
Reading out loud gets parents talking to their babies and the sound
of an adult’s voice stimulates that tiny yet rapidly growing brain. In the
statement, the academy advises pediatricians to tell parents to read
01. (CESGRANRIO) “A compulsive shopper told a researcher that she books to their children from birth.
could never go to a supermarket and buy just one bottle of milk”. Reading regularly with young children stimulates optimal patterns
Mark the question to which this sentence is an answer: of brain development and strengthens parent-child relationships at a
a) Where a compulsive shopper buys her milk? critical time in child development, which, in turn, builds language,
literacy, and social-emotional skills that last a lifetime. Research shows
b) What did a compulsive shopper tell a researcher?
that a child’s brain develops faster between 0 and 3 than at any other
c) Who did a compulsive shopper tell her habits to? time in life, making the early years a critical time for babies to hear
d) Why has a compulsive shopper told a researcher about her habits? rich oral language. The more words children hear directed at them by
e) How has a compulsive shopper told a researcher her routine? parents and caregivers, the more they learn.
While many babies are read Goodnight Moon and The Very
02. (CESGRANRIO) “The new telephone can deal with ‘hello’ and Hungry Caterpillar every night before bed, others never get a chance
other words well enough.” This sentence contains the answer to all to “pat the bunny.” Studies reveal that children from low-income,
question below EXCEPT one. Mark it. less-educated families have significantly fewer books than their more
affluent peers. By age 4, children in poverty hear 30 million fewer
a) What can the new telephone deal with?
words than those in higher-income households. These dramatic gaps
b) What can deal with “hello” and other words well enough? result in significant learning disadvantages that persist into adulthood.
c) How can the new telephone deal with “hello” and other words? The AAP hopes the new guidelines will encourage all parents to start
d) Whose words can the telephone deal with well enough? reading from day one.

e) Which words can the new telephone deal with well enough? Research shows that when pediatricians talk with parents about
reading, moms and dads are more likely to fill their home with books
and read. Also, to help get more parents reading, the AAP is partnering
03. (UNESP 2018)
with organizations such as Scholastic and Too Small to Fail to help get
WHEN IT COMES TO POLITICS AND FAKE NEWS, reading materials to new families who need books the most.
FACTS AREN’T ENOUGH
This is the first time the AAP has made a recommendation on
In today’s political climate, it sometimes feels like we can’t even agree children’s literary education and it seems the timing might be just
on basic facts. We bombard each other with statistics and figures, hoping right as more and more parents are leaning on screens and electronic
that more data will make a difference. A progressive person might show gadget to occupy their babies. “The reality of today’s world is that
you the same climate change graphs over and over while a conservative we’re competing with portable digital media,” Dr. Alanna Levine, a
person might point to the trillions of dollars of growing national debt. pediatrician in Orangeburg, N.Y., told The New York Times. “So you
We’re left wondering, “Why can’t they just see? It’s so obvious!” really want to arm parents with tools and rationale behind it about
Certain myths are so pervasive that no matter how many experts why it's important to stick to the basics of things like books.”
disprove them, they only seem to grow in popularity. There’s no (http://blog.seattlepi.com. Adaptado.)
shortage of serious studies showing no link between autism and
vaccines, for example, but these are no match for an emotional appeal No trecho do terceiro parágrafo “By age 4, children in poverty hear
to parents worried for their young children. Tali Sharot, a cognitive 30 million fewer words than those in higher-income households.”, o
neuroscientist at University College London, studies how our minds termo em destaque se refere às crianças
work and how we process new information. In her upcoming book, a) de famílias pobres.
The Influential Mind, she explores why we ignore facts and how we
b) de famílias com menor escolaridade.
can get people to actually listen to the truth.
c) de famílias de maior renda.
Tali shows that we’re open to new information – but only if it
confirms our existing beliefs. We find ways to ignore facts that d) com problemas de aprendizagem.
challenge our ideals. And as neuroscientist Bahador Bahrami and e) com mais de quatro anos de idade.
colleagues have found, we weigh all opinions as equally valid,
regardless of expertise. 05. (CEFET MG 2013)
So, having the data on your side is not always enough. For better “E” IS FOR EVERYDAY ETHICS
or for worse, Sharot says, emotions may be the key to changing minds. By Margaret R. McLean
(Shankar Vedantam. www.npr.org. Adaptado.)
When was the first time you had to choose between right and
No trecho do segundo parágrafo “but these are no match for an wrong, between telling the truth or whispering a lie? What was it that
emotional appeal to parents worried for their young children”, o first tweaked your sense of fair play — a warm gooey chocolate chip
termo sublinhado refere-se a cookie not quite divided in half, being “too little” to try-out for the
a) “experts”. d) “parents”. ball team? When was the first time 4you cried, “It's not fair!”? It really
b) “studies”. e) “myths”. is hard to imagine a time when we weren't trying to be good — or,
at least, trying to avoid being caught in that little lie. It really is hard
c) “autism and vaccines”. to imagine a time when we weren't doing ethics. Everyday, 3we face
ethical dilemmas, those moments when we think: Should I? Shouldn't
04. (UNESP 2015) I? Ethics — we can either do it well or do it badly.
PEDIATRIC GROUP ADVISES PARENTS TO READ TO KIDS There seem to be as many definitions of ethics as there are
June 26, 2014 ethicists. The one that has caught my eye recently is: “Ethics is
By Amy Graff individuals working together as a community to be at their best.”
This simple statement acknowledges what we all know to be true
Reading Go Dog Go to your 6 month old might seem like wasted about life — that we are in this alone and we are in this together. We
time because she’s more likely to eat the book than help you turn

PROMILITARES.COM.BR 223
INTERROGATIVE AND DEMONSTRATIVE PRONOUNS

are individuals — deciding, choosing, living, dying. But, we are also A pergunta que pode ser respondida com base nas informações do
members of communities — family, neighborhood, church, society. As texto é
individuals, we focus on will and choice — I decide what I am going to a) “How old was Mark Zuckerberg in 2004”?
do. As members of communities, we see that the results of our choices
affect others — my decision to lie is wrong because it hurts others as b) “When was the Facebook founded?”
well as myself. Ethics asks us to reflect on what 1it means to be at our c) “What makes the social network enter a new era?”
best both in our individual lives and in our relationships. What does it d) “Who directs the McKinsey Global Institute?”
take to be caring and committed people and communities?
e) “Where was Mark Zuckerberg born?”
Every day, we decide who we are — truthful or dishonest? Every
day, our actions have consequences — helpful or hurtful? Every day, 07. (EN 2017)
we either build up or tear down relationships. We tend to think of
ethics as coming into view only in congressional investigations or TOO MANY THIRD GRADERS CAN’T READ THIS SENTENCE
intensive care units. But, in reality, we practice ethics every day; we 9 Feb. 2017 – Editor’s Picks
work together to be at our best — or our worst — every day. Cloning,
assisted suicide, lying to Congress — 2these catch the headlines. Two-thirds of U.S. third graders face challenges that will impact
Munching on unpaid-for grapes in the market, cheating on an exam their future, including academic struggles that could lead to dimmer
or taxes, telling 5that little white lie — these catch our lives every day. academic and career prospects. Sadly, only one in three U.S. students
demonstrates reading proficiency at the end of third grade. This has
(Margaret R. McLean is director of healthcare and biotechnology ethics at the
Markkula Center for Applied Ethics. Posted August 2006 Available at: alarming consequences for these children, and for our country.
<http://www.scu.edu/ethics/practicing/focusareas/medical/ introduction A report released today from the Business Roundtable (BRT) shed
/everyday.html>. Accessed on: April 2013. collective.)
light on this troubling trend in American education, and advises business
leaders on how they can help put more children on a path to success.
Reading the text in details, one can say that
(…)
a) “it” (ref. 1) refers to “my decision to lie”.
I’ve heard it said that before third grade, students are learning
b) “these” (ref. 2) refers to “the headlines”.
to read, while after third grade, they’re reading to learn. Grade three
c) “we” (ref. 3) refers to the author and her colleagues. is a crossroads in a life’s journey. If you’ve read this far, then you
d) “you” (ref. 4) refers to Margaret McLean’s colleagues. understand why is so important. Not enough of our young learners
e) “that” (ref. 5) refers to any specific lie the reader remembers. can say the same.
I encourage you to read the BRT report. As you read, please
06. (UFRGS 2012) consider ways to help our schools and our teachers keep students on
paths to bright futures.
FACEBOOK IS THE WORLD’S LARGEST SOCIAL NETWORK, WITH 800
MILLION USERS WORLDWIDE AS OF SEPTEMBER 2011. Leave your comments below
More than any other company, it has defined what some see as MICHAEL JONAS
the “social” era of the Internet, in which connections made among
people replace algorithm-driven searches. And its policies, more than Really, all I read was blabla wa wa wa. Are you Charlie Brown’s
any others, seem to be driving the definition of privacy in this new age. teacher? If we can’t understand our children who are crying out for
help and direction, then there is a something wrong with you. Please
Every day, Facebook users comment or press the “like” button go back and check yourself! They are worth so much more.
more than 2 billion times and upload more than 250 million photos.
The McKinsey Global Institute has estimated that the network’s users RICK SHIRE
post 30 billion pieces of content every month. Thanks for sharing. With two young children, I increasingly think
The company, founded in 2004 by a Harvard sophomore, Mark about the importance of early childhood education. Pre-k care is far
Zuckerberg, began life catering first to Harvard students and then to too inaccessible, ultimately magnifying from the earliest stages of life.
all high school and college students. It has since evolved into a broadly TOM FRANK
popular online destination used by teenagers and adults of all ages. What exactly is education? Academic education doesn’t make
In country after country, Facebook has cemented itself as the leader, someone a better person or even a better employee. I would guess
often displacing other social networks. that anything we learn in the education process is at the most 10%
It is no surprise that Facebook has become one of the titans of the useful to us as people. Education should teach academia but also life
Internet, challenging even Google with its vision of a Web tied together skills such as budgeting, EQ skills, languages etc., all the elements to
by personal relationships and recommendations, rather than by search be a successful person and not necessarily a successful professional.
algorithms. In a major expansion, Facebook has spread itself across (Adapted from http://www.linkedin.com.)
other Web sites by offering members the chance to “Like” something -
share it with their network – without leaving the Web page they are on. By reading the text, we CANNOT state that the pronoun
At the Facebook developer conference in September, the a) “their” in “Two-thirds of U.S. third graders face challenges that will
company announced the release of a product called Timeline, which impact their future [...]” refers to “Two-thirds of U.S. third graders”.
offers a highly visual view of a user’s Facebook profile and organizes b) “these” in “This has alarming consequences for these children
content into photos, events and apps, all based on a timeline view that [...]” refers to “only one in three U.S. students”.
stretches back to the beginning of a user’s time on Facebook. Timeline
is designed to work on mobile devices, too. c) “they” in “[...] how they can help put more children on a path to
(Adaptado de: WYLD, Adrian. Facebook.
success.” refers to “business leaders”.
<http://topics.nytimes.com>. Acesso em 01 dez. 2011.) d) “they” in “[...] while after third grade, they’re reading to learn.”
refers to “students”.
e) “you” in “I encourage you to read the BRT report.” Refers to the
reader.

224 PROMILITARES.COM.BR
INTERROGATIVE AND DEMONSTRATIVE PRONOUNS

08. (PUC-PR 1999) Fill in the balloons with the right interrogative 04. (UNIRIO) Mark the correct option.
pronouns. Relate the numbers given to the pronouns. “Researchers at Ohio State have developed a way to speed up
the growth of native shade trees – and the local utility plans to help
promote _______ saplings to homeowners”.
a) which c) this e) these
b) it d) that

05. (EPCAR) Mark the option that is the correct question for the
sentence “One of the reasons is the great use of cell phones.”
a) When do children practice cyberbullying?
b) What kind of children practice cyberbullying?
c) Why do children often practice cyberbullying?
d) How often do children practice cyberbullying?

06. (JFS) Which item completes the paragraph below correctly?


“Faced with water shortages along the Yangtze River, WWF is
working in China with the government and local authorities to help
a) 1. whose - 2. who - 3. why - 4. how - 5. who communities best adapt to climate change impacts. _______ includes
b) 1. who - 2. which - 3. what - 4. why - 5. how developing a climate witness project in the Yangtze River basin so that
people affected by climate change can speak for themselves”.
c) 1. what - 2. whose - 3. what - 4. why - 5. what
a) Those c) These e) Its
d) 1. which - 2. who - 3. how - 4. what - 5. how
b) They d) This
e) 1. who - 2. whose - 3. what - 4. why - 5.how

07. (PUC-MG) “I had just participated in a project that was to determine


09. (AFA) Mark the correct question to the answer: “The government
the minimum size of forest fragment necessary to save native species
aims to transfer 30% of farmland to black South Africans by 2014.”
of animals and plants from extinction. With this information, scientists
a) Who transferred 30% of farmland? could then work to form preservation areas in the forest fragments left
b) Why did the government transfer 30% of farmland? behind by cattle ranchers”.
c) What does the government want to do? The word THIS refers to the:
d) Where does the government purchase part of the farmland? a) utilization of many valuable native species.
b) necessity of urgently saving birds from extinction.
10. (PUC-RS) Assinale a alternativa que completa corretamente a c) size of the forest needed for wildlife reserves.
frase: “Can you tell me _________?”.
d) destruction of thousands of native species.
a) how much does a box of matches cost
b) how much a box of matches costs 08. (EFOMM) Mark the alternative that presents the right sequence
c) how much do a box of matches cost of pronouns to fill in the gaps in: “__________ cat is sick because
d) how much is a box of matches cost __________ ate __________ spoiled food over there”.
e) how much costs a box of matches a) Its - he - that c) His - its - this e) His - it - that
b) Its - he - this d) Its - it - that

EXERCÍCIOS DE 09. (EN) Which question word completes the paragraph below?

COMBATE
“The National Gallery in London is among a growing number of
venues that have banned selfie sticks. ________ is so wrong with
walking around a gallery taking pictures of yourself? The gallery says
it needs to protect artworks and other visitors”.
a) Which c) What e) Who
01. (UNESP) Assinale a alternativa que completa corretamente a frase:
“These blue jeans are mine. _________ are those on the sofa? They're b) Why d) When
Peter's”.
a) Which c) Where e) Whose 10. (PUC-PR) Fill in the blanks below, choosing the best alternative:
b) What d) Whom I. _____ knows how to speak decent French to talk to the tourists?
II. The ticket costs $8. _____ are you going to pay?
02. (UEL) Assinale a letra correspondente à alternativa que preenche III. _____ can I take the subway to the Guggenhein Museum?
corretamente a lacuna da frase apresentada: “________ is the money IV. _____ of those buildings is the hospital?
you lent to me. Thank you very much!”
V. _____ will your sister travel to London?
a) Many c) Those e) This
b) None d) These a) I. Whom - II. What - III. Which - IV. Where - V. How
b) I. Whose - II. Who - III. How - IV. What - V. Why
03. (UNESP) Assinale a alternativa que completa corretamente a frase:
“_________ organisms pose no danger to human life”. c) I. Which - II. Why - III. When - IV. How - V. Whose
a) A c) Those e) This d) I. Who - II. How - III. Where - IV. Which - V. When
b) An d) That e) I. How. - II. When - III. What - IV. Why - V. Where

PROMILITARES.COM.BR 225
INTERROGATIVE AND DEMONSTRATIVE PRONOUNS

GABARITO
EXERCÍCIOS DE FIXAÇÃO
01. D 04. B 07. D 10. A
02. C 05. E 08. C
03. C 06. E 09. A
EXERCÍCIOS DE TREINAMENTO
01. B 04. C 07. B 10. B
02. D 05. E 08. B
03. B 06. B 09. C
EXERCÍCIOS DE COMBATE
01. E 04. E 07. C 10. D
02. E 05. C 08. E
03. C 06. D 09. C

ANOTAÇÕES

226 PROMILITARES.COM.BR
QUANTIFIERS AND
INDEFINITES

QUANTIFIERS E INDEFINITES MUCH, LITTLE, A LITTLE


Antes de substantivos incontáveis (singular).
COUNTABLE NOUNS UNCOUNTABLE NOUNS I did not have much coffee last night.
MANY MUCH
Muitos, muitas, bastantes Muito, muita, bastante incontável
He can’t go with us, he has little money.
FEW LITTLE
Poucos, poucas Pouco, pouca
incontável
A FEW A LITTLE
Uns poucos, umas poucas, Um pouco/algum/alguma incontável
alguns/algumas Alguns, o suficiente

A: Would you like some orange juice?


B: Just a little, thank you!
MANY, FEW, A FEW
Antes de substantivos no plural.
A estrutura MUCH/LITTLE/A LITTLE + OF + THE + SUBST.
I have many friends in Italy.
INCONTÁVEL:
Exemplo:
substantivo plural The little kid ate much/little/a little of the bread we offered
She talks to few people every day. him.

substantivo plural A estrutura MUCH/A LITTLE + FORMA COMPARATIVA:


They will travel to São Paulo in a few weeks Exemplo:
This table is much/a little heavier than that one.
That book is much/a little more interesting than this magazine.
substantivo plural

MORE (mais) também é a forma comparativa de MUCH:


• A estrutura MANY/FEW/A FEW + OF + OBJECT PRONOUN:
Exemplo:
Exemplos:
Shirley always had more time to study than her schoolmates.
The money was given to many/few/a few of us.
A estrutura MANY/FEW/A FEW + OF + THE + SUBSTANTIVO
PLURAL: LESS (menos) é a forma comparativa de LITTLE:
Exemplos: Exemplo:
That old man told a nice tale to many/few/a few of the kids. Shirley always had less time to study than her schoolmates.

Substantivos incontáveis se referindo a coisas individuais ou no


• MORE (mais) é a forma comparativa de MANY: plural nas seguintes estruturas:
Exemplo: Exemplos:
She read more books than we did. information → a/one piece of information → two pieces of
information
• FEWER (menos) é a forma comparativa de FEW: wheat → a/one grain of wheat → three grains of wheat
Exemplo: milk → a/one glass of milk → four glasses of milk
She wrote fewer compositions than we did. sunlight → a/one patch of sunlight → five patches of sunlight

PROMILITARES.COM.BR 227
QUANTIFIERS AND INDEFINITES

Usados antes de substantivos contáveis:


SEVERAL, A COUPLE OF, NONE OF, A GREAT NUMBER OF, A aluminum grief propaganda
LARGE NUMBER OF
anger ground protection
Exemplo:
several trees, a couple of trees, none of the trees apparatus growth psychology

applause hair publicity


Usados antes de substantivos incontáveis:
art happiness purity
A BIT OF, A GOOD DEAL OF, A GREAT DEAL OF, A LARGE
AMOUNT OF assistance harm rain
Exemplo:
atmosphere health reading
a bit of dancing, a good deal of dancing, a great deal of dancing
beauty heat reality
Usados antes de substantivos contáveis e incontáveis beer hair recreation
ALL OF, SOME, MOST OF, ENOUGH, A LOT OF, LOTS OF, PLENTY
OF behavior history relaxation
Exemplo:
biology hockey reliability
he has a lot of food to eat.
he has a lot of friends in Belo Horizonte. boating homework relief

boredom honesty religion


ESTRUTURAS ENFÁTICAS brandy hospitality research
• SO/TOO/VERY + MANY/FEW/MUCH/LITTLE + SUBSTANTIVO
Exemplos: bravery ice respect
Too much noise means very little concentration in the bread ice cream ribbon
classroom.
We know very few people with so many resources. butter ignorance rice

cake immigration sadness


• SO/TOO/VERY + MANY/FEW/MUCH/LITTLE + OF + OBJECT
PRONOUN cancer independence safety
Exemplos: capacity industry salad
The students heard the good news but so many of them did
not like it. chaos inferiority salt
They were offered a lot of food but they ate very little of it. cheese information sand

chemistry ink sauce


• Alguns substantivos incontáveis
childhood insecticide scenery
absence fuel physics
China insurance security

access fun piety Chinese integration sherry

accuracy fur plastic civics intelligence shopping

acoustics furniture pleasure clothing iron significance

adhesive garbage poetry coal irritability silence

admiration gas policy coffee isolation silver

advice generosity pollution cognac jam slang

aerobics genetics porcelain coke jelly sleep

aerodynamics gin poverty comfort jewelry smoking

aeronautics glass power comprehension joy snow

age glue preservative concern juice soap

aggression gold pride conduct junk soccer

agriculture grass productivity confidence justice softness

air gravity progress cotton knowledge soil

228 PROMILITARES.COM.BR
QUANTIFIERS AND INDEFINITES

courage labor soup fear paint wildlife

culture lager spaghetti fertilizer paper wind

currency laughter speed finance participation wine

curry leather statistics fire patience wisdom

dancing leisure status flesh pay wood

darkness liberty steel flour peace wool

death liqueur strength food perfume work

democracy literature stuff foolishness permission worth

deodorant logistics sugar freedom pesticide yarn

depression loneliness sunshine friendship philosophy yoghurt

design lotion superiority fruit photography youth

detergent love survival

disinfectant luck taste INDEFINITES


dust luggage tea
SOME - ALGUM
duty machinery teaching Usados em orações afirmativas.
Exemplos:
dye magic technology
I bought some books.
economics mail tennis She asked me some advice.
education marriage thermodynamics Observação
efficiency mathematics time Note que é utilizado tanto para substantivos contáveis quanto para
incontáveis.
electricity measles tolerance
Quando acompanhado por números, adquire o sentido de
energy meat trade
cerca de.
English mechanics traffic Exemplo:
I met some ten friends at the party.
enjoyment medicine training

entertainment melancholy transportation Também pode aparecer em orações interrogativas quando


houver um oferecimento.
equipment merchandise travel
Exemplos:
estimation mercy trouble Would you like some juice?
ethics metal trust Can I give you some help?

evidence milk truth ANY - QUALQUER / ALGUM / NENHUM


evil money violence Em afirmativas, é traduzido como qualquer.
Exemplos:
evolution mumps virtue
Any topic you study matters.
excitement music vodka Ana is very talkative. She chats to any person who sits next to her.

existence nature warmth


Em interrogativas em geral, traduz-se como algum.
experience news waste Exemplos:
fabric nonsense water Did you learn any topic so far?
Are you going to travel to any place next holiday?
failure oil watermelon

faith ointment wealth Em negativas, usa-se any para evitar a dupla negativa,
traduzido como nenhum.
fame ore welfare
Exemplos:
fashion oxygen whisky Lucas does not approve any attitude his girlfriend takes.
Leila is crabby. She doesn't like any person in her classroom.

PROMILITARES.COM.BR 229
QUANTIFIERS AND INDEFINITES

Usa-se any em IF-clauses. EXERCÍCIOS DE


Exemplos:
Tell me if you need any information. FIXAÇÃO
If you have any doubt, let me know.
01. (UPF-RS) Which is the wrong alternative?
NO - NENHUM
a) There is a lot of coffee in Brazil.
Usados em orações com estrutura afirmativa.
b) There is a lot of people in China.
Exemplos:
c) There is a lot of water in the ocean.
Bruno spent a lot this month! He has no money to lend you.
d) There is a lot of tea in India.
No judgment is free from mistakes.
e) There is a lot of wine in Europe.

Como evitar a dupla negativa: 02. (EFOMM) “_____ students in this class speak________ English”.
“We don’t need no education.” – Pink Floyd a) Fews - some
We need no education. b) The few – something
We don’t need any education. c) Few – some
d) A few – many
“I can’t get no satisfaction.” – The Rolling Stones e) A few – anybody
I can get no satisfaction.
I can’t get any satisfaction. 03. (EN) “Aside from the irrelevant fact that ________ might not then
take the opportunity to read the book”.
NONE - NENHUM(A) a) much people
Funciona como pronome. Nunca estará imediatamente do lado b) much peoples
do termo a que ele se refere, já que a função do pronome é substituir. c) very peoples
Exemplos: d) many people
None of my students gets distracted by social networks. (Refere- e) little people
se a students)
I bought some clothes to my mother but she liked none. (Refere-se 04. (FGV 2009)
a some clothes) UN STUDY FINDS BRAZIL'S WORKING
CONDITIONS UNSATISFACTORY
Formas derivadas de Some, Any e No September 9, 2008

[…] In 2006, the average income of non-black women in Brazil


SOME ANY NO
was 524.6 reais (327.8 U.S. dollars), while that of black women was
Someone/ Anyone/Anybody 367.2 reais (229.5 dollars). The average income of black men in 2006
No one/Nobody - was 451.1 reais (281.9 dollars), while that of non-black men was
Somebody - - Qualquer pessoa,
Ninguém 724.4 reais (452.75 dollars). In addition, Brazilians aged between 16
Alguém alguém, ninguém
and 24 face 3________ difficulties in finding a job than any other age
Anything - group, the study finds. [...]
Something - Algo Qualquer coisa, Nothing - Nada a) any
algo, nada
b) some
Anywhere – c) more
Somewhere – Qualquer lugar, Nowhere – Lugar d) as
Algum lugar algum lugar, lugar nenhum
nenhum e) least

Anyway – 05. (FATEC 2002) Assinale a alternativa que apresenta o uso gramatical
Somehow –
De qualquer jeito, No way – correto de "there is".
De algum jeito, de
de algum jeito, de De jeito nenhum a) There is few teachers and parents talking to the children.
algum modo
jeito nenhum
b) There is some children who need help.
Anytime – A c) There is a lot of frightened adults after the attack.
qualquer d) There is many events happening at the same time.
Sometime – Em momento, em
algum momento algum momento, e) There is much anger among people in New York.
em momento
nenhum 06. (UFSM 2001)
BRITAIN'S INDUSTRIAL HERITAGE
As formas derivadas seguem as mesmas regras dos usos de SOME, During the nineteenth century Britain was transformed
ANY e NO. from a mainly AGRICULTURAL SOCIETY into an industrial one.
This change has been called the INDUSTRIAL REVOLUTION
because of the dramatic effect it had on the British way of
life. People moved to the rapidly EXPANDING TOWNS and cities,

230 PROMILITARES.COM.BR
QUANTIFIERS AND INDEFINITES

RAILWAYS were developed to transport goods around the country and 09. (CESGRANRIO 1990) Which of the following sentences can be
by 1900 Britain had become a major WORLD POWER. The Industrial completed with the word MANY as in: "the laser has many applications"?
Revolution in Britain was built on the use of MACHINE in factories. a) The laser beam is being used by______ telephone companies.
Since the 1950s Britain's manufacturing industries have replaced
the machine operators with computers, and this “automation” b) The laser beam has caused_______ advance in various areas.
has led to a decline in the number, of employees in manufacturing c) Science has gained_____ from the latest applications of the laser.
industries. More manufactured goods are bought and used than ever d) ____ effort has resulted in significant technological improvement.
before but a lot of these goods are imported. By the beginning of
e) Scientists have devoted ______ time to research in the field
the twentieth century other industrial countries, like the USA, were
of communication.
competing with Britain's exports, and countries in the Far East have
been able to provide cheaper products since the 1970s. Areas where
HEAVY MANUFACTURING INDUSTRIES are located have suffered high 10. (UFSM 1999)
UNEMPLOYMENT. CALL WAITING
(LAVERY, Clare. Focus on Britain Today, MacMillan Publishers. p. 98-99.) Newsweek, March 2, 1998.

No segmento "A LOT OF these goods" a expressão em maiúsculo Soon, teenagers will have no excuse for not letting their parents
pode ser substituída, sem alteração do sentido, por: know where they are - and why they'll be home late. Some European
a) “a great deal of”. telecom firms - Sweden's Ericsson, and TIM, the mobile subsidiary of
Telecom Italia - are developing cell phones especially for kids.
b) “a few of”.
The gimmick: to prevent chatty kids ringing up their pals many
c) “more of”. time zones away, the new phones can be programmed to dial only a
d) “the majority of”. few numbers, like home, or a parent's office.
e) “much of”. The phones may debut late this year. And you can bet that by early
next year some kid will have learned a way to subvert the controls.
07. (UFV 2000)
A expressão "a few numbers" indica:
RELIGION AND HUMAN BEHAVIOR
a) número controlado.
Does life have meaning? What gives it meaning? Why do we act
b) grande quantidade.
the way we do? What is the best way to live? How can we find peace?
c) número insuficiente.
These are questions that people have struggled with throughout
history. Philosophers, psychologists, sociologists, and physicists are d) número ilimitado.
among the many thinkers who have tried to give us answers. We look e) número incompleto.
for answers within ourselves, but few are satisfactory. In the end, it is
religion that gives most of the world answers to these questions.
EXERCÍCIOS DE
Hundreds of religions exist in the world, yet all religions try to
answer the same questions. Every religion teaches basic ideas that
help humans understand their nature and their behavior. Every religion
describes two sides of human nature - the animal and the divine. It is
TREINAMENTO
these opposing sides that cause conflicts. Every religion gives people a
method that they can follow to resolve the conflicts. All religions have 01. (EPCAR/CPCAR 2017) Mark the sentence below that CANNOT
a goal, which is in one form or another the transformation of humans answer the following question correctly: “Which prejudice do you
from the animal to the divine. This spiritual transformation is common have?”.
to all religions, though it has many names: nirvana, heaven, salvation.
a) I have some.
All cultures in the world have religious beliefs. For that reason,
every part of life is affected by religions, whose teachings offer b) I don't have any.
guidelines on ways to live. c) I have none.
(WERNER, P. K. Mosaic: a content- based grammar. d) I don't have no prejudice.
New York: Random House, 1985.)

02. (ITA) Assinale a opção cuja frase esteja gramaticalmente correta:


In the sentence "MANY thinkers have tried to give us answers," the
capital word has a meaning close to: a) There is fewer people at the party than Mary expected.
a) a few. b) There is less people at the party than Mary expected.
b) few. c) There are less people at the party than Mary expected.
c) little. d) There are fewer people at the party than Mary expected
d) several. e) There, was less people at the party than Mary expected.
e) much.
03. (PUC-RIO) Mark the item which contains the right choice of FEW/
A FEW/ LITTLE/ A LITTLE to complete the following sentences:
08. (UNESP 1998) Assinale a alternativa que preenche corretamente a
lacuna da frase: “Must you always make so___ noise”? I. ________ politicians realize the importance of solar energy.
a) much II. Would you like _______ light to help you study this map?
b) many III. The new theory is very difficult, but fortunately there are ______
people who understand it.
c) most
IV. Regrettably, the government has _______ power over those who
d) few
are destroying the ozone in the atmosphere.
e) less
a) a few - a few – little - few
b) a little - a little – few - a little

PROMILITARES.COM.BR 231
QUANTIFIERS AND INDEFINITES

c) little - a few - a little - a little TEXTO PARA AS QUESTÕES 08 e 09:


d) few - little - a few - a few
When the first men arrived in Samoa they found blind men who
e) few - a little - a few – little
could see well (____I____) to describe things in detail just by holding
their hands over objects. In France, JUST after the First World War, Jules
04. (EN) Romain tested hundreds of blind people, and found a (____II____)
– Would you like some more coffee? that could tell the difference between light and dark. He narrowed
– No, thanks. I’ve had _________. their photosensitivity down to the nose or in the fingertips.
a) sufficiently
08. (UEL 1998) Assinale a alternativa que preenche corretamente a
b) additionally lacuna I do texto.
c) many a) so
d) enough b) very
e) a few c) enough
d) little
05. (ITA) Dadas as frases:
e) less
I. Only a few of you deserve this prize.
II. Not much of this wheat bread was eaten. 09. (UEL 1998) Assinale a alternativa que preenche corretamente a
III. We’ve heard too many bad news for today. lacuna II do texto.
IV. Drink less coffee and more water. a) some
Podemos afirmar que: b) any
a) todas estão corretas. c) one
b) apenas a II está correta. d) plenty
c) apenas a II e a III estão corretas. e) few
d) somente a I não está correta.
10. (PUC-PR 2000) Mark the correct alternative to fill the gaps of
e) I, II e IV estão corretas. the dialogue below:
At the supermarket...
06. (UFRGS 1998) Leia.
– Wife: Do we need ___I___ wheat?
“My house is haunted by the screams of slow digital death. I'm
referring to the last gasps of the Tamagotchi, a ‘digital craze’ that – Husband: Yes, we do. We haven't got ___II___ wheat.
infected my kids like chicken pox last spring. – Husband: We need ___III___ apples, don't we?
Since the day they were born, I have tried to give my children – Wife: No, we don't. We have got ___IV___ apples. But we have
the right toys. So when I first read about Tamagotchis, I felt here was ___V___ carrots and ___VI___. cheese. Let's get some...
a toy that would appeal to my daughters. In case you are childless a) I - some; II - much; III - any; IV - few; V - many; VI - little
or an alien from outer space and managed to avoid this craze, the
Tamagotchi is a Keychain-size plastic egg that houses a small LCD b) I - much; II - any; III - many; IV - too much; V - few; VI - few
(Liquid Crystal Display) in which ‘lives’ a creature that you nurture by c) I - few; II - some; III - little; IV - many; V - little; VI - little
pushing a variety of buttons. d) I - any; II - much; III - some; IV - many; V - few; VI - little
It was love at first beep for my kids. The silicon pets completely e) I - few; II - many; III - few; IV - no; V - much; VI - many
satisfied their ‘mothering needs’. The Tamagotchi had to be fed,
played with and even changed regularly. A ‘normal’ digital pet lives
for a few weeks, at which point it's ‘called back to the home planet’,
according to the instruction book. Anyway, the thing ‘dies’, so you EXERCÍCIOS DE
have to hit the reset button and grow another one. The cycle repeats
endlessly period before losing interest. But the Tamagotchis, bless their
little chips, keep on beeping, beeping, always beeping”.
COMBATE
A expressão "a variety of buttons" significa o mesmo que:
a) “little buttons”. 01. (EFOMM) Choose the option which completes the sentences
b) “several buttons”. below correctly:
c) “all the buttons”. I. It is ________ use trying to change her mind.
d) “very small buttons”. II. Slowly, ___________children began coming to school.
e) “too many buttons”. III. Unfortunately, he had __________ friends.
IV. Could you possibly give me ___________ help?
07. (EPCAR) “There are a few ways to prevent cyberbullying.” is a) I. a little - II. a few - III. few - IV. little
the same as:
b) I. a little - II. a few - III. little - IV. little
a) there are some ways to prevent cyberbullying.
c) I. a little - II. few - III. few - IV. a little
b) there are many ways to prevent cyberbullying.
d) I. little - II. few - III. little - IV. a little
c) there are lots of ways to prevent cyberbullying.
e) I. little - II. a few - III. few - IV. a little
d) there are no ways to prevent cyberbullying.

232 PROMILITARES.COM.BR
QUANTIFIERS AND INDEFINITES

02. (ITA) Dadas as sentenças: 08. (EFOMM) Complete the sentences with “too many”, “too much”
I. She does not spend much money on her health. or “enough”. Then, choose the correct alternative.
II. Not many of our friends usually travel abroad. 1. - I've been to rather __________ parties recently.
III. I have too much work to do and so little time available. 2. - You're spending far ________ time on your computer.
IV. Bruna says that London is much more exciting than Paris. 3. - Stop. You're asking me _______ questions.
4. - Help! I've got _________ luggage.
Constatamos que, relativamente, as expressões sublinhadas, está(ão)
corretamente empregada(s) na(s) frase(s) número(s) 5. - Oh, sorry, I didn't call you. I didn't have _________ time.
a) II e III. a) too many – too much – too many – too much - enough
b) I, II, e III. b) too much – enough – too many – too much – too much
c) II, III e IV. c) enough – enough – too much - enough – too many
d) I, III e IV. d) too many – too much – enough – too much – enough
e) Todas. e) too much – too many – enough – enough – enough

03. (PUC-PR) Put in the missing words: 09. (EFOMM) Choose the correct option.
I. I want _____ more tea, please. a) Dominoes is not played by many younger people these days.
II. _____ I go fishing. b) Where have you left your luggages?
III. It doesn't rain _____. c) I found many informations about this city.
IV. I'm sorry, but I have _____ to give you. d) Our rubbishes get collected on Thursdays.
V. _____ knows it's wrong. e) I can’t stand doing all these houseworks.
a) I - some; II - Every time; III - someday; IV - nothing; V - Somebody
10. (MACKENZIE 1998) Indicate the alternative that best completes
b) I - any; II - Sometimes; III - every day; IV - anything; V - Everybody the following sentences.
c) I - some; II - Sometimes; III - every day; IV - nothing; V - Everybody A. - I don't like strogonoff. Would you like __(I)__ pizza instead?
d) I - any; II - Every day; III - sometimes; IV - nothing; V - Everyone B. - Oh no! Let's buy __(II)__ loaves of bread and make sandwiches.
e) I - some; II - Everywhere; III - every time; IV - anything; V - Somebody A. - But we have __(III)__ time to do that. We're late for school.
B. - Alright, but we've got __(IV)__ ham and __(V)__ hot dogs.
04. (AFA) In the statement, “[…] we usually say a few words to people
in stores […]” the underlined words may be understood as: a) I - a little; II - a few; III - very little; IV - only a little; V - few
a) Many words could be spoken by people in stores. b) I - only a little; II - many; III - very little; IV - a few; V - any
b) Some words can be said by us. c) I - few; II - very few; III - more; IV - little; V - some
c) Lots of words are used to say people about us. d) I - very little; II - only a few; III - many; IV - a few; V - more
d) A small number of words are said by people in stores. e) I - little; II - much; III - few; IV - a few; V - very little

05. (EFOMM) “Much to Alexandre's surprise, ______________ had


secretly copied his private files from the computer”.
GABARITO
a) Somebody.
b) Somehow. EXERCÍCIOS DE FIXAÇÃO
c) Some. 01. B 04. C 07. D 10. A
d) Someway. 02. C 05. E 08. A
e) Something. 03. D 06. A 09. A
EXERCÍCIOS DE TREINAMENTO
06. (EN) Which of the alternatives completes the sentence correctly? 01. D 04. D 07. A 10. D
“If you need ________about what to remove from your _________ to 02. D 05. E 08. C
avoid problems at check in, this leaflet is for you”.
03. E 06. B 09. E
a) some information – luggage
EXERCÍCIOS DE COMBATE
b) the informations – luggage
01. E 04. B 07. B 10. A
c) information – luggages
02. E 05. A 08. A
d) an information – luggage
03. C 06. A 09. A
e) informations – luggages

07. (EEAR) The sentence: “We have learned nothing”, can be replaced by:
a) we have learned anything.
b) we haven´t learned anything.
c) we haven´t learned something.
d) we haven´t learned everything.

PROMILITARES.COM.BR 233
QUANTIFIERS AND INDEFINITES

ANOTAÇÕES

234 PROMILITARES.COM.BR
GENITIVE CASE

GENITIVE CASE Quando temos duas ou mais pessoas e queremos expressar posse
utilizamos ‘s apenas na última pessoa citada para indicar que todas
Teremos o ‘s para como partícula indicadora de posse, grau as pessoas possuem a mesma coisa ou colocamos ‘s em todas para
de parentesco/relacionamento e autoria. É utilizado com pessoas, indicar que cada uma possui algo diferente.
animais, lugares, organizações e expressões de tempo.
Exemplos:
Estrutura: possuídor + ‘s + objeto possuído.
Jim e Jill’s father.
Exemplos:
Jim’s and Jill’s fathers.
The boy’s name the bird’s egg
the woman’s daughter Mr. Smith’s car
No primeiro caso, entendemos que Jim e Jill são irmãos, uma vez
my father’s motorcycle today’s newspaper que father está no singular e só existe um ‘s denotando posse, ao final
the government’s decision the dog’s ball do último nome.
Já no segundo, temos dois ‘s ao final de cada nome, e fathers está
Observação
no plural. Portanto, um pai para cada pessoa.
Há casos onde devemos utilizar a estrutura possuidor + of + Exemplos:
possuído. É mais utilizada com objetos e com substantivos abstratos.
Jim and Jill’s house.
Exemplos:
Jim’s and Jill’s cars.
the name of the book
the back of the car
No primeiro caso, Jim and Jill moram juntos, portanto uma casa
in the name of love para duas pessoas.
No segundo, Jim and Jill possuem 2 carros, uma para cada.
Quando um nome próprio termina com S, temos a opção de
acrescentar ‘s ou apenas ‘ (apostrofo).
Exemplos: Com substantivos compostos o último termo recebe a partícula
de posse.
Mr. Jones’s book → Mr. Jones’ book.
Exemplos:
Marcos’s eyes → Marcos’ eyes.
My father-in-law’s only child is a woman.
Mrs. Williams’s office → Mrs. Williams’ office.
Stop talking! I can’t hear the police officer’s voice.
Observação Your stepson’s grades are great
Com nomes históricos terminados em –s ou –es usamos apenas (‘).
Exemplos:
This phenomenon is related to Archimedes’ Law. TEXT COMPREHENSION
I live Moses’ story. Text I:
PRESIDENT OBAMA LAUNCHES GUN-VIOLENCE TASK FORCE
Quando temos um substantivos comuns no plural, terminando Five days after deadliest elementary school shooting in U.S.
em S, SÓ ACRESCENTAMOS ‘ (apostrofo). history, President Obama said his administration plans immediate
Exemplos: action early next year on proposals to curb an “epidemic of gun
violence”. At a morning news conference, Obama announced the
The students’ uniform.
formation of a task force to be headed by Vice President Joe Biden
the companies’ staff that will formulate a package of policy recommendations by January.
the nurses’ wordplace. “The fact that this problem is complex can no longer be an excuse for
doing nothing”, Obama said. “The fact that we can’t prevent every
act of violence doesn’t mean that we can’t steadily reduce the violence
Já quando temos um substantivo de plural irregular (que não and prevent the very worst violence.” The president said he intends
termina em S), acrescentamos o ‘s. to push for implementation of the proposals “without delay”. “This is
Exemplos: a team that has a very specific task to pull together real reforms right
The women’s closet. now”, he said. While Obama did not offer specifics, he suggested
the task force would examine an array of steps to curb gun violence
the children’s toys.
and prevent mass shootings, including legislative measures, mental
men’s restroom. health resources and a “look more closely at a culture that all-too
often glorifies guns and violence”. “I will use all the powers of this
office to help advance efforts aimed at preventing more tragedies like
this”, Obama said. Obama made similar pronouncements following

PROMILITARES.COM.BR 289
GENITIVE CASE

at least four other mass shootings that marked his first term. But few 05. (IME) What kind of human behavior is central to the study
policy changes were made. “This is not the first incident of horrific mentioned in the text?
gun violence of your four years. Where have you been?”, asked ABC a) The alienation of the celebrities from the others around them.
News’ Jake Tapper. “I’ve been president of the United States, dealing
with the worst economic crisis since the Great Depression, an auto b) Addiction to technology.
industry on the verge of collapse, two wars. I don’t think I’ve been on c) The high scoring of college students in academic tests.
vacation”, Obama responded. d) The hard work of social psychologists.
Adaptado de http://abcnews.go.com/Politics/OTUS/president-obamalaunches-gun-
e) Excessive positive feelings and admiration of oneself.
violence-task-force/story?id=18015694#UOA3AOTAdyw

06. (IME) What has NOT been encouraging people to act the way
01. (ESPCEX) In the sentence “The fact that this problem is complex described in the text?
can no longer be an excuse for doing nothing...”, this problem refers to a) Technology.
a) task force. b) Narcissism.
b) economic crisis. c) Materialism.
c) police recommendations. d) Psychology.
d) an array of steps. e) Entitlement.
e) gun violence.
Text III:
02. (ESPCEX) In the sentence “This is a team that has a very specific MARCH OF THE MACHINES
task …”, who is the leader of this team?
We built lots of little robots on wheels. We gave them “eyes” to see
a) the press. with, a tiny brain to learn with – the same sort that humans have, but
b) Jake Tapper. much less powerful. And the rest we let them find out for themselves.
c) Joe Biden. We didn’t programme them; we didn’t give them instructions. And
what did they do in the new world they found themselves in? Like
d) the task force.
children, they learned. One of them became a “leader”. When they
e) ABC channel. wanted to, they chose a new leader. And when one of these little robots
became weak, with a low battery, the others ‘bullied’ him. They built
03. (ESPCEX) According to the text, Obama up a whole social order of their own. Yet they were machines with
a) is not trying to solve the mass shootings problem. less brain power than a bee. A brain’s power can be measured by the
number of cell-connections in it. Our robots have 50. A human brain
b) is not worried about gun violence.
has a billion. We humans, their creators, gave our robots just one basic
c) was on vacation during the shooting. instinct – survival. The rest they found out for themselves.
d) requests a task force to try to solve the problem. Developing Grammar in Context, Cambridge University Press
e) decides to take action after talking to the reporter.
07. (EEAR) The text above
04. (ESPCEX) In the sentences “... proposals to curb an “epidemic of a) shows that these machines became less feeble than a bee.
gun violence”...” and “an array of steps to curb gun violence...”, the b) proves the humans develop instincts on the robots insensitively.
word curb means c) describes robots designed to think almost like people.
a) control. d) finds out the power of the robots despite becoming leaders.
b) cultivate.
c) increase. 08. (EEAR) The word ‘built up’ is closest in meaning to
d) encourage. a) discharged.
e) originate. b) intensified.
c) burst.
Text II: d) ejected.
Is ‘Facebook’, the social networking website, making us narcissist?
A new book argues we’re much more self-absorbed nowadays, stating Text IV:
that technology is to blame. I tweet, therefore I am. Or is it, I tweet,
The Parthenon, one of the monuments erected on the sacred rock
therefore I aminsufferable? As if adult celebrities that pop out on the
of the Acropolis, is the major attraction for visitors of the modern
red carpets weren’t clue enough, we now have statistical evidence
city of Athenas. It was built between 447 and 438 b. C. as the holy
that we are a lot more in love with ourselves than we used to be. This
sanctuary of the goddess Athena. A good portion of the Parthenon
social phenomenon has raised fields of research to academic studies
is still standing having withstood centuries of such catastrophes as
nowadays. In the book ‘The Narcissism Epidemic: Living in the Age
earthquakes and bombardments by foreign invaders. However, this
of Entitlement’, Jean M. Twenge, a professor of psychology at San
moment of classical perfection is being slowly eaten away by man’s
Diego State University, and W. Keith Campbell, a social psychologist at
modern catastrophe, pollution. More damage has been done to the
the University of Georgia, look to the Narcissistic Personality Inventory,
Parthenon in the past 30 years than in all the 2.000 years preceding
which measures self-regard, materialism, and lack of empathy. They
them. While parts have been restored, plans are presently being
found that the number of college students scoring high on the test
discussed to remove the temple permanently from its ancient site and
has risen by 30 percent since the early 1980’s.
place it in a museum. Although the Parthenon is still being visited and
admired by thousands of people each year, the question now being
asked is “for how long?”
Source: ECKSTUT, S.; MILLER, T. Interlink. New York: “Prentice Hall”

290 PROMILITARES.COM.BR
GENITIVE CASE

09. (PUC-PR) According to the text, Athena was: 06. (G1) Assinale a alternativa que preenche corretamente a lacuna.
a) a holy sanctuary. How many ____________ nests did you find?
b) rebuilt between 447 and 438 b.C. a) birds’s
c) erected on a rock. b) bird’
d) the first name for Athens. c) birds’
e) a goddess. d) of birds’s
e) of the bird’
10. (PUC-PR) According to the text, the Parthenon has been damaged
by ________. 07. (G1) Assinale a alternativa que preenche corretamente a lacuna.
a) foreign invaders Could anybody please tell me how to find out the ____________?
b) earthquakes a) author address’s
c) men b) author’s address’s
d) catastrophes, bombardments and pollution c) author’s address
e) museums d) author address’
e) author’s addresses’
EXERCÍCIOS DE

FIXAÇÃO 08. (G1) Assinale a alternativa que preenche corretamente a lacuna.


Those dirty shoes are _____________.
a) the children’s
01. (EEAR) All the extracts below, in the text, present a possessive b) the childrens’
noun, except: c) the children’
a) “…the nation’s soccer federation…” d) of the children’
b) “…under Brazil’s military dictatorship…” e) of the children’s
c) “…a reflection of the investigation’s focus…”
d) “…the United States Justice Department’s charging Mr. Marin…” 09. (G1) Assinale a alternativa que preenche corretamente a lacuna.
Unfortunately, the ____________ ended up in the river.
02. (IBFC) I stayed at the house of my brother. Assinale a alternativa a) lady’ hat
correta. b) lady hat’
a) my brother house c) lady’s hat
b) my brother’s house d) ladies’s hat
c) my brother’s houses e) lady hats’
d) my house’s brother
10. (FATEC) Assinale a alternativa que apresenta o uso correto do caso
03. (G1) Assinale a alternativa que preenche corretamente a lacuna. possessivo, como no substantivo “media” em “the media’s collective
Where have you put your _____________? attention”.
a) mother purses’ a) mens’ garment.
b) mother purse’s b) womens’ wear.
c) mother’s purse’s c) mental’s disturbance.
d) mothers’ purse d) children’s clothes.
e) mother’s purse e) disappointment’s feeling.

04. (G1) Assinale a alternativa correta.


EXERCÍCIOS DE

TREINAMENTO
My little puppy doesn’t seem to like your _________ red handkerchief.
a) father’s-in-law
b) father-in-law’s
c) father’-in-law
d) father-in-law’ 01. (EPCAR) Mark the option which shows the same meaning as in “a
person’s body size or weight.”
e) fathers’-in-law
a) A person and body size or weight.
05. (G1) Assinale a alternativa que preenche corretamente a lacuna. b) The body size or weight of a person.
It will take us ____________ to fix this machine. c) Body size and person’s weight.
a) a whole day’s work d) The body’s size or weight of a person.
b) a whole’s day work
c) a day’s whole work
d) a whole’s work day
e) a whole day work’s

PROMILITARES.COM.BR 291
GENITIVE CASE

02. (CN) Mark the INCORRECT option according to the Genitive Case. d) “that’s a fact”;
a) The door’s car is open. e) “student’s best friend”.
b) The world’s population is increasing.
c) I’m going to Grandma’s. 09. (UFRGS) The use of ‘S is the same in ANYONE’S PARTYING SPIRIT
and in
d) Which are Fred’s and Eric’s cars?
a) Everyone’s invited for Carnival in Rio.
e) This is my mother-in-law’s house.
b) The American’s luggage was checked carefully.
03. (PM-BA) “So the government’s told the country’s menfolk to c) My friend Jeremy’s arrived.
shed their ties” d) Nobody’s pleased with the situation.
The’ s in “government’s” is the e) The Mexican tourist’s coming tomorrow.
a) contraction of is.
b) contraction of was. 10. (UNITAU) Assinale a alternativa que corresponde à tradução mais
adequada da frase a seguir:
c) contraction of has.
My mother’s maid has just bought the dog’s meat.
d) plural ending.
a) Minha mãe e a empregada acabam de comprar a carne do
e) genitive case. cachorro.
b) A empregada de minha mãe acaba de comprar a carne do
04. (UFAL) cachorro.
KING JUAN CARLOS’S DEMOCRATIC LEGACY IS ROOTED IN CLICHE c) Minha mãe acabou de fazer a carne do cachorro.
The Spanish king had no choice but to champion democracy. We d) Minha mãe fará compras com a empregada e o cachorro.
should remember the part played by ordinary people. e) Minha mãe é empregada e comprou carne de cachorro.
Disponível:<http://www.theguardian.com/commentisfree/2014/jun/03/
king-juan-carlos-legacy-spaincliche-democracy>. Acesso em: 03 jun. 2014.

No texto acima, os termos grifados correspondem, respectivamente, EXERCÍCIOS DE

COMBATE
aos elementos gramaticais:
a) plural e advérbio
b) conjunção e advérbio
c) pronome e substantivo 01. (AFA) Mark the option which shows the same meaning as in
d) substantivo e conjunção “Americans’ dependence on family”.
e) caso possessivo e adjetivo a) The family’s dependence on Americans’.
b) The Americans family dependence.
05. (CEV-URCA) You can find an example of genitive case in:
c) The Americans dependence of family’s.
a) “They are going to miss him when he’s gone.”
d) The dependence of Americans on family.
b) “It’s not so much his massive wealth or media influence…”
c) “Berlusconi´s not just any old charlatan.”  02. (AFA) No one could avoid noticing my _______ or ________
d) “…because he’s created it through his media influence.”  dresses during the party.
e) “Italy’s greatest weaknesses and its worst instincts.”  a) wives / daughter
b) wife / daughter’s
06. (EPCAR) Choose the option that has the same meaning of c) wife’s / daughter’s
“Parents usually don’t know their child is a cyberbully.”
d) wive’s / daughter’s
a) Parents’ child usually don’t know he is a cyberbully.
b) Parents child’s usually don’t know he is a cyberbully. 03. (AFA) Choose the correct answer to complete the sentence:
c) Child parents’ usually don’t know he is a cyberbully. The __________ offices are very modern.
d) Child’s parents usually don’t know he is a cyberbully. a) businessmen’
b) businessmens’
07. (UFRGS) Which of the alternatives below has the same structure
c) businessmans’
as The fans’ transgression of bourgeois taste?
d) businessmen’s
a) The government’s interference with middle class education.
b) The students’s discussion and their argument. 04. (ITA) Assinalar a alternativa onde o uso do caso genitivo esteja
c) The girl’s reading an interesting book. CORRETO:
d) The fan’s harsh criticizing went unnoticed. a) For goodness’ sake, this is my brother-in-law’s dog.
e) The teacher’s introduced quite complex matters. b) For goodness’ sake, this is my brother’s-in-law dog.
c) For goodness sake’s, this is my brother-in-law’s dog.
08. (UFF) Mark the option in which the apostrophe S is used as in
d) For goodness sake’s, this is my brother’s-in-law dog.
“curiosity’s virtue”.
e) For goodness sake’s, this is my brother-in-law dog’s.
a) “it’s the inner signal”;
b) “that’s still unknown”;
c) “it’s usually directed”;

292 PROMILITARES.COM.BR
GENITIVE CASE

05. (ESPCEX) The correct sentence is:


ANOTAÇÕES
a) My father’s friend called me yesterday.
b) The table’s leg is broken.
c) I have an appointment at the office’s doctor.
d) My brother neighbour’s sister is a nurse.
e) The girls school is far from St Bartholomew’s.

06. (EFOMM) Betty, Jane and I were invited to a party at __________


home.
a) your friend’s Carol
b) our friend Carol
c) our friend Carol’s
d) your friend’s Carol’s
e) her friends’ Carol

07. (EFOMM) His __________ sickness is worrying him very much.


a) mother’s-in-law
b) mother-in-law
c) mother’s-in-law’s
d) mother-in-law’s
e) mothers-in-law’s

08. (CFOE) The option which contains the use of the same rule of
Genitive Case (’s) applied to -from kids’ bodies is:
a) girls’ c) walrus’
b) Louis’ d) childrens’

09. (COMD AERONÁUTICA) Choose the alternative in which the (’s)


means the same as belongs to.
a) Mary’s an air hostess.
b) The pilot’s talking to the tower.
c) The pilot’s eaten a sandwich.
d) Mary’s uniform is fashionable.

10. (PUC-SP) Assinale a alternativa correta:


Ben and Bill are brothers. Do you know _______?
a) Ben and Bill’s parents
b) Ben’s and Bill parents
c) Ben’s and Bill’s parents.
d) Bem and Bill parents’
e) Bem and Bill parents’s

GABARITO
EXERCÍCIOS DE FIXAÇÃO
01. D 04. B 07. C 10. D
02. B 05. A 08. B
03. E 06. C 09. C
EXERCÍCIOS DE TREINAMENTO
01. B 04. E 07. A 10. B
02. A 05. E 08. E
03. C 06. D 09. B
EXERCÍCIOS DE COMBATE
01. D 04. A 07. D 10. A
02. C 05. A 08. A
03. D 06. C 09. D

PROMILITARES.COM.BR 293
GENITIVE CASE

ANOTAÇÕES

294 PROMILITARES.COM.BR
ARTICLES AND ADJECTIVE
DEGREE

INDEFINITE ARTICLES • Ideia anafórica (retoma um termo que foi mencionado anteriormente):
Exemplos:
A/AN – UM(A) I bought a book. The book is a best-seller.
Usados apenas com substantivos contáveis e no singular. A student asked a good question. The question was about articles.
A – Antes de sons consonantais e semivocálicos
AN – Antes de sons vocálicos • Com títulos e sobrenomes no plural (representando a família inteira):
Exemplos: Exemplos:
AN elephant A horse A European girl The President, The Queen, The Emperor, The Congresswoman.
AN hour A house AN intelligent woman Cuidado: Queen Elizabeth, King Henrique VIII (não usamos o
AN honor A unit artigo quando o nome próprio aparece junto ao título).
A table A uniform The Simpsons, The Kardashians, The Smiths.

USOS • Períodos históricos, décadas, séculos, eras e quando o dia vem


antes do mês na data:
• Antes de profissões:
Exemplos:
A teacher; He is a student.
The Renascence, the Middle Age, the 90s, the first of May, the
tenth of April.
• Com ideia indefinida ou representando o numeral 1:
I saw a girl before the class; a glass of water; a hundred.
• Países com nomes compostos com ideia de união (Union, United
or Republic) ou nome plural:
• Antes da locução (adjetivo + substantivo): Exemplos:
It is a beautiful night. The United Kingdom, The Republic of China, The Philippines, The
Bruna is an interested student. United States, The Czech Republic.

• Com alguns quantificadores/expressões numéricas: a lot, a few, a • Objetos únicos na espécie (geralmente corpos celestiais) e
little, a bit, a dozen, a couple, a great deal of… números ordinais:
Exemplos:
• Nas expressões que indicam duração de velocidade, tempo ou The world, the moon, the sun, the planet, the Earth, the satellite.
preço: seven days a week, twenty-four hours a day, sixty minutes The first, the last, the second, the third.
an hour, 80km an hour, 5 reais a kilo

• Nomes de museus, bibliotecas e monumentos:


• Após o advérbio what indicando surpresa: What a surprise! What
Exemplos:
a nonsense!
The Metropolitan Museum of Art, the National Gallery, the statue
of liberty.
• Em expressões como: It’s a pity! (É uma pena!), It’s a shame! (É
uma vergonha!) It’s a tragedy! (É uma tragédia!)
• Com nomes de aeronaves e embarcações:
Exemplos:
DEFINITE ARTICLE – THE
The Titanic, The Concorde, The Mayflower

USOS
• Ideia definida (retira da generalização e especifica o objeto em • Nomes de desertos, rios, mares, oceanos, canais, florestas (exceto
questão). nomes de praia, lagoa e lago):

Exemplos: Exemplos:

Love is hard to define but the love I feel is pure. The Amazon River, The Pacific Ocean, The Black Sea, The Panama
Canal, The Amazon rainforest.
Water is a precious liquid but the water we consume in Brazil is
not the purest.

PROMILITARES.COM.BR 235
ARTICLES AND ADJECTIVE DEGREE

• Nomes de montanhas, lagos e ilhas no plural (no singular não se Exemplos:


usa o artigo definido): Joana is more beautiful than her neighbor Julia.
Exemplos: João is more intelligent than Roberto.
The Andes, the Alps, the Bahamas, The Falkland Islands, The
Taurus Mountains.
Os comparativos de igualdade são usados com qualquer
adjetivo ou advérbio, independente do número de sílabas.
• Com adjetivos no grau superlativo ou substantivados: Exemplos:
Exemplos: Australia is as hot as Brazil.
The most important, the best, the greatest. Pedro is not as tall as Felipe.
The strong should help the weak. We cannot run as fast as horses.
Martha is as beautiful as Daniela.
• Com instrumentos musicais, nomes de jornal, organizações:
Exemplos: Os comparativos de inferioridade são usados com qualquer
The drums, the guitar, the bass. adjetivo ou advérbio, independente do número de sílabas.
The New Yorker, The Washington Post, The New York Times. Exemplos:
The United Nations, The International Monetary Fund, The World Bruna is less beautiful than Carla.
Health Organization. This house is less valuable than that one.
Observação Bruno is less tall than Carlos.

OMISSÃO DO THE SUPERLATIVO


Substantivos como church, hospital, school, prison, jail, sea, work
etc, quando aparecem no contexto cumprindo o propósito óbvio, SUPERIORIDADE INFERIORIDADE
o artigo definido deve ser omitido.
the ... –est
Exemplos:
(o(a) mais, os(as) mais...)
I go to church on Sundays. the least...
ou
Tourists went to the church yesterday to take some photos. (Não (o(a) menos, os(as) menos...)
foram à igreja rezar/orar) the most ...

Students go to school every weekday. (o(a) mais, os(as) mais...)

My mother is coming to the school to bring my lunch because


I forgot it at home this morning. (Minha mãe não está vindo à Os superlativos com the ... –est são usados com adjetivos e
escola estudar.) advérbios de até duas sílabas.
Exemplos:
That city is the coldest in the country.
ADJETIVOS COMPARATIVOS E My garden is the liveliest in town.
SUPERLATIVOS - ADJECTIVES
COMPARATIVE AND SUPERLATIVE Os superlativos com the most ... são usados com adjetivos e
advérbios com mais de duas sílabas.
COMPARATIVO Exemplos:
Pablo is the most foolish person of the office.
SUPERIORIDADE IGUALDADE INFERIORIDADE
This exercise is the most difficult of the book.
as...as...
-er ... than (tão...como/ Os superlativos de inferioridade são usados com qualquer
quanto) adjetivo ou advérbio, independente do número de sílabas.
(mais que...)
ou less... than Exemplos:
ou
not as...as (menos... que) Rita is the least tall of all the girls I know.
more... than
not so...as Lisa is the least beautiful woman of the course.
(mais que...)
(não tão...como/
quanto)
Os adjetivos abaixo, que terminam em -y e –ly, podem
formar o comparativo e superlativo de superioridade da
seguinte forma.
Os comparativos de superioridade com -er são usados com
adjetivos e advérbios de até duas sílabas. Exemplos:
Exemplos: heavy / heavier than / the heaviest
Americans are taller than Indians. handy / handier than / the handiest
Today is colder than yesterday. lonely / lonelier than / the loneliest
happy / happier than / the happiest
Os comparativos de superioridade formados com more ... than lively / livelier than / the liveliest
são usados com adjetivos e advérbios com mais de duas sílabas. funny / funnier than / the funniest

236 PROMILITARES.COM.BR
ARTICLES AND ADJECTIVE DEGREE

Os adjetivos e advérbios abaixo, embora tenham duas Exemplos:


sílabas, podem ser usados com o comparativo more ... than e Those teachers are getting / becoming fatter.
superlativo com the most. São exceções à regra de até duas
sílabas. Those students are getting / becoming fatter and fatter.

Exemplos: Those policemen are getting / becoming more and more fat.

afraid / more afraid than / the most afraid Those girls are getting / becoming more beautiful.

frequent / more frequent than / the most frequent Those women are getting / becoming more and more beautiful.

certain / more certain / the most certain


correct / more correct / the most correct A estrutura de “parallel” increase, ou crescimento em
paralelo (= quanto mais..., mais/menos... / quanto menos...,
exact / more exact / the most exact mais/menos...), é formada por the + comparativo de superioridade
often / more often / the most often (com -er ou more) e ... the + comparativo de superioridade (com
seldom / more seldom / the most seldom -er ou more).
modern / more modern / the most modern Exemplos:
normal / more normal / the most normal The hotter the weather is, the better for all of us.
recent / more recent / the most recent The more careful you are with your child, the less worried
you’ll be.
The better the computer, the more expensive it is.
Os adjetivos quiet, simple, clever, common, gentle, narrow,
pleasant e polite aceitam as duas formas comparativas de
superioridade. Adjetivos com duas consoantes e apenas uma vogal entre
Exemplos: essas consoantes dobram a última consoante.
clever / cleverer than / more clever than / the cleverest / the most Exemplos:
clever sad / sadder / the saddest
narrow / narrower than / more narrow than / the narrowest / the thin / thinner / the thinnest
most narrow fat / fatter / the fattest
shallow / shallower than / more shallow than / the shallowest /
the most shallow
EXERCÍCIOS DE

Alguns adjetivos e advérbios apresentam formas irregulares


para o comparativo e para o superlativo de superioridade. FIXAÇÃO
Exemplos:
good / well / better than / the best 01. (PUC-CAMP) Assinale a alternativa em que os artigos, a ou an
bad / badly / worse than / the worst foram incorretamente empregados.
little / less than / the least a) The General was wearing a uniform whose buttons were all
many / much / more than / the most made of gold.
far / farther than / the farthest (para indicar distância) b) Thank you, Sir, it’s an honor to me having worked with you.
further than / the furthest (para indicar tempo) c) Please, hand me a dozen eggs.
old / older than / the oldest (para pessoas, animais, coisas e eventos) d) What will you have, a coffee, a tea or water?
elder / the eldest (somente para pessoas, e da mesma família) e) A lot of people consider you a hero.
elder nunca é seguido de than. É usado em estruturas como “my elder
brother ...” 02. (EPCAR/AFA 2015)
JOBS AT HIGH RISK
Utilizamos as estruturas comparativas de superioridade It is an invisible force that goes by many names. Computerization.
referindo-se a quantidades ou a um extremo na quantidade Automation. Artificial intelligence. Technology. Innovation. And,
(muitas vezes essa regra vale para qualidades): everyone's favorite, ROBOTS. Whatever name you prefer, some form
Exemplos: of it has been stimulating progress and killing jobs - from tailors to
paralegals - for centuries. But this time is different: nearly half of
She spent less on clothes than Anna did. American jobs today could be automated in "a decade or two". The
The Hotel Copacabana has the most rooms. question is: which half?
The most gracious was Bahia. Another way of posing the same question is: Where do machines
work better than people? Tractors are more powerful than farmers.
Robotic arms are stronger and more tireless than assembly-line
A estrutura de gradual increase, ou crescimento gradual
workers. But in the past 30 years, software and robots have succeeded
(= cada vez mais, mais e mais), é formada através de be getting/
replacing a particular kind of occupation: the average-wage, middle-
becoming + uma forma comparativa de superioridade (com -er
skill, routine- heavy worker, especially in manufacturing and office
ou more) repetida ou não.
administration.
Indeed, it's projected that the next wave of computer progress will
continue to endanger human work where it already has: manufacturing,
administrative support, retail, and transportation. Most remaining
factory jobs are "likely to diminish over the next decades". Cashiers,
counter clerks, and telemarketers are similarly endangered. On the

PROMILITARES.COM.BR 237
ARTICLES AND ADJECTIVE DEGREE

other hand, health care workers, people responsible for our safety, a) the – the – a
and management positions are the least likely to be automated. b) the – its – a
The Next Big Thing c) the – it’s – the
We might be on the edge of an innovating moment in robotics d) an – it’s – the
and artificial intelligence. Although the past 30 years have reduced the
middle, high- and low-skill jobs have actually increased, as if protected e) an – its – the
from the invading armies of robots by their own moats. Higher-skill
workers have been protected by a kind of social-intelligence moat. 04. (UPE 2015)
Computers are historically good at executing routines, but they're SLEEPING ON STILTS IN THE AMAZON
bad at finding patterns, communicating with people, and making As 75-year-old villager Antônio Gomes told us stories of growing
decisions, which is what managers are paid to do. This is why some up in Boca do Mamirauá, a tiny settlement in the northern Amazon
people think managers are, for the moment, one of the largest rainforest, I tried to ignore the tiny blue flies biting through my trousers.
categories immune to the fast wave of AI. Meanwhile, lower-skill Despite my interest in hearing how locals survive in this remote part
workers have been protected by the Moravec moat. Hans Moravec of the Brazilian rainforest, now a part of the Mamirauá Sustainable
was a futurist who pointed out that machine technology copied a Development Reserve, I was grateful to escape when he finished,
savant infant: Machines could do long math equations instantly and finding refuge in one of the tall wooden houses. The houses hover
beat anybody in chess, but they can't answer a simple question or some 3m above the ground. They are not unusual: almost everything
walk up a flight of stairs. As a result, not skilled work done by people in the Mamirauá reserve is on stilts, even the chicken coop. It has to
without much education (like home health care workers, or fast-food be. Although much of Brazil is currently suffering one of the worst
attendants) have been saved, too. droughts in decades, this part of the Amazon is almost completely
The Human Half flooded for the six-month wet season. By April, the end of the rainy
In the 19th century, new manufacturing technology replaced what season, the river rises up to 10m high and overflows its banks. As a
was then skilled labor. In the second half of the 20th century, however, result, all living things in the forest, including locals, must adopt an
software technology took the place of median-salaried office work. amphibious lifestyle. Even the jaguars have learned to adapt by living
The first wave showed that machines are better at assembling things. in tree branches when the floods arrive.
The second showed that machines are better at organizing things. Only 1,000 tourists per year are allowed to visit Mamirauá, which,
Now data analytics and self-driving cars suggest they might be better at 57,000sqkm, is the largest wildlife reserve in the country. Created in
at pattern-recognition and driving. So what are we better at? 1984 to save the once-endangered uakari monkey, the reserve is the
The safest industries and jobs are dominated by managers, health- most carefully managed and protected part of the Amazon – and is
care workers, and a super-category that includes education, media, also home to what many consider Brazil’s most successful sustainable
and community service. One conclusion to draw from this is that tourist resort, the Uakari Floating Lodge. “If [the reserve] had not been
humans are, and will always be, superior at working with, and caring created,” guide Francisco Nogeuira said, “the rivers and lakes would be
for other humans. In this light, automation doesn't make the world empty of fish, and who knows how many trees would remain today?”
worse. Far from it: it creates new opportunities for human creativity. (Disponível em: http://www.bbc.com/travel/feature
/20140626-sleeping-on- stilts-in-the-amazon)
But robots are already creeping into diagnostics and surgeries.
Schools are already experimenting with software that replaces In the last paragraph, it is possible to find sentences in:
teaching hours. The fact that some industries have been safe from
a) comparative of equality.
automation for the last three decades doesn't guarantee that they'll
be safe for the next one. b) comparative of superiority.
It would be anxious enough if we knew exactly which jobs are next c) superlative of superiority.
in line for automation. The truth is scarier. We don't really have a clue. d) superlative of inferiority.
(Adapted from http://www.businessinsider.com/
e) comparative of inferiority.
robots-overtakingamerican- jobs-2014-1)

Glossary: 05. (ITA 2014) Substituindo os adjetivos long e comprehensive,


savant infant – a child with great knowledge and ability respectivamente, por easy e rich na oração “Harvard conducted
to assemble – to make something by joining separate parts
to creep – to move slowly, quietly and carefully
one of the longest and most comprehensive studies of human
development”, teremos:
Mark the option that contains an adjective in the same form as in “The a) the most easy - the richest
safest industries and jobs are dominated by managers [...]”. b) the easiest - the most rich
a) “The truth is scarier.” c) the more easy - the richer
b) “[...] the least likely to be automated.” d) the easiest - the richest
c) “Where do machines work better than people?” e) the most easy - the most rich
d) “Tractors are more powerful than farmers.”
06. (CN 2017) Complete the sentences using an article when necessary.
03. (UFRGS 2014) “Fan is 1__________ abbreviated form of fanatic, I. Is Mario __________ honest man?
which has 2__________ roots in 3__________ Latin word fanaticus,
which simply meant belonging to the temple, a devotee”. […] II. The students wear __________ uniform here.
(Adapted from: JENKINS, Henry. Textual Poachers: Television Fans and III. __________ Smiths live next to the supermarket.
Participatory Culture. New York / London: Routledge, 1992. p. 12-16.) IV. __________ Brasilia was made the capital in 1960.
Select the alternative which correctly fills in the gaps in references 1, 2 Choose the correct option:
and 3, in the order they appear.
a) a / an / - / - d) a / an / - / The
b) a / an / The / The e) an / a / The / -
c) an / a / The / The

238 PROMILITARES.COM.BR
ARTICLES AND ADJECTIVE DEGREE

07. (EEAR 2016) Select the alternative that best completes the normal sleep times, with most healthy adults naturally needing
extract below. seven to nine hours of sleep per night, according to the National
WORKPLACE ACCIDENT STATISTICS Sleep Foundation. Those over 65 need about seven to eight
hours, on average, while teenagers need eight to 10 hours,
Every year, millions of people in _____ United States are hurt on and school-age children nine to 11 hours. People’s performance
the job. Each day 16 workers die from injuries at work and more than continues to be poor while they are sleep deprived, Dr. Veasey said.
17,000 are injured. Accidents also cause the companies to spend Health issues like pain, sleep apnea or autoimmune disease can increase
more. Last year, _____ total cost was more than $121 billion. people’s need for sleep, said Andrea Meredith, a neuroscientist at the
a) the – a University of Maryland School of Medicine. A misalignment of the
b) a – the clock that governs our sleep-wake cycle can also drive up the need for
sleep, Dr. Meredith said. The brain’s clock can get misaligned by being
c) the – an
stimulated at the wrong time of day, she said, such as from caffeine
d) the – the in the afternoon or evening, digital screen use too close to bedtime,
or even exercise at a time of day when the body wants to be winding
08. (UFSM 2004) Leia. down.
EUROPE IS BORN (http://well.blogs.nytimes.com. Adaptado.)

In spring 1950 Europe was on the edge of the abyss. With the
No trecho do primeiro parágrafo “The more you deprive yourself of
onset of the Cold War, the threat of conflict between its eastern
sleep over long periods of time, the less accurate you are of judging
and western halves loomed over the continent. Five years after the
your own sleep perception”, os termos em destaque indicam:
end of World War Two, the old enemies were still 11a long way from
2
reconciliation. a) finalidade.
What could be done to avoid repeating the mistakes of the past b) preferência.
and to create the right conditions for a 1lasting peace between such c) proporcionalidade.
recent enemies? The nub of the problem was the relationship between d) exclusão.
France and Germany. 10A link had to be forged between the two and
all the 5free countries in Europe had to be 3united around them so that e) substituição.
they could work together on building a community with 9a shared
destiny. It was Jean Monnet, with his unique wealth of experience as 10. (EN 2016) Which is the correct way to complete the paragraph
a negotiator and man of peace, who 4suggested to the French Foreign below?
Minister, Robert Schuman, and the German Chancellor, Konrad No language is easy to learn well, though languages which are
Adenauer, that 8a community of interest be established between their related to our first language are __________. Learning a completely
countries, in the shape of a jointly managed market in coal and steel different writing system is a huge challenge, but that does not
under the control of an independent authority. The proposal was necessarily make a language __________ another. In the end, it
officially tabled by France on 9 May 1950, and was warmly received is impossible to say that there is one language that is __________
by Germany, Italy, the Netherlands, Belgium and Luxembourg. language in the world.
The treaty establishing the first European Community. the (Adapted from www.usingenglish.com)
European Coal and Steel Community or ECSC, was eventually
signed in April 1951, opening up the door to 12a Europe of 6practical a) easier – more difficult – harder
achievements. Further achievements were to follow until we finally b) the easiest – more difficult – harder
reached the European Union as it is today, 7a Union now opening up c) as easy as – the most difficult – the hardest
to the eastern half of the continent from which it has too long been
separated. d) easier – more difficult than – the hardest
(http://europa.eu.int/abc/obj/chrono/40years/7days/en.htm 06/06/03) e) the easiest – more difficult than – the harder

No fragmento "a Union", há uma eufonia. O mesmo processo ocorre em:


EXERCÍCIOS DE
a) “a community” (ref. 8).
b) “a shared destiny” (ref. 9).
c) “A link” (ref. 10).
TREINAMENTO
d) “a long way” (ref. 11).
e) “a Europe” (ref. 12). 01.
WHY BILINGUALS ARE SMARTER
09. (UNESP 2017)
Speaking two languages rather than just one has obvious practical
QUESTION: IS THERE ANYTHING I CAN DO TO benefits in an increasingly globalized world. But in recent years, scientists
TRAIN MY BODY TO NEED LESS SLEEP? have begun to show that the advantages of bilingualism are even more
Karen Weintraub June 17, 2016 fundamental than being able to converse with a wider range of people.
Being bilingual, it turns out, makes you smarter. It can have a profound
Many people think they can teach themselves to need less sleep, effect on your brain, improving cognitive skills not related to language
but they’re wrong, said Dr. Sigrid Veasey, a professor at the Center for and even protecting from dementia in old age.
Sleep and Circadian Neurobiology at the University of Pennsylvania’s
This view of bilingualism is remarkably different from the
Perelman School of Medicine. We might feel that we’re getting by fine
understanding of bilingualism through much of the 20th century.
on less sleep, but we’re deluding ourselves, Dr. Veasey said, largely
Researchers, educators and policy makers long considered a second
because lack of sleep skews our self-awareness. “The more you
language to be an interference, cognitively speaking, that delayed a
deprive yourself of sleep over long periods of time, the less accurate
child’s academic and intellectual development. They were not wrong
you are of judging your own sleep perception,” she said.
about the interference: there is ample evidence that in a bilingual’s
Multiple studies have shown that people don’t functionally brain both language systems are active even when he is using only
adapt to less sleep than their bodies need. There is a range of one language, thus creating situations in which one system obstructs

PROMILITARES.COM.BR 239
ARTICLES AND ADJECTIVE DEGREE

the other. But this interference, researchers are finding out, isn’t 02. (UNIOESTE 2012)
so much a handicap as a blessing in disguise. It forces the brain to BRAZIL POLICE OCCUPY RIO FAVELA IN WORLD CUP OPERATION
resolve internal conflict, giving the mind a workout that strengthens
its cognitive muscles. Bilinguals, for instance, seem to be more adept Brazilian security forces have occupied one of Rio de Janeiro's
than monolinguals at solving certain kinds of mental puzzles. In a biggest slums as part of a major crackdown ahead of the 2014 World
2004 study by the psychologists Ellen Bialystok and Michelle Martin- Cup and 2016 Olympics.
Rhee, bilingual and monolingual preschoolers were asked to sort blue Some 800 police and special forces moved into the Mangueira
circles and red squares presented on a computer screen into two shantytown, without needing to fire a shot, having announced the
digital bins — one marked with a blue square and the other marked raid in advance.
with a red circle. In the first task, the children had to sort the shapes by The slum – or favela – is close to Rio's famous Maracana stadium,
color, placing blue circles in the bin marked with the blue square and where the World Cup final will be played.
red squares in the bin marked with the red circle. Both groups did this
The pre-dawn operation involved armoured vehicles and helicopters.
with comparable ease. Next, the children were asked to sort by shape,
According to the newspaper, O Globo, leaflets were thrown out
which was more challenging because it required placing the images in
of the helicopters, some with photos of wanted criminals. Others
a bin marked with a conflicting color. The bilinguals were quicker at
were printed with the police special forces' telephone number so
performing this task. The collective evidence from a number of such
that residents could pass on information about drugs traffickers or
studies suggests that the bilingual experience improves the brain’s
weapons.
so-called executive function — a command system that directs the
attention processes that we use for planning, solving problems and BBC Brazil correspondent Paulo Cabral says most of Mangueira's
performing various other mentally demanding tasks. These processes residents co-operated with the operation, as they want to rid the area
include ignoring distractions to stay focused, switching attention of drug dealers.
willfully from one thing to another and holding information in mind He says that Rio's authorities are making an effort to gain the
— like remembering a sequence of directions while driving. trust of those living in the slums, who – after decades of abuse – have
Why does the fight between two simultaneously active language got used to seeing the police as their enemy. Mangueira – home to
systems improve these aspects of cognition? Until recently, researchers one of Rio's most famous samba schools – is the 18th favela that the
thought the bilingual advantage was centered primarily in an ability authorities have occupied recently.
for inhibition that was improved by the exercise of suppressing one (Adapted from: http://www.bbc.co.uk/news/world-latin-america-13833037)
language system: this suppression, it was thought, would help train
the bilingual mind to ignore distractions in other contexts. But that The only option that does not contain an adjective used in the
explanation increasingly appears to be inadequate, since studies have superlative form is:
shown that bilinguals perform better than monolinguals even at a) Mangueira is one of Rio de Janeiro's biggest slums.
tasks that do not require inhibition, like threading a line through an b) Mangueira is close to Rio's famous Maracanã stadium.
ascending series of numbers scattered randomly on a page.
c) Mangueira is the home to one of Rio's most famous samba
The bilingual experience appears to influence the brain from schools.
infancy to old age (and there is reason to believe that it may also apply
to those who learn a second language later in life). In a 2009 study led d) The oldest public park of Brazil is located in Rio de Janeiro.
by Agnes Kovacs of the International School for Advanced Studies in e) Maracanã is known as one of the largest football stadiums in the
Trieste, Italy, 7-month-old babies exposed to two languages from birth world.
were compared with peers raised with one language. In an initial set
of tests, the infants were presented with an audio stimulus and then 03. (PUC-MG 2010)
shown a puppet on one side of a screen. Both infant groups learned
GETTING REAL ABOUT THE HIGH PRICE OF CHEAP FOOD
to look at that side of the screen in anticipation of the puppet. But in
By Bryan Walsh Friday, Aug. 21, 2009.
a later set of tests, when the puppet began appearing on the opposite
side of the screen, the babies exposed to a bilingual environment Horror stories about the food industry have been with us since
quickly learned to switch their anticipatory gaze in the new direction 1906, when Upton Sinclair’s novel The Jungle told ugly truths about
while the other babies did not. how America produces its meat. Nowadays, things have got much
Bilingualism’s effects also extend into the twilight years. In a better, and in some ways much worse. The U.S. agricultural industry
recent study of 44 elderly Spanish-English bilinguals, scientists led by can now produce unlimited quantities of meat and grains at remarkably
the neuropsychologist Tamar Gollan of the University of California, cheap prices. But it does so at a high cost to the environment, animals
San Diego, found that individuals with a higher degree of bilingualism and humans. Some of those hidden prices are the erosion of fertile
— measured through a comparative evaluation of proficiency in each farmland and the rise of antibiotic-resistant bacteria among farm
language — were more resistant than others to the beginning of animals.
dementia and other symptoms of Alzheimer’s disease: the higher the Some Americans are noticing such warnings and working
degree of bilingualism, the later the age of occurrence. to transform the way the country eats — farmers who are raising
Nobody ever doubted the power of language. But who would sustainable food in ways that don't ruin the earth. Documentaries
have imagined that the words we hear and the sentences we speak and the work of journalists are reprising Sinclair's work, awakening a
might be leaving such a deep imprint? sleeping public to the realities of how we eat. Change is also coming
(Adapted from http://www.nytimes.com/2012/03/18/ from the very top. First Lady Michelle Obama's White House garden
opinion/sunday/the- benefitsof-bilingualism.html) has so far raised a lot of organic produce — and tons of powerful
symbolism. Nevertheless, despite increasing public awareness,
Mark the incorrect option: sustainable agriculture, remains a tiny enterprise: according to
a) […] the advantages of bilingualism are even more fundamental recent data from the U.S. Department of Agriculture, less than 1%
than being able to converse […] of American cropland is farmed organically. Sustainable food is also
b) […] with a wider range of people. pricier than conventional food and harder to find.
c) […] the understanding of bilingualism through much of the Unless Americans radically rethink the way they grow and consume
20th century. food, they face a future of eroded farmland and high health costs.
Sustainable food has an elitist reputation, but each of us depends on
d) The Bilinguals were quicker at performing this task.
the soil, animals and plants. And as every farmer knows, if you don't

240 PROMILITARES.COM.BR
ARTICLES AND ADJECTIVE DEGREE

take care of your land, it can't take care of you. face for each of these four categories, which were shown to a separate
(Adapted from: http://www.time.com/time/health/article/0,8599,1917458,00.html.) set of participants who had to pick which face appeared either more
trustworthy or stronger. Most of the participants found the computer-
The problem with organic food is that it is _______________________ generated averages to be good representations of trustworthiness or
than conventional food. strength – and generally saw the average “financial advisor” face as
a) more expensive and more difficult to find more trustworthy and the “powerlifter” face as stronger. The findings
from all four surveys were published in the Personality and Social
b) unhealthier and extremely more caloric Psychology Bulletin on June 18.
c) more fattening and harder to digest (Adaptado de www.scientific.american.com/article/your-facial-bone-
d) more harmful and more dangerous strecture- has-a-big-influence-on-how-people-see-you. Acesso em 20/8/2015)

Todas as frases abaixo contêm adjetivo com flexão de grau, exceto:


04. (ITA 2016)
a) […] photos of oneself convey more these days than snapshots
YOUR FACIAL BONE STRUCTURE HAS A BIG
ever did back in the Kodak era.
INFLUENCE ON HOW PEOPLE SEE YOU
b) […] it can be a stressful task to select the photo that conveys the
(…) Selfies, headshots, mug shots – photos of oneself convey more
best impression of ourselves.
these days than snapshots ever did back in the Kodak era. Most digitally
minded people continually post and update pictures of themselves at c) […] participants picked faces with happier expressions as
professional, social media and dating sites such as LinkedIn, Facebook, financial advisors […]
Match.com and Tinder. For better or worse, viewers then tend to d) […] and [participants] selected broader faces as belonging to
make snap judgments about someone’s personality or character from power-lifting champs.
a single shot. As such, it can be a stressful task to select the photo that e) […] and generally saw the average “financial advisor” face as
conveys the best impression of ourselves. For those of us seeking to more trustworthy […]
appear friendly and trustworthy to others, a new study underscores an
old, chipper piece of advice: Put on a happy face.
05. (UPE 2012) Considere o texto.
A newly published series of experiments by cognitive
Trying to predict what will happen as our planet warms up is not
neuroscientists at New York University is reinforcing the relevance
easy. We know that ice at the poles is melting and this is making sea
of facial expressions to perceptions of characteristics such as
levels rise. Warmer temperatures are likely to change other aspects
trustworthiness and friendliness. More importantly, the research also
of the weather. Some countries, such as those in North Africa, may
revealed the unexpected finding that perceptions of abilities such as
become ____I___ , while other areas, such as Northern Europe, may
physical strength are not dependent on facial expressions but rather
become _______II________. There will probably be more storms,
on facial bone structure.
droughts, and flooding.
The team’s first experiment featured photographs of 10 different (Adaptado de Impact of climate change. In: The New Children’s
people presenting five different facial expressions each. Study subjects Encyclopedia. London: 2009. p. 78)
rated how friendly, trustworthy or strong the person in each photo
appeared. A separate group of subjects scored each face on an As lacunas I e II no texto acima podem ser completadas, de forma
emotional scale from “very angry” to “very happy.” And three experts correta e na mesma sequência, pela opção:
not involved in either of the previous two ratings to avoid confounding a) more hotter and drier — more colder and wetter
results calculated the facial width-to-height ratio for each face. An
analysis revealed that participants generally ranked people with a b) as hotter and drier — as colder and wetter
happy expression as friendly and trustworthy but not those with angry c) hotter and drier — colder and wetter
expressions. Surprisingly, participants did not rank faces as indicative d) most hotter and dry — most colder and wet
of physical strength based on facial expression but graded faces that
e) the hotter and drier — the colder and wetter
were very broad as that of a strong individual.
In a second survey facial expression and facial structure were 06. (EPCAR/AFA 2012)
manipulated in computer-generated faces. Participants rated each
face for the same traits as in the first survey, with the addition of a HOW TO BECOME A STUNT DOUBLE
rating for warmth. Again, people thought a happy expression, but A stunt double stands in for the actor when the action or fight
not an angry one, indicated friendliness, trustworthiness — and in scene gets dangerous or goes beyond the capabilities of the actor. To
this case, warmth. The researchers then showed two additional sets become a stunt double, you must be in excellent physical condition
of participants the same faces, this time either with areas relevant and have special skills.
to facial expressions obscured or the width cropped. In the first INSTRUCTIONS:
variation, for faces lacking emotional cues, people could no longer
1. Exercise regularly if you want to become a stunt double. Eat
perceive personality traits but could still perceive strength based on
nutritiously for optimal health and strength.
width. Similarly, for those faces lacking structural cues, people could
no longer perceive strength but could still perceive personality traits 2. Take lots of lessons because the more skills you have,
based on facial expressions. the better. Gymnastics is extremely important in becoming
a stunt double. Get good at trampoline, skateboarding,
In a third iteration of the survey participants had to pick four faces
swimming and high board diving. Take scuba diving lessons.
out of a lineup of eight faces varied for expression and width that
Practice rock climbing and horseback riding. Learn to water ski
they might select either as their financial advisor or as the winner of a
and snow ski.
power-lifting competition. As might be expected, participants picked
faces with happier expressions as financial advisors and selected 3. Enroll in martial arts classes, especially judo. Judo is excellent for
broader faces as belonging to power-lifting champs. learning how to break falls.
In a final survey the researchers generated more than 100 4. Get training in CPR1 and First Aid. This training looks good on a
variations of one individual “base face” by varying facial features. résumé, especially for stunt double careers. Injuries happen.
Participants saw two faces at a time, and then picked one as either 5. Have valid driver's licenses for both car and motorcycle. Take
trustworthy or high in ability or as a good financial advisor or power- advanced driving classes so you'll be qualified for difficult driving
lifting winner. Using these results, a computer then created an average scenes.

PROMILITARES.COM.BR 241
ARTICLES AND ADJECTIVE DEGREE

6. Move to Hollywood and plan to work your way up from the different part of town. 2The rent is cheaper, but the move pretty much
bottom. You must get into the Screen Actors Guild2 and have a wiped out my savings.
union card3. Some of you may think that I’m a loser: an unmarried adult with
(Taken from Google) not much money. The old me would have been way too embarrassed
to admit all this. I was filled with useless pride. But I honestly don’t
1 - Cardiopulmonary resuscitation.
2 - Annual prize promoted by the American Syndicate of Actors. care about things like that any more. The reason is very simple: I’m
3 - A card certifying membership in an organization. perfectly happy just as I am. The reason? I got rid of most of my
material possessions.
Look at the bold comparative form (item 2). Choose the option that Minimalism is a lifestyle in which 3you reduce your possessions
contains a similar construction. to the least possible. Living with only the bare essentials has not only
a) The earlier we get there, the more likely we are to get good seats. provided superficial benefits such as the pleasure of a tidy room or the
b) More and more people travel to England. simple ease of cleaning, 4it has also led to a more fundamental shift.
It’s given me a chance to think about what it really means to be happy.
c) The smoothest Channel crossing you’ll ever have! Why not fly to
France with British Airways? It’ll be the best decision you’ve ever We think that 5the more we have, the happier we will be. 6We
made. never know what tomorrow might bring, so we collect and save as
much as we can. This means we need a lot of money, so we gradually
d) Our new jets are now far more luxurious. start judging people by how much money they have. You convince
yourself that you need to make a lot of money so you don’t miss out
07. (EFOMM 2018) Which alternative is correct? on success. And for you to make money, you need everyone else to
a) I visited the United Kingdom and the Brazil two years ago. spend their money. And so it goes.
b) The Bahamas is a group of islands in the West Indies. So I said goodbye to a lot of things, many of which I’d had for
c) Sahara is the largest hot desert in the world. years. And yet now I live each day with a happier spirit. 7I feel more
content now than I ever did in the past.
d) The coast of the country is bathed by Atlantic Ocean.
I wasn’t always a minimalist. I used to buy a lot of things, believing
e) My brother likes seaside, but I prefer mountains. that all those possessions would increase my self-worth and lead to
a happier life. I loved collecting a lot of useless stuff, and I couldn’t
08. (EN 2017) What is the correct option to complete the text below? throw anything away. I was a natural hoarder of knick-knacks that I
THE CURRENT SMARTPHONE MARKET thought made me an interesting person.
It’s __________ brand new year and already there have been At the same time, though, I was always comparing myself with
__________ new smartphones released onto the market. other people who had more or better things, 8which often made
me miserable. I couldn’t focus on anything, and I was always
Upgrading from your current device can be __________ exciting
wasting time. Alcohol was my escape, and I didn’t treat women
time, but it can also be __________ little confusing with all of
fairly. I didn’t try to change; I thought this was all just part of who
__________ options available.
I was, and I deserved to be unhappy.
(http://www.news.com.au)
My apartment wasn’t horribly messy; if my girlfriend was coming
a) a / - / the / - / - d) an / the / - / the / an over for the weekend, I could do enough tidying up to make it look
presentable. On a usual day, however, there were books stacked
b) the / a / an / the / an e) the / - / a / a / the
everywhere because there wasn’t enough room on my bookshelves.
c) a / - / an / a / the Most I had thumbed through once or twice, thinking that 9I would
read them when I had the time.
09. (UNIFOR 2014) Leia as sentenças abaixo e marque a opção correta The closet was crammed with what used to be my favorite
de acordo com o uso dos artigos definido e indefinido: clothes, most of which I’d only worn a few times. The room was filled
I. Can you play a guitar? with all the things I’d taken up as hobbies and then gotten tired of.
II. I once played the guitar which had only five strings. A guitar and amplifier, covered with dust. Conversational English
workbooks I’d planned to study once I had more free time. Even a
III. She started learning the piano at the age of five.
fabulous antique camera, 10which of course I had never once put a
IV. I’ve always had a flute, ever since I was a child. roll of film in.
V. I’m afraid the violin is an instrument I never mastered. 11
It may sound as if I’m exaggerating when I say I started to
become a new person. Someone said to me: “All you did is throw
a) Todas estão corretas. things away,” which is true. 12But by having fewer things around, I’ve
b) Todas estão incorretas. started feeling happier each day. I’m slowly beginning to understand
c) Apenas os itens I e II estão errados. what happiness is.
d) Apenas os itens III, IV, e V estão errados. If you are anything like I used to be – miserable, constantly
comparing yourself with others, or just believing your life sucks – 13I
e) Apenas os itens I, III e V estão corretos.
think you should try saying goodbye to some of your things. […]
Everyone wants to be happy. But trying to buy happiness only makes
10. (ITA 2018) us happy for a little while.
GOODBYE THINGS, HELLO MINIMALISM: 1CAN (adaptado de <https://www.theguardian.com/books/2017/apr/12/goodbye-things-
LIVING WITH LESS MAKE YOU HAPPIER? hello-minimalism-can-living-with-lessmake-you-happier>. Acesso em: 21 mai. 2017.)

Fumio Sasaki owns a roll-up mattress, three shirts and four pairs of
Todas as frases abaixo usam a forma comparativa do adjetivo, exceto:
socks. After deciding to scorn possessions, he began feeling happier.
He explains why. a) The rent is cheaper, (ref. 2)
Let me tell you a bit about myself. I’m 35 years old, male, single, b) […] you reduce your possessions to the least possible. (ref. 3)
never been married. I work as an editor at a publishing company. I c) […] the more we have, the happier we will be. (ref. 5)
recently moved from the Nakameguro neighbourhood in Tokyo, d) I feel more content now than I ever did in the past. (ref. 7)
where I lived for a decade, to a neighbourhood called Fudomae in a
e) But by having fewer things around, (ref. 12)

242 PROMILITARES.COM.BR
ARTICLES AND ADJECTIVE DEGREE

EXERCÍCIOS DE 08. (EFOMM) Choose the correct alternative to complete the

COMBATE sentences below.


I. Simon is in ______ prison because he didn’t pay his taxes.
II. You have made ____ very good progress.
III. We didn’t have time to visit ____ Louvre when we were in Paris.
01. (AFA) _____ man I don't know has called you but didn't leave any
IV. I’ve always wanted to visit ____ Netherlands.
message. _____ man just told me he'd call again during _____ week.
a) An - A - an c) The - A - the a) a / a / the / the
b) A - The - the d) The - The - a b) --- / --- / the / the
c) the / a / --- / ---
02. (EFOMM) I think you drive ______________ than your husband.
d) --- / a / --- / a
a) careful
e) a / --- / --- / the
b) carefully
c) more carefully 09. (EN) What is the correct option to complete the text below?
d) more careful MOSQUITO SCREENS TO BE USED AT RIO GAMES
e) most careful Even as athletes grow increasingly concerned about _______
outbreak of _______ Zika virus in Brazil, _______ organizing
03. (AFA) “Many adolescents act this way because they feel frustrated committee for the August Olympics in Rio de Janeiro said it would
or angry […]”. The comparative form of the underlined word is: charge national delegations to have mosquito screens on athletes'
rooms, _____ screens, one measure Brazilians are using to help
a) more angry.
ward off the mosquito that is the primary transmitter of Zika, will be
b) angrier than. installed in communal areas "where required," but affixed to lodging
c) more angrier. only if national delegations decide to pay for it, said Philip Wilkinson,
d) more angry than. ______ spokesman for the Rio 2016 organizing committee.
a) an / - / the / - /an
04. (ITA) Dadas as sentenças: b) the / the / the / the / a
I. Would you like to go to the movies with me? c) the / a / the / the / a
II. The Atlantic and The Pacific are very big oceans. d) an / the / - / the / a
III. I think she is the most beautiful girl in the neighborhood. e) the / the / the / the / an
Constatamos que está(ão) correta(s), relativamente ao uso do artigo the:
10. (EN) Which sequence best completes the quotation below?
a) apenas a I.
_____ Ebola outbreak in West Africa is already _______ global
b) apenas a II. threat to _______public health and it's vital that ______ UK remains at
c) apenas a III. _______ forefront of responding to ______epidemic.
d) apenas a II e III. Michael Fallon, Defense Secretary, 2014.
(Adapted from http://www.royalnavy.mod.uk)
e) todas as sentenças.
a) An / a / the / X / the / an
05. (EFOMM) Choose the alternative that correctly shows the
b) X / the / X / the / X / an
comparative form of the adjectives below.
c) The / the / the / X / the / the
far - good – bad – easy – old
d) An / X / X / the / X / X
a) further – best – worst – easier – oldest
e) The / a / X / the / the / the
b) farther – better – worse – easiest – older
c) further – better – worse – easier – elder
d) farther – best – worse – easier – oldest
GABARITO
06. (AFA) “Michael played the piano ______ when he was _______.
Now he stopped practicing”. EXERCÍCIOS DE FIXAÇÃO
a) better / younger 01. D 04. C 07. D 10. D

b) very well / newest 02. B 05. D 08. E 11. D

c) well / more young 03. E 06. E 09. C

d) more right / more young EXERCÍCIOS DE TREINAMENTO


01. C 04. A 07. B 10. B
07. (AFA) Choose the option which shows the same kind of 02. B 05. C 08. C
comparison in the underlined adjective in “friendship is considered to 03. A 06. A 09. C
be closer than association”.
EXERCÍCIOS DE COMBATE
a) Americans have no best friends.
01. B 04. E 07. C 10. E
b) While less restricted in Russia.
02. C 05. C 08. B
c) Friendships are often more intense than relationships.
03. B 06. A 09. B
d) Everyone has at least one best friend.

PROMILITARES.COM.BR 243
ARTICLES AND ADJECTIVE DEGREE

ANOTAÇÕES

244 PROMILITARES.COM.BR
ADVERBS

ADVÉRBIOS - ADVERBS widely → bastante – They differed widely.


Como em português, os advérbios modificam o sentido de um late → tarde – He arrived late.
verbo, de um adjetivo, ou de um outro advérbio. lately → ultimamente – I haven’t seen him lately
Exemplos:
She carried the heavy box carefully. (mudando verbo) • Tipos de advérbios:
The policeman got prudently suspicious. (mudando o adjetivo) Manner – (modo) fast, carefully, easily etc.
You really dance very well. (mudando o advérbio) Place – (lugar) here, there, everywhere etc.
Time – (tempo) yesterday, now, today etc.
• Formação dos advérbios Frequency – (frequência) always, never, usually etc.
Formamos a maioria dos advérbios (principalmente de modo e de Degree – (intensidade) very, too, quite etc.
intensidade) acrescentando-se ly ao adjetivo.
Exemplos: POSIÇÃO DOS ADVÉRBIOS
slow → slowly • Adverbs of manner (advébio de modo) – são colocados
general → generally geralmente no fim da oração.
clear → clearly
ADVERB OF
extreme → extremely SUBJECT VERB OBJECT
MANNER
heavy → heavily (o y final torna-se i)
easy → easily She can speak English fluently
comfortable → comfortably (a terminação able e ible perdem o
e final) Pedro works hard
marketable → marketably
simple → simply • Adverbs of place (advébio de lugar) – os advérbios de lugar,
Note: good → well assim como as locuções adverbiais de lugar, são geralmente
colocados no fim da oração.
• Adjetivos e advérbios que apresentam a mesma forma:
ADVERB OF
Exemplos: SUBJECT VERB OBJECT
PLACE
fast → fast – That was a fast train. That train goes fast.
hard→ hard – It’s a hard homework. He has worked hard. She put It on the table
high → high (alto) That is a high mountain. Some birds fly high.
low → low (baixo) She spoke in a low voice. Can you speak low? They are at home
late → late (atrasado/tarde, tardio) Don’t come at a late hour.
Don’t come late.
early → early (adiantado, cedo) Mary is early today. They got up • Adverbs of time (advérbio de tempo) – os advérbios de tempo,
very early. assim como as locuções adverbiais de tempo definido, podem
aparecer no início ou fim da oração, o que é mais comum.
enough → enough (suficiente, suficientemente) Do you have
enough money to lend me? You are smart enough to do it.
ADVERB ADVERB
SUBJECT VERB OBJECT
OF TIME OF TIME
• Advérbios com duas formas e significados diferentes.
Yesterday I saw him
Exemplos:
near → perto – He came near.
He died yesterday
nearly → (almost) quase – He nearly died.
high → alto – It is flying high.
highly → muito – She was highly admired. • Adverbs of frequency (advébio de frequência) – são
hard → árduo – He works hard. colocados:
hardly → (barely, scarcely), quase nunca – He hardly works. a) antes do verbo principal;
wide → totalmente – Our eyes were wide open. b) depois do auxiliar.

PROMILITARES.COM.BR 245
ADVERBS

intolerance are well-known, such as race. But others are less


AUXILIAR ADVERB OF MAIN acknowledged, even if more common:
SUBJECT
VERB FREQUENCY VERB
[…]
your Ability: Usually called ableism, a less well-known form of
You never study
lessons prejudice is discrimination against people with visible disabilities such
as those in wheelchairs or with a learning disability. The disabled face
in discrimination not only from their peers, but from institutions, schools,
Davi has always lived
Mexico employers, and landowners who are hesitant to accommodate the
disabled.
Sue is seldom at home
[…]
(Adapted from https://aloftyexistence.wordpress.com)

• Adverbs of degree (advérbios de intensidade) – geralmente In the sentence “Usually called ableism, a less well-known form of
aparecem diante da palavra (adjetivo, advérbio ou verbo) que prejudice”, the underlined expression means:
estiverem modificando.
a) always. c) frequently.
Exemplos:
b) rarely. d) seldom.
She can hardly walk. → verb
We are deeply sorry. → adjective 02. (ACAFE 2017) Answer the question based on the text below.
She dances very well. → adverb The Brazilian government has ratified its participation in the Paris
agreement on climate change, a significant step by Latin America’s
largest emitter of greenhouse gases that could spur other countries
Posições especiais dos advérbios:
to follow suit.
Quando tivermos os três tipos de advérbios – modo, lugar e
With a landmass larger than the continental US, Brazil emits
tempo – numa mesma oração, eles seguirão a seguinte ordem: modo,
about 2.5% of the world’s carbon dioxide and other polluting gases,
lugar e tempo.
according to United Nations data.
Exemplo:
“Our government is concerned about the future,” said President
Paul swam fast across the river this morning. Michel Temer during a signing ceremony in Brasilia. “Everything we do
↓ ↓ ↓ today is not aimed at tomorrow, but rather at a future that preserves
modo lugar tempo the living conditions of Brazilians.”
Temer said Brazil’s ratification would be presented formally to the
Observação UN later this month.
The Paris agreement will enter into force once 55 countries
Entretanto, com verbos de movimento (go, leave, travel) a
representing at least 55% of global emissions have formally joined it.
posição dos advérbios é a seguinte: lugar, modo e tempo.
Climate experts say that could happen later this year.
Exemplo:
Countries set their own targets for reducing emissions. The
They left for Italy by plane last week. targets are not legally binding, but nations must update them every
↓ ↓ ↓ five years. Using 2005 levels as the baseline, Brazil committed to
lugar modo tempo cutting emissions 37% by 2025 and an “intended reduction” of 43%
by 2030.
In the last decade, Brazil has achieved significant emissions cuts
Inversão negativa: thanks to efforts to reduce deforestation in the Amazon and increase
Para dar ênfase, alguns advérbios com ideias negativas podem ser in the use of energy from hydropower and other renewable sources
usados no início da oração. Nesses casos, faz-se necessário inverter o including wind, solar and biomass.
sujeito e o verbo como se fosse uma interrogativa. The Paris accord got a boost earlier this month when the US
Exemplos: president, Barack Obama, and China’s President, Xi Jinping, sealed
their nations’ participation.
He hardly composes music, but he plays brilliantly.
“Brazil is now the next major country to move forward. It will
Hardly does he compose music, but he also plays brilliantly.
add even greater momentum,” said David Waskow, director of the
International Climate Initiative at the Washington, DC-based think
Outros advérbios que podem iniciar orações causando tank the World Resources Institute.
inversão: (Source: https://www.theguardian.com/environment/2016/sep/13/
brazil-ratifies-paris-agreement-with-pledge-to-sharplyreduce-emissions.)
never, rarely, seldom, scarcely, little; e as expressões not only …
but also, no sooner …than.
From the words in bold below, which is not an adjective in the text:
a) Renewable sources (7th paragraph)
EXERCÍCIOS DE b) Polluting gases (2nd paragraph)

FIXAÇÃO c) Significant emissions (7th paragraph)


d) Legally binding (6th paragraph)

03. (EEAR 2016) In the sentence “Maria learns fast”, the word “fast”,
01. (EPCAR/CPCAR 2017) in bold type, is:
MOST COMMON PREJUDICES a) a noun. c) an adverb.
What are some of the most common ways people discriminate b) an article. d) an adjective.
against each other? Some of the areas where people show their

246 PROMILITARES.COM.BR
ADVERBS

04. (EEAR 2019) Read the text and answer the question. 07. (UERJ 2019) “The global health community has largely come to
It was a beautiful summer afternoon with the sun shining brightly. realize that public health preparedness is crucial”. (ref. 1)
I excitedly phoned my friends asking them to come over later for a Another word from the text that may replace the underlined one
barbecue. above without significant change in meaning is:
After making the calls I quickly drove into town to buy some a) widely
food and drink. __________ I arrived at the shops I was very surprised b) effectively
at __________ busy it was. Everyone must have been shopping for a
barbecue! c) particularly

The first butchers I visited had completely run out of sausages. The d) similarly
next shop had some left so I happily bought some. After visiting a few
more shops I had finally finished my shopping. But I was starting to 08. (UERJ 2019) “the question is not if another outbreak will happen,
worry as it was 6 p.m. and I had invited my friends to visit at 6:30 p.m. but when” (ref. 2)
I hastly rushed to the car park with all of my shopping and threw it The underlined words present the health community’s opinion
in the boot. Suddenly I notice that the cars were moving really slowly concerning new outbreaks of epidemics.
out of the car park and there was a bit of traffic jam. It was past 6:30 According to their opinion, future outbreaks are seen as:
p.m. __________ I arrived home and I was extremely worried. As a a) unlikely.
drove into my drive I smiled happily when I saw my friends sitting in
my front garden. Luckily they had realized I was stuck at the shops and b) certain.
they waited for me. c) probable.
(Fonte: www.bbc.co.uk/skillswise) d) impossible.

The four words in bold, in the text, are:


09. (EPCAR/CPCAR 2018) Read the text below and answer the
a) nouns. c) pronouns. question(s) according to it.
b) adverbs. d) adjectives. SOME OF THE INTERNET'S CRAZIEST CONSPIRACY THEORIES
[…]
05. (IME 2018) Leia.
THE EARTH IS HOLLOW
A DAY IN THE LIFE OF A NUCLEAR MATERIALS ENGINEER
Don't give up, readers. We're halfway through this list. We can
[...] make it to the end. Dig deep. Well, not 1too deep. You see, the Earth is
I definitely don’t have a “typical day”. I sometimes have a plan, hollow and accessible via portals at the north and south poles. Luckily
but _________ stick to it as much of my work is responsive to situations though, it's quite habitable down there, providing excellent living
which are transient. The range of things I can get involved in is huge quarters for the lost Viking colonies of Greenland and the Nazis, while
and includes specifying materials for use in challenging environments, "aliens" are in fact just visitors from the subterranean areas.
new plant designs and decommissioning activates. […]
(RATHBONE, Penny. Adapted from: The Guardian. A day in the life of a nuclear (Adapted from http://www.shortlist.com/entertainment/20-of-the-internets-
materials engineer. Disponível em: <https://www.theguardian.com/women-in- craziestconspiracy-theories/. Acesso em: 14 de fev 2017.)
leadership/2016/jan/22/a-day-in-the-life-of-a-nuclear-materials-engineer>.
Acesso em: 22/06/2017.)
Mark the alternative that CANNOT replace the word “too” (ref. 12)
Escolha a alternativa que completa corretamente a lacuna do texto. in the text.
a) rarely c) daintily e) chiefly a) extremely
b) endlessly d) primarily b) also
c) very
06. (UEMG 2013) In the line, “we talk too much, love too seldom, and d) so
hate too often”, what kind of adverb is seldom?
a) manner c) degree 10. (UNESP 2018) Examine a tira para responder à(s) questão(ões) a
b) frequency d) place seguir.

TEXTO PARA AS QUESTÕES 07 e 08:


THE EFFECT OF CLIMATE CHANGE ON EPIDEMIC RISK
[…]
It is important to take note of the impact of climate change on
epidemic risk, but it is equally important to prepare for its impact
on global health. 1The global health community has largely come
to realize that public health preparedness is crucial to responding
efficiently to infectious disease outbreaks. For this reason, our work
is, then, centered around helping governments manage and quantify No trecho do terceiro quadrinho “We’re not that dumb!”, o termo
infectious disease risk. Besides, regardless of weather patterns, em destaque pode ser substituído, sem alteração de sentido, por:
insights into epidemics and into mechanisms for ensuring adequate a) so.
support are critical for managing this risk.
b) which.
Since the public health community agrees that 2the question is
not if another outbreak will happen, but when, the steps we take in c) over.
the coming years to prepare for and reduce the increasing frequency d) more.
of outbreaks will determine the broader implications these diseases e) quite.
have on our world.
(contagionlive.com)

PROMILITARES.COM.BR 247
ADVERBS

EXERCÍCIOS DE 03. (UFRGS 2016) O segmento “deeply felt issues” tem a mesma

TREINAMENTO estrutura de:


a) “Well distributed indeed”.
b) “Costly building solutions”.
c) “Hard working rules”.
01. (MACKENZIE 2012)
d) “Often discussed themes”.
e) “Elderly experienced people”.

04. (PUC-MG 2015) Read the following passage and choose the
option which best completes the question, according to the text:
BUBBLEWS PAYS USERS TO USE SOCIAL NETWORK
Social media startup Bubblews wants to revolutionize social
networking by paying users to share their content. “It’s a new
perspective on the social networking model,” said Arvind Dixit, the
GO EASY ON YOURSELF 26 year-old Bubblews CEO (Chief Executive Officer). “One of our
By Stuart Bradford fundamental beliefs is that when someone is providing a service to
you, you should compensate them.” Bubblews is sharing a portion of
Do you treat yourself as well as you treat your friends and family? its ad revenue with users, paying a penny for a like, view, or comment
That simple question is the basis for a new area of psychological on its site. Payments are made to a ‘Bubblews bank’ and, after
research called self-compassion — how _____(I)_____ people view reaching $50, can be cashed out.
themselves. People who find it _____(II)_____ to be supportive and At least one analyst thinks that the Bubblews model could
understanding to others often score _____(III)_____ low on self- shake things up in a social networking world dominated by titans
compassion tests. They get _____(IV)_____ with themselves for like Facebook and Twitter. "The idea that social media users will
perceived failures like being overweight or not exercising. continue to contribute to networks for free has a fixed limit, because
most people have bills to pay and should, over time, favor sites that
ANXIOUS
share profits with them over those that do not,” wrote Rob Enderle,
The research suggests that accepting our imperfections may be the principal analyst at tech research firm Enderle Group.
first step toward better health. People who score high on tests of self-
Bubblews is encouraging people to share their passions in a
compassion have less depression and anxiety, and tend to be happier
minimum of 400 characters, supplemented by photos. 1Although in
and more optimistic. Preliminary data suggest that self-compassion
the last stage of testing up until now, the site has over 20 million
can even influence how much we eat and may help some people lose
visitors a month from more than 240 countries. Each post on Bubblews
weight. This idea does to contrast with the advice of many doctors
is known as a ‘bubble’, and the site uses a plus sign (+) instead of
and self-help books, which suggest that willpower and self-discipline
hashtags.
are the keys to better health. But Kristin Neff, a pioneer in the field,
says self-compassion is not to be confused with self-indulgence or Jason Zuccari, Bubblews president told FoxNews.com that the site
lower standards. has already had a positive financial impact on members of its user
community. “People aren’t going to be able to quit their job, but it’s
“I found in my research that the biggest reason people aren’t
good to add things to their life that they never had before,” he said,
more self-compassionate is that they are afraid they’ll become
using the example of a woman in the Philippines who was able to buy
self-indulgent,” said Dr. Neff, an associate professor of human
a refrigerator.
development at the University of Texas at Austin. “They believe self-
(http://www.foxnews.com. Acesso: 16/07/2014. Adaptado.)
criticism is what keeps them in line. Most people have gotten it wrong
because our culture says being hard on yourself is the way to be.”
The words up until now in “Although in the last stage of testing up
(www.nytimes.com)
until now” (ref. 1) convey an idea of:
The words that properly fill in the blanks I, II, III and IV in the text are: a) place. c) finality.
a) kind, easily, surprising and anger. b) time. d) conclusion.
b) kind, easily, surprise and angrily.
05. (ITA 2014)
c) kindly, easy, surprisingly and angry.
A HISTORY OF PI
d) kindly, easily, surprisingly and angrily.
The history of Pi, says the author, though a small part of the
e) kind, ease, surprising and anger. history of mathematics, is nevertheless a mirror of the history of
man. Petr Beckmann holds up this mirror, giving the background
02. (CN 2017) Read the sentences below. of the times when Pi made progress — and also when it did not,
I. I work hardly every day. because3science was being stifled by militarism or religious fanaticism.
II. My classmates speak French very well. The mathematical level of this book is flexible, and there is plenty for
readers of all ages and interests.
III. Ana drives incredibly fast.
IV. Our father is a very carefully driver. ABOUT THE AUTHOR
Petr Beckmann was born in Prague, Czechoslovakia, in 1924.
Choose the option according to the correct use of the adverbs and Until 1963, he worked as a research scientist for the Czechoslovak
modifiers. Academy of Sciences, when he was invited as a Visiting Professor to
a) Only the sentences I, III and IV are correct. the University of Colorado, where he decided to stay permanently as
b) Only the sentences I and III are correct. professor of electrical engineering.
c) Only the sentences II and III are correct. Dr. Beckmann has authored 11 books and more than 50
scientific papers, 1mostly on probability theory and electromagnetic
d) Only the sentences II, III and IV are correct.
wave propagation. History is one of his side interests; another is
e) Only the sentences III and IV are correct.

248 PROMILITARES.COM.BR
ADVERBS

linguistics (he is fluent in five languages and he has worked out a new At the Facebook 1developer 2conference in September, 3the
generative grammar which enables a computer to construct trillions of company announced the release of a 4product called Timeline, 5which
grammatical sentences from a dictionary of less than 100 unprocessed offers a 6highly visual view of a user’s Facebook profile and organizes
words). content into photos, events and apps, all based on a 7timeline view
He also publishes a monthly pro-science, pro-technology, pro- that stretches back to the beginning of a user’s time on Facebook.
free enterprise newsletter Access to Energy, in which he promotes the Timeline is designed to work on 8mobile devices, too.
viewpoint that clean energy can be made plentiful, but that access to (Adaptado de: WYLD, Adrian. Facebook. Disponível em:
<http://topics.nytimes.com>. Acesso em 01 dez. 2011.)
it is blocked by government interference and environmental paranoia.
(BECKMANN, Petr. A History of Pi. New York: Barnes & Noble Books, 1983.)
A melhor tradução para a palavra highly (ref. 6), como empregada
no texto, é:
Indique o item lexical que pode substituir o sublinhado no trecho “...
mostly on probability theory and electromagnetic wave propagation.” a) altamente. c) especial. e) completa.
(ref. 1), sem prejudicar o seu sentido. b) favoravelmente. d) levemente.
a) absolutely c) inherently e) utterly
b) chiefly d) randomly 09. (UNESP 2013)
BRAZIL WANTS TO COUNT TREES IN THE AMAZON RAINFOREST
06. (MACKENZIE 2013) Read the following text. By Channtal Fleischfresser
February 11, 2013
DO YOU WANT TO KNOW A SECRET?
John Lennon & Paul McCartney
Recorded 11 February 1963

Given away simultaneously to fellow Brian Epstein protégé Billy


J.Kramer (for a hit single), and to George Harrinson (for this LP), “Do
You Want To Know A Secret?” was a Lennon composition – inspired
by a line he remembered from a Disney song that his mother used to
sing. “I thought it would be a good vehicle for George because it only
had three notes and he wasn’t the best singer in the world,” Lennon
explained ______ in later years.

The word that properly fills in blank in


the text is:
Brazil is home to roughly 60 percent of the Amazon, about half of
a) charitable. d) charitably.
what remains of the world’s tropical rainforests. And now, the country
b) chariting. e) charitily. has plans to count its trees. A vast undertaking, the new National
c) charitingly. Forest Inventory hopes to gain “a broad panorama of the quality
and the conditions in the forest cover”, according to Brazil’s Forestry
07. (ITA 2012) Minister Antonio Carlos Hummel.
2045: THE YEAR MAN BECOMES IMMORTAL […]
(www.smartplanet.com. Adaptado.)
By Lev Grossman

No trecho do primeiro parágrafo – “Brazil is home to roughly 60


(…)
percent of the Amazon” –, a palavra roughly equivale, em português,
If you can swallow that idea, and Kurzweil and a lot of other very a:
smart people can, then all bets are off. 1From that point on, there’s no
a) evidentemente. c) aquém. e) cerca de.
reason to think computers would stop getting more powerful. 2They
would keep on developing until they were far more intelligent than b) exatamente. d) além de.
we are. Their rate of development would also continue to increase,
because they would take over their own development from their 10. (EN 2017) Mark the correct option.
slower-thinking human creators. Imagine a computer scientist that a) When times are tough, your friends will motivate you hardly.
was itself a super-intelligent computer. It would work incredibility
b) Enjoy the process of doing what you love and get excitedly.
quickly. It could draw on huge amounts of data effortlessly. It wouldn’t
even take breaks to play Farmville. c) Your parents will have a lot of reasons to be proudly of you.
(…) d) Working in a job you hate makes the days go slowly.
(http://www.time.com/printout/0,8816,2048138,00.html. e) Life without dreams can become very depressingly.
Acesso em 07/04/2011. Adaptado. )

Na sentença “They would keep on developing until they were far


more intelligent than we are” (ref. 2), o vocábulo grifado poderia ser EXERCÍCIOS DE
substituído por:
a) far away. c) much. e) many. COMBATE
b) incredible. d) distant.

08. (UFRGS 2012) 01. (UFV 2003)


FACEBOOK IS THE WORLD’S LARGEST SOCIAL NETWORK, WITH THE MOZART EFFECT
800 MILLION USERS WORLDWIDE AS OF SEPTEMBER 2011. Mozart makes you smarter! Researchers at the University of
[…] California at Irvine discovered that people who listened to ten minutes
of Mozart before taking an intelligence test scored higher than people

PROMILITARES.COM.BR 249
ADVERBS

who listened to ten minutes of relaxation instructions or who, for d) Did Mr. Burton hardly talk to me.
ten minutes, sat in silence. Scientists speculate that some kinds of e) Mr. Burton talked to me hardly.
music stimulate neural pathways in the brain. For a period of up to
fifteen minutes after listening, the group that heard Mozart improved
07. (ITA) Dadas as sentenças:
significantly in abstract and spatial reasoning. The one downer - that
improvement is 1temporary - may be because listening is a passive I. He hard works every day.
activity. No one knows if listening longer results in staying smarter II. He spoke hardly this morning at the debate.
longer. III. Don’t drive so fast!
Although some studies suggest that children as young as two
can benefit intellectually from music, you can be any age to take Constamos que está(ão) correta(s):
advantage of the Mozart Effect. You don't have to be a musician. a) apenas a I. d) apenas a I e a II.
You can profit from it regardless of your level of formal education. It b) apenas a II. e) todas as sentenças.
doesn't matter what kind of job you do, nor if you've never listened to
c) apenas a III.
a note of Mozart in your life. You don't even have to like music! The
Mozart Effect works automatically.
08. (ITA) Dadas as afirmações de que a forma adverbial de:
As a man, Mozart was playful, mercurial, ebullient: a quick thinker.
The rapidity with which he processed information and went from adjetivo advérbio:
one level of understanding to the next is echoed in the meticulous
organization of his 2frequently complicated but 3always clear music. I. fast (rápido) é fastly (rapidamente)
Mozart's usic induces widely varied emotional responses in us, but it
4
never allows us to wallow: it changes too 5fast. II. late (atrasado) é lately (atrasado)
Mozart had a notable career as a child virtuoso. His father,
III. low (baixo) é low (baixo)
Leopold, had him playing piano at four, composing by five. Mozart's
neural pathways, widened at an early age and stimulated constantly Constamos que está(ão) correta(s).
(Mozart composed more than six hundred works before he died at a) apenas a afirmação I. d) apenas as afirmações II e III.
thirty-five), facilitated his fluent expression of musical thought. What is
b) apenas a afirmação II. e) todas as afirmações.
it in Mozart that heightens our perceptivity? Perhaps it has something
to do with being able to pay attention. c) apenas a afirmação III.
(Source: Adapted from Mozart for Your Mind: Boost Your Brain Power with
Wolfgang Amadeus, Philips Classics Productions, CD 11.649.77.412, 1995.) 09. (ITA) Dadas as sentenças:
I. They go often to Rio de Janeiro.
All the following words are used as adverbs in the text, EXCEPT:
II. We have not yet bad news from him.
a) temporary (ref. 1). d) never (ref. 4).
III. She seldom is at home.
b) frequently (ref. 2). e) fast (ref. 5).
c) always (ref. 3). Constamos que está(ão) correta(s):
a) apenas a I. d) apenas a II e a III.
02. (PUC-RS) “The actors who had performed ______ received a b) apenas a II. e) todas as sentenças.
standing ovation and a big round of applause when the play ended”.
c) apenas a III.
a) immensely c) profoundly e) practically
b) extraordinarily d) fairly 10. (STA. CECÍLIA-SP) Assinale a alternativa em que os advérbios estão
na ordem correta:
03. (EFOMM) Which sentence is correct? a) She sang in the town hall perfectly last night.
a) He is guilty possibly of the crime. b) She sang perfectly in the town hall last night.
b) He is guilty of the crime possibly. c) She sang in the town hall last night perfectly.
c) He is possibly guilty of the crime. d) Last night she sang in the town hall perfectly.
d) Possibly is he guilty of the crime. e) She perfectly sang last night in the town hall.
e) He is guilty of possibly the crime.

04. (AFA) “Laila fared ______ in her test and was afraid to face her
mother”. GABARITO
a) well c) lowly
EXERCÍCIOS DE FIXAÇÃO
b) badly d) simply
01. C 04. B 07. A 10. A
02. D 05. A 08. B
05. (EFOMM) “They ran very _______ in the race but at the end they
could ______ breathe”. 03. C 06. B 09. B
a) fast - harder c) fast - hard e) fastly - hard EXERCÍCIOS DE TREINAMENTO
b) fast - hardly d) faster - hardly 01. C 04. B 07. C 10. D
02. C 05. B 08. A
06. (MACKANZE) The same as “Mr. Burton hardly talked to me.” is: 03. D 06. D 09. E
a) Hardly did Mr. Burton talked to me. EXERCÍCIOS DE COMBATE
b) Hardly Mr. Burton talked to me. 01. A 04. B 07. C 10. B
c) Hardly did Mr. Burton talk to me. 02. B 05. B 08. D
03. C 06. C 09. C

250 PROMILITARES.COM.BR
SIMPLE PAST AND
PAST PROGRESSIVE

SIMPLE PAST Geralmente, o “Simple Past” é usado na frase com um advérbio


de tempo no passado para indicar que a ação já ocorreu.
O passado é usado para indicar que uma ação ocorreu num
determinado tempo no passado: Exemplo:
Ex.: Thiago bought many books last Saturday.
L L
simple Past Adv. de
tempo
Em alguns casos, uma oração adverbial é que vai indicar quando
a ação aconteceu.
Exemplo:
Ex.: He visited Corcovado when he was in Rio.

Lsimple Past Loração adverbial

Todos os verbos regulares são acrescidos de –ed em suas formas


de passado e de particípio passado. Aqueles terminados em –e, ou
–ee, são apenas acrescidos de –d.

Exemplo: age aged


Jane went to Montreal last month. live lived
Usado para indicar um período de tempo no passado. agree agreed
hope hoped
order ordered
play played
ski skied

Verbos regulares terminados em Y precedidos de consoante


substituem o Y por –ied.

hurry hurried
study studied

Exemplo: cry cried


She worked at Petrobras from 2010 to 2014. apply applied

Sequência de ações ou fatos no passado: Verbos regulares terminados em Y precedidos de vogal recebem a
terminação –ed, normalmente.

play played
stay stayed

Verbos regulares com apenas uma sílaba, terminados em apenas


uma consoante precedida de uma só vogal, dobram essa última
consoante antes de receberem –ed. Essa regra não se aplica a verbos
regulares terminados em W, X ou Y, nas mesmas situações.

ban banned fax faxed


trim trimmed play played
bow bowed
Exemplo:
In 2000 she moved to São Paulo. Two years after she lived in Rio O auxiliar do “Simple Past” é o verbo “did” que é usado nas
and in 2014 she went to Curitiba. formas interrogativas e negativas.

PROMILITARES.COM.BR 251
SIMPLE PAST AND PAST PROGRESSIVE

Exemplos:
INTERROGATIVE FORM
I didn’t do my exercises.
Did she go to the movies alone? Was there?

Were there?
Alguns advérbios/expressões no passado:
Last month; last night; last year; yesterday; last spring; today; last
carnival; the day before yesterday; two months ago etc. Exemplos:
Was there a meeting at the office yesterday?
O PASSADO HABITUAL: WOULD AND USED TO Were there two trains at the platform this morning?
would e used to: são usados parar indicar ações repetidas no How many cars were there on the patio?
passado e que não mais acontecem no momento em que se fala.
Exemplos: PAST CONTINUOUS
As a child, Helen used to (would) spend all day in her room. O “Past Continuos” é formado com o verbo to be no passado
They used to (would) play kite for hours in their childhood. (como auxiliar) e mais o verbo principal no infinitivo com “ING” no
Did he use to attend Escola Naval or AFA? final.
Exemplo:
CONJUGAÇÃO DO “SIMPLE PAST”

AFIRMATIVA INTERROGATIVA NEGATIVA

You went Did you go You did not (didn’t) go


O “Past Continuous” é usado para indicar uma ação progressiva
no passado ou um período de tempo, num momento definido ou não.
They did Did they do They did not (didn’t) do

I had Did I have I did not (didn’t) have

You bought Did you buy You did not (didn’t) buy

He visited Did he visit He did not (didn’t) visit

She took Did she take She did not (didn’t) take

It fell Did it fall It did not (didn’t) fall


Exemplo:

We sank Did we sink We did not (didn’t) sink She was reading a book from 8 o’clock to 10 o’clock.

Usado para indicar simultaneidade no passado (Simple past + Past


Continuous ou Past Continuous + Past Continuous). Nesse caso, a
THERE WAS / THERE WERE ligação é feita principalmente pelos advérbios WHEN, WHILE, AS
SOON AS, TILL, UNTIL, BEFORE, AFTER, AS LONG AS, WHENEVER
AFIRMATIVE FORM e BY THE TIME.

There was havia/existia – Usado no singular

There were havia/existiam – Usado no plural

_ _ _ _ ___,.�
�---� �,---+----FUTURE
��-----'�
Exemplos: PA ST l'-
There was a scientist in the lab last night. PRESENT
There were many people on the beach yesterday.

NEGATIVE FORM 1 was studying last night


There was not There wasn’t CPast Continuous

There were not There weren’t When the light went

Simple past
Exemplos:
There wasn’t any student in class this morning.
There weren’t soldiers in the lodgment last night. Exemplo:
Peter was studying Math when I arrived.
My wife was talking on the phone while I was having lunch.

252 PROMILITARES.COM.BR
SIMPLE PAST AND PAST PROGRESSIVE

CONJUGAÇÃO DO “PAST CONTINUOUS” FUTURO - GOING TO OU


AFIRMATIVA INTERROGATIVA NEGATIVA
PRESENT CONTINUOUS
Indica um futuro com mais certeza, indica a situação como certa.
I was not (wasn’t) Exemplos:
I was going Was I going?
going
She’s going to start her classes tomorrow.
You were not They are visiting the museum this afternoon.
You were having Were you having?
(weren’t) having My sister is going to have a baby in December.
He was not (wasn’t)
He was taking Was he taking?
taking Ideias usadas com o futuro “going to”:
She was not Prediction (previsão com base em evidências no presente):
She was driving Was she driving?
(wasn’t) driving It’s going to rain soon.
She’s going to make an excellent nurse.
It was not (wasn’t)
It was showing Was it showing?
showing
Plan, decision or intention (plano, decisão ou intenção):
We were not
We were picking Were we picking? I’m going to start a new job on Monday. Or (I’m starting a new
(weren’t) picking
job on Monday.)
You were not I’m going to have a baby.
You were eating Were you eating?
(weren’t) eating

They were painting


Were they They were not FUTURO COM SIMPLE PRESENT
painting? (weren’t) painting Usado para indicar eventos oficiais e a partida ou chegada de
meios de transportes.
Exemplos:
SIMPLE FUTURE The show starts at 9 today at Ipanema Beach.
O futuro Will indica que a ação pode ou não acontecer, logo
The plane leaves at 8:00 P.M.
implica ideias como se fosse um plano, uma previsão para o futuro.
Caetano Veloso opens the season at Sambódromo tomorrow.
Exemplos:
She thinks she will study in the U.S.A. next year.
I believe I will leave in ten minutes.
THE FUTURE CONTINUOUS TENSE
Indica que as ações ou eventos estarão ocorrendo ou não em
My mother will bring me a DVD from The U.S.A.
um ponto no futuro, período de tempo no futuro ou relação de
simultaneidade.
Forma afirmativa:
Ponto no futuro:
Exemplo:
Exemplos:
I will go to the movies tonight.
What will the senator be doing at 4:00? He’ll be resting.
We will be meeting him at the office tomorrow.
Forma interrogativa:
Exemplo:
Período de tempo no futuro:
Will you take Jane at school today?
Exemplos:
Forma negativa: What will you be doing from 5:00 to 7:00? I’ll be studying at home.
Exemplo: She’ll be living in Gávea from June to July.
He will not (won’t) meet Mary during their trip next month.
Simultaneidade no futuro:
Ideias usadas com o futuro “will”: Exemplos:
Prediction (previsão, planos): By the time I get to Petrópolis, the sun will be rising.
Nancy will visit the Russia soon. Whenever you call, I will be waiting.
Don’t worry! You won’t feel a thing!
Time expressions: soon, sometime, before long, in a few minutes etc. EXERCÍCIOS DE

FIXAÇÃO
Promise (promessa): I’ll (I will) never do it again.
Request (pedido): It’s raining. Will you give me a ride?
Offer (oferta): You look cold. I’ll get you a jacket.
Willingness (disposição, boa vontade): Who will do the house 01. (FCC-SP) “His lawyer _______ here yesterday”.
cleaning? I will.
a) came
Unwillingness (refusal) (indisposição): I won’t (will not) leave
b) come
until I have seen Mr. Smith.
c) comes
Invitation (convite): Will you stay for dinner?
d) goes
e) went

PROMILITARES.COM.BR 253
SIMPLE PAST AND PAST PROGRESSIVE

02. (PUC-CAMP) Assinale a alternativa que traz os verbos entre how to code. He runs a tech company that depends on access to a
parênteses em sua forma correta para preencher a seguinte frase: constantly growing pipeline of talent. But it could be in your interest
“The dinosaur (live) 100 million years ago. It (walk) on two legs. Its too: studying coding could increase your chances of pulling in a big
arms (be) very small. Its mouth (be) very big. It (have) long teeth. salary. A computer-science education, at least in countries like the US,
A alternativa correta é: is one of the most viable and lucrative career paths open to young
people today.
a) lived - walked - were - was - had
But, Cook says, the benefits go beyond that. “It’s the language
b) was living - was walking - was - were - had that everyone needs, and not just for the computer scientists. It’s for
c) lived - walked - was - were - was having all of us”. He added that programming encourages students of all
d) lived - was walking - was - were - were having disciplines to be inventive and experimental: “Creativity is the goal.
Coding is just to allow that. Creativity is in the front seat; technology
e) lives - were walking - was - were - had
is in the backseat. With the combination of both of these you can do
such powerful things now.”
03. (UFRGS 2007) Leia.
(Adapted from https://www.cnbc.com/2017/10/12/apple-
Mr. Eugene Foster lives with his wife in a large house in New York ceo-tim-cook-learning-to-code-is-so-important.html.)
City, and they 1have four servants. On this particular morning, there is
a great deal of bustling about. One maid is distributing dust sheets to Which one from the underlined verbs in the text conveys a verb tense
every room, while another 2is draping them over the furniture. The butler that is different from the others?
is bringing down suitcases, and Mrs. Foster herself is flying from room a) says (paragraph 1)
to room and pretending to supervise these operations. Actually, she is
b) made (paragraph 1)
thinking of nothing at all except that she is going to miss her plane if her
husband doesn't come out of his study soon and get ready. c) think (paragraph 2)
Mr. Foster may possibly have a right to be irritated with his wife's d) runs (paragraph 3)
foolishness, but he can have no excuse for increasing her misery by e) needs (paragraph 4)
keeping her waiting 4unnecessarily. It is by no means certain that this
is what he 3does, yet whenever they go somewhere, his timing is so 05. (AFA) Mark the option that completes the gaps respectively: “I
accurate and his manner so bland that it is hard to believe he isn't ________ in Freeport, Illinois. My first job was at the local carwash.
purposely inflicting a nasty torture of his own on the 5unhappy lady. I ________ sixteen years old. I ________ every day, all summer, for a
And one thing he must know - that she would never dare to call out dollar an hour. I worked with ten other guys. All of us ________from
and tell him to hurry. He disciplined her too well for that. He must different racial, religious and economic backgrounds. We had to clean
also know that if he is prepared to wait even beyond the last moment cars in teams, and we quickly learned to work together”.
of safety, he can 6drive her nearly into hysterics. It seems almost as
a) woke up - were - came back - came
though he 'wanted' to miss the plane simply to intensify the poor
woman's suffering. b) was born - had - traveled - got
(Adapted from: DAHL, Roald. The way up to heaven. In: Tales c) grew up - was - worked - were
of the unexpected. London: Penguin Book, 1979. p. 179-180.)
d) lived - got - tried - lived
Se, em vez de narrar a história no presente, o autor a tivesse situado
no passado, as formas verbais HAVE (ref. 1), IS (ref. 2) e DOES (ref. 3) 06. (EFOMM) “On the way home I _________Jane at the bus stop.
teriam de ser substituídas, respectivamente, por: She __________ a beautiful dress but she _____________ happy”.
a) had, were e did. a) met – was wearing – didn’t look
b) have, was e done. b) meet – was wearing – didn’t look
c) had, was e did. c) met – wore – doesn’t looked
d) have, were e done. d) meet – is wearing – didn’t look
e) have, been e did. e) met – wears – doesn’t looked

04. (ESPCEX/AMAN 2019) Leia o texto a seguir e responda à questão 07. (ITA) Dadas as sentenças:
a seguir. I. I stayed at the Menezes for only two days.
LEARN TO CODE, IT’S MORE IMPORTANT THAN II. When she cried, her eyes used to get swollen.
ENGLISH AS A SECOND LANGUAGE III. He arrived and saw the patient.
Apple CEO Tim Cook says coding is the best foreign language
that a student in any country can learn. The tech executive made the Constamos que está (ão) correta (s):
remarks to French outlet Konbini while in the country for a meeting a) apenas a I.
with French President Emmanuel Macron. The tech leader gave some b) apenas a II.
brief thoughts on education:
c) apenas a III.
“If I were a French student and I were 10 years old, I think it would
d) apenas a II e a III.
be more important for me to learn coding than English. I’m not telling
people not to learn English in some form – but I think you understand e) todas estão corretas.
what I am saying is that this is a language that you can use to express
yourself to 7 billion people in the world. I think that coding should be 08. (ITA) Dadas as sentenças:
required in every public school in the world.” I. Shall we begin, now?
Of course, it’s in Cook’s best interest to have the world learning II. By 2050, Brazil will have overcome many of its social problems.
III. I can’t go out with you next Saturday, I will be working.
Constamos que está(ão) correta(s):

254 PROMILITARES.COM.BR
SIMPLE PAST AND PAST PROGRESSIVE

a) apenas a I. EXERCÍCIOS DE
b) apenas a II.
c) apenas a III. TREINAMENTO
d) apenas a I e a II.
e) todas as sentenças. 01. (UFSM 2007)
TEENS LIFE QUALITY AFFECTED BY A LACK OF SLEEP
09. (AFA) Complete the text: “Of course they are no angels, but they
______________ us pretty well and they __________ to kill us, like the According to a new 1survey of teenagers across the U.S., many of
Russian.” them are losing out on quality of life because of a lack of sleep.
(Yelena Kapran, one of the hundreds of Russian civilians taken hostage in The poll by the National Sleep Foundation (NSF) found that as
Budyonnovsk, after her release by Chechen Guerrillas.) consequence of insufficient sleep, teens are falling asleep in class, lack
the energy to exercise, feel depressed and are driving while feeling
a) cared - attacked 2
drowsy.
b) treated - didn’t try The 3poll results support previous studies by Brown Medical
c) haven’t treated - haven’t tried School, and Lifespan affiliates Bradley Hospital and Hasbro Children's
d) have worked - haven’t shooted Hospital, which found that adolescents are not getting enough sleep,
and suggest that 4this can lead to a number of physical and emotional
impairments.
10. (UNESP 2008)
Mary A. Carskadon, PhD, with Bradley Hospital and Brown Medical
HERE IS THE FIRST PART OF A LETTER, WRITTEN BY A
School, 5chaired the National Sleep Foundation poll taskforce and has
98-YEAR-OLD PENSIONED LADY TO HER BANK MANAGER.
been a leading authority on teen sleep for more than a decade.
Dear Sir,
Carskadon, director of the Bradley Hospital Sleep and
I am writing to thank you for bouncing my cheque with which Chronobiology Sleep Laboratory and a professor of psychiatry and
I endeavoured to pay my plumber last month. By my calculations, human behavior at Brown Medical School, says the old adage 6'early
three "nanoseconds" must have elapsed between his presenting to bed, early to rise' presents a real 7challenge for adolescents.
the cheque and the arrival in my account of the funds needed to
Her research on adolescent circadian rhythms indicates that the
honour it. I refer, of course, to the automatic monthly deposit of my
internal clocks of adolescents undergo maturational changes making
Pension, an arrangement, which, I admit, has been in place for only
them different from 8those of children or adults.
eight years You are to be commended for seizing that brief window
of opportunity, and also for debiting my account to the tune of 30 But teens must still meet the demands of earlier school start times
pounds by way of a penalty for the inconvenience caused to your that make it nearly impossible for them to get enough sleep.
bank. My thankfulness springs from the manner in which this incident Carskadon's work has been instrumental in influencing school
has caused me to rethink my errant financial ways. start times across the country.
I noticed that whereas I personally attend to your telephone Carskadon's newest finding indicates that, in addition to the
calls and letters, when I try to contact you, I am confronted by that changes in their internal clocks, adolescents experience slower sleep
impersonal, overcharging, prerecorded, faceless entity, which your pressure, which may contribute to an overall shift in teen sleep cycles
bank has become. to later hours.
From now on, I, like you, choose only to deal with a flesh- and- Judy Owens, MD, a national authority on children and sleep, is the
blood person. My mortgage and loan payment will therefore and director of the pediatric sleep disorders center at Hasbro Children's
hereafter no longer be automatic, but will arrive at your bank by Hospital and an associate professor of pediatrics at Brown Medical
cheque, addressed personally and confidentially to an employee at School, and says the results are especially important in light of the fact
your bank whom you must nominate. Be aware that it is an offence that 90% of the parents polled 9believed that their adolescents were
under the Postal Act for any other person to open such an envelope. getting enough sleep during the week.
Please find attached an Application Contact Status, which I require She says the message to parents is that teens are 10tired; but
your chosen employee to complete. I am sorry it runs to eight pages, parents can help by eliminating sleep stealers such as 11caffeinated
but in order that I know as much about him or her as your bank drinks and TV or computers in the teen's bedroom, as well as enforcing
knows about me, there is no alternative. Please note that a Solicitor reasonable bed times.
must countersign all copies of his or her medical history, and the
mandatory details of his/her financial situation (income, debts, assets A major, report last year by Carskadon, Owens, and Richard
and liabilities) must be accompanied by documented proof. Millman, MD, professor of medicine at Brown Medical School,
indicated that adolescents 12aged 13 to 22 need 9 to 10 hours of
(Adapted from: forums.film.com/showthead.php?t=15516)
sleep each night.
Indique a alternativa que completa a sentença: “The lady was sorry According to the National Center on Sleep Disorders Research at
the Application ________ to eight pages, but she ______ it to get the the National Institutes of Health, school-age children and teenagers
information she _______”. should get at least 9 hours of sleep a day. Other studies have also
a) runs - needs - wanted shown that young people between 16 and 29 years of age were the
most likely to be involved in 13crashes caused by the driver falling
b) runs - need - wanted asleep.
c) run - needs - wants The NIH also says without enough sleep, a person has trouble
d) ran - needed - wanted focusing and responding quickly and there is growing evidence linking
e) run - need - want a chronic lack of sleep with an 14increased risk of obesity, diabetes,
heart disease and infections.
(http://www.news-medical.net/?id=16969. Acesso em 03/7/06.)

O vocábulo "chaired" (ref. 5) tem a mesma função gramatical e


tempo verbal que:

PROMILITARES.COM.BR 255
SIMPLE PAST AND PAST PROGRESSIVE

a) believed (ref. 9) 04. (CN 2017) Complete the sentences with the correct use of the
b) tired (ref. 8) Simple Past and the Past Continuous.
c) caffeinated (ref. 11) – I was waiting for the bus when I __________ (see) her.
d) aged (ref. 12) – The children __________ (argue) when the teacher arrived.
e) increased (ref. 14) – Everyone __________ (listen) to music when the lights __________
(go) out.
02. (FATEC 2007) Leia. To fill in the gaps respectively, mark the right option.
OPTICAL FIBERS a) saw - was arguing - were listening - went
Optical fibers carry a dizzying amount of data each second, but a b) was seeing - was arguing - listened - were
great deal of communication still gets beamed, via slower microwaves, c) were weeing - argued - listenned - were
from one dish antenna to another. Engineers didn't think there was d) saw - were arguing - was listening - went
any improvement to tease out of this technology, but researchers at
the University of Paris recently reported in the journal Science that e) was seing - argued - listened - were going
they'd found a way of focusing microwaves into a narrow beam,
tripling the data rate. 05. (UNESP 2013) Leia a tira para responder à questão a seguir.
(Newsweek, March, 12, 2007)

Assinale a alternativa que corresponde à forma afirmativa do


segmento: "Engineers didn't think..."
a) “Engineers thought [...].”
b) “Engineers though [...].”
c) “Engineers through [...].”
d) “Engineers thru [...].”
e) “Engineers throw [...].” O trecho do segundo quadrinho – I’m going to – introduz uma:
a) possibilidade.
03. (IFBA 2018) b) dúvida.
My name is Luka c) resolução.
1
I live on the second floor
d) condição.
I live upstairs from you
2
Yes, I think you’ve 3seen me before e) obrigação.

If you hear something late at night 06. (EPCAR/CPCAR 2017) Read the text below and answer the
Some kind of trouble, some kind of fight question according to it.
4
Just don’t ask me what it 5was MOST COMMON PREJUDICES
What are some of the most common ways people discriminate
I think it’s ‘cause I’m clumsy
against each other? Some of the areas where people show their
I try not to talk too loud
intolerance are well-known, such as race. But 1others are less
Maybe it’s because I’m crazy 2
acknowledged, even if more common:
I try not to act too proud
[...]
They only hit until you cry Religion: Religious discrimination and 3persecution has been
After that you don’t ask why common throughout history. But 4prejudice based on religious
You just don’t argue anymore affiliation doesn’t end with organized religion; 5atheists are 6prone to
discrimination and being discriminated against.
Yes, I think I’m okay
[...]
I walked into the door again
6 7

8
If you ask that’s what I’ll say Which prejudice do you have? Which prejudice have you
18

And it’s not your business anyway experienced?


(Adapted from https://aloftyexistence.wordpress.com)
I guess I’d like to be alone
With nothing 9broken, nothing 10thrown Mark the correct question for the sentence “prejudice based on
Just don’t ask me how I am religious affiliation” (ref. 4).
(From: http://www.songtexte.com/songtext/suzanne- a) Which prejudice did you talk about?
vega/luka-2bd27c72.html.Access on Jul 24th, 2017.)
b) Which religion talked you about?
Which of the following verbs is regular? c) Which affiliation talk you about?
Check the correct alternative. d) Which religious prejudice you talked about?
a) seen (ref. 3).
07. (EEAR 2019) Choose the best alternative to complete the dialogue.
b) was (ref. 5). Jane: Hi Susan, how are you doing?
c) broken (ref. 9). Susan: Everything is Okay!
d) walked (ref. 7). Jane: Do you have any plans for this weekend?
e) thrown (ref. 10). Susan: Not sure… I _________ probably give a party this weekend.
Jane: Cool!

256 PROMILITARES.COM.BR
SIMPLE PAST AND PAST PROGRESSIVE

a) am going to 02. (CFOE) Mark the alternative that complete the excerpt: “There
b) am going __________ no special treatment for the women cadets during the
last war”.
c) will be
a) was
d) will
b) were
08. (CN 2017) Mark the option in which all the verbs are written in c) be
the Simple Past. d) is
a) put – drunk – ate – heard – taken
b) knew – brought – wanted – made – was 03. (AFA) “He ________ smoke a lot but he stopped because he was
running a big risk”.
c) wrote – come – felt – had – flew
a) has
d) told – were – begun – gave – read
b) loves
e) left – spent – slept – swum – traveled
c) hates
09. (CN 2017) Read the text to answer the question: “My name d) used to
is Patrick. I __________ on vacation to Brazil last Summer, and I
__________ in a five-star hotel front of the beach in Rio de Janeiro. I 04. (VUNESP) “They _________ about art last night”.
went to Rio by plane and I __________ a month there. I __________ a) talks
a lot of people and we __________ a great time! I want to go back to
Brazil as soon as possible”. b) talk

Choose the option which completes the gaps in the text respectively. c) was talking

a) traveled - stayed - spend - knew - had d) talking

b) traveled - were - spent - knew - did e) were talking

c) went - stayed - spent - met - had


05. (EFOMM) “Even though they ________ everything about the old
d) went - were - spend - met - did lady’s previous life, they still _______ her”.
e) were - went - stayed - knew - have a) know - hired
b) knew - hired
10. (EN 2016) Which is the correct way to complete the text below?
c) knew was - hiring
YOU CAN NOW EDIT AND FORMAT YOUR GOOGLE DOCS BY VOICE
d) were knowing - were hired
About six months ago, Google __________ voice typing for
Google Docs on the web __________ you to dictate your text into a e) know - is knowing
document. Today it’s taking this feature a step further by also allowing
you to edit and format your text by voice, too. 06. (COMD. AERONÁUTICA) There are several ways of expressing
future actions. Select the alternative which contains two of them.
This __________ you can now say think like “select all”, “align
center”, “bold”, “got to end of line”, or “increase font size” and a) I will stay here and rest. / I didn’t find that key.
Google Docs __________ and follow your commands. You can find a b) She is going to work and study abroad. / They will probably arrive
full list of available commands here […]. at nine o’clock.
(http://techcrunch.com//2016/02/2014/you-can- c) He thought he could win, but he didn’t. / I have to talk to that
now-edit-and-format-your-goole-docs-by-voice/)
official.
a) has introduced – to allow – means – understanding d) John and Peter want a new roommate. / I am going to have lunch
at noon.
b) introduces – allowed – meant – has understood
c) has introduced – to allow – is meaning – understood
07. (IME-RJ) “The phone _______ while I _______ my bath, as usual”.
d) introduced – to allow – means – will understand
a) rung – am having
e) introduced – allowed – meant – understand
b) rang – was having
c) rings – have
d) rung – had
EXERCÍCIOS DE

COMBATE
e) rang – have

08. (PUC-SP) Assinale a alternativa incorreta.


a) Please inform me when you will be delivering my order.
01. (AFA) b) The plane to Faro leaves at 6:30 p.m.
– ____________ you survive in that forest? c) Patrícia told me she is arriving on the 9 o’clock bus from
Campinas today.
– I had a survival course three years ago so I knew how to get food.
d) When summer is here, I will fulfill my promise of losing 20 pounds.
a) How did c) What can
e) They are going to move to Bauru when they finished the school year.
b) Why didn’t d) Who could

PROMILITARES.COM.BR 257
SIMPLE PAST AND PAST PROGRESSIVE

09. (EN) Mark the correct sentence.


a) The students was stopping talking when the teacher walked into
the room.
b) The students were stopping talking when the teacher walked into
the room.
c) The students talked when the teacher walks into the room.
d) The students were talking when the teacher were walking into
the room.
e) The students stopped talked when the teacher walked into the
room.

10. (CFO) Choose the option that correctly completes the sentence:
“When they _________ down the street they _________ Amelia”.
a) walk – saw
b) were walking – saw
c) walked – were seeing
d) was walking – was seeing
e) were walking – were seeing

GABARITO
EXERCÍCIOS DE FIXAÇÃO
01. A 04. B 07. E 10. D
02. A 05. C 08. E
03. C 06. A 09. B
EXERCÍCIOS DE TREINAMENTO
01. A 04. D 07. D 10. D
02. A 05. C 08. B
03. D 06. A 09. C
EXERCÍCIOS DE COMBATE
01. A 04. E 07. B 10. B
02. A 05. B 08. E
03. D 06. B 09. B

ANOTAÇÕES

258 PROMILITARES.COM.BR
GERUND AND INFINITIVE

INFINITIVO • Após adjetivos ou substantivos precedidos (ou não) de adjetivos.


Forma: to spend ou spend, to answer ou answer. Exemplos:
Negativa: not to spend / not to answer. I’m pleased to meet you.
O infinitivo é, normalmente, identificado pelo uso do “to” antes It was a terrible night to be out in.
do verbo.
Exemplo: • Como sujeito, objeto ou predicativo.
She wants to read a book. Exemplos:
To love is to believe. To love is to believe

verbo
sujeito infinitivo
principal sujeito infinitivo

USO DO INFINITIVO COM O “TO” To work means to give up much of your free time.
• Após os verbos afford, agree, appear, arrange, care, decide,
expect, fail, forget, happen, hope, learn, like, love, manage, need,
prepare, promise, prove, refuse, remember, seem, swear, turn out, objeto
want.
Exemplos: Observação
She decided to go to Mexico City. Em alguns casos, o infinitivo é apenas representado por to
Did Alex refuse to pay the bill? (especialmente quando usados com os verbos hate, like, love, try
e want).
Exemplos:
• Após os verbos discover, explain, find out, know, understand e
wonder, quando esses forem seguidos de “how”. Did you visit Sugar Loaf when you were in Rio?

Exemplos: Unfortunately not. I wanted to, but I didn’t have enough time.

Do you know how to fix this car? Have you booked us a table?
I’ve discovered how to solve this problem. I’ve tried to, but it was impossible.

• Após termos com a função de objeto.


Exemplos: USO DO INFINITIVO SEM O “TO”
Dr. Miller wants you to take these pills. • Após as formas auxiliares will, shall, would, do / does / did, can /
Are you asking your sister to lie to your parents? could, may / might, must, should, ought to, have to, (be) going to,
would rather e had better.
• Indicando finalidade (neste caso, pode aparecer a expressão Exemplos:
“only”). They could play any musical instrument.
Exemplos: You seem very tired, you had better take a nap.
They’ll go to São Paulo to buy some tools.
The teacher entered the room only to talk to the student. • Após os verbos “make” e “let”.
Exemplos:
• Após numerais ordinais (the first, the second,...); expressões the Ana makes me feel very happy.
last, the only; alguns superlatives como the best, the worst; e os Let me tell you about my new job.
advérbios what, when, where, why, which.
Exemplos:
• Após as expressões but e except.
Don’t let me be the last to know about your entrance to EFOMM.
Exemplos:
You didn’t know what to offer to our guests, did you?
Since you’ve let her, she’s done nothing but cry.
She will do anything except forgive your treason.

PROMILITARES.COM.BR 259
GERUND AND INFINITIVE

GERÚNDIO • Após os verbos:


Forma: forma verbal terminada em –ing.
admit deny involve resent
Reading / playing / quitting
Negativa: not reading / not playing / not quitting. anticipate detest mention resist

appreciate discuss mind risk


• Como sujeito ou complemento de frases.
avoid dislike miss spend time
Exemplos:
Singing is a lot of fun. Smoking is bad for you. be worth enjoy keep suggest

can’t help escape prevent tolerate


sujeito sujeito
complete excuse postpone understand

My favorite pastime is listening to music. consider finish practice

defer forgive recall

complemento delay imagine represent

He likes dancing on the weekends. Exemplos:


Keep practicing and you’ll reach perfection.
Do you mind opening that door for me?
complemento
• Verbos que são seguidos por verbos no gerúndio ou infinitivo:

• Como objeto da preposição. admit consider dislike love


Exemplos:
advise contemplate excuse need
I often relax by playing the guitar.
Good musicians improve by practicing every day. attempt continue hate neglect

begin deny imagine suggest


• Em frases compactas que expressam proibições.
Exemplos: cannot bear describe like start
No smoking / No trespassing
Exemplos:
• Após as expressões: I began studying electronics several years ago.
I began to study electronics several years ago.
be accustomed to be proud of rely on
Observação
be given to be sorry about speak about
Utilizamos o infinitivo quando temos a construção: would / ’d
be used to be successful at succeed at with hate, like, love, prefer, dislike.
Exemplo:
look forward to be (get) tired of take advantage of I would like to go to Maracanã.
plead innocent
care about think of • Uso do gerúndio ou infinitivo depois dos seguintes verbos: advise,
(guilty) to
allow, forbid, encourage, teach, permit.
(dis) approve of depend on worry about Exemplo:
verbo + gerúndio
be excited about forget about can’t help I advised taking your camera.
be afraid of in spite of can’t stand
verbo + (pronome) substantivo + infinitivo
be capable of insist on It’s no good They advised me to take my camera.

be good at instead of It’s no use • Uso do gerúndio ou infinitivo depois de verbos de percepção como:
feel, hear, see, smell, watch e phrasal verbs: listen to e look at.
be famous for plan on It’s useless
Exemplos:
I saw him pick up the papers and put them in his bag. (complete action)
Exemplos: I saw him walking along the road. = I saw him when he was
She told me that she’s looking forward to seeing you again. waking... (incomplete action)
It’s useless telephoning him at this time of the day. He felt the bullet penetrate his shoulder.
I can hear somebody moving in the other room.
You could have heard a pin drop.

260 PROMILITARES.COM.BR
GERUND AND INFINITIVE

EXERCÍCIOS DE 09. (UDESC 2013)

FIXAÇÃO ENGLISH AS AN INTERNATIONAL LANGUAGE


About one hundred years ago many educated people 4learned
and spoke French when they 5met people from other countries. Today
most people speak English when they meet foreigners. It has become
01. (FMU/FIAM/FAAM-SP) “She made us _____ an hour in her office”. the new international language. There are more people who speak
a) to wait d) waited English as a second language than people who speak English as a first
language. Why is this?
b) wait e) to waiting
There are many reasons why English has become so popular. One
c) waiting of them is that English has become the language of business. Another
important reason is that popular American culture (like movies, music,
02. (OSEC-SP) “She stopped _______ cigarettes. Her doctor told her and McDonald's) has quickly spread throughout the world. It has
not to do it”. 6
brought its language with it.
a) to smoke Is it good that English has spread to all parts of the world so
b) of smoking quickly? I don't know. It's important to have a language that the
people of the earth have in common. Our world has become very
c) smoke
global and we need to communicate with one another. 2On the other
d) by smoking hand, English is a fairly complicated language to learn and it brings 3its
e) smoking culture with it. Do we really need that?
Scientists have already 7tried to create an artificial language that
03. (PUC-RS) O verbo que completa corretamente a frase “Man isn't too difficult and doesn't include any one group's culture. It is called
avoids ______ like apes” é: Esperanto. But it hasn't become popular. But maybe the popularity of
a) acting English won't last that long either. Who knows? There are more people in
the world 1who speak Chinese than any other language. Maybe someday
b) acted
Chinese will be the new international language.
c) act (www.5minuteenglish.com - Accessed on June 19th)
d) have acted
e) to act Answer the following question according to the text above.
The infinitive of the verbs: “learned” (ref. 4), “met” (ref. 5), “brought”
04. (UNIFIC-RS) “He was angry about ______ failed the test”. (ref. 6), “tried” (ref. 7), is consecutively:
a) to have a) to learn, to meet, to bring and to try.
b) have b) to larne, to met, to broug and to trie.
c) having c) to learn, to met, to bring and to tri.
d) had d) to learne, to meet, to brough and to try.
e) has e) to lear, to meet, to bring and to tri.

05. (AFA) “The doctor advised her _________ while pregnant”. 10. (USF 2018) Read the text carefully.
a) stops to smoke GLASGOW SCHOOL OF ART PROJECT
PERSONALISES PROSTHETIC LIMBS
b) to stop smoking
21 August 2017
c) to stop to smoke
d) stopping smoking

06 (UFRJ) “Before _______ sent to prison, the thief will be given the
right to defend”.
a) being d) he will be
b) be e) he has been
c) to be

07. (VUNESP-SP) “Most people cannot learn verbs without ______


them”.
Craftspeople have been working with amputees who use artificial
a) to study d) studied
lower limbs to create more personalised prosthesis.
b) studying e) studies
Jeroen Blom, a researcher at Glasgow School of Art's Highlands
c) study and Islands Creative Campus, 1brought the team of three artisans
together.
08. (EEAR 2016) Choose the alternative that fills in the blank in the They are Karen Collins, from Rafford, and Scott Gleed, of Relugas,
sentence: “Scientists say that when a person stops __________ very both in Moray, and Roger Milton, from Auldearn in the Highlands.
soon, the chances of getting cancer and other diseases reduce”.
They are working with three people who use prosthetic limbs.
a) smoke
Among the materials being used to make what are known as
b) smokes greaves is wood, while the skills involved include weaving.
c) smoked
d) smoking

PROMILITARES.COM.BR 261
GERUND AND INFINITIVE

Mr Blom said: 2"Through this project three lower limb amputees […]
have been able to have a full involvement in the creation of something As a teenager, Mr. Barbosa moved to the capital, Brasília, finding
very personal and unique to them and in so doing had a much greater work as a janitor in a courtroom. Against the odds, he got into the
sense of involvement and ownership." University of Brasília, the only black student in its law program at the
"For the artisans, meanwhile, this has been an opportunity to time. Wanting to see the world, he later won admission into Brazil’s
apply and showcase their skills in a new area and to create very special diplomatic service, which promptly sent him to Helsinki, the Finnish
partnerships with their collaborators in the co-design process. capital on the shore of the Baltic Sea.
"The aesthetics of the 3resulting greaves reflect the identity of the […]”
amputee as well as the artisanal process." In the sentence “Wanting to see the world, he later won admission
One of the amputees, Caitlin McMullan, said discussions about into Brazil’s diplomatic service,” the underlined phrase can be correctly
her use of a prosthetic limb influenced the design work. rewritten as:
She said: "We spoke a bit about my experience of being an a) want to seeing the world.
amputee, and my experience of before my amputation. b) wanted to see the world.
"It was good to think about the design of that, and talk about c) because he wanted to see the world.
how I think the design is disability-awareness as well."
d) because he wanted seeing the world.
She added: 4"I like having choice, I like changing what my
prosthesis would look like.
02. (ITA 2012)
"I don't like to cover it up. I don't really see the point in 5trying
to hide it. That's what I like about this project. It's making something
really nice out of a prosthesis."
Chema Perez, who along with Carol Sloan is another of the
amputees, worked with craftsman Mr Milton on a wooden greave.
Mr Perez said: "The idea Roger had about having a piece of wood
which is not really nice and shiny but something that has some marks
of imperfection that tells you a story, was something that I was looking
for. 6Something that reflects my experience to make it more personal."
7
The art school's Highlands and Islands campus is in Forres in
Moray. Students and researchers use it as a base for studies in the
Highlands and Islands.
(Available in: <http://www.bbc.com/news/uk-scotland-highlands
-islands-40962175>. Access in: 11/10/2017. - Adapted.)

Read the text and choose the correct alternative.


a) “resulting” (ref. 3), “brought” (ref. 1) and “trying” (ref. 5) are
regular verbs.
b) The words “in” in the sentence “The art school's Highlands and
Islands campus is in Forres in Moray.” (ref. 7) are being used as
prepositions of time and place, respectively.
c) The sentence "Through this project three lower limb amputees
have been able to have a full involvement in the creation […]”
(ref. 2) is in the present perfect continuous tense.
d) In the sentence, “[…] I like having choice, I like changing what
my prosthesis would look like. “(ref. 4), we could replace the
Every working day, ARAMARK GmbH, _____ by Peter Amon,
verbs in bold by “to have” and “to change” because the verb like
prepares nutritious meals from fresh, sustainable ingredients. GE Capital
admits gerund and infinitive forms.
provides environmentally friendly vehicles to cater for the company’s long-
e) In the sentence “Something that reflects my experience to make it term transport needs. With its ______ international fleet platform, GE
more personal." (ref. 6) the word “it” refers to co-design process. Capital provides an enticing menu of leasing options.
As lacunas I e II devem ser preenchidas, respectivamente, por:
EXERCÍCIOS DE a) “led” e “leading”. d) “led” e “leader”.

TREINAMENTO b) “leader” e “led”.


c) “leading” e “led”.
e) “leader” e “leading”.

03. (UDESC 2012)


01. (UECE 2014) HOW DO I GET PAID?
“BRASÍLIA — Brazil’s highest court has long viewed itself as a All money generated by click throughs or customers signing on with
bastion of manners and formality. Justices call one another “Your a program are fed into a database on the affiliated advertisers site and
Excellency,” dress in billowing robes and wrap each utterance in tallied automatically (1). The affiliated advertiser transfers, wires, mails the
grandiloquence, as if little had changed from the era when marquises any money earned once an amount of $50 or more has been collected.
and dukes held sway from their vast plantations.
I only want certain types of ads on my site
In one televised feud, Mr. Barbosa questioned another justice
about whether he would even be on the court had he not been With most affiliate advertisers you can determine exactly what
appointed by his cousin, aformer president impeached in 1992. With kinds of ads appear on your site. The upside is you have more control,
another justice, Mr. Barbosa rebuked him over what the chief justice the down side is it is more labor intensive managing the ads. When an
considered his condescending tone, telling him he was not his ad is automatically generated, as is the case with Google AdSence, the
“capanga,” a term describing a hired thug.

262 PROMILITARES.COM.BR
GERUND AND INFINITIVE

nature of the ad depends on the content of your page where (2) the ad 05. (FGV 2010)
is located. This means you have little or no control over the contents of PETROBRAS APPROVES FIRST OFFSHORE HEAVY OIL DEVELOPMENT
the ads other than the content of your web page.
Petrobras has approved the development project for its Siri field in
It must (3) be hard to put the ads or banners up on the site the Campos basin, according to a news report from Brazil. The field will
In most cases it is a matter of cutting (4) the advertising code from be the first in the world ________ extra heavy oil from an offshore site.
the affiliated advertisers site and paste into (5) your web page. No Siri field, off the coast of Southeast Brazil, has been in production tests
messing with images or writing copy for the advertisments. since March and the company plans to contract production equipment
(Adapted from: www.bannermadness.com/articles.php. in 2011.
Accessed on: March 25th, 2012.) (www.ogfj.com)

Mark the correct sequence of grammar definitions from the underlined Assinale a alternativa que completa, correta e respectivamente, a
words, following the sequence of numbers (1), (2), (3), (4) e (5). primeira lacuna no texto.
a) subject - pronoun - verb - gerund - phrasal verb a) the product
b) adjective - objective adverb - auxiliary - past participle - phrasal verb b) in production
c) adverb - adverb - modal verb - gerund - preposition c) will produce
d) adverb - possessive pronoun - modal - passive voice - subject d) is produced
e) adverb - question word - simple past - past participle - noun e) to produce

04. (ITA 2011) 06. (UEM 2010)


OUR IMAGINARY, HOTTER SELVES ROLES REVERSED AS MEN DOMINATE BRITAIN’S
Avatars might serve therapeutic purposes, helping those ‘ONLINE HIGH STREET’
with social phobia become more confident. The stereotype of the female shopaholic could soon be an image
by Sharon Begley of the past, as new research shows that Britain’s ‘online high street’ is
¹increasingly dominated by men.
Anyone who has ever had a bad hair day, when looking like a latter-
The latest findings indicate that over 12.9 million men shop online
day Medusa makes you feel cranky and antisocial and plodding, can
today compared with 10.3 million women, and almost twice as many
sympathize with the Oakland Raiders - and not because the players get
men than women are joining ²the army of online shoppers each year.
helmet hair. The Raiders alternated between mostly black and mostly
white uniforms, 1depending on whether they were playing at home or Men are also spending more online than women. ³Over the last
away. 2Knowing that appearance affects people’s mood and outlook, six months, men spent online the average of £2,602 compared to
psychologists wondered whether uniform color influenced the Raiders’ £1,930 for women. Men significantly outspent women on travel,
aggressiveness. financial services and major purchases such as fridge freezers and
washing machines. In fact, men spent more than women in every
3
Using data from the 1970s and 1980s, 1they found that the team
online category except groceries and clothing.
racked up way more penalty yards – 4a measure of aggression – when
5
they wore black than when they wore white, for infractions both minor Men were quicker to turn to the Internet at the beginning of the
6
(encroachment) and major 7(roughing the kicker). The pattern held even economic downturn, looking to save money by shopping around, and
when the scientists took into account different conditions and styles of to research products 4in more depth before buying them.
play at home and away. 8But while the 1988 9finding has become a (Adaptação do texto disponível em . Acesso em 13/08/2009).
classic in psychology, the explanation remains controversial. Do referees,
because of black’s cultural baggage, see black-clad players as meaner Assinale a(s) alternativa(s) CORRETA(S) considerando o que se afirma a
and badder than 10those in, say, baby blue? Or does wearing black make respeito dos vocábulos extraídos do texto 1.
players see themselves as tougher and meaner – and therefore 11cause 01) “increasingly” (ref. 1) é o mesmo que “to a certain extent”.
them to play that way? 02) “army” é utilizado em “the army of online shoppers” (ref. 2)
Jeremy Bailenson and Nick Yee of Stanford University had this and other para indicar um grande número de pessoas envolvidas em uma
classic studies in mind 12when they started 13wondering about the effect of determinada atividade.
being able to alter one’s appearance. They weren’t going to study wardrobe 04) “Over the last six months” (ref. 3) poderia responder à pergunta
choices, however. 14Their quarry is avatars, 15digital representations of “How long have women taken to join online shopping?”.
players in such games as Second Life. “Your physical appearance changes
how people treat you,” says Bailenson. “But independent of that, when 08) “in more depth” (ref. 4) poderia ser substituído por “more
you perceive yourself in a certain way, you act differently.” He and Yee call economically”, sem alteração de significado.
it “the Proteus effect,” after the shape-changing Greek god. The effect of 16) A forma “-ing” foi empregada em “by shopping around” (ref. 5)
appearance on behavior, they find, carries over from the virtual world to the e “before buying” (ref. 6) porque os verbos foram usados após
real one, with intriguing consequences. (...) preposições.
(http://www.newsweek.com. Acesso em 5/6/2010.) Qual o somatório das opções corretas?

Assinale a opção em que o termo em negrito apresenta função gramatical


07. (MACKENZIE 2010)
diferente das demais.
GUITAR HERO INTERVIEW: NOEL GALLAGHER
a) [...] depending on whether they were playing at home or away.
(ref. 1) By Marcel Anders

b) Knowing that appearance affects people’s mood and outlook, [...] Oasis lead guitarist Noel Gallagher has been described as “the
(ref. 2) elder statesman” of British rock. Yet this is a description that he rejects,
c) Using data from the 1970s and 1980s, they found that [...] (ref. 3) pointing out that, at the age of 41, he is still a young man, when
d) But while the 1988 finding has become a classic in psychology, compared with the likes of Mick Jagger, Keith Richards and even Paul
[...] (ref. 9) Weller.
e) [...] when they started wondering about the effect of being able Gallagher, who has often been in conflict with the Oasis lead
to alter [...] (ref. 13) singer, his younger brother Liam, recently met with the press in order to
promote the band’s latest album, Dig Into Your Soul. As he explained,

PROMILITARES.COM.BR 263
GERUND AND INFINITIVE

some of its songs were inspired by his memories of LSD trips as a a drive-through window to grab something to eat. It is also easy
teenager in Manchester: __________ into a gas station __________ a bag of chips, a soda, and
Noel Gallagher: some candy. However, __________ this is not the best choice for our
bodies. Simply put – the more junk you put into your body, the worse
What do you write about when you’re 41? I don’t know. You you are going to feel. Try __________ your body with healthy food,
know, drink plenty of water, and skip fast food lines as much as you can to
__(I)__famous? Nobody wants to hear about that. __(II)__ in a feel healthy and happy.
band? (Abridged from http://www.teenadvice.about.com)
That was kind of what Definitely Maybe was about. Women? Too
boring, you know. Money? Nobody wants to hear about that. Politics? a) driving – stopping – to buy – to do – to nourish
__(III)__, you know. Save the planet? __(IV)__. You know, I don’t know. b) driving – stopping – to buy – doing – to nourish
So, the most __(V)__ thing is like, well, I remember when I was 16 and, c) to drive – to stop – to buy – doing – to nourish
you know, __(VI)__ acid, it was like, “Yeah, there’s stories to be told there,
I think.” d) to drive – to stop – buying –doing – nourishing
BACK ON TRACK e) to drive – to stop – to buy – to do – nourishing
Many critics say that Oasis are “back on track” with this album,
but Gallagher has his reservations: 09. (UEPB 2013)

Noel Gallagher: THE ART OF MEMORY

For me personally, I don’t know what anybody else in the band In the age before books and tablets, orators stored texts in lees
thinks, but when I sit down and write a song I don’t think: “Well, is reliable devices: their minds. To boost his memory capacity, Roman
this as good as what I’ve done?” I just write them, you know, and philosopher Cicero used tricks called mnemonics to bind his words to
record them and other people decide. So, if other people decided vivid mental images, “as if 1inscribing letters into wax.”
that we were back, you know, after “Don’t Believe The Truth,” then Such ancient techniques may no longer be needed, but this month
great, do you know what I mean? But I… it’s not something that I they’ll take center stage when some 50 “mental athletes” go head-to-
would ever… I wouldn’t enter into a debate about it. How can I, you head in the 15th USA Memory Championship in New York City. Their
know? I don’t… I’m not a professional songwriter, do you know what minds aren’t photographic, even memory experts need a 2coding
I mean? I kind of… it’s what… I’ve been writing songs for as long as system to remember strings of words, numbers, names, or 3playing
I can remember, you know, for fun, it’s just so now lots of people buy cards. The key is 4training – 12 hundreds of hours of it. And speed.
them, but if nobody bought the records tomorrow, I still wouldn’t stop 5
Linking items to celebrities is common practice because they’re easy
writing songs, but I’d write songs just to play to my mates. It’s kind to visualize. However, “an emotional tie makes the image louder” says
of what I do. last year’s champ. Nelson DeIlis. When creating his mnemonic cede for
ABBEY ROAD cards, he passed on a popular heartthrob for the king of hearts. “Brad
Pitt had to think about. But my dad – I can picture him in an instant”.
Dig Out Your Soul was recorded at London’s famous Abbey Road (Oliver Uberti, National Geographic, March 2012 )
studios, whose previous occupants have included The Beatles and Pink
Floyd. Rumour has it that Oasis had previously been banned from the The group of words from text which is made up of the words
studios on account of their unruly behaviour: “INSCRIBING” (ref. 1) “CODING” (ref. 2), “PLAYING” (ref. 3),
Noel Gallagher: “TRAINING” (ref. 4) and “LINKING” (ref. 5) includes
We had to pay the money upfront and promise to be on our best a) only nouns and present participles.
behaviour. b) only present and past participles.
We had to pay the money upfront, and so they said, “If… if you c) only present participles, adjectives and a noun.
have to leave, then you’re gonna lose the deposit.” Well, it’s different
d) only adjectives and present participles.
now, you know, when we were in there in ’97, everybody was in their
20s and whacked out on drugs all the time, so it’s different now. e) only past and present participles and an adjective.
Everybody’s got kids, you know. Everybody’s a little bit more… a little
bit less mental, you know, but we still have a good time, though. We 10. (MACKENZIE 2012)
made a great record, that’s the main thing about it, you know, and
it’s good for Abbey Road, that somebody’s finally made a great record
there!
(Speak Up)

The right form of the words be, bore, interest and take which
appropriately complete blanks I, II, III, IV, V and VI, in the excerpt of
the text, are:
a) been, been, boring, bored, interested and taking.
b) being, be, bored, boring, interesting and taken.
c) be, be, boring, boring, interested and take.
d) been, being, bored, boring, interesting and taken.
e) being, being, boring, boring, interesting and taking.

08. (EN 2016) Which option best completes the paragraph below?
TEN YEARS OF FEAR, GRIEF, REVENGE AND RESILIENCE
EAT HEALTHY The Samaritan By John Maguire
In today’s fast-paced world, it is so easy __________ through
It is one of the day’s most familiar and most chilling images: rescue
workers carrying the beloved Fire Department chaplain Father Mychal
Judge from the rubble. But the man nearest Judge was no rescue

264 PROMILITARES.COM.BR
GERUND AND INFINITIVE

worker: he was a 28-year-old associate at Goldman Sachs. Seeing a) used to taking d) used taking
papers flying in the air outside his office, he headed into the chaos on b) used to take e) used taking to
the ground. Just after the first tower fell, he found the men struggling
to carry the dying Judge. It was too late to perform CPR, so John c) uses to take
Maguire started back toward Ground Zero. Then the second tower
collapsed. “I jumped under a truck, and everything went black,” he 05. (EN) Which is the correct way to complete the paragraph below?
recalls. “I thought, ‘I guess this is how it’s going to end.’” HOW TO BECOME A PRO GAMER
Easy to understand, that is. ________ for a living isn't all fun and
Maguire eventually made it to games. It takes time, skill and determination ________ in today's
the apartment of his sister, who competitive circuits. If you’re up for the challenge, here's what you
showed him the photo the next day: need to do.
“It seemed surreal.” He found solace a) To play - to win
in an unexpected place. Recalled to
active duty, the West Point graduate b) Playing - winning
served as a civic-affairs officer in Iraq c) Play - winning
from 2006 to 2007. Doing so was his d) Playing - to win
therapy, alleviating a crushing sense
e) To play - winning
of futility. Today, Maguire is back at
Goldman. “_____(I)_____ because I commute through that site every
day.” 06. (PUC-SP) Assinale a alternativa que completa corretamente a frase:
(www.newsweek.com)
“When Mariana told me that joke I simply couldn’t help ________ my
pants off”.
The right sequence of words that properly fill in blank I in the text is: a) laughing
a) It’s hard for myself to really don’t think about […] b) laugh
b) It’s hard for me to really not think about […] c) to laugh
c) It’s hard for me to really think not about […] d) am laughing
d) It’s hard to really don’t think about for me […] e) “a” and “b” are correct.
e) It’s hard for me not to really think about […]
07. (AFA)
I. – I hate __________ up early in the morning.
EXERCÍCIOS DE II. – Mary suggested ________ to the country club next week.

COMBATE III. – I’m trying _______. Please, stop________.


IV. – I don’t mind__________ by train sometimes. Goodbye. I hope
________ you again.
a) to get - to go - to work - to quarrel - to travel - to see
01. (CFOE) Complete with the correct verb forms. “We need _______
b) to get - going - working - to quarrel - traveling - seeing
our hands before ______ ill people.”
c) getting - going - to work - quarreling - traveling - to see
a) wash – visit
d) getting - to go - to work - to quarrel - to travel - to seeing
b) to wash – to visit
c) to wash – visiting 08. (FATEC 2006) Considere o texto abaixo.
d) washing – visiting GRAND THEFT IDENTITY
Millions of people now have a new reason to dread the mailbox.
02. (ITA) “She insisted ________ to that out-of-the way farm just to
In addition to the tried-and-true collection of Letters You Never Want
know it”.
to See - the tax audit, the high cholesterol reading, the college-
a) on to go rejection letter - there is now the missive that reveals 1you are on
b) go the fast track to becoming a victim of identity theft. Someone may
have taken possession of your credit-card info, bank account or other
c) on going
personal data that would enable him or her to go on a permanent
d) to go shopping spree - leaving you to deal with the financial, legal and
e) in going psychic bills. Deborah Platt Majoras got the pain letter recently, from
DSW Shoe Warehouse. Hers was among more than a million credit-
03. (EFOMM) “It’s useless _______ for new changes”. card numbers that the merchant stored in an ill-protected database.
So when hackers busted in, they got the information to buy stuff in
a) to wait
her name - and 1.4 million other people's names. "It's scary", she
b) wait says. "Part of it is the uncertainty that comes with it, not knowing
c) waits whether sometime in the next year my credit-card number will be
d) waited abused". Now she must take steps to protect herself, including re-
examining charges closely, requesting a credit report and contacting
e) waiting the U.S. Federal Trade Commission to put her complaint into its ID-
theft database. The latter step should be easy for her, since Majoras is
04. (EFOMM) Choose the correct alternative: “My nephew is the FTC chairman.
_____________ a nap after meals”. (Newsweek, September 6, 2005)

Assinale a alternativa que apresenta o uso correto do gerúndio após


a preposição TO como em "you are on the fast track to becoming a
victim of identity theft"(ref.1).

PROMILITARES.COM.BR 265
GERUND AND INFINITIVE

a) I enjoy to becoming mature.


GABARITO
b) I object to becoming mature.
c) I need to becoming mature. EXERCÍCIOS DE FIXAÇÃO
d) I hope to becoming mature. 01. B 04. C 07. B 10. D
e) I insist to becoming mature. 02. E 05. B 08. D
03. A 06. A 09. A
09. (UFAL 2008) EXERCÍCIOS DE TREINAMENTO
"SHE HAS SHOWN SHE IS NOT AFRAID" 01. C 04. D 07. E 10. E
She has mended fences abroad, but is frustrated at home. 02. A 05. E 08. C
A pastor's daughter growing in communist East Germany, Angela 03. C 06. SOMA: 18 09. C
Merkel was trained as a physicist, who concentrated on her science to
EXERCÍCIOS DE COMBATE
the exclusion of all other pursuits. But Merkel, Germany's first female
chancellor, has emerged as the most important leader in Europe, and 01. C 04. A 07. C 10. D
enjoys the highest popularity rating of any German leader in postwar 02. C 05. D 08. B
history.
03. E 06. A 09. A
A lackluster campaigner, Merkel barely scraped into office two
years ago, after forging an uneasy coalition linking her Christian ANOTAÇÕES
Democrats with their ideological rivals, the Social Democrats. Since
taking office, she has been actively engaged in framing a new global
agenda, from climate change and energy security to sustaining 4,000
peacekeeping troops in Afghanistan - which for Germany would have
been unthinkable just a decade ago.
Merkel has shown she is not afraid to defy conventional wisdom
and her willingness to resist pressures from media and business has
bolstered her standing with the voters. But Merkel's most serious
challenges still lie ahead of her. She has achieved record approval
ratings on the strength of her foreign policy, but with economic
growth slowing and workers stepping up demands for higher wages,
political pressures are mounting within Merkel's coalition to backtrack
on the cuts in retirement benefits and other austerity measures that
propelled the recent recovery. And while her coalition has responded
to the country's plunging birthrate and aging population by extending
the retirement age to 67, Merkel has been frustrated in her efforts to
carry out a wider restructuring of the German economy.
(Adapted from She Has Shown She Is Not Afraid, Statesmanship,
Newsweek/October 29, 2007, page 19.)

In the last sentence of the text, the -ing words "plunging", "aging",
"extending", and "restructuring" function, consecutively, as:
a) adjective, adjective, verb and noun.
b) noun, noun, adjective and verb.
c) verb, verb, noun and adjective.
d) adjective, noun, adjective and verb.
e) noun, adjective, verb and noun.

10. (EFOMM) Choose the correct alternative to complete the sentences


below.
I. I can’t afford _____ for all your expenses.
II. He enjoys ______ on the weekends.
III. After many frustrated attempts, I finally managed _______ him
stay.
IV. You should always put on sunscreen before ______ to the beach.
a) paying - sailing - making - going
b) paying - sailing - to make - to go
c) pay - to sail - made - going
d) to pay - sailing - to make - going
e) to pay - to sail - making - to go

266 PROMILITARES.COM.BR
FUTURE TENSES

SIMPLE FUTURE Unwillingness (refusal) (indisposição).


Exemplo:
O futuro com WILL indica uma ação espontânea, sem planeja-
mento; previsão baseada no achismo, sem evidências; promessas e I won’t (will not) leave until I have seen Mr. Smith.
pedidos.
Exemplos: Invitation (convite).
She thinks she will study in the U.S.A. next year. Exemplo:
I believe I will leave in ten minutes. Will you stay for dinner?
My mother will bring me a DVD from The U.S.A.
FUTURO GOING TO OU
FORMA AFIRMATIVA PRESENT CONTINUOUS
Exemplo: Indica um futuro com mais certeza, indica a situação como certa.
I will go to the movies tonight. Exemplos:
She’s going to start her classes tomorrow.
FORMA INTERROGATIVA
They are visiting the museum this afternoon.
Exemplo:
My sister is going to have a baby in December.
Will you take Jane at school today?

FORMA NEGATIVA
IDEIAS USADAS COM O FUTURO “GOING TO”
Prediction (previsão com base em evidências no presente).
Exemplo:
Exemplos:
He will not (won’t) meet Mary during their trip next month.
It’s going to rain soon.
IDEIAS USADAS COM O FUTURO “WILL” She’s going to make an excellent nurse.
Prediction – GERALMENTE COM OS VERBOS THINK OU GUESS.
Plan, decision or intention (plano, decisão ou intenção).
Exemplos:
Exemplos:
I think Nancy will visit Rússia soon.
I’m going to start a new job on Monday. or (I’m starting a new
Don’t worry! You won’t feel a thing!
job on Monday.)
Time expressions: soon, sometime, before long, in a few minutes,
I’m going to have a baby.
etc.

Promise (promessa). FUTURO COM SIMPLE PRESENT


Exemplo: Usado para indicar eventos oficiais ou a partida ou chegada de
meios de transportes.
I’ll (I will) never do it again.
Exemplos:
The show starts at 9 today at Ipanema Beach.
Request (pedido).
The plane leaves at 8:00 P.M.
Exemplo:
Caetano Veloso opens the season at Sambódromo tomorrow.
It’s raining. Will you give me a ride?

Offer (oferta).
THE FUTURE CONTINUOUS OU
Exemplo: PROGRESSIVE
You look cold. I’ll get you a jacket. Esse tempo verbal indica uma ação que estará em andamento em
um determinado ponto futuro.
Estrutura: WILL + BE + VERB + ING
Willingness (disposição, boa vontade).
Exemplo: Ponto no futuro:
Who will do the house cleaning? I will. Exemplos:
What will the senator be doing at 4:00? He’ll be resting.
We will be meeting him at the office tomorrow.

PROMILITARES.COM.BR 283
FUTURE TENSES

Período de tempo no futuro: d) To declare anything loudly


Exemplos: e) To feel threatened
What will you be doing from 5:00 to 7:00? I’ll be studying at home.
She’ll be living in Gávea from June to July. 05. The word steaming, in “California steamin’”, gives an idea that
California is:
a) Extremely hot
Simultaneidade no futuro:
b) A cool place
Exemplos:
c) Cozy
By the time I get to Petrópolis, the sun will be rising.
d) Dirty
Whenever you call, I will be waiting.
e) Bohemian

TEXT COMPREHENSION
EXERCÍCIOS DE
CALIFORNIA STEAMIN’

All the trees are brown


And the sky is gray
FIXAÇÃO
I’ve been for a walk
On a greenhouse day. 01. - Hey, Jill, are you there?
- I’m here.
I should be safe and sound now
If I was miles from L.A. - The phone is ringing!
California steamin’ - _____ it!
On such a sweltering day. a) I’ll get

Stopped into a church b) I’ve rung


I stumbled along the way c) I ought answer
Got down on my knees d) I’d been taking
And prayed for a rainy day.
02.
You know the preacher likes it cold
Now that all his candles have melted away,
California steamin’
Please don’t take my fan away.
(cafe.cynicmag.com, September.20.2012)

01. In this poem, the author’s main aim is to:


a) Characterize the climate and biomes found in California
b) Give a detailed account of his daily life in California
c) Describe the weather conditions he is experiencing in California
d) Express how religious he and other Californians are
e) Tell a story about a church he has visited in California
In the cartoon, the future tense is used to express:
02. From this text, the reader concludes that: a) a plan
a) The current season is Autumn b) an order
b) It has been hot in California c) a possibility
c) Rain is expected over the next few days d) a prediction
d) Los Angeles is a polluted city e) an offer
e) The preacher likes drinking
03.
03. “I should be safe and sound now if I was miles from L.A.” means THE EARTH IS HOLLOW
that the poet
You see, the Earth is hollow and accessible via portals at the north
a) Isn’t in L.A. and south poles. Luckily thought, it’s quite habitable down there,
b) Feels free of danger in L.A. providing excellent places to live for the lost Viking colonies and the
c) Is likely to move to another city. Nazis, while aliens are in fact just visitors from the subterranean areas.
d) Wishes he was somewhere else Complete the fragment below with the grammatically correct verb tense.
e) Regrets a past situation According to the text, if the earth is hollow, you _____ it via
portals at the north and south poles.
04. “To be safe and sound”, underlined, means: a) Accessed
a) To feel excited b) Have entered
b) To feel out of danger c) Will reach
c) To make noise d) Stays

284 PROMILITARES.COM.BR
FUTURE TENSES

04. Can you lend me five hundred dollars? I _____ you back on Friday. EXERCÍCIOS DE

a) Am going to pay
b) Am paying
TREINAMENTO
c) Will pay
d) Pay 01. (EEAR)
e) Paid INSIDE LILIUM, THE WORLD’S FIRST VERTICAL TAKEOFF
AND LANDING PRIVATE JET
05. He __________ me up as soon as his daughter _____ home.
Wonder what’s in store for the future of private jet flying? Here’s
a) calls – will get a glimpse. A start-up company – hosted in a European Space Agency
b) will call – gets (ESA) business incubator center in Bavaria – released an idea for an
c) called – get egg-shaped two-seater plane called Lilium that’s currently in the
works. With a top speed of 250 mph and a range of 300 miles, the
d) called – will get
plane can travel roughly between Munich and Berlin in about 90
e) calls – got minutes. And according to the ESA, if testing succeeds, this _____ the
world’s first vertical takeoff and landing private jet.
06. Assinale a alternativa correta. The project came about when Daniel Wiegand – one of the four
He will ____ almost everything you ask him. founders of Lilium – wanted to realize flying for the masses in a fast,
a) Do inexpensive, efficient and eco-friendly way. ‘Our goal is to develop an
aircraft that doesn’t need the complex and expensive infrastructure
b) To do of an airport, can be used close to urban areas, and doesn’t produce
c) Doing too much noise and pollution,’ he said. So to produce this new
d) Does class of airplanes that could take off and land vertically anywhere
with a surface area of 250 square feet by 2018, Wiegand and his
e) Did
team in Germany came up with a design using electric engines and
incorporated movable fan turbines.
07. In the sentence “For the first time in human history, early in the www.forbes.com
next millennium, there will be more people living in cities than on the
rest of the planet”, the future form is used to express a prediction. In GLOSSARY
glimpse - uma ideia para entender melhor algo
which of the alternatives below is the future form used to express a
similar idea?
Choose the best verbal form to have thte text completed correctly:
a) Will someone help me with the luggage?
a) is
b) It will snow heavily in two days’ time.
b) was
c) If it rains, the match will be cancelled.
c) will be
d) Don’t worry. I’ll watch your dog carefully.
d) would be
e) Waiter, I’ll have some salad for lunch.
02. (CN) Mike Coleman is 19 and lives in Omaha, Nebraska, in the
08. A: Who turned up the air conditioner? It’s really cold in here. My United States. He wants to become a teacher but now he is living
nose and my fingers are cold. in Namibia. He's working in a hospital near Katima Mulilo. He says,
B: I _____ you a hot cup of tea. "I'm working with the doctors and nurses here to help sick people.
A: Thanks. That sounds good. I'm staying here for two months, and I'm living in a small house with
five other volunteers. The work is hard and the days are long, but I'm
a) Will bring enjoying my life here. When I finish the two months' work, I want to
b) Won’t bring travel in and around Namibia for three weeks."
c) Will not break Read the fragment from the text
d) Won’t break “When I finish the two months’ work, I want to travel in and
e) Will break around Namibia for three weeks.”
Because it’s a plan, it is possible to rewrite the sentence substituting
09. Read the following sentence: the underlined part for:
They’re going to have a baby in the spring. – It expresses a) Am traveling
something that: b) Like traveling
a) Is not probable to happen. c) Am going to travel
b) Is not being planned or expected. d) Can travel
c) Suddenly happens. e) Traveled
d) Is expected to happen
03. (ESFCEX) Complete the sentence with the appropriate word.
10. This time next year, we ____ round the Aegean Sea. Even though it’s ____, I ____ go to the beach.
a) Will be sailing a) Snowing – have
b) Are sailing b) Windy – don’t
c) Sail c) Raining – will
d) Sailing d) Sunny – did

PROMILITARES.COM.BR 285
FUTURE TENSES

04. (PM-MG) The negative form of the sentence “They’ll let you take a) Has left
it home” is: b) Will leave
a) They won’t let you take it home. c) Would have left
b) They don’t want to let you take it home. d) Has been leaving
c) They wouldn’t let you take it home.
d) They would never let you take it home. TEXTO PARA AS QUESTÕES 08 E 09:
Did you remember it’s April Fools Day today? My sister loves
05. (IME-ADAPTADA) Para a questão, escolha a alternativa que playing jokes. At breakfast, she said, “Robert and I ____ going to
complete a sentença CORRETAMENTE. elope! We’re going to Robben Island. It’s the place where Nelson
“I have a dream that one day, on the red hills of Georgia, Mandela was in prison. My guidebook say it is a “place forever
the sons of former slaves and the sons of former slave owners connected with the fight for freedom”. So, if you receive a phone call
_________________ sit down together at the table of brotherhood.” today about winning the lottery or something, don’t get excited. It’s
(Martin Luther King) probably an April Fools joke.
a) Will be able to (taken from Stories worth reading)

b) Should have been able to GLOSSARY


c) Are able to to elope: fugir de casa secretamente para se casar.

d) Would have been able to


08. (EEAR) Choose the correct alternative to fill in the blank.
06. (AFA) Read this dialogue between Melanie and Sam and then
a) Am c) Are
choose the correct alternatives according to it.
b) Is d) Have
Melanie: Hi, Sam. How was your interview? Hi, honey... Well, I
thought the interview itself went really well.
09. (EEAR) The sentence “We’re going to Robben Island” (line 3)
Melanie: Then why the sad face? refers to the
Sam: I think I failed. I was late getting there. a) Past
Melanie: Oh, no. Really? What happened? b) Future
Sam: I missed the train. The interview was supposed to start at 10. c) Present
Well, I was at the train station by 8:30. I bought my ticket, and then I
put my wallet down for a few seconds. When I turned around, it was d) Daily action
gone. I spent ten or fifteen minutes trying to find it. But I never did.
10. (EN) Which of the alternatives below completes the sentence
Melanie: What did you tell the interviewer?
correctly?
Sam: The truth. I said I'd missed the train. And I said I was sorry
Juliet and Romeo in the balcony scene: “Oh, Romeo, if my brother
for being late.
______ us together, he _________ you. Please, go away now.”
Melanie: Didn't you tell him about your wallet?
a) Will see – will kill
Sam: No. I was sure he'd think I was just making an excuse. I don't
b) Sees – will kill
think he even believed me about missing the train.
c) See – kill
Melanie: What did he say at the end of the interview?
d) Will see – kill
Sam: He said my qualifications were really good and he'd be in
touch. You know, "Don't call us. We'll call you." e) Will see – will kill
Adapted from True Colors - Longman

“He’d be in touch” underlined means that he


EXERCÍCIOS DE

COMBATE
a) Had already been in touch in the past
b) Has already been in touch
c) Will be in touch in the future
d) Was going to be in touch
01. There are several ways of expressing future actions. Select the
07. (EEAR) “A CARIBEAN HOLIDAY alternative which contains two of them.
Club Med is offering two lucky readers a fabulous holiday on a) I will stay here and rest. / I didn’t find that key.
the sensational island of St Lucia, with its marvelous white beaches, b) She is going to work and study abroad. / They will probably arrive
verdant rainforests and hot volcanic springs. at nine o’clock.
A two-week break at the newly renovated resort of Sainte-Lucie c) He thought he could win, but he didn’t. / I have to talk to that
________ you refreshed and revitalized, whether you’re chilling out official.
by the pool, riding the waves on a windsurf board, or exploring d) John and Peter want a new roommate. / I am going to have lunch
the glorious treasures of reefs at the foot of The Pitons, the island’s at noon.
unusual twin peaks.
How to participate 02. There are black clouds in the sky. It ______.
For your chance to win this fantastic prize, call 0901 601 4035 a) Is going to rain
and answer the following question:
b) Will rain
What are St Lucia’s twin mountains called?”
c) Is raining
Choose the correct verb form to fill in the blank.
d) Rains

286 PROMILITARES.COM.BR
FUTURE TENSES

03. I’m sorry, but I _____ able to meet you for lunch tomorrow.
a) Haven’t been RESOLUÇÃO EM VÍDEO
b) Can’t be Abra o ProApp, leia o QR Code, assista à resolução
de cada exercício e AVANCE NOS ESTUDOS!
c) Don’t be
d) Won’t be
e) Wasn’t

04. Assinale a alternativa que preenche corertamente a lacuna da GABARITO


seguinte frase:
TEXT COMPREHENSION
- What _____ to do when you get to Rio?
01. C 03. D 05. A
- I don’t know yet.
02. B 04. B
a) are you going
EXERCÍCIOS DE FIXAÇÃO
b) were you
01. A 04. C 07. B 10. A
c) did you
02. D 05. B 08. A
d) do you
03. C 06. A 09. D
e) you go
EXERCÍCIOS DE TREINAMENTO
05. You and I _____ together if we don’t want to fall at the admission 01. C 04. A 07. B 10. B
exam this year. 02. C 05. A 08. C
a) Will to study 03. C 06. C 09. B
b) Shall not to study EXERCÍCIOS DE COMBATE
c) Shall study 01. B 04. A 07. A 10. C
d) Will not to study 02. A 05. C 08. B
e) Won’t study 03. D 06. C 09. C

06. - You didn’t write to Sharon! ANOTAÇÕES

- Oh, no! I forgot! I ____ to her today.


a) Am going to write
b) Write
c) Will write
d) Am writing

07. By the end of this century, everyone _____ English.


a) Will be speaking
b) Is speaking
c) Spoke
d) speaks

08. This time next year we _____ round the Aegean Sea.
a) Are sailing
b) Will be sailing
c) Have sailed
d) Have been sailed

09. - Oh, no! I’ve split my wine.


- Don’t worry. _____ a cloth to wipe it up.
a) I get
b) I’m getting
c) I’ll get
d) I’ll be getting

10. I’m sure you ____ a lot better after a good night’s sleep.
a) Feel
b) Are feeling
c) Will feel
d) Are going to feel

PROMILITARES.COM.BR 287
FUTURE TENSES

ANOTAÇÕES

288 PROMILITARES.COM.BR
PRESENT PERFECT

PRESENT PERFECT TENSE Exemplos:


Formação: She has studied at IBEU since 2015.
I have worked here since 2017. Nesse caso, ela começou seus estudos no curso em 2015 e
ainda estuda no mesmo local. A ação não foi interrompida, e há
Auxiliar possibilidade de continuar no futuro. Outra forma de se expressar a
Sujeito “HAVE” Complemento Expressão mesma situação.
Verbo de tempo
She has studied at IBEU for about three years.
principal do
particípio As expressões que acompanham o tempo progressivo são “since”
e “for”.
• Usado para descrever uma ação ou situação que ocorreu (ou não
ocorreu) em um tempo não especificado no passado. • Indicado para mostrar que uma atitude aconteceu várias vezes
no passado.
Uma
vez Duas Três Quatro

FUTURO
PASSADO
PRESENTE

Exemplos: ? ? ?
I have been to Paris. (Nessa frase foi mencionado que o narrador Exemplo:
foi a Paris, mas não se sabe quando ele foi.)
She has been to Maracanã four times.
She has talked to her boss. (Sabe-se que ela falou com seu patrão,
mas não se sabe em que tempo no passado ou quando ela falou.) Expressões usadas nesse caso: once; twice; three times; many
Peter has visited his mother in Tijuca. times; never etc.
We have finished our exercises.
I have already been to Manaus. • Usado para indicar que uma ação aconteceu num tempo recente.
Have you ever learned Chinese?

As expressões usadas com a ideia de tempo indeterminado


geralmente são: ever, never, already, lately, today, recently, this
week etc.

• Usado para indicar uma ação progressiva de um certo ponto ou


tempo no passado até o momento em que se fala.

Exemplo:
(on the phone)
Nancy: Hello, Ms. Smith. Is John home?
John’s mother: No, Nancy. He has just left.

Nesse caso, o fato de João ter saído de casa aconteceu minutos


antes de Nancy ter ligado. O espaço de tempo entre o presente e o
passado pode ser maior do que o exemplo anterior.
Exemplo:
The scientists have just found out the human genome.

PROMILITARES.COM.BR 267
PRESENT PERFECT

• O “Present Perfect” também é usado quando a ação acontece Many ancient cultures 4have speculated about the appearance of
pela primeira vez, ou seja, ela (ação) não havia acontecido até o a second sun and this event appears to 1tie in very closely with the
momento em que se fala. Expressões geralmente usadas nessa December 21 2012 predictions.
ideia: before e never. Betelgeuse is the second 2biggest star in the universe and the
eighth 3brightest in the night sky, Scientists 5have determined that the
star is losing mass at a rapid rate, which indicates it will go supernova
very soon.
The light emitted from this exploding star will be so bright that it
will appear for a few weeks at the end of 2012 as a second sun in the
sky. There may be little if no period of darkness or night according to
senior lecturer of physics at the University of Southern Queensland,
X Brad Carter.
Exemplos: Earth will experience “brightness for a brief period of time for a
couple of weeks and then over the coming months it begins to fade
Jane hasn’t experienced this situation before. and then eventually it will be very hard to see at all,” explained the
We’ve never watched this film before. Australian scientist Brad Carter to news.com.au.
Scientist 6have known about this dying star which is 640 light
CONJUGAÇÃO DO “PRESENT PERFECT” years away from Earth, since 2005. It is believed that as Betelgeuse
goes supernova it will not be harmful to Earth. “There will be
AFIRMATIVA INTERROGATIVA NEGATIVA neutrinos emitted during the supernova process, said University of
Minnesota physics professor Priscilla Cushman, but neutrinos, even
I have not (haven’t) lots and lots of them, are only weakly interacting, so they won't affect
I have been Have I been?
been. life on earth,” but that is only speculation at this point.

You have not (haven’t) The fact is, we as human beings have never experienced anything
You have done Have you done? like this before so close to our home planet, and to be honest, we just
done.
don’t know for sure what this event could bring.
He has not (hasn’t) (www.december212012.com on 30/08/11)
He has made Has he made?
made.
The verb tenses “have speculated” (ref. 4), “have determined” (ref. 5)
She has not (hasn’t) and “have known” (ref. 6) are:
She has gone Has she gone?
gone. a) Past perfect
It has not (hasn’t) b) Simple present
It has opened Has it opened?
opened. c) Present perfect
d) Past participle
We have We have not (haven’t)
Have we visited? e) Gerund
visited visited.

You have not (haven’t) 03. (EPCAR/CPCAR 2018) “The advent of the internet hasn’t […]
You have read Have you read?
read. cooked up new conspiracy theories”.
They have Have they They have not (haven’t) Mark the correct option to make the sentence above interrogative.
cleaned cleaned? cleaned. a) Doesn’t the advent of the internet cook up new conspiracy theories?
b) Hasn’t the advent of the internet cooked up new conspiracy theories?
c) Didn’t the advent of the internet cook up new conspiracy theories?
d) Haven’t the advent of the internet cook up new conspiracy theories?
EXERCÍCIOS DE

FIXAÇÃO 04. (UECE 2014) In the sentences “Mr. Barbosa took on the entire legal
system,” “he is overseeing the precedent-setting trial,” and “Mr. Barbosa
has at times been exasperated,” the verbs are, respectively, in the:
a) simple present, present perfect, and present continuous.
01. (EPCAR/AFA 2017) Mark the option that contains the correct
negative form for the sentence “researchers have tried to identify b) past perfect, simple present, and present perfect.
learning styles”. c) simple past, present continuous, and present perfect.
a) Researchers have tried to not identify learning styles. d) simple past, present perfect, and present continuous.
b) Researchers not have tried to identify learning styles.
05. (UNIOESTE 2012) Em “it’s been bought and sold”, tem-se uma
c) Researchers do not have tried to identify learning styles.
construção verbal no tempo:
d) Researchers have tried not to identify learning styles.
a) Passado simples.
02. (UDESC 2012) b) Passado contínuo.
c) Particípio passado.
2012’S SECOND SUN
d) Presente perfeito.
Earth is believed to be getting a second sun burning in the sky near e) Passado perfeito contínuo.
the end of 2012, as the second biggest star in the universe, Betelgeuse,
is dying, which will lead to “multiple days of constant daylight.”

268 PROMILITARES.COM.BR
PRESENT PERFECT

06. (UERN 2012) Choose the item to complete the answer below. In 2001, when these attacks _____(1)_____, the cellular network was
still growing and was not as robust as it is today.
This letter is just one of the many objects that The National
Museum of American History _____(2)_____ since 2001. To learn
more, visit our online exhibition September 11th: Bearing Witness to
History.
(Adapted from http://americanhistory.si.edu/biog/handwritten-note-september-11-2001)

Choose the alternative containing the correct verb tenses to complete


gaps (1) and (2) in paragraphs 4 and 5 respectively.
a) have happened, collected
b) have happened, have collected
c) has happened, has collected
d) happened, has collected
e) happened, have collected

08. (EPCAR/AFA 2011)


a) They will go to.
Religion ________ central to people's lives in Africa. Although the
b) They have gone. majority of Africans are now Muslim or Christian, traditional religions
c) They went to. have endured and still play a big role. Religion runs like a thread
d) They would go to. through daily life, marked by prayers of gratitude in times of plenty
and prayers of supplication in times of need. Religion confirms identity
07. (ESPCEX/AMAN 2018) on the individual and the group.
(http://www.bbc.co.uk/worldservice/africa/features/storyofafrica)

Glossary:
Endure – to continue to exist for a long time
Thread – one part connecting with another

Mark the alternative that completes the gap from the text correctly.
a) has always been c) wasn’t
b) had never been d) will be

09. (UEMG 2013)


THE BIRTH OF A STORYTELLER
Jackie Torrence spent her childhood in North Carolina, in the
southern part of the United States. She was a shy child because she
had problems with her teeth, which made it hard for her to talk. Other
children teased her because of her speech problem, so she spent much
of her childhood playing alone. One of Jackie’s favorite games was to
pretend she was on television. She told stories out loud using gestures
and dramatic voices. At school, Jackie soon learned that she was good
at writing stories, and with the help of her favorite teacher, she started
to work on improving her speech.
Jackie’s first storytelling performance was in a library. She was
A HANDWRITTEN NOTE FROM SEPTEMBER 11 working as a librarian and was asked to entertain a group of children.
In moments of crisis, our first thoughts are usually to get in contact Jackie told them a story and they loved it! Before long, she began
with the people we love. September 11, 2001, was a day when many telling stories within her community. Many of her stories came from
people wanted to know that their loved ones were safe. At 9:37 a.m., old American and African-American folktales. Eventually, she started
the Pentagon was attacked by terrorists who crashed an airplane into telling stories across North America.
the western side of the building. As Jackie’s fame increased, her health decreased. She now
Many people tried using the mobile phones that existed then, has to use a wheelchair, but this has not stopped her storytelling
but few were successful. Franklin and Daria Gaillard (Frank and Chip) career. Jackie’s stories have been published in books, magazines,
were both members of the Air Force and worked at the Pentagon. and newspapers and she has appeared on radio and television. She
They worked in different parts of the building and had a previous has won awards for nine of her sound recordings and three of her
agreement that they would meet at their car in the parking lot if there television specials.
were any emergency. (Adapted from NorthStar 3: Listening and Speaking, 2nd Edition (Longman, p57),
Helen S. Solórzano and Jennifer P. L. Schmidt)
Daria was the first to arrive at the car and wrote a note to Franklin
saying “Frank - Sweetie I am okay. I’m w/ my office over by the Lyndon In the sentence, “She has won awards for nine of her sound recordings
B. Johnson Memorial Sign. I’ll stay there till you come. Love lots & lots, and three of her television specials”, the underlined expression shows that:
Chip.” Frank found the note and was able to locate his wife in the
a) the action is not expected to happen.
aftermath of the attack.
b) the action began in the past and is continuing now.
What makes this story so interesting is the handwritten note.
Today, in our digital culture, we have a variety of ways to let people c) the action expresses a past situation or habit.
know that we are safe. Text messages, voicemail, and different forms d) the action expresses an experience that happened at some time
of social media can be used to get the information out to loved ones. in one’s life.

PROMILITARES.COM.BR 269
PRESENT PERFECT

10. (EN 2016) Choose the correct option to complete this novel excerpt. “My boyfriend, John, and I 1__________ (be) together for about
“Where is Louisia?”, shouted Paul. six months. My 16th birthday was coming up and I was so excited
because my previous birthdays 2__________ (be) bad. Of course I
“Paul”, said Mrs. Schobert softly [...] “Louisia has not __________ was 3__________ (crush) when he told me his family was going away
nor __________. She has not __________ home – yet. I am sorry. I to Florida that weekend! While he was 4__________ (go), my sister
don’t know what to think.” 5__________ (take) me to the mall to get my mind off it. The whole
(HOOD, P. F. F.; HOOD, C. L. Tommorrow, soldier. Part Three: Himmler’s Gas Station. An time I kept texting him how much I missed him. I really 6__________
autobiographical novel in four parts. Milton Keynes: UK. Author House, 2007, p. 40.)
(start) to get bummed out and we 7__________ (shop) all day, so she
a) called – wrote – come decided to take me home. On the way, my sister said she wanted to
stop at her boyfriend’s house to say hi. When we 8__________ (walk)
b) called – wrote – came in my friends all screamed, ‘Surprise!’ I was happy about the party but
c) call – wrote – came still upset because John wasn’t there. Then my sister told me to go
d) call – written – came to the closet to get my presents. I walked over and opened the door,
and there stood John with a big red ribbon on his shirt! It turns out
e) called – written – come
he 9__________ (plan) the whole party just so I could finally have an
amazing birthday. I couldn’t 10__________ (ask) for a better day, or a
better boyfriend!”
EXERCÍCIOS DE

TREINAMENTO
(Disponível em: <http://www.seventeen.com/love/dating-advice/advice/a9286/love-
stories-present-perfect/>. Acesso em: junho de 2015.)

a) have been; had been; had crashed; gone; had taken; started; had
shopped; had walked; had planned; have asked.
01. (UFSJ 2013) Young Nina and her grandmother are having a b) were; have been; crush; went; taked; started; shopped; had
conversation: walked; have planned; ask.
“Grandma, how long have you and Grandpa been married?”, asked c) have be; were; have crushed; went; taken; had start; had shop;
Nina. have walk; have plan; have ask.
1
”We’ve been married for fifty years”, Grandma replied. d) have been; had been; crushed; gone; took; started; had shopped;
“That is so wonderful”, exclaimed Nina. “And I bet in all that walked; had planned; have asked.
time, you never once thought about divorce, right?” e) have been; were; had crushed; went; have taken; have start;
“Right Nina. Divorce, never. Murder, lots of times.” shopped; had walk; have planned; have asked.
(Adapted from http://www.sarasotawedding.com/jokes/divorce_jokes.html
Access on September 28th, 2012.) 04. (UDESC 2015)

In the joke, the sentence “We’ve been married for fifty years” (ref. 1) “TWERKING” BOUNCES INTO OXFORD DICTIONARY
means that Nina’s grandparents:
a) lived together for fifty years. LONDON – Twerking, the rump-busting up-and-down dance
move long beloved on America's hip-hop scene, has officially gone
b) were married for fifty years.
mainstream. 5It's got the English dictionary entry to prove it.
c) got married fifty years ago.
Britain's Oxford Dictionaries said the rapid-fire gyrations employed
d) were married for a long time. by U.S. pop starlet Miley Cyrus to bounce her way to the top of the
charts had become 3increasingly visible in the past 12 months and
02. (MACKENZIE 2012) would be added to its publications under the entry: "Twerk, verb."
Although Cyrus's eye-popping moves at Monday's MTV Video
Music Awards may have been many viewers' first introduction to
the practice, Oxford Dictionaries' Katherine Connor Martin said
"twerking" was some two decades old.
"There are many theories about the origin of this word, and since
it arose in oral use, we may never know the answer for sure," Martin
said. "We think the most likely theory is that it is an alteration of work,
because that word has a history of 4being used in similar ways, with
dancers being encouraged to 'work it.' The 't' could be a result of
“You have to motivate yourself with challenges. That’s how you blending with another word such as twist or twitch."
know you’re still alive. Once you start doing only what you _____
(I)_____ you can do, you _____(II)_____ on the road to death.” "Twerk" will be added to the dictionary as part of its quarterly
update, 1which includes words such as "selfie," the word typically
(Jerry Seinfeld) used to describe pouty smartphone self-portraits, "digital detox" for
time spent way from Facebook and Twitter, and "Bitcoin," for the
The verb forms that correctly complete the blanks I and II in the nationless electronic currency, 2whose gyrations have also caught the
quotation are: world's eye.
a) should have proved, will be Oxford Dictionaries is responsible for a range of reference works,
b) had proven, would be including Oxford Dictionaries Online, which focuses on modern usage,
and the historically-focused Oxford English Dictionary, which probably
c) have proven, are
won't be adding "twerk" to its venerable pages any time soon.
d) may have proven, can be
The definition: "Twerk, v.: dance to popular music in a sexually
e) will be proving, will have been 6
provocative manner involving thrusting hip movements and a low,
squatting stance."
03. (FPP MED 2016) Choose the correct alternative to complete the (By RAPHAEL SATTER Associated Press (www.mercurynews.com)
blanks in the correct tense with the verbs in parentheses. Accessed on: august 10th, 2014.)

270 PROMILITARES.COM.BR
PRESENT PERFECT

“Increasingly” (ref. 3), “being” (ref. 4), “It’s got” (ref. 5), “provocative” a) have already been there
(ref. 6) are consecutively: b) has already been there
a) adverb, gerund, present perfect, adjective c) was there already
b) adverb, present continuous, gerund, adverb d) will be there already
c) adjective, present continuous, adverb, adjective e) will have been there already
d) adverb, gerund, present perfect, verb
e) adverb, gerund, possessive case, adverb 07. (FUVEST) Qual destas sentenças está correta:
a) I don’t have never taken a course in Japanese;
05. (UEG 2018) Leia o texto a seguir para responder à questão. b) I have never taken a course in Japanese;
THE TRUE POTENTIAL OF TECHNOLOGY TO CHANGE BEHAVIOR c) I never didn’t take a course in Japanese still;
d) I ever did not take a course in Japanese;
Technology could successfully change behaviours where decades e) I took not a course in Japanese ever.
of campaigns and legislation have failed. With the quantified self
already walking among us and the internet of things within easy
reach, digital technology is creating unprecedented opportunities to 08. (F. SANTO ANDRÉ-SP) “This company _________ a really interesting
encourage, enable and empower more sustainable behaviours. software package.”

If we are to unlock the power of technology we must bemore a) is already began to ship
ambitious than simply digitising analogue strategies or creating b) has just begun shipping
another communications channel. c) have never begun shipping
The true potential of technology lies in its ability to do things d) has always begun to shipping
that nothing else can do. In behaviour change terms, the potential
e) was just begun to ship
to succeed where decades of education programmes, awareness
campaigns and product innovation have failed; to make a difference
where government policy and legislation has had limited impact. 09. (SANTA CASA-SP) Complete o seguinte diálogo entre dois amigos.
Using behavioural insights, it is possible to highlight the bottlenecks, – ____Peter recently?
drop out points and achilles heels of traditional behaviour change – Yes, I_____. I _____ him two days ago.
efforts — the reasons why we have failed in the past — and apply the – If you ______ him tomorrow, please give him my regards.
unique possibilities of technology to these specific challenges.
a) Did you see – did – saw – will see
OVERCOMING OUR LIMITATIONS b) Had you seen – had – had seen – will see
Luckily, the history of the human race is almost defined by its c) Do you see – do – see – saw
ability to invent stuff that bolsters its feeble capabilities. That stuff d) Have you seen – have – saw – see
is, of course, what we generically refer to as 'technology'. And in the
same way that the internal combustion engine and the light bulb 10. (MACKENZIE-SP)
allow us to overcome our relatively feeble powers of motion and
– _______ you ever ______ a dress?
perception, so digital technology can be directed to overcoming our
relatively feeble powers of reasoning, self-control, motivation, self- – Yes, I ______ one last month.
awareness and agency — the factors that make behaviour change a) Have; sewn; sewed
so difficult. b) Have; saw; sewed
Herein lies the true potential of technology: not in the laboratory c) Did; sew; have sewn
or the workshop, but in an understanding of the behavioural
dynamics that define the human condition, both generally and within d) Did; saw; have sawed
the context of a specific user-group, market segment or community. e) Do; sew; will sew
(Fonte: JOHNSON, Steven. Recognising the true potential of technology to change
behaviour.
Disponível em: <https://www.theguardian.com/sustainable-business/behavioural-
insights/true-potential-technology-change-behaviour>. EXERCÍCIOS DE

COMBATE
Acesso em: 23 ago. 2017. (Adaptado).)
Analisando-se aspectos linguísticos e estruturais do texto, constata-se que:
a) o termo “could”, em “Technology could successfully change
behaviours”, pode ser substituído por “should”, sem alteração de
sentido. 01. (AFA) In the sentence “it’s been 30 years since I developed the
b) a sentença “campaigns and legislation have failed” na forma notion of ‘multiple intelligences’”, the contraction refers to:
interrogativa seria “Do campaigns and legislation has failed?” a) It has c) It is
c) os vocábulos “successfully” e “unprecedented” são ambos b) It been d) It was
constituídos por prefixação em sua estrutura composional.
d) a sentença “legislation has had limited impact” na forma negativa 02. (EFOMM) Choose the word that correctly completes the sentence.
apresenta-se como “legislation hasn’t had limited impact”.
“I am not worried about the test. I ______ enough to pass.”
e) o vocábulo “that”, na sentença “the factors that make behaviour
a) learnt
change so difficult”, exerce a função de pronome demonstrativo.
b) have learnt
06. (EN) c) learn
– Why didn’t Peter want to go to Cabo Frio with his friends today? d) had learnt
– Because he ___________. e) learned

PROMILITARES.COM.BR 271
PRESENT PERFECT

03. (MACKENZIE)
GABARITO
A: Was that 9 o’clock train?
B: Yes, you ___________ it. EXERCÍCIOS DE FIXAÇÃO
a) have just missed d) have just missing 01. D 04. C 07. D 10. E
b) has just missed e) had just miss 02. C 05. D 08. A
c) have just miss 03. B 06. B 09. D
EXERCÍCIOS DE TREINAMENTO
04. (ITA) 01. C 04. A 07. B 10. A
A: Would you like some more whisky? 02. C 05. D 08. B
B: Yes, you ___________ it. 03. D 06. B 09. D
a) have already drunk d) has already drunk EXERCÍCIOS DE COMBATE
b) have already drink e) have already drinking 01. A 04. A 07. B 10. B
c) already drink 02. B 05. B 08. C
03. A 06. C 09. B
05. (EFOMM) Choose the word that correctly completes the sentence.
“I am not worried about the test. I ______ enough to pass.” ANOTAÇÕES
a) learnt d) had learnt
b) have learnt e) learned
c) learn

06. (SANTA CASA-SP) “Maybe she __________ any problems lately.”


a) wasn’t having d) hadn’t had
b) isn’t having e) haven’t had
c) hasn’t had

07. (UFSCAR-SP) “___________ from him for a long time.”


a) I not have heard d) I have heard not
b) I have not heard e) I haven’t hear not
c) Not I have heard

08. (ITA) “He ______ them up yesterday, although he _______ very


busy lately.”
a) rings – was d) has rung – has been
b) rang – was e) none of these
c) rang – has been

09. (CESCEM-SP) “Is Tom there? No, he hasn’t arrived ________.”


a) as far d) already
b) yet e) until
c) still

10. (EEAR) Select the alternative that best completes the dialogue below.
Jude: I ____ (see) Mary last Sunday.
John: Really? I ____ (not/see) her for years. How is she?
a) saw – haven’t see c) have seen – didn’t see
b) saw – haven’t seen d) have seen – haven’t seen

272 PROMILITARES.COM.BR
PERFECT TENSES

PRESENT PERFECT CONTINUOUS AFIRMATIVA INTERROGATIVA NEGATIVA


Esse tempo verbal é usado para indicar uma ação progressiva de
um certo ponto no passado até o presente, e que tem possibilidades de We have been Have we been We haven’t been
continuar no futuro. A diferença entre o “Present Perfect Continuous” traveling traveling? traveling
e o “Present Perfect” é que o primeiro expressa uma possibilidade
maior da ação continuar no futuro e também apresenta ênfase na You have been Have you been You haven’t been
progressão da ação até o presente. eating eating? eating

Exemplos: They have been Have they been They haven’t been
Pamela has been working at the plant for months. typing typing? typing
Judy and I have been studying English since last year.

Formação:
PAST PERFECT
John has be n wor n t IBM for two rears. Usado para indicar que uma ação (Past Perfect) aconteceu no
! r �
SujttoT Verbo Verbo
passado antes de outra ação (Simple Past) no passado.
Forma Expressão
Exemplos:
Aux. To Be Principal Gerúndio
do Joy had finished the article by 8:00 p.m.
Verbo She had never read that author before last night.
I tried to call her, but she had already left.
When I phoned she had just left.
1' 95 2004
*----+-------+
Time Expressions: by, before + (point in time), just, already,
never, after, when, before.

PASSADO I'\ FUTURO Formação:


PRESENTE

Exemplo: 1
The scientists have been working on the project since 1995. 2

CONJUGAÇÃO DO “PRESENT PERFECT


CONTINUOUS”
FUTURO
AFIRMATIVA INTERROGATIVA NEGATIVA

I have been Have I been I haven’t been


working working? working

You have been Have you been You haven’t been


teaching teaching? teaching

He has been Has he been He hasn’t been


driving driving? driving

She has been Has she been She hasn’t been


scrubbing scrubbing? scrubbing

It hasn’t been
It has been raining Has it been raining?
raining

PROMILITARES.COM.BR 273
PERFECT TENSES

CONJUGAÇÃO DO “PAST PERFECT” CONJUGAÇÃO DO “PAST PERFECT


CONTINUOUS”
AFIRMATIVA INTERROGATIVA NEGATIVA
AFIRMATIVA INTERROGATIVA NEGATIVA
I had had Had I had? I hadn’t had
I had been Had I been I hadn’t been
You had arrived Had you arrived? You hadn’t arrived
sleeping sleeping? seeping
He had thought Had he thought? He hadn’t thought
You had been Had you been You hadn’t been
She had made Had she made? She hadn’t made watching watching? watching

It had flown Had it flown? It hadn’t flown He had been Had he been He hadn’t been
washing washing? washing
We had seen Had we seen? We hadn’t seen
She had been Had she been She hadn’t been
You had gone Had you gone? You hadn’t gone walking walking? walking

They had sunk Had they sunk? They hadn’t sunk It had been Had it been It hadn’t been
working working? working

We had been Had we been We hadn’t been


listening listening? listening
PAST PERFECT CONTINUOUS
Esse tempo verbal descreve uma ação progressiva, iniciando no You had been Had you been You hadn’t been
passado e com término também no passado. painting painting? painting
Exemplos: They had been Had they been They hadn’t been
By midnight, Claudia had already been working for twelve straight calling calling? calling
hours.
She had been working for fourteen hours when I called her at
2:00 a.m.
EXERCÍCIOS DE

FIXAÇÃO
After she had been working for sixteen hours, she fell asleep.
She had been sleeping since 4:00 a.m. when she woke up at 8:00
a.m.

Formação: 01. (UERJ 2018)


By six o'clock yesterday, he had been stu yi for two exams.
��
1
rr T
OUR (IM)PERFECT BODIES
Verbo er bo prmc1pa
· · 1
Sujeito Since I write a lot about positive body image, you’d think that I
To Be no
am well over the idea that weight should be something that I allow to
Aux. particípio
passado define my life. Yet, the vestiges of my past life as a woman obsessed
with weight still linger. A good example is vacation pictures. If I show
you pictures of all the places I have been in my Iife, I can give you
1990 2000 minute details about the place itself, the food, the sights and the
weather. I can also tell you something else simply by looking at those
pictures: 1the exact number on the scale I was at that particular time
---------- in my life.
Sometimes my past catches up with me. I like to think of myself as
a recovering weight-a-holic.
The fear of being overweight is a constant one of despair at not
being personally successful in controlling your own body. What good
L ,o yea� (pe,iodo qee Efe/'"jefto is being in control of finances, major companies and businesses if
trabalhou em Los Angeles) you’re not in control of your body?! Silly idea, right? And yet that is
exactly the unconscious thought many intelligent women have.
Feeling satisfied with your appearance makes a tremendous
Before his stay in New York, he had been working for ten years amount of difference in how you present yourself to the world. Some
in Los Angeles. women live their entire lives on their perception of their physical selves.
2
But I’ve been there, done that. The hell with that idea! Personally, I
became tired of living my Iife this way.
My friend is an art historian who specializes in the Renaissance
period. Talking with him recently gave me a perspective on body
image. As we walked through the permanent exhibit of Renaissance
Art in the Metropolitan Museum of Art, he pointed out the paintings
done of women.
The women came in all sizes, all shapes. Some were curvier than
others, but all were beautiful. Some had what we refer to as love
handles; some had soft, fuller stomachs that had never suffered

274 PROMILITARES.COM.BR
PERFECT TENSES

through crunches in a gym. 3Though I had seen them many times, it 05. (ITA) “Paul ________ half the work, when they ______ in.”
was actually refreshing to view them in a new light.
a) finished – came d) has finished – had come
We are led to believe our self-worth must be a reflection of our
b) has finished – came e) had finished – came
looks. So, in essence, if we don’t believe we look good, we assume
we have no worth! Yet, self-worth should have nothing to do with c) had finished – had come
looks and everything to do with an innate feeling that you really are
worth it. 06. (FGV-SP) “He ________ him, and had seen light in his face.”
You are worth going after your dreams, you are worth being in a a) had being watching d) had been watching
good relationship, you are worth living a life that fulfills and nourishes
b) was looking e) had being looking at
you, and you are certainly worthy of being a successful woman.
c) has been looking
There is a quote attributed to Michelangelo that I’ve always
admired. When a friend complimented him on the glorious Sistine
Chapel, the great artist, referring to his art in the feminine form, was 07. (EPCAR/AFA 2020) Read the statement and mark the action that
said to have replied: “She is worthy of admiration simply because she happened first: “A study discovered that receptive music therapy had
exists; perfection and imperfection together”. decreased anxiety and stress levels before, during and after surgeries.
Also, music therapy can be applied to different levels of the disease”.
(BRISTEN HOUGHTON. Adaptado de twitter.com.)
a) discover c) can
The typical use of the underlined verb form in “Though I had seen b) decrease d) apply
them many times.“ (ref. 3) signals the following aspect of this action:
a) it happened after another c) it was a condition to another 08. (EEAR 2016 ADAPTADA)
b) it happened before another d) it was simultaneous with
another WILL’S EXPERIENCE AT THE AIRPORT

02. (UEPB 2014) After his return __________ Rome, Will couldn’t find his luggage
in the airport baggage area. He went to the lost luggage office and
HOW MONEY WORKS: WILL CHINA ON US ALL? told the woman there that his bags 1hadn’t arrived on the carousel.
She smiled and told him not to worry because they were trained
It’s no secret China has been booming while the West declines. In professionals and he was in good hands.
fact, it’s been growing so fast it’s expanding overseas, too: buying up Then she asked Will, “Has your plane arrived yet?”
businesses in the UK, U.S. and elsewhere. So, how worried should we be? (Adapted from academictips.org)
Napoleon once said, apparently. ‘Let China sleep because when
she wakes she’ll shake the world’. “Hadn’t arrived”, (ref. 1), is closest in meaning to:
Indeed, for much of the industrial revolution, China was taking a) hadn’t been found c) hadn’t been covered
a nap — so to speak. But in 1978 things began to change. The b) hadn’t been noticed d) hadn’t been delivered
Communist country encouraged private enterprise and unleashed its
biggest asset: 975 million citizens.
09. (UFSJ 2012)
Where then ensued mass migrations to urban areas where people
took up jobs in factories to manufacture goods for export. Since then
the economy dubbed ‘the dragon’ has doubled its slice of the global
economy and it’s predicted that by 2016 China will be the world’s
biggest economy.
Can anything stand in the way of the Asian powerhouse?
(From Yahoo Finance UK Friday Mar 8, 2013.)

In text, the verb forms booming, growing, expanding and buying


indicate that the events described are situated:
a) in the near future.
b) in the present.
c) long ago.
d) in the era of the Communist Revolution.
e) in the Napoleonic period.

03. (ESAN-RN) “After I ___________ a cup of coffee, I left home.”


a) have had d) have
b) had had e) had
c) has had

04. (PUC-CAMPINAS) “Although they _________ in the country since The man in the cartoon thinks the cat:
they got married, they are now moving to town.” a) was recently given some kind of “diversity product”.
a) live d) be living b) was given some “diversity product” a long time ago.
b) have been living e) None of the above applies. c) should have been given some kind of “diversity product”.
c) are living d) will have to be given some kind of “diversity product”.

PROMILITARES.COM.BR 275
PERFECT TENSES

10. (IME 2018) Leia o texto. 04. (UNESP 2008) Leia o que segue abaixo.

A DAY IN THE LIFE OF A NUCLEAR MATERIALS ENGINEER HERE IS THE SECOND PART OF THE LETTER, WRITTEN BY A
98-YEARS-OLD PENSIONED LADY TO HER BANK MANAGER.
[...] My PhD looked at auxetic polymeric materials. No one
__________ of them: they get fatter as you stretch them, which is very In due course, I will issue your employee with a PIN number,
novel, and at the time there were only a handful of researchers in the which he/she must quote in dealings with me. I regret that it cannot
world working on these. [...] be shorter than 28 digits but, again, I have modeled it on the number
(RATHBONE, Penny. Adapted from: The Guardian. A day in the life of a nuclear of button presses required of me to access my account balance on
materials engineer. Disponível em: <https://www.theguardian.com/women-in- your phone bank service. As they say, imitation is the sincerest form
leadership/2016/jan/22/a-day-in-the-life-of-a-nuclear-materials-engineer>. of flattery.
Acesso em: 22/06/2017.)
Let me level the playing field even further. When you call me, press
Escolha a alternativa que completa corretamente a lacuna do texto. buttons as follows:
a) hasn’t ever heard d) wasn’t ever heard 1. To make an appointment to see me.
b) ever heard e) had ever been heard 2. To query a missing payment.
c) had ever heard 3. To transfer the call to my living room in case I am there.
4. To transfer the call to my bedroom in case I am sleeping.
5. To transfer the call to my toilet in case I am attending to nature.
EXERCÍCIOS DE

TREINAMENTO
6. To transfer the call to my mobile phone if I am not at home.
7. To leave a message on my computer (a password to access my
computer is required. A password will be communicated to you at
a later date to the Authorized Contact).
01. (UPE 2013) Leia o seguinte texto: 8. To return to the main menu and to listen to options 1 through 8.
Sammy Sosa has been playing for the Chicago Cubs, a professional 9. To make a general complaint or inquiry, the contact will then be
baseball team located in Chicago, ____(I)____ 1993. In 1998, he and put on hold, pending the attention of my automated answering
McGwire _____(II)_____ the record for the most home runs in a single service. While this may, on occasion, involve a lengthy wait,
season. He has hit more than 50 home runs in four seasons in a row. uplifting music will play for the duration of the call.
When Sammy was a child, his family _____(III)_____ . They couldn’t Regrettably, but again following your example, I must also levy an
buy him a baseball bat, so he made one from a tree branch. Now the establishment fee to cover the setting up of this new arrangement.
Sammy Sosa Foundation raises money for poor children in Dominican Your Humble Client
Republic and Chicago. Sammy is married and has four children. (Adapted from: forums.film.com/showthread.php?t=15516)
(STEMPLESKI, S., MORGAN, J., DOUGLAS, Nancy. World Link: developing English
fluency - book 3B. Thomson Heinle: Boston, 2010. (Adaptado)) Indique a alternativa que completa a sentença: The lady _____ that she
______. The PIN number on the number of button presses required to
Vocabulary:
home run: tacada de beisebol que permite que o jogador (na posição de batedor) access her account balance.
percorra todas as bases e faça o ponto. a) wrote – has modeled d) wrote – has been modeling
Considerando a gramática e o contexto, a sequência cujas palavras b) writes – would be modeling e) wrote – had modeled
completam CORRETAMENTE as lacunas I, II e III está na alternativa:
c) was writing – modeled
a) in – broken – were very poor
b) for – reach – wasn’t very poor 05. (UFRGS 2001)
c) in – break down – was wealthy
THE TEACHER'S MAGAZINE
d) since – have broken – were careful
e) since – broke – was very poor The dramatic story of war among angels existed in heaven even
before earth was formed. The great 17th century poet John Milton
02. (EPCAR/AFA 2011) described in his masterpiece Paradise Lost what he considered the first
test of free will: the fall of angels. His story begins when Lucifer is
When football _____ professional in South Africa in 1959, 12
ordered to obey the Son of God.
clubs broke from the amateur ranks. However, in the strict days of
Apartheid, these pioneers _____ whites-only organizations and _____ Lucifer refuses, the rebellious angels join him and challenge the
today, all but a few, defunct. One of the survivors is Arcadia from power of God. On the first day, one of the powerful Seraphs and
Tshwane/ Pretoria, an outfit that today competes in the amateur ranks Lucifer meet, angel against angel. On the second day, the archangel
and concentrates on junior football. Michael enters the battle, and wounds Lucifer. Michael asks for
assistance and on the third day the Son of God comes forward. He
Mark the alternative which completes the gaps from the text correctly.
pursues the enemy to the bounds of heaven and the bad angels throw
a) had gone – have been – were themselves into the bottomless pit. The war in heaven is over, but
b) went – were – are Lucifer is far from finished. God has created a new race - humans. The
struggle between good and evil begins.
c) have been – have been – would be
(Fonte:The Teacher's Magazine, February 1998).
d) was – had been – will be
Complete the sentence below with the appropriate verb form.
03. (EFOMM) The Millers __________ Brazil for the last ten years. “When earth ______ to be, the angels' war in heaven________.”
a) has had touring d) is being touring a) came - had ended d) came - had been ending
b) have been touring e) had had touring b) comes - has ended e) comes - was ending
c) is been touring c) had come - ended

276 PROMILITARES.COM.BR
PERFECT TENSES

06. (CESGRANRIO 1994) 08. (Pucpr 2001) Select the best alternative to complete the sentences
below:
THE LANGUAGE BARRIER I. If they ____ money, they will build a big house.
Early last year, at a trade fair in Milan, a revolutionary telephone II. If you _____ out more, you would meet a few people.
system was unveiled. Developed by American and Japanese, the new III. If he ________ his restaurant, he would have got more customers.
machine provides instantaneous translation of the caller's speech. Say IV. If she ________ so fast, she wouldn’t have crashed her car.
"hello" in English and it will come out as "alô" in Portuguese or the
equivalent word in the language of your choice. It's remarkable! It V. If we _____ a car, we wouldn’t have to spend all our time waiting
might make you think that the whole business of language learning for buses.
could soon become redundant. But don't be hasty. A sophisticated
computer was programmed to perform language translation. It was a) have; went; had cleaned; hadn’t been driving; had
instructed to translate "out of sight, out of mind" into Russian. The b) have; go; cleaned; wasn’t driving; have
Russian translation was 6 then fed into the computer and translated c) had; went; was cleaned; hadn’t driven; has
back into English.
d) have had; have gone; had cleaned; hadn’t been driven; have had
The result was: "invisible lunatic". A typical error. Natural language
e) have; have gone; has cleaned; hadn’t driven; have
is so complex and ambiguous that a computer will invariably have
difficulty in making sense of it. The new phone can deal with "hello"
and other words well enough. 09. (UEG 2016) Leia o texto.
But if asked to translate a sentence, its limitations soon become MIGRANT OR REFUGEE? THERE IS A DIFFERENCE,
apparent. WITH LEGAL IMPLICATIONS
The dream of "machine translation" (MT) is almost as old as the
modern digital computer itself: the idea was promoted in 1949 and by In the first half of this year alone, at least 137,000 men, women and
the late 50's more than 20 MT projects were in development. By 1966 children crossed the Mediterranean Sea to reach the shores of Europe,
the Automatic Language Processing Advisory Committee concluded according to the United Nations. Thousands are traveling across the
that "there's no immediate or predictable prospect of useful machine Balkans now. However, are they refugee or migrants? Does it make any
translation." Research funds were cut. In the late 70's MT was re- difference? In search for these answers, let’s read the interview.
discovered. The new generation of programs is less ambitious. They
are limited to texts where the possibilities of error are minimal, such as Q: Does it matter what you call them?
technical reports and operating manuals. A: Yes. The terms “migrant” and “refugee” are sometimes used
Furthermore, the computers simply produce a workable draft interchangeably, but there is a crucial legal difference between the two.
translation, which a human "post-editor" will then correct. In spite
of their obvious limitations, MTs are extremely fast and reasonably Q: Who is a refugee?
accurate. Yet, even the most optimistic scientists admit that it'll be A: Briefly, a refugee is a person who has fled his or her country to
at least 20 years before computers are capable of translating more escape war or persecution, and can prove it.
sophisticated texts.
(Adapted from Speak Up - January 91.) Q: What does the distinction mean for European countries?
A: Refugees are entitled to basic protections under the 1951
Choose the alternative which completes the following sentences with convention and other international agreements. Once in Europe,
the adequate verb forms: refugees can apply for political asylum or another protected status,
I. The first translation program for computers _____ invented in the sometimes temporary. By law, refugees cannot be sent back to
late forties. (be) countries where their lives would be in danger. “One of the most
II. Scientists _____ research on machine translation since the 50's. fundamental principles laid down in international law is that refugees
(develop) should not be expelled or returned to situations where their life
and freedom would be under threat,” the refugee agency said in a
III. Most contemporary translators nowadays _____ computers to
statement on Thursday.
perform their task. (use)
Q: Who is a migrant?
a) (I) had been - (II) have developed - (III) will use
A: Anyone moving from one country to another is considered a
b) (I) was - (II) have been developing - (III) use
migrant unless he or she is specifically fleeing war or persecution.
c) (I) to be - (II) are developing - (III) are using Migrants may be fleeing dire poverty, or may be well-off and merely
d) (I) has been - (II) develop - (III) will be using seeking better opportunities, or may be migrating to join relatives who
have gone before them. There is an emerging debate about whether
e) (I) would have been - (II) developed - (III) have been using
migrants fleeing their homes because of the effects of climate change
– the desertification of the Sahel region, for example, or the sinking
07. (EEAR 2011 - ADAPTADA) of coastal islands in Bangladesh – ought to be reclassified as refugees.
THE ROAD TO SUCCESS
Q: Are migrants treated differently from refugees?
They live in New Jersey, where Nunes runs a trading company. A: Countries are free to deport migrants who arrive without
He has been living abroad for decades and prefers not to appear in legal papers, which they cannot do with refugees under the 1951
interviews or TV shows. convention. So it is not surprising that many politicians in Europe
“He has been living abroad for decades...”, (line 11), means that prefer to refer to everyone fleeing to the continent as migrants.
Nunes
(Disponível em: <http://www.nytimes.com/2015/08/28/world/migrants-refugees-
a) used to live abroad. europe-syria.html?_r=0>. Acesso em: 15 set. 2015.)
b) is still living abroad.
c) has been to different countries.
d) has recently moved to another country.

PROMILITARES.COM.BR 277
PERFECT TENSES

Considerando os aspectos estruturais do texto, tem-se o seguinte: 06. (PUC-PR) Mrs. Stone really thanked me for what I ________for her.
a) A sentença “refugees cannot be sent back to countries where a) had done d) doing
their lives would be in danger”, na voz ativa, seria: They could not b) done e) having done
send refugees back to countries where their lives will be in danger.
c) have done
b) A sentença “Thousands are traveling across the Balkans now”,
no tempo present perfect continuous, poderia ser assim expressa:
07. (EFOMM) The lecture ______ to an end before I ______ in the room.
Thousands have been traveling across the Balkans lately.
a) has come, am arriving
c) O modal “may” é usado nas sequências “…may be fleeing dire
poverty”, “…may be well-off” e “…may be migrating to join b) will come, had arrived
relatives”, indicando possibilidade futura. c) came, arrive
d) O termo em destaque em “...which they cannot do with refugees d) had come, arrived
under the 1951 convention”, no texto, é um pronome relativo e) comes, have arrived
que se refere a “legal papers”.
08. (MACKENZIE) Only then we ________ that we _______ enough
10. (PUC-PR 1998) Choose the RIGHT alternative to complete the for the test.
passage:
a) had realized, has studied
“Everything ____________ ready for the party. The room
_____________, the furniture _______. There ______________ bottles b) have realized, haven’t studied
of wine and food on the table. A jazz record __________ and the c) realized, hadn’t studied
atmosphere was just right.” d) had realized, have been studying
a) is - is clean - is moved - are - is playing e) have realized, hadn’t studying
b) was - had been cleaned - had been moved - were - was playing
c) had been - had been cleaned - had been moved - were - had 09. (EFOMM) Mark the correct alternative.
been played “When I saw a diamond necklace in a local jewelry store, I knew it was
d) was - had cleaned - had moved - was - had played exactly what I ______ for.”
e) were - was cleaned - was moved - were - was playing a) have looked
b) had looked
c) have been looking
EXERCÍCIOS DE d) had been looking

COMBATE e) looking

10. (EN) Brazil _____ more than 4,000 miles of coastline, and
Brazilians _______ its waves at least since the Australian surfer Peter
01. (CESESP-PE) “My parents are French but they ________ in Brazil Troy ______ a demonstration in Rio de Janeiro in 1964. So why the
since 1934.” sudden dominance? The answer is that the country itself ______.
a) live d) are living (http://nytimes.com)

b) lived e) has been living a) has/ have surfed/ has given/ has changed
c) have been living b) has/ have been surfing/ has given/ has changed
c) has/ have been surfing/ gave/ has changed
02. (FMU-SP) Paul ___________ in Germany since 1998.
d) has had/ have been surfing /gave/ changed
a) lived d) lives
e) has had/ have surfed/ has given/ changed
b) has been living e) live
c) will live
GABARITO
03. (PUC-RS) My brother _________ to me for months.
a) doesn’t write d) has not been writing EXERCÍCIOS DE FIXAÇÃO
b) don’t write e) have written 01. B 04. B 07. B 10. C
c) have not written 02. B 05. E 08. E
03. B 06. D 09. A
04. (FGV-SP) He ________ him before she left. EXERCÍCIOS DE TREINAMENTO
a) had being watching d) had been watching 01. E 04. E 07. B 10. B
b) was looking e) had being looking at 02. B 05. A 08. A
c) has been looking 03. B 06. B 09. B
EXERCÍCIOS DE COMBATE
05. (UNIP) Things ___________ a lot since October.
01. C 04. D 07. D 10. C
a) changed has d) change
02. B 05. C 08. C
b) are changing e) has changed
03. D 06. A 09. D
c) have been changing

278 PROMILITARES.COM.BR
MODAL VERBS

CARACTERÍSTICAS DOS MODAIS • CAN possibility /impossibility - possibilidade/impossibilidade


A forma infinitiva não apresenta to antes do verbo, é sempre Anyone can become rich and famous if they win the lottery.
seguido de infinitivo sem to. It can’t cost more than two hundred dollars.
Exemplos: You can’t be 62! I thought you were about 40 years old.
Mariane can speak Portuguese. (This use is usually a generalization or an supposition.)
We should take a laptop with us.
• COULD possibility - possibilidade
Não são acrescidos de s na 3ª pessoa do singular. Paul could be the one who entered to AFA.
Exemplos: Paul could have been the one who entered to AFA.
I’m sure Paul can do this for you. Paul could be charged with the crime when the Polícia Federal
He must be there at 7 a.m. finish the investigation.

Funcionam como auxiliares comuns, tanto na forma afirmativa, • COULD suggestion - sugestão/ conselho
negativa e interrogativa. You could study more.
Na forma interrogativa, eles se colocam na frente do sujeito e na You could have spent your vacation in Hawaii.
negativa basta acrescentar o “not” depois do verbo. You could spend your vacation in Hawaii.

Afirmativa: They should wake up earlier. • COULD past ability - habilidade no passado
Negativa: They should not (shouldn’t) wake up earlier. I could run ten miles in my twenties.
Interrogativa: Should they wake up earlier? I could speak Japanese when I was a kid.
Interrogativa-negativa: Shouldn’t they wake up earlier? (= I couldn’t run more than a mile in my twenties.
Should they not wake up earlier?)

• COULD polite request - pedido educado


• CAN general ability - habilidade / capacidade
Could I have something to drink?
I can speak Chinese.
Could borrow your stapler?
I could speak Chinese when I was a kid.
I will be able to speak Chinese by the time I finish my course.
• MAY possibility - possibilidade
Paul may be upset. I can’t really tell if he is annoyed or tired.
• CAN permission - permissão
Paul may have been upset. I couldn’t really tell if he was annoyed
I can drive my mother’s car when she is out of town. or tired.
I was allowed to drive my mother’s car while she was out of town
last week. (be allowed to denota permissão. O verbo to be pode
• MAY permission - permissão
estar conjugado nas diferentes formas verbais)
You may leave the classroom now that you’re finished with your
I can drive my mother’s car while she is out of town next week.
exercises.
Can I take your car? Yes you can / no you can’t.
You were allowed to leave the classroom after you finished your
exercises.
• CAN request - pedido You may leave the classroom when you finish your exercises.
Can I have a glass of water?
Can you give me a lift to school? • MIGHT possibility - possibilidade
(Requests usually refer to the near future.) She might be on the subway. I think her car is having problems.
She might have taken the subway. I’m not sure how she got home.

PROMILITARES.COM.BR 279
MODAL VERBS

• MUST certainty - certeza EXERCÍCIOS DE


That must be Patrick. They said he was tall with bright red hair.
That must have been the right school. There were no other FIXAÇÃO
schools on this street.
That must not be Patrick. He is supposed to have black hair.
01. (MACKENZIE-SP) In the sentence: “You may be wrong, and you
may be right”, “may” expresses:
• MUST (not) prohibition - proibição a) possibility. c) ability. e) obligation.
You must not walk on that street. It’s dangerous. b) permission. d) deduction.

• MUST strong recommendation - forte recomendação 02. (NESP) Assinale a alternativa que completa corretamente a frase:
She must take some time off and get some rest. “Could I__________ earlier tomorrow?”
She mustn’t drink so much. It’s not good for her. a) to leave c) leaves e) leaving
b) leave d) left
• MUST necessity - necessidade
03. (EEAR 2019) In the famous words by John Lennon: “You may say
You must have a permit to enter the U.S.A.
I’m a dreamer but I’m not the only one. I hope someday you’ll join us
We had to have a permit to enter the U.S.A. (have to denota and the world will be as one”, the modal verb in bold indicates that:
necessidade. Pode ser conjugado nas diferentes formas verbais)
a) He knew that he could dream about peace and try to help everybody.
We must get a permit to enter the park next week.
b) He understood that he could be seen as a dreamer.
We don’t have to get a permit to enter the Quinta da Boa Vista.
c) He was not able to dream about peace and love.
(don’t have to denota falta de obrigação, não necessidade)
d) He considered himself the only dreamer.

• SHOULD recommendation / advisability - recomendação / conselho 04. (UNESP 2018) Leia o texto.
People with high cholesterol should eat low fat foods.
WHEN IT COMES TO POLITICS AND “FAKE NEWS”,
John should have eaten low fat foods. That might have prevented FACTS AREN’T ENOUGH
his heart attack.
You really should start eating better.

• SHOULD assumption / expectation / probability - suposição /


expectativa / probabilidade
Peter should be in Salvador by now.
Mary should have arrived in Porto Alegre last week. Let’s call her
and see what she is up to.

• OUGHT TO recommendation / advisability - recomendação /


conselho
Frank ought to exercise more.
Frank ought to have exercised more so she would be better In today’s political climate, it sometimes feels like we can’t even
prepared for the marathon. agree on basic facts. We bombard each other with statistics and figures,
hoping that more data will make a difference. A progressive person
• OUGHT TO assumption / expectation / probability - suposição / might show you the same climate change graphs over and over while
expectativa / probabilidade a conservative person might point to the trillions of dollars of growing
national debt. We’re left wondering, “Why can’t they just see? It’s so
They ought to have the package by now. obvious!”
They ought to have received the package yesterday. Certain myths are so pervasive that no matter how many experts
disprove them, they only seem to grow in popularity. There’s no shortage
• HAD BETTER recommendation - recomendação / conselho. of serious studies showing no link between autism and vaccines, for
Desperate hope example, but these are no match for an emotional appeal to parents
worried for their young children. Tali Sharot, a cognitive neuroscientist
You had better unplug the oven before you try to clean it.
at University College London, studies how our minds work and how we
You had better not clean the house before 8 o’clock. process new information. In her upcoming book, The Influential Mind,
she explores why we ignore facts and how we can get people to actually
• SHALL future action - ação futura listen to the truth.

The manager shall be replaced by someone from the São Paulo Tali shows that we’re open to new information – but only if it
office. confirms our existing beliefs. We find ways to ignore facts that challenge
our ideals. And as neuroscientist Bahador Bahrami and colleagues have
Paul shall be there by 9:00. found, we weigh all opinions as equally valid, regardless of expertise.
So, having the data on your side is not always enough. For better
or for worse, Sharot says, emotions may be the key to changing minds.
(Shankar Vedantam. www.npr.org. Adaptado.)

280 PROMILITARES.COM.BR
MODAL VERBS

No trecho do quarto parágrafo “emotions may be the key to changing 6


Someone is in slavery if they are:
minds”, o termo sublinhado pode ser substituído, sem alteração de • forced to work – through coercion, or mental or physical threat;
sentido no texto, por:
• 7
owned or controlled by an ’employer’, through mental or
a) must. c) can. e) will. physical abuse or the 8threat of abuse;
b) has to. d) used to. • dehumanised, treated as a commodity or bought and sold as
9
property’;
05. (UNESP 2016) Leia o texto para responder às questões abaixo. • physically constrained or have restrictions placed on their
freedom of 10movement.
GENETICALLY MODIFIED FOODS
Slavery has been a disgraceful aspect of human society for most
Genetically modified (GM) foods are foods derived from organisms of human history. However, Anti-Slavery International has refused to
whose genetic material (DNA) has been modified in a way that does accept that this bloody status quo should be allowed to persist.
not occur naturally, e.g. through the introduction of a gene from a (Aidan McQuade, former director)
different organism. Currently available GM foods stem mostly from
plants, but in the future foods derived from GM microorganisms or
FORMS OF MODERN SLAVERY
GM animals are likely to be introduced on the market. Most existing
genetically modified crops have been developed to improve yield, [...]
through the introduction of resistance to plant diseases or of increased Forced and early marriage – when someone is married against
11

tolerance of herbicides. their will and cannot leave the marriage. Most child marriages can be
In the future, genetic modification could be aimed at altering considered slavery.
the nutrient content of food, reducing its allergenic potential, or [...]
improving the efficiency of food production systems. All GM foods In the topic “Forced and early marriage” (ref. 11), the modal verb can
should be assessed before being allowed on the market. FAO/WHO be replaced by __________ without changing the meaning.
Codex guidelines exist for risk analysis of GM food.
(www.who.int)
a) mustn’t
b) shouldn’t
No trecho do segundo parágrafo “All GM foods should be assessed c) doesn’t have to
before being allowed on the market.”, o termo em destaque pode ser
corretamente substituído, sem alteração de sentido, por: d) doesn’t need to

a) could. d) ought to.


08. (EFOMM 2018) Mark the correct alternative.
b) has to. e) used to.
a) We had better not to call him today.
c) might.
b) I ought to read this book, oughtn’t I?

06. (ESPM 2015) The question refers to the following comic strip: c) John is eighteen, so his sister must to be twenty.
d) I suggested she added this book to the list, and she finally would.
e) Last year I lived with my boyfriend but I knew I can live with my
parents again at any time.

09. (UNIFESP 2018)

In the last strip the modal verb couldn’t could be replaced, without
changing its meaning, by:
a) mustn’t d) might not
b) wasn’t supposed to e) am not able to
c) shouldn’t

07. (EPCAR/CPCAR 2019)

WHAT IS MODERN SLAVERY?


1
Slavery did not end with abolition in the 19th century. 2Slavery
continues today and harms people in every country in the world.
3

Women forced into prostitution. People forced to work in


agriculture, domestic work and factories. Children in 4sweatshops
producing 5goods sold globally. Entire families forced to work for
nothing to pay off generational debts. Girls forced to marry older men.
There are estimated 40.3 million people in modern slavery around
the world, including:
• 10 million children
• 24.9 million people in forced labour
• 15.4 million people in forced marriage
• 4.8 million people in forced sexual exploitation

PROMILITARES.COM.BR 281
MODAL VERBS

A lacuna no quarto quadrinho deve ser preenchida por: With a landmass larger than the continental US, Brazil emits
a) is writing. d) might write. about 2.5% of the world’s carbon dioxide and other polluting gases,
according to United Nations data.
b) have written. e) could have written.
“Our government is concerned about the future,” said President
c) used to write. Michel Temer during a signing ceremony in Brasilia. “Everything we do
today is not aimed at tomorrow, but rather at a future that preserves
10. (UNESP 2017) the living conditions of Brazilians.”
QUESTION: IS THERE ANYTHING I CAN DO TO Temer said Brazil’s ratification would be presented formally to the
TRAIN MY BODY TO NEED LESS SLEEP? UN later this month.
Karen Weintraub The Paris agreement will enter into force once 55 countries
June 17, 2016 representing at least 55% of global emissions have formally joined it.
Climate experts say that could happen later this year.
Countries set their own targets for reducing emissions. The
targets are not legally binding, but nations must update them every
five years. Using 2005 levels as the baseline, Brazil committed to
cutting emissions 37% by 2025 and an “intended reduction” of 43%
by 2030.
In the last decade, Brazil has achieved significant emissions cuts
thanks to efforts to reduce deforestation in the Amazon and increase
in the use of energy from hydropower and other renewable sources
including wind, solar and biomass.
The Paris accord got a boost earlier this month when the US
president, Barack Obama, and China’s President, Xi Jinping, sealed
Many people think they can teach themselves to need less sleep, their nations’ participation.
but they’re wrong, said Dr. Sigrid Veasey, a professor at the Center for
“Brazil is now the next major country to move forward. It will
Sleep and Circadian Neurobiology at the University of Pennsylvania’s
add even greater momentum,” said David Waskow, director of the
Perelman School of Medicine. We might feel that we’re getting by fine
International Climate Initiative at the Washington, DC-based think
on less sleep, but we’re deluding ourselves, Dr. Veasey said, largely
tank the World Resources Institute.
because lack of sleep skews our self-awareness. “The more you
deprive yourself of sleep over long periods of time, the less accurate (Source: https://www.theguardian.com/environment/2016/sep/13/brazil-ratifies-paris-
agreement-with-pledge-to-sharplyreduce-emissions.)
you are of judging your own sleep perception,” she said.
Multiple studies have shown that people don’t functionally adapt Why has the author used the modal “must” in the sentence:
to less sleep than their bodies need. There is a range of normal sleep “The targets are not legally binding, but nations must update them
times, with most healthy adults naturally needing seven to nine hours every five years”?
of sleep per night, according to the National Sleep Foundation. Those
over 65 need about seven to eight hours, on average, while teenagers a) Because the author meant to say that the update of nation’s set
need eight to 10 hours, and school-age children nine to 11 hours. targets every five years is a recommendation.
People’s performance continues to be poor while they are sleep b) Because the author meant to convey the idea of a future possibility
deprived, Dr. Veasey said. of nations update their set targets every five years.
Health issues like pain, sleep apnea or autoimmune disease can c) Because the author meant to convey the idea of what is right or
increase people’s need for sleep, said Andrea Meredith, a neuroscientist correct to do.
at the University of Maryland School of Medicine. A misalignment of d) Because the author meant to convey the idea of an obligation of
the clock that governs our sleep-wake cycle can also drive up the need nations update their set targets every five years.
for sleep, Dr. Meredith said. The brain’s clock can get misaligned by
being stimulated at the wrong time of day, she said, such as from
02. (ESPM 2017) The question refers to the following comic strip:
caffeine in the afternoon or evening, digital screen use too close to
bedtime, or even exercise at a time of day when the body wants to
be winding down.
(http://well.blogs.nytimes.com. Adaptado.)

No trecho do primeiro parágrafo “We might feel that we’re getting


by fine on less sleep”, o termo em destaque pode ser substituído, sem
alteração de sentido, por
a) could. c) will. e) has to.
b) ought to. d) should.
In the sentence “it had better not be!” in the first frame, the boy:
a) advises his mother not to put in rice in the food.
EXERCÍCIOS DE
b) says that they had better food the day before.

TREINAMENTO c) indicates that he is about to refuse to eat.


d) is thrilled with the possibility of eating the rice.
e) complains that the meal is not better today.
01. (ACAFE 2017) Answer the question based on the text below.
The Brazilian government has ratified its participation in the Paris
agreement on climate change, a significant step by Latin America’s
largest emitter of greenhouse gases that could spur other countries
to follow suit.

282 PROMILITARES.COM.BR
MODAL VERBS

03. (UFSJ 2012) “I decided to be in Lebanon because I believe that the voices of
the Syrian refugees need to be heard and they have been ignored
CLASSIC ADVERTISING MISTAKE: ARABIC VERSION for so long,” Malala told Reuters in a schoolroom decorated with
drawings of butterflies. The Malala Fund, a non-profit organization
A disappointed salesman of Coca Cola returns from his assignment that supports local education projects, provided most of the funding
in Arabia. for the school, set up by Lebanon’s Kayany Foundation in the Bekaa
A friend asked, “Why weren’t you successful with the Arabs?” Valley, close to the Syrian border. The Kayany Foundation, established
by Syrian Nora Joumblatt in response to Syria’s refugee crisis, has
The salesman explained, “When I got posted in the Middle East, I already completed three other new schools to give free education to
was very confident that I would make a good sales pitch in rural areas. Syrian children in Lebanon. The Malala school can welcome up to 200
But, I had a problem I didn’t know how to speak Arabic. So, I planned girls aged 14 to 18.
to convey the message through three posters...”
“Today on my first day as an adult, on behalf of the world’s
children, I demand of leaders we 1must invest in books 2instead
of bullets,” Malala said in a speech. Lebanon is home to at least
1.2 million of the 4 million refugees that have fled Syria’s war to
neighboring countries. There are about 500,000 Syrian school-age
children in Lebanon, but only a fifth are in formal education. “We
are in danger of losing generations of young Syrian girls due to the
lack of education,” Joumblatt said in a speech at the opening of the
• First poster – A man lying in the hot desert sand... totally school. “Desperate and displaced Syrians are increasingly seeing early
exhausted and fainting. marriage as a way to secure the social and financial future of their
• Second poster – man is drinking our Cola. daughters. We need to provide an alternative: Keep young girls in
school instead of being pressured into wedlock.”
• Third poster – Our man is now totally refreshed.
Lebanon, which allows informal settlements on land rented by
Then these posters were pasted all over the place. refugees, says it can no longer cope with the influx from Syria’s four-
1
“That should have worked,” said the friend. year conflict. More than one in four people living in Lebanon is a
The salesman replied “l didn’t realize that in Arabic you read from refugee. The United Nations says the number of Syrian refugees in
right to left.” neighboring countries is expected to reach 4.27 million by the end of
(Taken from: Clipmarks.com Ava ii able at: http://clipmarks.com/clipmark/BC4C6FAC- the year. “In Lebanon as well as in Jordan, an increasing number of
C464-4D58-A4B5-BAOE7267CF79/. Access: August 30th, 2011.) refugees are being turned back at the border,” Malala said. “This is
inhuman and this is shameful.”
In the text, the sentence “That should have worked” (ref. 1) means Her father Ziauddin said he was proud she was carrying on her
that the strategy used by the salesman activism into adulthood. “This is the mission we have taken for the last
a) will work properly, so the salesman can use it. 8-9 years. A small moment for the education of girls in Swat Valley: it
b) worked as expected in spite of the problems. is spreading now all over the world,” he said.
(www.reuters.com. Adaptado.)
c) did not work, but it was a good strategy.
d) will not work and the salesman must not try it. Termo “must” (ref. 1) pode ser substituído, sem alteração de sentido, por
a) has to. c) might. e) ought to.
04. (UNIFESP 2016) Leia o texto.
b) can. d) used to.
NOBEL WINNER MALALA OPENS SCHOOL FOR SYRIAN REFUGEES
05. (ITA 2018) Leia o que segue abaixo.
Sylvia Westall
July 13, 2015 GOODBYE THINGS, HELLO MINIMALISM:
Bekaa Valley, Lebanon CAN LIVING WITH LESS MAKE YOU HAPPIER?
1

Fumio Sasaki owns a roll-up mattress, three shirts and four pairs of
socks. After deciding to scorn possessions, he began feeling happier.
He explains why.
Let me tell you a bit about myself. I’m 35 years old, male, single,
never been married. I work as an editor at a publishing company. I
recently moved from the Nakameguro neighbourhood in Tokyo,
where I lived for a decade, to a neighbourhood called Fudomae in a
different part of town. 2The rent is cheaper, but the move pretty much
wiped out my savings.
Some of you may think that I’m a loser: an unmarried adult with
not much money. The old me would have been way too embarrassed
to admit all this. I was filled with useless pride. But I honestly don’t
care about things like that any more. The reason is very simple: I’m
perfectly happy just as I am. The reason? I got rid of most of my
material possessions.
Malala Yousafzai, the youngest winner of the Nobel Peace Prize,
celebrated her 18th birthday in Lebanon on Sunday by opening a Minimalism is a lifestyle in which 3you reduce your possessions
school for Syrian refugee girls and called on world leaders to invest to the least possible. Living with only the bare essentials has not only
in “books not bullets”. Malala became a symbol of defiance after provided superficial benefits such as the pleasure of a tidy room or the
she was shot on a school bus in Pakistan in 2012 by the Taliban for simple ease of cleaning, 4it has also led to a more fundamental shift.
advocating girls’ rights to education. She continued campaigning and It’s given me a chance to think about what it really means to be happy.
won the Nobel in 2014.

PROMILITARES.COM.BR 283
MODAL VERBS

We think that 5the more we have, the happier we will be. 6We 06. (PUC-RJ 2017)
never know what tomorrow might bring, so we collect and save as
much as we can. This means we need a lot of money, so we gradually FALCON SOARS INTO SPACE AND LANDS IN THE OCEAN
start judging people by how much money they have. You convince
yourself that you need to make a lot of money so you don’t miss out Musk’s Falcon 9 rocket had just shot 200 km up into space and
on success. And for you to make money, you need everyone else to flown almost horizontal to the planet at six times the speed of sound,
spend their money. And so it goes. before falling back to Earth. Then, somehow, it landed like a feather
on a robotic barge in the ocean. The Falcon even found time to put an
So I said goodbye to a lot of things, many of which I’d had for
inflatable space habitat into orbit, too.
years. And yet now I live each day with a happier spirit. 7I feel more
content now than I ever did in the past. [...]
I wasn’t always a minimalist. I used to buy a lot of things, believing Musk must prove that his Falcon 9 rocket can be re-flown with
that all those possessions would increase my self-worth and lead to modest modifications. After the first SpaceX landing in December, the
a happier life. I loved collecting a lot of useless stuff, and I couldn’t company performed a static firing test of the 1vehicle, 2which went
throw anything away. I was a natural hoarder of knick-knacks that I well until one of the nine engines showed thrust fluctuations. That
thought made me an interesting person. rocket will stand as a monument outside the company’s headquarters
in Hawthorne, California. Perhaps the Falcon 9 that landed at sea will
At the same time, though, I was always comparing myself with
be flown again.
other people who had more or better things, 8which often made me
miserable. I couldn’t focus on anything, and I was always wasting “It should fly again,” Musk said Friday. SpaceX will try to return
3

time. Alcohol was my escape, and I didn’t treat women fairly. I didn’t the booster back to Cape Canaveral, in Florida, by Sunday. After
try to change; I thought this was all just part of who I was, and I running a series of tests on the Falcon, the company plans to fire its
deserved to be unhappy. engines 10 times in a row on the ground. “If things look good, it will
be qualified for reuse,” Musk said. “We’re hoping to relaunch it on an
My apartment wasn’t horribly messy; if my girlfriend was coming
orbital mission, let’s say by June”, he added. [...]
over for the weekend, I could do enough tidying up to make it look
(By Eric Berger Retrieved and adapted from http://arstechnica.com/science/2016/04/
presentable. On a usual day, however, there were books stacked like-a-boss-falcon-soars-into-space-and-lands-in-the-ocean/
everywhere because there wasn’t enough room on my bookshelves. Access on July 2nd, 2016.)
Most I had thumbed through once or twice, thinking that 9I would
read them when I had the time. In the fragment of the text “‘It should fly again,’ Musk said Friday”
The closet was crammed with what used to be my favorite (ref. 3) the modal verb “should” implies an idea of
clothes, most of which I’d only worn a few times. The room was filled a) advice. d) possibility.
with all the things I’d taken up as hobbies and then gotten tired of. b) necessity. e) request.
A guitar and amplifier, covered with dust. Conversational English
workbooks I’d planned to study once I had more free time. Even a c) permission.
fabulous antique camera, 10which of course I had never once put a
roll of film in. 07. (UFRGS 2018) A questão a seguir está relacionada ao texto abaixo.
It may sound as if I’m exaggerating when I say I started to
11 _____1_____ September 11, 2001, at 8:46 A.M., a hijacked
become a new person. Someone said to me: “All you did is throw airliner crashed into the north tower of the World Trade Center in New
things away,” which is true. 12But by having fewer things around, I’ve York. At 9:03 A.M. a second plane crashed into the south tower. The
started feeling happier each day. I’m slowly beginning to understand resulting infernos caused the buildings to collapse, 1the south tower
what happiness is. after burning for an hour and two minutes, the north tower twenty-
three minutes after that. 2The attacks were masterminded by Osama
If you are anything like I used to be – miserable, constantly
bin Laden in an attempt to intimidate the United States and unite
comparing yourself with others, or just believing your life sucks – 13I
Muslims for a restoration of the caliphate.
think you should try saying goodbye to some of your things. […]
Everyone wants to be happy. But trying to buy happiness only makes 9/11, as the happenings of that day are now called, has set
us happy for a little while. off debates on a vast array of topics. But I would like to explore a
(Fonte: adaptado de <https://www.theguardian.com/books/2017/apr/12/goodbye- lesserknown debate triggered by it. Exactly how many events took
things-hello-minimalism-can-living-with-lessmake-you-happier>. place in New York on that morning _____2_____ September?
Acesso em: 21 mai. 2017.) 3
It could be argued that the answer is one. The attacks on the two
buildings were part of a single plan conceived by one man in service of
Marque a opção correta de acordo com o sentido com que os verbos
a single agenda. They unfolded _____3_____ a few minutes and yards
modais sublinhados são empregados no texto.
of each other, targeting the parts of a complex with a single name,
I. Can living with less make you happier? (ref. 1) → para indicar uma design, and owner. And they launched a single chain of military and
possibilidade. political events in their aftermath.
II. We never know what tomorrow might bring, (ref. 6) → para Or it could be argued that the answer is two. The towers were
indicar um estado contrário à realidade. distinct collections of glass and steel separated by an expanse of
III. It may sound as if I’m exaggerating… (ref. 11) → para indicar uma space, and they were hit at different times and went out of existence
probabilidade. at different times. The amateur video that showed the second plane
IV. I think you should try saying goodbye to some of your things. (ref.
4
closing in on the south tower as the north tower billowed with smoke
13) → para dar um conselho. makes the twoness unmistakable: while one event was frozen in the
past, the other loomed in the future.
Estão corretas:
The gravity of 9/11 would seem to make this discussion frivolous
a) I e II. to the point of impudence, a matter of mere “semantics,” as we say,
b) I, II e IV. with its implication of 5splitting hairs. But the relation of language to
c) I, III e IV. our inner and outer worlds is a matter of intellectual fascination and
real-world importance.
d) II, III e IV.
_____4_____ “importance” is often hard to quantify, _____5_____
e) II e IV.
this case I can put an exact value on it: 3,5 billion dollars. That was
the sum in a legal dispute for the insurance payout to Larry Silverstein,

284 PROMILITARES.COM.BR
MODAL VERBS

the leaseholder of the World Trade Center site. Silverstein’s insurance 09. (UEG 2015) Leia o texto a seguir para responder à questão.
policies stipulated a maximum reimbursement for each destructive
“event.” If 9/11 comprised a single event, he stood to receive 3,5 CATEGORIES OF TERRORIST GROUPS
billion dollars; if two, he stood to receive 7 billion. In the trials, the
attorneys disputed the applicable meaning of the term event. The There are many different categories of terrorism and terrorist
lawyers for the leaseholder defined it in physical terms (two collapses); groups that are currently in use. These categories serve to differentiate
those for the insurance companies defined it in mental terms (one terrorist organizations according to specific criteria, which are usually
plot). There is nothing “mere” about semantics! related to the field or specialty of whoever is selecting the categories.
(Adapted from: PINKER, Steven. The Stuff of Thought. New York: Penguin, 2007. p. 1-2.) Also, some categories are simply labels appended arbitrarily or
redundantly, often by the media. For example, every terrorist
Consider the following propositions for rephrasing the clause It could organization is by definition “radical”, as terror tactics are not the
be argued that the answer is one (ref. 3). norm for the mainstream of any group.
I. One might argue that the answer is one. Separatist. Separatist groups are those with the goal of separation
II. You could say that the answer is one. from existing entities through independence, political autonomy, or
religious freedom or domination.
III. They should argue that the answer is one.
Revolutionary. Dedicated to the overthrow of an established
If applied to the text, which ones would be correct and keep the literal order and replacing it with a new political or social structure.
meaning?
Political. Political ideologies are concerned with the structure and
a) Only I. c) Only III. e) I, II and III. organization of the forms of government and communities.
b) Only II. d) Only I and II. Religious. Religiously inspired terrorism is on the rise. While
Islamic terrorists and organizations have been the most active, and
08. (UEG 2017) Leia com atenção o texto a seguir e responda à questão. the greatest recent threat to the United States, all of the major world
religions have extremists that have taken up violence to further their
THE INTERNET OF THINGS perceived religious goals.
International or Transnational. International groups typically
The “Internet of Things” (IoT) is becoming an increasingly operate in multiple countries, but retain a geographic focus for their
growing topic of conversation both in the workplace and outside of activities. Transnational groups operate internationally, but are not tied
it. It’s a concept that not only has the potential to impact how we live to a particular country, or even region.
but also how we work. But what exactly is the “Internet of Things” (Disponível em: <http://www.terrorism-research.com/groups/categories.php>.
and what impact is it going to have on you, if any? There are a lot of Acesso em: 05 set. 2014. (Adaptado).)
complexities around the “Internet of Things” but we want to stick to
the basics. Lots of technical and policy-related conversations are being A estrutura linguística do texto evidencia que
had but many people are still just trying to grasp the foundation of a) a sentença “There are many different categories of terrorism
what the heck these conversations are about. and terrorist group” expressaria uma possibilidade na seguinte
Let’s start with understanding a few things. forma: There may be different categories of terrorism and terrorist
Broadband Internet is becoming more widely available, the cost of groups.
connecting is decreasing, more devices are being created with Wi-Fi b) o termo their na sentença “all of the major world religions have
capabilities and sensors built into them, technology costs are going extremists that have taken up violence to further their perceived
down, and smartphone penetration is sky-rocketing. All of these religious goals” refere-se a “all of the major world religions”.
things are creating a “perfect storm” for the IoT. c) os vocábulos “currently”, “arbitrarily”, “redundantly” e
So what is the Internet of Things? “religiously”, presentes no texto, são advérbios transformados a
Simply put, this is the concept of basically connecting any device partir de adjetivos, pelo processo de formação por prefixação e
with an on and off switch to the Internet (and/or to each other). sufixação.
This includes everything from cell phones, coffee makers, washing d) a afirmação “Separatist groups are those with the goal of
machines, headphones, lamps, wearable devices and almost anything separation from existing entities” responde adequadamente à
else you can think of. pergunta: What do separatist groups do to achieve their goal?
So what now?
The new rule for the future is going to be, “Anything that can be 10. (UDESC 2014) Answer the question, according to text.
connected, will be connected.”
ARCHAEOLOGISTS USE DRONES TO STUDY PERU’S RUINS
(Disponível em: <http://www.forbes.com/sites/jacobmorgan/2014/05/13/simple-
explanation-internet-things-that-anyone-can-understand/#29a0a1cd6828>.
Acesso em: 12 set. 2016. (Adaptado).) To get a bird’s-eye view of ancient sites, archaeologists often turn
to planes, helicopters and even hot air balloons. But today researchers
Analisando-se os aspectos estruturais do texto, verifica-se que have access to more agile and less expensive technology to map,
a) a sentença “more devices are being created” encontra-se na voz explore and protect archaeological treasures: tiny airborne drones.
passiva. Na voz ativa seria “they create many more devices”. In Peru – the home of Machu Picchu and other amazing ruins –
b) a sentença “technology costs are going down,” na forma the government is planning to purchase several drones to 5quickly and
negativa, seria “technology costs be not going down”. cheaply conduct archaeological surveys in areas targeted for building
or development, according to Reuters.
c) na sentença “Let’s start with understanding a few things”, o
termo “Let´s” é composto pela contração dos vocábulos “Let” Archaeologists working in the country have already been using
e “is”. small flying robots to study 4ancient sites, including the colonial
Andean town Machu Llacta, and the San José de Moro burial grounds,
d) na sentença “Anything that can be connected”, o modal “can”
which contain the tombs of Moche priestesses. Some researchers have
apresenta a ideia de possibilidade de ocorrência.
even built their own drones for less than $ 2,000, Reuters reported.
e) na sentença “many people are still just trying”, o termo “many”
“It’s like having a scalpel instead of a club,” Jeffrey Quilter, an
pode ser substituído pelo vocábulo “much”, sem alteração de
archaeologist at Harvard University, told the news agency. “You
sentido.

PROMILITARES.COM.BR 285
MODAL VERBS

can control it to a very fine degree. You can go up 3 meters and 06. (CFOE) “Peter, you must do your homework now.” The underlined
photograph a room, 300 meters and photograph a 3site, or you can verb means the same as:
go up 3,000 meters and photograph the entire valley.” a) have to
Cheap and effective drones could be a boon for Peru’s culture
6
b) may
ministry, which has a modest budget and is tasked with protecting
more than 13,000 archaeological sites that are threatened by looters, c) should
squatters and illegal mining, according to Reuters. d) can
Elsewhere robots have 1enabled archaeological discovery. A
remote-controlled robot the size of a 2lawn mower recently found 07. (FMU/FIAM-SP) You ______ finish your food, if you don’t want to.
burial chambers inside the Temple of the Feathered Serpent, an a) must c) needn’t e) mustn’t
ancient pyramid in Mexico. And in Russia, researchers used a miniature b) don’t need d) may not
airborne drone to capture images that could be used to create a 3-D
model of an ancient burial mound.
08. (FMU-SP) ________ I smoke here? Yes, but you _____ throw ashes
(www.foxnews.com/tech/2013. Accessed on: 26/08/2013.)
on the floor.
Some of the English grammar points which are present in ref. 6 are: a) May – should
a) adverb, present simple, present perfect. b) Might – must not
b) adjective, modal, relative pronoun. c) Can – mustn’t
c) adjectives, passive voice, simple past. d) Could – ought to
d) future perfect, possessive case, simple present. e) Would – shouldn’t
e) present perfect, modals, simple past.
09. (EN)
– (…) Where did you get this recipe for roast chicken, my dear? Quite
delicious.
EXERCÍCIOS DE

COMBATE
– It’s the same thing we have every day, Daddy, – replied Annabel.
– Is it really? It tastes quite different this week. Do you have a good
cook at Hartlepool Hall, Edward? Is Mrs. Horton still there? But she
______ be. She ______ be dead by now.
01. (UFSC) Assinale a alternativa correta. a) might not / can’t
a) You can’t drive a car unless you have a driver’s license. b) can’t / will
b) You can’t to drive a car unless you have a driver’s license. c) can’t / must
c) You can’t driving a car unless you have a driver’s license. d) shouldn’t / mustn’t
d) You don’t can drive a car unless you have a driver’s license. e) shouldn’t / can’t
e) You cannot to drive a car unless you have a driver’s license.
10. (EN) Ocean Acidification: It is a direct impact of excessive
production of CO2. 25% of CO2 is produced by humans. The ocean
02. (FCM/SANTA CASA-SP) She’s so good that there isn’t anything acidity has increased by the last 250 years, but by 2100, it _______
she ______do. shoot up by 150%.
a) can’t c) may e) isn’t able Acid Rain: Acid rain occurs due to the presence of certain pollutants in the
b) can d) has to atmosphere. Acid rain is a known environmental problem that _______
have serious effect on human health, wildlife and aquatic species.
03. (EPCAR) “Cyberbullying is getting extremely popular because a) may / can d) may / would
teens can stay anonymous”. The underlined verb expresses
b) would / can e) could / would
a) possibility. c) prohibition.
c) would / may
b) obligation. d) permission.

04. (PUC-CAMPINAS) Mother to child: “You _________ tell lies.”


GABARITO
a) don’t need
b) mustn’t EXERCÍCIOS DE FIXAÇÃO
c) may not 01. A 04. E 07. A 10. A
d) don’t must 02. B 05. D 08. B
e) haven’t 03. B 06. E 09. E
EXERCÍCIOS DE TREINAMENTO
05. (EEAR) Select the correct modal verb that fills in the blank in the 01. D 04. E 07. D 10. B
paragraph below. 02. C 05. C 08. D
“Paul feels very sick and dizzy. I think he has been drinking all night 03. C 06. D 09. A
again. He ____ get out of bed this morning.”
EXERCÍCIOS DE COMBATE
a) will
01. A 04. B 07. E 10. A
b) must
02. A 05. C 08. C
c) can’t
03. A 06. A 09. C
d) could

286 PROMILITARES.COM.BR
EITHER/NEITHER -
SO/TOO AND NUMERALS

EITHER/NEITHER - SO/TOO
Utilizamos a estrutura SO + AUXILIAR + SUJEITO para concordar com orações afirmativas. Já para concordância com orações negativas temos
a estrutura NEITHER + AUXILIAR + SUJEITO.
Exemplos:
I hate mushrooms → So do I
I don’t live in London → Neither do I
We live in London and so do they.
My parents don’t come here often. Neither does Alex.
She isn’t French and neither is he.
You don’t like cold weather. Neither do we.

Na tabela abaixo temos as concordâncias nos mais variados tempos verbais.

Present simple: use ‘do / does’ Lucy likes coffee. So do I. Lucy doesn’t like coffee. Neither does he.

Present simple with ‘be’: use ‘am / is / are’ John’s at the office. So is she. John isn’t at the office. Neither am I.

Present continuous: use ‘am / is / are’ Luke’s going out tonight. So am I. Luke isn’t going out tonight. Neither am I.

Past Simple: use ‘did’ Jill went to the cinema yesterday. So did I. Jill didn’t go to the cinema yesterday. Neither did I.

Past simple with ‘be’: use ‘was / were’ She was at the library. So was I. She wasn’t at the library. Neither were they.

Present perfect: use ‘have / has’ They’ve been to Colombia. So have I. They haven’t been to Colombia. Neither have I.

Future simple: use ‘will’ Edward will be at the cafe later. So will I. Edward won’t be at the cafe later. Neither will I.

He would like a cup of tea. So would I. He wouldn’t like a cup of tea. Neither would I.
Modal verbs: repeat the modal verb
Emma can speak Russian. So can I. Emma can’t speak Russian. Neither can I.

https://www.perfect-english-grammar.com/so-too-neither-either.html

Outras estruturas que podemos utilizar para concordância são as que possuem TOO e EITHER, onde teremos SUJEITO + AUXILIAR + TOO para
as orações afirmativas e SUJEITO + AUXILIAR NA NEGATIVA + EITHER para orações negativas.
Exemplos:
He hates mushrooms → She does too
I don’t live in London → I don’t either (= I also don’t live in London).

Abaixo exemplos em mais alguns tempos verbais:


Present simple: John’s at the office → I am too.
Present continuous: Luke isn’t going out tonight → I’m not either.
Present perfect: They’ve been to Colombia → I have too.
Modal verbs: Emma can’t speak Russian → I can’t either.

Por fim podemos ter a estrutura ‘me too’ e ‘me neither’. ‘Me too’ com mesmo sentido da estrutura SO + AUXILIAR + SUJEITO e ‘me neither’
com o mesmo sentido de NEITHER + AUXILIAR + SUJEITO. Tanto ‘me too’ quanto ‘me neither’ são informais:
I hate mushrooms → Me too
I don’t live in London → Me neither

PROMILITARES.COM.BR 55
EITHER/NEITHER - SO/TOO AND NUMERALS

NUMERAIS (ORDINAIS E CARDINAIS)


Na parte dos numerais é importante entender a diferença entre os cardinais e os ordinais. Cardinais são números mais comuns do dia a dia
(1, 2, 3...), já os ordinais são aqueles utilizados principalmente para indicar colocação (1º, 2º, 3º...). Ao olhar a tabela abaixo é importante
compreender que as letras que aparecem ao final do numeral ordinal escrito em forma de numeral são as duas últimas letras do mesmo número
escrito por extenso (first [primeiro] → 1st).

NUMBERS CARDINALS ORDINALS


0 Zero ------
1 One First 1st
2 Two Second 2nd
3 Three Third 3rd
4 Four Fourth 4th
5 Five Fifth 5th
6 Six Sixth 6th
7 Seven Seventh 7th
8 Eight Eighth 8th
9 Nine Ninth 9th
10 Ten Tenth 10th
11 Eleven Eleventh 11th
12 Twelve Twelfth 12th
13 Thirteen Thirteenth 13th
14 Fourteen Fourteenth 14th
15 Fifteen Fifteenth 15th
16 Sixteen Sixteenth 16th
17 Seventeen Seventeenth 17th
18 Eighteen Eighteenth 18th
19 Nineteen Nineteenth 19th
20 Twenty Twentieth 20th
21 Twenty-one Twenty-first 21st
22 Twenty-two Twenty-second 22nd
23 Twenty-three Twenty-third 23rd
24 Twenty-four Twenty-fourth 24th
... ... ...
29 Twenty-nine Twenty-ninth 29th
30 Thirty Thirtieth 30th
31 Thirty-one Thirty-first 31st
… … …
40 Forty Fortieth 40th
50 Fifty Fiftieth 50th
60 Sixty Sixtieth 60th
70 Seventy Seventieth 70th
80 Eighty Eightieth 80th
90 Ninety Ninetieth 90th
100 One hundred One hundredth 100th
101 One hundred and one One hundred and first 101st
… … …
1000 One thousand One thousandth
1001 One thousand and one One thousand and first
… … …
1540 One thousand five hundred and forty One thousand five hundred and fortieth

... ... …
1 000 000 One million One millionth

56 PROMILITARES.COM.BR
EITHER/NEITHER - SO/TOO AND NUMERALS

FALSOS COGNATOS
Falsos cognatos são termos de origem estrangeira que se parecem com palavras da nossa língua, porém possuem significado diferente. Alguns
desses podem ser vistos na tabela abaixo.

INGLÊS SIGNIFICADO EM PORTUGUÊS PORTUGUÊS SIGNIFICADO EM INGLÊS

actually na verdade atualmente nowadays

adept especialista adepto fan

anthem hino antena antenna

alias pseudônimo; apelido aliás indeed; in fact

to amass acumular amassar dent; knead (massa)

appointment compromisso com hora marcada apontamento note

apology pedido de desculpas apologia eulogy

argument discussão argumento reasoning

to assist ajudar assistir (reunião) to attend, (TV, cinema) to watch

to attend assistir, participar atender to answer

audience plateia audiência court appearance

balcony sacada balcão counter

braces aparelho (dentes) braços arms

cigar charuto cigarro cigarette

collar colarinho, coleira, gola colar necklace (adereço)

college faculdade colégio school

comprehensive abrangente compreensivo understanding

confident confiante confidente confidant

convict condenado convicto convinced

costume fantasia costume habit

data dados data date

deception ato de enganar; fraude decepção disappointment

to dent amassar dente tooth

to devolve transferir; delegar devolver to return; to give back

diversion desvio diversão fun

to enroll alistar, inscrever enrolar to roll

eventually finalmente; por fim eventualmente possibly

exit saída êxito success

exiting empolgante excitante (contexto sexual) sexy

exquisite sofisticado esquisito strange

fabric tecido fábrica factory

to grip agarrar firmemente gripe flu

idiom expressão idiomática idioma language

ingenuity criatividade ingenuidade naivety

inhabitable habitável inabitável uninhabitable

PROMILITARES.COM.BR 57
EITHER/NEITHER - SO/TOO AND NUMERALS

INGLÊS SIGNIFICADO EM PORTUGUÊS PORTUGUÊS SIGNIFICADO EM INGLÊS

injury ferimento injúria insult

to intend pretender entender to understand

jar pote jarra jug

journal revista de especialidade jornal newspaper (papel)

lace renda laço ribbon

lecture palestra leitura reading

legend lenda legenda subtitles

library biblioteca livraria bookstore

location localização locação lease

lunch almoço lanche snack

magazine revista magazine department store

mayor prefeito maior bigger

motel hotel de beira de estrada motel love motel

to notice notar notícia news

novel romance (literário) novela (TV) soap opera

office escritório oficial official

parents pais (pai e mãe) parentes relatives

pasta massa (macarrão, lasanha) pasta folder

physician médico físico physicist

policy política; diretriz polícia police

prejudice preconceito prejuízo damage, loss

preservative conservante (alimento) preservativo condom

to pull puxar pular to jump

to push empurrar puxar to pull

realize notar, reparar, perceber realizar to carry out, to put into practice

requirement requisito requerimento request; application

resume retomar resumo summary

retired aposentado retirado removed

retribution punição; retaliação retribuição remuneration (salário); reward (prêmio)

scholar intelectual; erudito escolar school (adjetivo)

sensible sensato sensível sensitive

silicon silício silicone silicone

https://www.todamateria.com.br/falsos-cognatos-no-ingles-false-friends/

58 PROMILITARES.COM.BR
EITHER/NEITHER - SO/TOO AND NUMERALS

Também existem os cognatos, que ao contrário do que foi visto TEXT COMPREHENSION
acima, querem dizer exatamente o que parecem. Text I:
INGLÊS PORTUGUÊS YOUR FACIAL BONE STRUCTURE HAS A BIG INFLUENCE
ON HOW PEOPLE SEE YOU
accident acidente
(…) Selfies, headshots, mug shots – 1photos of oneself convey more
comic cômico these days than snapshots ever did back in the Kodak era. 2Most digitally
creation criação minded people continually post and update pictures of themselves at
professional, social media and dating sites such as LinkedIn, Facebook,
different diferente Match.com and Tinder. For better or worse, viewers then tend to make
economy economia snap judgments about someone’s personality or character from a single
shot. As such, 3it can be a stressful task to select the photo that conveys
example exemplo the best impression of ourselves. 4For those of us seeking to appear
future futuro friendly and trustworthy to others, a new study underscores an old,
chipper piece of advice: Put on a happy face.
garage garagem
A newly published series of experiments by cognitive
important importante neuroscientists at New York University is reinforcing the relevance
minute minuto of facial expressions to perceptions of characteristics such as
trustworthiness and friendliness. 5More importantly, the research also
offensive ofensivo revealed the unexpected finding that perceptions of abilities such as
positive positivo physical strength are not dependent on facial expressions but rather
on facial bone structure.
television televisão
The team’s first experiment featured photographs of 10 different
https://www.todamateria.com.br/falsos-cognatos-no-ingles-false-friends/ people presenting five different facial expressions each. Study subjects
rated how friendly, trustworthy or strong the person in each photo
Por fim temos os casos de ambiguidade lexical, onde uma palavra appeared. A separate group of subjects scored each face on an
em inglês pode agir como cognato em um contexto, e como falso emotional scale from “very angry” to “very happy.” And three experts
cognato em outro. Observe a tabela: not involved in either of the previous two ratings to avoid confounding
results calculated the facial width-to-height ratio for each face. 6An
TERMO EM INGLÊS SIGNIFICADO 1 SIGNIFICADO 2 analysis revealed that participants generally ranked people with a
happy expression as friendly and trustworthy but not those with angry
Abstract Abstrato Resumo
expressions. 7Surprisingly, participants did not rank faces as indicative
Affluent Afluente Rico of physical strength based on facial expression but graded faces that
were very broad as that of a strong individual.
Apology Apologia Desculpa In a second survey facial expression and facial structure were
manipulated in computer-generated faces. Participants rated each
Apply Aplicar Inscrever-se face for the same traits as in the first survey, with the addition of a
rating for warmth. Again, people thought a happy expression, but
Argument Argumento Discussão
not an angry one, indicated friendliness, trustworthiness — and in
Arm Armar (dar armas) Braço this case, warmth. The researchers then showed two additional sets
of participants the same faces, this time either with areas relevant
Balance Balança Equilíbrio to facial expressions obscured or the width cropped. In the first
variation, for faces lacking emotional cues, people could no longer
Ball Bola Baile perceive personality traits but could still perceive strength based on
width. Similarly, for those faces lacking structural cues, people could
Capital Capital maiúscula no longer perceive strength but could still perceive personality traits
based on facial expressions.
Character Caráter Personagem
In a third iteration of the survey participants had to pick four faces
Cool Fresco (clima) Legal out of a lineup of eight faces varied for expression and width that
they might select either as their financial advisor or as the winner of a
Date Data Encontro power-lifting competition. As might be expected, 8participants picked
faces with happier expressions as financial advisors and 9selected
Interest Interesse Juros broader faces as belonging to power-lifting champs.
Mark Marcar Nota In a final survey the researchers generated more than 100
variations of one individual “base face” by varying facial features.
Medicine Medicina Remédio Participants saw two faces at a time, and then picked one as either
trustworthy or high in ability or as a good financial advisor or power-
Park Parque Estacionar lifting winner. Using these results, a computer then created an average
face for each of these four categories, which were shown to a separate
Race Raça Corrida
set of participants who had to pick which face appeared either more
Record Recorde Gravar trustworthy or stronger. Most of the participants found the computer-
generated averages to be good representations of trustworthiness or
Save Salvar Economizar strength – 10and generally saw the average “financial advisor” face as
more trustworthy and the “powerlifter” face as stronger. The findings
Turkey Turquia Peru from all four surveys were published in the Personality and Social
Psychology Bulletin on June 18.
http://www.pcs.uem.br/prointe/cidade-gaucha/inscricoes-oficina-de-ingles- Adaptado de www.scientific.american.com/article/your-facial-bone-strecture-has-a-
instrumental-1/material-das-oficinas/24-09-16/Tabela%20Falsos%20cognatos.pdf/view big-influence-on-how-people-see-you.(acesso em 20/8/2015)

PROMILITARES.COM.BR 59
EITHER/NEITHER - SO/TOO AND NUMERALS

01. (ITA) De acordo com o terceiro estudo, Robbee changed lanes and pulled a little closer. From the side,
a) rostos mais largos sinalizam pessoas mais felizes. she could see that the man had a slight smile on his face, the kind of
16absentminded smile a person might have when he’s all alone, happy
b) rostos mais finos indicam pessoas mais competentes. in his own thoughts. 12Robbee found herself thinking: “Wow, this is
c) rostos mais compridos indicam pessoas mais afetuosas. the epitome of a person appreciating this day and this moment.”
d) rostos mais finos sinalizam pessoas mais confiáveis. The convertible 18eventually turned the corner, and that’s when
e) rostos mais largos indicam pessoas mais fortes fisicamente. Robbee got a look at 9the man’s full face. “Oh my God,” she said to
herself. “It’s Randy Pausch!”
02. (ITA) De acordo com o texto, 2
She was so struck by the sight of me. She knew that my cancer
a) são relatados os resultados de quatro pesquisas realizadas por diagnosis was grim. And yet, 14as she wrote in her email, 3she was
neurocientistas ligados a empresas de recursos humanos. moved by how contented I seemed. In this private moment, 4I was
19
obviously in high spirits. Robbee wrote in her email: “You can never
b) todos os estudos utilizaram o mesmo método para analisar as know how much that glimpse of you made my day, reminding me of
fotos, mas os resultados são distintos. what life is all about.”
c) as pesquisas foram encomendadas por gerenciadores de redes I read 10Robbee’s email several times. I came to look at it 15
as a
sociais como o Facebook e o LinkedIn. feedback loop of sorts.
d) as pesquisas mostram que as pessoas avaliam a confiabilidade It has not always been easy to stay positive through my cancer
observando as expressões faciais do indivíduo. treatment. When you have a dire medical issue, it’s tough to know
e) os quatro estudos apresentam resultados totalmente distintos no how you’re 20really faring emotionally. I had wondered whether a part
que se refere à afetuosidade. of me was acting when I was with other people. Maybe at times I
forced myself to appear strong and upbeat. Many cancer patients feel
03. (ITA) Considere as sentenças a seguir: obliged to put up a brave front. Was I doing that, too?
I. O primeiro estudo foi realizado com um grupo de 10 participantes But Robbee had come upon me in an unguarded moment. I’d
e 3 avaliadores. like to think she saw me as I am. 5She 21certainly saw me as I was that
evening.
II. O segundo estudo ampliou o primeiro, incluindo a avaliação sobre
afetuosidade. Her mail was just a paragraph, but it meant a great deal to me.
6
She had given me a window into myself. I was still fully 17engaged. I
III. O terceiro estudo calculou a força física pela razão entre altura e
still knew life was good. I was doing OK.
largura da face.
Fonte: PAUSCH, R. The last lecture. New York, Hyperion, 2008. p.64-65.
IV. O quarto estudo utilizou as mesmas imagens do primeiro estudo.
Está(ao) correta(as)
05. (ITA) De acordo com as informações no texto, Robbee Kosak
a) apenas I e IV.
a) descreveu detalhadamente o cenário do seu primeiro encontro
b) apenas a II.
com Randy Pausch.
c) apenas II e III.
b) sentiu-se atraída pelo veículo de Randy Pausch devido à alta
d) apenas II e IV. velocidade dele.
e) apenas I e IV. c) escreveu palavras motivadoras a Randy Pausch porque desejava
reanimá-lo.
04. (ITA) De acordo com o texto, d) caracterizou o motorista do veículo como uma pessoa satisfeita e
a) fotos postadas em redes sociais determinam as habilidades e de bem com a vida.
competências de um candidato a emprego. e) ocupava o cargo de Vice-Presidente na empresa presidida por
b) fotos digitais postadas nas redes sociais causam as mesmas Randy Pausch.
impressões sobre um indivíduo que fotos analógicas.
c) a Universidade de Nova York pretende publicar as pesquisas 06. (ITA) O autor do texto
relatadas na reportagem para divulgar características de a) utiliza a dissertação e a descrição como tipologia textual
competência e confiabilidade. predominante.
d) além de credibilidade e competência profissional, a análise das b) é narrador observador e mescla discursos direto, indireto e
faces revelou dados sobre força física e condição socioeconômica. indireto livre.
e) a percepção da força física de um indivíduo está relacionada à c) usa foco narrativo em primeira pessoa configurando o texto
estrutura óssea da face e não à expressão facial do indivíduo. autobiográfico.
d) utiliza linguagem coloquial nos diálogos para externar seus
Text II: sentimentos.
THE MAN IN THE CONVERTIBLE e) recorre a figuras de pensamento para compor o gênero dramático.
One morning, well after I was diagnosed with cancer, I got an
email from Robbee Kosak, 7Carnegie Mellon’s vice president for 07. (ITA) Assinale a oração que não contém expressão ou termo
advancement. She told me a story. qualificador.
She said she had been driving home from work the night before,
22 a) It was a warm, gorgeous, early-spring evening… (ref. 1)
and 11she found herself behind a man in a convertible. 1It was a warm, b) She was so struck by the sight of me. (ref. 2)
gorgeous, early-spring evening, and the man had his top down and
c) … she was moved by how contented I seemed. (ref. 3)
all his windows lowered. His arm was hanging over 8the driver’s side
door, and his fingers were tapping along to the music on his radio. d) … I was obviously in high spirits. (ref. 4)
His head was bobbing along, too, 13as the wind blew through his hair. e) She certainly saw me as I was that evening. (ref. 5)

60 PROMILITARES.COM.BR
EITHER/NEITHER - SO/TOO AND NUMERALS

Text III: 09. (ITA) O autor do texto


STICKERNOMICS I. atribui ao roubo de milhares de figurinhas no Brasil a dificuldade
Football albums para compra e troca entre colecionadores.
Got, got, got, got, got, need II. deprecia as estratégias do Grupo Panini para comercializar álbuns
de figurinhas da Copa do Mundo.
THE World Cup is still two weeks away, but for children worldwide
(plus 6disturbing numbers of adults) the race to complete the Brazil III. descreve o mercado de figurinhas da Copa do Mundo e apresenta
2014 sticker book started long ago. 1Panini, an Italian firm, has aos colecionadores possibilidades de obtenção de figurinhas.
produced sticker albums for World Cups since Mexico 1970; this year’s Está(ão) correta(s)
version has 640 stickers to collect. 7Collecting them is no idle pursuit, a) apenas a I.
however. Getting every slot filled delivers an early lesson in probability,
the value of statistical tests and the importance of liquidity. b) apenas a II.

When you start an album, your first sticker (in Britain, they come
8 c) apenas a III.
in packs of five) has a 640/640 probability of being needed. 2As the d) apenas I e II.
spaces get filled, the odds of opening a pack and finding a sticker e) apenas I e III.
you want fall. 9According to Sylvain Sardy and Yvan Velenik, two
mathematicians at the University of Geneva, the number of sticker 10. (ITA) De acordo com o texto, Sardy e Velenik
packs that you would have to buy on average to fill the album by
mechanically buying pack after pack would be 899. 11That assumes a) insistem na formação de grupos de 10 colecionadores para
there is no supply shock to the market (the theft of hundreds of facilitar o preenchimento total de álbuns de figurinhas.
thousands of stickers in Brazil in April 12left many fearful that Panini b) fiscalizam a compatibilidade entre a produção de figurinhas e sua
would run short of cards). comercialização desde 2010.
It also assumes that 10the market is not being rigged. Panini c) verificaram na Suíça a repetição de aproximadamente 9 vezes
says that 3each sticker is printed in the same volumes and randomly cada figurinha em um lote de 6.000 figurinhas.
distributed. In a 2010 paper Messrs Sardy and Velenik gamely played d) são matemáticos pesquisadores da empresa Panini, responsáveis
the role of “regulator” by checking the distribution of stickers for a pela distribuição das figurinhas.
660-sticker album 13sold in Switzerland for that year’s World Cup. Out
e) consideram que as práticas de obtenção de figurinhas da Copa do
of their sample of 6,000 stickers, they expected to see each sticker
Mundo são injustas e manipuláveis.
9.09 times on average (6,000/660), 4which was broadly borne out in
practice.
Even in a fair market, it is inefficient to buy endless packs as an EXERCÍCIOS DE

FIXAÇÃO
individual (not to mention bloody expensive for the parents). The
answer is to create a market for collectors to swap their unwanted
stickers. The playground is 14one version of this market, 5where a child
who has a card prized by many suddenly understands the power of
limited supply. Sticker fairs are another. As with any market, liquidity
counts. The more people who can be attracted into the market with 01. (EEAR) The ordinal form for the number ninety is
their duplicate cards, the better the chances of finding the sticker you a) ninth
want. b) nineth
Messrs Sardy and Velenik reckon that a group of ten astute c) ninetieth
sticker-swappers would need a mere 1,435 packs between them to
complete all ten albums, if they take advantage of Panini’s practice of d) nineteenth
selling the final 50 missing stickers to order. Internet forums, where
potentially unlimited numbers of people can swap stickers, make this 02. (EEAR) The numbers “70 and 100” are expressed in English as
number fall even further. The idea of a totally efficient market 15should a) seventy and a hundred
dismay Panini, which will sell fewer packs as a result. But as in all
b) seventy and a thousand
markets, behaviour is not strictly rational. 16Despite entreaties, your
correspondent’s son is prepared to tear out most of his stickers to get c) seventeen and a hundred
hold of Lionel Messi. d) seventeen and a thousand
Fonte: http://www.economist.com/news/finance-and-economics/
21603019-got-got-got-got-got-need-stickernomics Acesso: 13/ago/2014 03. (EEAR) “A hundred thousand” is expressed in number as:
a) 100,000
08. (ITA) De acordo com o texto, b) 1,000,000
a) a empresa Panini comercializa álbuns de figurinhas da Copa do c) 10,000,000
Mundo há 30 anos. d) 100,000,000
b) é impossível completar o álbum sem que os colecionadores
recorram a feiras e redes sociais. 04. (G1) Assinale a alternativa que preenche corretamente a lacuna.
c) são necessárias 1.500 figurinhas para completar um álbum. _____________ is the cause of her success.
d) a empresa Panini disponibiliza a venda das 50 figurinhas faltantes a) Ignorant
aos colecionadores.
b) Intelligence
e) o processo de confecção e distribuição das figurinhas é feito
aleatoriamente pela Panini. c) Intelligent
d) Ignorantly

PROMILITARES.COM.BR 61
EITHER/NEITHER - SO/TOO AND NUMERALS

05. (G1) Assinale a alternativa que preenche corretamente as lacunas. 02. (EN) Choose the best reply to this statement.
She’s ____________ sorry she can’t talk to you now. She’s going to Mary: I have all the books the teacher told us about last class.
the ___________ to buy a book. You:___________ (1) .
a) terribly / bookstore a) Neither have I
b) terribly / library b) Nor have I
c) terrible / library c) So had I
d) terrible / bookstore d) So do I
e) So did I.
06. (G1) Assinale a alternativa que preenche corretamente a lacuna.
That is a lovely gold ___________ Helen is wearing. 03. (EEAR) Choose the best alternative for the written form of “1844”.
a) color a) eight fourty – four
b) collar b) one eight four four
c) bar c) eighteen forty-four
d) necklace d) eighteen fourth-four

07. (UERJ) Simplifying the culture merely for financial gain may 04. (EPCAR) “Websterts dictionary is now in its 11th edition.” The full
actually cost Hawaiians more than they think form of the underlined item is
The underlined word is used to express the notion of: a) eleven.
a) time b) eleventy.
b) doubt c) eleventh
c) manner d) elevent.
d) certainty
05. (EEAR) The Phoenix airport has introduced new technology that
08. (EEAR) The correct way of writing the cardinal number 30,000 can see through a person’s clothes. The new machine costs $100,000
using words is __________. to make and is designed to find out if a passenger is carrying a weapon
a) third thousand or explosives. Critics of the new X-Ray scanner, however, say it takes
away a person’s privacy.
b) thirty thousand
(adapted from www.inglesonline.com.br)
c) thirteen thousand
d) thirtieth thousand The number 100,000, in bold in the text, is expressed in words as
__________.
09. (UFF) In “I can not believe that someone actually funded this a) a million
project” (Bonnie, USA), ACTUALLY means: b) one hundred
a) as it happens c) one thousand
b) nowadays d) a hundred thousand
c) theoretically
d) really 06. (EN) Choose the best reply to this statement.
e) by the way Carol: I got caught up in the traffic.
You: ______________.
10. (EEAR) The numbers 12th and 25th, underlined in the text, are,
respectively: a) Neither did I.
a) twelve / twenty-fifth b) Nor did I.
b) twelfth / twenty-fifth c) So got I.
c) twelve / twentieth-fifth d) So did I.
d) twelfth / twentieth-five e) So do I.

07. (UFF) “False friends” are words in a foreign language which are
EXERCÍCIOS DE similar in form to words in our mother tongue, but which do not have

TREINAMENTO
the same meaning. Which pair of words below (taken from the text)
contains ONLY false friends?
a) Characteristics / cycle
b) Divide / cycle
01. (EEAR) Choose the best alternative for the written form of 137th. c) Laboratory / actually
a) hundredth thirtieth seventh. d) Realize / actually
b) one hundred thirty seven. e) Laboratory / characteristics
c) one hundred thirty seventh.
d) a hundred thirteen seventy.

62 PROMILITARES.COM.BR
EITHER/NEITHER - SO/TOO AND NUMERALS

08. (IBMEC-RJ) False cognates are usual in the English language. 03. (MACKENZIE) The meaning of EXQUISITELY in the sentence “Their
Which of the sentences extracted from the text contain a false cognate children were exquisitely dressed.“ is:
(false friend)? a) strangely or unexpected.
a) “Graddol anticipates a world where the share of people who are b) extremely beautiful in appearance.
native English speakers slips from 9 percent in the mid-twentieth
century to 5 percent in 2050.” c) oddly with an unusual appearance.

b) “... as the proportion of native speakers declines.” d) peculiarly with a special or unique quality.

c) “because varieties of Arabic spoken in say, Egypt and Morocco are e) very unattractive and unpleasant to look at.
mutually incomprehensible.”
04. (MACKENZIE) The meaning of CONSISTENT in the sentence “Mr.
d) “... the Internet it was dominated by sites in English”
Brown’s behavior is not consistent with his ideas.” is:
e) “Graddol does have words of consolation for those...”
a) coherent.

09. (UPE-SSA) Na frase: “The officials worried that he was drowning.”, b) contradictory.
o termo sublinhado está sendo empregado como substantivo, é um c) right.
falso cognato. Assim, o significado de ‘officials’, nesse contexto, d) persuasive.
corresponde a
e) strong.
a) ordenanças.
b) autoridades. 05. (EN) Which is the correct alternative to complete the dialogue?
c) oficinas. Peter: I don’t think I’ve got to study for the test.
d) oficiais. Jane: (1) ____ . This subject is really easy.
e) treinadores. a) So do I
b) Neither do I
10. (PM-PE) Assinale a alternativa cujas palavras destacadas são, de
c) So have I
acordo com o contexto, FALSAS COGNATAS.
d) Neither have I
I. He said, “you realize that there simply is no other option for you
out there.”. e) Neither am I
II. Younger people are less interested in farming, and look for
economic opportunities in other, more appealing industries. 06. (EN) Choose the correct option to complete the dialogue.
III. The trend is extreme in sub-Saharan Africa, where 60 percent of Megan: I’d like to live abroad. Julie: So ______ I. Which country
the total population is under the age of 25. And you can find do you have in mind? Megan: Probably Australia, because it’s hot. I
similar figures in other regions. wouldn’t like to live in a cold country. Julie: I wouldn’t ________. I hate
feeling cold.
IV. In his presentation, Mabaya warned against overgeneralizing
about the entire African continent. a) do/too.
V. Nevertheless, Mabaya emphasized, there is a lot to be optimistic b) do/neither.
about. c) would/too.
Estão CORRETAS apenas d) would/neither.
a) I e II. e) would/either.
b) III e V.
07. Which of the alternatives below correctly completes the sentence?
c) I, III e V.
She will never talk to him again, and (1)____ I.
d) I e III.
a) neither do
e) IV e V.
b) either do
c) neither will
EXERCÍCIOS DE d) either won’t

COMBATE
e) either will

08. (EN) Which of the options completes the excerpt below correctly?
You’re dehydrated - and ______ your skin
01. Choose the best reply to this statement. Most of us tend to think of dehydration as a short term problem
Dayse: I haven’t seen The King’s Speech yet. You: (1). solved by a glass of water, but board-certified dermatologist Dr. Janet
a) Neither have I. c) So have I. e) So did I. Prystowsky encourages viewing skin dehydration as a long-term
problem, as consistently failing to get your skin the water it needs can
b) Nor do I. d) So do I. have lasting results.
(Abridged from https ://w w w ,goodhousekeeping.com /beauty/
02. (EEAR) The correspondent ordinal forms for the numbers 60, 30 anti-aging/a 36993 /dull-skin-causes/)
and 90 are, respectively:
a) sixth / third / ninth a) so is
b) sixty / thirty / ninety b) so are
c) sixtieth / thirtieth / ninetieth c) nor is
d) sixteenth / thirteenth / nineteenth d) neither is
e) neither are

PROMILITARES.COM.BR 63
EITHER/NEITHER - SO/TOO AND NUMERALS

09. (EN) Which is the correct alternative to complete the dialogue?


John: I’ve never gone horseback riding.
Sue: ______ I’d be afraid to try.
a) I wouldn’t either
b) Neither did I
c) So have I
d) I haven’t either
e) Neither would I

10. (EN) Choose the best reply to the statement below.


I went to school yesterday.
a) So I did.
b) So did I .
c) Nor I did.
d) Neither went I.
e) Neither did I.

GABARITO
EXERCÍCIOS DE FIXAÇÃO
01. C 04. B 07. D 10. B
02. A 05. A 08. B
03. A 06. D 09. D
EXERCÍCIOS DE TREINAMENTO
01. C 04. C 07. D 10. D
02. D 05. D 08. A
03. C 06. D 09. B
EXERCÍCIOS DE COMBATE
01. A 04. A 07. C 10. B
02. C 05. B 08. A
03. B 06. E 09. D

ANOTAÇÕES

64 PROMILITARES.COM.BR
PASSIVE VOICE

A voz passiva é muito usada na língua inglesa, principalmente na Exemplos:


linguagem formal. Na estrutura, temos o verbo to be como auxiliar e Active: They have made a number of attempts to discover the
o verbo principal precisa estar no passado particípio. O tempo verbal mayas’ code.
da voz ativa é definido pelo verbo to be; neste caso, ele sempre
estará no mesmo tempo verbal que o verbo principal na oração ativa. Passive: A number of attempts have been made to discover the
Introduzimos o agente da passiva após a preposição by. mayas’ code.

Exemplos: Active: John gave Mary a present.


That young boy wears beautiful clothes. (VOZ ATIVA) Passive: Mary was given a present (by John).
Active: Someone built this house in 1850.
Passive: This house was built in 1850.
sujeito verbo objeto Active: The writers have written a lot of books about this subject.
Passive: A lot of books have been written about this subject.
Beautiful clothes are worn by that young boy. (VOZ PASSIVA) Active: We could have conserved more wheat last year.
Passive: More wheat could have been conserved last year.
Paciente da
ação verbal
Casos Especiais:
Da voz ativa para a passiva: principais transformações A voz passiva geralmente é usada quando o sujeito da oração é
VOZ ATIVA VOZ PASSIVA indeterminado e faz o papel de gerador da ação.
Exemplos:
AGENTE DA Material from the Earth’s interior is continually being brought
SUJEITO
PASSIVA to the surface.
TRANSFORMA-SE The ocean floors is recycled into the Earth’s interior, and is
VERBO V.AUX. + PP
EM replaced in less than 200 million years.
SUJEITO
OBJETO
(PACIENTE) Alguns anúncios são usados na voz passiva.
Principais mudanças nas formas verbais: Exemplos:
ENGLISH SPOKEN – KEYS CUT WHILE YOU WAIT – NO CHANGE
VOZ ATIVA VOZ PASSIVA GIVEN – CHILDREN UNDER 14 NOT ADMITTED – SECOND-HAND
BOOKS BOUGHT AND SOLD – CHEQUES NOT ACCEPTED.
moves is moved

moved was moved Expressões do tipo: They/ People + say/believe or Everybody +


says/ knows…etc., or are, frequentemente são usadas na voz passiva.
has moved has been moved
Exemplos:
had moved had been moved Active: Everybody says he is the best student in class.
Passive: It’s said he’s the best in class / He’s said to be the best in
will move will be moved class.
will have moved will have been moved Passive: You are supposed to be in class.
Active: Everyone supposes you are in class.
is moving is being moved
Observação
was moving was being moved
Duas formas diferentes de se dizer a mesma coisa na voz passiva.
to move to be moved Exemplos:
It is believed that the bride is wearing a white dress. / The bride is
to have moved to have been moved
believed to be wearing a white dress.
moving being moved
Voz passiva com os verbos: consider, report, expect, know,
could move could be moved
say, assume, feel, find, presume, repute, understand.
could have moved could have been moved

PROMILITARES.COM.BR 287
PASSIVE VOICE

Exemplos: family was academic; my father was an aerodynamic engineer and


I am considered to be an expert on crickets. my mother a mathematician, _____3_____ my sister studied geology.
The Millers are reported to be making a tour of the USA. At the age of 16, I attended a Women in Science and Engineering
careers week with school, just to have a look at what was available.
The report is expected to be published in May. This helped me decide that _____4_____ I really wanted to do was an
A camel is said to be able to go without water for 28 days. engineering degree, so I chose to do a BEng in materials science and
John Lennon is known to have been interested in oriental religions. engineering at Liverpool University, and then went on to do a PhD.
Da Vinci is said to have designed the first submarine. My PhD looked at auxetic polymeric materials. No one _____5_____
of them: they get fatter as you stretch them, _____6_____ is very
novel, and at the time there were only a handful of researchers in
Nota: other verbs which are used in this pattern are: the world working on these. The PhD started my interest in polymeric
materials. Towards the end of my PhD I _____7_____ two research
Voz passiva no infinitivo. roles, and ended up taking a job with British Nuclear Fuels Limited at
the Company Research Laboratory (CRL).
Exemplos:
(…)
It was the first/last/second/best one to be made/ to have been
made. During my time at CRL I _____8_____ on secondment to the
Sellafield site in Cumbria, which then turned into a permanent position
I would like to be invited/ I would like to have been invited. in the research and technology materials and inspection group.
During this time I became a chartered engineer and a full professional
Alguns casos de voz passiva com o verbo “to be”. member of the Institute of Materials Minerals and Mining. I now head
up one of Sellafield’s Centres of Expertise (CoE): I am the CoE lead and
Exemplos:
subject matter expert for polymeric materials. Recently I _____9_____
These cakes are not to be eaten until they are ready. as a fellow of the Institute of Materials.
I was to have been invited, but they lost my address! I definitely don’t have a “typical day”. I sometimes have a plan,
Not to be opened before christmas day. but _____10_____ stick to it as much of my work is responsive to
situations which are transient. The range of things I can get involved
To be taken three times a day before meal.
in is huge and includes specifying materials for use in challenging
The books to be catalogued are on your desk. environments, new plant designs and decommissioning activates.
There are a lot of letters to be written. (RATHBONE, Penny. Adapted from: The Guardian. A day in the life of a nuclear
materials engineer. Disponível em: <https://www.theguardian.com/women-in-
There is a lot to be done.
leadership/2016/jan/22/a-day-in-the-life-of-a-nuclear-materials-engineer>.
There is nothing to be done. Acesso em: 22/06/2017.)

Here are some books to be read.


Escolha a alternativa que completa corretamente a lacuna 8 do texto.
a) sent
Nota: a expressão “be born” já está na voz passiva.
b) was sent
Exemplos:
c) have sent
I was born in...
d) have been sending
How many babies are born every day?
e) am to be sent

A voz passiva usando “by” e “with”. 02. (EPCAR/AFA 2016)


Exemplos:
THE RELATIONSHIP BETWEEN FRIENDS AND TYPES OF FRIENDSHIP
Penicillin was discovered by Fleming in 1928.
Many towns in Mexico were destroyed by the earthquake. Everyone has at least one best friend, some maybe even more.
The building was occupied by security forces. There are also those people who are just friends and also arch-
The helicopter was piloted by the soldier. enemies. People may think that just because they are your friends it
means that they are your best friend. The thing is, even though they
The meat should be cut with a sharp knife. are your friend, the relationship between a best friend and a friend is
He was killed with a brick. different. Either way regardless of archenemies, friends or best friends,
This book was made with a new material. there are not many ways to compare any of these different types of
friends, but you can easily contrast them from one another.
Arch-enemies often know more about each other than two friends.
EXERCÍCIOS DE In a comparison of personal relationships, 1friendship is considered

FIXAÇÃO to be closer than association, although a wide range of degrees


of intimacy exists in friendships, arch-enemies, and associations.
Friendship and association can be thought of as spanning across the
same continuum. 2The study of friendship is included in the fields of
sociology, social psychology, anthropology, philosophy, and zoology.
01. (IME 2018) Leia o texto.
Even animals have familiars! Various academic theories of friendship
A DAY IN THE LIFE OF A NUCLEAR MATERIALS ENGINEER have been proposed, among which are social exchange theory, equity
theory, relational dialectics, and attachment styles. 3In Russia, one
typically bestows very few people the status of “friend”.
My career _____1_____ a planned one in any way. At school I
was athletic; I ran and played badminton to a high standard when I These friendships, however, make up in intensity what they lack
was young and always thought my career would be a sporting one in number. Friends are entitled to call each other by their first names
_____2_____ I suffered an injury during my teens. The rest of my alone, and to use diminutives. A customary example of polite behavior
is addressing “acquaintances” by full first name plus their patronymic.

288 PROMILITARES.COM.BR
PASSIVE VOICE

These could include relationships which elsewhere would be qualified a) Carole Walter is making great recipes.
as real friendships, such as workplace relationships of long standing, b) Carole Walter has made great recipes.
or neighbors with whom one shares an occasional meal or a social
c) Carole Walter makes great recipes.
drink with.
d) Carole Walter made great recipes.
Also in the Middle East and Central Asia, male friendships, while
less restricted than in Russia, tend to be reserved and respectable in
nature. They may use nicknames and diminutive forms of their first 04. (UFRGS 2012) Leia o que segue abaixo:
names. In countries like India, it is believed in some parts that friendship FACEBOOK IS THE WORLD’S LARGEST SOCIAL NETWORK, WITH 800
is a form of respect, not born out of fear or superiority. Friends are MILLION USERS WORLDWIDE AS OF SEPTEMBER 2011.
people who are equal in most standards, but still respect each other More than any other company, it has defined what 1______ see
regardless of their attributes or shortcomings. Most of the countries as the “social” era of the Internet, in which connections made among
previously mentioned (Russia, Asia, and even the Middle East) and 9
people 4replace 7algorithm-driven searches. And 8its 5policies, more than
even our own nation are suffering a decline in genuine friendships. any others, seem to be driving the definition of 14privacy in this new age.
According to a study documented in the June 2006 issue of the Every day, Facebook users comment or press the “like” button
Journal American Sociological Review, Americans are thought to be more than 2 billion times and upload more than 250 million photos.
suffering a loss in the quality and quantity of close friendships since The McKinsey Global Institute has estimated that the network’s users
at least 1985. The study’s results state that twenty-five percent of post 30 billion pieces of content 2______ month.
4
Americans have no close confidants, and the average total number
of confidants per citizen has dropped from four to two. According to The company, founded in 2004 by a Harvard 15sophomore, Mark
the study, 5Americans’ dependence on family as a safety net went up Zuckerberg, began life 16catering first to Harvard students and then to
from fifty-seven percent to eighty percent; Americans dependence on all high school and college students. It has since evolved into a broadly
a partner or spouse went up from five percent to nine percent. popular online destination used by teenagers and adults of all ages.
19
In country after country, Facebook has cemented itself as the leader,
Recent studies have found a link between fewer friendships, often displacing other social networks.
especially in quality, and psychological and physiological regression.
In the sequence of the emotional development of the individual, It is 3______ surprise that 11Facebook has become one of the titans
friendships come after parental bonding and before the pair bonding of the Internet, challenging even Google with 10its vision of a Web
engaged in at the approach of maturity. In the intervening period tied together by personal relationships and recommendations, rather
between the end of early childhood and the onset of full adulthood, than by search algorithms. In a major expansion, Facebook has spread
friendships are often the most important relationships in the emotional itself across other Web sites by offering members the chance to “Like”
life of the adolescent, and are often more intense than relationships something - share it with their network – without leaving the Web
experienced later in life. page they are on.
6
Unfortunately, making friends seems to trouble many of people. At the Facebook 20developer 6conference in September, 21the
Having no friends can be emotionally damaging for all ages, from company announced the release of a 13product called Timeline, 12which
young children to full grown adults. A study performed by researchers offers a 22highly visual view of a user’s Facebook profile and organizes
from Purdue University found that post-secondary-education content into photos, events and apps, all based on a 17timeline view
friendships, college and university last longer than the friendships that stretches back to the beginning of a user’s time on Facebook.
before it. Children with Asperger syndrome and autism usually have Timeline is designed to work on 18mobile devices, too.
some difficulty forming friendships. 7Socially crippling conditions like (Adaptado de: WYLD, Adrian. Facebook. Disponível em: <http://topics.nytimes.com>.
Acesso em 01 dez. 2011.)
these are just one way that the social world is so difficult to thrive
in. 8This does not mean that they are not able to form friendships,
Assinale a alternativa que preenche corretamente a lacuna do
however. With time, moderation and proper instruction, they are able
enunciado abaixo.
to form friendships after realizing their own strengths and weaknesses.
The passive version of the sentence [...] the company announced the
9
There is a number of theories that attempt to explain the link,
release of product called Timeline (ref. 21) is the sentence [...] the
including that; Good friends encourage their friends to lead more
release of a product called Timeline ______ by the company.
healthy lifestyles; 10Good friends encourage their friends to seek
help and access services, when needed; 11Good friends enhance a) was announced d) have been announced
their friend’s 12coping skills in dealing with illness and other health b) had been announced e) was being announced
problems; and/or Good friends actually affect physiological pathways c) were being announced
that are protective of health. Regardless of what we think, we can
clearly see that there are some ways that friends, best friends and
05. (EN 2016) Which option completes the paragraph below correctly?
archenemies are the same, but in the end they are clearly more
different. 13Nonetheless we all have every single type in our lives. ELECTRIC BIKES
(Adapted from: http://www.ukessays.com/essays/philosophy/therelationship-
between-friends-and-types-of-friendship-philosophyessay. php)
The US is different from other countries when it comes to electric
Choose the best option to complete the active form of the sentence: bikes. Nearly 32 m e-bikes __________ in 2014, most of them in
“The study of friendship is included in the fields of sociology, social China, where they are primarily used for transportation. They are
psychology, anthropology, philosophy, and zoology” (ref. 2). popular in much of Europe, too.
The fields of sociology, social psychology, anthropology, philosophy, They are common in the Netherland and Switzerland; German postal
and zoology __________ the study of friendship. workers use them to get around and BMW offers one for about $ 3,000.
a) include c) are including Electric bikes are different from motorcycles or mopeds, which
rely on motorized power; they are bicycles that __________ with – or
b) have included d) have been including
without – help from an electric motor.
Riding an e-bike feels like riding a normal bike with a strong wind
03. (EEAR 2019) Choose the alternative that corresponds to the
behind you; the motor just helps you to go faster or climb hills. Unlike
Active Voice of the following sentence: “Great recipes are made by
mopeds, e-bicycles __________ on bike paths and they cannot travel
Carole Walter”.
faster than 20 mph.
(Abridged from www.theguardian.com)

PROMILITARES.COM.BR 289
PASSIVE VOICE

a) were sold – can be pedaled – are usually permitted them in the West. But the phenomenon of Orientalism as studied here
b) were sold – can be pedal – usually permit deals principally, not with a correspondence between Orientalism and
Orient, but with the internal consistency of Orientalism and its ideas
c) are sold – can be pedaled – are usually permitted about the Orient 3__________ or beyond any correspondence, 6or lack
d) have been sold – can be pedal – usually permit thereof, with a ‘real’ Orient.
e) have been sold – can be pedaled – are usually permitted (Adaptado de: SAID, Edward. Orientalism. In: Ashcroft, Bill et al. (ed.) The
Postcolonial Studies Reader. London/ New York: Routledge, 1995. p. 87-91.)

06. (EN 2015) Which is the correct way to complete the text below?
Considere as seguintes propostas de reescrita do trecho Thus a very
large mass of writers and thinkers have accepted the basic distinction
MANY WOUNDER AS MOROCCAN POLICE BEAT PROTESTOR
between East and West (ref. 4).
(Reuters) – Moroccan police beat protesters who defied a ban on I. Thus the basic distinction between East and West have been
demonstrations across the country on Sunday, leading to arrests and accepted by a very large mass of writers and thinkers.
dozens of injuries, some of them life threatening, witnesses said. II. Thus the basic distinction between East and West has been
Much of the anger __________ at the Makhzen, Morocco’s royal accepted by a very large mass of writers and thinkers.
court. “Protest is a legal right, why is the Makhzen afraid?”, crowds III. Thus the basic distinction between East and West is being
in Casablanca chanted. “Makhzen get out. Down with despotism.” accepted by a very large mass of writers and thinkers.
A Reuters correspondent __________ seven riot police attacking Quais poderiam substituir o trecho destacado, sem prejuízo do sentido
one bearded man in his 30s, repeatedy hitting his head and body, literal e da correção gramatical?
causing severe bleeding. a) Apenas I.
“We __________ here to preserve order because of this b) Apenas II.
unauthorized protest”, said a senior police officer on the scene who
c) Apenas III.
__________ to give his name.
d) Apenas II e III.
No one was available at the Interior Ministry to comment on the
protesters’ reports. e) I, II e III.
(Adapted from http://www.linkedin.com)
08. (ITA 2014) Leia o texto.
a) directed/ was seen/ have called/ was declined
b) was directed/ was seen/ have called/ declined A HISTORY OF PI
c) directed/ saw/ had called/ were declined
The history of Pi, says the author, though a small part of the
d) was directed/ saw/ have been called/ declined history of mathematics, is nevertheless a mirror of the history of man.
e) directed/ was seen/ had called/ declined 5
Petr Beckmann holds up this mirror, 4giving the background of the
times when Pi made progress — and also when it did not, because
07. (UFRGS 2015) A questão a seguir está relacionada ao texto abaixo.
3
science was being 1stifled by militarism or religious fanaticism. The
mathematical level of this book is flexible, and there is plenty for
Orientalism means several interdependent things. The most readily
readers of all ages and interests.
accepted designation for Orientalism is an academic one. Anyone who
teaches, writes about, or researches the Orient, either in its specific or its ABOUT THE AUTHOR
general aspects, is an Orientalist, and what he or she does is Orientalism. Petr Beckmann was born in Prague, Czechoslovakia, in 1924.
Related to this academic tradition is a more general meaning for 6
Until 1963, he worked as a research scientist for the Czechoslovak
Orientalism as a style of thought based 1__________ a distinction made Academy of Sciences, when he was invited as a Visiting Professor to
between ‘the Orient’ and ‘the Occident.’ 4Thus a very large mass of the University of Colorado, where he decided to stay permanently as
writers and thinkers have accepted the basic distinction between East professor of electrical engineering.
and West as the starting point for elaborate theories, epics, novels, Dr. Beckmann has authored 11 books and more than 50 scientific
social descriptions, and political accounts concerning the Orient, its papers, 2mostly on probability theory and electromagnetic wave
people, customs, ‘mind,’ destiny, and so on. propagation. History is one of his side interests; another is linguistics
The interchange between the academic and the more or less (7he is fluent in five languages and he has worked out a new
imaginative meanings of Orientalism is a constant one, and since generative grammar which enables a computer to construct trillions of
the late 18th century there has been a considerable traffic between grammatical sentences from a dictionary of less than 100 unprocessed
the two. Here I come to the third meaning of Orientalism, which is words). 8He also publishes a monthly pro-science, pro-technology, pro-
something more historically and materially defined than either of the free enterprise newsletter Access to Energy, in which he promotes the
other two. Orientalism can be discussed and analyzed as the corporate viewpoint that clean energy can be made plentiful, but that access to
institution for dealing with the Orient—dealing with it by making it is blocked by government interference and environmental paranoia.
statements about it, authorizing views of it, describing it, by teaching it, (BECKMANN, Petr. A History of Pi. New York: Barnes & Noble Books, 1983.)
settling it: in short, Orientalism as a Western discourse for dominating,
restructuring, and having authority 2_________ the Orient. A opção que contém a reescrita correta de “...science was being
The Orient is not an inert fact of nature. It is not merely there, just stifled by militarism or religious fanaticism.” (ref. 3) é: militarism or
as the Occident itself is not just there either. As both geographical and religious fanaticism
cultural entities such regions as ‘Orient’ and ‘Occident’ are man-made. a) were stifling science.
Therefore as much as the West itself, the Orient is an idea that has a b) had been stifling science.
history and a tradition of thought, imagery, and vocabulary that have
given it reality and presence in and for the West. The two geographical c) were being stifling science.
entities thus support and to an extent reflect each other. It would be d) has stifling science.
wrong to conclude that the Orient is essentially an idea, or a creation e) have been stifling science.
with no corresponding reality. There are cultures and nations whose
location is in the East, and their lives, histories, and customs have a
5
brute reality obviously greater than anything that could be said about

290 PROMILITARES.COM.BR
PASSIVE VOICE

09. (UEG 2012) Observe a imagem. a) I and III. c) III and II.
b) II and IV. d) IV and I.

EXERCÍCIOS DE

TREINAMENTO
01. (UERN 2013) “These seeds are carried by the rivers” in the Active
Voice becomes
a) The rivers carry these seeds.
b) The rivers carried these seeds.
c) The rivers have carried these seeds.
d) The rivers are carrying these seeds.

02. (UECE 2014) The sentence “They are televising the court’s
proceedings” in the passive becomes
a) The court’s proceedings are being televised.
Em relação à fala da mulher, observa-se que:
b) The court’s proceedings can be televised.
a) as orações encontram-se respectivamente na voz passiva negativa
e afirmativa. c) The court’s proceedings are been televised.
b) o contrário da afirmação seria: “Yes, you were downloaded. You d) The court’s proceedings are to be televised.
were born”.
c) se refere a uma ação iniciada no passado e que se estende até o 03. (UERJ 2019)
presente.
THE EFFECT OF CLIMATE CHANGE ON EPIDEMIC RISK
d) se trata de uma resposta à pergunta do garoto: “How are people
downloaded?” 1
The potential impacts of climate change have returned to
headlines in recent weeks 2as scientists, activists and policy makers try
10. (EPCAR/AFA 2011) to understand the possible implications of a warming planet. 3While
THE LION KING rising temperatures and sea levels are important to be considered,
This article is about Disney’s 1994 film.
4
changing climate patterns can have vast implications for epidemic
risk as well.
The Lion King is a 1994 American animated feature produced
1

by Walt Disney Feature Animation. 2Released to theaters on June 15, Changes in global climate patterns have been 5widely discussed;
1994 by Walt Disney Pictures, it is the 32nd film in the Walt Disney however, rising temperatures also have implications for risk reduction
Animated Classics. 3The story, which was influenced by the Bible and management, including impacts on infectious disease epidemics.
stories of Joseph and Moses and the William Shakespeare play Hamlet, With 2016 the hottest year ever recorded and 2017 following suit, we
takes place in a kingdom of anthropomorphic animals in Africa. 4The anticipate a continued growth in the distribution of disease agents,
film was the highest grossing animated film of all time until the release like mosquitoes and ticks. 6These can spread illnesses such as zika,
of Finding Nemo. 5The Lion King still holds the record as the highest yellow fever and dengue to areas where they previously could not be
grossing traditionally animated film in history and belongs to an era
7
effectively transmitted.
known as the Disney Renaissance. As predicted by climate scientists, 8increases in extreme weather
The Lion King is the highest grossing 2D animated film of all time events may also lead to increases in infectious disease outbreaks.
in the United States, 6and received positive reviews from critics, who
9
Epidemics have previously been seen as a consequence of natural
praised the film for 8its music and story. During its release in 1994, the disasters, 10which can lead to displaced and crowded populations, the
film grossed more than $783 million worldwide, becoming the most ideal situation for infection transmission. Severe rainfall or flooding
successful film released that year, 7and it is currently the twenty-eighth is 11particularly effective at creating environments suitable for the
highest-grossing feature film. transmission and propagation of infectious diseases, such as measles
(http://en.wikipedia.org/wiki/The_Lion_King) or cholera.
Glossary: Even without rising to the level of a natural catastrophe, significant
Feature film – a film that is 90 or more minutes long variation in weather patterns can result in changes in human and
Gross – total animal interactions, increasing the potential for pathogens to move
Release – make public from animals into human populations. 12For example, unusually
Praise – show approval
heavy rains may predispose regions to ebola outbreaks by creating
Read the sentences below extracted from the text and mark the more favorable environments for bats hosting the virus. 13Similarly,
alternative that has the only ones in the Passive Voice. food scarcity brought about by drought, political instability or animal
disease may lead to more animal hunting, therefore raising the risk for
I. “The Lion King is a 1994 American animated feature produced by ebola virus epidemic.
Walt Disney...” (ref. 1)
It is important to take note of the impact of climate change on
II. “Released to theaters on June 15, 1994 by Walt Disney Pictures.” epidemic risk, but it is equally important to prepare for its impact
(ref. 2) on global health. 14The global health community has largely come
III. “The story, which was influenced by the Bible stories of Joseph to realize that public health preparedness is crucial to responding
and Moses and the William Shakespeare play Hamlet...” (ref. 3) efficiently to infectious disease outbreaks. For this reason, our work
IV. “The film was the highest grossing animated film of all time until is, then, centered around helping governments manage and quantify
the release of Finding Nemo.” (ref. 4) infectious disease risk. Besides, regardless of weather patterns,

PROMILITARES.COM.BR 291
PASSIVE VOICE

insights into epidemics and into mechanisms for ensuring adequate The history of the Brazilian military pilots schools goes back to
support are critical for managing this risk. 1913, when the Brazilian Aviation School was founded, at Campo dos
Since the public health community agrees that 15the question is Afonsos, State of Rio de Janeiro. Its mission was to provide instruction
not if another outbreak will happen, but when, the steps we take in at similar levels to those of the best European schools at the time;
the coming years to prepare for and reduce the increasing frequency Blériot and Farman aircraft, made in France, were available for the
of outbreaks will determine the broader implications these diseases instruction of the pupils. The Great War 1914-1918, however, forced
have on our world. its instructors to leave and the school was closed.
(contagionlive.com) At that time, both the Brazilian Army and Navy had their own air
arms, the Military Aviation and the Naval Aviation. The Navy bought
One of the marked characteristics of scientific texts is the presence of Curtiss F seaplanes in May 1916 to equip the latter, and in August of
passive voice. the same year, the Naval Aviation School was created.
An example from the text that illustrates this characteristic is indicated in: The Military Aviation, however, only activated its Military Aviation
a) “The potential impacts of climate change have returned to School after the Great War, on 10 July 1919. Among the aircrafts
headlines in recent weeks” (ref. 1) used at the school, one could find the Sopwith 1A2, Bréguet 14A2,
and Spad 7.
b) “increases in extreme weather events may also lead to increases in
infectious disease outbreaks.” (ref. 8) Until the beginning of the 1940s, both schools continued with
their activities. 1The Brazilian Government was concerned with the air
c) “Epidemics have previously been seen as a consequence of natural
war in Europe and decided to concentrate under a single command the
disasters,” (ref. 9)
military aviation activities. 6Thus, on 20 January 1941, the Air Ministry
d) “which can lead to displaced and crowded populations,” (ref. 10) was created and both the Army and Navy air arms were disbanded,
their personnel and equipment forming the Brazilian Air Force. On
04. (EPCAR/AFA 2019) Mark the option in which the sentence is an 25 March 1941, the Aeronautics School was based at Campo dos
example of passive voice. Afonsos, and its students became known as Aeronautics Cadets from
a) Sigmund Freud viewed human nature as inherently antisocial, 1943 to the current days.
biologically driven by the undisciplined id's pleasure principle. As early as 1942, it became clear that the Aeronautics School
b) People who haven't met their most basic needs will have difficulty would need to be transferred to another place, offering better climate
maturing. and little interference with the flight instruction of the future pilots.
2
The town of Pirassununga was chosen among others, and, in 1952,
c) Humans have been captivated by stories of heroes facing off the first buildings construction was initiated. The transfer of the
against superhuman foes. School activities to Pirassununga occurred from 1960 to 1971. 3The
d) We have needed heroes who rise to the occasion, overcome great School was redesigned as the Air Force Academy in 1969.
odds and take down giants. The motto of the Academy is the Latin expression “Macte Animo!
Generose Puer, sic itur ad astra”, extracted from the poem Thebaida,
05. (MACKENZIE 2016) The sentence “The information was given on by the Roman poet Tatius. It is an exhortation to the cadets, which
Thursday by the Minister of Justice José Eduardo Cardozo, at a forum can be translated as Courage! This is the way, oh noble youngster, to
held by Folha in São Paulo” in the active voice is the stars.
a) The information given on Thursday at the Folha forum in São The instruction of the Aeronautics Cadets, during the four-
Paulo by the Minister of Justice José Eduardo Cardozo. year-long course, has its activities centred in the words COURAGE –
b) The Minister of Justice José Eduardo Cardozo at a forum held by LOYALTY – HONOUR – DUTY – MOTHERLAND. The future officers take
Folha in São Paulo gave the information. courses on several subjects, including Calculus, Computer Science,
Mechanics, Portuguese and English, given by civilian lecturers, Air
c) The information that the Minister of Justice José Eduardo Cardozo
Force instructors and supervisors. The military instruction itself is given
gave on Thursday at a forum was held by Folha in São Paulo.
on a daily basis, and 4the Cadets are trained on different subjects,
d) The forum that Folha held in São Paulo on Thursday gave the including parachuting, and sea and jungle survival.
information by the Minister of Justice José Eduardo Cardozo.
e) On Thursday, The Minister of Justice José Eduardo Cardozo gave
the information at a forum that Folha held in São Paulo.

06. (EPCAR/AFA 2013)


BRAZILIAN AIR FORCE ACADEMY

Flight instruction at the Aademy with T-27 Tucano aircraft

According to the chosen specialization, the Cadet will receive


specific instruction:
Pilots: Instruction on precision maneuvering, aerobatics,
formation flying and by instruments, with 75 flying hours on the
primary/basic training aircraft T-25 Universal, beginning on the 2nd
AFA (Air Force Academy), located at Pirassununga, State of São
term of the 1st year and completed in the 3rd year. Advanced training
Paulo, is responsible for the training of Pilots, Administrative and
is given on T-27 Tucano aircraft, with 125 flying hours.
Aeronautics Infantry Officers for the Brazilian Air Force.

292 PROMILITARES.COM.BR
PASSIVE VOICE

Administrative: Training on the scientific and technological


modern foundations of economics and financial management, and
logistics training.
Aeronautics Infantry: Instruction on defense and security
techniques of military Aeronautics installations, anti-aircraft measures,
command of troops and firefighting teams, military laws and
regulations, armament usage, military service and call-up procedures.
During their leisure time, the Cadets participate on the activities of
seven different clubs: Aeromodelling, Literature, Informatics, Firearms
shooting, Gauchos Heritage (for those coming from the South of
Brazil), Gerais Club and Sail Flying. The clubs are directed by the
Cadets themselves, under supervision of Air Force officers.
City officials have now come round to thinking that cabs for
The Academy also houses the Brazilian Air Force Air Demonstration women would be safer for both passengers and drivers (Ms Sánchez
Squadron – The Smoke Squadron. began her crusade after being stabbed by a male client). From next
Flying as the eagles do! month, a fleet of pink taxis driven by and for women will roam the
(Adapted from http://www.rudnei.cunha.nom.br/FAB/en/afa.html) streets of the capital, charging the same fares as ordinary cabs. The
city government is training a first batch of approved drivers in security
Mark the alternative that has the fragment from the text INCORRECTLY and women’s rights.
changed into Active Voice. The suburban railway and buses have followed the metro in
a) The air war in Europe concerned The Brazilian Government. (ref. 1) providing women-only services. Victor Ramírez, a transport official,
b) Someone chose the town of Pirassununga among others. (ref. 2) says he is now fielding requests to segregate the pesero microbuses
which rattle around town.
c) Somebody redesigned the School as the Air Force Academy. (ref. 3)
(Pink cabs rev up - The Economist August 28th -3rd September 2010)
d) The officers trained the Cadets on different subjects. (ref. 4)
Das frases abaixo, retiradas do texto, indique a que apresenta sua
07. (FATEC 2012 ADAPTADA) Assinale a alternativa que apresenta construção na voz passiva.
a forma correta da voz ativa para o trecho em destaque na seguinte a) The latest controversy concerns women-only public...
passagem do texto: Kennedy’s Operation Ceasefire, as it has come to
be known, has been implemented in more than 70 cities. b) During rush hour, men have long been barred from a third...

a) They have implemented Kennedy’s Operation Ceasefire, as it has c) His latest move, cheered by environmentalists, was a ban…
come to be known, in more than 70 cities. d) The city government is training a first batch of approved…
b) They had implemented Kennedy’s Operation Ceasefire, as it has e) City officials have now come round to thinking that cabs...
come to be known, in more than 70 cities.
c) They were implemented Kennedy’s Operation Ceasefire, as it has 09. (EPCAR/AFA 2011) The sentence “the smallest butterfly in the
come to be known, in more than 70 cities. world is found in South Africa” in the Active Voice becomes
d) They had been implementing Kennedy’s Operation Ceasefire, as it a) “People would find the smallest butterfly in the world in South Africa.”
has come to be known, in more than 70 cities. b) “People found the smallest butterfly in the world in South Africa.”
e) They have been implementing Kennedy’s Operation Ceasefire, as c) “People find the smallest butterfly in the world in South Africa.”
it has come to be known, in more than 70 cities. d) “People are going to find the smallest butterfly in the world in
South Africa.”
08. (IFSP 2011) Leia o texto.

GENDER POLITICS IN MEXICO CITY 10. (UFRGS 2014) Leia o texto abaixo:
Pink cabs rev up I am happy to join 1__________ you today in what will go down
Mexico City in history as the greatest demonstration for freedom in the history of
our nation.
A blow for feminism−or against it? In the process 2__________ gaining our rightful place we must not
Since electing its first left-wing mayor in 1997, Mexico City has be guilty of wrongful deeds. Let us not seek to satisfy our thirst for
been a selfconsciously liberal oasis in a conservative country. The freedom 3__________ drinking from the cup of bitterness and hatred.
current mayor, Marcelo Ebrard, has legalised abortion on demand, We must forever conduct our struggle on the high plane of dignity
gay marriage and gay adoption in his first four years in office. His and discipline. We must not allow our creative protest to degenerate
latest move, cheered by environmentalists, was a ban on free plastic into physical violence. 4The marvelous new militancy which has
shopping bags, implemented on August 19th. Eye-catching reforms engulfed the 15Negro community must not lead us to distrust of 8all
such as these are enhancing Mr Ebrard’s profile ahead of a likely white people, for 13many of our white brothers, as evidenced by 5their
presidential bid in two years’ time. presence here today, have come to realize that their destiny is tied
up with our destiny and their freedom is 17inextricably bound to our
The latest controversy concerns women-only public transport.
freedom. We cannot walk alone.
During rush hour, men have long been barred from a third of the
carriages of metro trains. Some see that as offering a blessed sanctuary I have a 9dream that one day 11this nation will rise up and 18live
from wandering macho hands; for others it is a backward step on out the true 16meaning of 6its creed: “We hold these truths to be
the march to equality. But whereas Puebla, a nearby city of more self-evident: that all men are created equal.” I have a dream that my
conservative bent, runs a women-only “pinktaxi” service (pictured four little children will one day live in a nation where they will not be
above), Mexico City had resisted. Susana Sánchez, a Mexico City judged by the color of their skin but by the content of their character.
taxista, first requested permission to run such a service in 1998. She 19
This is our hope. This is the faith that I go back to the South with.
was told it would be discriminatory. When we allow 10freedom to 12ring, when we let 7it ring from every
state and every city, we will speed up that 14day when all of God’s
children, black men and white men, Jews and Gentiles, Protestants

PROMILITARES.COM.BR 293
PASSIVE VOICE

and Catholics, will join hands and sing the old Negro spiritual, “Free at 06. (UNESP) A voz passiva de “I’m reading the magazine” é:
last! free at last! thank God Almighty, we are free at last!” a) The magazine is being read (by me).
(Adaptado de: LUTHER KING JR., Martin. I have a dream. Disponível em:<http://www.
archives.gov/press/exhibits/dream-speech.pdf>. Acesso em: 06 set. 2013.) b) The magazine was being read (by me).
c) The magazine has been read (by me).
Assinale a alternativa que poderia substituir o trecho “The marvelous d) The magazine had been read (by me).
new militancy which has engulfed the Negro community” (ref. 4), sem
significativa alteração de sentido ou prejuízo da correção gramatical. e) The magazine were read (by me).
a) The Negro community which has engulfed in this marvelous new
07. (CESE-SP) Mude para passiva: “We’ll have to examine you again.”
militancy
a) We’ll have to be examined by you again.
b) The Negro community which has been engulfed in this marvelous
new militancy b) You’ll examine to have to.
c) The Negro community that was engulfed in this marvelous new c) To have been examined we’ll have to.
militancy d) You’ll have been examined again.
d) The marvelous new militancy in which the Negro community has e) You’ll have to be examined again.
been engulfed
e) The marvelous new militancy that the Negro community has been 08. (ESPCEX) Choose the alternative that has the sentence “Operation
engulfed Desert Storm was not won by smart weaponry” correctly changed
into active voice.
a) Smart weaponry hasn’t won operation desert storm.
EXERCÍCIOS DE b) Smart weaponry didn’t win operation desert storm.

COMBATE c) Smart weaponry doesn’t win operation desert storm.


d) Smart weaponry isn’t winning operation desert storm.
e) Smart weaponry won’t win operation desert storm.

01. (COM. AERONÁUTICA) This house ________ in 1970 by my


09. (IME) The passive voice of “Thousands of people welcomed the
grandfather.
President at the airport” is:
a) Built b) Was built c) Was build d) Has built
a) The President was welcomed by thousands of people at the airport.
b) The President at the airport welcomed thousands of people.
02. (AFA) Choose the best option to change the sentence “human
capacities are represented in the brain” into the active form. c) The President is welcome at the airport by thousands of people.
“The brain _______________ human capacities.” d) At the airport the President be welcome by thousands of people.
a) has represented c) has been represented e) The President was welcome at the airport by thousands of people.
b) represents d) representing
10. (OSEC-SP) Escolha a alternativa correta que indica a voz passiva
desta frase:
03. (FMU-SP) Indique a forma passiva de: “People found out all the truth.”
“You must write the answers on one side of the paper only.”
a) All the truth were found out by people.
a) Only on one side of the paper must be written the answers.
b) All the truth was found out.
b) The answers are written on one side of the paper only.
c) All the truth had been found out.
c) You must be written on one side of the paper only.
d) All the truth had been found out by the people.
d) The answers must be written on one side of the paper only.
e) All the truth has been found out.
e) One side of the paper must be written by you.
04. (AFA) What’s the passive voice for “Mr. McCannigan left this
leather jacket in the back seat of my car last night”?
a) Mr. McCanningan had been leaving this jacket in the back seat GABARITO
of my car.
EXERCÍCIOS DE FIXAÇÃO
b) Last night this leather jacket in the back left in my car by Mr.
McCannigan. 01. B 04. A 07. B 10. A
c) This leather jacket was left in the back seat of my car by Mr. 02. A 05. A 08. A
McCannigan last night. 03. C 06. D 09. A
d) This leather jacket had been left by the back seat of Mr. EXERCÍCIOS DE TREINAMENTO
McCannigan’s car last night. 01. A 04. C 07. A 10. D
02. A 05. E 08. B
05. (CFOE) The sentence “hypertext disrupts this cohesion” in the
passive voice is 03. C 06. D 09. C
a) this cohesion disrupted by hypertext. EXERCÍCIOS DE COMBATE
b) this cohesion is disrupts by hypertext. 01. B 04. C 07. E 10. D
c) this cohesion is disrupted by hypertext. 02. B 05. C 08. B
d) this cohesion has disrupted by hypertext. 03. B 06. A 09. A

294 PROMILITARES.COM.BR
CONDITIONAL SENTENCES

CONDITION: IF CLAUSES O USO DOS VERBOS MODAIS


Usamos frases condicionais para mostrar que uma ideia depende EM “IF CLAUSES”
da outra.
• Formas que apresentam ideias no futuro.
As “Conditional Tenses” são usadas em estruturas chamadas de
Exemplos:
“If-clauses”, onde existe uma correlação entre duas formas verbais,
gerada pela expressão “if” (se) “Unless” (a menos que, a não ser que). If it rains, we shall (will) stay at home.
Exemplos: If I have enough time, I may go and see Pamela this evening.
If you said it before, I would buy you some potatoes. If you study really hard, you should enter to EFOMM.
I won’t go there unless you ask me to. If they don’t come soon, they might miss the train.

CORREÇÃO DE TEMPOS VERBAIS NAS IF-CLAUSES • Formas que apresentam ideias no presente.
Exemplos:
Simple Past Simple Conditional
If I were you, I would take up a sport to keep fit.
If you studied harder, you would enter to AFA If you saved your money, you could afford a holiday abroad.
You would enter AFA if you studied harder.
Observação
Past Perfect Conditional Perfect /
Conditional Continuous Nesse caso acima usamos a forma do verbo “to be” no subjuntivo,
logo a forma correta será: I were, you were, he were, she were, it
If she had planned in advance, this wouldn’t have happened. were, we were, you were, they were.
This wouldn’t have happened if she had planned in advance. Algumas gramáticas aceitam, I was, he was, she was, It was.
If Paul had asked me, I would be helping him with his tasks.
I would be helping Paul with his tasks if he had asked me. • Formas que apresentam ideias no passado.
Exemplos:
Simple Present Simple Present
If I hadn’t come to Rio, we would never have met, darling.
If you put water in the freezer, it becomes ice. If you hadn’t reacted so quickly, we might well have had an
Water becomes ice if you put it in the freezer. accident.

Simple Present Simple Future / Future Continuous


As seguintes expressões também são usadas para introduzir
If you wait a minute, our maid will bring you some coffee orações condicionais: “unless (if not)”, “in case”, “provided or
Our maid will bring you some coffee if you wait a minute. providing”, “on condition that” e “supposing”.

If it rains, there are going to be floods all Rio de Janeiro. Exemplos:

There are going to be floods all around Rio de Janeiro if it rains. Supposing everyone did like that, (what would happen then)?

If you authorize me to, I will be writing to our students in five Here’s a ten-dollar bill. Don’t spend it unless you have to.
minutes I’ll give you some money in case you need to buy some clothes.
I will be writing to our students in five minutes if you authorize You can go out tonight, provided you have finished your homework.
me to.
Observação
Simple Present Imperative
A expressão “unless” significa “if not”.
If you drink, don’t drive. Exemplos:
Don’t drive if you drink. She will fail the exams, unless she studies harder.
If she doesn’t study harder, she will fail the exams.

PROMILITARES.COM.BR 295
CONDITIONAL SENTENCES

Nas “If-clauses”, a inversão do verbo com o sujeito elimina 05. (EFOMM 2018) Which option is NOT correct?
o uso do “if”. a) If your mother will fill in this form, I’ll prepare her ticket.
Exemplos: b) If Ann won’t be here on Monday, we’d better cancel the meeting.
Had you left earlier, this wouldn’t have happened (if you had left…) c) If you should run into Peter, tell him he owes me a letter.
Were I to be invited, I would go (If I were invited...) d) Hadn’t we missed the plane, we would all have been killed in the crash.
e) If it hadn’t been for your help, I don’t know what I’d have done.
As “Conditional Tenses” são usados com “wish” (desejar).
Exemplos: 06. (EN 2016) Which of the sentences below is INCORRECT?
I really wish it would stop raining. a) If the Earth dies, you die. If you die, the planet surely survives.
She wishes she would have rested more. b) If you don’t want to wait 1.5 seconds, press Control-Power button.
c) I will not go away if you promise to change and behave yourself.
Nas “if-clauses” e com “wish”, o passado de “be” é sempre d) You could see it through my eyes if you would realize how special
“were”, mesmo na 3ª pessoa do singular. you are.
Exemplos: e) My mother is going to kick me out tomorrow unless I stop all
If I were you, I wouldn’t say that. contact with my boyfriend.

She wishes she were married.


07. (FGV 2015) Read the text and answer the question.
ARGENTINA DEFAULTS – EIGHTH TIME UNLUCKY
EXERCÍCIOS DE
Cristina Fernández argues that her country’s latest default is different.

FIXAÇÃO She is missing the point.


Aug 2nd 2014

ARGENTINA’S first bond, issued in 1824, was supposed to have


had a lifespan of 46 years. Less than four years later, the government
01. (UNESP 2018) Assinale a alternativa que completa a lacuna da tira.
defaulted. Resolving the ensuing stand-off with creditors took 29
years. Since then seven more defaults have followed, the most recent
this week, when Argentina failed to make a payment on bonds issued
as partial compensation to victims of the previous default, in 2001.
Most investors think they can see a pattern in all this, but
Argentina’s president, Cristina Fernández de Kirchner, insists the latest
default is not like the others. Her government, she points out, had
transferred the full $539m it owed to the banks that administer the
bonds. It is America’s courts (the bonds were issued under American
law) that blocked the payment, at the behest of the tiny minority of
a) must c) can e) would owners of bonds from 2001 who did not accept the restructuring
Argentina offered them in 2005 and again in 2010. These “hold-
b) am going to d) have been outs”, balking at the 65% haircut the restructuring entailed, not only
persuaded a judge that they should be paid in full but also got him to
02. (UEMG 2015) Which alternative contains the correct conditional 1
freeze payments on the restructured bonds until Argentina coughs up.
to complete the gap below?
Argentina claims that paying the hold-outs was impossible. It
“He would have chatted with his Facebook friends last night if he is not just that they are “vultures” as Argentine officials often put
___________ so busy.” it, who bought the bonds for cents on the dollar after the previous
a) hadn’t been c) weren’t default and are now holding those who accepted the restructuring
(accounting for 93% of the debt) to ransom. The main problem is that
b) wasn’t d) hasn’t been
a clause in the restructured bonds prohibits Argentina from offering
the hold-outs better terms without paying everyone else the same.
03. (UFRGS 2014) Assinale a alternativa que preenche corretamente Since it cannot afford to do that, it says it had no choice but to default.
as lacunas do segmento abaixo, na ordem em que aparecem.
Yet it is not certain that the clause requiring equal treatment of
“If the present Queen __________ for another ten years, Charles all bondholders would have applied, given that Argentina would not
__________ to the throne at 75.” have been paying the hold-outs voluntarily, but on the courts’ orders.
a) survives – comes d) survived – can come Moreover, some owners of the restructured bonds had agreed to
b) survives – will come e) survives – would come waive their rights; 2had Argentina made a concerted effort to persuade
the remainder to do the same, it might have succeeded. Lawyers and
c) will survive – comes
bankers have suggested various ways around the clause in question,
which expires at the end of the year. But Argentina’s government was
04. (PM-SC 2017) The first conditional expresses a possibility in the slow to consider these options or negotiate with the hold-outs, hiding
future. Complete with the most suitable verb forms. instead behind indignant nationalism.
“If you __________ on the Internet, you ___________ that almost Ms Fernández is right that the consequences of America’s court
every day of the year celebrates some kind of food” rulings have been perverse, unleashing a big financial dispute in
a) were looking – would seeing an attempt to solve a relatively small one. But 3hers is not the first
b) are looking – would see government to be hit with an awkward verdict. Instead of railing
against it, she should have tried to minimize the harm it did.
c) had looked – would have seen
Defaulting has helped no one: none of the bondholders will now be
d) look – will see paid, Argentina looks like a pariah again, and its economy will remain
e) looks – sees starved of loans and investment.

296 PROMILITARES.COM.BR
CONDITIONAL SENTENCES

Happily, much of the damage can still be undone. It is not too 09. (PUC-RS 2014)
late to strike a deal with the hold-outs or back an ostensibly private EIGHT RULES FOR WALKS IN THE COUNTRY
effort to buy out their claims. A quick fix would make it easier for
Argentina to borrow again internationally. That, in turn, would speed Posted by Tom Cox
development of big oil and gas deposits, the income from which could
help ease its money troubles. “It’s quite an up and down kind of walk,” said my friend Emma.
More important, it would help to change perceptions of Argentina
4 “Oh,” added Emma, “and it gets a bit blowy up there, so I’d leave
as a financial rogue state. Over the past year or so Ms Fernández seems your credit card back here if I were you.” I gave her a searching look,
to have been trying to rehabilitate Argentina’s image and resuscitate wondering how a credit card might relate to a strong wind. “I took
its faltering economy. She settled financial disputes with government mine up there the other week and it blew out of my hand into the
creditors and with Repsol, a Spanish oil firm whose Argentine assets sea,” she clarified. “I had to order a new one.”
she had expropriated in 2012. This week’s events have overshadowed I fell in love with walking because it lifted my spirit and took me to
all that. For its own sake, and everyone else’s, 5Argentina should hold parts of my local area that I would never have _________ otherwise,
its nose and do a deal with the hold-outs. but also because there was something brilliantly ridiculous about the
(http://www.economist.com/news/leaders/21610263. Adapted) idea of _________ yourself, on a whim, alone, in a bit of countryside
you’d never _________ before, with no real goal apart from putting
The excerpt from the reference 2 – had Argentina made a concerted one foot in front of the other.
effort to persuade the remainder to do the same, it might have I’ve never really dressed in any walking-specific clothing or taken
succeeded. – denotes an idea of: any special supplies out with me, but I do think there are a few things
a) obligation. d) hypothesis. I’ve learned about “how to walk” in gentle terrain that might help
b) ability. e) necessity. others. I have compiled some of the main ones: always be assertive
in saying “Hello!” to fellow walkers, unless in a built-up area; learn
c) completion. to fold your map properly; show strange dogs and cows who is boss;
don’t be afraid of dictaphones*; try not to have a beard, but if you
08. (UFG 2013) The following poem presents a different version of the do have a beard, have a dog as well; try to avoid headwear, unless
popular song “California Dreamin’” by The Mammas and The Papas strictly necessary; choose an apt soundtrack for your walk; watch out
(1965). for fookwits and loonies!
This last one doesn’t apply specifically to country walks. It’s just
CALIFORNIA STEAMIN’ something that my dad tells me every time I see him, and it’s worked
fairly well as a general rule for life over the years, so it probably works
By Clinton VanInman – Contributing Poet
for walking as well.
All the tress are brown *voice recorders
And the sky is gray (Adapted from http://www.theguardian.com/lifeandstyle/
I’ve been for a walk 2013/sep/11/eight-rules-country-walks.)
On a greenhouse day.
To solve question, read paragraph 3 and select the correct words to
I should be safe and sound now complete the gaps.
If I was miles from L.A. _________ you _________ in a built-up area, you _________ greet
California steamin’ the fellow walkers.
On such a sweltering day. According to the idea in the text, the correct words to fill in the gaps
are, respectively,
Stopped into a church
I stumbled along the way a) Unless / aren’t / mustn’t d) If / are / should
Got down on my knees b) Unless / are / should e) If / aren’t / shouldn’t
And prayed for a rainy day. c) If / are / must

You know the preacher likes it cold 10. (EPCAR/AFA 2019)


Now that all his candles have melted away,
California steamin’ WHY DO SUPERVILLAINS FASCINATE US?
Please don’t take my fan away. A PSYCHOLOGICAL PERSPECTIVE
(Disponível em: <http://cafe.cynicmag.com/>. Acesso em: 20 set. 2012.)
Why are we fascinated by supervillains? Posing the question is
Glossário:
much like asking 1why evil itself intrigues us, but 2there's much more
steamin’: fumegante
sweltering: abafado to our continued interest in supervillains than meets the eye.
stumbled: cambaleei 3
Not only do Lex Luthor, Dracula and the Red Skull 4run
“I should be safe and sound now If I was miles from L.A.” means that unconstrained by conventional morality, 5they exist outside the limits
the poet: of reality itself. Their evil, even at its most realistic, retains a touch of
the unreal.
a) isn't in Los Angeles (L.A.).
But 6is our fascination with fantastic 7fiends healthy? From a
b) feels free of danger in L.A. psychological perspective, views vary on 8what drives our enduring
c) is likely to move to another city. interest in superhuman bad guys.
d) wishes he was somewhere else. Shadow confrontation: Psychiatrist Carl Jung believed we need to
e) regrets a past situation. confront and understand our own hidden nature to grow as human
beings. Healthy confrontation with our shadow selves can unearth
new strengths (e.g., Bruce Wayne creating his Dark Knight persona to
fight crime), whereas unhealthy attempts at confrontation may involve
dwelling on or unleashing the worst parts of ourselves.

PROMILITARES.COM.BR 297
CONDITIONAL SENTENCES

Wish fulfillment: 9Sigmund Freud viewed human nature as The sentence in the text “As you know, if you dedicate to your studies,
inherently antisocial, biologically driven by the undisciplined id's you will succeed in life.”, underlined in the text, is classified by
pleasure principle to get what we want when we want it – born to be a) Second Conditional. c) First Conditional.
bad but held back by society. Even if the psyche fully develops its ego
(source of self-control) and superego (conscience), Freudians say the b) Third Conditional. d) Zero Conditional.
id still 10dwells underneath, and it wishes for many selfish things – so
it would love to be supervillainous. 03. (EN 2017) Which of the sentences below is correct?
Hierarchy of needs: Humanistic psychologist 11Abraham Maslow a) You wouldn’t spend so much if you didn’t have a good salary.
held that 12people who haven't met their most basic needs will have b) If you would plan to use Wi-Fi, connect your TV with your router.
difficulty maturing. If starved for food, you're unlikely to feel secure. c) Unless he tried hard, he won’t get the job of his dreams.
If starved for love and companionship, you'll have trouble building
self-esteem. People who dwell on their deficits may envy and resent d) If he were married, he won’t travel alone so much.
others who have more than they do. Some people who are unable to e) She won’t buy a new car even if she'll have the money.
overcome social shortcomings fantasize about obtaining any means,
good or bad, to satisfy every need and greed. 04. (MACKENZIE 2016) The sentence “If Science Without Borders is
[…] suspended, another federal education program will be significantly
(Adapted from https://www.wired.com/2012/07/why-do-supervillainsfascinate-us/) expanded” written in the third conditional form is:
a) If Science Without Borders were suspended, another federal
The sentence “[…] Abraham Maslow held that people who haven't education program would be significantly expanded.
met their most basic needs will have difficulty maturing.” (ref. 11)
b) If Science Without Borders have been suspended, another federal
means the psychologist believes that:
education program will have been significantly expanded.
a) if people don’t become mature, they will have trouble meeting
c) If Science Without Borders could be suspended, another federal
their basic needs.
education program could have been significantly expanded.
b) if Abraham Maslow hadn’t met his basic needs, people would
d) If Science Without Borders had being suspended, another federal
have had difficulty maturing.
education program will have being significantly expanded.
c) unless people fulfill their basic necessities, getting mature won’t
e) If Science Without Borders had been suspended, another federal
be easy for them.
education program would have been significantly expanded.
d) unless one gets their basic necessities, they won’t have difficulty
maturing. 05. Which sentence is grammatically correct about the picture below?

EXERCÍCIOS DE

TREINAMENTO
01.

The sentence “Would you still love me if I did something wrong?” in


the third conditional form is
a) If I did something wrong, would you had loved me? a) If the 2 lions had been taller, they wouldn’t have to had helped
each other.
b) Would you still love me if I done something wrong?
b) If one lion was bigger, the other wouldn’t be help him.
c) If I had done something wrong, would you have loved me?
c) Had one lion taller, the other would have helped him.
d) Had I did something wrong, would you have loved me?
d) Should a lion be bigger, the other have helped him.
e) Would you have loved me if I would have done something wrong?
e) If one lion were taller, the other one wouldn’t have had to help him.
02. (EEAR 2019) Why Learn with English Podcasts? English language
podcasts are an excellent way to learn English quickly. You can listen 06. (IFBA 2012) “We need a new environmental consciousness on a
to them anytime anywhere – at your desk or while you’re on the global basis. To do this, we need to educate people.”
move. What better way to pass the time during a long commute (Mikhail Gorbachev
Disponível em http://edugreen.teri.res.in/misc/quotes.htm. Acesso em: 12.07.2011)
than by immersing yourself in an entertaining podcast? With a little
dedication, English language podcasts will help you quickly improve
A opção que melhor apresenta a mensagem da citação de Mikhail
your listening skills and proficiency. One perk (benefit) is that podcasts
Gorbachev é:
often have transcripts (a written version of the audio). This means that
you can listen and read at the same time, or look at a transcript if one a) If a new environmental consciousness is possible, the global basis
part of a podcast confuses you. As you know, if you dedicate to your will appreciate it.
studies, you will succeed in life. b) If we need a new environmental consciousness, we should do our
(Adapted https://www.fluentu.com/blog/english/esl-english-podcasts/) educated tasks.

298 PROMILITARES.COM.BR
CONDITIONAL SENTENCES

c) If we needed educated people, a new environmental consciousness 08. Which sentence is grammatically correct about the picture below?
would be gotten on a global basis.
d) If we were educated people, a new environmental consciousness
would be possible on a global basis.
e) If there is a new environmental consciousness on a global basis,
we will be educated people.

07. (UFSJ 2012)


BLAMING OTHERS CAN RUIN YOUR HEALTH
By Elizabeth Cohen, Senior Medical Correspondent
August 18, 2011 -- Updated 1837 GMT (0237 HKT)

(CNN) -- Kevin Benton had every reason


to feel bitter. During his sophomore year in
college, he says, 1white students harassed a) If you didn’t hold on, you would end the pain.
him and the only other African-American b) Should you wait, you won’t be painful.
living on the floor in his dorm in order to
get them to move out. “I felt like I was c) Had hope been practiced, we wouldn’t have had to wait.
being bullied, being targeted,” he says now d) People wish hope had been held on.
of his college experience 19 years ago. e) There is nothing hope doesn’t bring to ordinary people.
This was the first time in his life Benton had encountered racism
and it hit him hard. He had trouble sleeping, and then over the next 09. (ESPM 2013)
several months he suffered panic attacks. Admitted to the hospital, he
was found to have hypertrophic cardiomyopathy, or thickening of the
muscles in the heart. The disease is the leading cause of heart-related
sudden death in people under 30.
After some time, Benton could forgive the students who had
tormented them, and three days later, he walked out of the hospital.
2
”If I hadn’t forgiven them, I’d be dead,” says Benton, now healthy and
a social worker for the Philadelphia Department of Human Services.
Feeling persistently resentful toward other people can indeed affect
your physical health, according to a new book, “Embitterment: Societal, Calvin’s words in the third frame could be replaced, without changing
psychological, and clinical perspectives.” In fact, the negative power their meaning, by:
of feeling bitter is so strong that the authors call for the creation of a
new diagnosis called PTED, or post-traumatic embitterment disorder, to a) If you weren’t such a muttonhead, you mightn't think of it yourself.
describe people who can’t forgive others’ transgressions against them. b) If you hadn’t been such a muttonhead, you might have thought
Feeling bitter interferes with the body’s hormonal and immune of it yourself.
systems, according to Carsten Wrosch, an associate professor of c) Unless you had been such a muttonhead, you might have thought
psychology at Concordia University in Montreal and an author of of it yourself.
a chapter in the new book. 3Studies have shown that bitter, angry d) Unless you were such a muttonhead, you could have thought of
people have higher blood pressure and heart rate and are more likely it yourself.
to die of heart disease and other illnesses.
e) Weren’t you such a muttonhead, you could have thought of it yourself.
Physiologically, when we feel negatively towards someone, our
bodies instinctively prepare to fight that person, which leads to
10. (MACKENZIE 2013) The following text refers to the question.
changes such as an increase in blood pressure. “We run hot as our
inflammatory system responds to dangers and threats,” says Raison, JOIN MY NEW CLUB, “TGIT”
clinical director of the Mind-Body Program at Emory. Feeling this way
in the short term might not be dangerous -- it might even be helpful to
Until now, the business world was primarily made of two clubs.
fight off an enemy -- but the problem with bitterness is that it goes on
The most popular club, by far, was the “TGIF” club, or “Thank God
and on. When our bodies are constantly primed to fight someone, the
It’s Friday.” To be a member of this club, your primary focus is on
increase in blood pressure and in chemicals such as C-reactive protein
the weekend. Members think about, anticipate, and look forward to
eventually take a toll on the heart and other parts of the body.
Fridays so that they can get away from their work. Most members
“The data that negative mental states cause heart problems is just are highly stressed because only two days of the week are considered
stupendous,” Raison says. “The data is just as established as smoking, “good days.” Even Sunday is considered stressful because the next
and the size of the effect is the same.” day they have to go back to work.
CNN’s Sabriya Rice contributed to this report. The other business club is substantially smaller, yet in some
(Condensed from: CNN International ways the members are more dedicated to the club. This one is called
Available at: <http://edition.cnn.com/2011/HEALTH/08/17/bitter.resentful.ep/ “TGIM,” or “Thank God It’s Monday.” These members are usually
index.html?hpt=hp_c2>. Access: August 18th, 2011.)
workaholics who can’t stand weekends because they are away from
work! Members of this club are also highly stressed because while
In the text, the sentence “If I hadn’t forgiven them, I’d be dead, [...]”
there are generally five days of the week to be preoccupied with work,
(ref. 2) means that:
there is always that darn weekend that gets in the way! The most
a) Benton forgave the other boys because he didn’t want to die. difficult day of the week is usually Friday, because it often means the
b) Benton forgave the boys. As a result, he was healthy again. member won’t be able to get back to work for a few days. They may
c) after the boys who harassed him died, Benton forgave them. try to work on weekends, but the demands of family get in the way.
Needless to say, members of both clubs think members of the “other
d) before Benton died, he forgave the boys who harassed him. club” are completely nuts!

PROMILITARES.COM.BR 299
CONDITIONAL SENTENCES

I invite you to join an alternate club. My hope is that together, we 06. (FMABC-SP) If they had started earlier, they _____ in time.
can eventually achieve a 100 percent membership. In fact, I’d love to a) have arrived d) will arrive
put the other two clubs out of business altogether! This new club is
called “TGIT,” or “Thank God It’s Today.” Members of this club are b) arrived e) would arrive
happy seven days a week because they understand that every day is c) would have arrived
unique, and each brings with it different gifts. Members of this club
are grateful to be alive; they rejoice in their many blessings and expect 07. (FCM SANTA CASA-SP) If I _____ you, I _____ to him seriously.
each day to be full of wonder, surprise, and opportunity. a) were – would talk d) would be – talked
There are no qualifications necessary to join the “TGIT” club, b) were – will talk e) will be – talk
other than the desire to have a higher quality of life and the desire
to appreciate rather than dread each day. Members of this club c) was – will talk
understand that it’s useless to wish any day were different. They know
that Mondays don’t care if you like them or not – they simply go on 08. (EEAR) Select the alternative that completes the sentence below
being Mondays. Likewise, Fridays will come around every seventh day, in the third conditional.
___(I)___. It’s up to each of us to make every day as special as it can “If she hadn’t read the news magazine, she ________________.”
be. No amount of wishing will make the slightest bit of difference.
a) hadn’t seen the ad c) couldn’t see the ad
Don’t sweat the small stuff
b) would see the ad d) wouldn’t have seen the ad
(By Richard Carlson)
09. (EFOMM) Say if the sentences below are C (correct) or I (incorrect).
The sentence that properly fills in blank (I) in the text is:
( ) I wasn’t tired last night. If I were tired, I would have gone home.
a) if you don’t wish to be Friday.
( ) You’d be surprised if I told you how much it costs.
b) whether you wish it were Friday or not.
( ) I’d be able to visit Monica in the afternoon if I stay in Santiago
c) if you wish to be Friday or not. overnight.
d) whether or not you wish they were Friday. ( ) If Carrie had been honest, she would return the money.
e) if you are on a Friday or not. The correct sequence is:
a) ( I ) ( I ) ( C ) ( I ) d) ( I ) ( C ) ( I ) ( I )
b) ( I ) ( I ) ( I ) ( C ) e) ( C ) ( I ) ( I ) ( C )
EXERCÍCIOS DE
c) (I)(C)(C)(I)

COMBATE 10. (EN) Analyze these sentences.


I. Would you still love me if I hurt you?
01. (FCC-SP) They will go out if it _____ not rain. II. I wouldn’t travel alone if I were you.
a) will d) does III. Even if she would study, she didn’t pass.
b) would e) did IV. If you would stop smoking, you’d be able to run the marathon.
c) may V. She wouldn’t eat broccoli even if you’d cover it with cheese.
Choose the correct option.
02. (CFOE) Choose the item that completes the verb sequence of the a) Only I and II are grammatically correct.
sentence.
b) Only II and III are grammatically correct.
“If I win the lottery I______________ around the world.”
c) Only II and IV are grammatically correct.
a) to travel c) will travel
d) Only I and IV are grammatically correct.
b) traveled d) have traveled
e) Only II and V are grammatically correct.

03. (EEAR) The structure of the conditional clause in “… if you think


you can’t, you won’t.” is the same as in:
a) If you had overcome problems, you could have lived happily.
GABARITO
b) If you had kept on dreaming, your dreams would have come true.
c) If you don’t believe in yourself, you will not accomplish your EXERCÍCIOS DE FIXAÇÃO
mission in life. 01. E 04. D 07. D 10. C
d) If you had good feelings about the things you do, you would 02. A 05. D 08. D
succeed in your aims.
03. B 06. D 09. B
04. (EN) They could have stopped him if they ________ to. EXERCÍCIOS DE TREINAMENTO
a) want d) will want 01. C 04. E 07. B 10. B
b) were wanting e) had wanted 02. C 05. E 08. B
c) have wanted 03. A 06. D 09. E
EXERCÍCIOS DE COMBATE
05. (OSEC-SP) What would you do if _______ ? 01. D 04. E 07. A 10. A
a) the car breaks down. d) you see him at school. 02. C 05. B 08. D
b) she were here now. e) Marisa calls you. 03. C 06. C 09. D
c) the patient gets worse.

300 PROMILITARES.COM.BR
REPORTED SPEECH

O Reported Speech (discurso indireto) é uma estrutura utilizada Exemplos:


para reproduzir algo que tenha sido dito pela(s) própria(s) pessoa(s)
ou por outra(s). DIRECT SPEECH REPORTED SPEECH

TEMPOS VERBAIS NA TROCA DO Mary said, “I would like you Mary said (that) she would like
DISCURSO DIRETO PARA O INDIRETO to help us.” me to help the them.

NO REPORTED SPEECH Mary said, “These pages Mary said (that) those pages
DISCURSO DIRETO
MUDA PARA need work.” needed work.

Simple Present Simple Past Mary said, “We need the Mary said (that) they needed the
pages now.” pages then.
Present Continuous Past Continuous
Mary said (that) they would
Present Perfect Past Perfect Mary said, “We will need the
need the other pages the
other pages tomorrow.”
following day.
Simple Past Past Perfect
Mary said, “You should bring Mary said (that) I should take
Past Continuous Past Perfect Continuous
them here when you come.” them there when I went.
Past Perfect Past Perfect

Simple Future Simple Conditional PRINCIPAIS MUDANÇAS OCORRIDAS NA PASSAGEM DO


DISCURSO DIRETO PARA O REPORTED SPEECH
Simple Condtional Simple Conditional
I He / She
Exemplos: We They

DIRECT SPEECH REPORTED SPEECH You I / We

Mary said, “Pamela is at Mary said (that) Pamela was at Me Him / Her
work.” work.
My His / Her
Mary mentioned, “Pamela is Mary mentioned (that) Pamela
finishing a big project.” was finishing a big project. This That

Mary added, “She is going to Mary added (that) she was going These Those
stay until she finishes.” to stay until she finished.
Here There
Mary said, “Pamela hasn’t Mary said (that) Pamela hadn’t
Now Then
finished the project yet.” finished the project yet.
Ago Before
Mary remarked, “She Mary remarked (that) she had
was working on it all day been working (was working) on Today That day
yesterday.” it all day yesterday.
Yesterday The previous day / Day before
Mary told us, “She came Mary told us (that) she had come
home very late yesterday.” (came) home very late yesterday. Tomorrow The next / Following day

Next week The following week


REPORTED SPEECH E OS VERBOS E OS “MODALS”
Tonight That night
Can Could
Last Before
May Might - quando expressa possibilidade
Come Go
May Could - quando expressa permissão
Bring Take
Will Would

Must Had to - quando expressa necessidade

PROMILITARES.COM.BR 301
REPORTED SPEECH

Exemplos:
Quando o verbo no discurso direto estiver no imperativo, no
discurso indireto muda para o infinitivo ou podemos usar o modal
DIRECT SPEECH REPORTED SPEECH
“should”.
Susan says, “Chris is working Susan says (that) Chris is
today.” working today. Exemplos:
Susan said (that) Chris had
Susan says, “Chris was DIRECT SPEECH REPORTED SPEECH
been working ( was working )
working when I saw her.”
when she saw her. She said that we should finish
She said, “Finish your work.”
our work.
Casos Especiais:
She said to finish our work.
No “reported speech” quando o verbo da oração principal estiver
no passado como: “Susan said, etc” ele deve ser mantido no She told us, “Finish your She told us that we should finish
passado no “indirect speech”. work.” our work.

She told us to finish our work.


Exemplos:

DIRECT SPEECH REPORTED SPEECH Perguntas ou questões no discurso direto parecem com as
expressões “if” ou “whether” (se) no discurso indireto.
Susan said, “Ann can help Susan said (that) Ann could
Chris.” help Chris.
Exemplos:
Susan said, “John may help, Susan said (that) John might
too.” help, too. DIRECT SPEECH REPORTED SPEECH
Susan added, “You may help if Susan added (that) I could help She asked (me), “Could I use She asked (me) if she could use
you want.” if I wanted. your book?” my book.
Susan mentioned, “Jami will Susan mentioned (that) Jami She asked (me) to use my
help Chris, too.” would help Chris, too. book.
Susan added, “We all must Susan added (that) we all had She asked (me), “Will you help She asked (me) if I would help
help Chris.” to (must) help Chris. me?” her.

She asked (me) to help her.


No “reported speech”, não ocorre mudança quando o verbo da
oração principal estiver no “simple present” indicando fato ou She asked John, “Do you take She asked him if he took
verdade científica. a bus?” (takes) a bus.

I asked (her), “Should I do the I asked (her) if I should do the


Exemplos: assignment again?” assignment again.

DIRECT SPEECH REPORTED SPEECH I asked (her) whether (or not)


I should do it again.
Susan said, “Chris always Susan said (that) Chris always
I asked (her) , “Which pages I asked (her) which pages I
works on Saturdays.” works (worked) on Saturdays.
should I study?” should study.

Susan’s professor stated, John asked Paul, “What are John asked Paul what he was
Susan’s professor stated (that)
“Water freezes at 32 ºF ( 0 you going to do tomorrow?” going to do the following day.
water freezes at 32 ºF ( 0 ºC ).
ºC ).”

Outros verbos podem aparecer nas transformações de


Quando a frase no discurso direto não apresentar a ideia de discurso direto para discurso indireto, como: admit,
tempo, ou seja, a ideia é uma verdade aparente ou uma realidade, announce, explain, mention, notice, promise, realize, reply,
podemos usar no discurso indireto o mesmo tempo verbal do recommend, suggest e warn.
discurso direto.

Exemplos:
DIRECT SPEECH REPORTED SPEECH
DIRECT SPEECH REPORTED SPEECH
Mary said, “Pamela should be Mary said (that) Pamela should
home soon.” be home soon. He said: “Don’t play with He warned me not to play
fire!” with fire.
Mary said, “She must be Mary said (that) she must be
tired.” tired. She said: “I will give him ten
She promised him ten dollars.
dollars."
Mary said, “Pamela must have Mary said (that) Pamela must
left early.” have left early.

302 PROMILITARES.COM.BR
REPORTED SPEECH

EXERCÍCIOS DE 06. (UFRGS 2012)

FIXAÇÃO
01. (ESPCEX/AMAN 2019) What is the question the author refers to
when he says: “...I was asked if I’d like to have a look at Bert Krages’
book.”?
a) Did you like to have a look at Bert Krages’ book?
b) Should you like to have a look at Bert Krages’ book?
c) Need you like to have a look at Bert Krages’ book?
d) Would you like to have a look at Bert Krages’ book?
e) Do you like to have a look at Bert Krages’ book?

02. (EPCAR/AFA 2019) Mark the alternative which has the sentence
below correctly reported.
“[…] is our fascination with fantastic fiends healthy?”
The author:
a) replied: “is our fascination with fantastic fiends healthy?” Consider the following sentence and the three alternatives to
b) said that their fascination with fantastic fiends had been healthy. complete it.
c) told the readers their fascination with fantastic fiends has been healthy. Calvin said, “I will never teach maths”. In the indirect speech this becomes:
d) asked if people’s fascination with fantastic fiends was healthy. 1. Calvin said that he would never teach maths.
2. Calvin said that he is never going to teach maths.
03. (EPCAR/AFA 2017) Choose the option that shows the indirect
3. Calvin said that he was never going to teach maths.
speech form for “These distinctions are consequential”.
Gardner: Which of the alternatives above can be considered grammatically correct?
a) said that those distinctions were consequential. a) Only 1. d) Only 2 and 3.
b) told these distinctions are consequential. b) Only 2. e) 1, 2 and 3.
c) said us these distinctions were consequential. c) Only 1 and 2.
d) told those distinctions are consequential.
07. (EPCAR/AFA 2012)
04. (EPCAR/AFA 2013) In the question “Why does the fight between LEAVE OUT ALL THE REST
two simultaneously active language systems improve these aspects of Linking Park - Soundtrack of Twilight
cognition?” The author asked:
I dreamed I was missing
a) if the fight between two simultaneously active language systems
You were so scared
had improved these aspects of cognition.
But no one would listen
b) why does the fight between two simultaneously active language ‘Cause no one else cared
systems improved those aspects of cognition?
c) why the fight between two simultaneously active language After my dreaming
systems improved those aspects of cognition. I woke with this fear
d) if the fight between two simultaneously active language systems What am I leaving
improve these aspects of cognition? When I’m done here
[…]
05. (MACKENZIE 2013) The sentence “Gates would often ask women
(Chorus)
at remote clinics what else they needed” in the direct speech is
When my time comes
a) Gates will ask to the women at remote clinics “What else have Forget the wrong that I’ve done
you needed?” Help me leave behind some
b) “What else did you need?”, Gates asked women at remote clinics. Reasons to be missed
[…]
c) Gates often asked to the women at remote clinics: “What else
does she need?”
Don’t be afraid
d) “What else do you need?”, Gates usually asked women at remote I’ve taken my beating
clinics. I’ve shared what I made
e) Gates occasionally questioned the women at remote clinics about […]
what they needed.
Pretending
Someone else can come and save me from myself
I can’t be who you are

PROMILITARES.COM.BR 303
REPORTED SPEECH

Read the chorus of the song and choose the correct alternative. 09. (UEMG 2014)
The singer ________ ________ the wrong ________. THE MAN WHO SOLD THE EIFFEL TOWER
a) asked – to forget – he’d done Paris, 1925. World War I had finished and the city was full of
people with cash looking for business opportunities. Victor Lustig
b) said – forget – I did was reading the newspaper one day and found an article about the
c) advised – forgetting – I’ve done Eiffel Tower. It said the tower was being neglected because it was
d) told – not to forget – they’ve done too expensive to maintain. Lustig a great ‘business opportunity’ – he
would sell the Eiffel Tower!
08. (PUC-RS 2016) Answer the question in relation to the text bellow. Lustig wrote to six important businessmen in the city and invited
UNITED NATIONS, May 11, 2015 (IPS/GIN) – Speaking at the them to a secret meeting in a well-known Paris hotel. He said he was
U.N. Security Council, Federica Mogherini, High Representative of a government official and he told them that he wanted to talk about a
the European Union for Foreign Affairs, called on the international business deal. All six of the businessmen came to the meeting.
community to take urgent steps to end the Mediterranean crisis and
dismantle the human smuggling rings that facilitate it. At the meeting, Lustig told them that the
city wanted to sell the Eiffel Tower for scrap
“The EU is united and we will work, but we cannot work alone. metal and that he had been asked to find a
We need to share and act together, as it’s a EU responsibility and a buyer. He said that the deal was secret because
global responsibility,“ said Mogherini. it would not be popular with the public. The
In 2014, 3,300 migrants died while 1fleeing their countries businessmen believed him, perhaps the Eiffel
of origin to enter Europe. Three people out of four perished in the Tower was never planned to be permanent. It
Mediterranean Sea, and 2015 looks set to be even worse, added had been built as part of the 1889 Paris Expo,
Mogherini. and the original plan had been to remove it in
2
According to the U.N. Refugee Agency (UNHCR) about 60,000 1909.
men, women and children have crossed the Mediterranean this year, Lustig rented a limousine and took the men
and 1,800 of them have tragically died during the journey. to visit the tower. After the tour, he said that if
“Saving lives and preventing the loss of lives at sea is a top they were interested, they should contact him
responsibility that we all share, not only as Europeans but globally,” the next day. Lustig told them he would give the tower contract to
Mogherini said at the Council 3briefing, adding that an exceptional the person with the highest offer. One of the dealers, Andre Poisson,
situation requires an immediate strategy to solve the crisis. was very interested, but he was also worried. Why was Lustig in such
a hurry?
The Mediterranean problem is a structural problem rooted in
poverty, increasing inequality, conflicts and human rights violations The two men had a meeting, and Lustig confessed that he wasn’t
in African and Middle Eastern countries and beyond, including looking for the highest offer. He said he would give the contract to
the situation in Syria, Afghanistan and the Horn of Africa, said the anybody – for a price. Poisson understood: Lustig wanted a little extra
European High Representative. money “under the table” for himself. This was Lustig’s cleverest lie,
because now Poisson believed him completely.
Also speaking at the Council was Antonio Tete, Permanent
Representative Observer of the African Union to the U.N., who Lustig sold Poisson a false contract for the Eiffel Tower – and on
underlined that 4smuggling of migrants has emerged due to several top of that, Poisson paid him a little extra money “under the table”.
factors that lead people in many African countries to escape from Lustig put all the money in a suitcase and took the first train to
abject poverty, climate change, water scarcity, insufficient progress in Vienna. Poisson never told the police what had happened – he was
employment and rising inequality. too embarrassed. After a month, Lustig returned to Paris and tried to
sell the Eiffel Tower again, but this time somebody told the police and
“This humanitarian emergency is also a security crisis, since
he had to escape to America. There, he continued his criminal career
smuggling networks are linked to finance and terrorist activities,
and finished his days in the famous Alcatraz prison.
which contributes to instability in a region that is already unstable
(Oxford UP 2009 - English Result, p.62. Adapted.)
enough,” Mogherini said.
5
If the international community fails to frame its response to the Read the reported sentence below, from the text.
crisis, it will be a “moral failure,” said Peter Sutherland, the Special
“Lustig told them he would give the tower contract to the person with
Representative of the Secretary-General for International Migration.
the highest offer.”
(Ieri, Valentina. Global Information Network [New York] 11 May 2015.)
Which of the alternatives below corresponds to Lustig’s direct speech?
To answer the question, complete the rephrased speech of Peter a) “I will give the tower contract to the person with the highest offer”.
Sutherland (ref. 5), using the indirect speech. b) “I would give the tower contract to the person with the highest offer”.
“Peter Sutherland said that __________ the international community c) “I shall give the tower contract to the person with the highest offer”.
__________ to frame its response to the crisis, it __________ a ‘moral
failure’”. d) “I could give the tower contract to the person with the highest offer”.
The alternative that fills in the blanks of the text above correctly is:
10. (EEAR 2018) The correct form of the sentence “Flight operations to
a) unless – fails – wouldn’t be and from Chennai have been affected due to heavy rains – announced
b) unless – failed – will be IndiGo Airlines”, in the indirect speech is:
c) if – fails – would be a) IndiGo Airlines announced that flight operations to and from
d) if – failed – will be Chennai would be affected due to heavy rains.
e) if – failed – would be b) IndiGo Airlines announced that flight operations to and from
Chennai had been affected due to heavy rains.
c) IndiGo Airlines announced that flight operations to and from
Chennai will be affected due to heavy rains.
d) IndiGo Airlines announced that flight operations to and from
Chennai was affected due to heavy rains.

304 PROMILITARES.COM.BR
REPORTED SPEECH

EXERCÍCIOS DE 06. (ESPM 2012)

TREINAMENTO
01. (EEAR 2015) The mother said to the son:
– Do not watch TV after midnight!
a) The mother told her son to does not watch TV after midnight.
b) The mother told her son do not to watch TV after midnight.
Turning the first frame of the comic strip into the Reported Speech,
c) The mother told her son to not watch TV after midnight. we would have:
d) The mother told her son not to watch TV after midnight. a) Roosevelt once said to do what you could with what you had
where you were.
02. (EN 2014) Which alternative is correct?
b) Roosevelt once said that to do what you could with what you had
a) The teacher said the student do not eat in class.
where you were.
b) The teacher told the student not to eat in class.
c) Roosevelt once said to do what we could with what we had
c) The teacher said to the student to not to eat in class. where we were.
d) The teacher told to the student not to eat in class. d) Roosevelt once said did what we could with what we had where
e) The teacher told the student to do not eat in class. we were.
e) Roosevelt once said do what we could with what we had where
03. (EN 2012) Which alternative best reports the stranger's speech in we were.
this passage?
In 1953, I got married. A few weeks after the wedding, I suddenly 07. (EPCAR/AFA 2011)
fell ill. My husband took me to a hospital. I was there for almost a THE LION KING
week. I was in so much pain. And no one could say for sure what was This article is about Disney's 1994 film.
wrong. One night, in the hospital, a stranger came to see me. He told
me, "Janie, you’re going to die tomorrow". That was my name then, 1
The Lion King is a 1994 American animated feature produced
the name I was born with. by Walt Disney Feature Animation. 2Released to theaters on June 15,
(Adapted from http://www.notmyshoes.net/monologues/ hannah-march.html) 1994 by Walt Disney Pictures, it is the 32nd film in the Walt Disney
Animated Classics. 3The story, which was influenced by the Bible stories
a) He told her she was going to die the day after. of Joseph and Moses and the William Shakespeare play Hamlet, takes
b) He said to her she was going to die tomorrow. place in a kingdom of anthropomorphic animals in Africa. 4The film
c) He told she is going to die the day after. was the highest grossing animated film of all time until the release
of Finding Nemo. 5The Lion King still holds the record as the highest
d) He said to her she is going to die tomorrow.
grossing traditionally animated film in history and belongs to an era
e) He told her she was going to die tomorrow. known as the Disney Renaissance.
The Lion King is the highest grossing 2D animated film of all time
04. (UFRGS 2014) Considere o segmento a seguir.
in the United States, 6and received positive reviews from critics, who
THIS IS OUR HOPE. THIS IS THE FAITH THAT praised the film for 8its music and story. During its release in 1994, the
I GO BACK TO THE SOUTH WITH. film grossed more than $783 million worldwide, becoming the most
Assinale a alternativa que apresenta a reescrita mais adequada do successful film released that year, 7and it is currently the twenty-eighth
segmento acima, em discurso indireto. highest-grossing feature film.
a) This was my hope. This was the faith that I would go back to the (http://en.wikipedia.org/wiki/The_Lion_King)
South with.
Glossary:
b) That was their hope. That was the faith that he would go back to Feature film – a film that is 90 or more minutes long
the South with. Gross – total
Release – make public
c) That has been their hope. That has been the faith that they have Praise – show approval
gone back to the South with.
d) That was our hope. That was the faith with which we went back The sentences below taken from the text were changed into Indirect
to the South. Speech. Mark the one that was changed correctly.
e) Those were their hopes. Those were the faiths with which they a) The author quoted that the film is being the highest grossing
went back to the South. animated film of all time. (ref. 4)
b) The text mentioned that The Lion King still held the record as the
05. (UFG 2014) The excerpt “The young inventor's dream is to attend highest grossing traditionally animated film in history. (ref. 5)
medical school in the US and continue with projects concerning the
c) The text assumed that the movie was receiving positive reviews
environment” is written in direct speech as:
from critics, who praised the film for its music and story. (ref. 6)
a) “If only I had attended medical school in the US and continued
d) The movie has been currently the twenty-eighth highest-grossing
with projects concerning the environment.”
feature film. (ref. 7)
b) “I wish I had attended medical school in the US and continued
with projects concerning the environment.”
08. (EN 2017) Which is the correct option to complete the sentence?
c) “I wished I attended medical school in the US and continued with
projects concerning the environment.” “Peter: I saw Jane yesterday.”

d) “I hope I attend medical school in the US and continue with “Peter ____________________.”
projects concerning the environment.” a) said Jane that he saw her yesterday
e) “I would like to have attended medical school in the US and b) told he saw Jane the previous day
continued with projects concerning the environment.”

PROMILITARES.COM.BR 305
REPORTED SPEECH

c) says he had seen Jane the following day b) The text said us that good friends encourage their friends to seek
d) told Jane that he has seen her the next day help and access services.
e) said that he had seen Jane the day before c) The text told that good friends encourage their friends to seek
help and access services.
09. (AFA 2009) The Direct Speech of the two sentences "...I'd missed d) The text said that good friends encouraged their friends to seek
the train." and "... I was sorry..." is: help and access services.
a) "I missed the train" and "I am sorry".
06. (FMU/FIAN-SP) Then Mary asked, “How far away is the nearest
b) "I miss the train" and "I was sorry". post office?”
c) "I have been missing the train" and "I was sorry". a) She asked how far away were the nearest post office.
d) "I would miss the train" and "I have been sorry". b) She asked how far away the nearest post office was.
c) She asked how far away is the nearest post office.
10. (EEAR 2010) The indirect speech for “I was challenged by the
language barrier, he said.” is: d) She asked how far was away the nearest post office.
a) He said he is being challenged by the language barrier. e) She asked how far is away the nearest post office.
b) He said that he had been challenged by the language barrier.
07. (UFAC) The sentence “Martina asked me: ‘Will you do me a
c) He told us that he has been challenged by the language barrier. favor?'”, in the reported speech form, becomes:
d) He asked us if he was being challenged by the language barrier. a) Martina asked me if was to do a favor for her.
b) Martina asked me if she would do me a favor.
c) Martina asked me if I would do her a favor.
EXERCÍCIOS DE
d) Martina asked me if she will do me a favor.

COMBATE 08. (UEL-PR) Mother: John, bring this package over here.
Mother told John ___(I)___ ___(II)___ package over ___(III)___.
01. (EEAR) The Direct Speech for “people ask me if I prefer living in a) brings / hers / over here d) to take / that / there
France or the USA” is: b) takes / that / now e) take / his / today
a) Do you prefer living in France or the USA? c) bring / those / over there
b) Did you prefer living in France or the USA?
09. (AFA) My mother said to me: Don’t leave your little brother alone!
c) Will you prefer living in France or the USA?
The reported speech of the above sentence is:
d) Would you prefer living in France or the USA?
a) My mother told my little brother not to be left alone.
02. (UFGO) Mark the correct alternative which is the indirect speech of: b) My mother told me don’t leave my little brother alone.
He said, “I will never speak to her again.” c) My mother said me to not leave my little brother alone.
a) He said that he will never speak to her again. d) My mother told me not to leave my little brother alone.
b) He said that he was never going to speak to her again.
10. (EN) Which is the correct option to complete the dialogue?
c) He said that he is never going to speak to her again.
- What did John tell Mary last Saturday?
d) He said that he would never speak to her again.
- John told __________________ the day before.
e) He said that he never speaks to her again.
a) Mary that he will buy some flowers
03. (AFA) Choose the option that shows the indirect speech form for b) her that he had bought some flowers
“These distinctions are consequential.” c) him that he did buy some flowers
Gardner:
d) to Mary that he bought some flowers
a) said that those distinctions were consequential.
e) that he has to buy some flowers
b) told these distinctions are consequential.
c) said us these distinctions were consequential. GABARITO
d) told those distinctions are consequential. EXERCÍCIOS DE FIXAÇÃO
01. D 04. C 07. A 10. B
04. Which is the correct alternative to complete the dialogue?
02. D 05. D 08. E
Susan: What did George ask you yesterday?
03. A 06. A 09. A
Sandy: He asked me ______________________.
EXERCÍCIOS DE TREINAMENTO
a) do I need a ride home d) I needed a ride home
01. D 04. B 07. B 10. B
b) did I need a ride home e) would I need a ride home
02. B 05. D 08. E
c) if I needed a ride home
03. A 06. C 09. A
05. (AFA) Choose the option that shows the sentence “good friends EXERCÍCIOS DE COMBATE
encourage their friends to seek help and access services” in the 01. A 04. C 07. C 10. B
indirect speech form. 02. D 05. D 08. D
a) The text told good friends encourage their friends to seek help 03. A 06. B 09. D
and access services.

306 PROMILITARES.COM.BR
QUESTION TAG

Questions tags são pequenas frases acrescentadas no fim de EXERCÍCIOS DE


um período, cujo objetivo é obter uma confirmação ao que foi dito
anteriormente.
Exemplos:
FIXAÇÃO
It’s a wonderful city, isn’t it?
01. (EPCAR/CPCAR 2018) Mark the option that shows the appropriate
You don’t take sugar in your tea, do you?
question tag for the sentence.
“He's right there in the back seat, __________?”
• Quando a sentença é afirmativa, o “question tag” é negativo.
Neste caso, geralmente se espera uma resposta afirmativa. a) isn't he
Exemplos: b) hasn't he
Paulo will be here soon, won’t he? c) isn't there
Luciano should pass his exam, shouldn’t he? d) is he

• Quando a sentença é negativa, o “question tag” é afirmativo. 02. (EPCAR/CPCAR 2016) Mark the option to complete the sentence
Neste caso, geralmente se espera uma resposta negativa. with the correct tag question form.
Exemplos: “They bring together the parents of victims, __________”
You haven’t seen João today, have you? a) do they?
You didn’t go to EFOMM last week, did you? b) are they?
c) don't they?
• Quando a sentença está com o verbo no imperativo, forma-se o d) aren’t they?
“question tag” com a frase “will you”.
Exemplos: 03. (FUVEST 1998) Escolha a "question tag” correta para:
Close the door, will you? "I knew I would be a scientist."
Don’t say that again, will you? a) didn't I? d) don't I?
b) wasn't I? e) would I?
• Quando a primeira sentença apresentar a expressão “Let’s”, c) won't I?
forma-se o “question tag” com a frase “Shall we”.
Exemplos: 04. (UNESP 1988) Assinale a alternativa correta.
Let’s start now, shall we? “Politics is a science, ______?”
Let’s call the doctor, shall we? a) weren't they
b) isn't it
• Quando a sentença iniciar com o sujeito “I” com o verbo “to be” c) wouldn't he
e estiver na afirmativa, forma-se o “question tag” com a frase
“aren’t I”. d) wasn't it
Exemplos: e) won't they
I’m your teacher, aren’t I? = (am I not)
05. Choose the correct answer.
I’m working, aren’t I? = (am I not)
“They were cleaning the room when I arrived, _____ ?”

• Quando a sentença iniciar com os sujeitos “Everyone”, “Nobody” a) did they d) weren't they
etc., forma-se o “question tag” com o pronome “they”. b) were they e) were they not
Exemplos: c) didn't they
Everyone had a good time, didn’t they?
06. (FEI 1997) Complete:
Nobody will be late, will they?
“He'll be back soon, __________ ?”
Observação a) will he d) won't he

“Nobody” é uma expressão com ideia negativa, logo o “question b) doesn't he e) couldn't he
tag” passa ser afirmativo. c) shall he

PROMILITARES.COM.BR 307
QUESTION TAG

07. Choose the correct answer. 03. (EEAR 2017) Look at the following statements and choose the
“Jimmy has left his book at school, __________ ?” correct question tags:
a) has he 1 – “It snowed last night, _______________?”
b) hasn't he 2 – “She shouldn’t be aggressive to people, _____________?”
c) doesn't he 3 – “You haven’t closed the door,_________________?”
d) didn't he 4 – “You are going to the party with us, ____________?”
e) hasn't Jimmy a) doesn’t it – shouldn’t she – aren’t you – going you
b) didn’t it – should she – have you – aren’t you
08. (AFA 2009) Mark the option which contains the correct Question
Tag of the following sentence. c) did it – should she – haven’t you – aren’t you
"She has not yet demonstrated a link between increased brainwaves d) didn’t it – ought to – have you – will you
and suicidal tendencies, ________ "
a) hasn't she? 04. (EN 2013) Which is the correct alternative to complete this sentence?

b) has she? "There's been a rumor that Jane is seeing someone from the office,
________ ?"
c) does she?
a) is there
d) doesn't she?
b) isn't there
09. (EEAR 2016) Choose the alternative that completes the sentence c) has there
correctly. d) hasn't there
“You wouldn’t believe him, __________?” e) doesn't it
a) do you
b) don’t you 05. (EEAR 2015) Complete the statement with the correct question
tag:
c) would you
“Flight 9525, an Airbus A320, took off at 10:01 A.M. on March 24
d) wouldn’t you from Barcelona, ______?”
a) won’t it
10. (EN 2012) Which is the correct option to complete the sentence
below? b) didn’t it
“He's done a lot of work today, ____ ?” c) wasn’t it
a) hasn't he? d) doesn’t it
b) does he?
06. (EN 2011) Which of the alternatives below completes the sentence
c) isn't he? correctly?
d) doesn't he? “Let's go to the mall, _____?”
e) is he? a) will we
b) won't we
EXERCÍCIOS DE c) shall we

TREINAMENTO d) don't we
e) let we

07. (EN 2017 ADAPTADA) Which of the options completes the


01. (EPCAR/AFA 2013) One extracted fragment has its correct Tag dialogue correctly?
Question. Mark the item. THE LINDEN TREE
a) The bilingual experience appears to influence the brain from Mrs Linden: (...) You’d like some tea, ______ you, Rex?
infancy to old age, don’t they?
Rex: A cup, certainly.
b) Bilingualism’s effects also extend into the twilight years, has it?
Jean: And so would Marion and I.
c) These processes include ignoring distractions to stay focused,
(Priestley, J.B. “The Linden Tree". An inspector calls and other plays.
aren’t they? UK: Penguin, 2001.)
d) Nobody ever doubted the power of language, did they?
a) would d) hadn’t
02. (UDESC 1997) Complete with the CORRECT alternative: b) wouldn’t e) didn’t
“The sun shone the whole day, __________ it?” c) had
a) is
b) did 08. (EFOMM 2011) Choose the option with the correct tag questions
for the sentences below.
c) doesn't
1. You weren’t listening, ____________________?
d) didn't
2. She doesn’t know him, __________________?
e) isn't
3. I’m a bit overweight, _____________________?
4. Don’t open your eyes, ____________________?

308 PROMILITARES.COM.BR
QUESTION TAG

a) weren’t you / does she / aren’t I / do you 04. (EN) Which of the options completes the sentence correctly?
b) were you / doesn’t she / aren’t I / do you “Peter's got blue eyes, _____?”
c) were you / does she / aren’t I / will you a) isn't he
d) weren’t you / doesn’t she / am I not / do you b) does Peter
e) were you / doesn’t she / am I not / do you c) doesn't he
d) has Peter
09. (EFOMM 2010) Choose the option with the correct tag questions e) hasn't he
for the sentences below.
1) Come here _________________? 05. (EN) Which of the options completes the sentence correctly?
2) Let’s talk, _______________? “Mary has a brother, ______”
3) I’m late, ________________? a) does Mary?
4) Don’t close your eyes, _______________? b) doesn't she?
a) won’t you / won’t we / aren’t / do you c) has she?
b) won’t you / shall we / aren’t I / do you d) hasn't Mary?
c) will you / shall we / aren’t I / will you e) does he?
d) will you / won’t we / am I not / will you
06. (AFA) The question tag is CORRECT in the following sentences:
e) will you / shan’t we / am I not / do you
a) “Let’s have dinner now, haven’t us?”
10. (EFOMM 2012) Choose the option with the correct tag questions b) “You’d better careful, hadn’t you?”
for the sentences below c) “You’ll have to pay for this, haven’t you?”
I. Let’s start the presentation, ___________? d) “Nothing is wrong, aren’t they?”
II. Nobody phoned, __________?
III. Don’t open your books, ______________? 07. (AFA) “Cabral _________ Brazil many years ago and his discovery
opened new gates for Portugal, __________?”
IV. There is a nice restaurant near here, ______________?
a) discovered / didn’t it
a) won’t we / did they / do you / is there b) has discovered / did it
b) will we / didn’t they / don’t you / isn’t there c) had discovered / didn’t him
c) shall we / did they / do you / isn’t there d) has been discovered / did they
d) shall we / did they / will you / isn’t there
e) will we / didn’t they / will you / is there 08. (AFA) Choose the correct questions for the context:
You are speaking to your daughter. You want to make sure that she
turned off the stove. You ask her:
a) “You turned off the stove, did you?”
EXERCÍCIOS DE

COMBATE
b) “You do turn off the stove, didn’t you?”
c) “You didn’t turn off the stove, did you?”
d) “You did turn off the stove, didn’t you?”

01. (COMD AERONÁUTICA) “Kathrin is a famous top model in 09. (EFOMM) The question tags in the sentences below are correct,
Australia, ______?” except in:
a) isn’t she a) “It’s no good, isn’t it?”
b) isn’t Kathrin b) “Give me a hand, will you?”
c) is not c) “Let’s have a party, shall we?”
d) not is d) “Open a window, would you?”
e) “Shut up, can’t you?”
02. (AFA) “We can’t do without him, __________?”
a) can us 10. (AFA) Read the following passage and, then, mark the alternative
b) can we which contains the appropriate question tags to complete the gaps.
c) can ours “You think I’m nuts, __________? You’ve just had enough of my
voice, _________?
d) can’t he
I’m such a hindrance in your life, _________? Let’s make a last deal,
_________? Leave now and never look back over your shoulder,
03. (AFA) Mark the option which shows the appropriate question tag
_________?”
for the sentence:
a) don’t you / hadn’t you / am I / will we / shall you
“One unanticipated consequence has driven me to distraction”.
b) aren’t I / haven’t you / am not I / shall we / don’t you
a) Hasn’t driven it?
c) do you / have you / isn’t me / would we / won’t you
b) Not has it?
d) don’t you / haven’t you / aren’t I / shall we / will you
c) Has it?
d) Hasn’t it?

PROMILITARES.COM.BR 309
QUESTION TAG

GABARITO
EXERCÍCIOS DE FIXAÇÃO
01. A 04. B 07. B 10. A
02. C 05. D 08. B
03. A 06. D 09. C
EXERCÍCIOS DE TREINAMENTO
01. D 04. D 07. B 10. D
02. D 05. B 08. C
03. B 06. C 09. C
EXERCÍCIOS DE COMBATE
01. A 04. E 07. A 10. D
02. B 05. B 08. D
03. D 06. B 09. A

ANOTAÇÕES

310 PROMILITARES.COM.BR
CONJUNCTIONS - CONNECTIVES

CONJUNÇÕES, ADVERBIAL, Casos Especiais:


In Spite of / Despite (apesar de) geralmente introduzem frases
TRANSITIONS (phrases):
São palavras ou frases que ligam duas ideias entre si, ou seja, duas
Exemplos:
orações. O papel da conjunção é frisar a relação entre as duas orações
dando ideias de adição, conclusão, contraste, causa, propósito e etc. He was able to sleep in spite of the noise.
She entered the EFOMM despite her low grades.
Whereas, While, On The Contrary, On The Other Hand, Otherwise,
ADDITION (adição): moreover, furthermore, besides (além disso,
Conversely (enquanto que, ao passo que, por outro lado). Palavras
além do que), in addition (em adição) as well as (assim como), also,
que também expressam contraste:
not only ...but also, and (não somente... mas), both ... and (tão/ tanto
… quanto/ como). Exemplos:
Exemplos: Whereas /While While i like the south, my wife likes the north.
Jane is not only beautiful, but also intelligent.
Algumas palavras que também ligam orações:
RESULT/CONSEQUENCE/REASON (resultado, consequência, Time (tempo): after (depois), before (antes), when (quando), as
razão): thus, therefore, hence, consequently (portanto, consequente- (quando, à medida que), as soon as (assim que), until (até quando).
mente), as a result (como resultado), so, then (portanto, então), for
this reason (por essa razão), so that (de modo que). Place (lugar): where (onde), wherever (onde for).
Exemplos:
João studied hard, thus he entered to AFA. Manner (forma): as if (como se), as though, (como se), as (como).

CAUSE (causa): because, for, as (por causa) because of (por causa Condition (condição): if, provided that (contanto que, desde
de), since (uma vez que), due to (devido a). que), as long as (enquanto que, visto que), whether or not (se ou
Exemplos: não), unless (a menos que), if not (se não).
As you know Paulo better than I do, could you ask him for me? Exemplo:
He can stay here as long as he doesn’t make a noise.
PURPOSE (propósito): in order to (that) (com o propósito de), so
(that) (de modo que). Whether (dúvida): se
Exemplos: Exemplo:
Get an early night in order that you will be fresh in the morning. Sara does not know whether she will pick up Math or Chemistry.

CHOICE (escolha) either ...or, neither... nor, or (ou isso ...ou


aquilo, ou).
Exemplos: EXERCÍCIOS DE

FIXAÇÃO
We can either stay or go to the beach.
I neither smoke nor drink.

CONTRAST/CONCESSION (contraste, concessão): although,


though (embora), even though (muito embora), but, yet, however, 01. (ESPCEX/AMAN 2013)
nevertheless, nonetheless (mas, porém, entretanto, todavia), in spite APPLE MANUFACTURING PLANT WORKERS COMPLAIN
of, despite (apesar de, não obstante), still (ainda assim, mesmo assim, OF LONG HOURS AND MILITANT CULTURE
apesar de). Chengdu, China (CNN) — Miss Chen (we changed her name
Exemplos: for this story), an 18-year-old student from a village outside of the
Leo is antisocial, but the has a few friends. southern megacity of Chongqing, is one of more than one million
factory workers at a Chinese company that helps manufacture
products for Apple Inc.’s lucrative global empire, which ranked in a
record $46.3 billion in sales last quarter. They work day or night shifts,
eating and sleeping at company facilities, as they help build electronics
products for Apple and many other global brand names,such as
Amazon’s Kindleand Microsoft’s Xbox.

PROMILITARES.COM.BR 311
CONJUNCTIONS - CONNECTIVES

As a poor college student with no work experience, looking for 04. (UNIFESP 2015) Leia o texto para responder à questão.
a job in China’s competitive market is an uphill battle. So when Chen HEALTHY CHOICES
was offered a one-month position at Foxconn with promises of great
benefits and little overtime, she jumped at the chance. But when she How do we reduce waistlines in a country where we traditionally do
started working, she found out that only senior employees got such not like telling individuals what to do?
benefits. “During my first day of work, an older worker said to me, By Telegraph View 22 Aug 2014
‘Why did you come to Foxconn? Think about it again and leave right Every new piece of information about Britain’s weight problem
now’,” said Chen, who plans to return to her studies at a Chongqing makes for ever more depressing reading. Duncan Selbie, the Chief
university soon. Executive of Public Health England, today tells us that by 2034 some
[...] six million Britons will suffer from diabetes. Of course, many people
(Adaptado de http://edition.cnn.com, consulta em 06/02/2012.) develop diabetes through no fault of their own. But Mr Selbie’s
research concludes that if the levels of obesity returned to their 1994
In the sentence “But when she started working, she found out levels, 1.7 million fewer people would suffer from the condition.
that...”, the word “but” indicates Given that fighting diabetes already drains the National Health
a) addition. Service (NHS) by more than £1.5 million, or 10 percent of its budget
b) consequence. for England, the impact upon the Treasury in 20 years’ time from
unhealthy lifestyles could be catastrophic. 1Bad health not only impacts
c) result. on the individual but also on the rest of the community.
d) reason. Diagnosis of the challenge is straightforward. The tougher
e) contrast. question is what to do about reducing waistlines in a country where
we traditionally do not like telling individuals what to do.
TEXTO PARA AS QUESTÕES 02 E 03: It is interesting to note that Mr Selbie does not ascribe to the Big
PRESCRIPTIONS FOR FIGHTING EPIDEMICS Brother approach of ceaseless legislation and nannying. 2Rather, he is
keen to promote choices – making the case passionately that people
Epidemics have plagued humanity since the dawn of settled life. should be encouraged to embrace good health. One of his suggestions
Yet, success in conquering them remains patchy. Experts predict that is that parents feed their children from smaller plates. That way the
a global one that could kill more than 300 million people would come child can clear his or her plate, as ordered, without actually consuming
round in the next 20 to 40 years. What pathogen would cause it is too much. Like all good ideas, this is rooted in common sense.
anybody’s guess. Chances are that it will be a virus that lurks in birds or (www.telegraph.co.uk. Adaptado.)
mammals, or one that that has not yet hatched. The scariest are both
highly lethal and spread easily among humans. No trecho do segundo parágrafo (ref. 1), “Bad health not only impacts
Thankfully, bugs that excel at the first tend to be weak at the on the individual but also on the rest of the community”, a expressão
other. But mutations – ordinary business for germs – can change that “not only… but also” indica uma ideia de
in a blink. Moreover, when humans get too close to beasts, either wild a) negação. d) inclusão.
or packed in farms, an animal disease can become a human one. b) comparação. e) contraste.
A front-runner for global pandemics is the seasonal influenza c) alternativa.
virus, which mutates so much that a vaccine must be custom-made
every year. The Spanish flu pandemic of 1918, which killed 50 million
05. (UERJ SIMULADO 2018)
to 100 million people, was a potent version of the “swine flu” that
emerged in 2009. The H5N1 “avian flu” strain, deadly in 60% of cases, HOW TECHNOLOGY CAN EMPOWER THE ELDERLY
came about in the 1990s when a virus that sickened birds made the The elderly have often been neglected by technology developers
jump to a human. Ebola, HIV and Zika took a similar route. as a focus market. The stereotype is that they are technophobes, or
(www.economist.com, 08.02.2018. Adaptado.) at least slow to pick up new innovations. However, 1in reality not
only are the elderly very capable of using a range of complex modern
technologies, they are also very often in need of devices that can ease
02. (UNESP 2019) No trecho do segundo parágrafo “Moreover, when their lives and empower them in their range of abilities. Let’s look over
humans get too close to beasts”, o termo sublinhado indica: a few of the best examples out there.
a) acréscimo. It seems that we are currently obsessed with reducing the size
b) decorrência. of new devices to make them more and more portable. However,
c) comparação. according to researchers, most elderly people prefer to spend their
time without rushing and stressing and going from one place to the
d) condição. other, as many young people do. Many spend a great deal of time in
e) finalidade. their homes, which is often referred to as “ageing in place”. Therefore,
gadgets designed to support home living can be very useful, especially
03. (UNESP 2019) No trecho do primeiro parágrafo “Yet, success in when they are designed appropriately for the elderly. Some simple
conquering them remains patchy”, o termo sublinhado equivale, em examples include TV remote controllers, mobile phones and tablets
português, a: designed as lightweight and featuring large illuminated buttons.
TV audio amplifiers can also be very useful, as well as audiobooks
a) assim mesmo.
downloaded as MP3s or played on tablets and similar devices directly
b) portanto. from a browser or a playlist.
c) além disso. The improvements in home alarms and mobile phone security
d) ao invés disso. apps for seniors have been noticeable. There are sophisticated gadgets
e) no entanto. now available which can track activity patterns and create alerts for
carers and family or friends when there is an unexpected interrupt in
an elderly person’s routine. There are also a good range of wireless
alarm systems which can be placed around the home with ease.

312 PROMILITARES.COM.BR
CONJUNCTIONS - CONNECTIVES

For those who wander due to conditions such as Alzheimer’s or Fill in the blank with the option that best completes the text.
dementia, GPS Shoes and Smart soles are a great facility. GPS Shoes a) as soon as
update information periodically so caregivers can be informed about
b) instead of
the location of the user with frequencies ranging up to every 10
minutes. GPS Smart soles allow online tracking of a user’s location c) still
through any smartphone, tablet or browser with the login details. d) yet
A widening range of gadgets are now becoming more user
friendly, interesting and empowering for the elderly. Also, a broad 08. (EPCAR/AFA 2018)
range of gadgets are now custom-made for this market group. After
FOOD SHORTAGE CAUSES, EFFECTS AND SOLUTIONS
all, this is a segment of the population who should be respected and
should never be neglected. They brought us into this world, and we […]
will all arrive into this demographic in the end. Causes of food shortages
(psychcentral.com)
There are a number of social factors causing food shortages. The
rate of population increase is higher than increase in food production.
“In reality not only are the elderly very capable of using a range of
The world is consuming more than it is producing, leading to decline in
complex modern technologies,” (ref. 1)
food stock and storage level and increased food prices due to 2soaring
The underlined expression is used in the sentence to introduce an idea of: demand. Increased population has led to clearing of agricultural land
a) doubt for human settlement reducing agricultural production (Kamdor,
b) contrast 2007). 3Overcrowding of population in a given place results in
urbanization of previously rich agricultural fields. Destruction of
c) addition forests for human settlement, particularly tropical rain forest has led
d) restriction to climatic changes, such as prolonged droughts and desertification.
Population increase means more pollution as people use more fuel in
06. (UNESP 2015) Leia o texto para responder à questão. cars, industry, domestic cooking. The resultant effect is increased air
and water pollution which affect the climate and food production.
OXFAM STUDY FINDS RICHEST 1% IS LIKELY TO CONTROL
HALF OF GLOBAL WEALTH BY 2016 Environmental factors have greatly contributed to food shortage.
By Patricia Cohen January 19, 2015 Climatic change has reduced agricultural production. 4The change
in climate is majorly caused by human activities and to some small
The richest 1 percent is likely to control more than half of the globe’s extent natural activities. Increased combustion of fossil fuels due to
total wealth by next year, the anti-poverty charity Oxfam reported in increasing population through power plant, motor transport and
a study released on Monday. The warning about deepening global mining of coal and oil emits green house gases which have continued
inequality comes just as the world’s business elite prepare to meet this to affect world climate. 5Deforestation of tropical forest due to human
week at the annual World Economic Forum in Davos, Switzerland. pressure has changed climatic patterns and rainfall seasons, and led
to desertification which cannot support a crop production. 6Land
The 80 wealthiest people in the world altogether own $1.9 trillion
degradation due to increased human activities has impacted negatively
the report found, nearly the same amount shared by the 3.5 billion
on agricultural production (Kamdor, 2007). Natural disasters such as
people who occupy the bottom half of the world’s income scale.(Last
floods, tropical storms and prolonged droughts are on the increase and
year, it took 85 billionaires to equal that figure.) And the richest 1
have devastating impacts on food security particularly in developing
percent of the population controls nearly half of the world’s total
countries. There are several economic factors that contribute to food
wealth,a share that is also increasing.
shortage. Economic factors affect the ability of farmers to engage in
The type of inequality that currently characterizes the world’s agricultural production. 7Poverty situation in developing nations have
economies is unlike anything seen in recent years, the report explained. reduced their capacity to produce food, as most farmers cannot afford
“Between 2002 and 2010 the total wealth of the poorest half of the seed and fertilizers. They use poor farming methods that cannot 8yield
world in current U.S. dollars had been increasing more or less at the enough, even substantial use. Investments in agricultural research and
same rate as that of billionaires,” it said. “However, since 2010, it has developing are very low in developing nations. 9Recent global financial
been decreasing over that time.” crisis have led to increase in food prices and reduced investments in
[...] agriculture by individuals and governments in developed nations
(www.nytimes.com. Adaptado.) resulting in reduced food production.
[…]
No trecho do terceiro parágrafo “However, since 2010, it has been (Adapted from http://www.paypervids.com/food-shortage-causeseffects-solutions/.
decreasing over that time.”, o termo “however” pode ser substituído, Acesso em: 14 fev 2017.)
sem alteração de sentido, por:
Glossary:
a) meanwhile. d) but. 2. soaring – something that increases rapidly above the usual level
b) like. e) so. 8. yield – to supply or produce something such as profit or an amount or food

c) then.
Mark the option which best shows the meaning of the highlighted
expression in “Deforestation of tropical forest due to human
07. (EEAR 2019) Read the text and answer the question. pressure” (ref. 5).
Dear Mary, a) Owed by.
My younger sister just told us she’s been accepted to her first b) Arranged for.
choice university. Lee is very intelligent. She will be the first person in c) Caused by.
our family to go to college. I got good grades in high school, too, but
when I graduated I went into the family business __________ going d) Deserved by.
to college. I enjoy my new career, I’m sure that I’ve learned a lot of
new things.
With love, Lincon

PROMILITARES.COM.BR 313
CONJUNCTIONS - CONNECTIVES

09. (ITA) Dadas as sentenças: The Caledonian Sky, owned by British company Noble Caledonia,
I. They had arrived on time for the vestibular. Nevertheless, the was finishing a bird-watching trip on Waigeo Island on March 4 when
gates were already closed. it veered slightly off course and slammed into the reef. An investigation
into the incident found that the cruise ship allegedly entered the area
II. She was not sure whether you’d be there or not. without consulting local guides and that ship’s crew only relied on GPS
III. Even if you promise me not to complain, I won’t believe you. navigation without considering the tide.
Consideramos que está(ão) correta(s): “The skipper forced the ship to enter the area, which was not
open to cruise ships,” CII spokesman, Albert Nebore, said.
a) apenas a I. d) apenas a II e a III.
Noble Caledonia called the accident an “unfortunate” incident
b) apenas a II. e) todas as sentenças.
and added that the company is “firmly committed to protection of
c) apenas a III. the environment” and fully backed an investigation, but made no
mention of compensation.
10. (AFA) The item that correctly explains the sentences below is: The Caledonian Sky has since been refloated and an inspection
I. She was too excited, so she took a sleeping pill. revealed that “the hull was undamaged and remained intact,” the
II. You can’t drive a car since you are not eighteen yet. company said.
III. Although the waiter had a very sore throat, he managed to The ship itself “did not take on water, nor was any pollution
answer in a hoarse whisper. reported as a result of the grounding,” Noble Caledonia added.
IV. As soon as I have finished, I’ll explain him that I don’t feel up to Indonesia’s Environment and Forestry Ministry has deployed its
tidying the kitchen now. staff to identify the damage coral reefs and collect evidence that they
will use to demand compensation from the British company.
a) concession / time / reason / comparison
Ministry spokesman Djati Witjaksono said, “We will discuss with experts
b) result / reason / concession / time the amount of compensation the company must pay [to Indonesia].”
c) reason / result / cause / time Locals in Raja Ampat say that besides the damage to the reef, the
d) cause / time / result / comparison accident has also put a major strain on the local economy, which relies
heavily on snorkeling and scuba-diving tourism.
“Coral reefs are the main attraction for many tourists in the area.
EXERCÍCIOS DE It is counterproductive for our tourism prospects,” Laura Resti, from

TREINAMENTO Raja Ampat’s homestay association, told to BBC. “We have tried to
conserve those coral reefs for a long time, and just within few hours
they were gone”. Resti added: “I am so sad and feel ashamed to take
tourists there.”
01. (AFA) Read the sentences below and mark the alternative (Adapted from: www.foxnews.com.)
containing the ideas expressed by each sentence, respectively.
In paragraph 11:
I. We camped there since it was too dark to go on.
“Locals in Raja Ampat say that besides the damage to the reef, the
II. It froze hard that night, so there was ice everywhere next day. accident has also put a major strain on the local economy (…).”
III. Even though you don’t like him you can still be polite. The word in bold is closest in meaning to:
IV. As soon as he left university he was hired by a renowned industry. a) as soon as d) as long as
a) time / concession / reason / comparison b) instead of e) as well as
b) reason / result / concession / time c) in spite of
c) result / reason / cause / time
d) time / result / cause / comparison 03. (UNIFESP 2015) No trecho “Rather, he is keen to promote
choices”, o termo em destaque equivale, em português, a
02. (EFOMM 2018) a) por sinal.
CRUISE SHIP CRASH CAUSES MORE THAN $18M IN DAMAGE b) mesmo assim.
TO PRISTINE INDONESIAN REEF, EXPERT SAYS c) pelo contrário.
Published March 14, 2017. d) via de regra.

The damage caused by a British-owned cruise ship that accidentally e) além disso.
run aground on a pristine Indonesian coral reef could cause total more
than $18M million, according to academics and environmental groups 04. (UNESP 2018) Leia o texto para responder à questão a seguir.
working in the region. WHEN DOES THE BRAIN WORK BEST?
Researches for Conservation International Indonesia (CII), Papua The peak times and ages for learning
State University and the Regional Technical Implementing Unit (UPTD)
found that the grounding of the 295-foot Caledonian Sky cruise ship –
which weighs 4,200 tons and carried 102 passengers – caused massive
damage to several endemic reefs that are unique to Raja Ampat, a
remote and idyllic island chain west of Indonesian’s Papua province.
“The types of reefs that were damaged by the ship are Genus
Porites, Acropora, Poicilopora, Tubastrea, Montipora, Stylopora, Favia
and Pavites. It will take decades for restore the reefs,” Ricardo Tapilatu,
who headed the research, told the Jakarta Post. Tapilatu added that
damage area stretched for more than 145,000-square-miles.

314 PROMILITARES.COM.BR
CONJUNCTIONS - CONNECTIVES

What’s your ideal time of the day for brain performance? The good news is that the scientific community is gradually
Surprisingly, the answer to this isn’t as simple as being a morning or recognizing the importance of these ethical issues. For example, more
a night person. New research has shown that certain times of the day marine mammal scientists are steering away from doing research on
are best for completing specific tasks, and listening to your body’s captive dolphins. More significantly, a small group of experts who met
natural clock may help you to accomplish more in 24 hours. at the Helsinki Collegium for Advanced Studies in the spring of 2010 to
Science suggests that the best time for our natural peak evaluate the ethical implications of the scientific research on cetaceans
productivity is late morning. Our body temperatures start to rise just concluded that the evidence merited issuing a Declaration of Rights for
before we wake up in the morning and continue to increase through Cetaceans: Whales and Dolphins. This group included such prominent
midday, Steve Kay, a professor of molecular and computational biology scientists as Lori Marino and Hal Whitehead. Particularly important
at the University of Southern California told The Wall Street Journal. in this declaration was the recognition that whales and dolphins
This gradual increase in body temperature means that our working are persons who are "beyond use". Treating them as 'property' is
memory, alertness, and concentration also gradually improve, peaking indefensible. Unfortunately, while there has been consistent progress
at about mid morning. Our alertness tends to dip after this point, in scientists' sensitivity to the ethical issues, the same cannot be said
but one study suggested that midday fatigue may actually boost our for those who use cetaceans to generate revenue.
creative abilities. For a 2011 study, 428 students were asked to solve (Disponível em: www.abc.net.au/environment/articles. (Adaptado))
a series of two types of problems, requiring either analytical or novel
thinking. Results showed that their performance on the second type As palavras “Like” (ref. 1) e “However” (ref. 2) estabelecem,
was best at non-peak times of day when they were tired. respectivamente, relações de
As for the age where our brains are at peak condition, science has a) contraste e dúvida.
long held that fluid intelligence, or the ability to think quickly and recall b) condição e contraste.
information, peaks at around age 20. However, a 2015 study revealed c) temporalidade e dúvida.
that peak brain age is far more complicated than previously believed
and concluded that there are about 30 subsets of intelligence, all of d) comparação e causalidade.
which peak at different ages for different people. For example, the e) comparação e contraste.
study found that raw speed in processing information appears to peak
around age 18 or 19, then immediately starts to decline, but short- 06. (ESPCEX/AMAN 2018)
term memory continues to improve until around age 25, and then HOW DIVERSITY MAKES US SMARTER
begins to drop around age 35, Medical Xpress reported. The ability to
evaluate other people’s emotional states peaked much later, in the 40s Decades of research by organizational scientists, psychologists,
or 50s. In addition, the study suggested that out our vocabulary may sociologists, economists and demographers show that socially diverse
peak as late as our 60s’s or 70’s. groups (that is, those with a diversity of race, ethnicity, gender and
sexual orientation) are more innovative than homogeneous groups. It
Still, while working according to your body’s natural clock may means being around people who are different from us makes us more
sound helpful, it’s important to remember that these times may differ creative, more diligent and more hardworking.
from person to person. On average, people can be divided into two
distinct groups: morning people tend to wake up and go to sleep It seems obvious that a group of people with diverse individual
earlier and to be most productive early in the day. Evening people tend expertise would be better than a homogeneous group at solving
to wake up later, start more slowly and peak in the evening. If being a complex, non-routine problems. It is less obvious that social diversity
morning or evening person has been working for you the majority of should work in the same way - yet the science shows that it does.
your life, it may be best to not fix what’s not broken. This is not only because people with different backgrounds bring new
information. Simply interacting with individuals who are different
(Dana Dovey. www.medicaldaily.com, 08.08.2016. Adaptado.)
forces group members to prepare better, to anticipate alternative
No trecho do terceiro parágrafo “However, a 2015 study revealed”, o viewpoints and to expect that reaching consensus will take effort.
termo em destaque pode ser substituído, sem alteração de sentido, por: Diversity of expertise confers benefits that are obvious - you
a) although. c) inasmuch. e) whatever. would not think of building a new car without engineers, designers
and quality-control experts - but what about social diversity? The
b) nevertheless. d) meanwhile. same logic applies to social diversity. People who are different from
one another in race, gender and other dimensions bring unique
05. (UPE 2013) Leia o texto. information and experiences to bear on the task at hand. A male
WHALES ARE PEOPLE, TOO and a female engineer might have perspectives as different from one
another as an engineer and a physicist - and that is a good thing.
One of the most important features of science is that scientific
progress regularly leads to important ethical questions. This is The fact is that if you want to build teams or organizations
particularly true with research about cetaceans — whales, dolphins capable of innovating, you need diversity. Diversity enhances creativity.
and the like — because it has become increasingly apparent that the It encourages the search for novel information and perspectives,
inner life of these nonhumans is more complex than most humans leading to better decision making and problem solving. Diversity can
realize. We have learned that their capacity for suffering is significantly improve the bottom line of companies and lead to discoveries and
greater than has been imagined — which makes much human breakthrough innovations. Even simply being exposed to diversity can
behavior towards these nonhumans ethically problematic. change the way you think.
(Adapted from http://www.scientificamerican.com/article/how-diversity-makes-us-
There is now ample scientific evidence that capacities once thought smarter/)
to be unique to humans are shared by these beings. 1Like humans,
whales and dolphins are 'persons'. That is, they are self-aware beings Choose the alternative that correctly substitutes the word yet in the
with individual personalities and a rich inner life. They have the ability sentence "It is less obvious that social diversity should work in the
to think abstractly, feel deeply and choose their actions. Their lives same way - yet the science shows that it does." (paragraph 2).
are characterized by close, long-term relationships with conspecifics in a) however d) thus
communities characterized by culture. In short, whales and dolphins
are a “who”, not a “what”. b) for e) because
However, as the saying goes, there is good news and there is
2 c) such as
bad news.

PROMILITARES.COM.BR 315
CONJUNCTIONS - CONNECTIVES

07. (ESPM 2013) **DISCLAIMER: All characters and scenarios in this post are
THE EURO - THE FLIGHT FROM SPAIN fictional.**
Spain can be shored up for a while; but its woes contain an alarming Scott couldn’t believe his eyes when he checked Facebook this
lesson for the entire euro zone morning. A new page, “SCU Confessions,” had just been created,
and one of the first “confessions” was about him! Someone shared
The worst nightmares are the ones you cannot wake up from. a story where he had gotten really drunk last week and did a few
Just ask Spain. A year ago the cost of Spanish government borrowing things he wasn’t proud of. Granted, he wasn’t mentioned by name,
soared as euro contagion spread from Greece, Ireland and Portugal. but it was a unique enough situation that everyone he knew would
Panic seemed to subside with central-bank intervention and the recognize it as being about him.
promise of a new reforming government in Madrid. Since then Spain
has, broadly, been as good as its word and Mariano Rajoy’s government Scott had heard about other schools starting pages like this,
has, played its part in countless “make-or-break” summits in Brussels where people message the page administrator their secrets, hook-up
and secured up to €100 billion ($121 billion) to prop up its banks. Yet stories, dirty deeds, and anything else that they would want to share
despite all its efforts and pain, Spain cannot shake off that sense of anonymously. Scott initially thought these pages were hilarious, and
doom. On July 25th the yield on ten-year bonds touched a euro-era even “liked” the ones from other schools just 1so that he could be
record of 7.75%. Two-year bonds have climbed above 7%: investors entertained. However, now that he was reading something about him,
fear that Spain must soon ask for a bail-out—or default. Spain’s he felt embarrassed and upset. Already it had 50 “likes” and counting,
nightmare is a symptom of what is wrong with the entire euro zone. and several of his friends tagged him in the comments 2so that he
As the months drag on, the crisis is deepening. Europe’s leaders have would see it. To make matters worse, the post was anonymous, so he
asked the world to trust that they will do what it takes to save the had no way of knowing who was spreading the story around.
euro. They have also pleaded for more time to sort out the mess. Their [...]
task is indeed immense, but as they disappear to their chateaux and (Posted by Chloe Wilson Available at: http://www.scu.edu/r/ethics-center/ethicsblog/
beach villas, trust is draining away and time is not their friend. thebigq/15790/ Can-You-Keep-a- Secret?. Accessed on: April 2013.)
The bull and the horns
The linking word “so that” (ref. 1 e 2) expresses:
Spain’s situation today is all the more shocking because only this
month it had announced €65 billion of tax rises and spen- ding cuts a) reason. d) emphasis.
and won the funds for its bank rescue. This was meant to persuade b) purpose. e) comparison.
investors that the whole euro zone is serious about keeping Spain. c) addition.
Yet the message was obliterated by news that the government now
expects the recession to last into 2013 and, worse, that it will have to
09. (ITA 2019) Leia o fragmento abaixo.
find the money to bail out regions which have suddenly confessed to
being broke. […]
The prognosis for Spain is bleak. The economy is in recession, the The shift has not happened by accident. 1As Jacques Peretti argued
public sector is cuttings pending and the private sector is reluctant to in his film The Men Who Made Us Fat, food companies have invested
invest. This lack of domestic demand almost guarantees that Mr Rajoy heavily in designing products that use sugar to bypass our natural
will fail to meet the target to reduce the deficit. If that happens, Spain appetite control mechanisms, and in packaging and promoting these
will be asked to impose yet more austerity. That will undermine his products to break down what remains of our defenses, including
popularity, which has already fallen steeply since he was elected. Spain’s through the use of subliminal scents. They employ an army of food
resolve will be further damaged by rows over budget cuts between scientists and psychologists to trick us into eating more than we
Madrid and regional politicians, who control 40% of public spending— need, while their advertisers use the latest findings in neuroscience to
and who, even if they are from Mr Rajoy’s party, jealously guard their overcome our resistance.
autonomy. Political uncertainty will feed back into the economy, which […]
will only deteriorate more. And the vicious circle continues.
(Adaptado de: <https://www.theguardian.com/commentisfree/2018/aug/15/age-of-
(Jul 28th 2012 / www.economist.com) obesity-shaming-overweight-people/>. Acesso em: ago. 2018.)

In the underlined sentence “Yet despite all its efforts and pain, Spain Assinale a alternativa que pode substituir ‘as’ na sentença “As Jacques
cannot shake off that sense of doom.”, the word “yet” conveys an Peretti argued in his film The Men Who Made Us Fat, food companies
idea of have invested heavily in designing products [...]” (ref. 1) mantendo o
a) opposition mesmo sentido do texto e a correção gramatical.
b) addition a) In line with what
c) replacement b) In contempt of
d) cause c) During the time
e) result d) Considering that
e) Despite the fact that
08. (CEFET-MG 2013) The following text was published on the
Markkula Center for Applied Ethics website at Santa Clara University. 10. (ITA 2019) A questão a seguir refere-se ao texto abaixo:
It’s a blog post written by an Ethics teacher. Read it and answer
Artificial Intelligence (AI) is going to play an enormous role in
question.
our lives and in the global economy. It is the key to self-driving cars,
THE BIG Q - BACK TO BLOG the Amazon Alexa in your home, autonomous trading desks on Wall
Can You Keep a Secret? Street, innovation in medicine, and cyberwar defenses.
Monday, Mar. 25, 2013 Technology is rarely good nor evil — it’s all in how humans use
it. AI could do an enormous amount of good and solve some of the
The best student comment on “Can You Keep a Secret?” wins a world’s hardest problems, but that same power could be turned
$100 Amazon gift certificate. against us. AI could be set up to inflict bias based on race or beliefs,
Entries must be received by midnight, Sunday, April 7, 2013. invade our privacy, learn about and exploit our personal weaknesses
Subscribe to the blog (by RSS or by e-mail in the right hand column) — and do a lot of nefarious things we can’t yet foresee.
for updates.

316 PROMILITARES.COM.BR
CONJUNCTIONS - CONNECTIVES

Which means that our policymakers must understand 1and help d) I. para indicar confirmação; II. para expressar efeito; III. como
guide AI so it benefits society. […] We don’t want overreaching conjunção adversativa.
regulation that goes beyond keeping us safe and ends up stifling e) I. para indicar inclusão; II. como preposição; III. para indicar a
innovation. 2Regulators helped make it so difficult to develop atomic relevância do que será expresso.
energy, today the U.S. gets only 20% of its electricity from nuclear
power. 3So, while we need a Federal Artificial Intelligence Agency, or
FAIA, I would prefer to see it created as a public-private partnership.
Washington should bring in AI experts from the tech industry to a EXERCÍCIOS DE

COMBATE
federal agency designed to understand and direct AI and to inform
lawmakers. Perhaps the AI experts would rotate through Washington
on a kind of public service tour of duty.
Importantly, we’re at the beginning of a new era in government
— one where governance is software-defined. The nature of AI 01. (UFRS) Gases and liquids are two forms of the fluid states: gases are
and algorithms means we need to develop a new kind of agency generally compressible __________ liquids are often incompressible.
— one that includes both humans and software. The software will
a) as a result
help monitor algorithms. Existing, old-school regulations that rely on
manual enforcement are too cumbersome to keep up with technology b) while
and too “dumb” to monitor algorithms in a timely way. c) because
Software-defined regulation can monitor software-driven d) in other words
industries better than regulations enforced by squads of regulators.
e) whether
Algorithms can continuously watch emerging utilities such as
Facebook, looking for details and patterns that humans might never
catch, but nonetheless signal abuses. If Congress wants to make sure 02. (EN) Which of the options completes the sentence correctly?
Facebook doesn’t exploit political biases, it could direct the FAIA to “People who are middle-aged and older tend to know more than
write an algorithm to look for the behavior. young adults ________ they have been around longer, and score
It’s just as important to have algorithms that keep an eye on the higher on vocabulary tests, crossword puzzles and other measures of
role of humans inside these companies. We want technology that can so-called crystallized intelligence.”
tell if Airbnb hosts are illegally turning down minorities or if Facebook’s a) so
human editors are squashing conservative news headlines. b) when
The watchdog algorithms can be like open-source software — c) while
open to examination by anyone, while the companies keep private
d) because
proprietary algorithms and data. If the algorithms are public, anyone
can run various datasets against them and analyze for “off the rails” e) in case
behaviors and unexpected results.
Clearly, AI needs some governance. As Facebook is proving, we 03. (AFA) Which alternative completes meaningfully the sentence
can’t rely on companies to monitor and regulate themselves. Public below?
companies, especially, are incentivized to make the biggest profits “_______ the possibility of an awful storm they decided not to _______
possible, and their algorithms will optimize for financial goals, not the match that _______ scheduled.”
societal goals. But as a tech investor, I don’t want to see an ill-informed a) Because / win / is
Congress set up regulatory schemes for social networks, search and
b) Although / play / was
other key services that then make our dynamic tech companies as dull
and bureaucratic as electric companies. […] Technology companies c) However / cancel / isn’t
and policymakers need to come together soon and share ideas about d) In spite of / call off / had been
AI governance and the establishment of a software-driven AI agency.
[...] 04. (ITA) The test was ___________ no one passed.
Let’s do this before bad regulations get enacted — and before a) very hard that
AI gets away from us and does more damage. We have a chance b) too hard for that
right now to tee up AI so it does tremendous good. To unleash it in
a positive direction, we need to get the checks and balances in place c) too hard, so
right now. d) so hard so that
(Adaptado de <https://www.marketwatch.com/story/artificial-intelligence-is-too- e) too hard
powerful-to-be-left-to-facebook-amazon-and-other-tech-giants-2018-04-23>.
Acesso em: jun. 2018.)
05. (FATEC-SP) The buses are crowded and dirty; ________, they are
Observe o uso da palavra ‘so’ nas frases abaixo. never on time.
I. […] and help guide AI so it benefits society […] (ref. 1) a) then
II. Regulators helped make it so difficult to develop […] (ref. 2) b) instead
III. So, while we need a Federal Artificial Intelligence Agency, or FAIA c) in addition
[…] (ref. 3) d) for example
Assinale a alternativa que explica respectivamente, o uso de ‘so’. e) nevertheless
a) I. para expressar propósito; II. como intensificador; III. para
sintetizar ideias anteriores. 06. (FUVEST) Assinale a alternativa que completa corretamente a frase:
b) I. para expressar resultado; II. para indicar tamanho ou extensão; “Yuri and Vanessa are very good friends. _________, they sometimes
III. para substituir uma oração. stop talking to each other _______ both of them are very stubborn.”
c) I. para introduzir uma decisão; II. como advérbio de modo; III. com a) So that / otherwise
sentido de “até o momento”. b) However / because

PROMILITARES.COM.BR 317
CONJUNCTIONS - CONNECTIVES

c) But / whether
ANOTAÇÕES
d) Even if / for
e) “a” and “c” are correct

07. (AFA) Regardless of what we think, we can clearly see that there
are some ways that friends, best friends and archenemies are the
same, but in the and they are clearly more different.
Nonetheless we all have every single type in our lives.
The option that contains a synonym for the underlined expression is:
a) nevertheless.
b) due to.
c) therefore.
d) although.

08. (COMD. AERONÁUTICA) Mark the option that can replace the
word in bold.
“Despite being rich and famous, Kim is a lonely man.”
a) Although
b) However
c) In spite of
d) Whereas

09. (EEAR) A pilot from Northwest Airlines flight was less than pleased
with the food which was to be served on the flight.
He decided to find some better food, so he left his aeroplane, and
then left the airport to find food even though he had a plane to fly.
The conjunction “so”, in the text, can be replaced by:
a) because.
b) however.
c) even though.
d) and therefore.

10. (EN) Which of the options completes the sentence correctly?


“Surveys have found that even though 80% of smokers would like
to quit smoking, less than five percent are able to quit on their own
______ the highly addictive properties of nicotine.”
a) nonetheless
b) due to
c) moreover
d) however
e) instead of

GABARITO
EXERCÍCIOS DE FIXAÇÃO
01. E 04. D 07. B 10. B
02. A 05. C 08. C
03. E 06. D 09. E
EXERCÍCIOS DE TREINAMENTO
01. B 04. B 07. A 10. A
02. E 05. E 08. B
03. C 06. A 09. A
EXERCÍCIOS DE COMBATE
01. B 04. C 07. A 10. B
02. D 05. C 08. C
03. D 06. B 09. D

318 PROMILITARES.COM.BR
PREPOSITIONS

MOVEMENT • At - In the general direction of.


• Down (to) - movement downwards. Exemplo:
Exemplo: He threw a stone at me and it hit my leg.
He was lucky he didn’t break his neck when he fell so heavily
down the stairs. • Away from - Leaving an object, person or place.
Exemplo:
• Off - movement from one surface to a lower one. She ran away from home when she was only sixteen.
Exemplo:
The girls screamed as Michael Jackson came off the plane. • Down - Descending
Exemplo:
• Onto - Movement from one surface to a higher one. A tear ran down his cheek.
Exemplo:
He stepped onto the bus just as it set off from the stop. • For - With a destination of
Exemplo:
• Up (to) - Movement upwards. Cabral set sail for India in the late 15th century.
Exemplos:
The cat ran up the tree when she heard the dog. • Into - To a destination within something.
They traveled up to Paris form Bordeaux once every year. Exemplo:
The dog disappeared into the wood.
PASSING THROUGH
• Across - Go from one side to the other, usually on the surface. • Onto - To a destination on something.
Exemplo: Exemplo:
It took us more than two days to get across the desert. He put the open book onto the table and left.

• By - Pass beside/ next to • To - In the specific direction of


Exemplo: Exemplos:
We took the road which runs by the river. To Galeão airport, please. And Hurry! My plane leaves very soon.
Could you give this record to Paul, please?
• Through - Between the walls/ parts of.
Exemplo: • Towards - In the general direction of
I didn’t think I’d get the car through that narrow entrance. Exemplo:
They walked towards the car.
• Over - Go from one side to the other (similar to across, but usually
above). • Up - up to
Exemplo: Exemplo:
We flew over São Paulo on our way to Curitiba. The burglar climbed up the to the second floor and forced the
window.
DIRECTION
• Around - In a circular direction. • Up to - Ascending, but expressing destination.
Exemplo:
I’ve driven around this park three times and I still don’t know CATCHING UP AND DROPPING BACK
which road I want. • After - Following (pursuit)
Exemplo:
The cat ran playfully after the dog.

PROMILITARES.COM.BR 319
PREPOSITIONS

• Behind - Following • on the left of/on the right of - Immediately next to on the left/
Exemplo: right hand side
My building is behind the gas station. Exemplo:
I was a bridesmaid at her wedding. In that picture I’m on the left
of the bride.
• In front of - Preceding
Exemplo:
• Near - In the vicinity of
Run in front of her and ask the bus driver to wait.
Exemplo:
I’d like to send him to this school because we live very near it.
POSITION
• Above - Directly higher than something
• Next to - Very close, almost touching
Exemplo:
Exemplo:
The plane is now circling above Vitória.
We can’t possibly buy the house next to the cemetery.

• After - Slightly inferior to


Exemplo:
TIME
• About - Approximate time
Her opinion is second only after his.
Exemplo:
I’ll meet you about 4.30, at Maracanã.
• Below - Directly lower than something.
Exemplo:
• Around - Approximate time
There’s a huge bruise on your leg, just below the knee.
Exemplo:
We should arrive around three o’clock.
• Down - At or in a lower part of
Exemplo:
• At - Exact time
He lives further down the hill.
Exemplo:
The train leaves Nova Iguaçu at 8.30 am.
• On - As above, but with contact with the surface
Exemplo:
Occasions, special days
There’s a brilliant green parrot on his shoulder!
Exemplo:
We usually have four day’s holiday at Easter.
• Under - As below, but the object is closer
Exemplo:
• In - Parts of a day, season and years
In a few years there may be a tunnel under the Guanabara bay.
Exemplo:
• Up - As down, but expressing the opposite Very few shops are open in the evening.

CLOSE AT HAND • On - Names of days


• Along - To be placed by the side of something Exemplo:
Exemplo: The new teacher will start on Monday, 13 September.
There are emergency telephones all along the motorway.
• Within - Defined periods (rather formal)
• Around - Somewhere in the vicinity Exemplo:
Exemplo: I’ll be back with the shopping within an hour.
I know there’s a bar around here somewhere.
• After - Following a time
• At - Exact position Exemplos:
Exemplo: Our classes this year start after nine.
Wait for me right at the end of that narrow street. Following an event.

• Beside - At the side of • Before - Preceding a time


Exemplo: Exemplos:
The Rhine wines are produced mostly in vineyards beside the river. Please try to arrive before eight o’clock or we’ll miss the beginning
of the play.
• By - In the region of Preceding an event.
Exemplo:
He lives in a pretty little cottage by the sea.

320 PROMILITARES.COM.BR
PREPOSITIONS

• By - Preceding or at a time/point AT
Exemplo: arrive at, guess at, hint at, look at, stare at, marvel at, play at
Give me your homework by Friday, then I can mark it at the (=pretend) wonder at, work at, peep at, peer at
weekend.
AGAINST
• Past - Telling the time. be prejudiced against, insure against, protest against (or to), react
against (or to), rebel against, warn against (or about)
Exemplo:
It’s half-past eleven already.
ABOUT
joke about, laugh about, sing about, speak about, talk about,
• To - Telling the time think about, worry about, be/get upset about, tell someone about
Exemplos:
What time is it? IN
It’s already a quarter to six, let’s go home. believe in, delight in, be engaged in, be included in, indulge in,
take a pride in, be interested in, invest in, get involved in, persist in
• About - Approximate duration
FROM
Exemplo:
abstain from, borrow from, demand thing from someone, differ
The carnival lasts about eight days. from, discourage one from, distinguish A from B, draw money from,
emerge from, escape from, exclude from, hinder from, prevent
• Between - in the middle of two things/with someone/something someone from, prohibit from, recover from (illness), refrain from,
on each side or end separate A from B, suffer from
Exemplo:
FOR
The period between the two world wars was a period of
depression. account for, act for someone, apologize for, ask for something,
blame someone for, beg for call for (= require), charge for, exchange
A for B, be intended for, long for, hope for, pay for something, prepare
• During - Period of time for, provide for, search for, substitute A for B, thank someone for wait
Exemplo: for, vote for
The weather was fantastic during our holiday.
OF
• For - Duration of a given length accuse someone of, approve of, beware of, consist of, be
convicted of, convince someone of, cure someone of (an illness),
Exemplo: despair of, disapprove of, dream of, repent of, get rid of, smell of,
In 1972 India had been independent for 25 years. taste of, suspect of ,think of, tire of, get tired of, warn someone of

• In - Specified duration TO
Exemplo: accustom oneself to, appeal to, be attached to, attend to, attribute
A to B, belong to, apply to, get accustomed to, amount to, challenge
Wait for me here, please. I’ll be back in a few minutes.
someone to, compare to (or with), be condemned to, be confined
to, consent to, be converted to, be entitled to, entrust something to
• Since - Duration from a given point until the present. someone, invite someone to, listen to, look forward to, object to, react
to, be reduced to, resort to, respond to, reply to, be subjected to,
Exemplo:
submit to, succumb to, surrender to, subscribe to, be/get used to, talk
India has been independent since 1947. to, turn to, yield to

• Till - Duration to a given point. WITH


Exemplo: acquaint someone with, be afflicted with, agree with someone,
help someone with, be charged with, communicate with, compare A
She’ll be studying till 2020, when she takes her final exams.
with B, compete with (or against), comply with, conflict with, confuse
A with B, A contrasts with B, cope with, correspond with, be infected
with, interfere with, be threatened with, A doesn’t mix with B, part
VERBOS SEGUIDOS DAS SEGUINTES with, quarrel with, fight with (or against), reason with someone, be
satisfied with
PREPOSIÇÕES

ON EXPRESSÕES COMUNS COM PREPOSIÇÃO


act on, take revenge on, be based on, comment on, concentrate
on, congratulate one on something, count on rely), experiment on, OUT OF
decide on, depend on, embark on, impress on one that, intrude on, out of bounds, out of breath, out of control, out of danger, out
live on (food), play something on an instrument, pride oneself on, of date, out of doors, out of fashion, out of interest, out of luck, out
reckon on, rely on, make war on, work on (a project) of order, out of the ordinary, out of place, out of practice, out of all
proportion, out of the question, out of sight, out of stock, out of turn,
out of use

PROMILITARES.COM.BR 321
PREPOSITIONS

ON
on average on fire in practice
on (the) average, on board (ship), on business, on the contrary, on
fire, on foot, on holiday, on horseback, on the job, on leave, on the on balance on foot out of practice
level, on the other hand, on purpose, on second thoughts, on time,
on the way, on the whole on behalf of in future at present

out of breath in gear in private


IN
in the circumstances, in church, in doors (indoors), in difficulty, in in bulk in general in principle
difficulties, in fact, in fashion, in haste, in hospital, in a hurry, in love,
in order, in pain, in part, in the post, in practice, in prison, in any case, on business under guarantee in progress
in particular, in bed, in trouble, in reply, in school, in stock, in style,
in case on guard under protest
in tears, in theory, in time, in turn(s), in use, in vain, in common, in
public, in private, in a way, in half, in general, in the long run out of character for good in public

BY in charge of out of mind on purpose


by accident, by air, by all means, by car (other vehicles), by chance, in common with in half within reason
by coincidence, by default, by degrees, by design, (play) by ear, by
force, by heart, by no/any means, by mistake, by name (know one in company with by hand on receipt of
by name), by myself (etc.), by order, by sight (know one by sight), by
surprise, by the way in comparison with in hand in receipt of

in competition with in harmony with reference to


AT
at dinner (+ other meals), at ease, at first, at first sight, at home, on condition that by heart with regard to
at last, at least, at short/a moment’s notice, at the office, at once, at
present, at any rate, at school, at sea, at the theatre, at one time, one in connection with on heat in relation to
at a time, at the time (then), at times, at the same time, at war
under consideration for hire in reply to

FOR out of control on holiday on stage


for ever (forever). for example. for good (forever). for instance.
under control at home in step
for the record. for my sake (etc.). for Heaven’s sake. for sale. for the
time being under cover in honour of out of step

OUTRAS in credit for instance in stock


from memory, from now on, beside the point, within reason, by chance by invitation out of stock
without doubt, off the cuff, under the/no circumstances, under
protest, under repair, under suspicion, under weather, up to date, by choice on land in store
with interest, with luck, to now
in danger of at last on strike

out of danger by law in style


EXPRESSÕES USANDO PREPOSIÇÃO
out of date at length by surprise
COM SUBSTANTIVO
up to date at liberty in tears
by accident in difficulties in luck
by day for life in theory
on account of on duty by mail
in debt in line with on time
in addition to off duty by means of
without delay on loan in time
in advance at ease (call) to mind
on demand not for long at times
by agreement with on edge by mistake
by design in love in town
in answer to in effect in motion
in respect of on occasion out of town
by appointment in exchange for by word of mouth
in response to on offer on trial
on approval as a matter of fact by name
at rest in operation on top of
by arrangement in fact by nature
in return for by order in touch with
on arrival in fashion in need of
at risk in order (to) on tour
by association in favour of by night
for sale out of order on trust
under attack for fear that at night
(start) from scratch in pain in turn(s)
in attendance on file at short notice

322 PROMILITARES.COM.BR
PREPOSITIONS

EXERCÍCIOS DE

FIXAÇÃO
in search of in particular in use

in season on paper on vacation

in secret at peace in vain


01. (EN) Assinale a alternativa correta:
in sight in person in view
“Aspirin is the best drug to fight ______ headache.”
on sight by phone by virtue of a) on c) with e) for
by sight out of place at war b) against d) to

at (great) speed in place out of work 02. (PUC-SP) Assinale a alternativa correta:

in spite of in possession of “Very little is known ______ nuclear energy.”


a) of c) in e) about
b) over d) into

ADJETIVOS COM PREPOSIÇÕES 03. (UNESP) Assinale a alternativa correta:


“Fried potatoes are called “French Fries” __________ the United
AT States.”
amazed, amused, astonished, shocked, surprised, bad, brilliant, a) on c) of e) in
clever, good, hopeless, quick, weak
b) about d) from

FOR 04. (EEAR 2019) Read the text and answer the question:
answerable, bad, bound, convenient, due (~ ready), eager, Illegal levels of arsenic and mercury polluted a river in the days
inferior, famous, fit, good, grateful, late, prepared, qualified, ready, after a dam burst at an iron ore mine this month __________ Brazil’s
sorry, suitable, useful, responsible, eligible worst environmental disaster, according to tests by a state water
agency, the Institute for Water Management in Minas Gerais. The
TO agency found arsenic levels more than 10 times above the legal limit in
accustomed, allergic, answerable, attentive, averse, blind, close, one place __________ the river, the Rio Doce, after the dam burst on
contrary, due, faithful, harmful, inferior, married, new, peculiar, Nov. 5, killing at least 13 people. Mercury slightly above the permitted
sensitive, similar, superior, thankful, used (accustomed) level was also found in one area. Samarco, the mine operator, and
its co-owners, BHP Billiton and Vale, have repeatedly said that the
water and mineral waste unleashed by the dam burst were not toxic.
ABOUT On Wednesday, the United Nations human rights agency said “new
angry, annoyed, anxious, certain, clear, concerned (~worried), evidence” showed that the mud dumped by the flood “contained
crazy, curious, doubtful, excited, glad, happy, hesitant, honest, mad, high levels of toxic heavy metals and other chemicals.”
mistaken, positive, puzzled, sad, selfish, sensible, sensitive, serious, (Adopted from www.nytimes.com)
sincere, sorry, sure, suspicious, thrilled, uneasy, worried, wrong
Fill in the blanks with the option that best completes the text.
OF a) over / in
ahead, afraid, aware, ashamed, capable, certain, confident, b) in / along
conscious, critical, envious, fond, full, guilty, independent, jealous, c) at / along
proud, sick, sure, suspicious, tired
d) on / across
WITH
05. (FAMEMA 2019) Leia o texto para responder à questão a seguir.
angry with someone, annoyed with someone, busy, concerned,
Fake news can distort people’s beliefs even after being debunked. A
contented, delighted, disappointed, disgusted, familiar, level, patient,
study recently published in the journal Intelligence suggests that some
pleased, satisfied
people may have an especially difficult time rejecting misinformation.
Asked to rate a fictitious person on a range of character traits,
FROM people who scored low on a test of cognitive ability continued to be
absent from, different from, safe from, separate from influenced by damaging information about the person even after they
were explicitly told the information was false. The study is significant
ON because it identifies what may be a major risk factor for vulnerability
to fake news.
dependent on, intent on, keen on
One possible explanation for this finding is based on the theory
that a person’s cognitive ability reflects how well they can regulate
the contents of working memory – their “mental workspace” for
processing information. First proposed by the cognitive psychologists
Lynn Hasher and Rose Zacks, this theory holds that some people are
more prone to “mental clutter” than other people. In other words,
some people are less able to discard (or “inhibit”) information from
their working memory that is no longer relevant to the task at hand,
or information that has been discredited. Research on cognitive aging
indicates that, in adulthood, this ability declines considerably with
advancing age, suggesting that older adults may also be especially

PROMILITARES.COM.BR 323
PREPOSITIONS

vulnerable to fake news. Another reason why cognitive ability may 3


It could be argued that the answer is one. The attacks on the two
predict vulnerability to fake news is that it correlates highly with buildings were part of a single plan conceived by one man in service of
education. Through education, people may develop meta-cognitive a single agenda. They unfolded _____3_____ a few minutes and yards
skills – strategies for monitoring and regulating one’s own thinking – of each other, targeting the parts of a complex with a single name,
that can be used to combat the effects of misinformation. design, and owner. And they launched a single chain of military and
(www.scientificamerican.com, 06.02.2018. Adaptado.) political events in their aftermath.
Or it could be argued that the answer is two. The towers were
Considere o trecho do segundo parágrafo “Research on cognitive distinct collections of glass and steel separated by an expanse of
aging indicates that, in adulthood, this ability declines considerably”. space, and they were hit at different times and went out of existence
O termo sublinhado é empregado com o mesmo sentido em: at different times. The amateur video that showed the second plane
a) The program will be broadcast on BBC TV. 4
closing in on the south tower as the north tower billowed with smoke
b) Looking around the room I notice a diary on her bedside table. makes the twoness unmistakable: while one event was frozen in the
past, the other loomed in the future.
c) The glass of wine was on the kitchen counter.
The gravity of 9/11 would seem to make this discussion frivolous
d) There was a sign on the entrance door. to the point of impudence, a matter of mere "semantics," as we say,
e) Yesterday I watched a documentary on forensic science. with its implication of 5splitting hairs. But the relation of language to
our inner and outer worlds is a matter of intellectual fascination and
06. (FEI 1996) Leia o texto. real-world importance.
A NOBLE GIFT This "importance" is often hard to quantify, _____4_____ this
One of the most famous monuments in the world, the Statue case I can put an exact value on it: 3,5 billion dollars. That was the
of Liberty, was presented to the United States of America in the sum in a legal dispute for the insurance payout to Larry Silverstein,
nineteenth century by the people of France. The great statue, which the leaseholder of the World Trade Center site. Silverstein’s insurance
was designed by the sculptor August Bartholdi, took ten years to policies stipulated a maximum reimbursement for each destructive
complete. The actual figure was made of copper supported by a metal "event." If 9/11 comprised a single event, he stood to receive 3,5
framework which had been especially constructed by Eiffel. Before billion dollars; if two, he stood to receive 7 billion. In the trials, the
it could be transported to the United States, a site had to be found attorneys disputed the applicable meaning of the term event. The
for it and a pedestal had to be built. The site chosen was an island lawyers for the leaseholder defined it in physical terms (two collapses);
at the entrance of New York Harbour. By 1884, a statue which was those for the insurance companies defined it in mental terms (one
151 fast tall, had been erected In Paris. The following year, it was plot). There is nothing "mere" about semantics!
taken to pieces and sent to America. By the end of October 1886, the (Adapted from: PINKER, Steven. The Stuff of Thought. New York: Penguin, 2007. p. 1-2.)
statue had been put together again and was officially presented to the
American people by Bartholdi. Ever since then, the great monument Select the alternative that adequately fills in the gaps 1, 2, 3 and 4 in
has been a symbol of liberty for the millions of people who have this same order.
passed through New York Harbour to make their homes. a) In – in – within – in
Preencha a lacuna de acordo com o texto: "Who was the statue b) In – on – from – at
presented __________?", assinalando a resposta correta: c) On – in – from – at
a) from d) at d) On – on – from – at
b) for e) in e) On – in – within – in
c) by
09. (PUC-PR) Choose the RIGHT alternative to complete the spaces:
07. (UPE 2012) Read the paragraph and fill in the blanks using a I. I stayed in New York __________ two months.
preposition.
II. The film didn’t begin __________ nine o’clock.
“Freddie Mercury was born ______ September 5th, 1946. He was the
III. I go there __________ an hour.
vocalist _____ the band rock ‘Queen’ and composed many hits _____
the band. As the lead _____ ‘Queen’, he travelled all _____ the world. IV. They’ve been mending the road __________ last Monday.
Freddie Mercury died ____ 1991.” V. I’ll be working in a bank __________ three years.
a) for; of; of; of; under; in a) by – in – since – for – until
b) on; of; for; of; around; in b) for – until – in – since – for
c) in; of; of; all; over; on c) by – until – in – before – for
d) at; at; in; in; of; at d) since – by – before – until – by
e) to; at; on; at; around; at
10. (EN) Escolha a melhor alternativa para preencher as lacunas da
08. (UFRGS 2018) A questão a seguir está relacionada ao texto abaixo. frase a seguir: “________ 1948 an American woman was employed
_____1_____ September 11, 2001, at 8:46 A.M., a hijacked __________ the first time __________ a jet pilot __________ an
airliner crashed into the north tower of the World Trade Center in New American airline.
York. At 9:03 A.M. a second plane crashed into the south tower. The a) In – at – as – for
resulting infernos caused the buildings to collapse, 1the south tower b) During – by – like – in
after burning for an hour and two minutes, the north tower twenty-
three minutes after that. 2The attacks were masterminded by Osama c) From – on – with – by
bin Laden in an attempt to intimidate the United States and unite d) On – for – like – by
Muslims for a restoration of the caliphate. e) In – for – as – by
9/11, as the happenings of that day are now called, has set
off debates on a vast array of topics. But I would like to explore a
lesserknown debate triggered by it. Exactly how many events took
place in New York on that morning _____2_____ September?

324 PROMILITARES.COM.BR
PREPOSITIONS

EXERCÍCIOS DE differed significantly from the novel was written. Principal photography

TREINAMENTO
took 44 days and the film was primarily shot in Oregon.
Twilight was theatrically released ______ November 21 2010,
grossing over US$392 million worldwide and became the most
purchased DVD of the year. The soundtrack was released in the same
year. Following the success of the film, New Moon and Eclipse, the next
01. (MACKENZIE 2010) two novels in the series, were produced as films the following year.
BREADWINNER
(Adapted from Wikipedia)
As a single mom, Cordia Harrington just needed to
bring home the bacon. Choose the correct prepositions to fill in the gaps above.
She ended up rolling in dough. a) of – off – at – in
By Margaret Heffernan b) about – to – over – at
c) under – for – off – in
Cordia Harrington was tired __(I)__ standing up all day and
smelling like French fries__(II)__ night. A property developer, she also d) on – from – at – on
owned and operated three McDonald’s franchises in Illinois, but as a
divorced mother __(III)__ three boys, she yearned __(IV)__ a business 03. (ESPCEX/AMAN 2019) Leia o texto a seguir e responda à questão.
that would provide __(V)__ her children and let her spend more time MANY GRADUATES EARN “PALTRY RETURNS” FOR THEIR DEGREE
__(VI) __ them. Mr Halfon, a former skills minister, stated in his speech that the
Her aha moment struck, strangely enough, after she was nation has “become obsessed _____(1)_____ full academic degrees”.
nominated in 1992 to be on the McDonald’s bun committee. “The “We are creating a higher education system that overwhelmingly
other franchisees, all men, thought that was hilarious because of favours academic degrees, while intermediate and higher technical
the word bun,” she recalls. “But the joke was on them: They didn’t offerings are comparatively tiny. The labour market does not need an
know the company would be picking me up in a corporate jet to see ever-growing supply of academic degrees. Between a fifth and a third
bakeries around the world. Every time I went to a meeting, I loved it. of our graduates take non-graduate jobs. The extra return for having
This was global!” a degree varies wildly according to subject and institution. For many,
The experience opened her eyes to business possibilities. When the returns are paltry.”
McDonald’s decided it wanted a new bun supplier, Harrington became Mr Halfon said that there is a strong need for intermediate skills.
determined to win the contract, even though she had no experience “There are skills shortages in several sectors. And there are millions
running a bakery. _____(2)_____ people who want to get on in life – preferably without
“You see a tiny crack in the door, and you have to run through it,” spending £50,000 on academic degrees,” he added. “There has been
she says. “I really believed I could do this.” growing concern about the amount of debt students are accumulating
Harrington studied the bakery business and made sure she was and the interest being charged on that debt.”
never off executives’ radar. “If you have a dream, you can’t wait for A spokesman for UUK (a representative organisation for the UK’s
people to call you,” she says. “So I’d visit a mill and send them photos universities) said: “Official figures are clear that, on average, university
of myself in a baker’s hat and jacket, holding a sign that said ‘I want graduates continue to earn substantially more than non-graduates
to be your baker.’” and are more likely to be in employment. A university degree remains
After four years and 32 interviews, her persistence paid off. an excellent investment.”
Harrington sealed the deal with a handshake, sold her franchises, “We must, however, be careful to avoid using graduate salaries as
invested everything she owned, and borrowed $13.5 million. She was the single measure of success in higher education. Many universities
ready to build the fastest, most automated bakery in the world. specialise in fields such _____(3)_____ the arts, the creative industries,
(Reader’s Digest) nursing and public sector professions that, despite making an essential
contribution to society and the economy, pay less on average.”
The prepositions that appropriately fill in blanks I, II, III, IV, V and VI, (Adapted from http://www.bbc.co.uk/news/education-42923529)
in the text, are:
Choose the alternative containing the correct words to respectively
a) off, at, of, for, with and on.
complete gaps (1), (2) and (3).
b) of, at, of, for, for and with.
a) at, of, to
c) in, at, of, at, with and on.
b) to, on, a
d) at, in, for, at, for and with.
c) by, on, that
e) of, in, with, for, with and on.
d) in, with, an

02. (EPCAR/AFA 2012) e) with, of, as

TWILIGHT 04. (EFOMM) I made seven different plans for my vacation,


Twilight is a 2008 American romantic vampire film based ______ however__________ the end I went to the Bahamas again.
Stephenie Meyer’s popular novel of the same name. It is the first film in a) to
The Twilight Saga film series. This film focuses on the development of b) by
the relationship between Bella Swan and Edward Cullen (a vampire),
c) with
and the subsequent efforts of Cullen and his family to keep Swan safe
______ a coven of evil vampires. d) in
The project was in development for approximately 3 years ______ e) at
Paramount Pictures, during which time a screen adaptation that

PROMILITARES.COM.BR 325
PREPOSITIONS

05. (PUC-RS 2016) Answer the question according to the text. c) Although we are married, we don’t see eye to eye on a lot of things.
ARCHEOLOGY: STARTING OUT d) I cried my eyes away when my friend told me I failed the exam.

When my longtime childhood ambition of becoming a surgeon e) Working with poor children opened my eyes off their real needs.
was derailed by the onset of adolescent 1squeamishness – specifically, a
dread of blood – I turned to archeology. The sere, crumbly atmosphere 08. (EN 2016) Mark the correct option.
of a dig, as I envisioned it, seemed a welcome relief from the maelstrom a) Beth got married with an American engineer last week.
I now perceived to be teeming within the human body. This was in the
b) After 30 minutes standing in line, he was tired of waiting.
nineteen-seventies, when archeology was a glamour profession.
c) The disadvantage in having a car is the need with insurance.
I decided to take a year off before starting college and devote
myself to salaried excavation in exotic places. I had grown up in San d) During his high school years, he had always been good in math
Francisco and had left the United States only for occasional family trips and chemistry.
to Mexico. I’m not sure which part of this vision most enthralled me: e) There are some differences of living in a house and living in a flat.
myself prying human bones and lustrous vessels from the soil of Asia
or Africa, or the forgotten lives I pictured humming just 2__________ 09. (EN 2016) Analyze these sentences.
that soil, awaiting my discovery. After weeks of anxiously checking
I. The boss discussed about the new sales report.
the mail for job offers and plane tickets, I received a single reply, from
a professor at Berkeley. His avuncular tone failed to entirely blunt the II. Does the coefficient of kinetic friction depend on speed?
gist of his message: Our graduate students pay us to come on digs. III. My son finally succeeded in finding a new job.
And you are not even remotely qualified. IV. Some people still blame the driver on the accident.
Stung, I turned to some of the small pay-to-participate digs I’d V. He apologized for his girlfriend to being late.
seen advertised in the newsletter. In September of 1980, as most
of my high-school friends were starting college, I shelled out two Choose the correct option.
or three hundred dollars plus airfare (my earnings from long hours a) Only I and II are grammatically correct.
3
__________ the counter of a Haight Street cafe) to join a three-week
b) Only II and III are grammatically correct.
dig in Kampsville, Illinois.
c) Only II and IV are grammatically correct.
The exoticism of Kampsville was not the sort I’d craved. The real
shock was the square metre of earth – delineated by strings attached d) Only I and IV are grammatically correct.
to pegs – that was the extent of my archeological domain. We weren’t e) Only II and V are grammatically correct.
allowed to sit on our squares, only to squat. Nor were we to dig on
our dig, only to skim away fine layers of earth with a scalpel, lowering 10. (EN 2016) Which is the correct option to complete the sentence
the surface of our metre over the course of days, until the objects below?
embedded there – projectile points or pottery shards – rested on top.
This soil-shaving took place 4__________ a scouring sun, in ninety- 25 SIMPLES WELL-BEING TRICKS TO HEALTH-PROOF YOUR BODY
degree temperatures. By day two, I was craving stewed prunes long
Let’s be honest, we could all do with looking _____ ourselves
before lunchtime. By day three, I’d renounced my goal of becoming
better. And if you follow these simple well-being tricks to health-proof
an archeologist.
your body, you’ll soon feel the benefits.
Still, the archeology fantasy had been irrevocably dispelled, and
Here are 25 instant body boosters from top to toe.
by October I was back at my cafe job with a fresh goal: save enough
money to travel to Europe. But my sojourn in Kampsville has stayed (http://www.mirror.co.uk/lifestyle/health/25-simple-health-tips-boost-2305412)
with me – the sensation I had of scraping away the layers 5__________
myself and a lost world, in search of its occupants. a) for d) after
(Egan, Jennifer. The New Yorker 87.17 (Jun 13-Jun 20, 2011): n/a (adapted)) b) up e) into
c) to
Answer the question considering the words that correctly and
respectively complete the blanks in references 2, 3, 4, and 5.
According to the text, the words that fit in the blanks are:
EXERCÍCIOS DE
a) beneath – behind – under – between
b) beneath – between – behind – under
c) between – behind – under – beneath
COMBATE
d) behind – under – beneath – between
e) behind – beneath – between – under 01. (EEAR) Choose the best alternative to complete the blanks:
“Julie was born __________ July 3, __________ night__________
06. (EFOMM 2018) Which alternative is correct? New York.”

a) Can you translate this in Chinese? a) in / at / at c) in / at / in

b) I broke it into half. b) on / at / in d) on / in / at

c) Cut the onion into small pieces.


02. (EFOMM) Choose the best option to complete the sentence:
d) The ball rolled slowly in the goal.
“Bell used electricity to send the human voice __________ one
e) He sat down into the armchair, and I sat down onto the floor. place __________ another.”
a) on – in d) at – to
07. (EFOMM 2018) Choose the correct sentence.
b) from – to e) above – below
a) My father always kept a close eye in me when I played with my friends.
c) in – to
b) There must be more to him than meets my eyes, or else why
would she be interested in him?

326 PROMILITARES.COM.BR
PREPOSITIONS

03. (EEAR) Choose the alternative that best completes the sentence below. 09. (ITA) “It’s clear that Gossard and the rest of Pearl Jam no longer
“I stayed in London ___ a few days during my vacation. I came want to ‘rely’ __________ anger and craziness to drive the band.”
back to Brazil ___ May 1st.” A preposição que deve acompanhar o verbo “rely”, relacionado no
a) in – at texto, é:
b) for – in a) at
c) in – on b) on
d) for – on c) in
d) for
04. (ITA) “__________ what he says, she was born __________ March e) with
25, 1970.”
a) According to – in 10. (EFOMM) Which option is correct?
b) According with – on “She was afflicted ______ severe asthma.”
c) Accordance to – in a) to
d) According to – on b) by
e) Accordance with – in c) of
d) with
05. Choose the correct option to complete the sentences below. e) at
I. She congratulated me ______passing the driving test.
II. My parents discouraged me _____ quitting my job.
III. She got married______a foreigner. GABARITO
IV. Many young people dream______living abroad.
EXERCÍCIOS DE FIXAÇÃO
V. The mayor was forced to resign ______ his position.
01. B 04. B 07. B 10. E
a) for / from / with /about / of 02. E 05. E 08. E
b) for / for / with / about / of 03. E 06. C 09. B
c) on / from / to / of / from EXERCÍCIOS DE TREINAMENTO
d) on / from / to / with / from 01. B 04. D 07. C 10. D
e) by / of / to / with / from 02. D 05. A 08. B
03. E 06. C 09. B
06. (EN) Choose the correct option to complete the text below.
EXERCÍCIOS DE COMBATE
INTERNATIONAL CONGRESS 07. C 10. D
01. B 04. D
Join us ______ our 2016 International Educational Conference
02. B 05. C 08. A
_____ Orlando, at Disney’s Boardwalk Inn! The Boardwalk is located
within the Walt Disney World Resort and 10 minutes away _____ the 03. D 06. D 09. B
Epcot Theme Park.
(Abridged from http://www.eluteinstitute.com/education-conferences/)
ANOTAÇÕES

a) in / in / at
b) on / in / under
c) at / at / under
d) at / in / from

07. (CFOE) “They swam ______ a river ______ Mato Grosso to escape
_________ the police.”
a) through – in - of
b) in – on- to
c) across – in - from
d) along – at – out of

08. (CN) Complete the paragraph with a proper preposition.


“I'm Hannah and I work ______ an office in London. During the
week, I get up ______ six-thirty. I go _____ work by subway, but _____
Sundays I like waking up late because I don't work _____ weekends.”
Choose the right option to fill in the gaps with the correct prepositions.
a) in / at / to / on / on
b) at / at / to / in / at
c) on / about / at / at / on
d) at / about / at / on / at
e) in / about / to / in / at

PROMILITARES.COM.BR 327
PREPOSITIONS

ANOTAÇÕES

328 PROMILITARES.COM.BR
RELATIVE PRONOUNS AND
ADVERBS

PRONOMES RELATIVOS As orações “nonrestrictives” ou “nondefining” não definem ou


identificam o(s) substantivo(s); elas apenas dão informações extras
• Who; That - usados para pessoa, função na frase de sujeito. que são desnecessárias para a identificação do(s) substantivo(s).
Exemplo: Exemplos:
The man who sells fish in the market is john. President Ronald Reagan, who was shot, governed the USA
during the cold war.
• Who; That; Whom usados para pessoas, função na frase de The sun, which appears in many ancient drawings, was an
objeto. important religious symbol.
Exemplo:
Observação
The man who/whom you saw in the market is john.
Nesses casos os pronomes “who” e “which” não podem ser
substituídos por “that”.
• Whose usado para pessoas e indica possessão.
E o pronome “whose” é usado em orações “restrictive” assim
Exemplo: como em orações “nonrestective”, o pronome “whose” não pode
The writer whose books are bestsellers is Brazilian. ser omitido.

• Which; That usados para objetos ou animais, função na frase


de sujeito.
PRONOMES RELATIVOS
Exemplo:
The TV set which (that) is at the right side of the window is mine.
NA FUNÇÃO DE SUJEITO
Os pronomes relativos quando aparecem entre o sujeito e o verbo
The refrigerator which/that is my kitchen is from the USA.
não podem ser omitidos da frase, pois fazem papel de sujeito.
Exemplos:
• Which - That usados para objetos ou animais, função na frase
Pessoas:
de objeto.
The man who (that) died was John, my neighbor.
Exemplo:
I spoke to John, who was very bright.
The carpet which/that I bought in São Paulo is in the living room.
(Apenas o pronome “who” pode ser usado aqui.)

• Of Which - Whose usados para animais ou objetos e indica


possessão. Animais ou coisas:
Exemplo: The building which (that) is the most famous is “Avenida Rio
Branco 1”.
The house whose windows are big is John’s.
Architects draw the new building, which is near Tijuca.

ORAÇÕES ADJETIVAS
PRONOMES RELATIVOS
(ADJECTIVES CLAUSES)
Orações Adjetivas informam a respeito do substantivo ou
NA FUNÇÃO DE OBJETO
pronome, sujeitos ou não. Tais orações geralmente aparecem logo Os pronomes relativos quando substituem o objeto do verbo ou
depois das palavras que elas modificam. Temos dois tipos de Orações objeto de uma preposição podem ser omitidos da oração, menos
Adjetivas: “restrictive” (defining) ou “nonrestrictive” (nondefining). quando a oração for “nonretrictive” ou “non defining”.
As orações “restrictive” ou “defining” identificam o(s) Exemplos:
substantivo(s); elas descrevem; elas dão informações que são Pessoas:
necessárias que são importantes para o sentido da oração. The man who/whom John sold the dvd is peter.
Exemplos: (O pronome relativo pode ser omitido nesse caso.)
Many of the men who (that) work at Petrobras have medical care.
Most of the cars which (that) the police bought came from England. Animais ou coisas:
Observação John wrote about the American History, which he had studied
for years.
Nesses casos os pronomes “who” e “which” podem ser
(Nesse caso, em oração “nonrestrictive” ou “nondefining” o
substituídos por “that”.
pronome não pode ser omitido.)

PROMILITARES.COM.BR 329
RELATIVE PRONOUNS AND ADVERBS

PRONOMES OBJETOS DA PREPOSIÇÃO EXERCÍCIOS DE

Quando a preposição aparecer antes do pronome relativo, apenas


o “whom” ou o “which” podem ser usados, mas caso a preposição
FIXAÇÃO
apareça no final de uma oração “restrictive” o pronome “that” pode
ser usado ou o pronome relativo pode ser omitido.
01. (EEAR 2019) Read the text and answer question.
Exemplos:
When “Star Wars: The Force Awakens” opens on Dec. 18 and, for
Pessoas:
the first time in nearly a decade, moviegoers can return to that galaxy
The historians were British. Many ideas to the book came from of long ago and far, far away, they will find that it contains two new
them. stars. The film, __________ is directed by J. J. Abrams and continues
The historians from whom many ideas came were British. the interstellar saga of Luke Skywalker, Princess Leia and Han Solo
The historians who (that) many of our ideas came from were some 30 years after the events of “Return of the Jedi” (1983), is also a
British. launching pad for two young actors who are barely half as old as the
“Star Wars” franchise itself.
Daisy Ridley, who plays a mysterious scavenger named Rey, and
Animais ou coisas: John Boyega, who plays a disaffected stormtrooper named Finn, are
The books were written by Brazilians. Most of the information among the new heroes of “The Force Awakens” and are bracing
came from these books. themselves for the biggest roles of their careers.
The books from which most of the information came were written (www.nytimes.com.br)
by Brazilians.
Choose the best alternative to complete the text.
The books (that, which) most of the information came from were
written by Brazilians. a) whose
b) which
USO DO “WHERE” c) where
Exemplos: d) who
This is a farm where (on which) a famous writer was brought up.
02. (UNESP 2017) Examine a tira e o texto, para responder à questão
This is Easter Island, where (on which) John Kennedy lived for years.
a seguir.

PRONOMES RELATIVOS COM


EXPRESSÕES DE “QUANTITY”
Exemplos:
They selected five candidates. One candidate would get the job.
They selected five candidates, one of whom would get the job.
These statues are world famous. Many of them weigh over twenty
tons.
These statues, many of which weigh over twenty tons, are world
famous.

WHAT → WHICH

What = O que → o que antecede é desconhecido.


Exemplo: Tell me What she said = Tell me the things that she said.

Which = O que → o que acontece está na oração anterior.


Exemplo: He invited us to the meeting which was very good.

RELATIVES ADVERBS

Ligação de orações usando conectivos de “TIME”, “PLACE” OU


“REASON” “A study from Brigham Young University reported that teenagers
________ sleep seven hours, compared to nine hours of sleep perform
When → in/ on which better academically. This study contradicts federal guidelines, stating
Exemplo: We don’t know the day on which (= When) he’ll come. teenagers should sleep as much as they need to.” - THESTATECOLUMN.
COM
Where → in/ at which (http://lolalollipop.com. Adaptado)
Exemplo: That’s the house at which (= Where) she has been
raised. Assinale a alternativa que completa corretamente a lacuna no texto:
a) which d) whoever
Why → for which
b) when e) who
Exemplo: I don’t know the reason for which (= why) he is late.
c) while

330 PROMILITARES.COM.BR
RELATIVE PRONOUNS AND ADVERBS

03. (UNIFESP 2017) Leia o texto para responder à questão a seguir. The best alternative to complete the text is:
a) that / how c) that / which
b) when / that d) which / when

06. (EEAR 2017)


CAN YOU BE TOO OLD TO DONATE BLOOD?
In general, you can never be too old to donate blood. Though you
can be too young or too thin.
The American Association of Blood Banks used to bar people over
65 from donating blood, but it scrapped the rule in 1978 after studies
found that older people _____ stored their own blood prior to surgery
did well, said Dr. Steven Kleinman, the association’s senior medical
adviser. Now most blood banks are happy to accept blood from older
volunteers.
(Adapted from: http://well.blogs.nytimes.com)

In developing countries there are high levels of what is known


Choose the best alternative to complete the blank in the text.
as “food loss”, which is unintentional wastage, often due to poor
equipment, transportation and infrastructure. In wealthy countries, a) which c) when
there are low levels of unintentional losses but high levels of “food b) where d) who
waste”, which involves food being thrown away by consumers
because they have purchased too much, or by retailers who reject 07. (EEAR 2016)
food because of exacting aesthetic standards.
ANOTHER PLANE RETURNS TO GUARULHOS AIRPORT
(www.theguardian.com)
For the second day consecutive, an airplane had to return to
No trecho “which involves food being thrown away by consumers”, Guarulhos airport, after being hit by birds.
o termo em destaque se refere a
The incident happened on Wednesday (27) with a Boeing 737-
a) food. 800 belonging to the airline Gol _____ took off for Fortaleza. The bird
b) wealthy countries. entered one of the two motors, causing serious troubles. Because of
the problem, the plane returned to Guarulhos almost two hours after
c) food loss.
taking off. The aircraft had to use fuel to land with less weight.
d) consumers.
Choose the word to have the text completed:
e) food waste.
a) which
04. (EEAR 2017) b) whom
ECONOMIC CRISIS INCREASES CONSUMPTION OF c) whose
RICE AND BEANS IN BRAZIL d) who
The economic crisis is making the Brazilian consumer exchange
meat for the traditional dish of rice and beans. High unemployment 08. (EFOMM 2018) Choose the correct sentence.
and falling incomes, together with the low prices of these products, a) I saw a girl who hair was blue.
caused by good harvest, are responsible for the increase in demand,
b) Mr. Smith, which has never smiled, seems very happy today.
__________ will be 15% to 20% this month, compared to the
prediction for the year. The average consumption per capita is around c) He got fired again, what surprised everybody.
3, 5 kilos of rice and 1, 5 kilo of beans. d) The girl who I gave the form to was very nervous.
(Fonte: Folha de São Paulo – Internacional -10/05/2017) e) It was like a dream from that I feared I would wake sooner or later.
Choose the alternative that best completes the blank in the text:
09. (ESPCEX/AMAN 2016)
a) who
b) which CHILDREN EXPERIENCE BASIC TRAINING AND MOCK DEPLOYMENT
c) whom The 460th Force Support Squadron (FSS) hosted the 6th Annual
Operation Future Forces (OFF) Sept. 13, 2014, at Camp Rattlesnake.
d) whose
OFF allows children ages 8-18 to experience what military members
endure from basic training to technical school to a mock deployment,
05. (EEAR 2017) ending with a homecoming party.
JETBLUE... THE CRISIS? NEVER HEARD OF IT... “The youth mock deployment was developed to alleviate many of
The domestic air transport market in the USA must seem highly the stresses commonly experienced by young family members when
unstable to newcomers, including low-cost carriers. Of the 82 airlines one or both parents are deployed,” said Thomas Cox, 460th FSS youth
formed in ten years ______ followed deregulation in 1978, only two program chief. “Everything from basic training, tech schools, camp
have survived - America West and Mid-west Express. Statistics show activities and accomplishing their mission as a team made the event a
that three-quarters of all projects never even get off the ground and one of a kind opportunity for military kids.”
that most of the others crash in less than five years. A few of the boys had their heads shaved before heading off to
Paradoxically, Southwest Airlines, the No.1 success story in the “Basic Military Training.” The training consisted of doing push-ups
US air transport today, was founded before deregulation (in 1972), and sit-ups, jumping through hula-hoops and running through an
though it did take advantage of it. Many of those who tried to imitate inflatable castle. They were also taught how to stand at attention,
the Southwest model came to grief. Not JetBlue, ______ just four salute and do an about-face.
years after it was formed is continuing to grow at a spectacular rate.
(Fonte: Re-vista Planet Aero-Space)

PROMILITARES.COM.BR 331
RELATIVE PRONOUNS AND ADVERBS

After basic training, each child attended “technical school” and "There are many theories about the origin of this word, and since
was taught a specific Air Force Specialty Code to help them in their it arose in oral use, we may never know the answer for sure," Martin
mock deployment. Some children were taught lifesaving self-aid and said. "We think the most likely theory is that it is an alteration of work,
buddy care skills while others learned about the importance of radio because that word has a history of 4being used in similar ways, with
communications while on a deployment. dancers being encouraged to 'work it.' The 't' could be a result of
The tech-school graduates were then issued water pistols before blending with another word such as twist or twitch."
heading out. During their deployment, the children encountered "Twerk" will be added to the dictionary as part of its quarterly
hostile and non-hostile citizens, a water-balloon fight and injured allies update, 1which includes words such as "selfie," the word typically
who needed help along the way. used to describe pouty smartphone self-portraits, "digital detox" for
At the end of a long day, loved ones waited outside the youth time spent way from Facebook and Twitter, and "Bitcoin," for the
center on base with homemade signs welcoming the “troops” back nationless electronic currency, 2whose gyrations have also caught the
home. world's eye.
“This event was great,” said Senior Airman Jasmine Madison, Oxford Dictionaries is responsible for a range of reference works,
460th FSS Force Support Force Management technician. “It’s a way including Oxford Dictionaries Online, which focuses on modern usage,
for kids to get a hands-on understanding of what their parents do and the historically-focused Oxford English Dictionary, which probably
when they are separated from them during deployments.” won't be adding "twerk" to its venerable pages any time soon.
(Adapted from http://www.buckley.af.mil/news/story.asp?id=123424927) The definition: "Twerk, v.: dance to popular music in a sexually
6
provocative manner involving thrusting hip movements and a low,
In the sentence “During their deployment, the children encountered squatting stance."
hostile and non-hostile citizens, a water-balloon fight and injured allies (By RAPHAEL SATTER Associated Press (www.mercurynews.com)
who needed help along the way.” (paragraph 5), the words “their” Accessed on: august 10th, 2014.)
and “who” respectively refer to
The words in bold: “which” (ref. 1) and “whose” (ref. 2) are
a) the children and citizens.
consecutively related to:
b) water pistols and injured allies.
a) part and Bitcoin
c) the children and injured allies.
b) twerk and gyrations
d) deployment and non-hostile citizens.
c) quarterly update and Bitcoin
e) graduates and a water-balloon fight.
d) dictionary and currency
10. (UEMG 2015) Read the passage below to complete the gaps with e) words and world´s eye
the relative pronouns (1 - 4):
02. (IFBA 2018)
“Online friends are people _______ always post messages and
pictures of the places _______ they are, _______ they are with and SUSTAINABLE TOURISM
______ they are doing.” Tourism is one of the world’s fastest growing industries and is a
1. what major source of income for many countries. Being a people-oriented
industry, tourism also provides many jobs 1which have helped 2revitalise
2. who
local economies.
3. whom
However, like other forms of development, tourism can also cause
4. where 3
its share of problems, such as social 4dislocation, loss of cultural
The CORRECT sequence is: heritage, economic 5dependence and ecological 6degradation.
Learning about the impacts of tourism has led many people to seek
a) (1), (3), (2), (4) more responsible holidays. 7These include various forms of alternative
b) (3), (1), (4), (2) or sustainable tourism such as: ‘nature-based tourism’, ‘ecotourism’
c) (4), (2), (1), (3) and ‘cultural tourism’. Sustainable tourism is becoming so popular
that some say that what we 8presently call ‘alternative’ will be the
d) (2), (4), (3), (1)
‘mainstream’ in a decade.
All tourism activities of whatever motivation – holidays, business
EXERCÍCIOS DE travel, conferences, adventure travel and ecotourism – need to

TREINAMENTO
be sustainable. Sustainable tourism is defined as “tourism that
respects both local people and the traveller, cultural heritage and
the environment”. It seeks to provide people with an exciting and
educational holiday 9that is also of benefit to the people of the host
country.
01. (UDESC 2015) (Adapted from: http://www.unesco.org/education/tlsf/mods/theme_c/mod16.html.
'TWERKING' BOUNCES INTO OXFORD DICTIONARY Access on Jul 24th, 2017.)

LONDON – Twerking, the rump-busting up-and-down dance


Concerning grammar reference, mark the sentences below true (T)
move long beloved on America's hip-hop scene, has officially gone
or false (F):
mainstream. 5It's got the English dictionary entry to prove it.
I. The relative pronoun which (ref. 1) refers to economy.
Britain's Oxford Dictionaries said the rapid-fire gyrations employed
by U.S. pop starlet Miley Cyrus to bounce her way to the top of the II. The possessive adjective its (ref. 3) refers to tourism.
charts had become 3increasingly visible in the past 12 months and III. The demonstrative pronoun these (ref. 7) refers to people.
would be added to its publications under the entry: "Twerk, verb." IV. The relative pronoun that (ref. 9) refers to holiday.
Although Cyrus's eye-popping moves at Monday's MTV Video
Music Awards may have been many viewers' first introduction to Now, check the correct alternative below.
the practice, Oxford Dictionaries' Katherine Connor Martin said a) T, T, F, F. c) F, T, T, F. e) F, T, F, T.
"twerking" was some two decades old. b) T, F, T, F. d) F, F, T, T.

332 PROMILITARES.COM.BR
RELATIVE PRONOUNS AND ADVERBS

03. (UPF 2018) 05. (ESFCEX 2016) Which of the relative clauses below modifies the
BRAZIL HAS OPENED A MASSIVE SWATH object of the sentence?
OF THE AMAZON TO MINING. a) The boy who lives near me has four cats.
Ivana Kottasová
b) Stories which end in death make me cry.
c) The idea that she could fly is completely nonsense.
d) The girl whose father died yesterday will move to China.
e) I can’t find the person to whom you talked.

06. (ITA 2016) Marque a opção em que o item sublinhado NÃO exerce
a função de agente da oração.
a) [...] Patrick Makanza, 51, an M.B.A. and veteran of Unilever and
Barclays Bank, who quit a cushy job at a top Zimbabwe private
equity firm [...]
The government has abolished a reserve that 1straddles the
b) That fund will make loans [...] for an average of four years to small
northern states of Pará and Amapá, a move that opens the vast area
and medium-size businesses that want to expand [...]
to mineral exploration and commercial mining. The reserve, 2which
was established in 1984, is huge: It covers 18,000 square miles, an c) [...] Tracy Washington [...] lobs a personal query at Makanza, a
area twice the size of New Jersey. father of four who is partial to conservative business suits and
golf.
Brazil said that mineral extraction would only be allowed in areas
where there are no conservation controls or indigenous lands. An d) Makanza responds that he worked in venture capital back in the
official report from 2010 said that up to two-thirds of the reserve is 1990s and came to miss the highs and lows of investing in early-
subject to such protections. stage entrepreneurs.
The government, 3which has previously said that the region is rich e) Impact investing – which aims to produce both financial and social
in minerals, gold and iron, framed the decision as an effort to bring or environmental returns – is in vogue.
new investment and jobs to a country that recently emerged from the
longest recession in 4its history. 07. (IME 2014) Para a questão, encontram-se em destaque cinco
Brazil announced a plan in July to revitalize 5its mining sector, termos. Assinale a alternativa correspondente ao termo cujo emprego
and increase its share of the economy from 4% to 6%. The industry está INCORRETO.
employs 200,000 people in a country where a record 14 million are If mankind can learn to respect other human beings in thoughts,
out of work. The government wants to encourage more growth, and words, and actions, humanity may survive on this planet, Earth. If
has announced plans to open 10% of all protected rainforest areas to parents teach children clearly not only to respect their elders but to
mining. The true scale of mining in the country is unknown because treat everyone with respect and courtesy, children may grow up to be
small, illegal mining operations are difficult to track. responsible adults whose influence other people to respect human
The elimination of the reserve sparked an immediate backlash feeling, rights and property. They may grow up to cherish human life,
from activists and environmental groups. Opposition politicians called not annihilate it. All people want respect, so they must give it to earn it.
it "the biggest crime against the Amazon forest since the 1970s." a) on
World Wildlife Fund Brazil warned that deforestation would result, b) but
along with a loss of biodiversity and water resources. It said that even
c) whose
areas that remain under formal protection are at risk. "Opening up
these areas for mining without discussing environmental safeguards d) not
is a social and environmental international affront," said Mauricio e) All
Voivodic, executive director of WWF Brazil. "A gold rush in the region
will create irreversible damage to local cultures as well," he added. 08. (EPCAR/AFA 2020) Read the text below and answer the question
Deforestation and mining are destroying the rainforest at a according to it.
stunning rate. The Rainforest Foundation estimates that about 1 acre MUSIC THERAPY WITH CANCER PATIENTS
is wiped out every second, and an estimated 20% of the rainforest has
been destroyed over the past 40 years. The Amazon covers 1.2 billion Cancer is the second leading cause of death in the United States, in
acres and produces 20% of the world's oxygen. Germany and in many other industrialized countries. In 2007, about 12
million people were diagnosed with cancer worldwide with a mortality
(Vasco Cotovio contributed reporting.)
rate of 7.6 million (American Cancer Society, 2007). In the industrial
The expressions ‘which’ (reference 2); ‘which’ (reference 3); ‘its’ countries, 1the most commonly diagnosed cancers in men are prostate
(reference 4) and ‘its’ (reference 5) refer, respectively, to: cancer, lung cancer and colorectal cancer. Women are most commonly
diagnosed with breast cancer, gastric cancer and lung cancer.
a) Pará and Amapá / region / job / July.
The symptoms of cancer depend on the type of the disease, but
b) New Jersey / country / decision / mining sector. there are common symptoms caused by cancer and/or by its medical
c) reserve / decision / investment / economy. treatment (e.g., chemotherapy and radiation). Common physical
d) commercial mining / investment / effort / industry. symptoms are pain, fatigue, sleep disturbances, loss of appetite,
nausea (feeling sick, vomiting), dizziness, limited physical activity, hair
e) reserve / government / country / Brazil.
loss, a sore mouth/throat and bowel problems. 2Cancer also often
causes psychological problems such as depression, anxiety, mood
04. (EEAR 2017) Fill in the blank with the correct pronoun. disturbances, stress, insecurity, grief and decreased self-esteem. This,
“An archeologist is a man _____ work is the study of ancient things.” in turn, can implicate social consequences. Social isolation can occur
a) whose due to physical or psychological symptoms (for example, feeling too
tired to meet friends, cutting oneself off due to depressive complaints).
b) which
Besides conventional pharmacological treatments of cancer,
c) how there are treatments to meet psychological and physical needs of the
d) who

PROMILITARES.COM.BR 333
RELATIVE PRONOUNS AND ADVERBS

patient. Psychological consequences of cancer, such as depression, express the wish to become aware of themselves again. They may
anxiety or loss of control, can be counteracted by psychotherapy. wish to grapple with negative emotions due to their disease. Other
For example, within cognitive therapy cancer patients may develop patients wish to experience positive feelings, such as enjoyment and
3
coping strategies to handle the disease. Research indicates that vitality.
4
music therapy, which is a form of psychotherapy, can have positive The results indicate that music therapy can also have positive
effects on both physiological and psychological symptoms of cancer influences on well-being of cancer patients in the post-hospital curative
patients as well as in acute or palliative situations. stage as well as they offer valuable information about patients' needs
There are several definitions of music therapy. According to the in this state of treatment and how effects can be dealt with properly.
World Federation of Music Therapy (WFMT, 1996), music therapy is: (Adapted from https://essay.utwente.nl/59115/1/scriptie_F_Teiwes.pdf -
“the use of music and/or its music elements (sound, rhythm, melody Access on 25/02/19)
and harmony) by a qualified music therapist, with a client or group,
in a process designed to facilitate and promote communication, Mark the sentence in which “that” can correctly replace the pronoun.
relationship, learning, mobilization, expression, organization, and a) Music therapy, which is a form of psychotherapy, can have positive
other relevant therapeutic objectives, in order to meet physical, effects (reference 4).
emotional, mental, social and cognitive needs”. b) A methodological form of assistance in which musical means are
The Dutch Music Therapy Association (NVCT, 1999) defines used (reference 5).
music therapy as “5a methodological form of assistance in which c) For patients who have difficulties in expressing emotions [...]
musical means are used within a therapeutic relation to manage (reference 7).
changes, developments, stabilisation or acceptance on the emotional,
behavioural, cognitive, social or on the physical field”. d) Patients often express the wish to become aware of themselves
again (reference 13).
The assumption is that the patient's musical behaviour conforms
6

to their general behaviour. The starting points are the features of


09. (ITA 2018) Leia o que segue.
the patient's specific disorder or disease pattern. There is an analogy
between psychological problems and musical behaviour, which means GOODBYE THINGS, HELLO MINIMALISM:
that emotions can be expressed musically. 7For patients who have CAN LIVING WITH LESS MAKE YOU HAPPIER?
1

difficulties in expressing emotions, music therapy can be a useful


Fumio Sasaki owns a roll-up mattress, three shirts and four
medium. Music therapy might be a useful intervention for breast
pairs of socks. After deciding to scorn possessions,
cancer patients in order to facilitate and enhance their emotional
he began feeling happier. He explains why.
expressivity. 8Besides analogy, there are further qualities of music that
can be beneficial within therapeutic treatment. One of these qualities is Let me tell you a bit about myself. I’m 35 years old, male, single,
symbolism: music can symbolize persons, objects, incidents, experiences never been married. I work as an editor at a publishing company. I
or memories of daily life. 9Therefore, music is a reality, which represents recently moved from the Nakameguro neighbourhood in Tokyo,
another reality. The symbolism of the musical reality enables the patient where I lived for a decade, to a neighbourhood called Fudomae in a
to deal safely with the other reality 10for it evokes memories about different part of town. 2The rent is cheaper, but the move pretty much
persons, objects or incidents. These associations can be perceived as wiped out my savings.
positive or negative, so they release emotions in the patient. Some of you may think that I’m a loser: an unmarried adult with
Music therapy both addresses physical and psychological needs not much money. The old me would have been way too embarrassed
of the patient. Numerous studies indicate that music therapy can be to admit all this. I was filled with useless pride. But I honestly don’t
beneficial to both acute cancer patients and palliative cancer patients care about things like that any more. The reason is very simple: I’m
in the final stage of disease. perfectly happy just as I am. The reason? I got rid of most of my
Most research with acute cancer patients receiving chemotherapy, material possessions.
surgery or stem cell transplantation examined the effectiveness of Minimalism is a lifestyle in which 3you reduce your possessions
receptive music therapy. Listening to music during chemotherapy, either to the least possible. Living with only the bare essentials has not only
played live by the music therapist or from tape has a positive effect on provided superficial benefits such as the pleasure of a tidy room or the
pain perception, relaxation, anxiety and mood. There was also found a simple ease of cleaning, 4it has also led to a more fundamental shift.
decrease in diastolic blood pressure or heart rate and an improvement It’s given me a chance to think about what it really means to be happy.
in fatigue; insomnia and appetite loss could be significantly decreased We think that 5the more we have, the happier we will be. 6We
in patients older than 45 years. Further improvements by receptive never know what tomorrow might bring, so we collect and save as
music therapy were found for physical comfort, vitality, dizziness and much as we can. This means we need a lot of money, so we gradually
tolerability of the chemotherapy. A study with patients undergoing start judging people by how much money they have. You convince
surgery found that receptive music therapy led to decreased anxiety, yourself that you need to make a lot of money so you don’t miss out
stress and relaxation levels before, during and after surgery. Music on success. And for you to make money, you need everyone else to
therapy can also be applied in palliative situations, for example to spend their money. And so it goes.
patients with terminal cancer who live in 11hospices.
So I said goodbye to a lot of things, many of which I’d had for
Studies indicate that music therapy may be beneficial for cancer years. And yet now I live each day with a happier spirit. 7I feel more
patients in acute and palliative situations, but the benefits of music content now than I ever did in the past.
therapy for convalescing cancer patients remain unclear. Whereas music
therapy interventions for acute and palliative patients often focus on I wasn’t always a minimalist. I used to buy a lot of things, believing
physiological and psychosomatic symptoms, such as pain perception that all those possessions would increase my self-worth and lead to
and reducing medical side-effects, 12music therapy with posthospital a happier life. I loved collecting a lot of useless stuff, and I couldn’t
curative treatment could have its main focus on psychological aspects. throw anything away. I was a natural hoarder of knick-knacks that I
A cancer patient is not free from cancer until five years after the thought made me an interesting person.
tumour ablation. The patient fears that the cancer has not been At the same time, though, I was always comparing myself with
defeated. In this stage of the disease, patients frequently feel insecure, other people who had more or better things, 8which often made me
depressive and are emotionally unstable. How to handle irksome and miserable. I couldn’t focus on anything, and I was always wasting
negative emotions is an important issue for many oncology patients. time. Alcohol was my escape, and I didn’t treat women fairly. I didn’t
After the difficult period of the medical treatment, which they often try to change; I thought this was all just part of who I was, and I
have overcome in a prosaic way by masking emotions, 13patients often deserved to be unhappy.

334 PROMILITARES.COM.BR
RELATIVE PRONOUNS AND ADVERBS

My apartment wasn’t horribly messy; if my girlfriend was coming Accelerator, a first-of-its-kind venture whose initial investors include
over for the weekend, I could do enough tidying up to make it look Microsoft cofounder (and world‘s richest man) Bill Gates. [...]
presentable. On a usual day, however, there were books stacked O termo “whose” em: “They’ll head home with golden contacts
everywhere because there wasn’t enough room on my bookshelves. (investor cocktail hours were built into the packed schedule) and
Most I had thumbed through once or twice, thinking that 9I would a commitment for up to $500,000 in seed capital from Capria
read them when I had the time. Accelerator, a first-of-its-kind venture whose initial investors include
The closet was crammed with what used to be my favorite Microsoft cofounder (and world’s richest man) Bill Gates.”, refere-se a
clothes, most of which I’d only worn a few times. The room was filled a) golden contacts.
with all the things I’d taken up as hobbies and then gotten tired of.
A guitar and amplifier, covered with dust. Conversational English b) a commitment.
workbooks I’d planned to study once I had more free time. Even a c) seed capital.
fabulous antique camera, 10which of course I had never once put a d) Capria Accelerator.
roll of film in.
e) Bill Gates.
It may sound as if I’m exaggerating when I say I started to
11

become a new person. Someone said to me: “All you did is throw
things away,” which is true. 12But by having fewer things around, I’ve
started feeling happier each day. I’m slowly beginning to understand EXERCÍCIOS DE
what happiness is.
If you are anything like I used to be – miserable, constantly
comparing yourself with others, or just believing your life sucks – 13I
COMBATE
think you should try saying goodbye to some of your things. […]
Everyone wants to be happy. But trying to buy happiness only makes 01. (ITA) “______are these shoes and ______ is this hat?”
us happy for a little while.
a) Who; whom
(Fonte: adaptado de <https://www.theguardian.com/books/2017/apr/12/
goodbye-things-hello-minimalism-can-living-with-lessmake-you-happier>. b) Those; that
Acesso em: 21 mai. 2017.) c) These; this
As palavras sublinhadas nos excertos da coluna I foram utilizadas d) Whose; whose
tendo os referentes respectivamente indicados na coluna II. e) What; which

COLUNA I COLUNA II 02. (EPCAR/AFA 2017) In the sentence “there was a single kind that
could be measured by standardized tests”, it is possible to find an
I. ... it has also led to a more living with only the bare option to substitute the pronoun “that” accordingly in
fundamental shift. (ref. 4) essentials
a) when
II. … which often made me other people who had more or b) which
miserable. (ref. 8) better things
c) how
III. ... I would read them when I d) whom
my bookshelves
had the time. (ref. 9)

IV. … which of course I had 03. (CFOE) In “... our ancestors who enjoyed spices with their food,
a fabulous antique camera were apt to live longer...”, the Relative Pronoun can be replaced by
never once … (ref. 10)
a) that.
Estão corretas b) what.
a) I, II e III. c) which.
b) I e III. d) whose
c) I e IV.
d) II, III e IV. 04. (AFA) In the sentence “there is a number of theories that attempt
to explain the link”, it is possible to find an option to substitute the
e) todas.
pronoun accordingly in

10. (ITA 2016) Leia o texto. a) when.

INSIDE THE BILL GATES-BACKED ACCELERATOR THAT'S TRAINING b) how.


THE NEXT GENERATION OF VENTURE CAPITALISTS c) whom.
Lauren Gensler, FORBES STAFF d) which.

In an airy converted furniture store in Seattle‘s Pioneer Square 05. (EN) Which of the options completes the sentence correctly?
neighborhood, five novice impact fund managers from Zimbabwe,
Guatemala and the Netherlands are rehearsing the sales pitches they‘ll “That’s the businessman_________ daughter suffered an accident this
make the next day to 60 mostly institutional investors, representing morning.”
$10 billion in capital. a) which
The presentations will be a graduation ceremony of sorts. Despite b) who
their impressive resumes, the five men have just completed a four- c) whose
week boot camp covering everything from term sheets, accounting
d) whom
and mezzanine debt structures to dealing with corruption to defining
and marketing their brands. They‘ll head home with golden contacts e) when
(investor cocktail hours were built into the packed schedule) and
a commitment for up to $500,000 in seed capital from Capria

PROMILITARES.COM.BR 335
RELATIVE PRONOUNS AND ADVERBS

06. (ITA) “The man ______ came here and _____ you talked with is
my relative.” GABARITO
a) whom - who EXERCÍCIOS DE FIXAÇÃO
b) which - whom 01. B 04. B 07. A 10. D
c) that - which 02. E 05. C 08. D
d) who - whom 03. E 06. D 09. C
e) none of these EXERCÍCIOS DE TREINAMENTO
01. C 04. A 07. C 10. D
07. (AFA) Choose the option that completes the sentences below
02. E 05. E 08. C
correctly:
03. E 06. D 09. C
I. The ladder ________ I was standing began to slip.
EXERCÍCIOS DE COMBATE
II. My roof leaks ________ it rains.
01. D 04. D 07. D 10. D
III. The wine, ________ was in the cellar, was ruined.
02. B 05. C 08. A
IV. _________ happens don’t forget to call us.
03. A 06. D 09. C
a) on which / whichever / when / wherever
b) If / when / what / whenever ANOTAÇÕES
c) which / when / that / whichever
d) that / whenever / which / whatever

08. (EFOMM) Choose the correct answer.


“I’m not sure _______________.”
a) whom she is staying with.
b) with whom is she staying.
c) with who she is staying.
d) with who is she staying.
e) with which is she staying.

09. (EFOMM) Say if the sentences below are C (correct) or I (incorrect).


( ) This is my sister Jane, whom you met last week.
( ) She remarried six months later, what surprised everyone.
( ) I visited Mr. Rogers, whose son I used to go to school with.
( ) He doesn’t like the people with who he works.
The correct sequence is:
a) (I) (C) (C) (I)
b) (C) (C) (I) (C)
c) (C) (I) (C) (I)
d) (C) (I) (I) (C)
e) (I) (I) (I) (C)

10. (EFOMM 2016) Choose the option that correctly completes the
text below, respectively.
“______ half-past twelve next day Lord Henry Wotton strolled
from Curzon Street over to the Albany to call on his uncle, Lord
Fermor, a genial if somewhat rough-mannered old bachelor, ______
the outside world called selfish, ______ it derived no particular benefit
from him, but ______ was considered generous by Society as he fed
the people who amused him.”
(WILDE, Oscar. The Picture of Dorian Gray. Collins Classics.)

a) On / which / nonetheless / that


b) About / where / notwithstanding / which
c) In / who / instead / whom
d) At / whom / because / who
e) Around / that / consequently / which

336 PROMILITARES.COM.BR
CAUSATIVE FORM, WISH
CLAUSES AND PHRASAL VERBS

CAUSATIVE FORM I wish that we didn’t need to work today.


A causative form é formada pelos verbos have/get + objeto + I wish that you lived close by.
passado particípio do verbo, e sempre será usada quando alguém faz I wish that John wasn’t busy tomorrow.
alguma coisa por nós (seja porque pagamos, pedimos, convencemos, I wish I were rich.
etc), mas esse “alguém” não precisa aparecer na frase.
I wish that I could speak Spanish.
Exemplos:
I wish that I could drive.
I had my car washed.
John will have his house painted.
Wish + (that) + would:
The students get their essays checked.
Utilizado quando queremos expressar alguma coisa que nos
I’ll get my hair cut next week. desagrada sobre algo ou alguém.
He got his washing machine fixed. Exemplos:
I wish that John wouldn’t eat all the chocolate.
Podemos também utilizar a estrutura have + pessoa + infinitivo I wish that the neighbours would be quiet!
sem “to” do verbo, onde será citada a pessoa que foi persuadida a
I wish that you wouldn’t smoke so much!
fazer algo.
I wish that you wouldn’t work late so often.
Exemplos:
I wish that it would stop raining!
I had the electrician look at my broken light.
The doctor will have the nurse call the patients. ProBizu
The teacher had the students write the answers on the whiteboard.
Quando há uma forte possiblidade de algo se realizar, é mais
comum a utilização do termo “hope” do que “wish”.
ProBizu
Exemplos:
Podemos utilizar a estrutura have + objeto + verbo-ing ou infinitivo
I hope that you pass your exam.
sem to para falarmos sobre um evento ou experiência. Utilizamos
verbo-ing para um evento em progresso e infinitivo sem to para I hope that it’s sunny tomorrow.
um evento completado: I hope that Julie has a lovely holiday.
Exemplos:
We had a man singing to us as we sat in the restaurant having Wish + (that) + past perfect:
our meal. Aqui temos uma estrutura que representa arrependimentos do
We had a strange woman come to the door selling pictures. passado. Passa uma ideia muito similar a Condicional 3.
Exemplos:
Mais uma estrutura que temos é get + pessoa + to + infinitivo do I wish that I had studied harder at school.
vebo, passando a ideia de que uma pessoa persuadiu a outra a fazer
I wish that I hadn’t eaten so much yesterday!
algo. Essa estrutura dá uma ideia mais forte de que a pessoa precisou
ser convencida a praticar a ação. I wish that the train had been on time.
Exemplos:
She gets her son to do his homework by promising him ice cream Wish + to + infinitive:
when he’s finished. Neste caso, wish significa o mesmo que want, porém é mais
I got the cleaner to clean under the cupboards. formal. Da mesma forma que o verbo “want”, normalmente não
utilizamos este verbo na forma contínua.
Exemplos:
WISH CLAUSES
I wish to speak to the headmaster.
Neste tópico abordaremos as mais variadas estruturas que
apresentam o termo wish (desejar). Podemos utilizar o wish para I wish to go now.
expressar um arrependimento; quando desejamos que algo fosse
diferente no presente, passado ou futuro. Wish + object + to + infinitive:
Wish + (that) + past simple: Aqui possuímos o mesma ideia do tópico acima, porém colocamos
Podemos utilizar ‘wish’ para falar de algo que gostaríamos que um objeto após “wish”.
fosse diferente. Utilizados em situações impossíveis ou improváveis. Exemplos:
Exemplos: I do not wish you to publish this article.
I wish that I had a big house. I wish these people to leave.

PROMILITARES.COM.BR 57
CAUSATIVE FORM, WISH CLAUSES AND PHRASAL VERBS

PHRASAL VERBS TO GO (IR)


Um verbo frasal ou phrasal verb é um verbo composto, formado
por um verbo comum e mais uma ou duas palavras, geralmente uma Go after ir atrás, perseguir
preposição ou um advérbio. Esse acréscimo de um segundo termo
Go at atacar; fazer algo com muito entusiasmo
ao verbo por vezes modifica por completo seu significado. Na tabela
abaixo temos exemplos de alguns phrasal verbs: Go back retornar

TO CALL (CHAMAR) Go away partir, ir embora

Call for exigir, requerer Go up subir

Call in chamar (alguém para ajudar a resolver algo) Go down descer

Call off cancelar ir atrás de algo, tentar conseguir/conquistar


Go for algo; gostar de determinado tipo de coisa ou
Call out gritar (algo para chamar a atenção de alguém) pessoa; escolher

Call up telefonar Go off explodir; sair (de um local)

Call back retornar uma ligação telefônica Go on continuar; acontecer

entrar (em um local); ficar escondido pelas


Go in
TO GET (OBTER, RECEBER, PEGAR) nuvens (sol, lua)

Get in chegar (em casa; no trabalho) Go out sair (para se divertir)

Get out sair (de um lugar); escapar, fugir Go over verificar (algo) cuidadosamente

Get into entrar (em um local); começar a curtir algo Go with combinar com (roupas, sapatos)

Get off sair, descer (de ônibus, trem); sair do trabalho Go against ser contrário a algo; ir contra algo

entrar (em ônibus, avião, trem); continuar


Get on
(fazendo algo) TO LOOK (OLHAR)
Get up levantar-se Look after cuidar; tomar conta
Get at alcançar; insinuar; pegar no pé Look at olhar para
Get back voltar, retornar Look for procurar
Get away escapar, fugir; sair de férias procurar informação (em livro, revista,
Look up
dicionário)
escapar de algo sem ser punido; virar-se,
Get away with
safar-se Look up to
admirar, respeitar (alguém)
(somebody)
Get around viajar; persuadir
Look into investigar, examinar, analisar
Get on with
continuar, prosseguir com algo
(something) Look over examinar, inspecionar
Get along with dar-se com alguém Look out! Cuidado!
Get over superar, ultrapassar algo (problema; Look forward
(something) experiência desagradável) aguardar ansiosamente por (algo)
to (something)
Get over with terminar, acabar Look down on menosprezar, desfazer-se de (alguém)

TO GIVE (DAR, OFERECER) TO MAKE (FAZER)


Give up desistir Make into transformar
Give in ceder, aceitar Make off fugir, escapar
Give off exalar, liberar (luz, cheiro, calor) entender, captar (com dificuldade); preencher
Make out
(cheque)
Give out distribuir; chegar ao fim; parar de funcionar
inventar, criar (história, explicação); maquiar;
Give away revelar (informação); doar Make up
fazer as pazes
Give back devolver

Give onto dar para (porta, corredor, vistas de uma janela)

58 PROMILITARES.COM.BR
CAUSATIVE FORM, WISH CLAUSES AND PHRASAL VERBS

people. Being bilingual, it turns out, makes you smarter. It can have a
TO PUT (PÔR, COLOCAR) profound effect on your brain, improving cognitive skills not related to
language and even protecting from dementia in old age.
Put aside ignorar (algo); guardar, economizar (dinheiro)
This view of bilingualism is 1remarkably different from 12the
Put away guardar, pôr no lugar understanding of bilingualism through much of the 20th century.
Researchers, educators and policy makers long considered a second
humilhar, colocar para baixo; colocar em uma language to be an interference, cognitively speaking, that delayed a
Put down
superfície (por exemplo, no chão) child’s academic and intellectual development. They were not wrong
about the interference: there is ample evidence that in a bilingual’s
Put on vestir; passar na pele (óleo, creme)
brain both language systems are active even when he is using only
Put off adiar one language, thus creating situations in which one system obstructs
the other. But this interference, researchers are finding out, isn’t
pôr do lado de fora; apagar fogo (cigarro, so much a handicap as a blessing in disguise. It forces the brain to
Put out resolve internal conflict, giving the mind a workout that strengthens
incêndio)
its cognitive muscles.
Put up construir; hospedar (alguém) Bilinguals, 2for instance, seem to be more adept than monolinguals
at solving certain kinds of mental puzzles. In a 2004 study by the
Put
preparar, montar; organizar psychologists Ellen Bialystok and Michelle Martin-Rhee, bilingual and
together
monolingual preschoolers were asked to sort blue circles and red
Put up with tolerar, suportar squares presented on a computer screen into two digital bins — one
marked with a blue square and the other marked with a red circle. In
the first task, the children had to sort the shapes by color, placing blue
TO TAKE (PEGAR) circles in the bin marked with the blue square and red squares in the
bin marked with the red circle. Both groups did this with comparable
Take apart desmontar ease. Next, the children were asked to sort by shape, which was more
challenging because it required placing the images in a bin marked
Take after puxar, assemelhar-se (a parente) with a conflicting color. 13The bilinguals were quicker at performing
this task.
Take away levar embora; tirar 6
The collective evidence from a number of such studies suggests
devolver (item comprado); aceitar alguém de volta that the bilingual experience improves the brain’s 3so-called executive
Take back function — a command system that directs the attention processes
(relacionamento, emprego)
that we use for planning, solving problems and performing various
Take down derrubar; desmontar other mentally demanding tasks. These processes include ignoring
distractions to stay focused, switching attention willfully from one
Take in enganar; incluir thing to another and holding information in mind — like remembering
a sequence of directions while driving.
Take off tirar (roupa, sapato); decolar 14
Why does the fight between two simultaneously active language
Take on contratar systems improve these aspects of cognition? Until recently, researchers
thought 7the bilingual advantage was centered primarily in an ability
Take out tirar (de dentro de algo); levar para sair for inhibition that was improved by the exercise of suppressing one
language system: this suppression, it was thought, would help train
Take over assumir o controle (de algo) the bilingual mind to ignore distractions in other contexts. But that
explanation increasingly appears to be inadequate, since studies have
shown that bilinguals perform better than monolinguals 4even at
tasks that do not require inhibition, like threading a line through an
TO TURN (VIRAR)
ascending series of numbers scattered randomly on a page.
Turn up aumentar; chegar, aparecer (inesperadamente) The bilingual experience appears to influence the brain from
infancy to old age (and 8there is reason to believe that it may also
Turn into tornar-se apply to those who learn a second language later in life).

Turn down rejeitar, recusar; abaixar In a 2009 study led by Agnes Kovacs of the International School
for Advanced Studies in Trieste, Italy, 7-month-old babies exposed to
Turn on ligar, abrir (água, gás); acender (luz) two languages from birth were compared with peers raised with one
language. In an initial set of tests, the infants were presented with an
Turn off desligar; fechar (água, gás); apagar (luz) audio stimulus and then shown a puppet on one side of a screen. Both
infant groups learned to look at that side of the screen in anticipation
Turn over virar of the puppet. But in a later set of tests, when the puppet began
appearing on the opposite side of the screen, the babies exposed to a
https://www.todamateria.com.br/phrasal-verbs/ bilingual environment quickly learned to switch their anticipatory gaze
in the new direction while the other babies did not.
TEXT COMPREHENSION Bilingualism’s effects also extend into the twilight years. In a
TEXT I recent study of 44 elderly Spanish-English bilinguals, scientists led by
the neuropsychologist Tamar Gollan of the University of California,
WHY BILINGUALS ARE SMARTER San Diego, found that individuals with a higher degree of bilingualism
Speaking two languages 5rather than just one has obvious practical — measured through a comparative evaluation of proficiency in each
benefits in an increasingly globalized world. But in recent years, scientists language — were more resistant than others to the beginning of
have begun to show that 10the advantages of bilingualism are even dementia and other symptoms of Alzheimer’s disease: the higher the
more fundamental than being able to converse with 11a wider range of degree of bilingualism, the later the age of occurrence.

PROMILITARES.COM.BR 59
CAUSATIVE FORM, WISH CLAUSES AND PHRASAL VERBS

Nobody ever doubted the power of language. 9But who would TEXT III
have imagined that the words we hear and the sentences we speak HOW TO BECOME A STUNT DOUBLE
might be leaving such a deep imprint?
A stunt double stands in for the actor when the action or fight
Adapted from http://www.nytimes.com/2012/03/18/opinion/sunday/
the-benefitsof-bilingualism.html scene gets dangerous or goes beyond the capabilities of the actor. To
become a stunt double, you must be in excellent physical condition
and have special skills.
01. (AFA) The last two sentences of the second paragraph mean that Instructions
the interference of bilingualism
1. Exercise regularly if you want to become a stunt double. Eat
a) was considered positive in the past, but nowadays this view has nutritiously for optimal health and strength.
changed.
2. Take lots of lessons because the more skills you have, the
b) has always been a problem, since the brain has to solve an internal better. Gymnastics is extremely important in becoming a stunt
conflict. double. Get good at trampoline, skateboarding, swimming and high
c) brings to the brain an internal conflict that improves its cognition. board diving. Take scuba diving lessons.
d) has proved to increase the disabilities of the brain and reduce the Practice rock climbing and horseback riding. Learn to water ski
blessings it can have. and snow ski.
3. Enroll in martial arts classes, especially judo. Judo is excellent for
02. (AFA) Based on the text, it is NOT correct to state that bilingualism learning how to break falls.
a) delays the symptoms of diseases related to old age. 4. Get training in CPR(1) and First Aid. This training looks good on
b) has effect on children’s brains. a résumé, especially for stunt double careers. Injuries happen.
c) is irrelevant for the elderly. 5. Have valid driver’s licenses for both car and motorcycle. Take
advanced driving classes so you’ll be qualified for difficult driving
d) develops the ability of performing difficult tasks.
scenes.
6. Move to Hollywood and plan to work your way up from the
03. (AFA) Mark the INCORRECT option. According to the text, recent
bottom. You must get into the Screen Actors Guild(2) and have a
researches prove that bilingualism
union card(3).
a) causes general cognitive development. Taken from Google
b) enables people to communicate better in both languages only.
(1) Cardiopulmonary resuscitation.
c) prevents people from suffering from problems related to memory
and other mental disorders or delay these problems. (2) Annual prize promoted by the American Syndicate of Actors.
(3) A card certifying membership in an organization.
d) is seen as positive cognitive interference.

06. (AFA) Look at the bold comparative form (item 2). Choose the
04. (AFA) The psychological study done in 2004 (3rd paragraph)
option that contains a similar construction.
showed that
a) The earlier we get there, the more likely we are to get good seats.
a) the children in preschool had the same performances in both tests.
b) More and more people travel to England.
b) bilingual children were more efficient in the most complex test.
c) The smoothest Channel crossing you’ll ever have! Why not fly to
c) monolinguals are better at solving mental puzzles.
France with British Airways? It’ll be the best decision you’ve ever
d) blue and red are confusing colors for both groups. made.
d) Our new jets are now far more luxurious.
TEXT II
07. (AFA) After reading this text, we can deduce that it
a) requests new stunt doubles.
b) briefs about what people should do to start any career.
c) recommends ways to deal with wounds and provides tips to be
healthy to be a double.
d) questions the problems stunt doubles usually have to face.

08. (AFA) One of the instructions below IS NOT stated in the text.
Choose it.
In order to become a stunt double you
a) must have specific qualification for driving.
b) have to consume the necessary substances for your health.
c) need to be a black belt in martial arts.
d) have to show up in some specific surroundings.

05. (AFA) The first boy 09. (AFA) In the article, the author DOES NOT
a) has never intended to be a Rolling Stone. a) advise future doubles to train in First Aid to face possible injuries.
b) has never been a Rolling Stone, but wanted it had happened. b) recommend to eat junky food.
c) has already been a Rolling Stone. c) instruct and give tips for developing a lot of different abilities.
d) hasn’t wished to be a Rolling Stone yet. d) mention that people should be qualified to join the career.

60 PROMILITARES.COM.BR
CAUSATIVE FORM, WISH CLAUSES AND PHRASAL VERBS

10. (AFA) After reading the first item of the instructions, mark the 06. (EEAR) “Put off” is similar in meaning to
option that completes the gap in the converted sentence below. a) forget.
“If you want to become a stunt double you ________ exercise b) cancel.
regularly.”
c) expect.
a) had to
d) postpone.
b) might
c) could 07. (EEAR) “switch off” can be replaced by
d) must a) go off.
b) put on.
EXERCÍCIOS DE c) turn on.

FIXAÇÃO d) turn off.

08. (QUADRO COMPLEMENTAR DA MARINHA) According to the text,


the phrasal verb ruled out is closest in meaning to
01. (EEAR) Read the cartoon and answer question. a) prevented something from happening.
b) took into account.
c) gave order.
d) removed from consideration.
e) made a law.

09. (FUVEST) Investigators trying to find out what happened to a


Malaysia Airlines jet that disappeared en route to Beijing on Saturday
morning were examining the causes of plane crashes: mechanical
“I find the easiest way to expand my vocabulary is to make up words.” failure, pilot error, bad weather. But the discovery that two of the
The phrasal verb “make up” in this cartoon can be replaced by: passengers were carrying stolen passports also raised the possibility
of criminal violence.
a) build up
(Adapted from “Passport Theft adds mystery of missing Malaysia Airlines Jet”)
b) go away
c) look up GLOSSARY: raised – aumentou, ampliou

d) go on Assinale a alternativa que melhor traduz a seguinte frase:


I have run out of vinegar.
02. (PM-MG) Regarding the right use of participle adjectives, choose a) Corri para buscar vinagre.
the best alternative to fill the sentences:
b) Derramei o vinagre.
Why Peter and Chris are so _________? Are they going to a party?
c) Preciso sair para pegar vinagre.
a) Looked up
d) Meu vinagre acabou.
b) Dried up
e) Joguei fora o vinagre.
c) Dressed up
d) Carried out 10. (UDESC) Find the correct alternative:
a) to put on – to remove
03. (PM-MG) Regarding the right use of participle adjectives, choose
b) to put out – to increase
the best alternative to fill the sentences:
c) to put off – to dress
I truly hope it rains in São Paulo soon. Sistema Cantareira is almost
_________. d) to put on weight – to grow fat
a) Set up e) to put together – to keep
b) Dressed up
c) Fixed up EXERCÍCIOS DE
d) Dried up

04. (EEAR) The expression “pick (...) out” has the same meaning as
TREINAMENTO
a) choose
b) invent 01. (VUNESP) In the fragment “Poverty also pushes up fire risk”, the
expression in bold means
c) come
a) causes.
d) enter
b) accumulates.
05. (EEAR) The phrasal verb “tracked down” is closest in meaning to c) raises.
a) arrested. d) blames.
b) attacked. e) mitigates.
c) localized.
d) patrolled.

PROMILITARES.COM.BR 61
CAUSATIVE FORM, WISH CLAUSES AND PHRASAL VERBS

02. (CBM-DF) a) didn’t drink/ wouldn’t finish/ had paid/ weren’t


FIRE IN SOUTH KOREA b) hadn’t drunk/ wouldn’t finish/ had paid/ hadn’t been
Four people died after a fire broke out in a shopping centre in c) hadn’t drunk/ wouldn’t have finished/ have paid/ wasn’t
South Korea. The fire broke out in a children´s play area inside the d) didn’t drink/ wouldn’t have finished/ paid/ weren’t
centre. There were no children inside at the time.
e) hadn’t drunk/ wouldn’t have finished/ paid/ hadn’t been
The centre is next to some flats. More than a hundred people from
the flats immediately evacuated.
07. (UDESC) Choose the CORRECT answer:
(Available: https://www.newsinlevels.com/products/fire-in-south-korea.)
The woman is TAKING CARE of her baby.
“Broke out” (1st par) means. Read the text and choose the correct – She is:
option a) looking back on her baby.
a) hit. b) looking for her baby.
b) started. c) looking after her baby.
c) forbade. d) looking down on her baby.
d) changed. e) looking over her baby.

03. (ETAM) 08. (EN) Which is the correct option to complete the sentence below?
THE FOUR-THOUSAND-YEAR-OLD COMPUTER I don’t mind paying to go to the dentist or_______ my eyes
In 1901, a group of divers excavating an ancient Roman shipwreck ______.
near the island of Antikythera, off the southern coast of Greece, a) to have - checked
found a mysterious object - a lump of calcified stone that contained b) to have - checking
within it several gearwheels welded together after years under the
sea. The 2,000-year-old object, no bigger than a modern laptop, is c) to having - checked
now regarded as the world’s oldest computer, devised to predict solar d) have - checking
eclipses and, according to recent findings, calculate the timing of the e) have - checked
ancient Olympics. Following the efforts of an international team of
scientists, the mysteries of the Antikythera Mechanism are uncovered,
09. (EN) Which sequence best completes the conversation below?
revealing surprising and awe-inspiring details of the object that
continues to mystify. Jane: Did you paint your bedroom when you (1) ______ to your
(From: http://www.bbc.co.uk/programmes/b01hlkcq)
new flat?
Mary: Yes, I did.
The verb “found” can be replaced by: Jane: But did you do it yourself or did you call a painter?
a) came for. Mary: Oh, I don’t think I’d able to do it myself. I (2) _______ my
b) came out. bedroom (3) _______ .
c) came into. a) moved - have - painted
d) came across. b) moved - had - paint
c) move - have - painted
04. (PM-MG) Regarding the right use of phrasal verbs, choose the
d) move - had - painted
best alternative to fill the sentences:
e) moved - had - painted
Sarah __________ her best clothes for the prom.
a) Dressed up 10. (EN) Choose the correct option to complete the dialogue.
b) Took out John: I’ll have to buy a new smartphone. Mine is broken. Paul:
c) Put away Why don’t you have it ______ instead?
d) Put out a) repair. d) to repair.
b) repairing. e) repaired.
05. (QUADRO COMPLEMENTAR DA MARINHA) In: ‘Can you pick up
c) repairs.
survívors?’, the phrasal verb in bold can be replaced by
a) hide.
b) call.
EXERCÍCIOS DE

COMBATE
c) get.
d) buy.
e) find.

06. (EFOMM) Mark the correct answer 01. (EFOMM) Which is the correct option to replace the verb “reach”
I. I wish you _____________ all the water! I’m thirsty. in the paragraph below so that the meaning remains the same?
II. If you hadn’t helped me, I ___________ the task so easily. Nowadays, it is difficult for parents to ______ their image of what
ideal parenting should look like.
III. If you __________ me back, I wouldn’t have to borrow money
from my parents. a) live up to d) get away with
IV. If it ________________ for the goalkeeper, our team would have b) look down on e) look in on
lost. c) run out of

62 PROMILITARES.COM.BR
CAUSATIVE FORM, WISH CLAUSES AND PHRASAL VERBS

02. (EFOMM) Which is the correct option to complete the sentence hearing me complain so many times. If I had known how easy the
below? recovery was, I would have done it a long time ago!
Ruth wanted to be transferred to another department, but her (https://www.realself.com)
application was________ because her own department is understaffed.
a) it had fixed/getting it done
a) turned down b) had fixed it/getting it done
b) turned out c) it had fixed/it getting done
c) turned up d) had fixed it/it getting done
d) turned over e) had it fixed/getting it done
e) turned away
08. Choose the correct option to complete this paragraph.
03. (EN) Which is the correct option to complete the sentence below? WHEN TO HAVE AN EYE EXAM
25 SIMPLE WELL-BEING TRICKS TO HEALTH-PROOF YOUR BODY School-age children and adolescents should have their
Let’s be honest, we could all do with looking _________ ourselves vision_______ before they enter first grade. If your child has no
better. And if you follow these simple well-being tricks to health-proof symptoms of vision problems and no family history of vision problems,
your body, you’ll soon feel the benefits . ______ every one to two years. Otherwise, schedule eye exams based
Here are 25 instant body boosters from top to toe. on the advice of your eye doctor.
(http://www.mirror.co.uk/lifestyle/health/25-simple-health-tips-boost- 2305412) (Adapted from hltps://www.mayoclinic.org)

a) for a) checked/have their vision rechecked


b) up b) checked/have to recheck their vision
c) to c) check/recheck their vision
d) after d) check/have their vision rechecked
e) into e) to check/recheck their vision

04. (EFOMM) Choose the option that correctly completes the 09. (EN) Choose the best reply to this statement.
sentences below, respectively. Michelle: Your flat needs painting.
I. Factories are warning that they may have to ______ workers. You: I know. I __________shortly.
II. He wore a dark grey suit that would not ______ in a workplace. a) have it paint
III. This plan might ______ costing us more money. b) have painted it
c) have been painting it
a) put down / stand up / end up
d) am having it painted
b) count on / rule out / come down
e) am having painted it
c) lay off / stick out / wind up
d) wipe out / set aside / go up 10. (EFOMM) Which option best completes the gap?
e) give away / turn around / finish up “Are you going to the movies alone?”
“Yes, but I wish Susan _____________ with me.”
05. (EFOMM) In, “(...) they have added extra forms to cope with
the needs of their users (...)”, the phrasal verb in bold is closest in a) might come
meaning to: b) had come
a) foresee c) will come
b) support d) would come
c) respect e) can come
d) realize
e) handle
GABARITO
06. (IME) Para a questão, escolha a alternativa que complete a
sentença corretamente: EXERCÍCIOS DE FIXAÇÃO
Using a high-tech kit, the police found a single clue, tracked it 01. A 04. A 07. D 10. D
_________ and saved the girl. 02. C 05. C 08. D
a) down 03. D 06. D 09. D
b) over EXERCÍCIOS DE TREINAMENTO
c) by 01. C 04. D 07. C 10. E
d) on 02. B 05. C 08. A
e) under 03. D 06. E 09. E
EXERCÍCIOS DE COMBATE
07. (EN) Choose the correct option to complete this forum post. 01. A 04. C 07. E 10. D
ALWAYS HATED MY NOSE AND FINALLY ______ 02. A 05. E 08. A
My “beak-like” nose needed to be shaped, but I was afraid the 03. D 06. A 09. D
surgery would hurt too much. A friend talked me into_________ after

PROMILITARES.COM.BR 63
CAUSATIVE FORM, WISH CLAUSES AND PHRASAL VERBS

ANOTAÇÕES

64 PROMILITARES.COM.BR
INGLÊS – EXERCÍCIOS

Leia o texto a seguir e responda às questões 01 do not serve the customer at the bar.
e 02 [C] pubs have a very important role in social life.
[D] pubs are always located in very old
What is a pub? The word pub is short for public buildings.
house. Pubs are popular social meeting places. [E] friends can’t buy ‘rounds’ of drinks in pubs.
They are an important part of British life. People
talk, eat, drink, meet their friends and relax there. Leia o texto a seguir e responda às questões
Many have a garden where people can sit in the 03, 04 e 05.
summer. One of the oldest pubs, Fighting Cocks “Hello”, said a quiet, musical voice.
in St. Albans, Herts, is located in a building that I looked up, stunned that he was speaking to me.
dates back to the eleventh century. Groups of He was sitting as far away from me as the desk
friends normally buy ‘rounds’ of drinks, where allowed, but his chair was angled toward me. His
the person whose turn it is will buy drinks for all hair was dripping wet – he looked like someone
the members of the group. In most pubs in in a commercial for hair gel. His dazzling face
Britain, you must go to the bar to order drinks was friendly, open, a slight smile on his flawless
and food and pay for your purchase immediately, lips. But his eyes were careful.
there is no table service. Bartenders do not expect “My name is Edward Cullen,” he continued. “I
frequent tipping. To tip them, it is customary to didn’t have a chance to introduce myself last
say: “Would you like a drink yourself?” week. You must be Bella Swan.”
Adaptado de http://projectbritain.com/pubs.htm, consulta em 22/06/2012
My mind was spinning with confusion. He was
perfectly polite now. I had to speak; he was
01. (ESPCEX) According to the text, pubs are
waiting. But I couldn’t think of anything
popular places where people
conventional to say.
[A] get together.
“H-how do you know my name?” I stammered.
[B] just drink.
He laughed a soft laugh.
[C] have to tip the bartender.
“Oh, I think everyone knows your name. The
[D] meet only in the winter.
whole town was waiting for you to arrive.”
[E] pay the bartender a drink.
MEYER, S. Twilight. New York: Megan Tingley
Books, 2006. Page 43.
02. (ESPCEX) According to the text, it is correct
to say that in Britain
03. (ESPCEX) According to the sentence “I
[A] there is table service in most pubs. [B] pubs
looked up, stunned that he was speaking to me.”,

www.projeto-militar.blogspot.com.br
INGLÊS – EXERCÍCIOS

Bella empirical literature on leadership is emotional


[A] didn’t expect Edward to approach her. intelligence (EI), which in recent years has been
[B] was feeling comfortable. the focus of considerable attention in relationship
[C] didn’t want to be friends with Edward. to leadership efficacy. Emotional intelligence
[D] didn’t want to talk to Edward. involves an awareness of one’s own emotions as
[E] was offended by Edward’s attitude. well as the ability to control them, social
awareness of others and their emotions, and the
04. (ESPCEX) “My mind was spinning with capacity to understand and manage relationship
confusion. He was perfectly polite now. I had to and social networks.
speak; he was waiting. But I couldn’t think of In understanding others’ emotions, an important
anything conventional to say. contributing factor to the success of the more
“H-how do you know my name?” I stammered.” effective military officers is their ability to
The excerpt above describes a feeling of empathize with their subordinates. In discussing
[A] sadness. empathy, FM (Field Manual) 6-22 defines it as
[B] deception. “the ability to see something from another
[C] pain. person’s point of view, to identify with and enter
[D] nervousness. into another person’s feelings and emotions”.
[E] anger. Empathy is not typically a quality that most
soldiers would readily identify as an essential
05. (ESPCEX) According to Bella’s descriptions characteristic to effective leadership or necessary
in the text, Edward was to producing positive organizational outcomes,
[A] intelligent. but it is an important quality for competent
[B] boring. leadership, especially as it relates to EI.
Adaptado de McDONALD, Sean P. Military Review, Jan-Feb, 2013.
[C] charming.
[D] inconvenient.
06. (ESPCEX) According to the text, we can
[E] dangerous.
state that
[A] empathy is part of emotional intelligence.
Leia o texto a seguir e responda às questões
[B] emotional intelligence does not include
06, 07.
empathy.
[C] emotional intelligence is the ability to avoid
Empirically Based Leadership
empathy.
A significant area of interest within the US Army

www.projeto-militar.blogspot.com.br
INGLÊS – EXERCÍCIOS

[D] the US Army wants soldiers to hide feelings. bringing Spain’s economy down, Rocio lost her
[E] the US Army wants leaders to control two jobs - in a shop, and as a cleaner. For a
subordinates’ feelings. while, Rocio got by on benefits, but then those
stopped too. She is an example of the crisis many
07. (ESPCEX) In the sentence “...an awareness Spaniards face as the country deals with the
of one’s own emotions as well as the ability to highest unemployment rate since the Civil War in
control them...”, the expression as well as has the the 1930s, and a recession entering its second
same meaning as year. “I can’t stand the thought of living on the
[A] but. streets with my son, but I have no idea where else
[B] thus. to go”, she says.
[C] also. Rocio’s story is echoed by others all over Spain.
[D] unless. It is this fear that took many Spanish citizens to
[E] then. action. Many of those people who are outside the
door of Rocio’s apartment block are supporters of
Leia o texto a seguir e responda às questões “Stop Desahucios” (Stop Evictions), part of the
08, 09 e 10. Platform of People Affected by Mortgages (PAH
– Plataforma de Afectados por la Hipoteca), a
Welcome to Madrid: City of Protests group that campaigns to prevent banks and
Madrid (CNN) – “The people, united, will never authorities from eviction because of the country’s
be divided!” yells the crowd, angrily waving economic crisis. They accuse the banks and
banners and placards. “To fight is the only way!” authorities of “real estate terrorism”.
Dog-walkers, mothers with strollers, and There are also the mass marches of the 15-M
pensioners carrying shopping bags join the movement - also known as the “Indignados”.
crowd. These people on the sidewalk are no Activist Dante Scherma, 24, says citizens were
curious neighbors. Indeed, many of them are not used to speaking out on political issues. “The
complete strangers to the family living on the 15-M movement made people talk about social
fifth floor, but they are all here to protect Rocio issues, and about politics in normal conversations
from eviction - being forced to leave her property - in cafés, restaurants, bars - where before they
by legal process. Rocio and her son, now 17 and only talked about football or fashion.”
in high school, moved from Ecuador in 2003, Back in Vicalvaro, the moment of truth has
when times were good and jobs plentiful in arrived, but the crowd - now shouting at the
Spain. But then the global financial crisis hit, police, insisting they have to stop forcing

www.projeto-militar.blogspot.com.br
INGLÊS – EXERCÍCIOS

families to leave their properties - appears to have to stop forcing families to leave their
have had an impact. Lawyers from the PAH properties...”, the words they and their
explain that Rocio will be able to stay - for a respectively refer to
while, at least. For those working to stop Spain’s [A] the crowd and families.
eviction epidemic, today has seen a small and [B] the crowd and the police.
temporary victory. For those demonstrating about [C] the police and families.
cuts, corruption and lack of cash, the protests will [D] the families and the properties.
go on. [E] the police and the properties.
Adaptado de http://edition.cnn.com/2013/02/20/world/europe/madrid-city-of-protests/index.html

Leia o texto a seguir e responda às questões 11


08. (ESPCEX) According to the text, Rocio
e 12.
[A] moved to Spain when there were many jobs
there.
Fire at Antarctica station kills 2 Brazilian sailors
[B] was the only person affected by the crisis in
Two Brazilian sailors died and one was injured
Spain.
Saturday after a fire broke out at a naval research
[C] is getting benefits from Spain’s government
station in Antarctica, authorities reported. The
nowadays.
fire occurred at the Comandante Ferraz Station
[D] had two jobs in Ecuador before moving to
on King George Island, said Adm. Julio Soares
Spain.
de Moura Neto, commander of the Brazilian
[E] has the best job in Spain nowadays.
Navy. The three sailors were trying to extinguish
a fire that broke out in the engine room of the
09. (ESPCEX) According to the text, the 15-M
facility.
movement made
Brazilian military police are investigating the
[A] Spaniards’ lives lose their importance.
cause. The station is home to researchers who
[B] citizens in Spain more interested in social
conduct
issues.
studies on the effects of climate change in
[C] Spaniards talk more about football and
Antarctica and its implications on the planet,
fashion.
according to the Ministry of Science and
[D] people go to cafés, restaurants and bars.
Technology and Innovation. Researchers at the
[E] Spaniards accept evictions.
base also study marine life and the atmosphere.
Adaptado de http://articles.cnn.com, consulta em 26/02/2012

10. (ESPCEX) In the sentence “...insisting they

www.projeto-militar.blogspot.com.br
INGLÊS – EXERCÍCIOS

11. (ESPCEX) According to the text, it is correct $46.3 billion in sales last quarter. They work day
to state that or night shifts, eating and sleeping at company
[A] the Brazilian sailors were responsible for the facilities, as they help build electronics products
fire incident. for Apple and many other global brand names,
[B] the fire started outside the engine room. such as Amazon’s Kindle and Microsoft’s Xbox.
[C] Brazilian military police still don’t know the As a poor college student with no work
cause. experience, looking for a job in China’s
[D] researchers are studying the cause. competitive market is an uphill battle. So when
[E] climate change caused the fire. Chen was offered a one-month position at
Foxconn with promises of great benefits and little
12. (ESPCEX) In the sentence “The station is overtime, she jumped at the chance. But when
home to researchers who conduct studies...”, the she started working, she found out that only
word who refers to senior employees got such benefits.
[A] station. “During my first day of work, an older worker
[B] researchers. said to me, ‘Why did you come to Foxconn?
[C] home. Think about it again and leave right now’,” said
[D] studies. Chen, who plans to return to her studies at a
[E] Ministry of Science and Technology and Chongqing university soon.
Innovation. Foxconn recently released a statement defending
its corporate practices, stating its employees are
Leia o texto a seguir e responda às questões entitled to numerous benefits including access to
13, 14, 15, 16 e 17 health care and opportunities for promotions and
training. In response to questions from CNN,
Apple manufacturing plant workers complain of Apple also released a statement: “We care about
long hours and militant culture every worker in our worldwide supply chain. We
Chengdu, China (CNN) — Miss Chen (we insist that our suppliers provide safe working
changed her name for this story), an 18-year-old conditions, treat workers with dignity and
student from a village outside of the southern respect, and use environmentally responsible
megacity of Chongqing, is one of more than one manufacturing processes wherever Apple
million factory workers at a Chinese company products are made. Our suppliers must live up to
that helps manufacture products for Apple Inc.’s these requirements if they want to keep doing
lucrative global empire, which ranked in a record business with Apple.”

www.projeto-militar.blogspot.com.br
INGLÊS – EXERCÍCIOS

After three weeks of applying more than 4,000 [D] it’s difficult to get a job in China.
stickers a day onto iPad screens by hand and [E] you have to be an excellent athlete.
working 60 hours a week in an assembly line,
Chen says she’s ready to go back to school and 15. (ESPCEX) In the sentence “But when she
study hard so she’ll never have to return to started working, she found out that...”, the word
Foxconn. “It’s so boring, I can’t bear it anymore. but indicates
Everyday is like: I get off from work and I go to [A] addition.
bed. I get up in the morning, and I go to work. It [B] consequence.
is my daily routine and I almost feel like an [C] result.
animal,” said Miss Chen. When asked why [D] reason.
humans do machine-ike [E] contrast.
work at Foxconn, she responds, “Well, humans
are cheaper.” 16. (ESPCEX) In the sentence “Foxconn
Adaptado de http://edition.cnn.com, consulta em 06/02/2012
recently released a statement defending its
corporate practices...”, the word its refers to
13. (ESPCEX) It’s correct to say that Miss Chen
[A] statement.
[A] is very satisfied with her job at Foxconn.
[B] Foxconn.
[B] is a special factory worker at Foxconn.
[C] health care.
[C] has lots of benefits and little overtime at
[D] practices.
Foxconn.
[E] employees.
[D] works day or night shifts, eating and sleeping
at Foxconn.
17. (ESPCEX) According to the text, workers at
[E] worked in another company before working
Foxconn company are compared to
at Foxconn.
[A] machines and animals.
[B] machines and humans.
14. (ESPCEX) In the sentence “As a poor
[C] animals and men.
college student with no work experience, looking
[D] suppliers and machines.
for a job in China’s competitive market is an
[E] animals and suppliers.
uphill battle.”, the author means that
[A] you cannot find a job in China.
Leia o texto a seguir e responda às questões
[B] you have to go up a hill.
19, 19 e 20.
[C] it’s exciting to get a job in China.

www.projeto-militar.blogspot.com.br
INGLÊS – EXERCÍCIOS

Facebook Song lyrics 18. (ESPCEX) The text above can be considered
I wouldn’t call myself a social butterfly [A] obscene.
And there’s not much that separates me from the [B] obsolete.
other guy [C] religious.
But when I log in I begin to live [D] ironic.
There’s an online world where I am king [E] thrilling.
Of a little website dedicated to me
With pictures of me and a list of my friends 19. (ESPCEX) The sentence “But when I login I
And an unofficial record of the groups that I’m in begin to live” brings the readers a reflection
Before the internet, friendship was so tough about the extreme importance that people, in our
You actually had to be in people’s presence and society, give to
stuff [A] school and internet.
Who would have thought that with a point and a [B] love and life.
click [C] life and death.
I could know that Hope Floats is your favorite [D] girls and computers.
flick [E] the online world.
Facebook
I’m hooked on Facebook 20. (ESPCEX) In the sentence “Before the
I used to meet girls hanging out at the mall internet, friendship was so tough”, the word
Now I just wait for them to write on my wall tough can be replaced by
Oh! Link’s status changed, it says he’s playing [A] easy.
the recorder... [B] difficult.
How do you know this person? [C] respectable.
Did you hook up with this person? [D] nice.
Do you need to request confirmation? [E] flexible.
Or did you just think they looked cute...
From their picture on Facebook? Leia o trecho abaixo e responda às questões de
If the internet crashed all across the land 21 a 25.
Or my Facebook account was deleted by the man Mark Zuckerberg’s 650 Million
I’d carry around a picture of my face Friends (and counting)
And a summary of me typed out on a page Back in June 2009, the globe’s potpourri of
Adaptado de http://www.lyricsmode.com/lyrics/r/rhett_and_link/facebook_song.html
social-networking sites was extremely diverse:

www.projeto-militar.blogspot.com.br
INGLÊS – EXERCÍCIOS

Google’s Orkut dominated India and Brazil; [A] Five.


Central and South America preferred Hi5; [B] Seven.
Maktoob was king in the Arab world. The [C] Nine.
Vietnamese liked Zing, the Czechs loved Lidé, [D] Eleven.
South Koreans surfed Cyworld. Two years after [E] Thirteen.
that, and Facebook has stolen users away from its
rivals very fast. It’s completely knocked Hi5 off 23. (ESPCEX) According to the text, Facebook
the map in former strongholds such as Peru, is not number one in
Mexico, and Thailand. After a tense back-and- [A] South America.
forth with Orkut in India, Facebook has emerged [B] Peru, Mexico and Thailand.
victorious. And it’s becoming more popular in [C] Russia and China.
Armenia, Georgia, and the Netherlands, where [D] India and Brazil.
local providers are making a [E] South Korea.
desperate last stand.
There are some glaring exceptions to 24. (ESPCEX) In the sentence “And it’s
Facebook’s colonization kick. Russians continue becoming more popular in Armenia, Georgia, and
to use Vkontakte and Odnoklassniki, with the Netherlands...”, the pronoun it refers to
Facebook a distant fourth in the rankings. China [A] Orkut.
remains highly committed to domestic sites such [B] India.
as Qzone and Renren. But for the rest of us, [C] Armenia.
we’re living in Zuckerberg’s world. [D] Hi5.
(endereço eletrônico omitido propositadamente)
[E] Facebook.
21. (ESPCEX) According to the text, Facebook’s
boom on the Internet was in
25. (ESPCEX) Which alternative represents the
[A] 2007.
main idea of the text ?
[B] 2008.
[A] Facebook dominates the world of social
[C] 2009.
networks.
[D] 2010.
[B] Orkut is the most popular site in Latin
[E] 2011.
America.
[C] Facebook is used by people all over the
22. (ESPCEX) How many different social-
world except in China.
networking sites are mentioned in the text ?
[D] Orkut won the battle in India.

www.projeto-militar.blogspot.com.br
INGLÊS – EXERCÍCIOS

[E] Facebook is one of the top three in popularity Shifting into overdrive. Oxford, Macmillan, 2005.

26. (ESPCEX) In the sentence “All teenagers


in Russia.
have the same problems - life, love, school work,
Leia o trecho abaixo e responda às questões de
and parents”,
26 a 30.
the conjunction and indicates
Life and the Movies
[A] contrast.
Joey Potter looked at her friend Dawson Leery
[B] result.
and she smiled sadly.
[C] reason.
“Life isn’t like a movie, Dawson,” she said. “We
[D] consequence.
can’t write happy endings to all our
[E] addition.
relationships.”
Joey was a pretty girl with long brown hair. Both
27. (ESPCEX) Which alternative best explains
Joey and Dawson were nearly sixteen years old.
the sentence “We can’t write happy endings to all
The two teenagers had problems. All teenagers
our relationships.” ?
have the same problems - life, love, school work,
[A] It’s not possible to decide what happens in
and parents. It isn’t easy to become an adult.
our future.
Dawson loved movies. He had always loved
[B] We need permission to write books about
movies. He took film classes in school. He made
happy endings in relationships.
short movies himself. Dawson wanted to be a
[C] Our relationships are the happiest things in
film director. His favorite director was Steven
our lives.
Spielberg.
[D] Happy endings are always possible in real
Dawson spent a lot of his free time filming with
life.
his video camera. He loved watching videos of
[E] People expect unhappy endings when they
great movies from the past. Most evenings, he
watch a movie.
watched movies with Joey.
“These days, Dawson always wants us to behave
28. (ESPCEX) According to the sentence “Joey
like people in movies,” Joey thought. And life in
looked at the handsome, blond boy who was
the little seaside town of Capeside wasn’t like the
sitting next to her”, it is correct to say that
movies.
Dawson was
Joey looked at the handsome, blond boy who was
[A] strange.
sitting next to her. She thought about the years of
[B] calm.
their long friendship. They were best friends...
ANDERS, C. J. Retold by CORNISH, F. H. Dawson’s Creek. [C] ugly.

www.projeto-militar.blogspot.com.br
INGLÊS – EXERCÍCIOS

[D] good-looking. shantytown complex, providing a rare moment of


[E] intelligent. happiness and celebration in a decades-long
battle to rid this city’s violent slums of drug
29. (ESPCEX) According to the text, which gangs.
alternative is correct about Dawson? An air of calm and relief swept through the
[A] He first met Joey a couple of weeks ago. neighborhood, as residents opened their windows
[B] He spent most of his evenings with Joey. and began walking the streets. Dozens of children
[C] He had dark hair. ran from their houses in shorts and bikinis to
[D] He didn’t have any problems. jump into a swimming pool that used to belong to
[E] He was older than Joey. a gang leader. Residents congregated around
televisions in bars and restaurants, cheering for
30. (ESPCEX) According to the text, “The two the police as if they were cheering for their
teenagers had problems” because favorite soccer teams. “Now the community is
[A] they wanted to be film directors. ours,” Jovelino Ferreira, a 60-year-old pastor,
[B] it is difficult to become an adult. said, his eyes filling with tears. “This time it will
[C] they wanted to behave like people in movies. be different. We have to have faith. Many people
[D] they were best friends. who didn’t deserve have suffered here.”
[E] they watched movies until late at night. http://www.nytimes.com, consulta em 28/11/2010

31. (ESPCEX) In the sentence “An air of calm


Leia o trecho abaixo e responda às questões de
and relief swept through the neighborhood, as
31 a 35.
residents opened their windows and began
walking the streets”, the possessive adjective
Brazilian Forces Claim Victory in Gang
their refers to
Haven
[A] an air of calm and relief.
RIO DE JANEIRO - In a quick and decisive
[B] neighborhood.
military operation, Brazilian security forces took
[C] residents.
control of this city’s most notorious slum on
[D] streets.
Sunday, celebrating victory over drug gangs after
[E] calm and relief.
a weeklong battle.
In the early afternoon, the military police raised
32. (ESPCEX) According to the text, Brazilian
the flags of Brazil and Rio de Janeiro atop a
security forces fought against
building on the highest hill in the Alemão

www.projeto-militar.blogspot.com.br
INGLÊS – EXERCÍCIOS

[A] Alemão shanty town complex. [E] tragedy.


[B] drug gangs.
[C] Jovelino Ferreira. As questões de 36 a 39 referem-se ao texto a
[D] the military police. seguir:
[E] Jovelino Ferreira’s community.
THE MAN IN THE CONVERTIBLE
33. (ESPCEX) After Brazilian security forces’ One morning, well after I was diagnosed
victory, with cancer, I got an email from Robbee Kosak,
[A] residents went to streets. Carnegie Mellon’s vice president for
[B] children were afraid. advancement. She told me a story.
[C] the gang leader used the swimming pool. She said she had been driving home from
[D] residents cheered for their favorite soccer work the night before, and she found herself
team. behind a man in a convertible. It was a warm,
[E] residents prayed with pastor Jovelino gorgeous, early-spring evening, and the man had
Ferreira. his top down and all his windows lowered. His
arm was hanging over the driver’s side door, and
34. (ESPCEX) It is correct to say that, after the his fingers were tapping along to the music on his
decisive military operation in the Alemão radio. His head was bobbing along, too, as the
shantytown wind blew through his hair.
complex, Jovelino Ferreira was Robbee changed lanes and pulled a little
[A] angry. closer. From the side, she could see that the man
[B] sick. had a slight smile on his face, the kind of
[C] sad absentminded smile a person might have when
[D] hungry he’s all alone, happy in his own thoughts. Robbee
[E] hopeful. found herself thinking: “Wow, this is the epitome
of a person appreciating this day and this
35. (ESPCEX) In the last paragraph, the author moment.”
is describing a scene of The convertible eventually turned the
[A] sadness. corner, and that’s when Robbee got a look at the
[B] conflict. man’s full face. “Oh my God,” she said to
[C] happiness. herself. “It’s Randy Pausch!”
[D] fear. She was so struck by the sight of me. She

www.projeto-militar.blogspot.com.br
INGLÊS – EXERCÍCIOS

knew that my cancer diagnosis was grim. And direto, indireto e indireto livre.
yet, as she wrote in her email, she was moved by C) usa foco narrativo em primeira pessoa
how contented I seemed. In this private moment, configurando o texto autobiográfico.
I was obviously in high spirits. Robbee wrote in D) utiliza linguagem coloquial nos diálogos para
her email: “You can never know how much that externar seus sentimentos.
glimpse of you made my day, reminding me of E) recorre a figuras de pensamento para compor
what life is all about.” o gênero dramático.
I read Robbee’s email several times. I
came to look at it as a feedback loop of sorts. 37. (ITA) De acordo com as informações no
It has not always been easy to stay texto, Robbee Kosak
positive through my cancer treatment. When you A) descreveu detalhadamente o cenário do seu
have a dire medical issue, it’s tough to know how primeiro encontro com Randy Pausch.
you’re really faring emotionally. I had wondered B) sentiu-se atraída pelo veículo de Randy
whether a part of me was acting when I was with Pausch devido à alta velocidade dele.
other people. Maybe at times I forced myself to C) escreveu palavras motivadoras a Randy
appear strong and upbeat. Many cancer patients Pausch porque desejava reanimá-lo.
feel obliged to put up a brave front. Was I doing D) caracterizou o motorista do veículo como uma
that, too? pessoa satisfeita e de bem com a vida.
But Robbee had come upon me in an E) ocupava o cargo de Vice-Presidente na
unguarded moment. I’d like to think she saw me empresa presidida por Randy Pausch.
as I am. She certainly saw me as I was that
evening. 38. (ITA) A frase “She had given me a window
Her mail was just a paragraph, but it into myself” expressa:
meant a great deal to me. She had given me a A) percepção que Robbee Kosak transmitiu de si
window into myself. I was still fully engaged. I própria para Pausch.
still knew life was good. I was doing OK. B) visão reduzida que o autor transmitiu sobre
Fonte: PAUSCH, R. The last lecture. New York, Hyperion, 2008. p.64-65. seu lado otimista.
C) aparência distorcida de uma personalidade
36. (ITA) O autor do texto
extrovertida.
A) utiliza a dissertação e a descrição como
D) constatação de que Randy Pausch não
tipologia textual predominante.
transmitia vontade de viver.
B) é narrador observador e mescla discursos
E) percepção do narrador sobre algo de que ele

www.projeto-militar.blogspot.com.br
INGLÊS – EXERCÍCIOS

não se dava conta. design body armour for the US military.


39. (ITA) Na frase “She said she had been Legacy Effects, a Hollywood design
driving home from work the night before, and studio based in California, has previously worked
she found herself behind a man in a on power suits for films such as RoboCop,
convertible”, a formação correta quanto ao uso Captain America, The Terminator and Iron Man.
do discurso direto é: Now, the company is building body armour
A) She said: “I was driving home from work last equipped with an "agile exoskeleton" that will
night, and I found myself behind a man in a allow soldiers to carry hundreds of pounds of
convertible”. equipment, the Wall Street Journal reports.
B) She said: “I had been driving home from work "We are trying to be revolutionary," said
last night, and I found me behind a man in a Mike Fieldson, who manages the US military
convertible”. project known as the Tactical Assault Light
C) She said: “I drove home yesterday night from Operator Suit (Talos).
work, and I had found myself behind a man in a Three prototypes have been presented to
convertible”. the Pentagon by teams of bioengineers,
D) She said: “I had driven home the night before, technologists and a Canadian company that
and I found myself behind a man in a studies insect and animal exoskeletons. The
convertible”. prototypes will contribute to the creation of a
E) She said: “I was driving home from work new generation of body armour which the US
yesterday, and I was finding myself behind a man Special Operations Command aims to complete
in a convertible”. within four years.
The suits are designed to protect soldiers
As questões de 40 a 42 referem-se ao texto a from bullets, explosions and bayonet attacks.
seguir: Legacy Effects admits that bringing an
Iron Man to life presents significant challenges.
IRON MAN DESIGNERS TO BUILD BODY For one thing, a real-life version of the suit would
ARMOUR FOR US ARMY add extra bulk to a soldier limiting his or her
Hollywood special effect team is working on a agility. Also, the company estimates that the Iron
new Iron Man 'agile exoskeleton' for US Man suit would probably weigh about 180kg, and
soldiers would need to be supported by a mobile
The Oscar-nominated special effects team exoskeleton, but "none of the exoskeletons in the
behind the Iron Man suit has been contracted to industry are capable of moving that much

www.projeto-militar.blogspot.com.br
INGLÊS – EXERCÍCIOS

weight", SlashGear reports. desenvolveu a armadura do Iron Man.


Russ Angold of Ekso Bionics, a company E) faz parte de um projeto mais amplo
that designs exoskeletons for medical use, says desenvolvido pela empresa americana Legacy
that power armour in films offer an unrealistic Effects.
model, so engineers are presently trying to make
the suits more practical. "Hollywood has 41. (ITA) A empresa Legacy Effects
definitely made the Iron Man suit impossibly A) tem experiência em criar roupas especiais
thin, impossibly light, impossibly agile and para filmes americanos famosos.
impossibly energy efficient. So we're really B) tem como sede o estado da Califórnia e prevê
trying to solve the problem and ask the question: um gasto de 10 milhões no projeto Talos.
What would Iron Man look like if it was real?" C) é uma das parceiras do exército americano na
The US military has so far spent about idealização de exoesqueleto para uso médico.
$10 million on Talos, prompting the armed D) aceitou o desafio do projeto Talos e garante
services committee to request a briefing on the cumprir todos os objetivos que o projeto impõe.
project to ensure taxpayer money is not being E) baseou-se nos estudos de exoesqueletos de
wasted. animais e de insetos para criar o protótipo
"Will you ever have an Iron Man? I don't americano.
know," said Brian Dowling, a former soldier
involved in the project. "But you'll have some 42. (ITA) A vestimenta idealizada no projeto
greatly improved technology along the way". Talos deverá satisfazer apenas uma das condições
Fonte: http://www.theweek.co.uk/world-news/59323/iron-man-designers-to-build-
abaixo:
body-armour-for-us-army
A) não ultrapassar o orçamento de 10 milhões de

40. (ITA) O projeto Talos dólares previsto pelo governo americano.

A) tem por objetivo construir uma prótese a ser B) ajustar-se ao corpo humano

usada por soldados americanos. independentemente do peso e do tamanho do

B) foi idealizado há quatro anos e três protótipos usuário.

foram apresentados. C) oferecer condições de realizar operações

C) é constituído por uma equipe formada por militares carregando muito peso.

militares americanos e pesquisadores D) auxiliar o soldado em combate, aumentando o

aposentados. tempo em incursões militares não motorizadas.

D) conta com a participação do studio que E) ser funcional e conter bateria duradoura e
recarregável por energia solar.

www.projeto-militar.blogspot.com.br
INGLÊS – EXERCÍCIOS

It also assumes that the market is not


As questões de 43 a 46 referem-se ao texto a being rigged. Panini says that each sticker is
seguir: printed in the same volumes and randomly
distributed. In a 2010 paper Messrs Sardy and
STICKERNOMICS Velenik gamely played the role of “regulator” by
Football albums checking the distribution of stickers for a 660-
Got, got, got, got, got, need sticker album sold in Switzerland for that year’s
World Cup. Out of their sample of 6,000 stickers,
THE World Cup is still two weeks away, but for they expected to see each sticker 9.09 times on
children worldwide (plus disturbing numbers of average (6,000/660), which was broadly borne
adults) the race to complete the Brazil 2014 out in practice.
sticker book started long ago. Panini, an Italian Even in a fair market, it is inefficient to
firm, has produced sticker albums for World buy endless packs as an individual (not to
Cups since Mexico 1970; this year’s version has mention bloody expensive for the parents). The
640 stickers to collect. Collecting them is no idle answer is to create a market for collectors to
pursuit, however. Getting every slot filled swap their unwanted stickers.
delivers an early lesson in probability, the value The playground is one version of this
of statistical tests and the importance of liquidity. market, where a child who has a card prized by
When you start an album, your first many suddenly understands the power of limited
sticker (in Britain, they come in packs of five) supply. Sticker fairs are another. As with any
has a 640/640 probability of being needed. As the market, liquidity counts. The more people who
spaces get filled, the odds of opening a pack and can be attracted into the market with their
finding a sticker you want fall. According to duplicate cards, the better the chances of finding
Sylvain Sardy and Yvan Velenik, two the sticker you want.
mathematicians at the University of Geneva, the Messrs Sardy and Velenik reckon that a
number of sticker packs that you would have to group of ten astute sticker-swappers would need
buy on average to fill the album by mechanically a mere 1,435 packs between them to complete all
buying pack after pack would be 899. That ten albums, if they take advantage of Panini’s
assumes there is no supply shock to the market practice of selling the final 50 missing stickers to
(the theft of hundreds of thousands of stickers in order. Internet forums, where potentially
Brazil in April left many fearful that Panini unlimited numbers of people can swap stickers,
would run short of cards). make this number fall even further. The idea of a

www.projeto-militar.blogspot.com.br
INGLÊS – EXERCÍCIOS

totally efficient market should dismay Panini, completar um álbum.


which will sell fewer packs as a result. But as in D) a empresa Panini disponibiliza a venda das 50
all markets, behaviour is not strictly rational. figurinhas faltantes aos colecionadores.
Despite entreaties, your correspondent’s son is E) o processo de confecção e distribuição das
prepared to tear out most of his stickers to get figurinhas é feito aleatoriamente pela Panini.
hold of Lionel Messi.
Fonte: http://www.economist.com/news/finance-and- 45. (ITA) De acordo com o texto, Sardy e
economics/21603019-got-got-got-got-got-need-stickernomics
Velenik
A) insistem a formação de grupos de 10
43. (ITA) O autor do texto
colecionadores para facilitar o preenchimento
I. atribui ao roubo de milhares de figurinhas no
total de álbuns de figurinhas.
Brasil a dificuldade para compra e troca entre
B) fiscalizam a compatibilidade entre a produção
colecionadores.
de figurinhas e sua comercialização desde 2010.
II. deprecia as estratégias do Grupo Panini para
C) verificaram na Suíça a repetição de
comercializar álbuns de figurinhas da Copa do
aproximadamente 9 vezes cada figurinha em um
Mundo.
lote de 6.000 figurinhas.
III. descreve o mercado de figurinhas da Copa do
D) são matemáticos pesquisadores da empresa
Mundo e apresenta aos colecionadores
Panini, responsáveis pela distribuição das
possibilidades de obtenção de figurinhas.
figurinhas.
Está(ão) correta(s)
E) consideram que as práticas de obtenção de
A) apenas a I.
figurinhas da Copa do Mundo são injustas e
B) apenas a II.
manipuláveis.
C) apenas a III.
D) apenas I e II.
46. (ITA) Em “Despite entreaties, your
E) apenas I e III.
correspondent’s son is prepared to tear out
most of his stickers to get hold of Lionel
44. (ITA) De acordo com o texto,
Messi”, depreende-se que o autor
A) a empresa Panini comercializa álbuns de
A) reconhece que também faz parte do grupo de
figurinhas da Copa do Mundo há 30 anos.
colecionadores fanáticos por álbuns de
B) é impossível completar o álbum sem que os
figurinhas.
colecionadores recorram a feiras e redes sociais.
B) se dispõe a tudo para conseguir a figurinha de
C) são necessárias 1.500 figurinhas para
Lionel Messi para o filho.

www.projeto-militar.blogspot.com.br
INGLÊS – EXERCÍCIOS

C) busca adquirir as figurinhas mais disputadas E) reinforcement.


para seu filho por meio das redes sociais.
D) inclui-se no grupo de colecionadores 49. (ITA) A reportagem anunciada na tirinha
insensatos de figurinhas de Copa de Mundo A) mostra a relação direta entre obesidade e
2014. consumo de produtos calóricos.
E) vivencia em casa o esforço de um B) divulga um estudo científico com o objetivo
colecionador para obter uma única figurinha. de mudar o comportamento da audiência.
C) demonstra indiferença com relação ao tema.
As questões de 47 a 49 referem-se à tirinha a D) estimula o uso de redes sociais para divulgar
seguir: produtos dos patrocinadores.
E) revela que 70% das pessoas com sobrepeso
são sedentárias.

As questões de 50 a 55 referem-se ao texto a


seguir:

A HISTORY OF PI
The history of Pi, says the author, though
47. (ITA) Marque a opção que pode substituir
a small part of the history of mathematics, is
“due to” sem alterar o sentido do período.
nevertheless a mirror of the history of man. Petr
A) by means of
Beckmann holds up this mirror, giving the
B) in case of
background of the times when Pi made progress
C) in spite of

D) instead of
and also when it did not, because science was
E) because of
being stifled by militarism or religious
fanaticism. The mathematical level of this book
48. (ITA) A relação semântica entre os dois
is flexible, and there is plenty for readers of all
quadros é de
ages and interests.
A) corroboration.
ABOUT THE AUTHOR
B) contradiction.
Petr Beckmann was born in Prague,
C) substantiation.
Czechoslovakia, in 1924. Until 1963, he worked
D) establishment.
as a research scientist for the Czechoslovak

www.projeto-militar.blogspot.com.br
INGLÊS – EXERCÍCIOS

Academy of Sciences, when he was invited as a B) é direcionado apenas para iniciantes em


Visiting Professor to the University of Colorado, matemática.
where he decided to stay permanently as C) conta a história de Petr Beckmann em tempos
professor of electrical engineering. de repressão ao conhecimento.
Dr. Beckmann has authored 11 books and D) associa conceitos matemáticos a fatos da vida
more than 50 scientific papers, mostly on cotidiana.
probability theory and electromagnetic wave E) é acessível a um público diversificado.
propagation. History is one of his side interests;
another is linguistics (he is fluent in five 52. (ITA) No contexto deste texto, o item lexical
languages and he has worked out a new “stifled” pode ser traduzido por
generative grammar which enables a computer to A) sufocada.
construct trillions of grammatical sentences from B) desmascarada.
a dictionary of less than 100 unprocessed words). C) organizada.
He also publishes a monthly pro-science, D) promulgada.
pro-technology, pro-free enterprise newsletter E) institucionalizada.
Access to Energy, in which he promotes the
viewpoint that clean energy can be made 53. (ITA) Dentre os interesses de Petr Beckmann,
plentiful, but that access to it is blocked by NÃO se encontra(m)
government interference and environmental A) a divulgação científica.
paranoia. B) a Geografia.
BECKMANN, Petr. A History of Pi. New York: Barnes & Noble Books, 1983.
C) a História.
D) a pesquisa científica.
50. (ITA) O texto foi extraído de um(a)
E) as línguas estrangeiras.
A) aba / orelha de livro.
B) prefácio de livro.
54. (ITA) Indique o item lexical que pode
C) roteiro de leitura.
substituir o sublinhado no trecho “... mostly on
D) resenha literária.
probability theory and electromagnetic wave
E) ensaio literário.
propagation.”, sem prejudicar o seu sentido.
A) absolutely
51. (ITA) O livro A History of Pi
B) chiefly
A) descreve grande parte da história da
C) inherently
matemática e da humanidade.
D) randomly

www.projeto-militar.blogspot.com.br
INGLÊS – EXERCÍCIOS

E) utterly warm relationships between parents, spouses,


55. (ITA) A opção que contém a reescrita correta children and friends have the greatest impact on
de “... science was being stifled by militarism your health and happiness in old age. The study
or religious fanaticism.” é: Militarism or found that 93 percent of the sample group who
religious fanaticism were thriving at age 65, had a close relationship
A) were stifling science. with a sibling when they were younger. As
B) had been stifling science. George Vaillant, the lead director of the study
C) were being stifling science. states, it can all be boiled down into five simple
D) has stifling science. words: “Happiness is love. Full stop.” (Business
E) have been stifling science. Insider.)
http://www.goodnet.org/articles/1055 (acesso em 10/06/2013).

As questões de 56 a 5 referem-se ao texto a


56. (ITA) A Grant Study, pesquisa realizada
seguir:
pela Universidade de Harvard,
A) teve por objetivo investigar o comportamento
Harvard conducted one of the longest and
de pessoas idosas e felizes.
most comprehensive studies of human
B) possibilitou o levantamento gigantesco de
development — the 75 year old Grant Study —
dados sobre pesquisadores de Harvard.
that’s reached some fascinating conclusions
C) comprovou que John F. Kennedy foi um
regarding the recipe for leading a happy life. The
homem extremamente feliz.
sample group was comprised of healthy male
D) chama-se the 75 year old Grant Study por ser
Harvard college students who, over the course of
homenagem à faixa etária analisada.
their lifetime, agreed to meet with an array of
E) comprovou que felicidade na vida adulta está
scientists and researchers who measured their
atrelada às relações afetivas ao longo da vida.
psychological, physical and anthropological
traits. Though all identities are confidential, it
57. (ITA) Assinale a opção cuja reescrita não
was recently discovered that John F. Kennedy
altera o sentido de: “Though all identities are
was a sample participant. Following these men
confidential, it was recently discovered that
through times of war, their careers, parenthood
John F. Kennedy was a sample participant.”
and old age, the Grant Study has amassed an
A) John F. Kennedy was a sample participant,
exorbitant amount of data that deeply reflects the
although nobody knew that.
human condition. What can be concluded from
B) In spite of being a sample participant, John F.
seven decades of data? It is quite simple actually;

www.projeto-militar.blogspot.com.br
INGLÊS – EXERCÍCIOS

Kennedy’s identity was never discovered. B) a empresa contratante exige uma série de
C) The study was confidential, thus the exames clínicos que atestem a saúde do
participation of John F. Kennedy was never candidato.
discovered. C) a atitude do candidato é comprovada através
D) Besides being a confidential study, John F. de detalhada investigação laboratorial.
Kennedy said he used to be a participant. D) o desempenho do entrevistado é de suma
E) In spite of the fact that all identities are kept importância para a construção de sua imagem.
confidential, it was recently found out that John E) as informações sobre o entrevistado,
F. Kennedy was a sample participant. disponíveis online, não são mais importantes do
que sua atitude e apresentação pessoal.
As questões de 58 a 60 referem-se à tirinha a
seguir: 60. (ITA) “Tanning bed”, no penúltimo quadro
da tirinha
A) foi mencionado para ocultar um MRI.
B) refere-se a uma atitude do entrevistado.
C) refere-se a um tipo de cama utilizada para
relaxamento.
D) é sinônimo de MRI.
E) é um tipo de exame.
58. (ITA) No contexto em que se insere,
“external stuff”, no quarto quadro da tirinha, foi 61. (ITA) A palavra “landed”, na sentença
interpretado, pelo entrevistado, como “apparently some of your sample landed [...]”,
A) funcionários terceirizados. no
B) exames de rotina para contratação. sexto quadro da tirinha, pode ser substituída por
C) informações de menor importância. A) stopped.
D) dados de veracidade questionável. B) ended up.
E) dados investigados externamente. C) was included.
D) arrived.
59. (ITA) Segundo a tirinha, em uma entrevista E) was caught
de trabalho
A) está cada vez mais difícil falsear informações 62. (ITA) Leia o anúncio abaixo e assinale a
pessoais. opção que substitui corretamente a afirmação “so

www.projeto-militar.blogspot.com.br
INGLÊS – EXERCÍCIOS

should your airline”. detained several times. During one period in


custody, he was allegedly beaten so badly that he
required brain surgery. This arrest comes amid a
widespread crackdown touched off by online
calls for a Tunisian-style ―jasmine revolution.‖
Over the past several weeks, at least 26 activists
have been detained, 200 have been put under
house arrest, and more than 30 have disappeared.
Time, April 18, 2011.
A) Your airline should offer its clients a wider
range of businesses. 63. (ITA) Segundo o texto, Ai Weiwei
B) Business should cross borders and also should A) alegou ter sido severamente torturado.
your airline. B) foi preso devido a um recrudescimento da
C) Your airline should invest more in business repressão na China.
worldwide. C) embarcou num voo para Hong Kong.
D) Business crosses borders and your airline D) foi preso por incitar uma revolução nos
should, too. moldes da tunisiana.
E) Your airline should keep on doing business E) foi quem projetou o estádio olímpico de
abroad so as to improve its results. Pequim.

As questões 63 e 64 referem-se ao seguinte 64. (ITA) Segundo o texto,


texto: A) Liu Xiaobo foi preso em 3 de abril no
aeroporto de Pequim.
Artist Detained In Growing Crackdown B) houve, na China, incitação à revolução via
BEIJING Internet.
Ai Weiwei, China’s most prominent dissident C) Ai Weiwei é o mais proeminente dissidente
after imprisoned Nobel laureate Liu Xiaobo, was chinês.
detained April 3 at the Beijing airport as he tried D) a prisão domiciliar é prática frequente em
to board a flight to Hong Kong. Perhaps best território chinês.
known for codesigning the 2008 Beijing Olympic E) Ai Weiwei faz críticas veladas ao regime
stadium known as the Bird’s Nest, Ai is an vigente.
outspoken critic of the government an has been

www.projeto-militar.blogspot.com.br
INGLÊS – EXERCÍCIOS

As questões 65 e 66 referem-se ao seguinte players see themselves as tougher and meaner -


texto: and therefore cause them to play that way?
Jeremy Bailenson and Nick Yee of Stanford
Our Imaginary, Hotter Selves University had this and other classic studies in
Avatars might serve therapeutic purposes, mind when they started wondering about the
helping those with social phobia effect of being able to alter one's appearance.
become more confident. They weren't going to study wardrobe choices,
by Sharon Begley however. Their quarry is avatars, digital
Anyone who has ever had a bad hair day, representations of players in such games as
when looking like a latter-day Medusa makes you Second Life. "Your physical appearance changes
feel cranky and antisocial and plodding, can how people treat you," says Bailenson. "But
sympathize with the Oakland Raiders - and not independent of that, when you perceive yourself
because the players get helmet hair. The Raiders in a certain way, you act differently." He and Yee
alternated between mostly black and mostly call it "the Proteus effect," after the shape-
white uniforms, depending on whether they were changing Greek god. The effect of appearance on
playing at home or away. Knowing that behavior, they find, carries over from the virtual
appearance affects people's mood and outlook, world to the real one, with intriguing
psychologists wondered whether uniform color consequences. (…)
http://www.newsweek.com. Acesso em 5/6/2010
influenced the Raiders' aggressiveness. Using
data from the 1970s and 1980s, they found that
65. (ITA) De acordo com o título e o subtítulo do
the team racked up way more penalty yards - a
texto, avatares
measure of aggression - when they wore black
A) proporcionam efeitos terapêuticos e ajudam a
than when they wore white, for infractions both
prevenir doenças como a fobia social.
minor (encroachment) and major (roughing the
B) são versões imaginárias e mais atraentes de
kicker). The pattern held even when the scientists
nós mesmos.
took into account different conditions and styles
C) são mais confiáveis e, por isso, não despertam
of play at home and away. But while the 1988
fobias.
finding has become a classic in psychology, the
D) têm uma proposta de entretenimento, que
explanation remains controversial. Do referees,
torna as pessoas mais sociáveis.
because of black's cultural baggage, see black-
E) são mais confiáveis do que algumas propostas
clad players as meaner and badder than those in,
terapêuticas disponíveis em nossa sociedade.
say, baby blue? Or does wearing black make

www.projeto-militar.blogspot.com.br
INGLÊS – EXERCÍCIOS

66. (ITA) Assinale a opção CORRETA. 67. (ITA) A palavra breakthrough, na charge, tem
A) Os estudiosos da Universidade de Stanford o mesmo sentido de
não consideraram, em seus experimentos, a A) customary.
descoberta realizada em 1988, cuja explicação B) inept.
ainda é controversa. C) conventional.
B) Psicólogos ainda questionam se, de fato, a D) innovative.
aparência afeta o humor e opinião das pessoas. E) ordinary.
C) Jeremy Bailenson e Nick Yee afirmam que a
aparência transforma o modo como as pessoas 68. (ITA) A mensagem transmitida pela charge
nos tratam e disso depende a maneira como NÃO denota
percebemos a nós mesmos. A) crítica.
D) A aparência física afeta o comportamento das B) lentidão.
pessoas e traz consequências para o mundo real e C) arrependimento.
não apenas para o virtual. D) ironia.
E) O foco dos estudiosos está no figurino dos E) evolução.
avatares e no modo como isso afeta a
agressividade dos jogadores. 69. (ITA) Assinale a opção que mais se aproxima
da ideia central do texto.
As questões de 67 a 69 referem-se à charge a A) O trabalho dignifica o homem.
seguir: B) Uma andorinha só não faz verão.
C) Quem tudo quer, nada tem.
D) A ociosidade é a mãe de todos os vícios.
E) Mais vale prevenir que remediar

As questões 70 e 71 referem-se à seguinte figura:

www.projeto-militar.blogspot.com.br
INGLÊS – EXERCÍCIOS

A) apenas a I.
B) apenas a II.
C) apenas a III.
D) apenas a I e II.
E) todas.

As questões de 72 a 75 referem-se ao seguinte


texto:

Ethical abuses in the authorship of scientific


papers
Problems regarding the order of authorship of
scientific papers have become more frequent and
more abusive. These problems may have
heightened due to the ever increasing pressure to
“publish or perish” in the academic world, given
that the publication of scientific articles has
70. (ITA) Assinale a opção que NÃO descreve
become the benchmark of success in a field with
benefícios apontados na figura.
few job opportunities. This article reviews the
A) Sapatos com velcro e fáceis de fechar.
abuses in the authorship of scientific papers.
B) Calça comprida com elástico na cintura.
Different examples are given of the most
C) Bolso com fecho especial para guloseimas.
common problems and recommendations are
D) Blusa sintética e aderente à pele.
provided for authors and journal editors
E) Jaqueta resistente ao vento.
Rev. Bras. Entomol. Vol. 51 no. 1 São Paulo, Jan./Mar. 2007

71. (ITA) Considere as seguintes afirmações:


72. (ITA) O objetivo do artigo ao qual se refere
I. As listas verticais indicadas afinam a silhueta.
o texto é
II. A figura mostra sapatos que não se desgastam
A) divulgar as dificuldades no mercado de
com o tempo.
trabalho acadêmico.
III. Inactive Wear é apropriada para praticantes
B) publicar diferentes textos científicos.
de exercícios físicos.
C) estimular a publicação de artigos científicos.
Está(ão) correta(s):
D) divulgar as recomendações de editores para a

www.projeto-militar.blogspot.com.br
INGLÊS – EXERCÍCIOS

elaboração de artigos científicos. substituída por


E) analisar abusos relacionados à autoria de A) where
artigos científicos. B) when
C) which
73. (ITA) Considere as seguintes afirmações: D) whose
I. O artigo ao qual o texto se refere divulga uma E) whether
lista de publicações científicas com problemas
relacionados ao plágio. As questões de 76 a 78 referem-se à charge a
II. As oportunidades de trabalho no mundo seguir:
acadêmico são mais restritas para os
pesquisadores que não publicam artigos Luis Suárez joins anti-racism calls after
científicos. Dani Alves banana incident The Barcelona
III. Para que o pesquisador seja reconhecido defender Dani Alves has sparked a social media
diante da comunidade acadêmica, a publicação campaign against racism in football as support
de artigos científicos é importante. flooded in from fellow professionals for his
Está(ão) correta(s) decision to eat a banana thrown at him by an
A) apenas a I. opposition fan.
B) apenas II e III. Luis Suárez, Neymar, Hulk, Mario
C) apenas I e II. Balotelli and Sergio Agüero were among those
D) apenas I e III. who posted pictures of themselves taking bites
E) todas. out of bananas in tribute to Alves' actions in his
side's La Liga match at Villarreal on Sunday.
74. (ITA) Os termos heightened, benchmark e The Fifa president Joseph Blatter has
are provided podem ser traduzidos, branded the abuse directed at Alves an "outrage"
respectivamente, como: and promised zero tolerance towards
A) complicado, símbolo, sugerem discrimination at the World Cup, while Villarreal
B) aumentado, problema, são sugeridas took swift action by identifying the culprit and
C) solucionado, determinante, têm mostrado handing him a lifetime stadium ban.
D) crescido, referência, são fornecidas Alves' response to the banana being
E) diminuído, causa, mostram thrown on to the pitch in front of him as he
prepared to take a corner was to nonchalantly
75. (ITA) A expressão given that pode ser pick it up, peel it and take a bite before

www.projeto-militar.blogspot.com.br
INGLÊS – EXERCÍCIOS

continuing with the game. The 30-yearold, who C) even Suárez, who has already been racially
has been the victim of racist abuse before during abused by Patrice Eva paid homage to Dani
his time in La Liga, said: "You need to take these Alves.
situations with a dose of humour." D) Dani Alves’ decision to eat a banana thrown at
Players across Europe paid homage on him during a game ignited a racism discussion in
Twitter and Instagram, including Suárez, who the social media.
served an eight-match ban for racially abusing E) Villarreal managed to find who the offender
Patrice Evra. was with the help of Neymar.
Alves's Barça and Brazil team-mate
Neymar led the way after posting a picture on 77. (IME) In the sentence “Alves' response to the
Instagram of himself holding a banana, while banana being thrown on to the pitch in front of
writing "We are all monkeys". Balotelli, Milan's him as he prepared to take a corner was to
former Manchester City striker, posted a picture nonchalantly pick it up, peel it and take a bite
of himself in a similar pose. before continuing with the game.”, the word in
Suárez posted a picture on Twitter of himself and bold could be replaced by:
Liverpool team-mate Philippe Coutinho taking A) calmly.
bites out of bananas, along with the words: B) flawlessly.
"#SayNoToRacism #WeAreAllMonkeys." C) furiously.
(...) D) intently.
Barça gave their player their "complete E) heatedly.
support and solidarity" and thanked Villarreal for
their "immediate condemnation" of the incident. 78. (IME) According to text, which of the
Villarreal later revealed they had, with the help of following is true about Dani Alves’ racism
fans, found out who the culprit was, had episode?
withdrawn his season ticket and banned him from A) The Fifa president himself posted a photo on
the El Madrigal stadium for life. Twitter taking a bite of a banana.
Disponível em: <http://www.theguardian.com/football/2014/apr/29/luis-suarez-anti-racism-dani-
alvesbanana>. Acesso em 29 abr.2014 (texto adaptado)
B) The Fifa president stated that episodes of
racism would not be be accepted during the
76. (IME) It is implied in text that World Cup.
A) Villareal took the racism episode for granted. C) The offender will serve an eight-match ban for
B) the offender was banned from the stadium racially abusing Alves.
because of Neymar’s photo on Instagram. D) Alves declared that he handled the situation

www.projeto-militar.blogspot.com.br
INGLÊS – EXERCÍCIOS

with a dose of humor because he thought it was table of brotherhood.” (Martin Luther King)
just witty. A) would be able to
E) Barcelona was in charge of banning the culprit B) will be able to
from the El Madrigal stadium for good. C) should have been able to
D) are able to
Para as questões 79 a 8, escolha a alternativa E) would have been able to
que complete a sentença CORRETAMENTE.
82. (IME) On average, women continue to earn
79. (IME) During the Second World War, considerably less than men. In 2012, female full-
approximately 6 million european jews time workers made only 77 cents for every dollar
__________ mass murdered in concentration earned by men, a gender wage gap ____ 23
camps and forced labour. percent.
A) has been A) at
B) been B) by
C) would have been C) on
D) are D) of
E) were E) with

80. (IME) _______________ the legislation 83. (IME) There are many forms of prejudice
promising them a fair share of opportunity, Dalits and oppression, __________ based on race, but
(lower caste) Hindus continue to form among the on gender, class, sexual orientation, etc.
poorest sections of indian society. A) as well as
A) Even though B) not just
B) Nevertheless C) in addition to
C) Since D) simply
D) Despite E) on the contrary
E) While
84. (IME) ___________ the Fifa president and
81. (IME) “I have a dream that one day, on the vice president will be in Brazil for the World
red hills of Georgia, the sons of former slaves Soccer Cup.
and the sons of former slave owners A) Either
_________________ sit down together at the B) Also

www.projeto-militar.blogspot.com.br
INGLÊS – EXERCÍCIOS

C) Too their elders but to treat everyone with respect and


D) Both courtesy, children may grow up to be responsible
E) Neither adults whose influence other people to respect
human feeling, rights and property. They may
85. (IME) The player was about to take corner grow up to cherish human life, not annihilate it.
when he _______________ at him. All people want respect, so they must give it to
A) would had a banana thrown earn it.
B) would have throw a banana A) on
C) is throwing a banana B) but
D) would be thrown a banana C) whose
E) had a banana thrown D) not
E) All
86. (IME) Russian Sports Minister says he
___________ by the slow pace of designing the 88. (IME) The history of modern-day soccer was
country’s stadiums for the 2018 World Cup and established in 1863. In October 1863, eleven
threatened heads will roll if the situation is not representatives from London clubs and schools
rectified. met at the Freemason’s Tavern to set up common
A) is alarming fundamental rules to control the matches
B) is alarmed amongst themselves. The outcome of this
C) has alarmed meeting was the formation of the Football
D) has been alarming Association. In December 1863, the Rugby
E) alarmed Football and Football Association finally split as
the supporters of the Rugby School rules walked
Para as questões de 87 e 88, encontram-se em in.
destaque cinco termos. Assinale a alternativa A) up
correspondente ao termo cujo emprego está B) to
INCORRETO. C) amongst
D) outcome
87. (IME) If mankind can learn to respect other E) in
human beings in thoughts, words, and actions,
humanity may survive on this planet, Earth. If Para as questões 89 a 98, escolha a alternativa
parents teach children clearly not only to respect que complete a sentença corretamente:

www.projeto-militar.blogspot.com.br
INGLÊS – EXERCÍCIOS

89. (IME) I grew up in Brisbane, Australia, court pending a trial.


_______ a shady quiet street in the old part of A) riot
town. B) demonstration
A) at C) law
B) in D) decision
C) on E) affray
D) over
E) next 93. (IME) Fat? No way! Jane isn’t fat at all.
_______________________, she is quite skinny.
90. (IME) Using a high-tech kit, the police found A) In any case
a single clue, tracked it _________ and saved the B) By rights
girl. C) Nevertheless
A) down D) Although
B) over E) On the contrary
C) by
D) on 94. (IME) Don’t be ridiculous! That man
E) under _______________ possibly be Barrack Obama!
A) mustn’t
91. (IME) In 2013, agents rescued 337 children B) can’t
and took 964 alleged predators __________ the C) shouldn’t
street. D) won’t
A) on E) doesn’t
B) away
C) off 95. (IME) Not only _____________ his house,
D) by but his wife also walked out on him.
E) apart A) did he lose
B) lost
92. (IME) Thousands gathered at Taksim Square C) has lost
in Turkey to protest the court ________________ D) loses
on Ethem Sarisülük’s case. Ethem Sarisülük was E) he didn’t lose
shot in the head by a policeman during Gezi
protests and the murderer was released by the 96. (IME) If we don’t hurry up, all the best seats

www.projeto-militar.blogspot.com.br
INGLÊS – EXERCÍCIOS

___________. to a word or word group it can logically describe.


A) will take A) Stopped for speeding, the ticket was not his
B) take first.
C) will be taken B) Stopped for speeding, the court decided
D) are taken against the driver.
E) would be taken C) Stopped for speeding, the driver paid his fine
properly.
97. (IME) _____________ the cost of a college D) Stopped for speeding, a warning was all that
education at Central Wyoming College is was given.
relatively low, many E) Stopped for speeding, the policeman gave him
students need and receive financial aid. a ticket.
A) Although 100. (IME) Encontram-se em destaque cinco
B) Besides termos ou expressões. Assinale a
C) No sooner alternativa correspondente ao termo cujo
D) Despite emprego está incorreto.
E) However The spreading branches of the tree swayed in
the breeze. In the distance, I heard a barked
98. (IME) Coptic Christians in Egypt dog.
________________ persecution at the hands of a) spreading
the government. Claims against them under b) swayed
Mubarak’s regime were rarely punished. They c) In
have faced open discrimination d) heard
while remaining peaceful. e) barked
A) have long tolerated
B) has long tolerated
C) had long tolerated
D) used to long tolerate
E) long tolerate

99. (IME) Para a questão a seguir, escolha a


alternativa correta.
Choose the sentence in which the modifier refers

www.projeto-militar.blogspot.com.br
INGLÊS – EXERCÍCIOS

GABARITO

1 2 3 4 5 6 7 8 9 10 11 12 13 14 15 16 17 18 19 20

A C A D C A C A B C C B D D E B A D E B

21 22 23 24 25 26 27 28 29 30 31 32 33 34 35 36 37 38 39 40

A D C E A E A D B B C B A E C C D E A D

41 42 43 44 45 46 47 48 49 50 51 52 53 54 55 56 57 58 59 60

A C C D C E E B C A E A B B A E E C A A

61 62 63 64 65 66 67 68 69 70 71 72 73 74 75 76 77 78 79 80

B D B B B D D E E D A E B D A D A B E D

81 82 83 84 85 86 87 88 89 90 91 92 93 94 95 96 97 98 99 100

B D B D E B C E C A C D E B A C A A C E

www.projeto-militar.blogspot.com.br

Você também pode gostar